*NURSING > EXAM > Keiser University: NURSING NUR 340 Principles of Pediatric Nursing: Caring for Children, 7e (Ball et (All)

Keiser University: NURSING NUR 340 Principles of Pediatric Nursing: Caring for Children, 7e (Ball et al.),100% CORRECT

Document Content and Description Below

Keiser University: NURSING NUR 340 Principles of Pediatric Nursing: Caring for Children, 7e (Ball et al.) Chapter 1 Nurse's Role in Care of the Child: Hospital, Community, and Home 1) Which nurs... ing role is not directly involved when providing family-centered approach to the pediatric population? 1. Advocacy 2. Case management 3. Patient education 4. Researcher Answer: 4 Explanation: 1. A researcher is not involved in the family-centered approach to patient care of children and their families. Advocacy, case management, and patient education are all roles directly involved in the care of children and their families. 2. A researcher is not involved in the family-centered approach to patient care of children and their families. Advocacy, case management, and patient education are all roles directly involved in the care of children and their families. 3. A researcher is not involved in the family-centered approach to patient care of children and their families. Advocacy, case management, and patient education are all roles directly involved in the care of children and their families. 4. A researcher is not involved in the family-centered approach to patient care of children and their families. Advocacy, case management, and patient education are all roles directly involved in the care of children and their families. Page Ref: 4 Cognitive Level: Analyzing Client Need &Sub: Safe and Effective Care Environment: Management of Care Standards: QSEN Competencies: Patient-centered care | AACN Essential Competencies: Essential II: Basic organizational and systems leadership for quality care and patient safety | NLN Competencies: Professional identity | Nursing/Integrated Concepts: Nursing Process: Assessment/Coordination of care Learning Outcome: LO 1.2 Compare the roles of nurses in child healthcare. MNL LO: Family-centered care 2) A nurse is working with pediatric clients in a research facility. The nurse recognizes that federal guidelines are in place that delineate which pediatrics clients must give assent for participation in research trials. Based upon the client's age, the nurse would seek assent from which children? Select all that apply. 1. The precocious 4-year-old commencing as a cystic fibrosis research-study participant. 2. The 7-year-old leukemia client electing to receive a newly developed medication, now being researched. 3. The 10-year-old commencing in an investigative study for clients with precocious puberty. 4. The 13-year-old client commencing participation in a research program for Attention Deficit Hyperactivity Disorder (ADHD) treatments. Answer: 2, 3, 4 Explanation: 1. Federal guidelines mandate that research participants 7 years old and older must receive developmentally appropriate information about healthcare procedures and treatments and give assent. 2. Federal guidelines mandate that research participants 7 years old and older must receive developmentally appropriate information about healthcare procedures and treatments and give assent. 3. Federal guidelines mandate that research participants 7 years old and older must receive developmentally appropriate information about healthcare procedures and treatments and give assent. 4. Federal guidelines mandate that research participants 7 years old and older must receive developmentally appropriate information about healthcare procedures and treatments and give assent. Page Ref: 11, 12 Cognitive Level: Applying Client Need &Sub: Psychosocial Integrity Standards: QSEN Competencies: Patient-centered care | AACN Essential Competencies: Essential V: Healthcare policy, finance, and regulatory environments | NLN Competencies: Nursing judgement | Nursing/Integrated Concepts: Nursing Process: Planning/Coordination of care Learning Outcome: LO 1.6 Examine three unique pediatric legal and ethical issues in pediatric nursing practice. MNL LO: Developmentally appropriate care 3) The nurse in a pediatric acute care unit is assigned the following tasks. Which task is not appropriate for the nurse to complete? 1. Diagnose an 8-year-old with acute otitis media and prescribe an antibiotic. 2. Listen to the concerns of an adolescent about being out of school for a lengthy surgical recovery. 3. Provide information to a mother of a newly diagnosed 4-year-old diabetic about local support- group options. 4. Diagnose a 6-year-old with Diversional Activity Deficit related to placement in isolation. Answer: 1 Explanation: 1. The role of the pediatric nurse includes providing nursing assessment, directing nursing care interventions, and educating client and family at developmentally appropriate levels; client advocacy, case management, minimization of distress, and enhancement of coping. Advanced practice nurse practitioners perform assessment, diagnosis, and management of health conditions. 2. The role of the pediatric nurse includes providing nursing assessment, directing nursing care interventions, and educating client and family at developmentally appropriate levels; client advocacy, case management, minimization of distress, and enhancement of coping. Advanced practice nurse practitioners perform assessment, diagnosis, and management of health conditions. 3. The role of the pediatric nurse includes providing nursing assessment, directing nursing care interventions, and educating client and family at developmentally appropriate levels; client advocacy, case management, minimization of distress, and enhancement of coping. Advanced practice nurse practitioners perform assessment, diagnosis, and management of health conditions. 4. The role of the pediatric nurse includes providing nursing assessment, directing nursing care interventions, and educating client and family at developmentally appropriate levels; client advocacy, case management, minimization of distress, and enhancement of coping. Advanced practice nurse practitioners perform assessment, diagnosis, and management of health conditions. Page Ref: 2-4 Cognitive Level: Applying Client Need &Sub: Safe and Effective Care Environment: Management of Care Standards: QSEN Competencies: Patient-centered care | AACN Essential Competencies: Essential II: Basic organizational and systems leadership for quality care and patient safety | NLN Competencies: Professional identity | Nursing/Integrated Concepts: Nursing Process: Implementation/Coordination of care Learning Outcome: LO 1.2 Compare the roles of nurses in child healthcare. MNL LO: Pediatric nursing care 4) A 7-year-old child is admitted for acute appendicitis. The parents are questioning the nurse about expectations during the child's recovery. Which information tool would be most useful in answering a parent's questions about the timing of key events? 1. Healthy People 2020 2. Clinical pathways 3. Child mortality statistics 4. National clinical practice guidelines Answer: 2 Explanation: 1. Clinical pathways are interdisciplinary documents provided by a hospital to suggest ideal sequencing and timing of events and interventions for specific diseases to improve efficiency of care and enhance recovery. This pathway serves as a model outlining the typical hospital stay for individuals with specified conditions. Healthy People 2020 contains objectives set by the U.S. government to improve the health and reduce the incidence of death in the twenty-first century. Child mortality statistics can be compared with those from other decades for the evaluation of achievement toward health-care goals. National clinical practice guidelines promote uniformity in care for specific disease conditions by suggesting expected outcomes from specific interventions. 2. Clinical pathways are interdisciplinary documents provided by a hospital to suggest ideal sequencing and timing of events and interventions for specific diseases to improve efficiency of care and enhance recovery. This pathway serves as a model outlining the typical hospital stay for individuals with specified conditions. Healthy People 2020 contains objectives set by the U.S. government to improve the health and reduce the incidence of death in the twenty-first century. Child mortality statistics can be compared with those from other decades for the evaluation of achievement toward health-care goals. National clinical practice guidelines promote uniformity in care for specific disease conditions by suggesting expected outcomes from specific interventions. 3. Clinical pathways are interdisciplinary documents provided by a hospital to suggest ideal sequencing and timing of events and interventions for specific diseases to improve efficiency of care and enhance recovery. This pathway serves as a model outlining the typical hospital stay for individuals with specified conditions. Healthy People 2020 contains objectives set by the U.S. government to improve the health and reduce the incidence of death in the twenty-first century. Child mortality statistics can be compared with those from other decades for the evaluation of achievement toward health-care goals. National clinical practice guidelines promote uniformity in care for specific disease conditions by suggesting expected outcomes from specific interventions. 4. Clinical pathways are interdisciplinary documents provided by a hospital to suggest ideal sequencing and timing of events and interventions for specific diseases to improve efficiency of care and enhance recovery. This pathway serves as a model outlining the typical hospital stay for individuals with specified conditions. Healthy People 2020 contains objectives set by the U.S. government to improve the health and reduce the incidence of death in the twenty-first century. Child mortality statistics can be compared with those from other decades for the evaluation of achievement toward health-care goals. National clinical practice guidelines promote uniformity in care for specific disease conditions by suggesting expected outcomes from specific interventions. Page Ref: 1, 2 Cognitive Level: Applying Client Need &Sub: Psychosocial Integrity Standards: QSEN Competencies: Patient-centered care | AACN Essential Competencies: Essential II: Basic organizational and systems leadership for quality care and patient safety | NLN Competencies: Nursing judgement | Nursing/Integrated Concepts: Nursing Process: Planning/Health teaching and health promotion Learning Outcome: LO 1.1 Describe the continuum of pediatric healthcare. MNL LO: Family-centered care 5) The nurse recognizes that the pediatric client is from a cultural background different from that of the hospital staff. Which goal is most appropriate for this client when planning nursing care? 1. Overlook or minimize the differences that exist. 2. Facilitate the family's ability to comply with the care needed. 3. Avoid inadvertently offending the family by imposing the nurse's perspective. 4. Encourage complementary beneficial cultural practices as primary therapies. Answer: 2 Explanation: 1. The incorporation of the family's cultural perspective into the care plan is most likely to result in the family's ability to accept medical care and comply with the regimen prescribed. Since culture develops from social learning, attempts to ignore or minimize cultural consideration will result in mistrust, suspicion, or offenses that can have negative effects upon the health of children by reducing the resources available to promote health and prevent illness. Complementary therapy may be used later if other primary therapies prove to be ineffective. 2. The incorporation of the family's cultural perspective into the care plan is most likely to result in the family's ability to accept medical care and comply with the regimen prescribed. Since culture develops from social learning, attempts to ignore or minimize cultural consideration will result in mistrust, suspicion, or offenses that can have negative effects upon the health of children by reducing the resources available to promote health and prevent illness. Complementary therapy may be used later if other primary therapies prove to be ineffective. 3. The incorporation of the family's cultural perspective into the care plan is most likely to result in the family's ability to accept medical care and comply with the regimen prescribed. Since culture develops from social learning, attempts to ignore or minimize cultural consideration will result in mistrust, suspicion, or offenses that can have negative effects upon the health of children by reducing the resources available to promote health and prevent illness. Complementary therapy may be used later if other primary therapies prove to be ineffective. 4. The incorporation of the family's cultural perspective into the care plan is most likely to result in the family's ability to accept medical care and comply with the regimen prescribed. Since culture develops from social learning, attempts to ignore or minimize cultural consideration will result in mistrust, suspicion, or offenses that can have negative effects upon the health of children by reducing the resources available to promote health and prevent illness. Complementary therapy may be used later if other primary therapies prove to be ineffective. Page Ref: 3 Cognitive Level: Applying Client Need &Sub: Health Promotion and Maintenance Standards: QSEN Competencies: Patient-centered care | AACN Essential Competencies: Essential II: Basic organizational and systems leadership for quality care and patient safety | NLN Competencies: Professional identity | Nursing/Integrated Concepts: Nursing Process: Planning/Coordination of care Learning Outcome: LO 1.3 Analyze the current societal influences on pediatric healthcare and nursing practice. MNL LO: Family-centered care 6) The telephone triage nurse at a pediatric clinic knows each call is important. Which call would require attentiveness from the nurse because of an increased risk of mortality? 1. A 3-week-old infant born at 35 weeks' gestation with gastroenteritis 2. A term 2-week-old infant of American Indian descent with an upper respiratory infection 3. A postterm 4-week-old infant non-Hispanic black descent with moderate emesis after feeding 4. A 1-week-old infant born at 40 weeks' gestation with symptoms of colic Answer: 1 Explanation: 1. The leading causes of death in the neonatal period (birth to 28 days of age) are short gestation, low birth weight, and congenital malformations. The preterm infant experiencing gastroenteritis at 3 weeks of age is at the greatest risk for mortality; therefore, would require extra attentiveness from the registered nurse. 2. The leading causes of death in the neonatal period (birth to 28 days of age) are short gestation, low birth weight, and congenital malformations. The preterm infant experiencing gastroenteritis at 3 weeks of age is at the greatest risk for mortality; therefore, would require extra attentiveness from the registered nurse. 3. The leading causes of death in the neonatal period (birth to 28 days of age) are short gestation, low birth weight, and congenital malformations. The preterm infant experiencing gastroenteritis at 3 weeks of age is at the greatest risk for mortality; therefore, would require extra attentiveness from the registered nurse. 4. The leading causes of death in the neonatal period (birth to 28 days of age) are short gestation, low birth weight, and congenital malformations. The preterm infant experiencing gastroenteritis at 3 weeks of age is at the greatest risk for mortality; therefore, would require extra attentiveness from the registered nurse. Page Ref: 6, 7 Cognitive Level: Applying Client Need &Sub: Health Promotion and Maintenance Standards: QSEN Competencies: Informatics/Patient-centered care | AACN Essential Competencies: Essential IV: Information management and application of patient care technology | NLN Competencies: Professional identity | Nursing/Integrated Concepts: Nursing Process: Planning/Coordination of care Learning Outcome: LO 1.4 Report the most common causes of child mortality by age group and reasons for hospitalization. MNL LO: Developmentally appropriate care 7) Despite the availability of Children's Health Insurance Programs (CHIP), many eligible children are not enrolled. Which nursing intervention would be the most appropriate to help children become enrolled in CHIP? 1. Assess details of the family's income and expenditures 2. Case management to limit costly, unnecessary duplication of services 3. Advocate for the child by encouraging the family to investigate SCHIP eligibility 4. Educate the family about the need for keeping regular well-child-visit appointments Answer: 3 Explanation: 1. In the role of an advocate, a nurse will advance the interests of another; by suggesting the family investigate its CHIP eligibility, the nurse is directing their action toward the child's best interest. Financial assessment is more commonly the function of a social worker. The case-management activity mentioned will not provide a source of funding nor will the educational effort described. 2. In the role of an advocate, a nurse will advance the interests of another; by suggesting the family investigate its CHIP eligibility, the nurse is directing their action toward the child's best interest. Financial assessment is more commonly the function of a social worker. The case- management activity mentioned will not provide a source of funding nor will the educational effort described. 3. In the role of an advocate, a nurse will advance the interests of another; by suggesting the family investigate its CHIP eligibility, the nurse is directing their action toward the child's best interest. Financial assessment is more commonly the function of a social worker. The case- management activity mentioned will not provide a source of funding nor will the educational effort described. 4. In the role of an advocate, a nurse will advance the interests of another; by suggesting the family investigate its CHIP eligibility, the nurse is directing their action toward the child's best interest. Financial assessment is more commonly the function of a social worker. The case- management activity mentioned will not provide a source of funding nor will the educational effort described. Page Ref: 2-4 Cognitive Level: Applying Client Need &Sub: Health Promotion and Maintenance Standards: QSEN Competencies: Patient-centered care | AACN Essential Competencies: Essential VI: Interprofessional communication and collaboration for improving patient health outcomes | NLN Competencies: Human flourishing | Nursing/Integrated Concepts: Nursing Process: Planning/Coordination of care Learning Outcome: LO 1.2 Compare the roles of nurses in child healthcare. MNL LO: Family-centered care 8) A supervisor is reviewing documentation of the nurses in the unit. Which client documentation is the most accurate and contains all the required part for a narrative entry? 1. "2/2/05 1630 Catheterized using an 8 French catheter, 45 mL clear yellow urine obtained, specimen sent to lab, squirmed and cried softly during insertion of catheter. Quiet in mother's arms following catheter removal. M. May RN" 2. "1/9/05 2 pm nasogastric tube placement confirmed and irrigated with 30 ml sterile water. Suction set at low, intermittent. Oxygen via nasal canal at 2 L/min. Nares patent, pink, and nonirritated. K. Earnst RN" 3. "4:00 tracheostomy dressing removed with dime-size stain of dry serous exudate. Site cleansed with normal saline. Dried with sterile gauze. New sterile tracheostomy sponge and trach ties applied. Respirations regular and even throughout the procedure. F. Luck RN" 4. "Feb. '05 Port-A-Cath assessed with Huber needle. Blood return present. Flushed with NaCl solution, IV gamma globulins hung and infusing at 30 cc/hr. Child smiling and playful throughout the procedure. P. Potter, RN" Answer: 1 Explanation: 1. The client record should include the date and time of entry, nursing care provided, assessments, an objective report of the client's physiologic response, exact quotes, and the nurse's signature and title. 2. The client record should include the date and time of entry, nursing care provided, assessments, an objective report of the client's physiologic response, exact quotes, and the nurse's signature and title. 3. The client record should include the date and time of entry, nursing care provided, assessments, an objective report of the client's physiologic response, exact quotes, and the nurse's signature and title. 4. The client record should include the date and time of entry, nursing care provided, assessments, an objective report of the client's physiologic response, exact quotes, and the nurse's signature and title. Page Ref: 2-4 Cognitive Level: Applying Client Need &Sub: Safe and Effective Care Environment: Management of Care Standards: QSEN Competencies: Informatics | AACN Essential Competencies: Essential IV: Information management and application of patient care technology | NLN Competencies: Professional identity | Nursing/Integrated Concepts: Nursing Process: Assessment/Coordination of care Learning Outcome: LO 1.2 Compare the roles of nurses in child healthcare. MNL LO: Pediatric nursing care 9) A 12-year-old pediatric client is in need of surgery. Which member of the healthcare team is legally responsible for obtaining informed consent for an invasive procedure? 1. Nurse 2. Physician 3. Unit secretary 4. Social worker Answer: 2 Explanation: 1. Informed consent is legal preauthorization for an invasive procedure. It is the physician's legal responsibility to obtain this, because it consists of an explanation about the medical condition, a detailed description of treatment plans, the expected benefits and risks related to the proposed treatment plan, alternative treatment options, the client's questions, and the guardian's right to refuse treatment. 2. Informed consent is legal preauthorization for an invasive procedure. It is the physician's legal responsibility to obtain this, because it consists of an explanation about the medical condition, a detailed description of treatment plans, the expected benefits and risks related to the proposed treatment plan, alternative treatment options, the client's questions, and the guardian's right to refuse treatment. 3. Informed consent is legal preauthorization for an invasive procedure. It is the physician's legal responsibility to obtain this, because it consists of an explanation about the medical condition, a detailed description of treatment plans, the expected benefits and risks related to the proposed treatment plan, alternative treatment options, the client's questions, and the guardian's right to refuse treatment. 4. Informed consent is legal preauthorization for an invasive procedure. It is the physician's legal responsibility to obtain this, because it consists of an explanation about the medical condition, a detailed description of treatment plans, the expected benefits and risks related to the proposed treatment plan, alternative treatment options, the client's questions, and the guardian's right to refuse treatment. Page Ref: 11, 12 Cognitive Level: Applying Client Need &Sub: Safe and Effective Care Environment: Management of Care Standards: QSEN Competencies: Informatics | AACN Essential Competencies: Essential IV: Information management and application of patient care technology | NLN Competencies: Professional identity | Nursing/Integrated Concepts: Nursing Process: Planning/Communication Learning Outcome: LO 1.5 Contrast the policies for obtaining informed consent of minors to policies for adults. MNL LO: Developmentally appropriate care 10) A child is being prepared for an invasive procedure. The mother of the child has legal custody but is not present. After details of the procedure are explained, who can provide legal consent on behalf of a minor child for treatment? 1. The divorced parent without custody 2. A cohabitating boyfriend of the child's mother 3. A grandparent who lives in the home with the child 4. A babysitter with written proxy Answer: 4 Explanation: 1. A parent may grant proxy consent in writing to another adult so that children are not denied necessary healthcare. In the case of divorced parents, the parent with custody may be the only parent allowed by some states to give informed consent. Residence in the same household with a child does not authorize an adult to sign consent for treatment. 2. A parent may grant proxy consent in writing to another adult so that children are not denied necessary healthcare. In the case of divorced parents, the parent with custody may be the only parent allowed by some states to give informed consent. Residence in the same household with a child does not authorize an adult to sign consent for treatment. 3. A parent may grant proxy consent in writing to another adult so that children are not denied necessary healthcare. In the case of divorced parents, the parent with custody may be the only parent allowed by some states to give informed consent. Residence in the same household with a child does not authorize an adult to sign consent for treatment. 4. A parent may grant proxy consent in writing to another adult so that children are not denied necessary healthcare. In the case of divorced parents, the parent with custody may be the only parent allowed by some states to give informed consent. Residence in the same household with a child does not authorize an adult to sign consent for treatment. Page Ref: 11, 12 Cognitive Level: Applying Client Need &Sub: Safe and Effective Care Environment: Management of Care Standards: QSEN Competencies: Informatics | AACN Essential Competencies: Essential IV: Information management and application of patient care technology | NLN Competencies: Professional identity | Nursing/Integrated Concepts: Nursing Process: Planning Learning Outcome: LO 1.5 Contrast the policies for obtaining informed consent of minors to policies for adults. MNL LO: Family-centered care 11) A 12-year-old child is admitted to the unit for a surgical procedure. The child is accompanied by two parents and a younger sibling. What is the level of involvement in treatment decision making for this child? 1. Emancipated minor 2. Mature minor 3. Assent 4. None Answer: 3 Explanation: 1. Assent requires the ability to generally understand what procedure and treatments are planned, to understand what participation is required, and to make a statement of agreement or disagreement with the plan. Usually, in Piaget's stage of formal operations, 11- to 13-year-olds should be able to problem solve using abstract concepts and are able to give valid assent when parents sign the informed consent. An emancipated minor is a self-supporting adolescent who is not subject to the control of a parent or guardian. A mature minor is a 14- or 15-year-old whom the state law designates as being able to understand medical risks and who is thus permitted to give informed consent for treatment. 2. Assent requires the ability to generally understand what procedure and treatments are planned, to understand what participation is required, and to make a statement of agreement or disagreement with the plan. Usually, in Piaget's stage of formal operations, 11- to 13-year-olds should be able to problem solve using abstract concepts and are able to give valid assent when parents sign the informed consent. An emancipated minor is a self-supporting adolescent who is not subject to the control of a parent or guardian. A mature minor is a 14- or 15-year-old whom the state law designates as being able to understand medical risks and who is thus permitted to give informed consent for treatment. 3. Assent requires the ability to generally understand what procedure and treatments are planned, to understand what participation is required, and to make a statement of agreement or disagreement with the plan. Usually, in Piaget's stage of formal operations, 11- to 13-year-olds should be able to problem solve using abstract concepts and are able to give valid assent when parents sign the informed consent. An emancipated minor is a self-supporting adolescent who is not subject to the control of a parent or guardian. A mature minor is a 14- or 15-year-old whom the state law designates as being able to understand medical risks and who is thus permitted to give informed consent for treatment. 4. Assent requires the ability to generally understand what procedure and treatments are planned, to understand what participation is required, and to make a statement of agreement or disagreement with the plan. Usually, in Piaget's stage of formal operations, 11- to 13-year-olds should be able to problem solve using abstract concepts and are able to give valid assent when parents sign the informed consent. An emancipated minor is a self-supporting adolescent who is not subject to the control of a parent or guardian. A mature minor is a 14- or 15-year-old whom the state law designates as being able to understand medical risks and who is thus permitted to give informed consent for treatment. Page Ref: 11, 12 Cognitive Level: Analyzing Client Need &Sub: Safe and Effective Care Environment: Management of Care Standards: QSEN Competencies: Patient-centered care | AACN Essential Competencies: Essential V: Healthcare policy, finance, and regulatory environments | NLN Competencies: Professional identity | Nursing/Integrated Concepts: Nursing Process: Planning/Communication Learning Outcome: LO 1.5 Contrast the policies for obtaining informed consent of minors to policies for adults. MNL LO: Developmentally appropriate care 12) Which nursing intervention is most appropriate when providing education to the pediatric client and family? 1. Giving primary care for high-risk children who are in hospital settings 2. Giving primary care for healthy children 3. Working toward the goal of informed choices with the family 4. Obtaining a physician consultation for any technical procedures at delivery Answer: 3 Explanation: 1. The educator works with the family toward the goal of making informed choices through education and explanation. 2. The educator works with the family toward the goal of making informed choices through education and explanation. 3. The educator works with the family toward the goal of making informed choices through education and explanation. 4. The educator works with the family toward the goal of making informed choices through education and explanation. Page Ref: 3 Cognitive Level: Applying Client Need &Sub: Safe and Effective Care Environment: Management of Care Standards: QSEN Competencies: Patient-centered care | AACN Essential Competencies: Essential II: Basic organizational and systems leadership for quality care and patient safety | NLN Competencies: Professional identity | Nursing/Integrated Concepts: Nursing Process: Planning/Health teaching and health promotion Learning Outcome: LO 1.2 Compare the roles of nurses in child healthcare. MNL LO: Family-centered care 13) What is the pediatric nurse's best defense against an accusation of malpractice or negligence? 1. Following the physician's written orders 2. Meeting the scope and standards of practice for pediatric nursing 3. Being a nurse practitioner or clinical nurse specialist 4. Acting on the advice of the nurse manager Answer: 2 Explanation: 1. Meeting the scope and standards of practice for pediatric nursing would cover the pediatric nurse against an accusation of malpractice or negligence because the standards are rigorous and cover all bases of excellent nursing practice. Following the physician's written orders or acting on the advice of the nurse manager are not enough to defend the nurse from accusations because the orders and/or advice may be wrong or unethical. Being a clinical nurse specialist or nurse practitioner does not defend the nurse against these accusations if he or she does not follow the Society of Pediatric Nurses standards of practice. 2. Meeting the scope and standards of practice for pediatric nursing would cover the pediatric nurse against an accusation of malpractice or negligence because the standards are rigorous and cover all bases of excellent nursing practice. Following the physician's written orders or acting on the advice of the nurse manager are not enough to defend the nurse from accusations because the orders and/or advice may be wrong or unethical. Being a clinical nurse specialist or nurse practitioner does not defend the nurse against these accusations if he or she does not follow the Society of Pediatric Nurses standards of practice. 3. Meeting the scope and standards of practice for pediatric nursing would cover the pediatric nurse against an accusation of malpractice or negligence because the standards are rigorous and cover all bases of excellent nursing practice. Following the physician's written orders or acting on the advice of the nurse manager are not enough to defend the nurse from accusations because the orders and/or advice may be wrong or unethical. Being a clinical nurse specialist or nurse practitioner does not defend the nurse against these accusations if he or she does not follow the Society of Pediatric Nurses standards of practice. 4. Meeting the scope and standards of practice for pediatric nursing would cover the pediatric nurse against an accusation of malpractice or negligence because the standards are rigorous and cover all bases of excellent nursing practice. Following the physician's written orders or acting on the advice of the nurse manager are not enough to defend the nurse from accusations because the orders and/or advice may be wrong or unethical. Being a clinical nurse specialist or nurse practitioner does not defend the nurse against these accusations if he or she does not follow the Society of Pediatric Nurses standards of practice. Page Ref: 9-11 Cognitive Level: Analyzing Client Need &Sub: Safe and Effective Care Environment: Management of Care Standards: QSEN Competencies: Patient-centered care | AACN Essential Competencies: Essential V: Healthcare policy, finance, and regulatory environments | NLN Competencies: Professional identity | Nursing/Integrated Concepts: Nursing Process: Planning/Ethics Learning Outcome: LO 1.6 Examine three unique pediatric legal and ethical issues in pediatric nursing practice. MNL LO: Pediatric nursing care 14) Which legal or ethical offense would be committed if a nurse tells family members the condition of a newborn baby without first consulting the parents? 1. A breach of privacy 2. Negligence 3. Malpractice 4. A breach of ethics Answer: 1 Explanation: 1. A breach of privacy would have been committed in this situation, because it violates the right to privacy of this family. The right to privacy is the right of a person to keep his or her person and property free from public scrutiny, including other family members. Negligence and malpractice are punishable legal offenses and are more serious. A breach of ethics would not apply to this situation. 2. A breach of privacy would have been committed in this situation, because it violates the right to privacy of this family. The right to privacy is the right of a person to keep his or her person and property free from public scrutiny, including other family members. Negligence and malpractice are punishable legal offenses and are more serious. A breach of ethics would not apply to this situation. 3. A breach of privacy would have been committed in this situation, because it violates the right to privacy of this family. The right to privacy is the right of a person to keep his or her person and property free from public scrutiny, including other family members. Negligence and malpractice are punishable legal offenses and are more serious. A breach of ethics would not apply to this situation. 4. A breach of privacy would have been committed in this situation, because it violates the right to privacy of this family. The right to privacy is the right of a person to keep his or her person and property free from public scrutiny, including other family members. Negligence and malpractice are punishable legal offenses and are more serious. A breach of ethics would not apply to this situation. Page Ref: 9-11 Cognitive Level: Applying Client Need &Sub: Safe and Effective Care Environment: Management of Care Standards: QSEN Competencies: Patient-centered care | AACN Essential Competencies: Essential V: Healthcare policy, finance, and regulatory environments | NLN Competencies: Professional identity | Nursing/Integrated Concepts: Nursing Process: Implementation/Ethics Learning Outcome: LO 1.6 Examine three unique pediatric legal and ethical issues in pediatric nursing practice. MNL LO: Family-centered care 15) Pediatric nurses have foundational knowledge obtained in nursing school and add specific competencies related to the pediatric client. Which would be considered an additional specific expected competency of the pediatric nurse? 1. Physical assessment 2. Anatomical and developmental differences 3. Nursing process 4. Management of healthcare conditions Answer: 2 Explanation: 1. Assessing anatomical and developmental differences would be a specific expected competency for the pediatric nurse that would not be learned in nursing school. Physical assessment, nursing process, and management of health conditions are all foundational knowledge learned in nursing school. 2. Assessing anatomical and developmental differences would be a specific expected competency for the pediatric nurse that would not be learned in nursing school. Physical assessment, nursing process, and management of health conditions are all foundational knowledge learned in nursing school. 3. Assessing anatomical and developmental differences would be a specific expected competency for the pediatric nurse that would not be learned in nursing school. Physical assessment, nursing process, and management of health conditions are all foundational knowledge learned in nursing school. 4. Assessing anatomical and developmental differences would be a specific expected competency for the pediatric nurse that would not be learned in nursing school. Physical assessment, nursing process, and management of health conditions are all foundational knowledge learned in nursing school. Page Ref: 2-4 Cognitive Level: Applying Client Need &Sub: Safe and Effective Care Environment: Management of Care Standards: QSEN Competencies: Patient-centered care | AACN Essential Competencies: Essential II: Basic organizational and systems leadership for quality care and patient safety | NLN Competencies: Nursing judgement | Nursing/Integrated Concepts: Nursing Process: Assessment/Education Learning Outcome: LO 1.2 Compare the roles of nurses in child healthcare. MNL LO: Pediatric nursing care 16) Which of the following are components of family-centered care? Select all that apply. 1. Recognizing and building on family strengths 2. Meeting the emotional, social, and developmental needs of the child and family 3. Respect all parenting practices 4. Support all cultural practices 5. Encourage parent-to-parent support Answer: 1, 2, 5 Explanation: 1. Recognizing and building on family strengths are one of the components of family-centered care. 2. Meeting the emotional, social, and developmental needs of the child and family are included in the components of family-centered care. 3. Respecting all parenting practices is not one of the components of family-centered care. 4. Supporting all cultural practices is not one of the components of family-centered care. 5. Encouraging parent-to-parent support is one of the components of family-centered care. Page Ref: 6 Cognitive Level: Applying Client Need &Sub: Safe and Effective Care Environment: Management of Care Standards: QSEN Competencies: Patient-centered care | AACN Essential Competencies: Essential II: Basic organizational and systems leadership for quality care and patient safety | NLN Competencies: Human flourishing | Nursing/Integrated Concepts: Nursing Process: Intervention/Coordination of care Learning Outcome: LO 1.1 Describe the continuum of pediatric healthcare. MNL LO: Family-centered care 17) A three-week-old infant is returned post-pyloromyotomy three hours ago. The father is refusing pain medication for the infant and states, "The baby is hungry. Can I give the baby a bottle?" How should the nurse best advocate for the infant? Select all that apply. 1. Call the physician to ask if the child can feed yet. 2. The FLACC scale rating is 8 out of 10; try swaddling and rocking the infant. 3. Ask the parent to obtain a FLACC scale rating and let the nurse know what rating they get. 4. Educate the parent about the surgery and why the infant should not have anything by mouth. 5. Inform the parent about the meaning of the pain scale and the need for pain medication. Answer: 4, 5 Explanation: 1. Calling the physician to ask if the infant can feed yet is not the best way to advocate for the infant. 2. Swaddling and rocking the infant may calm the child but is not the best way to advocate for the infant. 3. Asking the parent to obtain a FLACC scale rating and let the nurse know what rating they get. This is not the parents' duty. It is the nurse's responsibility to assess pain. 4. Educating the parent about the surgery and why the infant should not have anything by mouth is a good way to advocate for the infant. 5. Informing the parent about the meaning of the pain scale and the need for pain medication is a good way to advocate for the infant. Page Ref: 3 Cognitive Level: Analyzing Client Need &Sub: Safe and Effective Care Environment: Management of Care Standards: QSEN Competencies: Patient-centered care | AACN Essential Competencies: Essential II: Basic organizational and systems leadership for quality care and patient safety | NLN Competencies: Human flourishing | Nursing/Integrated Concepts: Nursing Process: Assessment/Health teaching and health promotion Learning Outcome: LO 1.2 Compare the roles of nurses in child healthcare. MNL LO: Developmentally appropriate care Principles of Pediatric Nursing: Caring for Children, 7e (Ball et al.) Chapter 2 Family-Centered Care and Cultural Considerations 1) The nurse is planning care for an adolescent client who will be hospitalized for several weeks following a traumatic brain injury. Which interventions will enhance family-centered care for this client and family? Select all that apply. 1. Making all ADL decisions for the adolescent and family 2. Asking the adolescent what foods to include during meal time 3. Allowing the family time to pray each day with the adolescent 4. Encouraging the adolescent's friends to visit during visiting hours 5. Leaving all questions for the healthcare provider Answer: 2, 3, 4 Explanation: 1. Interventions that will enhance family-centered care for this client and family include asking the adolescent to be an active member of care by making food choices, allowing the family to pray each day with the adolescent, and encouraging the adolescent's friends to visit during visiting hours. These interventions each promote the concepts of family-centered care. Making all decisions for the adolescent and family and leaving all questions for the healthcare provider do not promote the concepts of family-centered care. 2. Interventions that will enhance family-centered care for this client and family include asking the adolescent to be an active member of care by making food choices, allowing the family to pray each day with the adolescent, and encouraging the adolescent's friends to visit during visiting hours. These interventions each promote the concepts of family-centered care. Making all decisions for the adolescent and family and leaving all questions for the healthcare provider do not promote the concepts of family-centered care. 3. Interventions that will enhance family-centered care for this client and family include asking the adolescent to be an active member of care by making food choices, allowing the family to pray each day with the adolescent, and encouraging the adolescent's friends to visit during visiting hours. These interventions each promote the concepts of family-centered care. Making all decisions for the adolescent and family and leaving all questions for the healthcare provider do not promote the concepts of family-centered care. 4. Interventions that will enhance family-centered care for this client and family include asking the adolescent to be an active member of care by making food choices, allowing the family to pray each day with the adolescent, and encouraging the adolescent's friends to visit during visiting hours. These interventions each promote the concepts of family-centered care. Making all decisions for the adolescent and family and leaving all questions for the healthcare provider do not promote the concepts of family-centered care. 5. Interventions that will enhance family-centered care for this client and family include asking the adolescent to be an active member of care by making food choices, allowing the family to pray each day with the adolescent, and encouraging the adolescent's friends to visit during visiting hours. These interventions each promote the concepts of family-centered care. Making all decisions for the adolescent and family and leaving all questions for the healthcare provider do not promote the concepts of family-centered care. Page Ref: 18, 19 Cognitive Level: Applying Client Need &Sub: Health Promotion and Maintenance Standards: QSEN Competencies: Patient-centered care | AACN Essential Competencies: Essential II: Basic organizational and systems leadership for quality care and patient safety | NLN Competencies: Nursing judgement | Nursing/Integrated Concepts: Nursing Process: Implementation/Coordination of care Learning Outcome: LO 2.7 Develop a family-centered nursing care plan for the child and family. MNL LO: Examine health promotion, stress reduction therapies, and safety for hospitalized child and family. 2) A new pediatric hospital will open soon. While planning nursing care, the hospital administration is considering two models of providing healthcare: family-focused care and family-centered care. Which action best demonstrates family-centered care? 1. Telling the family what must be done for the family's health 2. Assuming the role of an expert professional to direct the healthcare 3. Intervening for the child and family as a unit 4. Conferring with the family in deciding which healthcare option will be chosen Answer: 4 Explanation: 1. The benefit of employing the family-centered-care philosophy is that the priorities and needs as seen by the family are addressed as a partnership between a family and a nurse develops. In family-focused care, the healthcare worker assumes the role of professional expert while missing the multiple contributions the family brings to the healthcare meeting. 2. The benefit of employing the family-centered-care philosophy is that the priorities and needs as seen by the family are addressed as a partnership between a family and a nurse develops. In family-focused care, the healthcare worker assumes the role of professional expert while missing the multiple contributions the family brings to the healthcare meeting. 3. The benefit of employing the family-centered-care philosophy is that the priorities and needs as seen by the family are addressed as a partnership between a family and a nurse develops. In family-focused care, the healthcare worker assumes the role of professional expert while missing the multiple contributions the family brings to the healthcare meeting. 4. The benefit of employing the family-centered-care philosophy is that the priorities and needs as seen by the family are addressed as a partnership between a family and a nurse develops. In family-focused care, the healthcare worker assumes the role of professional expert while missing the multiple contributions the family brings to the healthcare meeting. Page Ref: 18, 19 Cognitive Level: Applying Client Need &Sub: Psychosocial Integrity Standards: QSEN Competencies: Patient-centered care | AACN Essential Competencies: Essential II: Basic organizational and systems leadership for quality care and patient safety | NLN Competencies: Professional identity | Nursing/Integrated Concepts: Nursing Process: Planning/Coordination of care Learning Outcome: LO 2.1 Describe key concepts of family-centered care. MNL LO: Examine the concepts related to the hospitalization of a child. 3) A school-age client tells you that "Grandpa, Mommy, Daddy, and my brother live at my house." Which type of family will the nurse identify in the medical record based on this description? 1. Binuclear family 2. Extended family 3. Gay or lesbian family 4. Traditional nuclear family Answer: 2 Explanation: 1. An extended family contains a parent or a couple who share the house with their children and another adult relative. A binuclear family includes the divorced parents who have joint custody of their biologic children, while the children alternate spending varying amounts of time in the home of each parent. A gay or lesbian family is comprised of two same-sex domestic partners; they may or may not have children. The traditional nuclear family consists of an employed provider parent, a homemaking parent, and the biologic children of this union. 2. An extended family contains a parent or a couple who share the house with their children and another adult relative. A binuclear family includes the divorced parents who have joint custody of their biologic children, while the children alternate spending varying amounts of time in the home of each parent. A gay or lesbian family is comprised of two same-sex domestic partners; they may or may not have children. The traditional nuclear family consists of an employed provider parent, a homemaking parent, and the biologic children of this union. 3. An extended family contains a parent or a couple who share the house with their children and another adult relative. A binuclear family includes the divorced parents who have joint custody of their biologic children, while the children alternate spending varying amounts of time in the home of each parent. A gay or lesbian family is comprised of two same-sex domestic partners; they may or may not have children. The traditional nuclear family consists of an employed provider parent, a homemaking parent, and the biologic children of this union. 4. An extended family contains a parent or a couple who share the house with their children and another adult relative. A binuclear family includes the divorced parents who have joint custody of their biologic children, while the children alternate spending varying amounts of time in the home of each parent. A gay or lesbian family is comprised of two same-sex domestic partners; they may or may not have children. The traditional nuclear family consists of an employed provider parent, a homemaking parent, and the biologic children of this union. Page Ref: 19, 20 Cognitive Level: Applying Client Need &Sub: Psychosocial Integrity Standards: QSEN Competencies: Patient-centered care | AACN Essential Competencies: Essential IX: Baccalaureate generalist nursing practice | NLN Competencies: Professional identity | Nursing/Integrated Concepts: Nursing Process: Assessment/Coordination of care Learning Outcome: LO 2.2 Compare the characteristics of different types of families. MNL LO: Examine health promotion, stress reduction therapies, and safety for hospitalized child and family. 4) The nurse is performing an assessment of a child's biologic family history. Which situation would necessitate the nurse's asking the mother for information should use the term "child's father" instead of "your husband"? 1. Traditional nuclear family 2. Traditional extended family 3. Two-income nuclear family 4. Cohabitating informal stepfamily Answer: 4 Explanation: 1. The mother from the cohabitating informal stepfamily does not have a husband; the nurse should be asking about the "child's father." In the traditional nuclear family, the traditional extended family, and the two-income nuclear family, the child's father is the same person as the mother's husband. 2. The mother from the cohabitating informal stepfamily does not have a husband; the nurse should be asking about the "child's father." In the traditional nuclear family, the traditional extended family, and the two-income nuclear family, the child's father is the same person as the mother's husband. 3. The mother from the cohabitating informal stepfamily does not have a husband; the nurse should be asking about the "child's father." In the traditional nuclear family, the traditional extended family, and the two-income nuclear family, the child's father is the same person as the mother's husband. 4. The mother from the cohabitating informal stepfamily does not have a husband; the nurse should be asking about the "child's father." In the traditional nuclear family, the traditional extended family, and the two-income nuclear family, the child's father is the same person as the mother's husband. Page Ref: 19, 20 Cognitive Level: Applying Client Need &Sub: Psychosocial Integrity Standards: QSEN Competencies: Patient-centered care | AACN Essential Competencies: Essential IX: Baccalaureate generalist nursing practice | NLN Competencies: Nursing judgement | Nursing/Integrated Concepts: Nursing Process: Assessment/Coordination of care Learning Outcome: LO 2.2 Compare the characteristics of different types of families. MNL LO: Examine the role of the nurse in promoting culturally competent family-centered care. 5) Several children arrived at the emergency department accompanied by their fathers. Which father may legally sign emergency medical consent for treatment? 1. The divorced one from the binuclear family 2. The stepfather from the blended or reconstituted family 3. The divorced one when the single-parent mother has custody 4. The nonbiologic one from the heterosexual cohabitating family Answer: 1 Explanation: 1. The divorced father from the binuclear family may sign informed consent because he has equal legal rights with the mother under joint-custody arrangements. The nonbiologic stepfather from the blended or reconstituted family, the divorced biologic father when the single-parent mother has custody, and the nonbiologic father from the heterosexual cohabitating family are without legal authority to seek emergency medical care for the child. 2. The divorced father from the binuclear family may sign informed consent because he has equal legal rights with the mother under joint-custody arrangements. The nonbiologic stepfather from the blended or reconstituted family, the divorced biologic father when the single-parent mother has custody, and the nonbiologic father from the heterosexual cohabitating family are without legal authority to seek emergency medical care for the child. 3. The divorced father from the binuclear family may sign informed consent because he has equal legal rights with the mother under joint-custody arrangements. The nonbiologic stepfather from the blended or reconstituted family, the divorced biologic father when the single-parent mother has custody, and the nonbiologic father from the heterosexual cohabitating family are without legal authority to seek emergency medical care for the child. 4. The divorced father from the binuclear family may sign informed consent because he has equal legal rights with the mother under joint-custody arrangements. The nonbiologic stepfather from the blended or reconstituted family, the divorced biologic father when the single-parent mother has custody, and the nonbiologic father from the heterosexual cohabitating family are without legal authority to seek emergency medical care for the child. Page Ref: 20 Cognitive Level: Applying Client Need &Sub: Psychosocial Integrity Standards: QSEN Competencies: Patient-centered care | AACN Essential Competencies: Essential II: Basic organizational and systems leadership for quality care and patient safety | NLN Competencies: Professional identity | Nursing/Integrated Concepts: Nursing Process: Planning/Coordination of care Learning Outcome: LO 2.2 Compare the characteristics of different types of families. MNL LO: Examine the role of the nurse in promoting culturally competent family-centered care. 6) The community health nurse is assessing several families for various strengths and needs in regard to after-school and backup childcare arrangements. Which family type will benefit the most from this assessment and subsequent interventions? 1. The binuclear family 2. The extended family 3. The single-parent family 4. The traditional nuclear family Answer: 3 Explanation: 1. The single-parent family most typically lacks social, emotional, and financial resources. Nursing considerations for such families should include referrals to options that will enable the parent to fulfill work commitments while providing the child with access to resources that can support the child's growth and development. The binuclear family, the extended family, and the traditional nuclear family generally have at least two adults who can share in the care and the nurturing of its children. 2. The single-parent family most typically lacks social, emotional, and financial resources. Nursing considerations for such families should include referrals to options that will enable the parent to fulfill work commitments while providing the child with access to resources that can support the child's growth and development. The binuclear family, the extended family, and the traditional nuclear family generally have at least two adults who can share in the care and the nurturing of its children. 3. The single-parent family most typically lacks social, emotional, and financial resources. Nursing considerations for such families should include referrals to options that will enable the parent to fulfill work commitments while providing the child with access to resources that can support the child's growth and development. The binuclear family, the extended family, and the traditional nuclear family generally have at least two adults who can share in the care and the nurturing of its children. 4. The single-parent family most typically lacks social, emotional, and financial resources. Nursing considerations for such families should include referrals to options that will enable the parent to fulfill work commitments while providing the child with access to resources that can support the child's growth and development. The binuclear family, the extended family, and the traditional nuclear family generally have at least two adults who can share in the care and the nurturing of its children. Page Ref: 19, 20 Cognitive Level: Applying Client Need &Sub: Psychosocial Integrity Standards: QSEN Competencies: Patient-centered care | AACN Essential Competencies: Essential VII: Clinical prevention and population health | NLN Competencies: Nursing judgement | Nursing/Integrated Concepts: Nursing Process: Assessment/Health teaching and health promotion Learning Outcome: LO 2.2 Compare the characteristics of different types of families. MNL LO: Apply key concepts of family-centered care. 7) The nurse is working on parenting skills with a group of mothers. Which mother would need the fewest discipline-related suggestions? 1. Authoritarian 2. Authoritative 3. Indifferent 4. Permissive Answer: 2 Explanation: 1. The parental style that results in positive outcomes for the behavior and learning of its children is the authoritative style. Nurses have observed that children from homes using this parental style more frequently have personalities manifesting self-reliance, self-control, and social competence. These parents should be praised for using the preferred approach. Children in the authoritarian parenting family are denied opportunity to develop some skills in the areas of self-direction, communication, and negotiation. Under the permissive parental style, children do not learn the socially acceptable limits of behaviors. The indifferent parental style results in children who often exhibit destructive behaviors and delinquency. 2. The parental style that results in positive outcomes for the behavior and learning of its children is the authoritative style. Nurses have observed that children from homes using this parental style more frequently have personalities manifesting self-reliance, self-control, and social competence. These parents should be praised for using the preferred approach. Children in the authoritarian parenting family are denied opportunity to develop some skills in the areas of self-direction, communication, and negotiation. Under the permissive parental style, children do not learn the socially acceptable limits of behaviors. The indifferent parental style results in children who often exhibit destructive behaviors and delinquency. 3. The parental style that results in positive outcomes for the behavior and learning of its children is the authoritative style. Nurses have observed that children from homes using this parental style more frequently have personalities manifesting self-reliance, self-control, and social competence. These parents should be praised for using the preferred approach. Children in the authoritarian parenting family are denied opportunity to develop some skills in the areas of self-direction, communication, and negotiation. Under the permissive parental style, children do not learn the socially acceptable limits of behaviors. The indifferent parental style results in children who often exhibit destructive behaviors and delinquency. 4. The parental style that results in positive outcomes for the behavior and learning of its children is the authoritative style. Nurses have observed that children from homes using this parental style more frequently have personalities manifesting self-reliance, self-control, and social competence. These parents should be praised for using the preferred approach. Children in the authoritarian parenting family are denied opportunity to develop some skills in the areas of self-direction, communication, and negotiation. Under the permissive parental style, children do not learn the socially acceptable limits of behaviors. The indifferent parental style results in children who often exhibit destructive behaviors and delinquency. Page Ref: 23 Cognitive Level: Applying Client Need &Sub: Psychosocial Integrity Standards: QSEN Competencies: Patient-centered care | AACN Essential Competencies: Essential VII: Clinical prevention and population health | NLN Competencies: Nursing judgement | Nursing/Integrated Concepts: Nursing Process: Implementation/Health teaching and health promotion Learning Outcome: LO 2.3 Contrast four different parental styles and analyze their impact on child personality development. MNL LO: Apply key concepts of family-centered care. 8) The nurse in the pediatric clinic observes a parental lack of warmth and interest toward the child. Which parental style will the nurse most likely document in this situation? 1. Authoritarian 2. Authoritative 3. Indifferent 4. Permissive Answer: 3 Explanation: 1. Parents displaying the indifferent parental style fail to demonstrate consistent warmth and interest in their children. Parents who favor the authoritarian style may exhibit a punitive attitude toward the child who is misbehaving but are not disinterested. Parents employing the authoritative style and the permissive style have children who report that "my parent loves me and shows affection regularly." 2. Parents displaying the indifferent parental style fail to demonstrate consistent warmth and interest in their children. Parents who favor the authoritarian style may exhibit a punitive attitude toward the child who is misbehaving but are not disinterested. Parents employing the authoritative style and the permissive style have children who report that "my parent loves me and shows affection regularly." 3. Parents displaying the indifferent parental style fail to demonstrate consistent warmth and interest in their children. Parents who favor the authoritarian style may exhibit a punitive attitude toward the child who is misbehaving but are not disinterested. Parents employing the authoritative style and the permissive style have children who report that "my parent loves me and shows affection regularly." 4. Parents displaying the indifferent parental style fail to demonstrate consistent warmth and interest in their children. Parents who favor the authoritarian style may exhibit a punitive attitude toward the child who is misbehaving but are not disinterested. Parents employing the authoritative style and the permissive style have children who report that "my parent loves me and shows affection regularly." Page Ref: 23 Cognitive Level: Applying Client Need &Sub: Psychosocial Integrity Standards: QSEN Competencies: Patient-centered care | AACN Essential Competencies: Essential II: Basic organizational and systems leadership for quality care and patient safety | NLN Competencies: Nursing judgement | Nursing/Integrated Concepts: Nursing Process: Assessment/Education Learning Outcome: LO 2.3 Contrast four different parental styles and analyze their impact on child personality development. MNL LO: Develop a family-centered nursing care plan for the child and family. 9) The nurse is working on parenting skills with a mother of three children. The nurse demonstrates a strategy that uses reward to increase positive behavior. Which strategy will the nurse document in the medical record based on this description? 1. Time out 2. Reasoning 3. Behavior modification 4. Experiencing consequences of misbehavior Answer: 3 Explanation: 1. Behavior modification identifies and gives rewards for desired behaviors. Time out and experiencing consequences of misbehavior show the child that unacceptable behavior brings undesirable outcomes. Reasoning attempts to use explanation to end misbehavior. 2. Behavior modification identifies and gives rewards for desired behaviors. Time out and experiencing consequences of misbehavior show the child that unacceptable behavior brings undesirable outcomes. Reasoning attempts to use explanation to end misbehavior. 3. Behavior modification identifies and gives rewards for desired behaviors. Time out and experiencing consequences of misbehavior show the child that unacceptable behavior brings undesirable outcomes. Reasoning attempts to use explanation to end misbehavior. 4. Behavior modification identifies and gives rewards for desired behaviors. Time out and experiencing consequences of misbehavior show the child that unacceptable behavior brings undesirable outcomes. Reasoning attempts to use explanation to end misbehavior. Page Ref: 24 Cognitive Level: Applying Client Need &Sub: Psychosocial Integrity Standards: QSEN Competencies: Patient-centered care | AACN Essential Competencies: Essential IX: Baccalaureate generalist nursing practice | NLN Competencies: Nursing judgement | Nursing/Integrated Concepts: Nursing Process: Implementation/Health teaching and health promotion Learning Outcome: LO 2.3 Contrast four different parental styles and analyze their impact on child personality development. MNL LO: Develop a family-centered nursing care plan for the child and family. 10) The nurse is assessing a family's effective coping strategies and ineffective defensive strategies. Which family-social-system theory is the nurse using in this assessment of the family? 1. Family-stress theory 2. Family-development theory 3. Family-systems theory 4. Family life-cycle theory Answer: 1 Explanation: 1. Family-stress theory indicates an array of coping strategies that effectively help reduce stress, in contrast with the defensive strategies of dysfunctional families. Family- development theory suggests developmental tasks for families in each stage. Family-systems theory looks at the relationships among and between family members and the environment. The family life cycle is not a family social system theory. 2. Family-stress theory indicates an array of coping strategies that effectively help reduce stress, in contrast with the defensive strategies of dysfunctional families. Family-development theory suggests developmental tasks for families in each stage. Family-systems theory looks at the relationships among and between family members and the environment. The family life cycle is not a family social system theory. 3. Family-stress theory indicates an array of coping strategies that effectively help reduce stress, in contrast with the defensive strategies of dysfunctional families. Family-development theory suggests developmental tasks for families in each stage. Family-systems theory looks at the relationships among and between family members and the environment. The family life cycle is not a family social system theory. 4. Family-stress theory indicates an array of coping strategies that effectively help reduce stress, in contrast with the defensive strategies of dysfunctional families. Family-development theory suggests developmental tasks for families in each stage. Family-systems theory looks at the relationships among and between family members and the environment. The family life cycle is not a family social system theory. Page Ref: 29, 30 Cognitive Level: Applying Client Need &Sub: Psychosocial Integrity Standards: QSEN Competencies: Patient-centered care | AACN Essential Competencies: Essential IX: Baccalaureate generalist nursing practice | NLN Competencies: Human flourishing | Nursing/Integrated Concepts: Nursing Process: Assessment/Health teaching and health promotion Learning Outcome: LO 2.5 List the categories of family strengths that help families develop and cope with stressors. MNL LO: Compare and contrast known family theories and assessment strategies. 11) The nurse is assigned to a child in a spica cast for a fractured femur suffered in an automobile accident. The child's teenage brother was driving the car, which was totaled. The nurse learns that the father lost his job three weeks ago and the mother has just accepted a temporary waitress job. Which nursing diagnosis will the nurse use when planning care for this child and family? 1. Compromised Family Coping Related to the Effects of Multiple Simultaneous Stressors 2. Impaired Social Interaction (Parent and Child) Related to the Lack of Family or Respite Support 3. Interrupted Family Processes Related to Child with Significant Disability Requiring Alteration in Family Functioning 4. Risk for Caregiver Role Strain Related to Child with a Newly Acquired Disability and the Associated Financial Burden Answer: 1 Explanation: 1. Compromised Family Coping Related to the Effects of Multiple Simultaneous Stressors best fits the multiple crises to which this family is responding. The spica cast may require alteration in family functioning; however, fractures are generally not considered a significant long-term disability. Lack of family members and lack of respite support was not mentioned in the scenario. 2. Compromised Family Coping Related to the Effects of Multiple Simultaneous Stressors best fits the multiple crises to which this family is responding. The spica cast may require alteration in family functioning; however, fractures are generally not considered a significant long-term disability. Lack of family members and lack of respite support was not mentioned in the scenario. 3. Compromised Family Coping Related to the Effects of Multiple Simultaneous Stressors best fits the multiple crises to which this family is responding. The spica cast may require alteration in family functioning; however, fractures are generally not considered a significant long-term disability. Lack of family members and lack of respite support was not mentioned in the scenario. 4. Compromised Family Coping Related to the Effects of Multiple Simultaneous Stressors best fits the multiple crises to which this family is responding. The spica cast may require alteration in family functioning; however, fractures are generally not considered a significant long-term disability. Lack of family members and lack of respite support was not mentioned in the scenario. Page Ref: 29, 30 Cognitive Level: Applying Client Need &Sub: Psychosocial Integrity Standards: QSEN Competencies: Patient-centered care | AACN Essential Competencies: Essential IX: Baccalaureate generalist nursing practice | NLN Competencies: Nursing judgement | Nursing/Integrated Concepts: Nursing Process: Diagnosis/Coordination of care Learning Outcome: LO 2.6 Summarize the advantages of using a family or cultural assessment tool. MNL LO: Develop a family-centered nursing care plan for the child and family. 12) A nurse is working with a pediatric client. When obtaining an accurate family assessment, which initial step is the most appropriate? 1. Establish a trusting relationship with the family. 2. Select the most relevant family-assessment tool. 3. Focus primarily upon the mother, while learning her greatest concern. 4. Observe the family in the home setting, since this step always proves indispensable. Answer: 1 Explanation: 1. Establishment of a trusting relationship between the family and the nurse is the essential preliminary step in obtaining an accurate family assessment. There is benefit when the tool used matches the family's strengths and resources; however, selecting the most relevant family-assessment tool is not indispensable to accuracy in the assessment. Focusing primarily upon the mother while learning her greatest concern is counterproductive and prevents the nurse from acknowledging multiple perceptions held by the family's members. Observing the family in the home setting is only recommended in some cases. 2. Establishment of a trusting relationship between the family and the nurse is the essential preliminary step in obtaining an accurate family assessment. There is benefit when the tool used matches the family's strengths and resources; however, selecting the most relevant family- assessment tool is not indispensable to accuracy in the assessment. Focusing primarily upon the mother while learning her greatest concern is counterproductive and prevents the nurse from acknowledging multiple perceptions held by the family's members. Observing the family in the home setting is only recommended in some cases. 3. Establishment of a trusting relationship between the family and the nurse is the essential preliminary step in obtaining an accurate family assessment. There is benefit when the tool used matches the family's strengths and resources; however, selecting the most relevant family- assessment tool is not indispensable to accuracy in the assessment. Focusing primarily upon the mother while learning her greatest concern is counterproductive and prevents the nurse from acknowledging multiple perceptions held by the family's members. Observing the family in the home setting is only recommended in some cases. 4. Establishment of a trusting relationship between the family and the nurse is the essential preliminary step in obtaining an accurate family assessment. There is benefit when the tool used matches the family's strengths and resources; however, selecting the most relevant family- assessment tool is not indispensable to accuracy in the assessment. Focusing primarily upon the mother while learning her greatest concern is counterproductive and prevents the nurse from acknowledging multiple perceptions held by the family's members. Observing the family in the home setting is only recommended in some cases. Page Ref: 31 Cognitive Level: Applying Client Need &Sub: Psychosocial Integrity Standards: QSEN Competencies: Patient-centered care | AACN Essential Competencies: Essential IX: Baccalaureate generalist nursing practice | NLN Competencies: Human flourishing | Nursing/Integrated Concepts: Nursing Process: Assessment/Coordination of care Learning Outcome: LO 2.6 Summarize the advantages of using a family or cultural assessment tool. MNL LO: Apply key concepts of family-centered care. 13) The nurse is assessing a group of children attending summer camp. The nurse will expect which children to most likely have problems perceiving a sense of belonging? 1. Children whose parents divorced recently 2. Children who gained a stepparent recently 3. Children recently placed into foster care 4. Children adopted as infants Answer: 3 Explanation: 1. Children in foster care are more likely to have problems perceiving a sense of belonging. Children whose parents divorce often fear abandonment. Children who gain a stepparent may have problems trusting the new parent. Infants who are adopted at birth can have minimal problems with acceptance when parents follow preadoption counseling about disclosure. 2. Children in foster care are more likely to have problems perceiving a sense of belonging. Children whose parents divorce often fear abandonment. Children who gain a stepparent may have problems trusting the new parent. Infants who are adopted at birth can have minimal problems with acceptance when parents follow preadoption counseling about disclosure. 3. Children in foster care are more likely to have problems perceiving a sense of belonging. Children whose parents divorce often fear abandonment. Children who gain a stepparent may have problems trusting the new parent. Infants who are adopted at birth can have minimal problems with acceptance when parents follow preadoption counseling about disclosure. 4. Children in foster care are more likely to have problems perceiving a sense of belonging. Children whose parents divorce often fear abandonment. Children who gain a stepparent may have problems trusting the new parent. Infants who are adopted at birth can have minimal problems with acceptance when parents follow preadoption counseling about disclosure. Page Ref: 27 Cognitive Level: Applying Client Need &Sub: Psychosocial Integrity Standards: QSEN Competencies: Patient-centered care | AACN Essential Competencies: Essential IX: Baccalaureate generalist nursing practice | NLN Competencies: Nursing judgement | Nursing/Integrated Concepts: Nursing Process: Assessment/Coordination of care Learning Outcome: LO 2.4 Explain the effects of major family changes on children. MNL LO: Examine the role of the nurse in promoting culturally competent family-centered care. 14) There are several tools that help with obtaining a cultural assessment of a client and his family. Which tool would be appropriate to gather 12 major concepts of cultural assessment? 1. Sunrise enabler 2. Model for cultural competence 3. Transcultural assessment model 4. Health traditions model Answer: 2 Explanation: 1. The sunrise enabler examines influences on care and culture. The model for cultural competence will gather information on 12 major concepts. The transcultural assessment model is based on 6 phenomena. The health traditions model is predicated on holistic health. 2. The sunrise enabler examines influences on care and culture. The model for cultural competence will gather information on 12 major concepts. The transcultural assessment model is based on 6 phenomena. The health traditions model is predicated on holistic health. 3. The sunrise enabler examines influences on care and culture. The model for cultural competence will gather information on 12 major concepts. The transcultural assessment model is based on 6 phenomena. The health traditions model is predicated on holistic health. 4. The sunrise enabler examines influences on care and culture. The model for cultural competence will gather information on 12 major concepts. The transcultural assessment model is based on 6 phenomena. The health traditions model is predicated on holistic health. Page Ref: 34 Cognitive Level: Applying Client Need &Sub: Psychosocial Integrity Standards: QSEN Competencies: Patient-centered care | AACN Essential Competencies: Essential IX: Baccalaureate generalist nursing practice | NLN Competencies: Nursing judgement | Nursing/Integrated Concepts: Nursing Process: Assessment/Coordination of care Learning Outcome: LO 2.6 Summarize the advantages of using a family or cultural assessment tool. MNL LO: Examine the role of the nurse in promoting culturally competent family-centered care. 15) Cultures have many different childrearing practices. Which culture is known to value the male child more than the female child, and often teaches children to avoid displaying emotion? 1. Mexican 2. Amish 3. Chinese 4. Navajo Answer: 3 Explanation: 1. The Chinese culture values the male child more than the female child, and often teaches children to avoid showing emotion. The other cultures do not have this component. 2. The Chinese culture values the male child more than the female child, and often teaches children to avoid showing emotion. The other cultures do not have this component. 3. The Chinese culture values the male child more than the female child, and often teaches children to avoid showing emotion. The other cultures do not have this component. 4. The Chinese culture values the male child more than the female child, and often teaches children to avoid showing emotion. The other cultures do not have this component. Page Ref: 35 Cognitive Level: Applying Client Need &Sub: Psychosocial Integrity Standards: QSEN Competencies: Patient-centered care | AACN Essential Competencies: Essential IX: Baccalaureate generalist nursing practice | NLN Competencies: Professional identity | Nursing/Integrated Concepts: Nursing Process: Assessment/Health teaching and health promotion Learning Outcome: LO 2.8 Describe cultural influences on the family's beliefs about health, illness, and treatments. MNL LO: Examine the role of the nurse in promoting culturally competent family-centered care. 16) The nurse is planning care for a school-age client and family who have expressed wanting to use complementary and alternative modalities (CAM) in the treatment plan. Which interventions can the nurse safely implement into the plan of care? Select all that apply. 1. Substituting an herbal remedy for a prescribed medication 2. Encouraging the parents to share which modalities they would like to implement 3. Educating on the benefits and risks for each modality 4. Using essential oils to decrease nausea 5. Discouraging the use of faith-based therapies Answer: 2, 3, 4 Explanation: 1. Appropriate interventions for this client and family include encouraging the parents to share which modalities they want to implement, educating about the risks and benefits of each modality, and using modalities that are safe, such the use of essential oils to decrease nausea. An herbal remedy should not be substituted for a prescribed medication, but can be used if deemed safe with the prescribed medication. Discouraging the use of faith-based therapies does not support the client and family who want to use CAM in the treatment plan. 2. Appropriate interventions for this client and family include encouraging the parents to share which modalities they want to implement, educating about the risks and benefits of each modality, and using modalities that are safe, such the use of essential oils to decrease nausea. An herbal remedy should not be substituted for a prescribed medication, but can be used if deemed safe with the prescribed medication. Discouraging the use of faith-based therapies does not support the client and family who want to use CAM in the treatment plan. 3. Appropriate interventions for this client and family include encouraging the parents to share which modalities they want to implement, educating about the risks and benefits of each modality, and using modalities that are safe, such the use of essential oils to decrease nausea. An herbal remedy should not be substituted for a prescribed medication, but can be used if deemed safe with the prescribed medication. Discouraging the use of faith-based therapies does not support the client and family who want to use CAM in the treatment plan. 4. Appropriate interventions for this client and family include encouraging the parents to share which modalities they want to implement, educating about the risks and benefits of each modality, and using modalities that are safe, such the use of essential oils to decrease nausea. An herbal remedy should not be substituted for a prescribed medication, but can be used if deemed safe with the prescribed medication. Discouraging the use of faith-based therapies does not support the client and family who want to use CAM in the treatment plan. 5. Appropriate interventions for this client and family include encouraging the parents to share which modalities they want to implement, educating about the risks and benefits of each modality, and using modalities that are safe, such the use of essential oils to decrease nausea. An herbal remedy should not be substituted for a prescribed medication, but can be used if deemed safe with the prescribed medication. Discouraging the use of faith-based therapies does not support the client and family who want to use CAM in the treatment plan. Page Ref: 38-40 Cognitive Level: Applying Client Need &Sub: Psychosocial Integrity Standards: QSEN Competencies: Patient-centered care | AACN Essential Competencies: Essential VII: Clinical prevention and population health | NLN Competencies: Human flourishing | Nursing/Integrated Concepts: Nursing Process: Implementation/Coordination of care Learning Outcome: LO 2.9 Discuss nursing interventions for providing culturally sensitive and competent care to the child and family. MNL LO: Analyze the role of the nurse and the role of the family in pediatric care. 17) A nurse and the family of an 8-year-old with acute renal failure are reviewing family strengths helpful in managing stressors. Which family strengths should the nurse recommend this family utilize? Select all that apply. 1. Meeting member needs 2. Support by extended family 3. Effective communication 4. Receiving and giving love 5. Prior life experiences Answer: 2, 3, 5 Explanation: 1. Meeting member needs is one of the roles of a family. Strengths that enable families to develop and adapt to stressors include: education, prior experiences, finances, effective communication, collaborative problem solving, emotional awareness, emotional stability and developing shared meaning about the experience. 2. Support by extended family is one of the family strengths. 3. Effective communication is one of the family strengths. 4. Receiving and giving love is one of the roles of a family. Strengths that enable families to develop and adapt to stressors include: education, prior experiences, finances, effective communication, collaborative problem solving, emotional awareness, emotional stability and developing shared meaning about the experience. 5. Prior life experiences are one of the family strengths. Page Ref: 31 Cognitive Level: Applying Client Need &Sub: Psychosocial Integrity Standards: QSEN Competencies: Patient-centered care | AACN Essential Competencies: Essential IX: Baccalaureate generalist nursing practice | NLN Competencies: Human flourishing | Nursing/Integrated Concepts: Nursing Process: Implementation/Coordination of care Learning Outcome: LO 2.5 List the categories of family strengths that help families develop and cope with stressors. MNL LO: Apply key concepts of family-centered care. 18) The nurse is counseling the parents of a 13-year-old regarding the behaviors they may encounter after telling the child about their plans to divorce. Which behaviors could the child demonstrate? Select all that apply. 1. Sorrow 2. Skipping school 3. Risk-taking 4. Withdraw from friends and activities 5. Temper tantrums Answer: 2, 3 Explanation: 1. Preschool behaviors include: fear, anxiety, worry, self-blame, sorrow, grief, anger, regression, searching and questioning, temper tantrums, crankiness and aggression, loneliness, unhappiness, and depression. 2. Adolescent behaviors include: panic, fear, depression, guilt, risk-taking, fear of loneliness and abandonment, denial, anger, sadness, aggressiveness, skipping or dropping out of school, use of drugs and alcohol, and sexual acting out. 3. Adolescent behaviors include: panic, fear, depression, guilt, risk-taking, fear of loneliness and abandonment, denial, anger, sadness, aggressiveness, skipping or dropping out of school, use of drugs and alcohol, and sexual acting out. 4. School age behaviors include: worry, anxiety depression, sadness, insecurity, fantasy, grief, guilt, self-blame, inability to concentrate on schoolwork, lower academic achievement, regression, aggression, confusion, anger resentment, behavioral problems at school and home, manipulation of parents, withdrawal from friends and activities, fear, and loneliness. 5. Preschool behaviors include: fear, anxiety, worry, self-blame, sorrow, grief, anger, regression, searching and questioning, temper tantrums, crankiness and aggression, loneliness, unhappiness, and depression. Page Ref: 25, 26 Cognitive Level: Applying Client Need &Sub: Psychosocial Integrity Standards: QSEN Competencies: Patient-centered care | AACN Essential Competencies: Essential IX: Baccalaureate generalist nursing practice | NLN Competencies: Human flourishing | Nursing/Integrated Concepts: Nursing Process: Implementation/Coordination of care Learning Outcome: LO 2.4 Explain the effects of major family changes on children. MNL LO: Compare the developmental stages for pediatric clients. Principles of Pediatric Nursing: Caring for Children, 7e (Ball et al.) Chapter 3 Genetic and Genomic Influences 1) Personalized healthcare for health promotion and maintenance can be based on environmental factors and which other item? 1. The genes a person inherited 2. Common conditions with known treatment strategies 3. Teaching strategies 4. The health of the person Answer: 1 Explanation: 1. Personalized healthcare is based on environmental factors and the genes the person inherited. Common conditions and the current health of the person are not part of personalized healthcare. Teaching strategies are not part of personalized healthcare. 2. Personalized healthcare is based on environmental factors and the genes the person inherited. Common conditions and the current health of the person are not part of personalized healthcare. Teaching strategies are not part of personalized healthcare. 3. Personalized healthcare is based on environmental factors and the genes the person inherited. Common conditions and the current health of the person are not part of personalized healthcare. Teaching strategies are not part of personalized healthcare. 4. Personalized healthcare is based on environmental factors and the genes the person inherited. Common conditions and the current health of the person are not part of personalized healthcare. Teaching strategies are not part of personalized healthcare. Page Ref: 49, 50 Cognitive Level: Applying Client Need &Sub: Health Promotion and Maintenance Standards: QSEN Competencies: Patient-centered care | AACN Essential Competencies: Essential VII: Clinical prevention and population health | NLN Competencies: Professional identity | Nursing/Integrated Concepts: Nursing Process: Assessment/Health teaching and health promotion Learning Outcome: LO 3.1 Explain the role of genetic and genomic concepts in health promotion, disease prevention, screening, diagnostics, selection of treatment, and monitoring of treatment effectiveness. MNL LO: Examine the role of the nurse in promoting culturally competent family-centered care. 2) A three-generation pedigree is constructed around the designated "index" patient. Based on this knowledge, which explanation of the term proband is the most accurate? 1. The "index" patient has the disorder of interest. 2. One parent of the "index" patient has the disorder of interest. 3. The "index" patient does not have the disorder of interest. 4. Siblings of the "index" patient do not have the disorder of interest. Answer: 1 Explanation: 1. The proband indicates that the "index" patient has the disorder of interest. A consultand is an "index" patient seeking genetic counseling for a disorder she is not affected by at present. 2. The proband indicates that the "index" patient has the disorder of interest. A consultand is an "index" patient seeking genetic counseling for a disorder she is not affected by at present. 3. The proband indicates that the "index" patient has the disorder of interest. A consultand is an "index" patient seeking genetic counseling for a disorder she is not affected by at present. 4. The proband indicates that the "index" patient has the disorder of interest. A consultand is an "index" patient seeking genetic counseling for a disorder she is not affected by at present. Page Ref: 58 Cognitive Level: Applying Client Need &Sub: Health Promotion and Maintenance Standards: QSEN Competencies: Patient-centered care | AACN Essential Competencies: Essential VII: Clinical prevention and population health | NLN Competencies: Professional identity | Nursing/Integrated Concepts: Nursing Process: Assessment/Health teaching and health promotion Learning Outcome: LO 3.2 Elicit a family health history and construct a genetic pedigree. MNL LO: Examine the role of the nurse in promoting culturally competent family-centered care. 3) When discussing inheritance with parents of a child with a genetic disorder, which statement by the parents indicates they understand inheritance risk? 1. "This child has a genetic disorder, so future children will not have it." 2. "Each pregnancy carries the same percent risk of inheritance." 3. "I cannot have any more children, because they will all have the disorder." 4. "There is a good chance future children will be normal." Answer: 2 Explanation: 1. Each pregnancy carries the same percent risk of having a child with the disorder in question. The other statements indicate the need for further education regarding inheritance risk. 2. Each pregnancy carries the same percent risk of having a child with the disorder in question. The other statements indicate the need for further education regarding inheritance risk. 3. Each pregnancy carries the same percent risk of having a child with the disorder in question. The other statements indicate the need for further education regarding inheritance risk. 4. Each pregnancy carries the same percent risk of having a child with the disorder in question. The other statements indicate the need for further education regarding inheritance risk. Page Ref: 45, 46 Cognitive Level: Analyzing Client Need &Sub: Health Promotion and Maintenance Standards: QSEN Competencies: Patient-centered care | AACN Essential Competencies: Essential VII: Clinical prevention and population health | NLN Competencies: Nursing judgement | Nursing/Integrated Concepts: Nursing Process: Assessment/Health teaching and health promotion Learning Outcome: LO 3.3 Incorporate knowledge of genetic and genomic influences and risk factors into assessment, planning, and implementation of nursing care. MNL LO: Apply the nursing process to the care of the child and family related to test results and diagnosis. 4) Parents of a child with a congenital heart defect ask what the chances are of recurrence in future pregnancies. Which response by the nurse is the most appropriate? 1. "There is a 50 percent chance of recurrence in a future pregnancy." 2. "There is a very low chance of recurrence." 3. "It should not happen again with a future pregnancy." 4. "There is a strong chance of recurrence." Answer: 2 Explanation: 1. There is a very low rate of recurrence with congenital heart defects. The other statements are not appropriate for the nurse to make in this situation. 2. There is a very low rate of recurrence with congenital heart defects. The other statements are not appropriate for the nurse to make in this situation. 3. There is a very low rate of recurrence with congenital heart defects. The other statements are not appropriate for the nurse to make in this situation. 4. There is a very low rate of recurrence with congenital heart defects. The other statements are not appropriate for the nurse to make in this situation. Page Ref: 46, 47 Cognitive Level: Applying Client Need &Sub: Health Promotion and Maintenance Standards: QSEN Competencies: Patient-centered care | AACN Essential Competencies: Essential VII: Clinical prevention and population health | NLN Competencies: Human flourishing | Nursing/Integrated Concepts: Nursing Process: Implementation/Health teaching and health promotion Learning Outcome: LO 3.4 Integrate basic genetic and genomic concepts into child and family education. MNL LO: Cardiovascular and Hematological Disorders/Examine etiology, risk factors, pathophysiology, and clinical manifestations as seen in children. 5) A family with a child who had a cleft lip and palate at birth are planning another pregnancy. What intervention should be recommended prior to conception? 1. A genetic family history 2. A family pedigree 3. A genetic physical assessment 4. A maternal health history Answer: 1 Explanation: 1. A genetic family history is recommended when there is history of a congenital anomaly, such as cleft lip and palate. A pedigree is a more comprehensive family history, and could follow a genetic family history if needed. The previous anomaly is already known, so a genetic history would be recommended over a genetic physical assessment. A maternal health history is not comprehensive enough for this case. 2. A genetic family history is recommended when there is history of a congenital anomaly, such as cleft lip and palate. A pedigree is a more comprehensive family history, and could follow a genetic family history if needed. The previous anomaly is already known, so a genetic history would be recommended over a genetic physical assessment. A maternal health history is not comprehensive enough for this case. 3. A genetic family history is recommended when there is history of a congenital anomaly, such as cleft lip and palate. A pedigree is a more comprehensive family history, and could follow a genetic family history if needed. The previous anomaly is already known, so a genetic history would be recommended over a genetic physical assessment. A maternal health history is not comprehensive enough for this case. 4. A genetic family history is recommended when there is history of a congenital anomaly, such as cleft lip and palate. A pedigree is a more comprehensive family history, and could follow a genetic family history if needed. The previous anomaly is already known, so a genetic history would be recommended over a genetic physical assessment. A maternal health history is not comprehensive enough for this case. Page Ref: 57-59 Cognitive Level: Applying Client Need &Sub: Health Promotion and Maintenance Standards: QSEN Competencies: Patient-centered care | AACN Essential Competencies: Essential VII: Clinical prevention and population health | NLN Competencies: Human flourishing | Nursing/Integrated Concepts: Nursing Process: Planning/Health teaching and health promotion Learning Outcome: LO 3.3 Incorporate knowledge of genetic and genomic influences and risk factors into assessment, planning, and implementation of nursing care. MNL LO: Evaluate healthcare issues related to pediatric nursing care. 6) A father is a known carrier of an X-linked condition, and asks when he will know whether his newborn son has the condition he carries. Which response by the nurse is the most appropriate? 1. "Genetic studies have been ordered, and they will take about a week to determine the results." 2. "We plan to run additional tests this afternoon, and should have results by the end of the day." 3. "Your son cannot have the condition because the condition is X-linked and cannot be passed on to him." 4. "There is a 50 percent chance you passed it on, but further tests are not recommended until he is a month old." Answer: 3 Explanation: 1. A male child does not inherit any X chromosome from the father; therefore, the male child will not have the condition. 2. A male child does not inherit any X chromosome from the father; therefore, the male child will not have the condition. 3. A male child does not inherit any X chromosome from the father; therefore, the male child will not have the condition. 4. A male child does not inherit any X chromosome from the father; therefore, the male child will not have the condition. Page Ref: 53, 54 Cognitive Level: Applying Client Need &Sub: Health Promotion and Maintenance Standards: QSEN Competencies: Patient-centered care | AACN Essential Competencies: Essential VII: Clinical prevention and population health | NLN Competencies: Nursing judgement | Nursing/Integrated Concepts: Nursing Process: Implementation/Health teaching and health promotion Learning Outcome: LO 3.4 Integrate basic genetic and genomic concepts into child and family education. MNL LO: Cellular Alterations/Examine etiology, risk factors, pathophysiology, and clinical manifestations as seen in children. 7) A student nurse asks, "What is carrier testing?" Which response by the nurse educator is most appropriate to answer the student nurse's question? 1. "Carrier testing involves testing an asymptomatic individual for carrier status for a genetic condition." 2. "Carrier testing is used to establish a diagnosis of a genetic disorder in an individual who is symptomatic or has had a positive screening test." 3. "Carrier testing is testing to identify a fetus with a genetic disease or condition. Some prenatal testing is offered routinely; other testing may be initiated due to family history or maternal factors." 4. "Carrier testing follows in vitro fertilization (IVF) testing to identify embryos with a particular genetic condition." Answer: 1 Explanation: 1. Carrier testing involves testing an asymptomatic individual for carrier status for a genetic condition. Diagnostic testing is testing to identify a fetus with a genetic disease or condition. Some prenatal testing is offered routinely; other testing may be initiated due to family history or maternal factors. Prenatal testing is testing to identify a fetus with a genetic disease or condition. Some prenatal testing is offered routinely; other testing may be initiated due to family history or maternal factors. Pre-implantation testing follows in vitro fertilization (IVF) testing to identify embryos with a particular genetic condition. 2. Carrier testing involves testing an asymptomatic individual for carrier status for a genetic condition. Diagnostic testing is testing to identify a fetus with a genetic disease or condition. Some prenatal testing is offered routinely; other testing may be initiated due to family history or maternal factors. Prenatal testing is testing to identify a fetus with a genetic disease or condition. Some prenatal testing is offered routinely; other testing may be initiated due to family history or maternal factors. Pre-implantation testing follows in vitro fertilization (IVF) testing to identify embryos with a particular genetic condition. 3. Carrier testing involves testing an asymptomatic individual for carrier status for a genetic condition. Diagnostic testing is testing to identify a fetus with a genetic disease or condition. Some prenatal testing is offered routinely; other testing may be initiated due to family history or maternal factors. Prenatal testing is testing to identify a fetus with a genetic disease or condition. Some prenatal testing is offered routinely; other testing may be initiated due to family history or maternal factors. Pre-implantation testing follows in vitro fertilization (IVF) testing to identify embryos with a particular genetic condition. 4. Carrier testing involves testing an asymptomatic individual for carrier status for a genetic condition. Diagnostic testing is testing to identify a fetus with a genetic disease or condition. Some prenatal testing is offered routinely; other testing may be initiated due to family history or maternal factors. Prenatal testing is testing to identify a fetus with a genetic disease or condition. Some prenatal testing is offered routinely; other testing may be initiated due to family history or maternal factors. Pre-implantation testing follows in vitro fertilization (IVF) testing to identify embryos with a particular genetic condition. Page Ref: 56 Cognitive Level: Applying Client Need &Sub: Health Promotion and Maintenance Standards: QSEN Competencies: Patient-centered care | AACN Essential Competencies: Essential IX: Baccalaureate generalist nursing practice | NLN Competencies: Professional identity | Nursing/Integrated Concepts: Nursing Process: Implementation/Health teaching and health promotion Learning Outcome: LO 3.6 Discuss the significance of recent advances in human genetics and genomics and their impact on healthcare delivery. MNL LO: Cellular Alterations/Examine etiology, risk factors, pathophysiology, and clinical manifestations as seen in children. 8) Which genetic test would be best for the prospective father who recently had a positive screen for a genetic condition? 1. Carrier testing 2. Predictive testing 3. Diagnostic testing 4. Prenatal testing Answer: 3 Explanation: 1. Diagnostic testing is best for an individual who has a positive screen for a genetic disorder. Prenatal testing would be done with a pregnancy. Carrier testing is done with an asymptomatic individual who wishes to know whether he or she is a carrier of a condition. Predictive testing predicts the likelihood of a condition later in life. 2. Diagnostic testing is best for an individual who has a positive screen for a genetic disorder. Prenatal testing would be done with a pregnancy. Carrier testing is done with an asymptomatic individual who wishes to know whether he or she is a carrier of a condition. Predictive testing predicts the likelihood of a condition later in life. 3. Diagnostic testing is best for an individual who has a positive screen for a genetic disorder. Prenatal testing would be done with a pregnancy. Carrier testing is done with an asymptomatic individual who wishes to know whether he or she is a carrier of a condition. Predictive testing predicts the likelihood of a condition later in life. 4. Diagnostic testing is best for an individual who has a positive screen for a genetic disorder. Prenatal testing would be done with a pregnancy. Carrier testing is done with an asymptomatic individual who wishes to know whether he or she is a carrier of a condition. Predictive testing predicts the likelihood of a condition later in life. Page Ref: 55-57 Cognitive Level: Applying Client Need &Sub: Health Promotion and Maintenance Standards: QSEN Competencies: Patient-centered care | AACN Essential Competencies: Essential VII: Clinical prevention and population health | NLN Competencies: Human flourishing | Nursing/Integrated Concepts: Nursing Process: Assessment/Health teaching and health promotion Learning Outcome: LO 3.6 Discuss the significance of recent advances in human genetics and genomics and their impact on healthcare delivery. MNL LO: Cellular Alterations/Examine etiology, risk factors, pathophysiology, and clinical manifestations as seen in children. 9) The nurse is preparing a three-generation family pedigree. A student asks the nurse the significance of the darkened circles. Which response by the nurse is the most appropriate? 1. "Males unaffected by the disease." 2. "Males affected by the disease." 3. "Females unaffected by the disease." 4. "Females affected by the disease." Answer: 4 Explanation: 1. A circle is the standard symbol for a female, and darkening the circle represents a female affected by a disease. A male is represented by a square. 2. A circle is the standard symbol for a female, and darkening the circle represents a female affected by a disease. A male is represented by a square. 3. A circle is the standard symbol for a female, and darkening the circle represents a female affected by a disease. A male is represented by a square. 4. A circle is the standard symbol for a female, and darkening the circle represents a female affected by a disease. A male is represented by a square. Page Ref: 57, 58 Cognitive Level: Applying Client Need &Sub: Health Promotion and Maintenance Standards: QSEN Competencies: Patient-centered care | AACN Essential Competencies: Essential VII: Clinical prevention and population health | NLN Competencies: Nursing judgement | Nursing/Integrated Concepts: Nursing Process: Assessment/Health teaching and health promotion Learning Outcome: LO 3.2 Elicit a family health history and construct a genetic pedigree. MNL LO: Cellular Alterations/Examine etiology, risk factors, pathophysiology, and clinical manifestations as seen in children. 10) A family desires genetic testing for their adolescent. What response by the clinic nurse is appropriate? 1. "The child is a minor and cannot give consent." 2. "It is not advisable because insurance does not pay for this test." 3. "Let me discuss this with the adolescent and then we can discuss it more fully." 4. "There is a chance the adolescent might be discriminated against because of the test." Answer: 3 Explanation: 1. The adolescent is old enough to understand the pros and cons of testing. It would be advisable to discuss the matter with the adolescent and then more fully with the parents. That the minor is not able to give consent is true, but this answer cuts off discussion and is not appropriate. Insurance and discrimination can play a role in the decision, but still are not the appropriate answers because they do not address the issue of the request for testing. 2. The adolescent is old enough to understand the pros and cons of testing. It would be advisable to discuss the matter with the adolescent and then more fully with the parents. That the minor is not able to give consent is true, but this answer cuts off discussion and is not appropriate. Insurance and discrimination can play a role in the decision, but still are not the appropriate answers because they do not address the issue of the request for testing. 3. The adolescent is old enough to understand the pros and cons of testing. It would be advisable to discuss the matter with the adolescent and then more fully with the parents. That the minor is not able to give consent is true, but this answer cuts off discussion and is not appropriate. Insurance and discrimination can play a role in the decision, but still are not the appropriate answers because they do not address the issue of the request for testing. 4. The adolescent is old enough to understand the pros and cons of testing. It would be advisable to discuss the matter with the adolescent and then more fully with the parents. That the minor is not able to give consent is true, but this answer cuts off discussion and is not appropriate. Insurance and discrimination can play a role in the decision, but still are not the appropriate answers because they do not address the issue of the request for testing. Page Ref: 59-61 Cognitive Level: Analyzing Client Need &Sub: Health Promotion and Maintenance Standards: QSEN Competencies: Patient-centered care | AACN Essential Competencies: Essential VII: Clinical prevention and population health | NLN Competencies: Professional identity | Nursing/Integrated Concepts: Nursing Process: Implementation/Health teaching and health promotion Learning Outcome: LO 3.3 Incorporate knowledge of genetic and genomic influences and risk factors into assessment, planning, and implementation of nursing care. MNL LO: Cellular Alterations/Examine etiology, risk factors, pathophysiology, and clinical manifestations as seen in children. 11) A nurse is planning an education session on genetic testing. What would not concern the nurse when planning the session? 1. Cultural beliefs 2. Religious beliefs 3. Family values 4. Insurance reimbursement Answer: 4 Explanation: 1. Cultural and religious beliefs and family values are all considerations when planning a teaching session on genetic testing. Insurance plays a factor in determining whether the test is done, but is not a consideration in the teaching session itself. 2. Cultural and religious beliefs and family values are all considerations when planning a teaching session on genetic testing. Insurance plays a factor in determining whether the test is done, but is not a consideration in the teaching session itself. 3. Cultural and religious beliefs and family values are all considerations when planning a teaching session on genetic testing. Insurance plays a factor in determining whether the test is done, but is not a consideration in the teaching session itself. 4. Cultural and religious beliefs and family values are all considerations when planning a teaching session on genetic testing. Insurance plays a factor in determining whether the test is done, but is not a consideration in the teaching session itself. Page Ref: 62, 63 Cognitive Level: Applying Client Need &Sub: Health Promotion and Maintenance Standards: QSEN Competencies: Patient-centered care | AACN Essential Competencies: Essential VII: Clinical prevention and population health | NLN Competencies: Professional identity | Nursing/Integrated Concepts: Nursing Process: Planning/Health teaching and health promotion Learning Outcome: LO 3.4 Integrate basic genetic and genomic concepts into child and family education. MNL LO: Cellular Alterations/Examine etiology, risk factors, pathophysiology, and clinical manifestations as seen in children. 12) A nurse is planning care for a family who is undergoing genetic screening. Which expected outcome will the nurse include in the plan of care for this family? 1. Consult an attorney before making a decision. 2. Make a voluntary decision related to genetic health issues. 3. Not consider the influence of genetics on health promotion. 4. Look closely at the present before considering the future as it relates to genetic screening. Answer: 2 Explanation: 1. The goal of nursing care is to allow informed, voluntary decisions when it comes to genetic screening. 2. The goal of nursing care is to allow informed, voluntary decisions when it comes to genetic screening. 3. The goal of nursing care is to allow informed, voluntary decisions when it comes to genetic screening. 4. The goal of nursing care is to allow informed, voluntary decisions when it comes to genetic screening. Page Ref: 61 Cognitive Level: Applying Client Need &Sub: Health Promotion and Maintenance Standards: QSEN Competencies: Patient-centered care | AACN Essential Competencies: Essential VII: Clinical prevention and population health | NLN Competencies: Human flourishing | Nursing/Integrated Concepts: Nursing Process: Planning/Health teaching and health promotion Learning Outcome: LO 3.5 Understand implications of genome science on the nursing role with particular attention to ethical, legal, and social issues. MNL LO: Cellular Alterations/Examine etiology, risk factors, pathophysiology, and clinical manifestations as seen in children. 13) The nurse notes some dysmorphic facial features when examining a toddler in the well-child clinic. Which measurement taken by the nurse would not be considered when looking at dysmorphic facial features? 1. Interpupillary distance 2. Intercanthal distance 3. The distance from the outer canthus to the pinna 4. Outer cantus distance Answer: 3 Explanation: 1. The distance from the outer canthus to the pinna does not apply to the face. The other measurements would be necessary when evaluating facial dysmorphic features. 2. The distance from the outer canthus to the pinna does not apply to the face. The other measurements would be necessary when evaluating facial dysmorphic features. 3. The distance from the outer canthus to the pinna does not apply to the face. The other measurements would be necessary when evaluating facial dysmorphic features. 4. The distance from the outer canthus to the pinna does not apply to the face. The other measurements would be necessary when evaluating facial dysmorphic features. Page Ref: 58-60 Cognitive Level: Applying Client Need &Sub: Health Promotion and Maintenance Standards: QSEN Competencies: Patient-centered care | AACN Essential Competencies: Essential IX: Baccalaureate generalist nursing practice | NLN Competencies: Human flourishing | Nursing/Integrated Concepts: Nursing Process: Assessment/Quality of practice Learning Outcome: LO 3.3 Incorporate knowledge of genetic and genomic influences and risk factors into assessment, planning, and implementation of nursing care. MNL LO: Cellular Alterations/Examine etiology, risk factors, pathophysiology, and clinical manifestations as seen in children. 14) When conducting a health history on a late school-age client, what would the nurse document as a dysmorphic feature? 1. A repaired cleft palate 2. A 10 percent burn to the face 3. A severed finger 4. A flat anterior fontanel Answer: 1 Explanation: 1. A dysmorphic feature was present at birth. A cleft palate, even though repaired, would be included in a health history as a dysmorphic feature. The burns and a severed digit were not present at birth, and would not be considered dysmorphic. A soft fontanel would be considered normal. 2. A dysmorphic feature was present at birth. A cleft palate, even though repaired, would be included in a health history as a dysmorphic feature. The burns and a severed digit were not present at birth, and would not be considered dysmorphic. A soft fontanel would be considered normal. 3. A dysmorphic feature was present at birth. A cleft palate, even though repaired, would be included in a health history as a dysmorphic feature. The burns and a severed digit were not present at birth, and would not be considered dysmorphic. A soft fontanel would be considered normal. 4. A dysmorphic feature was present at birth. A cleft palate, even though repaired, would be included in a health history as a dysmorphic feature. The burns and a severed digit were not present at birth, and would not be considered dysmorphic. A soft fontanel would be considered normal. Page Ref: 60 Cognitive Level: Applying Client Need &Sub: Health Promotion and Maintenance Standards: QSEN Competencies: Quality improvement | AACN Essential Competencies: Essential IV: Information management and application of patient care technology | NLN Competencies: Human flourishing | Nursing/Integrated Concepts: Nursing Process: Implementation/Quality of practice Learning Outcome: LO 3.3 Incorporate knowledge of genetic and genomic influences and risk factors into assessment, planning, and implementation of nursing care. MNL LO: Cellular Alterations/Examine etiology, risk factors, pathophysiology, and clinical manifestations as seen in children. 15) When completing a pedigree, which factors should be included? Select all that apply. 1. Full siblings only 2. Begin with the proband 3. Mark each generation with a Roman numeral 4. Include at least three generations 5. Use only standard pedigree symbols Answer: 2, 3, 4, 5 Explanation: 1. It is important to include half-siblings in addition to full siblings, as half-siblings have half the genetic history that the full siblings do. The other answers are all important to include in a pedigree. 2. It is important to include half-siblings in addition to full siblings, as half-siblings have half the genetic history that the full siblings do. The other answers are all important to include in a pedigree. 3. It is important to include half-siblings in addition to full siblings, as half-siblings have half the genetic history that the full siblings do. The other answers are all important to include in a pedigree. 4. It is important to include half-siblings in addition to full siblings, as half-siblings have half the genetic history that the full siblings do. The other answers are all important to include in a pedigree. 5. It is important to include half-siblings in addition to full siblings, as half-siblings have half the genetic history that the full siblings do. The other answers are all important to include in a pedigree. Page Ref: 57, 58 Cognitive Level: Applying Client Need &Sub: Health Promotion and Maintenance Standards: QSEN Competencies: Patient-centered care | AACN Essential Competencies: Essential VII: Clinical prevention and population health | NLN Competencies: Nursing judgement | Nursing/Integrated Concepts: Nursing Process: Planning/Quality of practice Learning Outcome: LO 3.2 Elicit a family health history and construct a genetic pedigree. MNL LO: Cellular Alterations/Examine etiology, risk factors, pathophysiology, and clinical manifestations as seen in children. 16) A nurse from a pediatric clinic performs assignments and counsels parents. Which families should the nurse refer for genetic counseling? Select all that apply. 1. Female with hypoactive thyroid disease 2. Couple with multiple stillbirths 3. Couple with family history of heart disease 4. Couple with three years of infertility 5. Child born with ophthalmia neonatorum Answer: 2, 4 Explanation: 1. A female with hypoactive thyroid disease should not be referred for genetic counseling. 2. A couple with multiple stillbirths should be referred for genetic counseling. 3. A couple with family history of heart disease should not be referred for genetic counseling. 4. The couple with three years of infertility should be referred for genetic counseling. 5. The family with a child born with a child born with ophthalmia neonatorum should not be referred for genetic counseling. Page Ref: 49 Cognitive Level: Applying Client Need &Sub: Safe and Effective Care Environment Standards: QSEN Competencies: Patient-centered care | AACN Essential Competencies: Essential VI: Interprofessional communication and collaboration for improving patient health outcomes | NLN Competencies: Nursing judgement | Nursing/Integrated Concepts: Nursing Process: Assessment/Collaboration Learning Outcome: LO 3.1 Explain the role of genetic and genomic concepts in health promotion, disease prevention, screening, diagnostics, selection of treatment, and monitoring of treatment effectiveness. MNL LO: Cellular Alterations/Examine etiology, risk factors, pathophysiology, and clinical manifestations as seen in children. 17) The nurse is discussing genetic conditions with a family of a newborn. Which genetic conditions fall under the inheritance pattern of autosomal recessive conditions? Select all that apply. 1. Achondroplasia 2. Marfan syndrome 3. Hemophilia A 4. Cystic fibrosis 5. Sickle cell disease Answer: 4, 5 Explanation: 1. Achondroplasia is not an autosomal recessive condition. Beta-thalassemia, cystic fibrosis, Gaucher disease, phenylketonuria, sickle cell disease, and Tay-sachs disease all are autosomal recessive conditions. 2. Marfan syndrome is not an autosomal recessive condition. Beta-thalassemia, cystic fibrosis, Gaucher disease, phenylketonuria, sickle cell disease, and Tay-sachs disease all are autosomal recessive conditions. 3. Hemophilia A is not an autosomal recessive condition. Beta-thalassemia, cystic fibrosis, Gaucher disease, phenylketonuria, sickle cell disease, and Tay-sachs disease all are autosomal recessive conditions. 4. Cystic fibrosis is an autosomal recessive condition. Beta-thalassemia, cystic fibrosis, Gaucher disease, phenylketonuria, sickle cell disease, and Tay-sachs disease all are autosomal recessive conditions. 5. Sickle cell disease is an autosomal recessive condition. Beta-thalassemia, cystic fibrosis, Gaucher disease, phenylketonuria, sickle cell disease, and Tay-sachs disease all are autosomal recessive conditions. Page Ref: 52 Cognitive Level: Applying Client Need &Sub: Health Promotion and Maintenance Standards: QSEN Competencies: Patient-centered care | AACN Essential Competencies: Essential IX: Baccalaureate generalist nursing practice | NLN Competencies: Human flourishing | Nursing/Integrated Concepts: Nursing Process: Diagnosis/Education Learning Outcome: LO 3.4 Integrate basic genetic and genomic concepts into child and family education. MNL LO: Cellular Alterations/Examine etiology, risk factors, pathophysiology, and clinical manifestations as seen in children. Principles of Pediatric Nursing: Caring for Children, 7e (Ball et al.) Chapter 4 Growth and Development 1) While in the pediatrician's office for their child's 12-month well-child exam, the parents ask the nurse for advice on age-appropriate toys for their child. Based on the child's developmental level, which types of toys would the nurse suggest? Select all that apply. 1. Soft toys that can be manipulated 2. Small toys that can pop apart and go back together 3. Jack-in-the-box toys 4. Toys with black and white patterns 5. Push-and-pull toys Answer: 1, 3, 5 Explanation: 1. Both gross and fine motor skills are becoming more developed, and children at this age enjoy toys that can help them refine these skills. They tend to enjoy more colorful toys at this age and are more mobile and thus have less interest in placing toys in their mouths and more interest in toys that can be manipulated. 2. Both gross and fine motor skills are becoming more developed, and children at this age enjoy toys that can help them refine these skills. They tend to enjoy more colorful toys at this age and are more mobile and thus have less interest in placing toys in their mouths and more interest in toys that can be manipulated. 3. Both gross and fine motor skills are becoming more developed, and children at this age enjoy toys that can help them refine these skills. They tend to enjoy more colorful toys at this age and are more mobile and thus have less interest in placing toys in their mouths and more interest in toys that can be manipulated. 4. Both gross and fine motor skills are becoming more developed, and children at this age enjoy toys that can help them refine these skills. They tend to enjoy more colorful toys at this age and are more mobile and thus have less interest in placing toys in their mouths and more interest in toys that can be manipulated. 5. Both gross and fine motor skills are becoming more developed, and children at this age enjoy toys that can help them refine these skills. They tend to enjoy more colorful toys at this age and are more mobile and thus have less interest in placing toys in their mouths and more interest in toys that can be manipulated. Page Ref: 67 Cognitive Level: Applying Client Need &Sub: Health Promotion and Maintenance Standards: QSEN Competencies: Patient-centered care | AACN Essential Competencies: Essential VII: Clinical prevention and population health | NLN Competencies: Human flourishing | Nursing/Integrated Concepts: Nursing Process: Implementation/Health teaching and health promotion Learning Outcome: LO 4.7 Describe the role of play in the growth and development of children. MNL LO: Compare the developmental stages for pediatric clients. 2) A mother of a school-age client who recently had surgery for the removal of tonsils and adenoids complains that the child has begun sucking his thumb again. Which coping mechanisms is the child using to cope with the surgery and hospitalization? 1. Repression 2. Rationalization 3. Regression 4. Fantasy Answer: 3 Explanation: 1. The correct answer is regression, which is a return to an earlier behavior. Repression is the involuntary forgetting of uncomfortable situations, rationalization is an attempt to make unacceptable feelings acceptable, and fantasy is a creation of the mind to help deal with an unacceptable fear. 2. The correct answer is regression, which is a return to an earlier behavior. Repression is the involuntary forgetting of uncomfortable situations, rationalization is an attempt to make unacceptable feelings acceptable, and fantasy is a creation of the mind to help deal with an unacceptable fear. 3. The correct answer is regression, which is a return to an earlier behavior. Repression is the involuntary forgetting of uncomfortable situations, rationalization is an attempt to make unacceptable feelings acceptable, and fantasy is a creation of the mind to help deal with an unacceptable fear. 4. The correct answer is regression, which is a return to an earlier behavior. Repression is the involuntary forgetting of uncomfortable situations, rationalization is an attempt to make unacceptable feelings acceptable, and fantasy is a creation of the mind to help deal with an unacceptable fear. Page Ref: 68 Cognitive Level: Applying Client Need &Sub: Psychosocial Integrity Standards: QSEN Competencies: Patient-centered care | AACN Essential Competencies: Essential IX: Baccalaureate generalist nursing practice | NLN Competencies: Human flourishing | Nursing/Integrated Concepts: Nursing Process: Diagnosis/Coordination of care Learning Outcome: LO 4.5 Identify major developmental milestones for infants, toddlers, preschoolers, school-age children, and adolescents. MNL LO: Compare the developmental stages for pediatric clients. 3) While being comforted in the emergency department, a young school-age sibling of a pediatric trauma victim blurts out to the nurse, "It's my fault! When we were fighting yesterday, I told him I wished he was dead!" Which response is most appropriate by the nurse? 1. Asking the child if she would like to sit down and drink some water 2. Sitting the child down in an empty room with markers and paper so that she can draw a picture 3. Calmly discussing the catheters, tubes, and equipment that the patient requires and explaining to the sibling why the patient needs them 4. Reassuring the child that it is normal to get angry and say things that we do not mean but that we have no control over whether or not an accident happens Answer: 4 Explanation: 1. "Magical thinking" is the belief that events occur because of one's thoughts or actions, and the most therapeutic way to respond to this is to correct any misconceptions that children may have and reassure them that they are not to blame for any accidents or illness. 2. "Magical thinking" is the belief that events occur because of one's thoughts or actions, and the most therapeutic way to respond to this is to correct any misconceptions that children may have and reassure them that they are not to blame for any accidents or illness. 3. "Magical thinking" is the belief that events occur because of one's thoughts or actions, and the most therapeutic way to respond to this is to correct any misconceptions that children may have and reassure them that they are not to blame for any accidents or illness. 4. "Magical thinking" is the belief that events occur because of one's thoughts or actions, and the most therapeutic way to respond to this is to correct any misconceptions that children may have and reassure them that they are not to blame for any accidents or illness. Page Ref: 89, 90 Cognitive Level: Applying Client Need &Sub: Psychosocial Integrity Standards: QSEN Competencies: Patient-centered care | AACN Essential Competencies: Essential I: Liberal education for Baccalaureate generalist nursing practice | NLN Competencies: Human flourishing | Nursing/Integrated Concepts: Nursing Process: Implementation/Coordination of care Learning Outcome: LO 4.5 Identify major developmental milestones for infants, toddlers, preschoolers, school-age children, and adolescents. MNL LO: Compare the developmental stages for pediatric clients. 4) While assessing the development of a 9-month-old infant, the nurse asks the mother if the child actively looks for toys when they are placed out of sight. Which developmental task is the nurse assessing this infant for? 1. Object permanence 2. Centration 3. Transductive reasoning 4. Conservation Answer: 1 Explanation: 1. A child who has developed object permanence has the ability to understand that even though something is out of sight, it still exists. In centration, a child focuses only on a particular aspect of a situation. Transductive reasoning happens when a child connects two events in a cause-effect relationship because they have occurred at the same time. Conservation describes when a child knows that matter is not changed when its form is altered. 2. A child who has developed object permanence has the ability to understand that even though something is out of sight, it still exists. In centration, a child focuses only on a particular aspect of a situation. Transductive reasoning happens when a child connects two events in a cause- effect relationship because they have occurred at the same time. Conservation describes when a child knows that matter is not changed when its form is altered. 3. A child who has developed object permanence has the ability to understand that even though something is out of sight, it still exists. In centration, a child focuses only on a particular aspect of a situation. Transductive reasoning happens when a child connects two events in a cause- effect relationship because they have occurred at the same time. Conservation describes when a child knows that matter is not changed when its form is altered. 4. A child who has developed object permanence has the ability to understand that even though something is out of sight, it still exists. In centration, a child focuses only on a particular aspect of a situation. Transductive reasoning happens when a child connects two events in a cause- effect relationship because they have occurred at the same time. Conservation describes when a child knows that matter is not changed when its form is altered. Page Ref: 86 Cognitive Level: Analyzing Client Need &Sub: Health Promotion and Maintenance Standards: QSEN Competencies: Patient-centered care | AACN Essential Competencies: Essential VII: Clinical prevention and population health | NLN Competencies: Human flourishing | Nursing/Integrated Concepts: Nursing Process: Assessment/Coordination of care Learning Outcome: LO 4.5 Identify major developmental milestones for infants, toddlers, preschoolers, school-age children, and adolescents. MNL LO: Compare the developmental stages for pediatric clients. 5) The nurse is counseling the parents of a 6-1/2-month-old infant. Which age-appropriate toy is most appropriate for the nurse to suggest to these parents? 1. Soft, fluid-filled ring that can be chilled in the refrigerator 2. Colorful rattle 3. Jack-in-the-box toy 4. Push-and-pull toy Answer: 1 Explanation: 1. Teething toys would be appropriate for this age. The rattle might be better enjoyed by a 3- to 6-month-old infant, and the jack-in-the-box and push-and-pull toys are better suited for a 9- to 12-month-old child. 2. Teething toys would be appropriate for this age. The rattle might be better enjoyed by a 3- to 6-month-old infant, and the jack-in-the-box and push-and-pull toys are better suited for a 9- to 12-month-old child. 3. Teething toys would be appropriate for this age. The rattle might be better enjoyed by a 3- to 6-month-old infant, and the jack-in-the-box and push-and-pull toys are better suited for a 9- to 12-month-old child. 4. Teething toys would be appropriate for this age. The rattle might be better enjoyed by a 3- to 6-month-old infant, and the jack-in-the-box and push-and-pull toys are better suited for a 9- to 12-month-old child. Page Ref: 81 Cognitive Level: Applying Client Need &Sub: Health Promotion and Maintenance Standards: QSEN Competencies: Patient-centered care | AACN Essential Competencies: Essential VII: Clinical prevention and population health | NLN Competencies: Human flourishing | Nursing/Integrated Concepts: Nursing Process: Implementation/Coordination of care Learning Outcome: LO 4.7 Describe the role of play in the growth and development of children. MNL LO: Compare the developmental stages for pediatric clients. 6) A nurse is assessing language development in all the infants presenting at the doctor's office for well-child visits. At which age range would the nurse expect a child to verbalize the words "dada" and "mama"? 1. 3 and 5 months 2. 6 and 8 months 3. 9 and 12 months 4. 13 and 18 months Answer: 3 Explanation: 1. Children should be able to verbalize "mama" or "dada" to identify their parents by 1 year of age. 2. Children should be able to verbalize "mama" or "dada" to identify their parents by 1 year of age. 3. Children should be able to verbalize "mama" or "dada" to identify their parents by 1 year of age. 4. Children should be able to verbalize "mama" or "dada" to identify their parents by 1 year of age. Page Ref: 87 Cognitive Level: Applying Client Need &Sub: Health Promotion and Maintenance Standards: QSEN Competencies: Patient-centered care | AACN Essential Competencies: Essential VII: Clinical prevention and population health | NLN Competencies: Human flourishing | Nursing/Integrated Concepts: Nursing Process: Evaluation/Coordination of care Learning Outcome: LO 4.5 Identify major developmental milestones for infants, toddlers, preschoolers, school-age children, and adolescents. MNL LO: Compare the developmental stages for pediatric clients. 7) The parents of an 8-year-old state that their son seems very interested in trying new activities. When the parents ask for suggested activities for this age child, the nurse recommends scouts as an activity that will foster growth and development. In which stage of Erikson's "psychosocial stages of development" is this child? 1. Trust versus mistrust 2. Initiative versus guilt 3. Industry versus inferiority 4. Identity versus role confusion Answer: 3 Explanation: 1. Trust versus mistrust (birth to 1 year)—The task of the first year of life is to establish trust in the people providing care. Trust is fostered by provision of food, clean clothing, touch, and comfort. If basic needs are not met, the infant will eventually learn to mistrust others. Initiative versus guilt (3 to 6 years)—The young child initiates new activities and considers new ideas. This interest in exploring the world creates a child who is involved and busy. Constant criticism, on the other hand, leads to feelings of guilt and a lack of purpose. Identity versus role confusion (12 to 18 years)—In adolescence, as the body matures and thought processes become more complex, a new sense of identity or self is established. The self, family, peer group, and community are all examined and redefined. The adolescent who is unable to establish a meaningful definition of self will experience confusion in one or more roles of life. Industry versus inferiority (6 to 12 years)—The middle years of childhood are characterized by development of new interests and by involvement in activities. The child takes pride in accomplishments in sports, school, home, and community. If the child cannot accomplish what is expected, however, the result will be a sense of inferiority. 2. Trust versus mistrust (birth to 1 year)—The task of the first year of life is to establish trust in the people providing care. Trust is fostered by provision of food, clean clothing, touch, and comfort. If basic needs are not met, the infant will eventually learn to mistrust others. Initiative versus guilt (3 to 6 years)—The young child initiates new activities and considers new ideas. This interest in exploring the world creates a child who is involved and busy. Constant criticism, on the other hand, leads to feelings of guilt and a lack of purpose. Identity versus role confusion (12 to 18 years)—In adolescence, as the body matures and thought processes become more complex, a new sense of identity or self is established. The self, family, peer group, and community are all examined and redefined. The adolescent who is unable to establish a meaningful definition of self will experience confusion in one or more roles of life. Industry versus inferiority (6 to 12 years)—The middle years of childhood are characterized by development of new interests and by involvement in activities. The child takes pride in accomplishments in sports, school, home, and community. If the child cannot accomplish what is expected, however, the result will be a sense of inferiority. 3. Trust versus mistrust (birth to 1 year)—The task of the first year of life is to establish trust in the people providing care. Trust is fostered by provision of food, clean clothing, touch, and comfort. If basic needs are not met, the infant will eventually learn to mistrust others. Initiative versus guilt (3 to 6 years)—The young child initiates new activities and considers new ideas. This interest in exploring the world creates a child who is involved and busy. Constant criticism, on the other hand, leads to feelings of guilt and a lack of purpose. Identity versus role confusion (12 to 18 years)—In adolescence, as the body matures and thought processes become more complex, a new sense of identity or self is established. The self, family, peer group, and community are all examined and redefined. The adolescent who is unable to establish a meaningful definition of self will experience confusion in one or more roles of life. Industry versus inferiority (6 to 12 years)—The middle years of childhood are characterized by development of new interests and by involvement in activities. The child takes pride in accomplishments in sports, school, home, and community. If the child cannot accomplish what is expected, however, the result will be a sense of inferiority. 4. Trust versus mistrust (birth to 1 year)—The task of the first year of life is to establish trust in the people providing care. Trust is fostered by provision of food, clean clothing, touch, and comfort. If basic needs are not met, the infant will eventually learn to mistrust others. Initiative versus guilt (3 to 6 years)—The young child initiates new activities and considers new ideas. This interest in exploring the world creates a child who is involved and busy. Constant criticism, on the other hand, leads to feelings of guilt and a lack of purpose. Identity versus role confusion (12 to 18 years)—In adolescence, as the body matures and thought processes become more complex, a new sense of identity or self is established. The self, family, peer group, and community are all examined and redefined. The adolescent who is unable to establish a meaningful definition of self will experience confusion in one or more roles of life. Industry versus inferiority (6 to 12 years)—The middle years of childhood are characterized by development of new interests and by involvement in activities. The child takes pride in accomplishments in sports, school, home, and community. If the child cannot accomplish what is expected, however, the result will be a sense of inferiority. Page Ref: 69 Cognitive Level: Analyzing Client Need &Sub: Psychosocial Integrity Standards: QSEN Competencies: Patient-centered care | AACN Essential Competencies: Essential IV: Information management and application of patient care technology | NLN Competencies: Human flourishing | Nursing/Integrated Concepts: Nursing Process: Evaluation/Coordination of care Learning Outcome: LO 4.8 Use data collected during developmental assessments to implement activities that promote development of children and adolescents. MNL LO: Compare the developmental stages for pediatric clients. 8) Two 3-year-olds are playing in a hospital playroom together. One is working on a puzzle while the other is stacking blocks. Which type of play are these children exhibiting? 1. Cooperative play 2. Associative play 3. Parallel play 4. Solitary play Answer: 3 Explanation: 1. Parallel play describes when two or more children play together, each engaging in their own activities. Cooperative play happens when children demonstrate the ability to cooperate with others and play a part in order to contribute to a unified whole. Associative play is characterized by children interacting in groups and participating in similar activities. In solitary play, a child plays alone. 2. Parallel play describes when two or more children play together, each engaging in their own activities. Cooperative play happens when children demonstrate the ability to cooperate with others and play a part in order to contribute to a unified whole. Associative play is characterized by children interacting in groups and participating in similar activities. In solitary play, a child plays alone. 3. Parallel play describes when two or more children play together, each engaging in their own activities. Cooperative play happens when children demonstrate the ability to cooperate with others and play a part in order to contribute to a unified whole. Associative play is characterized by children interacting in groups and participating in similar activities. In solitary play, a child plays alone. 4. Parallel play describes when two or more children play together, each engaging in their own activities. Cooperative play happens when children demonstrate the ability to cooperate with others and play a part in order to contribute to a unified whole. Associative play is characterized by children interacting in groups and participating in similar activities. In solitary play, a child plays alone. Page Ref: 86 Cognitive Level: Applying Client Need &Sub: Health Promotion and Maintenance Standards: QSEN Competencies: Patient-centered care | AACN Essential Competencies: Essential II: Basic organizational and systems leadership for quality care and patient safety. | NLN Competencies: Human flourishing | Nursing/Integrated Concepts: Nursing Process: Implementation/Coordination of care Learning Outcome: LO 4.5 Identify major developmental milestones for infants, toddlers, preschoolers, school-age children, and adolescents. MNL LO: Compare the developmental stages for pediatric clients. 9) The nurse, talking with the parents of a toddler who is struggling with toilet training, reassures them that their child is demonstrating a typical developmental stage. According to Erikson, which developmental stage will the nurse document in the medical record for this toddler? 1. Trust versus mistrust 2. Autonomy versus shame and doubt 3. Initiative versus guilt 4. Industry versus inferiority Answer: 2 Explanation: 1. Erikson's stage of "autonomy versus shame and doubt" marks a period of time when the toddler is trying to gain some independence while still wanting to please adults. 2. Erikson's stage of "autonomy versus shame and doubt" marks a period of time when the toddler is trying to gain some independence while still wanting to please adults. 3. Erikson's stage of "autonomy versus shame and doubt" marks a period of time when the toddler is trying to gain some independence while still wanting to please adults. 4. Erikson's stage of "autonomy versus shame and doubt" marks a period of time when the toddler is trying to gain some independence while still wanting to please adults. Page Ref: 69 Cognitive Level: Applying Client Need &Sub: Health Promotion and Maintenance Standards: QSEN Competencies: Patient-centered care | AACN Essential Competencies: Essential I: Liberal education for Baccalaureate generalist nursing practice | NLN Competencies: Nursing judgement | Nursing/Integrated Concepts: Nursing Process: Implementation/Coordination of care Learning Outcome: LO 4.1 Describe the major theories of development as formulated by Freud, Erikson, Piaget, Kohlberg, social learning theorists, and behaviorists. MNL LO: Compare the developmental stages for pediatric clients. 10) A new parent group inquires about the stages through which their children will progress as they grow older. The nurse is discussing Piaget's developmental stages. In what order would the nurse expect the child to progress through Piaget's stages of development? 1. Sensorimotor 2. Formal operational 3. Preoperational 4. Concrete operational Answer: 1, 3, 4, 2 Sensorimotor Preoperational Concrete operational Formal operational Explanation: Sensorimotor (birth to 2 years), preoperational (2 to 7 years), concrete operational (7 to 11 years), formal operational (11 years to adulthood). Page Ref: 72 Cognitive Level: Applying Client Need &Sub: Psychosocial Integrity Standards: QSEN Competencies: Patient-centered care | AACN Essential Competencies: Essential VII: Clinical prevention and population health | NLN Competencies: Human flourishing | Nursing/Integrated Concepts: Nursing Process: Implementation/Health teaching and health promotion Learning Outcome: LO 4.1 Describe the major theories of development as formulated by Freud, Erikson, Piaget, Kohlberg, social learning theorists, and behaviorists. MNL LO: Compare the developmental stages for pediatric clients. 11) While trying to inform a young school-age client about what will occur during an upcoming CT scan, the nurse notices that the child is engaged in a collective monologue, talking about a new puppy. Which response by the nurse is the most appropriate in this situation? 1. "Please stop talking about your puppy. I need to tell you about your CT scan." 2. Ignore the child's responses and continue discussing the procedure. 3. "I'll come back when you are ready to talk with me more about your CT scan." 4. "You must be so excited to have a new puppy! They are so much fun. Now, let me tell you again about going downstairs in a wheelchair to a special room." Answer: 4 Explanation: 1. When a child becomes engaged in a collective monologue, it is best to respond to the content of his or her conversation and then attempt to reinsert facts about the content that needs to be covered. 2. When a child becomes engaged in a collective monologue, it is best to respond to the content of his or her conversation and then attempt to reinsert facts about the content that needs to be covered. 3. When a child becomes engaged in a collective monologue, it is best to respond to the content of his or her conversation and then attempt to reinsert facts about the content that needs to be covered. 4. When a child becomes engaged in a collective monologue, it is best to respond to the content of his or her conversation and then attempt to reinsert facts about the content that needs to be covered. Page Ref: 90 Cognitive Level: Applying Client Need &Sub: Health Promotion and Maintenance Standards: QSEN Competencies: Patient-centered care | AACN Essential Competencies: Essential IX: Baccalaureate generalist nursing practice | NLN Competencies: Nursing judgement | Nursing/Integrated Concepts: Nursing Process: Implementation/Coordination of care Learning Outcome: LO 4.3 Plan nursing interventions for children that are appropriate for each child's developmental state, based on theoretical frameworks. MNL LO: Compare the developmental stages for pediatric clients. 12) An adolescent client with cystic fibrosis suddenly becomes noncompliant with the medication regime. Which intervention by the nurse will most likely improve compliance for this client? 1. Give the child a computer-animated game that presents information on the management of cystic fibrosis. 2. Arrange for the physician to sit down and talk to the child about the risks related to noncompliance with medications. 3. Set up a meeting with some older teens with cystic fibrosis who have been managing their disease effectively. 4. Discuss with the child's parents the privileges that can be taken away, such as cell phone, if compliance fails to improve. Answer: 3 Explanation: 1. Providing an adolescent with positive role models who are in his peer group is the intervention most likely to improve compliance. Interest in games may begin to wane, adults' opinions may be viewed negatively and challenged, and threatening punishment may further incite rebellion. 2. Providing an adolescent with positive role models who are in his peer group is the intervention most likely to improve compliance. Interest in games may begin to wane, adults' opinions may be viewed negatively and challenged, and threatening punishment may further incite rebellion. 3. Providing an adolescent with positive role models who are in his peer group is the intervention most likely to improve compliance. Interest in games may begin to wane, adults' opinions may be viewed negatively and challenged, and threatening punishment may further incite rebellion. 4. Providing an adolescent with positive role models who are in his peer group is the intervention most likely to improve compliance. Interest in games may begin to wane, adults' opinions may be viewed negatively and challenged, and threatening punishment may further incite rebellion. Page Ref: 96 Cognitive Level: Applying Client Need &Sub: Health Promotion and Maintenance Standards: QSEN Competencies: Patient-centered care | AACN Essential Competencies: Essential I: Liberal education for Baccalaureate generalist nursing practice | NLN Competencies: Nursing judgement | Nursing/Integrated Concepts: Nursing Process: Planning/Coordination of care Learning Outcome: LO 4.3 Plan nursing interventions for children that are appropriate for each child's developmental state, based on theoretical frameworks. MNL LO: Analyze the impact the diagnosis of a chronic illness has on the pediatric client and family. 13) A neonatal nurse who encourages parents to hold their baby and provides opportunities for Kangaroo Care most likely is demonstrating concern for which aspect of the infant's psychosocial development? 1. Attachment 2. Assimilation 3. Centration 4. Resilience Answer: 1 Explanation: 1. Attachment is a strong emotional bond between a parent and child that forms the foundation for the fulfillment of the basic need of trust in the infant. Assimilation describes the child's incorporation of new experiences, centration is the ability to consider only one aspect of a situation at a time, and resilience is the ability to maintain healthy function even under significant stress and adversity. 2. Attachment is a strong emotional bond between a parent and child that forms the foundation for the fulfillment of the basic need of trust in the infant. Assimilation describes the child's incorporation of new experiences, centration is the ability to consider only one aspect of a situation at a time, and resilience is the ability to maintain healthy function even under significant stress and adversity. 3. Attachment is a strong emotional bond between a parent and child that forms the foundation for the fulfillment of the basic need of trust in the infant. Assimilation describes the child's incorporation of new experiences, centration is the ability to consider only one aspect of a situation at a time, and resilience is the ability to maintain healthy function even under significant stress and adversity. 4. Attachment is a strong emotional bond between a parent and child that forms the foundation for the fulfillment of the basic need of trust in the infant. Assimilation describes the child's incorporation of new experiences, centration is the ability to consider only one aspect of a situation at a time, and resilience is the ability to maintain healthy function even under significant stress and adversity. Page Ref: 80, 83 Cognitive Level: Applying Client Need &Sub: Psychosocial Integrity Standards: QSEN Competencies: Patient-centered care | AACN Essential Competencies: Essential III: Scholarship for evidence-based practice | NLN Competencies: Professional identity | Nursing/Integrated Concepts: Nursing Process: Implementation/Coordination of care Learning Outcome: LO 4.8 Use data collected during developmental assessments to implement activities that promote development of children and adolescents. MNL LO: Examine health promotion, stress reduction therapies, and safety for hospitalized child and family. 14) Which statement by the nurse is most appropriate prior to giving an intramuscular injection to a 2-1/2-year-old child? 1. "We will give you your shot when your mommy comes back." 2. "This is medicine that will make you better. First we will hold your leg, then I will wipe it off with this magic cloth that kills the germs on your leg right here, then I will hold the needle like this and say 'one, two, three . . . go' and give you your shot. Are you ready?" 3. "It is all right to cry, I know that this hurts. After we are done you can go to the box and pick out your favorite sticker." 4. "This is a magic sword that will give you your medicine and make you all better." Answer: 3 Explanation: 1. The most appropriate response would be to acknowledge the child's feelings and give her something to look forward to (picking out a sticker). Waiting for the mother to come back would be inappropriate because toddlers do not have an understanding of time. Giving elaborate descriptions and using colorful language are inappropriate. The instructions should not end with a "are you ready" statement because the toddler will say no. You also don't want to frighten and/or confuse the child by using statements such as use of a magic sword. 2. The most appropriate response would be to acknowledge the child's feelings and give her something to look forward to (picking out a sticker). Waiting for the mother to come back would be inappropriate because toddlers do not have an understanding of time. Giving elaborate descriptions and using colorful language are inappropriate. The instructions should not end with a "are you ready" statement because the toddler will say no. You also don't want to frighten and/or confuse the child by using statements such as use of a magic sword. 3. The most appropriate response would be to acknowledge the child's feelings and give her something to look forward to (picking out a sticker). Waiting for the mother to come back would be inappropriate because toddlers do not have an understanding of time. Giving elaborate descriptions and using colorful language are inappropriate. The instructions should not end with a "are you ready" statement because the toddler will say no. You also don't want to frighten and/or confuse the child by using statements such as use of a magic sword. 4. The most appropriate response would be to acknowledge the child's feelings and give her something to look forward to (picking out a sticker). Waiting for the mother to come back would be inappropriate because toddlers do not have an understanding of time. Giving elaborate descriptions and using colorful language are inappropriate. The instructions should not end with a "are you ready" statement because the toddler will say no. You also don't want to frighten and/or confuse the child by using statements such as use of a magic sword. Page Ref: 74 Cognitive Level: Applying Client Need &Sub: Psychosocial Integrity Standards: QSEN Competencies: Patient-centered care | AACN Essential Competencies: Essential I: Liberal education for Baccalaureate generalist nursing practice | NLN Competencies: Nursing judgement | Nursing/Integrated Concepts: Nursing Process: Planning/Coordination of care Learning Outcome: LO 4.3 Plan nursing interventions for children that are appropriate for each child's developmental state, based on theoretical frameworks. MNL LO: Examine the pathophysiology of pain perception and associated concepts seen in children. 15) The parents of a 1-year-old infant are concerned that this baby seems more shy and scared of new situations than their other child and ask the nurse if this is normal. The nurse knows that the infant is exhibiting a characteristic of the "slow-to-warm-up." Which statement to the parents is most appropriate by the nurse? 1. "Your infant is showing a regularity in patterns of eating." 2. "Your infant displays a predominately negative mood." 3. "Your infant initially reacts to new situations by withdrawing." 4. "Your infant has intense reactions to the environment." Answer: 3 Explanation: 1. "Slow-to-warm-up" children adapt slowly to new situations and initially will withdraw. Showing regularity in patterns of eating is a characteristic of an "easy" child, and displaying a predominately negative mood and commonly having intense reactions to the environment are characteristics of "difficult" children. 2. "Slow-to-warm-up" children adapt slowly to new situations and initially will withdraw. Showing regularity in patterns of eating is a characteristic of an "easy" child, and displaying a predominately negative mood and commonly having intense reactions to the environment are characteristics of "difficult" children. 3. "Slow-to-warm-up" children adapt slowly to new situations and initially will withdraw. Showing regularity in patterns of eating is a characteristic of an "easy" child, and displaying a predominately negative mood and commonly having intense reactions to the environment are characteristics of "difficult" children. 4. "Slow-to-warm-up" children adapt slowly to new situations and initially will withdraw. Showing regularity in patterns of eating is a characteristic of an "easy" child, and displaying a predominately negative mood and commonly having intense reactions to the environment are characteristics of "difficult" children. Page Ref: 77, 78 Cognitive Level: Applying Client Need &Sub: Health Promotion and Maintenance Standards: QSEN Competencies: Patient-centered care | AACN Essential Competencies: Essential I: Liberal education for Baccalaureate generalist nursing practice | NLN Competencies: Human flourishing | Nursing/Integrated Concepts: Nursing Process: Implementation/Coordination of care Learning Outcome: LO 4.8 Use data collected during developmental assessments to implement activities that promote development of children and adolescents. MNL LO: Differentiate developmentally appropriate care environments for the pediatric client and family. 16) The nurse educator is presenting a lecture about risks to developmental progression. Which items will the educator include in the lecture? Select all that apply. 1. Family support 2. Access to the Internet 3. Recent loss of employment 4. Terminal illness of a family member 5. Hazards within the home environment Answer: 3, 4, 5 Explanation: 1. Risk factors that can inhibit developmental progression include financial problems, stresses and worries, family and job instability, neighborhood and home hazards, and lack of resources. Family support and access to the Internet are both considered protective factors. 2. Risk factors that can inhibit developmental progression include financial problems, stresses and worries, family and job instability, neighborhood and home hazards, and lack of resources. Family support and access to the Internet are both considered protective factors. 3. Risk factors that can inhibit developmental progression include financial problems, stresses and worries, family and job instability, neighborhood and home hazards, and lack of resources. Family support and access to the Internet are both considered protective factors. 4. Risk factors that can inhibit developmental progression include financial problems, stresses and worries, family and job instability, neighborhood and home hazards, and lack of resources. Family support and access to the Internet are both considered protective factors. 5. Risk factors that can inhibit developmental progression include financial problems, stresses and worries, family and job instability, neighborhood and home hazards, and lack of resources. Family support and access to the Internet are both considered protective factors. Page Ref: 94-96 Cognitive Level: Applying Client Need &Sub: Health Promotion and Maintenance Standards: QSEN Competencies: Patient-centered care | AACN Essential Competencies: Essential VII: Clinical prevention and population health | NLN Competencies: Professional identity | Nursing/Integrated Concepts: Nursing Process: Implementation/Health teaching and health promotion Learning Outcome: LO 4.2 Recognize risks to developmental progression and factors that protect against those risks. MNL LO: Use the nursing process to provide developmentally appropriate care for the pediatric client. 17) The nurse is performing an assessment of the ecological systems of childhood. What will the nurse include when assessing mesosystems? Select all that apply. 1. Parental involvement in school 2. Local political influences 3. Libraries in the community 4. Influences of the religious community 5. Age of each family member Answer: 1, 4 Explanation: 1. When assessing a child's mesosystem, the nurse will assess parental involvement in school and the influences of the religious community on the child and family. Local political influences and the libraries in the community are assessed in an exosystem assessment. The age of each family member is assessed during chronosystem assessment. 2. When assessing a child's mesosystem, the nurse will assess parental involvement in school and the influences of the religious community on the child and family. Local political influences and the libraries in the community are assessed in an exosystem assessment. The age of each family member is assessed during chronosystem assessment. 3. When assessing a child's mesosystem, the nurse will assess parental involvement in school and the influences of the religious community on the child and family. Local political influences and the libraries in the community are assessed in an exosystem assessment. The age of each family member is assessed during chronosystem assessment. 4. When assessing a child's mesosystem, the nurse will assess parental involvement in school and the influences of the religious community on the child and family. Local political influences and the libraries in the community are assessed in an exosystem assessment. The age of each family member is assessed during chronosystem assessment. 5. When assessing a child's mesosystem, the nurse will assess parental involvement in school and the influences of the religious community on the child and family. Local political influences and the libraries in the community are assessed in an exosystem assessment. The age of each family member is assessed during chronosystem assessment. Page Ref: 74, 76 Cognitive Level: Applying Client Need &Sub: Psychosocial Integrity Standards: QSEN Competencies: Patient-centered care | AACN Essential Competencies: Essential IX: Baccalaureate generalist nursing practice | NLN Competencies: Professional identity | Nursing/Integrated Concepts: Nursing Process: Assessment/Coordination of care Learning Outcome: LO 4.4 Explain contemporary developmental approaches such as temperament theory, ecologic theory, and the resilience framework. MNL LO: Compare and contrast known family theories and assessment strategies. 18) The nurse is assessing a toddler's development of communication skills. The nurse recognizes that a toddler communicates in what ways? Select all that apply. 1. Expressive jargon 2. Interpersonal skills and contact with other children 3. Uses all parts of speech 4. Temper tantrums 5. Enjoys talking Answer: 1, 2, 4, 5 Explanation: 1. Toddlers use expressive jargon as a communication skill. 2. Toddlers learn interpersonal skills while being in contact with other children. 3. Preschool-age children can use all parts of speech with frequent errors. 4. Toddlers use temper tantrums occasionally as a communication skill. 5. Toddlers enjoy talking. Page Ref: 87 Cognitive Level: Applying Client Need &Sub: Health Promotion and Maintenance Standards: QSEN Competencies: Patient-centered care | AACN Essential Competencies: Essential II: Basic organizational and systems leadership for quality care and patient safety. | NLN Competencies: Human flourishing | Nursing/Integrated Concepts: Nursing Process: Evaluation/Quality of practice Learning Outcome: LO 4.8 Use data collected during developmental assessments to implement activities that promote development of children and adolescents. MNL LO: Implement developmentally appropriate communication strategies for pediatric clients and families. 19) A child with hives weighing 40 pounds is prescribed diphenhydramine (Benadryl), 5 mg/kg/day in four divided doses. How many milligrams should the nurse give for each dose? Answer: 22.75 mg/dose Explanation: Convert 40 pounds to kilograms (18.18) multiply by 5 mg = 90.9 divided by 4 doses = 22.75 mg/dose Page Ref: N/A Cognitive Level: Applying Client Need &Sub: Physiological Integrity: Pharmacological and Parenteral Therapies Standards: QSEN Competencies: Patient-centered care | AACN Essential Competencies: Essential VII: Clinical prevention and population health | NLN Competencies: Human flourishing | Nursing/Integrated Concepts: Nursing Process: Evaluation/Coordination of care Learning Outcome: LO 4.3 Plan nursing interventions for children that are appropriate for each child's developmental state, based on theoretical frameworks. MNL LO: Demonstrate safe medication administration for the pediatric client. Principles of Pediatric Nursing: Caring for Children, 7e (Ball et al.) Chapter 5 Pediatric Assessment 1) The nurse is taking a health history from a family of a 3-year-old child. Which statement by the nurse would most likely establish rapport and elicit an accurate response from the family? 1. "Does any member of your family have a history of asthma, heart disease, or diabetes?" 2. "Hello, I would like to talk with you and get some information on you and your child." 3. "Tell me about the concerns that brought you to the clinic today." 4. "You will need to fill out these forms; make sure that the information is as complete as possible." Answer: 3 Explanation: 1. Asking the parents to talk about their concerns is an open-ended question and one which is more likely to establish rapport and an understanding of the parent's perceptions. Giving the family a list of items to answer at once may be confusing to the parents. Giving an introduction before asking the parents for information is likely to establish rapport, but giving an explanation of why the information would be needed would be even more effective at establishing rapport and also for getting more accurate, pertinent information. Simply asking the parents to fill out forms is very impersonal, and more information is likely to be obtained and clarified if the nurse is directing the interview. 2. Asking the parents to talk about their concerns is an open-ended question and one which is more likely to establish rapport and an understanding of the parent's perceptions. Giving the family a list of items to answer at once may be confusing to the parents. Giving an introduction before asking the parents for information is likely to establish rapport, but giving an explanation of why the information would be needed would be even more effective at establishing rapport and also for getting more accurate, pertinent information. Simply asking the parents to fill out forms is very impersonal, and more information is likely to be obtained and clarified if the nurse is directing the interview. 3. Asking the parents to talk about their concerns is an open-ended question and one which is more likely to establish rapport and an understanding of the parent's perceptions. Giving the family a list of items to answer at once may be confusing to the parents. Giving an introduction before asking the parents for information is likely to establish rapport, but giving an explanation of why the information would be needed would be even more effective at establishing rapport and also for getting more accurate, pertinent information. Simply asking the parents to fill out forms is very impersonal, and more information is likely to be obtained and clarified if the nurse is directing the interview. 4. Asking the parents to talk about their concerns is an open-ended question and one which is more likely to establish rapport and an understanding of the parent's perceptions. Giving the family a list of items to answer at once may be confusing to the parents. Giving an introduction before asking the parents for information is likely to establish rapport, but giving an explanation of why the information would be needed would be even more effective at establishing rapport and also for getting more accurate, pertinent information. Simply asking the parents to fill out forms is very impersonal, and more information is likely to be obtained and clarified if the nurse is directing the interview. Page Ref: 98-100 Cognitive Level: Analyzing Client Need &Sub: Health Promotion and Maintenance Standards: QSEN Competencies: Patient-centered care | AACN Essential Competencies: Essential VII: Clinical prevention and population health | NLN Competencies: Professional identity | Nursing/Integrated Concepts: Nursing Process: Assessment/Communication Learning Outcome: LO 5.2 Apply communication strategies to improve the quality of historical data collected. MNL LO: Apply key concepts of family-centered care. 2) When assessing the cognitive development, which technique would be appropriate to test the remote memory of a 5-year-old? 1. Say the name of an object and after 5 minutes ask the child to tell you what you said the object was. 2. Ask the child to repeat his address. 3. Ask the child to say a poem and listen to the child's speech articulation. 4. Have the child point to various parts of the body as you name them. Answer: 2 Explanation: 1. Repeating the name of an object after 5 to 10 minutes is assessing recent memory. Asking the child to repeat his address is assessing remote memory. Listening to speech articulation and pointing to body parts both assess communication skills. 2. Repeating the name of an object after 5 to 10 minutes is assessing recent memory. Asking the child to repeat his address is assessing remote memory. Listening to speech articulation and pointing to body parts both assess communication skills. 3. Repeating the name of an object after 5 to 10 minutes is assessing recent memory. Asking the child to repeat his address is assessing remote memory. Listening to speech articulation and pointing to body parts both assess communication skills. 4. Repeating the name of an object after 5 to 10 minutes is assessing recent memory. Asking the child to repeat his address is assessing remote memory. Listening to speech articulation and pointing to body parts both assess communication skills. Page Ref: 136 Cognitive Level: Analyzing Client Need &Sub: Health Promotion and Maintenance Standards: QSEN Competencies: Patient-centered care | AACN Essential Competencies: Essential VII: Clinical prevention and population health | NLN Competencies: Nursing judgement | Nursing/Integrated Concepts: Nursing Process: Assessment/Quality of practice Learning Outcome: LO 5.1 Describe the elements of a health history for infants and children of different ages. MNL LO: Use the nursing process to provide developmentally appropriate care for the pediatric client. 3) Place the nursing assessments of a toddler in the best order. 1. Examination of eyes, ears, and throat 2. Auscultation of chest 3. Palpation of abdomen 4. Developmental assessment Answer: 4, 2, 3, 1 Developmental assessment Auscultation of chest Palpation of abdomen Examination of eyes, ears, and throat Explanation: In examining a toddler, it is usually best to go from least invasive to most invasive examination in order to build trust and cooperation. Developmental assessment involves visual inspection and activities that the toddler may view as games and will likely cooperate with. Auscultation is usually less threatening to the toddler than palpation, especially if the nurse were to use the stethoscope on a parent or a toy. The most uncomfortable, invasive exam for the toddler is most likely to be the examination of the eyes, ears, and throat, so that should be performed last. Page Ref: 103-105 Cognitive Level: Analyzing Client Need &Sub: Health Promotion and Maintenance Standards: QSEN Competencies: Patient-centered care | AACN Essential Competencies: Essential VII: Clinical prevention and population health | NLN Competencies: Human flourishing | Nursing/Integrated Concepts: Nursing Process: Assessment/Quality of practice Learning Outcome: LO 5.4 Describe the differences in sequence of the physical assessment for infants, children, and adolescents. MNL LO: Examine the role of the nurse in promoting culturally competent family-centered care. 4) While assessing a 10-month-old African American infant, the nurse notices that the sclerae have a yellowish tint. Which organ system should the nurse further evaluate to determine an ongoing disease process? 1. Cardiac 2. Respiratory 3. Gastrointestinal 4. Genitourinary Answer: 3 Explanation: 1. This infant's sclerae are showing signs of jaundice, which most likely is secondary to a failure or malfunction of the liver in the gastrointestinal system. Cyanosis of the skin and mucous membranes is generally a sign of problems with the cardiac and/or respiratory system. Tenting of the skin and dry mucous membranes could be a sign of dehydration, and edema could be a sign of fluid overload. Both of these conditions could be secondary to problems with functioning of the genitourinary system. 2. This infant's sclerae are showing signs of jaundice, which most likely is secondary to a failure or malfunction of the liver in the gastrointestinal system. Cyanosis of the skin and mucous membranes is generally a sign of problems with the cardiac and/or respiratory system. Tenting of the skin and dry mucous membranes could be a sign of dehydration, and edema could be a sign of fluid overload. Both of these conditions could be secondary to problems with functioning of the genitourinary system. 3. This infant's sclerae are showing signs of jaundice, which most likely is secondary to a failure or malfunction of the liver in the gastrointestinal system. Cyanosis of the skin and mucous membranes is generally a sign of problems with the cardiac and/or respiratory system. Tenting of the skin and dry mucous membranes could be a sign of dehydration, and edema could be a sign of fluid overload. Both of these conditions could be secondary to problems with functioning of the genitourinary system. 4. This infant's sclerae are showing signs of jaundice, which most likely is secondary to a failure or malfunction of the liver in the gastrointestinal system. Cyanosis of the skin and mucous membranes is generally a sign of problems with the cardiac and/or respiratory system. Tenting of the skin and dry mucous membranes could be a sign of dehydration, and edema could be a sign of fluid overload. Both of these conditions could be secondary to problems with functioning of the genitourinary system. Page Ref: 109, 110 Cognitive Level: Analyzing Client Need &Sub: Physiological Integrity Standards: QSEN Competencies: Patient-centered care | AACN Essential Competencies: Essential VII: Clinical prevention and population health | NLN Competencies: Human flourishing | Nursing/Integrated Concepts: Nursing Process: Assessment/Quality of practice Learning Outcome: LO 5.8 Distinguish between expected and unexpected physical signs to identify at least five signs that require urgent nursing intervention. MNL LO: Gastrointestinal Disorders/Examine etiology, risk factors, pathophysiology, and clinical manifestations as seen in children. 5) A nurse caring for a school-age client notices some swelling in the child's ankles. The nurse presses against the ankle bone for five seconds, then releases the pressure and notices a markedly slow disappearance of the indentation. Which priority nursing assessment is appropriate? 1. Skin integrity, especially in the lower extremities 2. Urine output 3. Level of consciousness 4. Range of motion and ankle mobility Answer: 2 Explanation: 1. Dependent, pitting edema, especially in the lower extremities, can be a symptom of both kidney and cardiac disorders. Decreases in urine output can also indicate compromise in both the renal and cardiac systems. Changes in level of consciousness, if present, would more than likely be a later effect in this situation. While ankle edema could lead to both decreased ankle mobility and compromise in skin integrity, diagnosing and treating the underlying cause of the edema is most important. 2. Dependent, pitting edema, especially in the lower extremities, can be a symptom of both kidney and cardiac disorders. Decreases in urine output can also indicate compromise in both the renal and cardiac systems. Changes in level of consciousness, if present, would more than likely be a later effect in this situation. While ankle edema could lead to both decreased ankle mobility and compromise in skin integrity, diagnosing and treating the underlying cause of the edema is most important. 3. Dependent, pitting edema, especially in the lower extremities, can be a symptom of both kidney and cardiac disorders. Decreases in urine output can also indicate compromise in both the renal and cardiac systems. Changes in level of consciousness, if present, would more than likely be a later effect in this situation. While ankle edema could lead to both decreased ankle mobility and compromise in skin integrity, diagnosing and treating the underlying cause of the edema is most important. 4. Dependent, pitting edema, especially in the lower extremities, can be a symptom of both kidney and cardiac disorders. Decreases in urine output can also indicate compromise in both the renal and cardiac systems. Changes in level of consciousness, if present, would more than likely be a later effect in this situation. While ankle edema could lead to both decreased ankle mobility and compromise in skin integrity, diagnosing and treating the underlying cause of the edema is most important. Page Ref: 125-127 Cognitive Level: Analyzing Client Need &Sub: Physiological Integrity Standards: QSEN Competencies: Patient-centered care | AACN Essential Competencies: Essential VII: Clinical prevention and population health | NLN Competencies: Human flourishing | Nursing/Integrated Concepts: Nursing Process: Assessment/Quality of practice Learning Outcome: LO 5.8 Distinguish between expected and unexpected physical signs to identify at least five signs that require urgent nursing intervention. MNL LO: Renal and Genitourinary Disorders/Examine etiology, risk factors, pathophysiology, and clinical manifestations as seen in children. 6) A new mother is worried about a "soft spot" on the top of her newborn infant's head. The nurse informs her that this is a normal physical finding called the anterior fontanel. At what age will the nurse educate the mother that the soft spot will close? 1. 2 to 3 months of age 2. 6 to 9 months of age 3. 12 to 18 months of age 4. Approximately 2 years of age Answer: 3 Explanation: 1. The anterior fontanel is located at the top of the head and is the opening at the intersection of the suture lines. As the infant grows, the suture lines begin to fuse, and the anterior fontanel closes at 12 to 18 months of age. 2. The anterior fontanel is located at the top of the head and is the opening at the intersection of the suture lines. As the infant grows, the suture lines begin to fuse, and the anterior fontanel closes at 12 to 18 months of age. 3. The anterior fontanel is located at the top of the head and is the opening at the intersection of the suture lines. As the infant grows, the suture lines begin to fuse, and the anterior fontanel closes at 12 to 18 months of age. 4. The anterior fontanel is located at the top of the head and is the opening at the intersection of the suture lines. As the infant grows, the suture lines begin to fuse, and the anterior fontanel closes at 12 to 18 months of age. Page Ref: 109 Cognitive Level: Applying Client Need &Sub: Health Promotion and Maintenance Standards: QSEN Competencies: Patient-centered care | AACN Essential Competencies: Essential VII: Clinical prevention and population health | NLN Competencies: Human flourishing | Nursing/Integrated Concepts: Nursing Process: Implementation/Quality of practice Learning Outcome: LO 5.1 Describe the elements of a health history for infants and children of different ages. MNL LO: Examine the role of the nurse in promoting culturally competent family-centered care. 7) While inspecting a 5-year-old child's ears, the nurse notes that the right pinna protrudes outward and that there is a mass behind the right ear. In light of these findings, which vital-sign parameter would the nurse assess on priority? 1. Temperature 2. Heart rate 3. Respirations 4. Blood pressure Answer: 1 Explanation: 1. Swelling behind an ear could indicate mastoiditis, and the presence of a fever would indicate a higher index of suspicion for this. There could also be changes in other vital- sign parameters, but they would not be specific for the presence of infection. 2. Swelling behind an ear could indicate mastoiditis, and the presence of a fever would indicate a higher index of suspicion for this. There could also be changes in other vital-sign parameters, but they would not be specific for the presence of infection. 3. Swelling behind an ear could indicate mastoiditis, and the presence of a fever would indicate a higher index of suspicion for this. There could also be changes in other vital-sign parameters, but they would not be specific for the presence of infection. 4. Swelling behind an ear could indicate mastoiditis, and the presence of a fever would indicate a higher index of suspicion for this. There could also be changes in other vital-sign parameters, but they would not be specific for the presence of infection. Page Ref: 112-115 Cognitive Level: Analyzing Client Need &Sub: Physiological Integrity Standards: QSEN Competencies: Patient-centered care | AACN Essential Competencies: Essential VII: Clinical prevention and population health | NLN Competencies: Human flourishing | Nursing/Integrated Concepts: Nursing Process: Assessment/Quality of practice Learning Outcome: LO 5.8 Distinguish between expected and unexpected physical signs to identify at least five signs that require urgent nursing intervention. MNL LO: Examine the role of the nurse in promoting culturally competent family-centered care. 8) A 7-year-old child presents to the clinic with an exacerbation of asthma symptoms. On physical examination, the nurse would expect which assessment findings? Select all that apply. 1. Wheezing 2. Increased tactile fremitus 3. Decreased vocal resonance 4. Decreased tactile fremitus 5. Bronchophony Answer: 1, 3, 4 Explanation: 1. Wheezing is caused by air passing through mucus or fluids in a narrowed lower airway, which is a condition present in asthma exacerbations. The air trapping in the lungs that occurs in asthma causes a decrease in the sensation of vibrations felt, not an increase in tactile fremitus, which is indicative of pneumonia. Bronchophony is an increase in the intensity and clarity of transmitted sounds. This is also indicative of pneumonia but not asthma, which causes a decrease in vocal resonance. 2. Wheezing is caused by air passing through mucus or fluids in a narrowed lower airway, which is a condition present in asthma exacerbations. The air trapping in the lungs that occurs in asthma causes a decrease in the sensation of vibrations felt, not an increase in tactile fremitus, which is indicative of pneumonia. Bronchophony is an increase in the intensity and clarity of transmitted sounds. This is also indicative of pneumonia but not asthma, which causes a decrease in vocal resonance. 3. Wheezing is caused by air passing through mucus or fluids in a narrowed lower airway, which is a condition present in asthma exacerbations. The air trapping in the lungs that occurs in asthma causes a decrease in the sensation of vibrations felt, not an increase in tactile fremitus, which is indicative of pneumonia. Bronchophony is an increase in the intensity and clarity of transmitted sounds. This is also indicative of pneumonia but not asthma, which causes a decrease in vocal resonance. 4. Wheezing is caused by air passing through mucus or fluids in a narrowed lower airway, which is a condition present in asthma exacerbations. The air trapping in the lungs that occurs in asthma causes a decrease in the sensation of vibrations felt, not an increase in tactile fremitus, which is indicative of pneumonia. Bronchophony is an increase in the intensity and clarity of transmitted sounds. This is also indicative of pneumonia but not asthma, which causes a decrease in vocal resonance. 5. Wheezing is caused by air passing through mucus or fluids in a narrowed lower airway, which is a condition present in asthma exacerbations. The air trapping in the lungs that occurs in asthma causes a decrease in the sensation of vibrations felt, not an increase in tactile fremitus, which is indicative of pneumonia. Bronchophony is an increase in the intensity and clarity of transmitted sounds. This is also indicative of pneumonia but not asthma, which causes a decrease in vocal resonance. Page Ref: 120-124 Cognitive Level: Analyzing Client Need &Sub: Physiological Integrity Standards: QSEN Competencies: Patient-centered care | AACN Essential Competencies: Essential VII: Clinical prevention and population health | NLN Competencies: Nursing judgement | Nursing/Integrated Concepts: Nursing Process: Assessment/Quality of practice Learning Outcome: LO 5.8 Distinguish between expected and unexpected physical signs to identify at least five signs that require urgent nursing intervention. MNL LO: Apply the general concepts related to caring for a child with a chronic illness. 9) The nurse is caring for a newly-admitted infant diagnosed with "failure to thrive." The nurse begins to implement the healthcare provider prescribed orders by taking blood pressures in all four extremities. Which congenital cardiac defect does the nurse anticipate based on the prescribed order? 1. Tetralogy of Fallot 2. Pulmonary atresia 3. Coarctation of the aorta 4. Ventricular septal defect Answer: 3 Explanation: 1. Normally, blood pressures in the lower extremities are the same as or higher than upper-extremity blood pressures. But in coarctation of the aorta, the narrowing of the aorta causes decreased blood flow to the lower extremities and thus lower-extremity blood-pressure readings are significantly lower than upper-extremity readings. There are minimal differences between upper and lower blood-pressure readings in tetralogy of Fallot, pulmonary atresia, and ventricular septal defect. 2. Normally, blood pressures in the lower extremities are the same as or higher than upper- extremity blood pressures. But in coarctation of the aorta, the narrowing of the aorta causes decreased blood flow to the lower extremities and thus lower-extremity blood-pressure readings are significantly lower than upper-extremity readings. There are minimal differences between upper and lower blood-pressure readings in tetralogy of Fallot, pulmonary atresia, and ventricular septal defect. 3. Normally, blood pressures in the lower extremities are the same as or higher than upper- extremity blood pressures. But in coarctation of the aorta, the narrowing of the aorta causes decreased blood flow to the lower extremities and thus lower-extremity blood-pressure readings are significantly lower than upper-extremity readings. There are minimal differences between upper and lower blood-pressure readings in tetralogy of Fallot, pulmonary atresia, and ventricular septal defect. 4. Normally, blood pressures in the lower extremities are the same as or higher than upper- extremity blood pressures. But in coarctation of the aorta, the narrowing of the aorta causes decreased blood flow to the lower extremities and thus lower-extremity blood-pressure readings are significantly lower than upper-extremity readings. There are minimal differences between upper and lower blood-pressure readings in tetralogy of Fallot, pulmonary atresia, and ventricular septal defect. Page Ref: 125-127 Cognitive Level: Applying Client Need &Sub: Physiological Integrity Standards: QSEN Competencies: Patient-centered care | AACN Essential Competencies: Essential VII: Clinical prevention and population health | NLN Competencies: Nursing judgement | Nursing/Integrated Concepts: Nursing Process: Assessment/Quality of practice Learning Outcome: LO 5.8 Distinguish between expected and unexpected physical signs to identify at least five signs that require urgent nursing intervention. MNL LO: Cardiovascular and Hematological Disorders/Examine etiology, risk factors, pathophysiology, and clinical manifestations as seen in children. 10) During an examination, a nurse asks a 5-year-old child to repeat his address. What is the nurse evaluating with this action? 1. Recent memory 2. Language development 3. Remote memory 4. Social-skill development Answer: 3 Explanation: 1. Asking children to remember addresses, phone numbers, and dates assesses remote-memory development. To evaluate recent memory, the nurse would have the child name something and then ask him to name it again in 10 to 15 minutes. Listening to how the child talks and his sentence structure evaluates the child's language development, and assessing how he interacts with others evaluates social-skill development. 2. Asking children to remember addresses, phone numbers, and dates assesses remote-memory development. To evaluate recent memory, the nurse would have the child name something and then ask him to name it again in 10 to 15 minutes. Listening to how the child talks and his sentence structure evaluates the child's language development, and assessing how he interacts with others evaluates social-skill development. 3. Asking children to remember addresses, phone numbers, and dates assesses remote-memory development. To evaluate recent memory, the nurse would have the child name something and then ask him to name it again in 10 to 15 minutes. Listening to how the child talks and his sentence structure evaluates the child's language development, and assessing how he interacts with others evaluates social-skill development. 4. Asking children to remember addresses, phone numbers, and dates assesses remote-memory development. To evaluate recent memory, the nurse would have the child name something and then ask him to name it again in 10 to 15 minutes. Listening to how the child talks and his sentence structure evaluates the child's language development, and assessing how he interacts with others evaluates social-skill development. Page Ref: 136 Cognitive Level: Analyzing Client Need &Sub: Health Promotion and Maintenance Standards: QSEN Competencies: Patient-centered care | AACN Essential Competencies: Essential VII: Clinical prevention and population health | NLN Competencies: Nursing judgement | Nursing/Integrated Concepts: Nursing Process: Assessment/Communication Learning Outcome: LO 5.2 Apply communication strategies to improve the quality of historical data collected. MNL LO: Compare the developmental stages for pediatric clients. 11) During the newborn examination, the nurse assesses the infant for signs of developmental dysplasia of the hip. A finding that would strongly indicate this disorder would be: 1. soles are flat with prominent fat pads. 2. positive Babinski reflex. 3. metatarsus varus. 4. asymmetric thigh and gluteal folds. Answer: 4 Explanation: 1. A positive Babinski reflex and flat soles are normal newborn findings. Metatarsus varus is an in-toeing of the feet that usually occurs secondary to intra-uterine positioning and frequently resolves on its own, but approximately 10 percent of infants with metatarsus varus also have developmental dysplasia of the hip. Asymmetric thigh and gluteal folds are a positive finding for developmental dysplasia of the hip requiring follow-up with ultrasound. 2. A positive Babinski reflex and flat soles are normal newborn findings. Metatarsus varus is an in-toeing of the feet that usually occurs secondary to intra-uterine positioning and frequently resolves on its own, but approximately 10 percent of infants with metatarsus varus also have developmental dysplasia of the hip. Asymmetric thigh and gluteal folds are a positive finding for developmental dysplasia of the hip requiring follow-up with ultrasound. 3. A positive Babinski reflex and flat soles are normal newborn findings. Metatarsus varus is an in-toeing of the feet that usually occurs secondary to intra-uterine positioning and frequently resolves on its own, but approximately 10 percent of infants with metatarsus varus also have developmental dysplasia of the hip. Asymmetric thigh and gluteal folds are a positive finding for developmental dysplasia of the hip requiring follow-up with ultrasound. 4. A positive Babinski reflex and flat soles are normal newborn findings. Metatarsus varus is an in-toeing of the feet that usually occurs secondary to intra-uterine positioning and frequently resolves on its own, but approximately 10 percent of infants with metatarsus varus also have developmental dysplasia of the hip. Asymmetric thigh and gluteal folds are a positive finding for developmental dysplasia of the hip requiring follow-up with ultrasound. Page Ref: 133, 134 Cognitive Level: Analyzing Client Need &Sub: Physiological Integrity Standards: QSEN Competencies: Patient-centered care | AACN Essential Competencies: Essential VII: Clinical prevention and population health | NLN Competencies: Human flourishing | Nursing/Integrated Concepts: Nursing Process: Assessment/Quality of practice Learning Outcome: LO 5.8 Distinguish between expected and unexpected physical signs to identify at least five signs that require urgent nursing intervention. MNL LO: Integumentary and Musculoskeletal Disorders/Examine etiology, risk factors, pathophysiology, and clinical manifestations as seen in children. 12) The nurse must assess each of the 2-year-olds listed below. Which one should be evaluated first? 1. A child with a temperature of 101 degrees F 2. A child who has stridor 3. A child who has absent Babinski sign 4. A child who has a pot belly appearance Answer: 2 Explanation: 1. A child with stridor is at risk for airway compromise; a child with a temperature of 101 degrees F, while sick, is not as ill as the child with stridor; and the child with an absent Babinski sign and the pot-bellied child are normal. 2. A child with stridor is at risk for airway compromise; a child with a temperature of 101 degrees F, while sick, is not as ill as the child with stridor; and the child with an absent Babinski sign and the pot-bellied child are normal. 3. A child with stridor is at risk for airway compromise; a child with a temperature of 101 degrees F, while sick, is not as ill as the child with stridor; and the child with an absent Babinski sign and the pot-bellied child are normal. 4. A child with stridor is at risk for airway compromise; a child with a temperature of 101 degrees F, while sick, is not as ill as the child with stridor; and the child with an absent Babinski sign and the pot-bellied child are normal. Page Ref: 120-124 Cognitive Level: Analyzing Client Need &Sub: Physiological Integrity Standards: QSEN Competencies: Patient-centered care | AACN Essential Competencies: Essential VII: Clinical prevention and population health | NLN Competencies: Nursing judgement | Nursing/Integrated Concepts: Nursing Process: Assessment/Quality of practice Learning Outcome: LO 5.8 Distinguish between expected and unexpected physical signs to identify at least five signs that require urgent nursing intervention. MNL LO: Evaluate healthcare issues related to pediatric nursing care. 13) The nurse notes a history of a grade III heart murmur in a small infant. When assessing the heart, the nurse would expect to: 1. Auscultate a quiet but easily heard murmur. 2. Auscultate a moderately loud murmur without a palpable thrill. 3. Auscultate a very loud murmur with easily palpable thrill. 4. Listen without a stethoscope and hear a murmur at chest wall. Answer: 2 Explanation: 1. A quiet but easily heard murmur is a grade II. A moderately loud murmur without palpable thrill is a grade III. A very loud murmur with easily palpable thrill is a grade V. A murmur heard at the chest wall without the aid of a stethoscope is a grade VI. 2. A quiet but easily heard murmur is a grade II. A moderately loud murmur without palpable thrill is a grade III. A very loud murmur with easily palpable thrill is a grade V. A murmur heard at the chest wall without the aid of a stethoscope is a grade VI. 3. A quiet but easily heard murmur is a grade II. A moderately loud murmur without palpable thrill is a grade III. A very loud murmur with easily palpable thrill is a grade V. A murmur heard at the chest wall without the aid of a stethoscope is a grade VI. 4. A quiet but easily heard murmur is a grade II. A moderately loud murmur without palpable thrill is a grade III. A very loud murmur with easily palpable thrill is a grade V. A murmur heard at the chest wall without the aid of a stethoscope is a grade VI. Page Ref: 125,126 Cognitive Level: Analyzing Client Need &Sub: Physiological Integrity Standards: QSEN Competencies: Patient-centered care | AACN Essential Competencies: Essential VII: Clinical prevention and population health | NLN Competencies: Professional identity | Nursing/Integrated Concepts: Nursing Process: Implementation/Quality of practice Learning Outcome: LO 5.8 Distinguish between expected and unexpected physical signs to identify at least five signs that require urgent nursing intervention. MNL LO: Cardiovascular and Hematological Disorders/Examine etiology, risk factors, pathophysiology, and clinical manifestations as seen in children. 14) The nurse is measuring an abdominal girth on a child with abdominal distension. Identify the area on the child's abdomen where the tape measure should be placed for an accurate abdominal girth. 1. Just above the umbilicus, around the largest circumference of the abdomen 2. Below the umbilicus 3. Just below the sternum 4. Just above the pubic bone Answer: 1 Explanation: 1. An abdominal girth should be taken around the largest circumference of the abdomen, in this case, just above the umbilicus. The circumference below the umbilicus or just below the sternum would not be an accurate abdominal girth. 2. An abdominal girth should be taken around the largest circumference of the abdomen, in this case, just above the umbilicus. The circumference below the umbilicus or just below the sternum would not be an accurate abdominal girth. 3. An abdominal girth should be taken around the largest circumference of the abdomen, in this case, just above the umbilicus. The circumference below the umbilicus or just below the sternum would not be an accurate abdominal girth. 4. An abdominal girth should be taken around the largest circumference of the abdomen, in this case, just above the umbilicus. The circumference below the umbilicus or just below the sternum would not be an accurate abdominal girth. Page Ref: 127, 128 Cognitive Level: Applying Client Need &Sub: Health Promotion and Maintenance Standards: QSEN Competencies: Patient-centered care | AACN Essential Competencies: Essential VII: Clinical prevention and population health | NLN Competencies: Human flourishing | Nursing/Integrated Concepts: Nursing Process: Assessment/Quality of practice Learning Outcome: LO 5.5 Modify physical assessment techniques according to the age and developmental stage of the child. MNL LO: Evaluate healthcare issues related to pediatric nursing care. 15) The nurse is preparing to assess a toddler client. Which activities would gain cooperation from the toddler? Select all that apply. 1. Asking the parents to wait outside 2. Allowing the client to sit in the parent's lap 3. Administering vaccinations prior to the assessment 4. Handing the client a stethoscope while taking the health history 5. Making a game out of the assessment process Answer: 2, 4 Explanation: 1. Allowing the client to stay on the parents lap and allowing the client to play with instruments that will be used in the assessment process are activities the nurse can implement to gain the toddler's cooperation during the assessment process. Asking the parents to wait outside may cause the toddler to become fearful. Vaccinations should be administered at the end of the visit. While making a game out of the assessment process may be appropriate for older children, this is not an appropriate strategy for a toddler client. 2. Allowing the client to stay on the parents lap and allowing the client to play with instruments that will be used in the assessment process are activities the nurse can implement to gain the toddler's cooperation during the assessment process. Asking the parents to wait outside may cause the toddler to become fearful. Vaccinations should be administered at the end of the visit. While making a game out of the assessment process may be appropriate for older children, this is not an appropriate strategy for a toddler client. 3. Allowing the client to stay on the parents lap and allowing the client to play with instruments that will be used in the assessment process are activities the nurse can implement to gain the toddler's cooperation during the assessment process. Asking the parents to wait outside may cause the toddler to become fearful. Vaccinations should be administered at the end of the visit. While making a game out of the assessment process may be appropriate for older children, this is not an appropriate strategy for a toddler client. 4. Allowing the client to stay on the parents lap and allowing the client to play with instruments that will be used in the assessment process are activities the nurse can implement to gain the toddler's cooperation during the assessment process. Asking the parents to wait outside may cause the toddler to become fearful. Vaccinations should be administered at the end of the visit. While making a game out of the assessment process may be appropriate for older children, this is not an appropriate strategy for a toddler client. 5. Allowing the client to stay on the parents lap and allowing the client to play with instruments that will be used in the assessment process are activities the nurse can implement to gain the toddler's cooperation during the assessment process. Asking the parents to wait outside may cause the toddler to become fearful. Vaccinations should be administered at the end of the visit. While making a game out of the assessment process may be appropriate for older children, this is not an appropriate strategy for a toddler client. Page Ref: 105 Cognitive Level: Applying Client Need &Sub: Physiological Integrity Standards: QSEN Competencies: Patient-centered care | AACN Essential Competencies: Essential VII: Clinical prevention and population health | NLN Competencies: Human flourishing | Nursing/Integrated Concepts: Nursing Process: Assessment/Quality of practice Learning Outcome: LO 5.3 Demonstrate strategies to gain cooperation of a young child for assessment. MNL LO: Compare the developmental stages for pediatric clients. 16) The nurse is assessing an infant client during a health supervision visit. Which assessment findings are considered normal variations for this client? Select all that apply. 1. Sucking pads in the mouth 2. A rounded chest 3. Hearing breath sounds over the entire chest 4. Pubertal development 5. Knock-knees Answer: 1, 2, 3 Explanation: 1. Normal variations for the infant client include sucking pads in the mouth, a rounded chest, and hearing breath sounds over the entire chest. Pubertal development and knock- knees are not normal variations for the infant client. 2. Normal variations for the infant client include sucking pads in the mouth, a rounded chest, and hearing breath sounds over the entire chest. Pubertal development and knock-knees are not normal variations for the infant client. 3. Normal variations for the infant client include sucking pads in the mouth, a rounded chest, and hearing breath sounds over the entire chest. Pubertal development and knock-knees are not normal variations for the infant client. 4. Normal variations for the infant client include sucking pads in the mouth, a rounded chest, and hearing breath sounds over the entire chest. Pubertal development and knock-knees are not normal variations for the infant client. 5. Normal variations for the infant client include sucking pads in the mouth, a rounded chest, and hearing breath sounds over the entire chest. Pubertal development and knock-knees are not normal variations for the infant client. Page Ref: 106-107; 123-134 Cognitive Level: Applying Client Need &Sub: Physiological Integrity Standards: QSEN Competencies: Patient-centered care | AACN Essential Competencies: Essential VII: Clinical prevention and population health | NLN Competencies: Human flourishing | Nursing/Integrated Concepts: Nursing Process: Assessment/Quality of practice Learning Outcome: LO 5.6 List five normal variations in pediatric physical findings (such as breast budding in a girl) found during a physical assessment. MNL LO: Compare the developmental stages for pediatric clients. 17) The nurse is conducting a health surveillance visit with a 6-month-old infant. Which methods are appropriate to monitor the infant's growth pattern since birth? Select all that apply. 1. Weight the infant twice and average together 2. Measure the infant's height 3. Measure the infant's head circumference 4. Determine the infant's body mass index 5. Plot the infant's growth on appropriate chart Answer: 1, 3, 5 Explanation: 1. In order to determine the infant's growth pattern, the nurse will obtain two weights and average them together, measure the infant's head circumference, and obtain the infant's length, not height. After the measurements have been obtained the nurse will plot the measurements on the appropriate growth chart and monitor the infant's growth pattern. Body mass index is not determined during infancy. 2. In order to determine the infant's growth pattern, the nurse will obtain two weights and average them together, measure the infant's head circumference, and obtain the infant's length, not height. After the measurements have been obtained the nurse will plot the measurements on the appropriate growth chart and monitor the infant's growth pattern. Body mass index is not determined during infancy. 3. In order to determine the infant's growth pattern, the nurse will obtain two weights and average them together, measure the infant's head circumference, and obtain the infant's length, not height. After the measurements have been obtained the nurse will plot the measurements on the appropriate growth chart and monitor the infant's growth pattern. Body mass index is not determined during infancy. 4. In order to determine the infant's growth pattern, the nurse will obtain two weights and average them together, measure the infant's head circumference, and obtain the infant's length, not height. After the measurements have been obtained the nurse will plot the measurements on the appropriate growth chart and monitor the infant's growth pattern. Body mass index is not determined during infancy. 5. In order to determine the infant's growth pattern, the nurse will obtain two weights and average them together, measure the infant's head circumference, and obtain the infant's length, not height. After the measurements have been obtained the nurse will plot the measurements on the appropriate growth chart and monitor the infant's growth pattern. Body mass index is not determined during infancy. Page Ref: 105 Cognitive Level: Applying Client Need &Sub: Physiological Integrity Standards: QSEN Competencies: Patient-centered care | AACN Essential Competencies: Essential VII: Clinical prevention and population health | NLN Competencies: Human flourishing | Nursing/Integrated Concepts: Nursing Process: Assessment/Quality of practice Learning Outcome: LO 5.7 Evaluate the growth pattern of an infant or child. MNL LO: Compare the developmental stages for pediatric clients. 18) The nurse is assessing a school-age child's extraocular movements. The nurse recognizes which cranial nerves that involve testing extraocular movements? Select all that apply. 1. VII 2. III 3. IV 4. XII 5. VI Answer: 2, 3, 5 Explanation: 1. VII is the facial nerve and is not involved in testing extraocular movements. Cranial nerves III, IV, and VI dominate eye movements and pupil constriction. Cranial nerve VII dominates the person's ability to smile and frown and cranial nerve XII dominates tongue movements. 2. III is the nerve and is involved in testing extraocular movements. 3. IV is the nerve and is involved in testing extraocular movements. 4. XII is the hypoglossal nerve and is not involved in testing extraocular movements. Cranial nerves III, IV, and VI dominate eye movements and pupil constriction. Cranial nerve VII dominates the person's ability to smile and frown and cranial nerve XII dominates tongue movements. 5. VI is the nerve and is involved in testing extraocular movements. Page Ref: 87 Cognitive Level: Applying Client Need &Sub: Health Promotion and Maintenance Standards: QSEN Competencies: Patient-centered care | AACN Essential Competencies: Essential II: Basic organizational and systems leadership for quality care and patient safety. | NLN Competencies: Human flourishing | Nursing/Integrated Concepts: Nursing Process: Evaluation/Quality of practice Learning Outcome: LO 5.4 Describe the differences in sequence of the physical assessment for infants, children, and adolescents. MNL LO: Evaluate healthcare issues related to pediatric nursing care. Principles of Pediatric Nursing: Caring for Children, 7e (Ball et al.) Chapter 6 Introduction to Health Promotion and Maintenance 1) A nurse is helping the parents of 2-year-old twins cope with the daily demands of life in an active household. Which strategy is most appropriate for the nurse to use? 1. Health maintenance 2. Health promotion 3. Health protection 4. Health supervision Answer: 2 Explanation: 1. In health promotion, nurses partner with families to promote family strategies in the areas of lifestyle and coping. The definition of health maintenance and health supervision makes the other answers incorrect. Health protection is another term for health maintenance. 2. In health promotion, nurses partner with families to promote family strategies in the areas of lifestyle and coping. The definition of health maintenance and health supervision makes the other answers incorrect. Health protection is another term for health maintenance. 3. In health promotion, nurses partner with families to promote family strategies in the areas of lifestyle and coping. The definition of health maintenance and health supervision makes the other answers incorrect. Health protection is another term for health maintenance. 4. In health promotion, nurses partner with families to promote family strategies in the areas of lifestyle and coping. The definition of health maintenance and health supervision makes the other answers incorrect. Health protection is another term for health maintenance. Page Ref: 146 Cognitive Level: Applying Client Need &Sub: Health Promotion and Maintenance Standards: QSEN Competencies: Patient-centered care | AACN Essential Competencies: Essential VII: Clinical prevention and population health | NLN Competencies: Nursing judgement | Nursing/Integrated Concepts: Nursing Process: Implementation/Collaboration Learning Outcome: LO 6.1 Define health promotion and health maintenance. MNL LO: Develop a family-centered nursing care plan for the child and family. 2) A nurse in the outpatient pediatric clinic is reviewing the records of a preschool-age child and notes that because the parents often miss routine healthcare visits the child has not received the second measles, mumps, and rubella (MMR) vaccine. Which action by the nurse is most appropriate in this situation? 1. Speak firmly with the parents about the importance of being compliant. 2. Notify the physician that the child's immunizations are no longer up to date. 3. Call the parents and encourage them to bring the child for recommended care. 4. Plan to discuss the principles of health supervision at the next scheduled visit. Answer: 3 Explanation: 1. The nurse in the pediatric healthcare setting is responsible for reviewing the health supervision of the child. Partnering with the parents and encouraging the parents to follow health-supervision guidelines are the best strategies to use. Speaking firmly with the parents about compliance will alienate the parents at this time. A discussion of the principles of health supervision without an intervention at this visit would mean a delay in needed healthcare for the child in this example. Discussing with the physician that the immunizations are not up to date is not necessary in an outpatient clinic. Immunizations are given per schedule. 2. The nurse in the pediatric healthcare setting is responsible for reviewing the health supervision of the child. Partnering with the parents and encouraging the parents to follow health-supervision guidelines are the best strategies to use. Speaking firmly with the parents about compliance will alienate the parents at this time. A discussion of the principles of health supervision without an intervention at this visit would mean a delay in needed healthcare for the child in this example. Discussing with the physician that the immunizations are not up to date is not necessary in an outpatient clinic. Immunizations are given per schedule. 3. The nurse in the pediatric healthcare setting is responsible for reviewing the health supervision of the child. Partnering with the parents and encouraging the parents to follow health-supervision guidelines are the best strategies to use. Speaking firmly with the parents about compliance will alienate the parents at this time. A discussion of the principles of health supervision without an intervention at this visit would mean a delay in needed healthcare for the child in this example. Discussing with the physician that the immunizations are not up to date is not necessary in an outpatient clinic. Immunizations are given per schedule. 4. The nurse in the pediatric healthcare setting is responsible for reviewing the health supervision of the child. Partnering with the parents and encouraging the parents to follow health-supervision guidelines are the best strategies to use. Speaking firmly with the parents about compliance will alienate the parents at this time. A discussion of the principles of health supervision without an intervention at this visit would mean a delay in needed healthcare for the child in this example. Discussing with the physician that the immunizations are not up to date is not necessary in an outpatient clinic. Immunizations are given per schedule. Page Ref: 147 Cognitive Level: Applying Client Need &Sub: Health Promotion and Maintenance Standards: QSEN Competencies: Patient-centered care | AACN Essential Competencies: Essential II: Basic organizational and systems leadership for quality care and patient safety | NLN Competencies: Human flourishing | Nursing/Integrated Concepts: Nursing Process: Implementation/Communication Learning Outcome: LO 6.4 Analyze the nurse's role in providing health promotion and health maintenance for children and families. MNL LO: Examine the role of the nurse in promoting culturally competent family-centered care. 3) A mother brings a child to the pediatric office for a sick visit. Which action by the nurse is the most appropriate? 1. Focus exclusively on the reported illness. 2. Review health-promotion and health-maintenance activities. 3. Ask the mother to leave the room after obtaining the history. 4. Obtain a comprehensive history, including sociodemographic data. Answer: 2 Explanation: 1. A nurse should use every opportunity during an office visit to review health- promotion and health-maintenance activities. Focusing exclusively on the reported illnesses ignores the opportunity to use health-promotion strategies. There is not enough data in this scenario to determine whether the mother should be asked to leave the room. There is not enough information to indicate that a comprehensive history should be taken at this visit. 2. A nurse should use every opportunity during an office visit to review health-promotion and health-maintenance activities. Focusing exclusively on the reported illnesses ignores the opportunity to use health-promotion strategies. There is not enough data in this scenario to determine whether the mother should be asked to leave the room. There is not enough information to indicate that a comprehensive history should be taken at this visit. 3. A nurse should use every opportunity during an office visit to review health-promotion and health-maintenance activities. Focusing exclusively on the reported illnesses ignores the opportunity to use health-promotion strategies. There is not enough data in this scenario to determine whether the mother should be asked to leave the room. There is not enough information to indicate that a comprehensive history should be taken at this visit. 4. A nurse should use every opportunity during an office visit to review health-promotion and health-maintenance activities. Focusing exclusively on the reported illnesses ignores the opportunity to use health-promotion strategies. There is not enough data in this scenario to determine whether the mother should be asked to leave the room. There is not enough information to indicate that a comprehensive history should be taken at this visit. Page Ref: 148 Cognitive Level: Applying Client Need &Sub: Health Promotion and Maintenance Standards: QSEN Competencies: Patient-centered care | AACN Essential Competencies: Essential VII: Clinical prevention and population health | NLN Competencies: Human flourishing | Nursing/Integrated Concepts: Nursing Process: Implementation/Health teaching and health promotion Learning Outcome: LO 6.4 Analyze the nurse's role in providing health promotion and health maintenance for children and families. MNL LO: Develop a family-centered nursing care plan for the child and family. 4) Which of these strategies would be most effective for a teachable moment during a routine office visit for the parents of a 6-year-old child? 1. Select one topic and present a brief amount of information on the topic. 2. Review all 6-year-old anticipatory guidelines with the parents. 3. Review 7-year-old anticipatory guidelines with the parents. 4. Discuss signs of malnutrition with the parents. Answer: 1 Explanation: 1. Children and families often learn best when presented with small bits of information. Do not give too much information to the parents at one time; therefore, selecting one topic and presenting information is appropriate. It is not appropriate to discuss malnutrition with these parents, since nothing in the stem of the question indicates that the child has a problem with nutrition. 2. Children and families often learn best when presented with small bits of information. Do not give too much information to the parents at one time; therefore, selecting one topic and presenting information is appropriate. It is not appropriate to discuss malnutrition with these parents, since nothing in the stem of the question indicates that the child has a problem with nutrition. 3. Children and families often learn best when presented with small bits of information. Do not give too much information to the parents at one time; therefore, selecting one topic and presenting information is appropriate. It is not appropriate to discuss malnutrition with these parents, since nothing in the stem of the question indicates that the child has a problem with nutrition. 4. Children and families often learn best when presented with small bits of information. Do not give too much information to the parents at one time; therefore, selecting one topic and presenting information is appropriate. It is not appropriate to discuss malnutrition with these parents, since nothing in the stem of the question indicates that the child has a problem with nutrition. Page Ref: 148 Cognitive Level: Analyzing Client Need &Sub: Health Promotion and Maintenance Standards: QSEN Competencies: Patient-centered care | AACN Essential Competencies: Essential I: Liberal education for Baccalaureate generalist nursing practice | NLN Competencies: Nursing judgement | Nursing/Integrated Concepts: Nursing Process: Evaluation/Health teaching and health promotion Learning Outcome: LO 6.3 Describe the components of a health supervision visit. MNL LO: Analyze the role of the nurse and the role of the family in pediatric care. 5) The clinic administrator has asked each nurse to classify the nursing activities as a beginning step of clinic reorganization. Which of these strategies can be identified as health promotion and health maintenance? Select all that apply. 1. Administration of the flu vaccine for infants from 6 months to 23 months old 2. Daily feeding schedules for infants 3. Instruction to adolescents on how to use dental floss 4. Treatment for a child with a diagnosis of acute otitis media Answer: 1, 2, 3 Explanation: 1. Administering flu vaccines, discussion of feeding schedules, and instructions to adolescents are all health-promotion and health-maintenance topics. Treatment of an acute ear infection (otitis media) would not be a topic for health promotion and health maintenance since it is an acute illness. 2. Administering flu vaccines, discussion of feeding schedules, and instructions to adolescents are all health-promotion and health-maintenance topics. Treatment of an acute ear infection (otitis media) would not be a topic for health promotion and health maintenance since it is an acute illness. 3. Administering flu vaccines, discussion of feeding schedules, and instructions to adolescents are all health-promotion and health-maintenance topics. Treatment of an acute ear infection (otitis media) would not be a topic for health promotion and health maintenance since it is an acute illness. 4. Administering flu vaccines, discussion of feeding schedules, and instructions to adolescents are all health-promotion and health-maintenance topics. Treatment of an acute ear infection (otitis media) would not be a topic for health promotion and health maintenance since it is an acute illness. Page Ref: 148 Cognitive Level: Analyzing Client Need &Sub: Health Promotion and Maintenance Standards: QSEN Competencies: Teamwork and collaboration | AACN Essential Competencies: Essential V: Healthcare policy, finance, and regulatory environments | NLN Competencies: Nursing judgement | Nursing/Integrated Concepts: Nursing Process: Implementation/Health teaching and health promotion Learning Outcome: LO 6.1 Define health promotion and health maintenance. MNL LO: Explore the practice of pediatric healthcare. 6) A mother of a 2-year-old child becomes very anxious when the child has a temper tantrum in the medical office. Which response by the nurse is the most appropriate? 1. "What do you usually do or say during a temper tantrum?" 2. "Let's ignore this behavior; it will stop sooner or later." 3. "Pick up and cuddle your child now, please." 4. "This is definitely a temper tantrum; I know exactly what you are feeling right now." Answer: 1 Explanation: 1. Asking the mother to describe her usual behavior via an open-ended question will encourage the mother to talk about home management and will lead the nurse to assist the mother in making a plan of care for temper tantrums. Ignoring the behavior, instructing the mother to cuddle the child, or sympathizing with the mother ("I know exactly what you are feeling") are not effective ways to problem solve for temper tantrums. 2. Asking the mother to describe her usual behavior via an open-ended question will encourage the mother to talk about home management and will lead the nurse to assist the mother in making a plan of care for temper tantrums. Ignoring the behavior, instructing the mother to cuddle the child, or sympathizing with the mother ("I know exactly what you are feeling") are not effective ways to problem solve for temper tantrums. 3. Asking the mother to describe her usual behavior via an open-ended question will encourage the mother to talk about home management and will lead the nurse to assist the mother in making a plan of care for temper tantrums. Ignoring the behavior, instructing the mother to cuddle the child, or sympathizing with the mother ("I know exactly what you are feeling") are not effective ways to problem solve for temper tantrums. 4. Asking the mother to describe her usual behavior via an open-ended question will encourage the mother to talk about home management and will lead the nurse to assist the mother in making a plan of care for temper tantrums. Ignoring the behavior, instructing the mother to cuddle the child, or sympathizing with the mother ("I know exactly what you are feeling") are not effective ways to problem solve for temper tantrums. Page Ref: 149 Cognitive Level: Analyzing Client Need &Sub: Health Promotion and Maintenance Standards: QSEN Competencies: Patient-centered care | AACN Essential Competencies: Essential VI: Interprofessional communication and collaboration for improving patient health outcomes | NLN Competencies: Nursing judgement | Nursing/Integrated Concepts: Nursing Process: Implementation/Health teaching and health promotion Learning Outcome: LO 6.8 Apply the nursing process in assessment, diagnosis, goal setting, intervention, and evaluation of health promotion and health maintenance activities for children and families. MNL LO: Develop a family-centered nursing care plan for the child and family. 7) A nurse says to the mother of a 6-month-old infant, "Does the baby sit without assistance, and is the baby crawling?" Which process is the nurse using in this interaction? 1. Health promotion 2. Health maintenance 3. Disease surveillance 4. Developmental surveillance Answer: 4 Explanation: 1. The question asked by the nurse is seeking information about developmental milestones; therefore, the nurse is involved in developmental surveillance. While health- promotion and health-maintenance activities are related to developmental surveillance, this question is looking specifically at the milestones; therefore, the answers "health promotion" and "health maintenance" are incorrect. The questions asked in the stem are not classified as disease- surveillance questions. 2. The question asked by the nurse is seeking information about developmental milestones; therefore, the nurse is involved in developmental surveillance. While health-promotion and health-maintenance activities are related to developmental surveillance, this question is looking specifically at the milestones; therefore, the answers "health promotion" and "health maintenance" are incorrect. The questions asked in the stem are not classified as disease- surveillance questions. 3. The question asked by the nurse is seeking information about developmental milestones; therefore, the nurse is involved in developmental surveillance. While health-promotion and health-maintenance activities are related to developmental surveillance, this question is looking specifically at the milestones; therefore, the answers "health promotion" and "health maintenance" are incorrect. The questions asked in the stem are not classified as disease- surveillance questions. 4. The question asked by the nurse is seeking information about developmental milestones; therefore, the nurse is involved in developmental surveillance. While health-promotion and health-maintenance activities are related to developmental surveillance, this question is looking specifically at the milestones; therefore, the answers "health promotion" and "health maintenance" are incorrect. The questions asked in the stem are not classified as disease- surveillance questions. Page Ref: 149 Cognitive Level: Analyzing Client Need &Sub: Health Promotion and Maintenance Standards: QSEN Competencies: Patient-centered care | AACN Essential Competencies: Essential VII: Clinical prevention and population health | NLN Competencies: Human flourishing | Nursing/Integrated Concepts: Nursing Process: Assessment/Health teaching and health promotion Learning Outcome: LO 6.6 Synthesize the areas of assessment and intervention for health supervision visits–growth and developmental surveillance, nutrition, physical activity, oral health, mental and spiritual health, family and social relations, disease prevention strategies, and injury prevention strategies. MNL LO: Compare the developmental stages for pediatric clients. 8) A parent says to a nurse, "How do you know when my child needs these screening tests the doctor just mentioned?" Which response by the nurse is the most appropriate? 1. "Screening tests are administered at the ages when a child is most likely to develop a condition." 2. "Screening tests are done in the newborn nursery and from these results, additional screening tests are ordered throughout the first two years of life." 3. "Screening tests are most often done when the doctor suspects something is wrong with the child." 4. "Screening tests are done at each office visit." Answer: 1 Explanation: 1. Screening tests administered at ages when a child is most likely to develop a condition provide a good basis for health promotion. The remaining answers all provide incorrect information to the parent. Abnormal newborn screening tests require immediate follow-up. Screening tests are done to detect the possibility of problems, not when a problem is suspected; at that point, a child needs diagnostic testing. Screening tests are not done at each office visit. 2. Screening tests administered at ages when a child is most likely to develop a condition provide a good basis for health promotion. The remaining answers all provide incorrect information to the parent. Abnormal newborn screening tests require immediate follow-up. Screening tests are done to detect the possibility of problems, not when a problem is suspected; at that point, a child needs diagnostic testing. Screening tests are not done at each office visit. 3. Screening tests administered at ages when a child is most likely to develop a condition provide a good basis for health promotion. The remaining answers all provide incorrect information to the parent. Abnormal newborn screening tests require immediate follow-up. Screening tests are done to detect the possibility of problems, not when a problem is suspected; at that point, a child needs diagnostic testing. Screening tests are not done at each office visit. 4. Screening tests administered at ages when a child is most likely to develop a condition provide a good basis for health promotion. The remaining answers all provide incorrect information to the parent. Abnormal newborn screening tests require immediate follow-up. Screening tests are done to detect the possibility of problems, not when a problem is suspected; at that point, a child needs diagnostic testing. Screening tests are not done at each office visit. Page Ref: 151 Cognitive Level: Applying Client Need &Sub: Health Promotion and Maintenance Standards: QSEN Competencies: Patient-centered care | AACN Essential Competencies: Essential III: Scholarship for evidence-based practice | NLN Competencies: Human flourishing | Nursing/Integrated Concepts: Nursing Process: Implementation/Health teaching and health promotion Learning Outcome: LO 6.7 Plan health promotion and health maintenance strategies employed during health supervision visits. MNL LO: Compare the developmental stages for pediatric clients. 9) Which nursing assessment activities should be included for the child and family at each health-supervision visit? Select all that apply. 1. Interview to obtain an updated health history 2. Performing an age-appropriate development assessment 3. Monitoring parents' ability to pay for services 4. Performing age-appropriate screening examinations 5. Physical assessment for genetic abnormalities Answer: 1, 2, 4 Explanation: 1. The interview, the developmental assessment, and age-appropriate screenings are all included in the nursing assessment of a child and family during each health-supervision visit. A nurse would not assess the parents' financial status at each health-supervision visit. Physical assessments for genetic abnormalities would be done based on history and physical findings, not at each routine visit. 2. The interview, the developmental assessment, and age-appropriate screenings are all included in the nursing assessment of a child and family during each health-supervision visit. A nurse would not assess the parents' financial status at each health-supervision visit. Physical assessments for genetic abnormalities would be done based on history and physical findings, not at each routine visit. 3. The interview, the developmental assessment, and age-appropriate screenings are all included in the nursing assessment of a child and family during each health-supervision visit. A nurse would not assess the parents' financial status at each health-supervision visit. Physical assessments for genetic abnormalities would be done based on history and physical findings, not at each routine visit. 4. The interview, the developmental assessment, and age-appropriate screenings are all included in the nursing assessment of a child and family during each health-supervision visit. A nurse would not assess the parents' financial status at each health-supervision visit. Physical assessments for genetic abnormalities would be done based on history and physical findings, not at each routine visit. 5. The interview, the developmental assessment, and age-appropriate screenings are all included in the nursing assessment of a child and family during each health-supervision visit. A nurse would not assess the parents' financial status at each health-supervision visit. Physical assessments for genetic abnormalities would be done based on history and physical findings, not at each routine visit. Page Ref: 150-151 Cognitive Level: Applying Client Need &Sub: Health Promotion and Maintenance Standards: QSEN Competencies: Patient-centered care | AACN Essential Competencies: Essential VII: Clinical prevention and population health | NLN Competencies: Human flourishing | Nursing/Integrated Concepts: Nursing Process: Implementation/Coordination of care Learning Outcome: LO 6.3 Describe the components of a health supervision visit. MNL LO: Examine the role of the nurse in promoting culturally competent family-centered care. 10) The nurse of an outpatient clinic is sitting with the parents while their adolescent goes for a test. The parents are complaining about their child's behavior. Which statement by the nurse fosters family-centered communication? 1. "I agree with you; discipline is an important part of parenting." 2. "I know just how you feel. I had the same experience with my children." 3. "You are so right. Adolescents function in the "me-first" mode all the time." 4. "Tell me what concerns you about your child's behavior." Answer: 4 Explanation: 1. Using an open-ended question allows the parents to discuss a family concern. All the other questions or statements are blocking statements and would not foster family- centered communication. 2. Using an open-ended question allows the parents to discuss a family concern. All the other questions or statements are blocking statements and would not foster family-centered communication. 3. Using an open-ended question allows the parents to discuss a family concern. All the other questions or statements are blocking statements and would not foster family-centered communication. 4. Using an open-ended question allows the parents to discuss a family concern. All the other questions or statements are blocking statements and would not foster family-centered communication. Page Ref: 151 Cognitive Level: Applying Client Need &Sub: Health Promotion and Maintenance Standards: QSEN Competencies: Patient-centered care | AACN Essential Competencies: Essential VI: Interprofessional communication and collaboration for improving patient health outcomes | NLN Competencies: Nursing judgement | Nursing/Integrated Concepts: Nursing Process: Implementation/Communication Learning Outcome: LO 6.2 Describe how health promotion and health maintenance are addressed by partnering with families during health supervision visits. MNL LO: Implement developmentally appropriate communication strategies for pediatric clients and families. 11) The nurse is assessing an adolescent client whose weight is in the 5th percentile. Based on this information, which question is most appropriate for the nurse to ask the adolescent client? 1. "Do you eat the school lunches?" 2. "Do you have any concerns about your weight?" 3. "Do you eat fruits, vegetables, and drink milk?" 4. "How many meals do you eat each day?" Answer: 2 Explanation: 1. The only question that addresses the adolescent's weight, which is below the expected norm, is "Do you have any concerns about your weight?" Asking about school lunches, eating fruits and vegetables, and how many meals eaten each day should be used to obtain a nutritional history; however, those questions do not address the underweight status of the adolescent. 2. The only question that addresses the adolescent's weight, which is below the expected norm, is "Do you have any concerns about your weight?" Asking about school lunches, eating fruits and vegetables, and how many meals eaten each day should be used to obtain a nutritional history; however, those questions do not address the underweight status of the adolescent. 3. The only question that addresses the adolescent's weight, which is below the expected norm, is "Do you have any concerns about your weight?" Asking about school lunches, eating fruits and vegetables, and how many meals eaten each day should be used to obtain a nutritional history; however, those questions do not address the underweight status of the adolescent. 4. The only question that addresses the adolescent's weight, which is below the expected norm, is "Do you have any concerns about your weight?" Asking about school lunches, eating fruits and vegetables, and how many meals eaten each day should be used to obtain a nutritional history; however, those questions do not address the underweight status of the adolescent. Page Ref: 151 Cognitive Level: Analyzing Client Need &Sub: Health Promotion and Maintenance Standards: QSEN Competencies: Patient-centered care | AACN Essential Competencies: Essential VI: Interprofessional communication and collaboration for improving patient health outcomes | NLN Competencies: Nursing judgement | Nursing/Integrated Concepts: Nursing Process: Assessment/Coordination of care Learning Outcome: LO 6.8 Apply the nursing process in assessment, diagnosis, goal setting, intervention, and evaluation of health promotion and health maintenance activities for children and families. MNL LO: Analyze socioeconomic factors that impact childhood nutrition. 12) In the pediatric well-child clinic, the nurse explains the reason for an immunization series to the child's mother. This action represents which item? 1. Health assessment 2. Health promotion 3. Health maintenance 4. Health screening Answer: 2 Explanation: 1. The explanation to the mother by the nurse provides an understanding of the immunization series to the mother and enables the mother to make an intelligent choice. While administering immunizations is considered health maintenance, the activity described in the question is clearly health promotion. A health assessment would be completed to determine what immunizations are needed. Health maintenance is the actual administration of the immunization and health screening involves looking at the immunization record to determine which immunizations are needed. 2. The explanation to the mother by the nurse provides an understanding of the immunization series to the mother and enables the mother to make an intelligent choice. While administering immunizations is considered health maintenance, the activity described in the question is clearly health promotion. A health assessment would be completed to determine what immunizations are needed. Health maintenance is the actual administration of the immunization and health screening involves looking at the immunization record to determine which immunizations are needed. 3. The explanation to the mother by the nurse provides an understanding of the immunization series to the mother and enables the mother to make an intelligent choice. While administering immunizations is considered health maintenance, the activity described in the question is clearly health promotion. A health assessment would be completed to determine what immunizations are needed. Health maintenance is the actual administration of the immunization and health screening involves looking at the immunization record to determine which immunizations are needed. 4. The explanation to the mother by the nurse provides an understanding of the immunization series to the mother and enables the mother to make an intelligent choice. While administering immunizations is considered health maintenance, the activity described in the question is clearly health promotion. A health assessment would be completed to determine what immunizations are needed. Health maintenance is the actual administration of the immunization and health screening involves looking at the immunization record to determine which immunizations are needed. Page Ref: 146 Cognitive Level: Applying Client Need &Sub: Health Promotion and Maintenance Standards: QSEN Competencies: Patient-centered care | AACN Essential Competencies: Essential VII: Clinical prevention and population health | NLN Competencies: Human flourishing | Nursing/Integrated Concepts: Nursing Process: Implementation/Health teaching and health promotion Learning Outcome: LO 6.2 Describe how health promotion and health maintenance are addressed by partnering with families during health supervision visits. MNL LO: Examine the role of the nurse in promoting culturally competent family-centered care. 13) A pediatric nurse who is employed in a busy ambulatory clinic setting is informed by the nurse manager that average nursing time allocated for each child and family is being reduced to 10 minutes to more efficiently manage the clinic. The nursing activities must include a nursing assessment and discussion on anticipatory guidance. Which of these strategies should the nurse utilize in the plan of care delivery? 1. Attempt to complete the assessment and education in 10 minutes, but extend the time whenever the nurse deems necessary. 2. Plan to do the anticipatory guidance first since either the nurse practitioner or the physician can perform the assessment of the child. 3. Encourage the parent to ask for specific time to talk with the nurse privately at each office visit. 4. Focus anticipatory guidance strategies on topics that the parent or child have expressed as an area of interest. Answer: 4 Explanation: 1. With limited time for each visit, the nurse should focus on anticipatory guidance strategies that will most benefit the parent and child during that office visit. 2. With limited time for each visit, the nurse should focus on anticipatory guidance strategies that will most benefit the parent and child during that office visit. 3. With limited time for each visit, the nurse should focus on anticipatory guidance strategies that will most benefit the parent and child during that office visit. 4. With limited time for each visit, the nurse should focus on anticipatory guidance strategies that will most benefit the parent and child during that office visit. Page Ref: 148 Cognitive Level: Applying Client Need &Sub: Safe and Effective Care Environment Standards: QSEN Competencies: Patient-centered care | AACN Essential Competencies: Essential IX: Baccalaureate generalist nursing practice | NLN Competencies: Nursing judgement | Nursing/Integrated Concepts: Nursing Process: Implementation/Health teaching and health promotion Learning Outcome: LO 6.4 Analyze the nurse's role in providing health promotion and health maintenance for children and families. MNL LO: Explore the practice of pediatric healthcare. 14) Which assessment would not be included with a 17-year-old's screening during a routine health supervision visit? 1. STI evaluation 2. Autism screening 3. Hemoglobin test 4. Vision screening Answer: 2 Explanation: 1. Autism screening would not be appropriate at this age. If autism were present, it would have presented before this age. STI evaluation, hemoglobin test, and vision screening are all appropriate for a 17-year-old. 2. Autism screening would not be appropriate at this age. If autism were present, it would have presented before this age. STI evaluation, hemoglobin test, and vision screening are all appropriate for a 17-year-old. 3. Autism screening would not be appropriate at this age. If autism were present, it would have presented before this age. STI evaluation, hemoglobin test, and vision screening are all appropriate for a 17-year-old. 4. Autism screening would not be appropriate at this age. If autism were present, it would have presented before this age. STI evaluation, hemoglobin test, and vision screening are all appropriate for a 17-year-old. Page Ref: 151 Cognitive Level: Analyzing Client Need &Sub: Health Promotion and Maintenance Standards: QSEN Competencies: Patient-centered care | AACN Essential Competencies: Essential IX: Baccalaureate generalist nursing practice | NLN Competencies: Human flourishing | Nursing/Integrated Concepts: Nursing Process: Implementation/Coordination of care Learning Outcome: LO 6.3 Describe the components of a health supervision visit. MNL LO: Use the nursing process to provide developmentally appropriate care for the pediatric client. 15) A nurse is discussing health promotion activities with parents of a 4-year-old client. What health-promotion activity is most appropriate for this family? 1. Make arrangements to tour the kindergarten in which the child will enroll next year. 2. Plan a "movie afternoon" with the child's big brother. 3. Maintain appropriate immunizations. 4. Teach the child the proper method for brushing the teeth. Answer: 4 Explanation: 1. Teaching proper oral hygiene through proper teeth brushing is a health- promotion activity. Touring the kindergarten might alleviate anxiety, but is not health promotion. A movie afternoon with the big brother is sedentary, and also not a health-promotion activity. Maintaining immunizations is a health-maintenance, not health-promotion, activity. 2. Teaching proper oral hygiene through proper teeth brushing is a health-promotion activity. Touring the kindergarten might alleviate anxiety, but is not health promotion. A movie afternoon with the big brother is sedentary, and also not a health-promotion activity. Maintaining immunizations is a health-maintenance, not health-promotion, activity. 3. Teaching proper oral hygiene through proper teeth brushing is a health-promotion activity. Touring the kindergarten might alleviate anxiety, but is not health promotion. A movie afternoon with the big brother is sedentary, and also not a health-promotion activity. Maintaining immunizations is a health-maintenance, not health-promotion, activity. 4. Teaching proper oral hygiene through proper teeth brushing is a health-promotion activity. Touring the kindergarten might alleviate anxiety, but is not health promotion. A movie afternoon with the big brother is sedentary, and also not a health-promotion activity. Maintaining immunizations is a health-maintenance, not health-promotion, activity. Page Ref: 149 Cognitive Level: Applying Client Need &Sub: Health Promotion and Maintenance Standards: QSEN Competencies: Patient-centered care | AACN Essential Competencies: Essential VII: Clinical prevention and population health | NLN Competencies: Human flourishing | Nursing/Integrated Concepts: Nursing Process: Implementation/Health teaching and health promotion Learning Outcome: LO 6.6 Synthesize the areas of assessment and intervention for health supervision visits–growth and developmental surveillance, nutrition, physical activity, oral health, mental and spiritual health, family and social relations, disease prevention strategies, and injury prevention strategies. MNL LO: Examine the role of the nurse in promoting culturally competent family-centered care. 16) The nurse educator is teaching a group of students about the key concepts of a medical home during the developmental years of the pediatric client. Which items should the educator include in the teaching session? Select all that apply. 1. Financial accessibility 2. Consistent, ongoing care 3. Coordination of care 4. No individualization of care 5. A paternalistic view of care Answer: 1, 2, 3 Explanation: 1. All children need a medical home, where accessible, continuous, and coordinated health supervision is provided during the developmental years. Accessibility refers to both financial and geographic access; continuous indicates that the care is ongoing with consistent care providers; coordination refers to the need for communication among health professionals to provide for the needs of the child. Care is individualized and is not paternalistic. 2. All children need a medical home, where accessible, continuous, and coordinated health supervision is provided during the developmental years. Accessibility refers to both financial and geographic access; continuous indicates that the care is ongoing with consistent care providers; coordination refers to the need for communication among health professionals to provide for the needs of the child. Care is individualized and is not paternalistic. 3. All children need a medical home, where accessible, continuous, and coordinated health supervision is provided during the developmental years. Accessibility refers to both financial and geographic access; continuous indicates that the care is ongoing with consistent care providers; coordination refers to the need for communication among health professionals to provide for the needs of the child. Care is individualized and is not paternalistic. 4. All children need a medical home, where accessible, continuous, and coordinated health supervision is provided during the developmental years. Accessibility refers to both financial and geographic access; continuous indicates that the care is ongoing with consistent care providers; coordination refers to the need for communication among health professionals to provide for the needs of the child. Care is individualized and is not paternalistic. 5. All children need a medical home, where accessible, continuous, and coordinated health supervision is provided during the developmental years. Accessibility refers to both financial and geographic access; continuous indicates that the care is ongoing with consistent care providers; coordination refers to the need for communication among health professionals to provide for the needs of the child. Care is individualized and is not paternalistic. Page Ref: 147 Cognitive Level: Applying Client Need &Sub: Health Promotion and Maintenance Standards: QSEN Competencies: Patient-centered care | AACN Essential Competencies: Essential IX: Baccalaureate generalist nursing practice | NLN Competencies: Nursing judgement | Nursing/Integrated Concepts: Nursing Process: Planning/Health teaching and health promotion Learning Outcome: LO 6.5 Perform the general observations made of children and their families as they come to the pediatric healthcare home for health supervision visits. MNL LO: Examine the role of the nurse in promoting culturally competent family-centered care. 17) The nurse is preparing to perform a hearing screening on a 6-year-old child. The nurse knows this screening is what level of prevention? 1. Primary prevention 2. Secondary prevention 3. Tertiary prevention 4. Quaternary prevention Answer: 2 Explanation: 1. Primary prevention includes immunizations, teaching regarding seatbelts, helmets, and so on. 2. Secondary prevention includes developmental, hearing and vision screenings. 3. Tertiary prevention includes rehab, PT, OT, and so on. 4. Quaternary prevention includes advanced levels of medicine, extensive tests. Page Ref: 146 Cognitive Level: Analyzing Client Need &Sub: Safe and Effective Care Environment: Management of Care Standards: QSEN Competencies: Patient-centered care | AACN Essential Competencies: Essential VII: Clinical prevention and population health | NLN Competencies: Professional identity | Nursing/Integrated Concepts: Nursing Process: Planning/Coordination of care Learning Outcome: LO 6.6 Synthesize the areas of assessment and intervention for health supervision visits–growth and developmental surveillance, nutrition, physical activity, oral health, mental and spiritual health, family and social relations, disease prevention strategies, and injury prevention strategies. MNL LO: Evaluate healthcare issues related to pediatric nursing care. 18) Match the Development surveillance questionnaire with its description. A. Denver II B. Ages and stages questionnaire C. Child development inventory D. Parents evaluation of developmental status E. Prescreening developmental questionnaire 1. Questionnaire of specific ages, 10 to 15 items in each area: fine motor, gross motor, communication, adaptive, personal, and social skills. 2. Consists of 10 questions for parent to answer in interview, based on research regarding parents' concerns. 3. Consists of 60, yes-no descriptions for three separate instruments to identify child with developmental difficulties. 4. Consists of observation of child in 4 domains; personal, social, fine-motor-adaptive, language, and gross motor. 5. Helps identify children who need Denver II assessment. Answer: 1/B, 2/D, 3/C, 4/A, 5/E 1. Ages and stages questionnaire 2. Parents evaluation of developmental status 3. Child development inventory 4. Denver II 5. Prescreening developmental questionnaire Explanation: Ages and stages questionnaire: Questionnaire of specific ages, 10 to 15 items in each area: fine motor, gross motor, communication, adaptive, personal, and social skills. Parents evaluation of developmental status: Consists of 10 questions for parent to answer in interview, based on research regarding parents' concerns. Child development inventory: Consists of 60, yes-no descriptions for three separate instruments to identify child with developmental difficulties. Denver II: Consists of observation of child in four domains; personal, social, fine-motor- adaptive, language, and gross motor. Prescreening developmental questionnaire: Helps identify children who need Denver II assessment. These are among many developmental questionnaires available for use with children. Page Ref: 149, 150 Cognitive Level: Analyzing Client Need &Sub: Safe and Effective Care Environment: Management of Care Standards: QSEN Competencies: Patient-centered care | AACN Essential Competencies: Essential VII: Clinical prevention and population health | NLN Competencies: Human flourishing | Nursing/Integrated Concepts: Nursing Process: Evaluation/Coordination of care Learning Outcome: LO 6.6 Synthesize the areas of assessment and intervention for health supervision visits–growth and developmental surveillance, nutrition, physical activity, oral health, mental and spiritual health, family and social relations, disease prevention strategies, and injury prevention strategies. MNL LO: Compare the developmental stages for pediatric clients. Principles of Pediatric Nursing: Caring for Children, 7e (Ball et al.) Chapter 7 Health Promotion and Maintenance for the Newborn and Infant 1) A nursery nurse is planning care for the newborns currently in the newborn nursery. Which activities does the nurse plan for the first 48 hours of life? Select all that apply. 1. Monitor feeding behaviors 2. Perform a hearing screening 3. Perform a heel stick to obtain blood for the newborn screen 4. Monitor the mother as she performs the first newborn bath to remove blood and amniotic fluids 5. Administer folic-acid injection to the infant to prevent bleeding Answer: 1, 2, 3 Explanation: 1. The nurse should assess feeding behaviors of the infant whether the infant is breastfed or bottle-fed. A hearing screening is performed on all newborn infants prior to discharge. The newborn screen is performed prior to infant discharge from the newborn unit. The nurse, not the mother, performs the first bath to remove blood and amniotic fluids. Vitamin K is administered, not folic acid. 2. The nurse should assess feeding behaviors of the infant whether the infant is breastfed or bottle-fed. A hearing screening is performed on all newborn infants prior to discharge. The newborn screen is performed prior to infant discharge from the newborn unit. The nurse, not the mother, performs the first bath to remove blood and amniotic fluids. Vitamin K is administered, not folic acid. 3. The nurse should assess feeding behaviors of the infant whether the infant is breastfed or bottle-fed. A hearing screening is performed on all newborn infants prior to discharge. The newborn screen is performed prior to infant discharge from the newborn unit. The nurse, not the mother, performs the first bath to remove blood and amniotic fluids. Vitamin K is administered, not folic acid. 4. The nurse should assess feeding behaviors of the infant whether the infant is breastfed or bottle-fed. A hearing screening is performed on all newborn infants prior to discharge. The newborn screen is performed prior to infant discharge from the newborn unit. The nurse, not the mother, performs the first bath to remove blood and amniotic fluids. Vitamin K is administered, not folic acid. 5. The nurse should assess feeding behaviors of the infant whether the infant is breastfed or bottle-fed. A hearing screening is performed on all newborn infants prior to discharge. The newborn screen is performed prior to infant discharge from the newborn unit. The nurse, not the mother, performs the first bath to remove blood and amniotic fluids. Vitamin K is administered, not folic acid. Page Ref: 156 Cognitive Level: Applying Client Need &Sub: Health Promotion and Maintenance Standards: QSEN Competencies: Evidence-based practice | AACN Essential Competencies: Essential III: Scholarship for evidence-based practice | NLN Competencies: Nursing judgement | Nursing/Integrated Concepts: Nursing Process: Implementation/Coordination of care Learning Outcome: LO 7.1 Synthesize the areas of assessment and intervention for health supervision visits of newborns and infants: growth and developmental surveillance, nutrition, physical activity, oral health, mental and spiritual health, family and social relations, disease prevention strategies, and injury prevention strategies. MNL LO: Use the nursing process to provide developmentally appropriate care for the pediatric client. 2) The nurse is planning care for clients seen in a newborn clinic. Which is the priority for a newborn client during the first clinic visit? 1. Providing pamphlets to reinforce information provided at the visit 2. Assessing the newborn-family interactions 3. Modeling infant-nurturing behaviors 4. Informing the parents of the infant's gains in height and weight Answer: 2 Explanation: 1. The first step in the nursing process is assessment; therefore, the nurse should assess the interactions of the parents with the newborn. Providing pamphlets to help educate the parents should be done at each appropriate office visit; however, the pamphlets would be distributed after assessment of parent needs. While the nurse should be a role model for nurturing behaviors during the office visit, this would not be the first thing the nurse performs at the office visit. While parents are informed of the infant's gains in height and weight, this activity does not take priority. 2. The first step in the nursing process is assessment; therefore, the nurse should assess the interactions of the parents with the newborn. Providing pamphlets to help educate the parents should be done at each appropriate office visit; however, the pamphlets would be distributed after assessment of parent needs. While the nurse should be a role model for nurturing behaviors during the office visit, this would not be the first thing the nurse performs at the office visit. While parents are informed of the infant's gains in height and weight, this activity does not take priority. 3. The first step in the nursing process is assessment; therefore, the nurse should assess the interactions of the parents with the newborn. Providing pamphlets to help educate the parents should be done at each appropriate office visit; however, the pamphlets would be distributed after assessment of parent needs. While the nurse should be a role model for nurturing behaviors during the office visit, this would not be the first thing the nurse performs at the office visit. While parents are informed of the infant's gains in height and weight, this activity does not take priority. 4. The first step in the nursing process is assessment; therefore, the nurse should assess the interactions of the parents with the newborn. Providing pamphlets to help educate the parents should be done at each appropriate office visit; however, the pamphlets would be distributed after assessment of parent needs. While the nurse should be a role model for nurturing behaviors during the office visit, this would not be the first thing the nurse performs at the office visit. While parents are informed of the infant's gains in height and weight, this activity does not take priority. Page Ref: 156 Cognitive Level: Analyzing Client Need &Sub: Health Promotion and Maintenance Standards: QSEN Competencies: Patient-centered care | AACN Essential Competencies: Essential III: Scholarship for evidence-based practice | NLN Competencies: Human flourishing | Nursing/Integrated Concepts: Nursing Process: Implementation/Coordination of care Learning Outcome: LO 7.2 Plan health promotion and health maintenance strategies employed during health supervision visits of newborns and infants. MNL LO: Apply key concepts of family-centered care. 3) The nurse in the newborn nursery is admitting a neonate. To determine the health and development of the newborn, what will the nurse include in the assessment? Select all that apply. 1. Head circumference 2. Body length 3. Weight 4. Length of pregnancy 5. Hearing screens Answer: 1, 2, 3, 4 Explanation: 1. The nurse should assess almost all of these parameters to determine the health of the newborn. However, hearing screens are typically done after the first 12 hours after birth and are not part of newborn assessment. 2. The nurse should assess almost all of these parameters to determine the health of the newborn. However, hearing screens are typically done after the first 12 hours after birth and are not part of newborn assessment. 3. The nurse should assess almost all of these parameters to determine the health of the newborn. However, hearing screens are typically done after the first 12 hours after birth and are not part of newborn assessment. 4. The nurse should assess almost all of these parameters to determine the health of the newborn. However, hearing screens are typically done after the first 12 hours after birth and are not part of newborn assessment. 5. The nurse should assess almost all of these parameters to determine the health of the newborn. However, hearing screens are typically done after the first 12 hours after birth and are not part of newborn assessment. Page Ref: 156-157 Cognitive Level: Applying Client Need &Sub: Health Promotion and Maintenance Standards: QSEN Competencies: Patient-centered care | AACN Essential Competencies: Essential IX: Baccalaureate generalist nursing practice | NLN Competencies: Human flourishing | Nursing/Integrated Concepts: Nursing Process: Assessment/Coordination of care Learning Outcome: LO 7.1 Synthesize the areas of assessment and intervention for health supervision visits of newborns and infants: growth and developmental surveillance, nutrition, physical activity, oral health, mental and spiritual health, family and social relations, disease prevention strategies, and injury prevention strategies. MNL LO: Differentiate developmentally appropriate care environments for the pediatric client and family. 4) An infant weighs 9 pounds 3 ounces at birth. The nurse plans to make a home visit to the mother and infant when the infant is 7 days old. What is the lowest acceptable weight the infant should be at this age? 1. 7 pounds 12 ounces 2. 8 pounds 2 ounces 3. 8 pounds 12 ounces 4. 9 pounds Answer: 2 Explanation: 1. In the first week of life, most infants lose about one-tenth of their birth weight; therefore, this infant's weight should be 8 pounds 2 ounces at 7 days of age. A weight loss to 7 pounds 12 ounces would be too much for this infant. A decline to 8 pounds 12 ounces is less than the expected one-tenth weight loss after birth, and an infant would not be expected to lose only 3 ounces during the first week of life. 2. In the first week of life, most infants lose about one-tenth of their birth weight; therefore, this infant's weight should be 8 pounds 2 ounces at 7 days of age. A weight loss to 7 pounds 12 ounces would be too much for this infant. A decline to 8 pounds 12 ounces is less than the expected one-tenth weight loss after birth, and an infant would not be expected to lose only 3 ounces during the first week of life. 3. In the first week of life, most infants lose about one-tenth of their birth weight; therefore, this infant's weight should be 8 pounds 2 ounces at 7 days of age. A weight loss to 7 pounds 12 ounces would be too much for this infant. A decline to 8 pounds 12 ounces is less than the expected one-tenth weight loss after birth, and an infant would not be expected to lose only 3 ounces during the first week of life. 4. In the first week of life, most infants lose about one-tenth of their birth weight; therefore, this infant's weight should be 8 pounds 2 ounces at 7 days of age. A weight loss to 7 pounds 12 ounces would be too much for this infant. A decline to 8 pounds 12 ounces is less than the expected one-tenth weight loss after birth, and an infant would not be expected to lose only 3 ounces during the first week of life. Page Ref: 157 Cognitive Level: Applying Client Need &Sub: Health Promotion and Maintenance Standards: QSEN Competencies: Patient-centered care | AACN Essential Competencies: Essential III: Scholarship for evidence-based practice | NLN Competencies: Human flourishing | Nursing/Integrated Concepts: Nursing Process: Assessment/Coordination of care Learning Outcome: LO 7.1 Synthesize the areas of assessment and intervention for health supervision visits of newborns and infants: growth and developmental surveillance, nutrition, physical activity, oral health, mental and spiritual health, family and social relations, disease prevention strategies, and injury prevention strategies. MNL LO: Differentiate developmentally appropriate care environments for the pediatric client and family. 5) The nurse is teaching a new mother developmental expectations. Which activity should the nurse expect a newborn to do within the first month of life? 1. Bring hands to eyes and mouth 2. Push up with hands, moving chest up 3. Keep hands in a relaxed position 4. Roll over from back to abdomen Answer: 1 Explanation: 1. Newborns at one month of age can bring hands to their eyes and mouths, move their heads from side to side when lying on their abdomens, and attempt to lift their heads only when prone. Newborn hands are kept in tight fist position, and the newborn cannot roll over until 4 months of age. 2. Newborns at one month of age can bring hands to their eyes and mouths, move their heads from side to side when lying on their abdomens, and attempt to lift their heads only when prone. Newborn hands are kept in tight fist position, and the newborn cannot roll over until 4 months of age. 3. Newborns at one month of age can bring hands to their eyes and mouths, move their heads from side to side when lying on their abdomens, and attempt to lift their heads only when prone. Newborn hands are kept in tight fist position, and the newborn cannot roll over until 4 months of age. 4. Newborns at one month of age can bring hands to their eyes and mouths, move their heads from side to side when lying on their abdomens, and attempt to lift their heads only when prone. Newborn hands are kept in tight fist position, and the newborn cannot roll over until 4 months of age. Page Ref: 157 Cognitive Level: Analyzing Client Need &Sub: Health Promotion and Maintenance Standards: QSEN Competencies: Patient-centered care | AACN Essential Competencies: Essential I: Liberal education for Baccalaureate generalist nursing practice | NLN Competencies: Human flourishing | Nursing/Integrated Concepts: Nursing Process: Assessment/Health teaching and health promotion Learning Outcome: LO 7.1 Synthesize the areas of assessment and intervention for health supervision visits of newborns and infants: growth and developmental surveillance, nutrition, physical activity, oral health, mental and spiritual health, family and social relations, disease prevention strategies, and injury prevention strategies. MNL LO: Use the nursing process to provide developmentally appropriate care for the pediatric client. 6) The nurse is providing anticipatory guidance instructions to the parents of a newborn. Which instruction should the nurse give as a strategy for illness/disease prevention? 1. Don't allow visitors for the first month 2. Smoke outside only 3. Take the newborn to weekly child-stimulation classes 4. SIDS risk-reduction measures Answer: 4 Explanation: 1. Several disease-prevention strategies are used during anticipatory guidance for the parents of newborns. Not allowing visitors is unreasonable but screening for illness is appropriate. Smoking outside will not prevent disease. Attending weekly stimulation classes is not a disease prevention strategy. SIDS risk-reduction measures can reduce the risk of sudden infant death syndrome. 2. Several disease-prevention strategies are used during anticipatory guidance for the parents of newborns. Not allowing visitors is unreasonable but screening for illness is appropriate. Smoking outside will not prevent disease. Attending weekly stimulation classes is not a disease prevention strategy. SIDS risk-reduction measures can reduce the risk of sudden infant death syndrome. 3. Several disease-prevention strategies are used during anticipatory guidance for the parents of newborns. Not allowing visitors is unreasonable but screening for illness is appropriate. Smoking outside will not prevent disease. Attending weekly stimulation classes is not a disease prevention strategy. SIDS risk-reduction measures can reduce the risk of sudden infant death syndrome. 4. Several disease-prevention strategies are used during anticipatory guidance for the parents of newborns. Not allowing visitors is unreasonable but screening for illness is appropriate. Smoking outside will not prevent disease. Attending weekly stimulation classes is not a disease prevention strategy. SIDS risk-reduction measures can reduce the risk of sudden infant death syndrome. Page Ref: 157 Cognitive Level: Applying Client Need &Sub: Health Promotion and Maintenance Standards: QSEN Competencies: Patient-centered care | AACN Essential Competencies: Essential VII: Clinical prevention and population health | NLN Competencies: Nursing judgment | Nursing/Integrated Concepts: Nursing Process: Implementation/Health teaching and health promotion Learning Outcome: LO 7.3 Recognize the importance of family in newborn and infant health care, and include family assessment in each health supervision visit. MNL LO: Use the nursing process to provide developmentally appropriate care for the pediatric client. 7) A nurse assesses the height and weight measurements on an infant and documents these measurements at the 75th percentile. The nurse notes that the previous measurements two months ago were at the 25th percentile. Which interpretation by the nurse is the most accurate? 1. The infant is not gaining enough weight. 2. The infant has gained a significant amount of weight. 3. The previous measurements were most likely inaccurate. 4. These measurements are most likely inaccurate. Answer: 2 Explanation: 1. A comparison of these two sets of measurements shows that the infant has crossed two percentiles going from the 25th to the 75th percentile and therefore has gained a significant amount of weight. There is neither indication that the previous measurements are inaccurate nor that the current measurement is inaccurate. 2. A comparison of these two sets of measurements shows that the infant has crossed two percentiles going from the 25th to the 75th percentile and therefore has gained a significant amount of weight. There is neither indication that the previous measurements are inaccurate nor that the current measurement is inaccurate. 3. A comparison of these two sets of measurements shows that the infant has crossed two percentiles going from the 25th to the 75th percentile and therefore has gained a significant amount of weight. There is neither indication that the previous measurements are inaccurate nor that the current measurement is inaccurate. 4. A comparison of these two sets of measurements shows that the infant has crossed two percentiles going from the 25th to the 75th percentile and therefore has gained a significant amount of weight. There is neither indication that the previous measurements are inaccurate nor that the current measurement is inaccurate. Page Ref: 157 Cognitive Level: Applying Client Need &Sub: Health Promotion and Maintenance Standards: QSEN Competencies: Patient-centered care | AACN Essential Competencies: Essential I: Liberal education for Baccalaureate generalist nursing practice | NLN Competencies: Nursing judgement | Nursing/Integrated Concepts: Nursing Process: Evaluation/Coordination of care Learning Outcome: LO 7.1 Synthesize the areas of assessment and intervention for health supervision visits of newborns and infants: growth and developmental surveillance, nutrition, physical activity, oral health, mental and spiritual health, family and social relations, disease prevention strategies, and injury prevention strategies. MNL LO: Compare the developmental stages for pediatric clients. 8) A nurse asks the mother of a 4-month-old infant to undress the infant. The nurse observes the mother taking off several layers of clothing and knows that the outdoor temperature is 70 degrees Fahrenheit. Which statement by the nurse is most appropriate in this situation? 1. "My, you are dressing your infant warmly today." 2. "Did you think it was cold when you left your home this morning?" 3. "I see that you have many layers of clothing on your baby. This may cause your baby's temperature to rise." 4. "When you leave the office, only put one layer of clothing on your baby." Answer: 3 Explanation: 1. In this scenario, the mother has overdressed the infant. The nurse needs to gently inform the mother of this problem and to provide information to the mother on why it is a problem. Just making a statement on how warmly the child is dressed will not accomplish this goal or just telling the mother to only put one layer of clothing on the child does not provide a rationale for the mother to make a better decision the next time, so this statement also is not helpful to the mother. 2. In this scenario, the mother has overdressed the infant. The nurse needs to gently inform the mother of this problem and to provide information to the mother on why it is a problem. Just making a statement on how warmly the child is dressed will not accomplish this goal or just telling the mother to only put one layer of clothing on the child does not provide a rationale for the mother to make a better decision the next time, so this statement also is not helpful to the mother. 3. In this scenario, the mother has overdressed the infant. The nurse needs to gently inform the mother of this problem and to provide information to the mother on why it is a problem. Just making a statement on how warmly the child is dressed will not accomplish this goal or just telling the mother to only put one layer of clothing on the child does not provide a rationale for the mother to make a better decision the next time, so this statement also is not helpful to the mother. 4. In this scenario, the mother has overdressed the infant. The nurse needs to gently inform the mother of this problem and to provide information to the mother on why it is a problem. Just making a statement on how warmly the child is dressed will not accomplish this goal or just telling the mother to only put one layer of clothing on the child does not provide a rationale for the mother to make a better decision the next time, so this statement also is not helpful to the mother. Page Ref: 157 Cognitive Level: Applying Client Need &Sub: Health Promotion and Maintenance Standards: QSEN Competencies: Patient-centered care | AACN Essential Competencies: Essential IX: Baccalaureate generalist nursing practice | NLN: Professional identity | Nursing/Integrated Concepts: Nursing Process: Implementation/Health teaching and health promotion Learning Outcome: LO 7.1 Synthesize the areas of assessment and intervention for health supervision visits of newborns and infants: growth and developmental surveillance, nutrition, physical activity, oral health, mental and spiritual health, family and social relations, disease prevention strategies, and injury prevention strategies. MNL LO: Develop a family-centered nursing care plan for the child and family. 9) The nurse working with a family has observed that the older children have a large number of dental caries and plans to provide the mother with information to prevent the development of dental caries in her new infant. Which interventions will prevent the development of dental caries in the infant? Select all that apply. 1. Avoiding nursing or giving the infant a bottle at bedtime 2. Giving foods high in sugar only at breakfast time 3. Using a soft moist gauze for cleaning 4. Using a topical anesthetic daily beginning as soon as the first tooth begins to erupt Answer: 1, 3 Explanation: 1. The only interventions that will assist in the prevention of dental caries listed in this question are wiping the gums with a soft, moist gauze and avoiding putting the infant to bed with a bottle. Foods high in sugar should be avoided in the infant period. Topical anesthetic should not be applied daily. 2. The only interventions that will assist in the prevention of dental caries listed in this question are wiping the gums with a soft, moist gauze and avoiding putting the infant to bed with a bottle. Foods high in sugar should be avoided in the infant period. Topical anesthetic should not be applied daily. 3. The only interventions that will assist in the prevention of dental caries listed in this question are wiping the gums with a soft, moist gauze and avoiding putting the infant to bed with a bottle. Foods high in sugar should be avoided in the infant period. Topical anesthetic should not be applied daily. 4. The only interventions that will assist in the prevention of dental caries listed in this question are wiping the gums with a soft, moist gauze and avoiding putting the infant to bed with a bottle. Foods high in sugar should be avoided in the infant period. Topical anesthetic should not be applied daily. Page Ref: 159 Cognitive Level: Applying Client Need &Sub: Health Promotion and Maintenance Standards: QSEN Competencies: Patient-centered care | AACN Essential Competencies: Essential VII: Clinical prevention and population health | NLN Competencies: Professional identity | Nursing/Integrated Concepts: Nursing Process: Implementation/Health teaching and health promotion Learning Outcome: LO 7.1 Synthesize the areas of assessment and intervention for health supervision visits of newborns and infants: growth and developmental surveillance, nutrition, physical activity, oral health, mental and spiritual health, family and social relations, disease prevention strategies, and injury prevention strategies. MNL LO: Develop a family-centered nursing care plan for the child and family. 10) A nurse is assessing an 11-month-old infant and notes that the infant's height and weight are at the 5th percentile on the growth chart. Family history reveals that the infant's two siblings are at the 50th percentile for height and at the 75th percentile for weight. Psychosocial history reveals that the parents are separated and are planning to divorce. Which of these nursing diagnoses takes priority? 1. Alteration in Growth Pattern Related to Parental Anxiety 2. Alteration in Growth Pattern Secondary to Familial Short Stature 3. Nutritional Intake: Excessive Secondary to Maternal Feeding Patterns 4. At Risk for Constitutional Growth Delay Related to Decreased Appetite Answer: 1 Explanation: 1. The scenario reveals parental anxiety due to marital problems. The most appropriate nursing diagnosis is alteration in growth patterns related to parental anxiety. There is no data that indicates familial short stature. Since height and weight are at the 5th percentile, there is no indication of increased nutritional intake. This infant is not at risk for constitutional growth delay. 2. The scenario reveals parental anxiety due to marital problems. The most appropriate nursing diagnosis is alteration in growth patterns related to parental anxiety. There is no data that indicates familial short stature. Since height and weight are at the 5th percentile, there is no indication of increased nutritional intake. This infant is not at risk for constitutional growth delay. 3. The scenario reveals parental anxiety due to marital problems. The most appropriate nursing diagnosis is alteration in growth patterns related to parental anxiety. There is no data that indicates familial short stature. Since height and weight are at the 5th percentile, there is no indication of increased nutritional intake. This infant is not at risk for constitutional growth delay. 4. The scenario reveals parental anxiety due to marital problems. The most appropriate nursing diagnosis is alteration in growth patterns related to parental anxiety. There is no data that indicates familial short stature. Since height and weight are at the 5th percentile, there is no indication of increased nutritional intake. This infant is not at risk for constitutional growth delay. Page Ref: 162-163 Cognitive Level: Analyzing Client Need &Sub: Health Promotion and Maintenance Standards: QSEN Competencies: Patient-centered care | AACN Essential Competencies: Essential VII: Clinical prevention and population health | NLN Competencies: Professional identity | Nursing/Integrated Concepts: Nursing Process: Diagnosis/Coordination of care Learning Outcome: LO 7.5 Evaluate data about the family and other social relationships to promote and maintain health of newborns and infants. MNL LO: Develop a family-centered nursing care plan for the child and family. 11) While teaching parents of a newborn about normal growth and development, which statement is most appropriate for the nurse to include in the session? 1. Weight should triple by 6 months of age. 2. Weight should double by 1 year of age. 3. Weight should double by 4 months of age. 4. Weight should triple by 1 year of age. Answer: 4 Explanation: 1. An infant should triple its birth weight by 1 year of age. The other answers are not appropriate weight gains. 2. An infant should triple its birth weight by 1 year of age. The other answers are not appropriate weight gains. 3. An infant should triple its birth weight by 1 year of age. The other answers are not appropriate weight gains. 4. An infant should triple its birth weight by 1 year of age. The other answers are not appropriate weight gains. Page Ref: 158 Cognitive Level: Applying Client Need &Sub: Health Promotion and Maintenance Standards: QSEN Competencies: Patient-centered care | AACN Essential Competencies: Essential VII: Clinical prevention and population health | NLN Competencies: Human flourishing | Nursing/Integrated Concepts: Nursing Process: Implementation/Health teaching and health promotion Learning Outcome: LO 7.1 Synthesize the areas of assessment and intervention for health supervision visits of newborns and infants: growth and developmental surveillance, nutrition, physical activity, oral health, mental and spiritual health, family and social relations, disease prevention strategies, and injury prevention strategies. MNL LO: Compare the developmental stages for pediatric clients. 12) A mother who is bottle feeding her newborn asks to be discharged 24 hours post-delivery, because she also has twin 2-year-old children at home. When should the nurse schedule the first office visit for this newborn? 1. Within 48 hours of discharge 2. Within 1 week of discharge 3. Within 2 weeks of discharge 4. When the infant is 1-month old Answer: 1 Explanation: 1. Newborns discharged before 48 hours old should be seen within 48 hours of discharge. Waiting 1 or 2 weeks after discharge of a 24-hour-old infant increases the chance that several common newborn conditions can go undiagnosed (e.g., jaundice, failure to gain weight). Waiting one month is too long for any infant who is discharged at 24 hours old. 2. Newborns discharged before 48 hours old should be seen within 48 hours of discharge. Waiting 1 or 2 weeks after discharge of a 24-hour-old infant increases the chance that several common newborn conditions can go undiagnosed (e.g., jaundice, failure to gain weight). Waiting one month is too long for any infant who is discharged at 24 hours old. 3. Newborns discharged before 48 hours old should be seen within 48 hours of discharge. Waiting 1 or 2 weeks after discharge of a 24-hour-old infant increases the chance that several common newborn conditions can go undiagnosed (e.g., jaundice, failure to gain weight). Waiting one month is too long for any infant who is discharged at 24 hours old. 4. Newborns discharged before 48 hours old should be seen within 48 hours of discharge. Waiting 1 or 2 weeks after discharge of a 24-hour-old infant increases the chance that several common newborn conditions can go undiagnosed (e.g., jaundice, failure to gain weight). Waiting one month is too long for any infant who is discharged at 24 hours old. Page Ref: 156 Cognitive Level: Applying Client Need &Sub: Health Promotion and Maintenance Standards: QSEN Competencies: Patient-centered care | AACN Essential Competencies: Essential VII: Clinical prevention and population health | NLN Competencies: Nursing judgement | Nursing/Integrated Concepts: Nursing Process: Implementation/Coordination of care Learning Outcome: LO 7.5 Evaluate data about the family and other social relationships to promote and maintain health of newborns and infants. MNL LO: Examine the role of the nurse in promoting culturally competent family-centered care. 13) A follow-up visit for a newborn client is scheduled with the pediatric nurse practitioner 3 days after discharge. What will the nurse include in the assessment during the scheduled visit for this newborn? Select all that apply. 1. Feeding pattern 2. Jaundice 3. Length 4. Vision screen 5. Sleep pattern Answer: 1, 2, 5 Explanation: 1. Feeding pattern, sleep pattern, and jaundice assessment would be appropriate 3 days after discharge. It would not be necessary to do a length or vision screen at this age. 2. Feeding pattern, sleep pattern, and jaundice assessment would be appropriate 3 days after discharge. It would not be necessary to do a length or vision screen at this age. 3. Feeding pattern, sleep pattern, and jaundice assessment would be appropriate 3 days after discharge. It would not be necessary to do a length or vision screen at this age. 4. Feeding pattern, sleep pattern, and jaundice assessment would be appropriate 3 days after discharge. It would not be necessary to do a length or vision screen at this age. 5. Feeding pattern, sleep pattern, and jaundice assessment would be appropriate 3 days after discharge. It would not be necessary to do a length or vision screen at this age. Page Ref: 156 Cognitive Level: Analyzing Client Need &Sub: Health Promotion and Maintenance Standards: QSEN Competencies: Patient-centered care | AACN Essential Competencies: Essential VII: Clinical prevention and population health | NLN Competencies: Human flourishing | Nursing/Integrated Concepts: Nursing Process: Planning/Coordination of care Learning Outcome: LO 7.1 Synthesize the areas of assessment and intervention for health supervision visits of newborns and infants: growth and developmental surveillance, nutrition, physical activity, oral health, mental and spiritual health, family and social relations, disease prevention strategies, and injury prevention strategies. MNL LO: Examine the role of the nurse in promoting culturally competent family-centered care. 14) A mother asks which developmental milestones she can expect when her baby is 6 months old. Which response by the nurse is the most appropriate? 1. Lifts head momentarily when prone 2. Has well-developed pincer grasp 3. Transfers objects from one hand to the other 4. Rolls from front to back Answer: 3 Explanation: 1. Lifting head when prone is a milestone at 1 month. A well-developed pincer grasp is a milestone at 12 months. Transferring objects from one hand to the other is a milestone at 6 months. Rolling from front to back is a milestone at 4 months. 2. Lifting head when prone is a milestone at 1 month. A well-developed pincer grasp is a milestone at 12 months. Transferring objects from one hand to the other is a milestone at 6 months. Rolling from front to back is a milestone at 4 months. 3. Lifting head when prone is a milestone at 1 month. A well-developed pincer grasp is a milestone at 12 months. Transferring objects from one hand to the other is a milestone at 6 months. Rolling from front to back is a milestone at 4 months. 4. Lifting head when prone is a milestone at 1 month. A well-developed pincer grasp is a milestone at 12 months. Transferring objects from one hand to the other is a milestone at 6 months. Rolling from front to back is a milestone at 4 months. Page Ref: 157-158 Cognitive Level: Analyzing Client Need &Sub: Health Promotion and Maintenance Standards: QSEN Competencies: Patient-centered care | AACN Essential Competencies: Essential I: Liberal Education for Baccalaureate generalist nursing practice | NLN Competencies: Human flourishing | Nursing/Integrated Concepts: Nursing Process: Implementation/Health teaching and health promotion Learning Outcome: LO 7.1 Synthesize the areas of assessment and intervention for health supervision visits of newborns and infants: growth and developmental surveillance, nutrition, physical activity, oral health, mental and spiritual health, family and social relations, disease prevention strategies, and injury prevention strategies. MNL LO: Compare the developmental stages for pediatric clients. 15) Injury prevention is an important aspect of parent teaching. Which injury prevention strategy would reduce the risk of suffocation? 1. Measure crib slat spacing at 2-3/8 inches or less. 2. Never leave an infant alone in a bath. 3. Position the infant on her back to sleep. 4. Use only approved restraint systems. Answer: 3 Explanation: 1. Measuring crib slats will reduce strangulation. Not leaving an infant alone in a bath will reduce drowning. Positioning an infant on her back will reduce suffocation. Using approved restraint systems will reduce motor vehicle injury. 2. Measuring crib slats will reduce strangulation. Not leaving an infant alone in a bath will reduce drowning. Positioning an infant on her back will reduce suffocation. Using approved restraint systems will reduce motor vehicle injury. 3. Measuring crib slats will reduce strangulation. Not leaving an infant alone in a bath will reduce drowning. Positioning an infant on her back will reduce suffocation. Using approved restraint systems will reduce motor vehicle injury. 4. Measuring crib slats will reduce strangulation. Not leaving an infant alone in a bath will reduce drowning. Positioning an infant on her back will reduce suffocation. Using approved restraint systems will reduce motor vehicle injury. Page Ref: 162 Cognitive Level: Applying Client Need &Sub: Health Promotion and Maintenance Standards: QSEN Competencies: Patient-centered care | AACN Essential Competencies: Essential II: Basic organizational and systems leadership for quality care and patient safety | NLN Competencies: Nursing judgement | Nursing/Integrated Concepts: Nursing Process: Implementation/Health teaching and health promotion Learning Outcome: LO 7.1 Synthesize the areas of assessment and intervention for health supervision visits of newborns and infants: growth and developmental surveillance, nutrition, physical activity, oral health, mental and spiritual health, family and social relations, disease prevention strategies, and injury prevention strategies. MNL LO: Differentiate developmentally appropriate care environments for the pediatric client and family. 16) The nurse is assessing an infant client and parents during a routine health supervision visit at 2 months of age. Which items will the nurse assess to determine if the infant's mental health needs are being addressed? Select all that apply. 1. Immunization record 2. Newborn screen results 3. Temperament during the visit 4. Feeding schedule 5. Sleep-wake patterns Answer: 3, 4, 5 Explanation: 1. When addressing mental health issues, the nurse would assess the infant's temperament during the visit, feeding schedule, and sleep-wake patterns. The infant's mental health is related to early experiences, inborn characteristics such as temperament and resilience, and relationships with caregivers. The first year of life provides many opportunities for the infant to develop positive mental health; interventions during this important period can enhance the child's future mental status. The immunization record and the newborn screen results will not provide the needed information for the nurse in terms of whether the infant's mental health needs are being addressed. 2. When addressing mental health issues, the nurse would assess the infant's temperament during the visit, feeding schedule, and sleep-wake patterns. The infant's mental health is related to early experiences, inborn characteristics such as temperament and resilience, and relationships with caregivers. The first year of life provides many opportunities for the infant to develop positive mental health; interventions during this important period can enhance the child's future mental status. The immunization record and the newborn screen results will not provide the needed information for the nurse in terms of whether the infant's mental health needs are being addressed. 3. When addressing mental health issues, the nurse would assess the infant's temperament during the visit, feeding schedule, and sleep-wake patterns. The infant's mental health is related to early experiences, inborn characteristics such as temperament and resilience, and relationships with caregivers. The first year of life provides many opportunities for the infant to develop positive mental health; interventions during this important period can enhance the child's future mental status. The immunization record and the newborn screen results will not provide the needed information for the nurse in terms of whether the infant's mental health needs are being addressed. 4. When addressing mental health issues, the nurse would assess the infant's temperament during the visit, feeding schedule, and sleep-wake patterns. The infant's mental health is related to early experiences, inborn characteristics such as temperament and resilience, and relationships with caregivers. The first year of life provides many opportunities for the infant to develop positive mental health; interventions during this important period can enhance the child's future mental status. The immunization record and the newborn screen results will not provide the needed information for the nurse in terms of whether the infant's mental health needs are being addressed. 5. When addressing mental health issues, the nurse would assess the infant's temperament during the visit, feeding schedule, and sleep-wake patterns. The infant's mental health is related to early experiences, inborn characteristics such as temperament and resilience, and relationships with caregivers. The first year of life provides many opportunities for the infant to develop positive mental health; interventions during this important period can enhance the child's future mental status. The immunization record and the newborn screen results will not provide the needed information for the nurse in terms of whether the infant's mental health needs are being addressed. Page Ref: 159-160 Cognitive Level: Applying Client Need &Sub: Health Promotion and Maintenance Standards: QSEN Competencies: Patient-centered care | AACN Essential Competencies: Essential IX: Baccalaureate generalist nursing practice | NLN Competencies: Human flourishing | Nursing/Integrated Concepts: Nursing Process: Assessment/Coordination of care Learning Outcome: LO 7.4 Integrate pertinent mental health care into health supervision visits for newborns and infants. MNL LO: Mental Health and Cognition/Apply the nursing process in providing care for the child and family. 17) The nurse is teaching a mother of a 2-month-old that she will begin to introduce certain foods to the diet between 4 and 6 months. The nurse should recommend what foods? Select all that apply. 1. Vegetables 2. Pasta 3. Rice cereal 4. Fruits 5. Soups Answer: 1, 3, 4 Explanation: 1. Reinforce proper introduction of new foods, to include rice cereal, vegetables, and fruits. Discuss any unusual food reactions observed. Pasta and soups are not advised at this time. 2. Reinforce proper introduction of new foods, to include rice cereal, vegetables, and fruits. Discuss any unusual food reactions observed. Pasta and soups are not advised at this time. 3. Reinforce proper introduction of new foods, to include rice cereal, vegetables, and fruits. Discuss any unusual food reactions observed. Pasta and soups are not advised at this time. 4. Reinforce proper introduction of new foods, to include rice cereal, vegetables, and fruits. Discuss any unusual food reactions observed. Pasta and soups are not advised at this time. 5. Reinforce proper introduction of new foods, to include rice cereal, vegetables, and fruits. Discuss any unusual food reactions observed. Pasta and soups are not advised at this time. Page Ref: 158 Cognitive Level: Analyzing Client Need &Sub: Physiological Integrity: Basic Care and Comfort Standards: QSEN Competencies: Patient-centered care | AACN Essential Competencies: Essential VII: Clinical prevention and population health | NLN Competencies: Human flourishing | Nursing/Integrated Concepts: Nursing Process: Planning/Health teaching and health promotion Learning Outcome: LO 7.1 Synthesize the areas of assessment and intervention for health supervision visits of newborns and infants: growth and developmental surveillance, nutrition, physical activity, oral health, mental and spiritual health, family and social relations, disease prevention strategies, and injury prevention strategies. MNL LO: Compare the nutritional needs of the infant, toddler, preschool-age child, school-age child, and adolescent. 18) The nurse is teaching the mothers of three-month-olds about oral health. Which of the following should the nurse include? Select all that apply. 1. Include iron vitamins once a day. 2. Avoid breastfeeding or drinking from a bottle when sleeping. 3. Allow to drink from a bottle at will during the day. 4. Cleanse gums 1 to 2 times a day. 5. Put baby to bed with a bottle of 2 percent milk only. Answer: 2, 4 Explanation: 1. The parents should wipe the infant's gums with soft moist gauze once or twice daily. Families are also cautioned to avoid having the infant breastfeed when sleeping, to avoid use of bottles in bed, and not to allow the infant to drink at will from a bottle during the day. These practices are linked to early childhood caries and can lead to tooth decay. 2. The parents should wipe the infant's gums with soft moist gauze once or twice daily. Families are also cautioned to avoid having the infant breastfeed when sleeping, to avoid use of bottles in bed, and not to allow the infant to drink at will from a bottle during the day. These practices are linked to early childhood caries and can lead to tooth decay. 3. The parents should wipe the infant's gums with soft moist gauze once or twice daily. Families are also cautioned to avoid having the infant breastfeed when sleeping, to avoid use of bottles in bed, and not to allow the infant to drink at will from a bottle during the day. These practices are linked to early childhood caries and can lead to tooth decay. 4. The parents should wipe the infant's gums with soft moist gauze once or twice daily. Families are also cautioned to avoid having the infant breastfeed when sleeping, to avoid use of bottles in bed, and not to allow the infant to drink at will from a bottle during the day. These practices are linked to early childhood caries and can lead to tooth decay. 5. The parents should wipe the infant's gums with soft moist gauze once or twice daily. Families are also cautioned to avoid having the infant breastfeed when sleeping, to avoid use of bottles in bed, and not to allow the infant to drink at will from a bottle during the day. These practices are linked to early childhood caries and can lead to tooth decay. Page Ref: 159 Cognitive Level: Analyzing Client Need &Sub: Physiological Integrity: Basic Care and Comfort Standards: QSEN Competencies: Patient-centered care | AACN Essential Competencies: Essential VII: Clinical prevention and population health | NLN Competencies: Human flourishing | Nursing/Integrated Concepts: Nursing Process: Planning/Health teaching and health promotion Learning Outcome: LO 7.1 Synthesize the areas of assessment and intervention for health supervision visits of newborns and infants: growth and developmental surveillance, nutrition, physical activity, oral health, mental and spiritual health, family and social relations, disease prevention strategies, and injury prevention strategies. MNL LO: Evaluate healthcare issues related to pediatric nursing care. Principles of Pediatric Nursing: Caring for Children, 7e (Ball et al.) Chapter 8 Health Promotion and Maintenance for the Toddler and Preschooler 1) Which of these aspects of developmental health supervision should be included in each healthcare visit of young children? Select all that apply. 1. Assessment 2. Discipline 3. Education 4. Intervention 5. Toilet training Answer: 1, 3, 4 Explanation: 1. The main recommendations for developmental health supervision of young children include assessment, education, intervention, and care coordination. This standard framework should be used as guidelines for each healthcare visit. Discipline and toilet training, while important to the care of children, are age specific and not part of the main developmental plans. 2. The main recommendations for developmental health supervision of young children include assessment, education, intervention, and care coordination. This standard framework should be used as guidelines for each healthcare visit. Discipline and toilet training, while important to the care of children, are age specific and not part of the main developmental plans. 3. The main recommendations for developmental health supervision of young children include assessment, education, intervention, and care coordination. This standard framework should be used as guidelines for each healthcare visit. Discipline and toilet training, while important to the care of children, are age specific and not part of the main developmental plans. 4. The main recommendations for developmental health supervision of young children include assessment, education, intervention, and care coordination. This standard framework should be used as guidelines for each healthcare visit. Discipline and toilet training, while important to the care of children, are age specific and not part of the main developmental plans. 5. The main recommendations for developmental health supervision of young children include assessment, education, intervention, and care coordination. This standard framework should be used as guidelines for each healthcare visit. Discipline and toilet training, while important to the care of children, are age specific and not part of the main developmental plans. Page Ref: 168 Cognitive Level: Applying Client Need &Sub: Health Promotion and Maintenance Standards: QSEN Competencies: Patient-centered care | AACN Essential Competencies: Essential VII: Clinical prevention and population health | NLN Competencies: Human flourishing | Nursing/Integrated Concepts: Nursing Process: Planning/Health teaching and health promotion Learning Outcome: LO 8.1 Describe the areas of assessment and intervention for health supervision visits for toddlers and preschool children: growth and developmental surveillance, nutrition, physical activity, oral health, mental and spiritual health, family and social relations, disease prevention strategies, and injury prevention strategies. MNL LO: Use the nursing process to provide developmentally appropriate communication strategies for pediatric clients and families. 2) A 27-month-old toddler who is in the pediatric office for a well-child visit begins to cry the moment he is placed on the examination table. The parent attempts to comfort the toddler; however, nothing is effective. Which of these actions by the nurse takes priority? 1. Instruct the father to hold the toddler down tightly to complete the examination. 2. Allow the toddler to sit on the parent's lap and begin the assessment. 3. Allow the toddler to stand on the floor until he stops crying. 4. Ask another nurse in the office to hold the toddler, because the parent is not able to control the toddler's behavior. Answer: 2 Explanation: 1. Toddlers are most comfortable when sitting with the parents. Much of the examination can be completed in this way. Allowing the toddler to stand on the floor is inappropriate. A nurse can assist if the parent is unable to hold the child during the examination of the throat and ears to prevent injury from movement. 2. Toddlers are most comfortable when sitting with the parents. Much of the examination can be completed in this way. Allowing the toddler to stand on the floor is inappropriate. A nurse can assist if the parent is unable to hold the child during the examination of the throat and ears to prevent injury from movement. 3. Toddlers are most comfortable when sitting with the parents. Much of the examination can be completed in this way. Allowing the toddler to stand on the floor is inappropriate. A nurse can assist if the parent is unable to hold the child during the examination of the throat and ears to prevent injury from movement. 4. Toddlers are most comfortable when sitting with the parents. Much of the examination can be completed in this way. Allowing the toddler to stand on the floor is inappropriate. A nurse can assist if the parent is unable to hold the child during the examination of the throat and ears to prevent injury from movement. Page Ref: 168, 169 Cognitive Level: Analyzing Client Need &Sub: Health Promotion and Maintenance Standards: QSEN Competencies: Patient-centered care | AACN Essential Competencies: Essential IX: Baccalaureate generalist nursing practice | NLN Competencies: Nursing judgement | Nursing/Integrated Concepts: Nursing Process: Implementation/Health teaching and health promotion Learning Outcome: LO 8.3 Plan health promotion and health maintenance strategies employed during health supervision visits of toddlers and preschoolers. MNL LO: Develop a family-centered nursing care plan for the child and family. 3) At a routine healthcare visit, a nurse measures a toddler and plots the height and weight on the growth charts. The nurse documents that the toddler is above the 95th percentile for weight and is at the 5th percentile for height. How should the nurse interpret these data? 1. The toddler is proportionate for the age. 2. The toddler needs to eat more at each feeding. 3. The height and weight are disproportionate, and the toddler needs further evaluation. 4. The family is most likely short. Answer: 3 Explanation: 1. Usually height and weight are at approximately the same percentile. When the weight of a child is found to be at the 95th percentile, the child's height is also greater than the 50th percentile. The height and weight for the child described in this question are a concern, and the child may need further endocrine testing. 2. Usually height and weight are at approximately the same percentile. When the weight of a child is found to be at the 95th percentile, the child's height is also greater than the 50th percentile. The height and weight for the child described in this question are a concern, and the child may need further endocrine testing. 3. Usually height and weight are at approximately the same percentile. When the weight of a child is found to be at the 95th percentile, the child's height is also greater than the 50th percentile. The height and weight for the child described in this question are a concern, and the child may need further endocrine testing. 4. Usually height and weight are at approximately the same percentile. When the weight of a child is found to be at the 95th percentile, the child's height is also greater than the 50th percentile. The height and weight for the child described in this question are a concern, and the child may need further endocrine testing. Page Ref: 168 Cognitive Level: Analyzing Client Need &Sub: Health Promotion and Maintenance Standards: QSEN Competencies: Patient-centered care | AACN Essential Competencies: Essential IX: Baccalaureate generalist nursing practice | NLN Competencies: Human flourishing | Nursing/Integrated Concepts: Nursing Process: Assessment/Health teaching and health promotion Learning Outcome: LO 8.1 Describe the areas of assessment and intervention for health supervision visits for toddlers and preschool children: growth and developmental surveillance, nutrition, physical activity, oral health, mental and spiritual health, family and social relations, disease prevention strategies, and injury prevention strategies. MNL LO: Compare the nutritional needs of the infant, toddler, preschool-age child, school-age child, and adolescent. 4) A nurse is preparing to perform a physical assessment on a toddler. Which action is most appropriate for the nurse to take? 1. Perform the assessment from head to toe. 2. Leave intrusive procedures such as ear and eye examinations until the end. 3. Explain each part of the examination to the child before performing it. 4. Ask the mother to tell the child not to be afraid. Answer: 2 Explanation: 1. Intrusive procedures such as examination of the ears, throat, eye, and genital areas should be done last to decrease the anxiety of the child during the initial phases of the examination, which includes the heart and lungs. 2. Intrusive procedures such as examination of the ears, throat, eye, and genital areas should be done last to decrease the anxiety of the child during the initial phases of the examination, which includes the heart and lungs. 3. Intrusive procedures such as examination of the ears, throat, eye, and genital areas should be done last to decrease the anxiety of the child during the initial phases of the examination, which includes the heart and lungs. 4. Intrusive procedures such as examination of the ears, throat, eye, and genital areas should be done last to decrease the anxiety of the child during the initial phases of the examination, which includes the heart and lungs. Page Ref: 168 Cognitive Level: Applying Client Need &Sub: Health Promotion and Maintenance Standards: QSEN Competencies: Patient-centered care | AACN Essential Competencies: Essential IX: Baccalaureate generalist nursing practice | NLN Competencies: Nursing judgement | Nursing/Integrated Concepts: Nursing Process: Implementation/Health teaching and health promotion Learning Outcome: LO 8.3 Plan health promotion and health maintenance strategies employed during health supervision visits of toddlers and preschoolers. MNL LO: Use the nursing process to provide developmentally appropriate communication strategies for pediatric clients and families. 5) Which of these developmental milestones should the nurse expect to find in children who are between 2 and 3 years old? Select all that apply. 1. Always feeds self 2. Scribbles and draws on paper 3. Kicks a ball 4. Throws ball overhand 5. Goes up and down stairs Answer: 2, 3, 5 Explanation: 1. Children between the ages of 2 and 3 years can scribble and draw on paper, kick a ball, and go up and down the stairs. Children who are between the ages of 3 and 4 years can feed themselves. Children between the ages of 4 and 5 years can throw a ball overhand. 2. Children between the ages of 2 and 3 years can scribble and draw on paper, kick a ball, and go up and down the stairs. Children who are between the ages of 3 and 4 years can feed themselves. Children between the ages of 4 and 5 years can throw a ball overhand. 3. Children between the ages of 2 and 3 years can scribble and draw on paper, kick a ball, and go up and down the stairs. Children who are between the ages of 3 and 4 years can feed themselves. Children between the ages of 4 and 5 years can throw a ball overhand. 4. Children between the ages of 2 and 3 years can scribble and draw on paper, kick a ball, and go up and down the stairs. Children who are between the ages of 3 and 4 years can feed themselves. Children between the ages of 4 and 5 years can throw a ball overhand. 5. Children between the ages of 2 and 3 years can scribble and draw on paper, kick a ball, and go up and down the stairs. Children who are between the ages of 3 and 4 years can feed themselves. Children between the ages of 4 and 5 years can throw a ball overhand. Page Ref: 171 Cognitive Level: Analyzing Client Need &Sub: Health Promotion and Maintenance Standards: QSEN Competencies: Patient-centered care | AACN Essential Competencies: Essential IX: Baccalaureate generalist nursing practice | NLN Competencies: Human flourishing | Nursing/Integrated Concepts: Nursing Process: Assessment/Health teaching and health promotion Learning Outcome: LO 8.1 Describe the areas of assessment and intervention for health supervision visits for toddlers and preschool children: growth and developmental surveillance, nutrition, physical activity, oral health, mental and spiritual health, family and social relations, disease prevention strategies, and injury prevention strategies. MNL LO: Compare the developmental stages for pediatric clients. 6) A nurse observes the parent/child interaction during the 4-year-old well-child checkup and notes that the parent speaks harshly to the child and uses negative remarks when speaking with the nurse. Which statement by the nurse would be most beneficial? 1. "Perhaps you should leave the room so that I can speak with your child privately." 2. "I am going to refer you for counseling since your interactions with your child seem so negative." 3. "Let's talk privately. Let's discuss the way you speak with your child and possible ways to be more positive." 4. Addressing the child, the nurse says, "Are you unhappy when Mommy talks to you like this?" Answer: 3 Explanation: 1. The best approach to this encounter would be for the nurse to discuss concerns with the parent privately, since the nurse wants to help the parent develop a good relationship with the child. The child should not be a part of this conversation. Because the child is only 4 years old, it would be difficult to ask the parent to leave the room. If the nurse also wants to speak alone with the child, the nurse perhaps would escort the child to another area and speak briefly with the child. Referring to counseling without a discussion with the parent is not appropriate. The nurse should not ask the child if she is "unhappy" with the parent. 2. The best approach to this encounter would be for the nurse to discuss concerns with the parent privately, since the nurse wants to help the parent develop a good relationship with the child. The child should not be a part of this conversation. Because the child is only 4 years old, it would be difficult to ask the parent to leave the room. If the nurse also wants to speak alone with the child, the nurse perhaps would escort the child to another area and speak briefly with the child. Referring to counseling without a discussion with the parent is not appropriate. The nurse should not ask the child if she is "unhappy" with the parent. 3. The best approach to this encounter would be for the nurse to discuss concerns with the parent privately, since the nurse wants to help the parent develop a good relationship with the child. The child should not be a part of this conversation. Because the child is only 4 years old, it would be difficult to ask the parent to leave the room. If the nurse also wants to speak alone with the child, the nurse perhaps would escort the child to another area and speak briefly with the child. Referring to counseling without a discussion with the parent is not appropriate. The nurse should not ask the child if she is "unhappy" with the parent. 4. The best approach to this encounter would be for the nurse to discuss concerns with the parent privately, since the nurse wants to help the parent develop a good relationship with the child. The child should not be a part of this conversation. Because the child is only 4 years old, it would be difficult to ask the parent to leave the room. If the nurse also wants to speak alone with the child, the nurse perhaps would escort the child to another area and speak briefly with the child. Referring to counseling without a discussion with the parent is not appropriate. The nurse should not ask the child if she is "unhappy" with the parent. Page Ref: 174, 175 Cognitive Level: Applying Client Need &Sub: Health Promotion and Maintenance Standards: QSEN Competencies: Patient-centered care | AACN Essential Competencies: Essential II: Basic organizational and systems leadership for quality care and patient safety | NLN Competencies: Professional identity | Nursing/Integrated Concepts: Nursing Process: Implementation/Health teaching and health promotion Learning Outcome: LO 8.4 Discuss the importance of family in child health care, and include family assessment in each health supervision visit. MNL LO: Implement developmentally appropriate communication strategies for pediatric clients and families. 7) A nurse who is the manager of an ambulatory pediatric healthcare center is planning protocols for the routine healthcare visits of the children. Children at this care center have a high incidence of obesity. At which age should the nurses at this clinic calculate the body mass index (BMI) for all pediatric clients? 1. 12 months 2. 24 months 3. 36 months 4. 4 years Answer: 2 Explanation: 1. The body mass index is first calculated at 2 years of age and gives information about the relationship between the height and weight of the child. With this information, the nurse would be able to develop strategies that may reduce the incidence of obesity. 2. The body mass index is first calculated at 2 years of age and gives information about the relationship between the height and weight of the child. With this information, the nurse would be able to develop strategies that may reduce the incidence of obesity. 3. The body mass index is first calculated at 2 years of age and gives information about the relationship between the height and weight of the child. With this information, the nurse would be able to develop strategies that may reduce the incidence of obesity. 4. The body mass index is first calculated at 2 years of age and gives information about the relationship between the height and weight of the child. With this information, the nurse would be able to develop strategies that may reduce the incidence of obesity. Page Ref: 168 Cognitive Level: Applying Client Need &Sub: Health Promotion and Maintenance Standards: QSEN Competencies: Quality improvement | AACN Essential Competencies: Essential V: Healthcare policy, finance, and regulatory environments | NLN Competencies: Nursing judgement | Nursing/Integrated Concepts: Nursing Process: Assessment/Health teaching and health promotion Learning Outcome: LO 8.3 Plan health promotion and health maintenance strategies employed during health supervision visits of toddlers and preschoolers. MNL LO: Develop a family-centered nursing care plan for the child and family. 8) Which of these measures used by a nurse will help relieve parental anxiety related to the changing appetite in the toddler who is gaining weight along the 50th percentile? 1. Discussing the growth of the toddler as compared to the growth chart 2. Suggesting ways to have the toddler eat higher calorie foods 3. Instructing the mother to feed the toddler alone without any distractions such as TV or music 4. Teaching the mother to avoid disciplining the toddler within one-half hour of eating Answer: 1 Explanation: 1. Showing the parents the growth pattern of the child as compared to the normal growth chart will help relieve parental anxiety related to eating less food during the toddler years. Toddlers who are at the 50th percentile do not need additional high-calorie foods. Toddlers eat to their personal needs and there is no reason to restrict watching TV or other environmental stimuli during meals. There is no reason to relate timing of discipline and eating. 2. Showing the parents the growth pattern of the child as compared to the normal growth chart will help relieve parental anxiety related to eating less food during the toddler years. Toddlers who are at the 50th percentile do not need additional high-calorie foods. Toddlers eat to their personal needs and there is no reason to restrict watching TV or other environmental stimuli during meals. There is no reason to relate timing of discipline and eating. 3. Showing the parents the growth pattern of the child as compared to the normal growth chart will help relieve parental anxiety related to eating less food during the toddler years. Toddlers who are at the 50th percentile do not need additional high-calorie foods. Toddlers eat to their personal needs and there is no reason to restrict watching TV or other environmental stimuli during meals. There is no reason to relate timing of discipline and eating. 4. Showing the parents the growth pattern of the child as compared to the normal growth chart will help relieve parental anxiety related to eating less food during the toddler years. Toddlers who are at the 50th percentile do not need additional high-calorie foods. Toddlers eat to their personal needs and there is no reason to restrict watching TV or other environmental stimuli during meals. There is no reason to relate timing of discipline and eating. Page Ref: 170 Cognitive Level: Applying Client Need &Sub: Health Promotion and Maintenance Standards: QSEN Competencies: Quality improvement | AACN Essential Competencies: Essential IX: Baccalaureate generalist nursing practice | NLN Competencies: Nursing judgement | Nursing/Integrated Concepts: Nursing Process: Implementation/Health teaching and health promotion Learning Outcome: LO 8.1 Describe the areas of assessment and intervention for health supervision visits for toddlers and preschool children: growth and developmental surveillance, nutrition, physical activity, oral health, mental and spiritual health, family and social relations, disease prevention strategies, and injury prevention strategies. MNL LO: Develop a family-centered nursing care plan for the child and family. 9) Parents of a preschool-age child report that they find it necessary to spank the child at least once a day. Which response should the nurse make to the parents? 1. "Spanking is one form of discipline; however, you want to be certain that you do not leave any marks on the child." 2. "Let's talk about other forms of discipline that have a more positive effect on the child." 3. "Can you try only spanking the child every other day for one week and see how that affects the child's behavior?" 4. "I think you are not parenting your child properly, so let's talk about ways to improve your parenting skills." Answer: 2 Explanation: 1. The behavior reported by the parent was excessive. The only response that is appropriate is to find a more positive way of influencing behavior in this age child. The nurse's response needs to reflect these feelings. To suggest spanking as an appropriate form of discipline is inappropriate, especially when the parent is describing daily spanking of the child. 2. The behavior reported by the parent was excessive. The only response that is appropriate is to find a more positive way of influencing behavior in this age child. The nurse's response needs to reflect these feelings. To suggest spanking as an appropriate form of discipline is inappropriate, especially when the parent is describing daily spanking of the child. 3. The behavior reported by the parent was excessive. The only response that is appropriate is to find a more positive way of influencing behavior in this age child. The nurse's response needs to reflect these feelings. To suggest spanking as an appropriate form of discipline is inappropriate, especially when the parent is describing daily spanking of the child. 4. The behavior reported by the parent was excessive. The only response that is appropriate is to find a more positive way of influencing behavior in this age child. The nurse's response needs to reflect these feelings. To suggest spanking as an appropriate form of discipline is inappropriate, especially when the parent is describing daily spanking of the child. Page Ref: 173,174 Cognitive Level: Applying Client Need &Sub: Health Promotion and Maintenance Standards: QSEN Competencies: Patient-centered care | AACN Essential Competencies: Essential II: Basic organizational and systems leadership for quality care and patient safety | NLN Competencies: Professional identity | Nursing/Integrated Concepts: Nursing Process: Implementation/Health teaching and health promotion Learning Outcome: LO 8.5 Integrate pertinent mental health care into health supervision visits for young children. MNL LO: Develop a family-centered nursing care plan for the child and family. 10) A parent questions how her toddler plays with other toddlers. Which response by the nurse displays the best description of the differences in play between the toddler and the preschool-age child? 1. Toddlers play side by side, while preschool-age children play cooperatively. 2. Toddlers play house and imitate adult roles, while preschool-age children become the Mom or Dad while playing house. 3. Toddlers play cooperatively, while preschool-age children play interactive games. 4. There are no differences between toddlers and preschool-age children because both groups play cooperatively. Answer: 1 Explanation: 1. Toddlers will play side by side with another child, but they do not interact with the child during play. Preschoolers play cooperatively with other children. 2. Toddlers will play side by side with another child, but they do not interact with the child during play. Preschoolers play cooperatively with other children. 3. Toddlers will play side by side with another child, but they do not interact with the child during play. Preschoolers play cooperatively with other children. 4. Toddlers will play side by side with another child, but they do not interact with the child during play. Preschoolers play cooperatively with other children. Page Ref: 174 Cognitive Level: Analyzing Client Need &Sub: Health Promotion and Maintenance Standards: QSEN Competencies: Patient-centered care | AACN Essential Competencies: Essential VII: Clinical prevention and population health | NLN Competencies: Human flourishing | Nursing/Integrated Concepts: Nursing Process: Implementation/Health teaching and health promotion Learning Outcome: LO 8.1 Describe the areas of assessment and intervention for health supervision visits for toddlers and preschool children: growth and developmental surveillance, nutrition, physical activity, oral health, mental and spiritual health, family and social relations, disease prevention strategies, and injury prevention strategies. MNL LO: Compare the developmental stages for pediatric clients. 11) A mother of an 18-month old asks the nurse whether she can begin to introduce low-fat milk like the rest of the family drinks. The nurse answers the mother based on the knowledge that low-fat milk can safely be introduced at what age? 1. 18 months 2. 24 months 3. 3 years 4. 4 years Answer: 2 Explanation: 1. Health promotion for the toddler includes whole milk until age 2. Age 1 is too early for low-fat milk; and it can safely be introduced before ages 3 and 4. 2. Health promotion for the toddler includes whole milk until age 2. Age 1 is too early for low- fat milk, and it can safely be introduced before ages 3 and 4. 3. Health promotion for the toddler includes whole milk until age 2. Age 1 is too early for low- fat milk, and it can safely be introduced before ages 3 and 4. 4. Health promotion for the toddler includes whole milk until age 2. Age 1 is too early for low- fat milk, and it can safely be introduced before ages 3 and 4. Page Ref: 170, 171 Cognitive Level: Applying Client Need &Sub: Health Promotion and Maintenance Standards: QSEN Competencies: Patient-centered care | AACN Essential Competencies: Essential VII: Clinical prevention and population health | NLN Competencies: Human flourishing | Nursing/Integrated Concepts: Nursing Process: Assessment/Health teaching and health promotion Learning Outcome: LO 8.1 Describe the areas of assessment and intervention for health supervision visits for toddlers and preschool children: growth and developmental surveillance, nutrition, physical activity, oral health, mental and spiritual health, family and social relations, disease prevention strategies, and injury prevention strategies. MNL LO: Compare the nutritional needs of the infant, toddler, preschool-age child, school-age child, and adolescent. 12) The nurse is evaluating the car seat of a 3-year-old who weighs 42 pounds. Which recommendation should the nurse make about the car seat to the parents? 1. Convertible, rear-facing seat 2. Belt-positioning booster seat 3. A car seat with a harness approved for higher weights and heights 4. A regular seat with lap and shoulder strap Answer: 3 Explanation: 1. The American Academy of Pediatrics and the National Highway Safety Administration recommend booster seats for children over 40 pounds and 4 years of age. A 3- year-old should be in a regular car seat with approved harness for higher-weight/height children so that the child is protected from injury. Rear-facing seats and regular seat with lap and shoulder strap are not appropriate for a 3-year-old. 2. The American Academy of Pediatrics and the National Highway Safety Administration recommend booster seats for children over 40 pounds and 4 years of age. A 3-year-old should be in a regular car seat with approved harness for higher-weight/height children so that the child is protected from injury. Rear-facing seats and regular seat with lap and shoulder strap are not appropriate for a 3-year-old. 3. The American Academy of Pediatrics and the National Highway Safety Administration recommend booster seats for children over 40 pounds and 4 years of age. A 3-year-old should be in a regular car seat with approved harness for higher-weight/height children so that the child is protected from injury. Rear-facing seats and regular seat with lap and shoulder strap are not appropriate for a 3-year-old. 4. The American Academy of Pediatrics and the National Highway Safety Administration recommend booster seats for children over 40 pounds and 4 years of age. A 3-year-old should be in a regular car seat with approved harness for higher-weight/height children so that the child is protected from injury. Rear-facing seats and regular seat with lap and shoulder strap are not appropriate for a 3-year-old. Page Ref: 175 Cognitive Level: Applying Client Need &Sub: Health Promotion and Maintenance Standards: QSEN Competencies: Safety | AACN Essential Competencies: Essential II: Basic organizational and systems leadership for quality care and patient safety | NLN Competencies: Professional identity | Nursing/Integrated Concepts: Nursing Process: Implementation/Health teaching and health promotion Learning Outcome: LO 8.3 Plan health promotion and health maintenance strategies employed during health supervision visits of toddlers and preschoolers. MNL LO: Evaluate healthcare issues related to pediatric nursing care. 13) During a clinic visit, the parents of a 15-month-old ask what disease and injury prevention topics would be appropriate to discuss at this age. Which response by the nurse is the most appropriate? 1. "It's never too early to teach a child to wear a helmet when riding a bicycle." 2. "Teaching simple handwashing is a good topic at this age." 3. "Tell the child over and over to stay away from water unless you are with him." 4. "Tell him firmly 'no' when he tries to cross the street." Answer: 2 Explanation: 1. Disease and injury prevention are ongoing topics at all ages. Simple handwashing is appropriate for a 15-month-old child. A 15-month-old is too young for bicycle riding, so this can be delayed. A 15-month-old is too young to understand water safety and crossing the street, and should never be left unattended in these situations. 2. Disease and injury prevention are ongoing topics at all ages. Simple handwashing is appropriate for a 15-month-old child. A 15-month-old is too young for bicycle riding, so this can be delayed. A 15-month-old is too young to understand water safety and crossing the street, and should never be left unattended in these situations. 3. Disease and injury prevention are ongoing topics at all ages. Simple handwashing is appropriate for a 15-month-old child. A 15-month-old is too young for bicycle riding, so this can be delayed. A 15-month-old is too young to understand water safety and crossing the street, and should never be left unattended in these situations. 4. Disease and injury prevention are ongoing topics at all ages. Simple handwashing is appropriate for a 15-month-old child. A 15-month-old is too young for bicycle riding, so this can be delayed. A 15-month-old is too young to understand water safety and crossing the street, and should never be left unattended in these situations. Page Ref: 175 Cognitive Level: Applying Client Need &Sub: Health Promotion and Maintenance Standards: QSEN Competencies: Patient-centered care | AACN Essential Competencies: Essential VII: Clinical prevention and population health | NLN Competencies: Human flourishing | Nursing/Integrated Concepts: Nursing Process: Implementation/Health teaching and health promotion Learning Outcome: LO 8.3 Plan health promotion and health maintenance strategies employed during health supervision visits of toddlers and preschoolers. MNL LO: Differentiate developmentally appropriate care environments for the pediatric client and family. 14) Which assessment question would get the most accurate response when a nurse is assessing learning and reading skills in the early childhood years? 1. "What rewards do you use when your child does something good?" 2. "What is your child's language like now?" 3. "Does your child get along well with others?" 4. "Do you keep books for your child readily available?" Answer: 4 Explanation: 1. Keeping books readily available will stimulate reading skills. This is the question that will provide the most information about learning and reading skills. Language and getting along with others are more communication skills. Rewards are more closely related to discipline. 2. Keeping books readily available will stimulate reading skills. This is the question that will provide the most information about learning and reading skills. Language and getting along with others are more communication skills. Rewards are more closely related to discipline. 3. Keeping books readily available will stimulate reading skills. This is the question that will provide the most information about learning and reading skills. Language and getting along with others are more communication skills. Rewards are more closely related to discipline. 4. Keeping books readily available will stimulate reading skills. This is the question that will provide the most information about learning and reading skills. Language and getting along with others are more communication skills. Rewards are more closely related to discipline. Page Ref: 178 Cognitive Level: Applying Client Need &Sub: Health Promotion and Maintenance Standards: QSEN Competencies: Patient-centered care | AACN Essential Competencies: Essential IX: Baccalaureate generalist nursing practice | NLN Competencies: Human flourishing | Nursing/Integrated Concepts: Nursing Process: Implementation/Health teaching and health promotion Learning Outcome: LO 8.3 Plan health promotion and health maintenance strategies employed during health supervision visits of toddlers and preschoolers. MNL LO: Implement developmentally appropriate communication strategies for pediatric clients and families. 15) The nurse is asked to teach injury prevention measures to a classroom of 4-year-old preschoolers. Which teaching points are most appropriate at this age? Select all that apply. 1. Stop, drop and roll if clothes catch fire 2. Never go into the road alone. 3. Acceptable places for climbing 4. Safe meeting place outside the house in case of fire 5. Car seat safety Answer: 1, 2, 4, 5 Explanation: 1. Acceptable places to climb should be introduced in the toddler years when children are learning to walk, climb, and explore. It is not a topic for a preschool class. All the other topics are appropriate for this age. 2. Acceptable places to climb should be introduced in the toddler years when children are learning to walk, climb, and explore. It is not a topic for a preschool class. All the other topics are appropriate for this age. 3. Acceptable places to climb should be introduced in the toddler years when children are learning to walk, climb, and explore. It is not a topic for a preschool class. All the other topics are appropriate for this age. 4. Acceptable places to climb should be introduced in the toddler years when children are learning to walk, climb, and explore. It is not a topic for a preschool class. All the other topics are appropriate for this age. 5. Acceptable places to climb should be introduced in the toddler years when children are learning to walk, climb, and explore. It is not a topic for a preschool class. All the other topics are appropriate for this age. Page Ref: 175 Cognitive Level: Applying Client Need &Sub: Health Promotion and Maintenance Standards: QSEN Competencies: Quality improvement | AACN Essential Competencies: Essential VII: Clinical prevention and population health | NLN Competencies: Nursing judgement | Nursing/Integrated Concepts: Nursing Process: Implementation/Health teaching and health promotion Learning Outcome: LO 8.3 Plan health promotion and health maintenance strategies employed during health supervision visits of toddlers and preschoolers. MNL LO: Use the nursing process to provide developmentally appropriate care for the pediatric client. 16) The nurse is performing a well-child exam on a child who turned 4 years old 3 months ago. What can the nurse ask the child to do to assess appropriate milestones for this age? 1. Jump up and down 2. Throw a ball 3. Stack three or more blocks 4. Draw lines on paper Answer: 2 Explanation: 1. Jumping up and down, stacking three or more blocks, and drawing lines on paper are activities that represent milestones for young children. Throwing a ball and observing how it is thrown would assess a milestone for this age. By 4 to 5 years, a child begins to throw a ball overhand. 2. Jumping up and down, stacking three or more blocks, and drawing lines on paper are activities that represent milestones for young children. Throwing a ball and observing how it is thrown would assess a milestone for this age. By 4 to 5 years, a child begins to throw a ball overhand. 3. Jumping up and down, stacking three or more blocks, and drawing lines on paper are activities that represent milestones for young children. Throwing a ball and observing how it is thrown would assess a milestone for this age. By 4 to 5 years, a child begins to throw a ball overhand. 4. Jumping up and down, stacking three or more blocks, and drawing lines on paper are activities that represent milestones for young children. Throwing a ball and observing how it is thrown would assess a milestone for this age. By 4 to 5 years, a child begins to throw a ball overhand. Page Ref: 171, 172 Cognitive Level: Analyzing Client Need &Sub: Health Promotion and Maintenance Standards: QSEN Competencies: Patient-centered care | AACN Essential Competencies: Essential IX: Baccalaureate generalist nursing practice | NLN Competencies: Human flourishing | Nursing/Integrated Concepts: Nursing Process: Assessment/Health teaching and health promotion Learning Outcome: LO 8.1 Describe the areas of assessment and intervention for health supervision visits for toddlers and preschool children: growth and developmental surveillance, nutrition, physical activity, oral health, mental and spiritual health, family and social relations, disease prevention strategies, and injury prevention strategies. MNL LO: Use the nursing process to provide developmentally appropriate care for the pediatric client. 17) Which health promotion activities can the nurse recommend to the parents of a preschool-age child in order to enhance the child's self-concept? Select all that apply. 1. Encourage a play date with a school-age child. 2. Praise the child for staying dry at night. 3. Tell the child there will be a punishment for bathroom accidents. 4. Set aside time for the child each day. 5. Discuss appropriate activities to engage in with the daycare provider. Answer: 2, 4, 5 Explanation: 1. Health promotion activities focus on development of a healthy self-concept in the toddler and young child by helping parents to set up successful play experiences, to praise the child for successes, to use effective limit-setting techniques, and to realize and appreciate the child's unique characteristics. Health maintenance seeks to avoid the poor self-image that can occur with constant criticism or expectations not in alignment with the toddler's or preschooler's developmental capabilities. 2. Health promotion activities focus on development of a healthy self-concept in the toddler and young child by helping parents to set up successful play experiences, to praise the child for successes, to use effective limit-setting techniques, and to realize and appreciate the child's unique characteristics. Health maintenance seeks to avoid the poor self-image that can occur with constant criticism or expectations not in alignment with the toddler's or preschooler's developmental capabilities. 3. Health promotion activities focus on development of a healthy self-concept in the toddler and young child by helping parents to set up successful play experiences, to praise the child for successes, to use effective limit-setting techniques, and to realize and appreciate the child's unique characteristics. Health maintenance seeks to avoid the poor self-image that can occur with constant criticism or expectations not in alignment with the toddler's or preschooler's developmental capabilities. 4. Health promotion activities focus on development of a healthy self-concept in the toddler and young child by helping parents to set up successful play experiences, to praise the child for successes, to use effective limit-setting techniques, and to realize and appreciate the child's unique characteristics. Health maintenance seeks to avoid the poor self-image that can occur with constant criticism or expectations not in alignment with the toddler's or preschooler's developmental capabilities. 5. Health promotion activities focus on development of a healthy self-concept in the toddler and young child by helping parents to set up successful play experiences, to praise the child for successes, to use effective limit-setting techniques, and to realize and appreciate the child's unique characteristics. Health maintenance seeks to avoid the poor self-image that can occur with constant criticism or expectations not in alignment with the toddler's or preschooler's developmental capabilities. Page Ref: 173, 174 Cognitive Level: Applying Client Need &Sub: Health Promotion and Maintenance Standards: QSEN Competencies: Patient-centered care | AACN Essential Competencies: Essential VII: Clinical prevention and population health | NLN Competencies: Human flourishing | Nursing/Integrated Concepts: Nursing Process: Implementation/Health teaching and health promotion Learning Outcome: LO 8.2 State components of self-concept for toddlers and preschoolers. MNL LO: Develop a family-centered nursing care plan for the child and family. 18) The nurse is planning care for a preschool-age child and family. In order to assess the family, what should the nurse plan to do during each health supervision visit? Select all that apply. 1. Discuss the child's developmental status 2. Observe interactions among the family members 3. Discuss concerns with the parents 4. Administer age appropriate vaccinations 5. Record height and weight Answer: 1, 2, 3 Explanation: 1. In order to assess the child and family, the nurse would plan to discuss the child's developmental status, observe interactions among the family members, and discuss any concerns with the parents. Administering age appropriate vaccinations and recording height and weight are appropriate interventions, but are not included during the family assessment process. 2. In order to assess the child and family, the nurse would plan to discuss the child's developmental status, observe interactions among the family members, and discuss any concerns with the parents. Administering age appropriate vaccinations and recording height and weight are appropriate interventions, but are not included during the family assessment process. 3. In order to assess the child and family, the nurse would plan to discuss the child's developmental status, observe interactions among the family members, and discuss any concerns with the parents. Administering age appropriate vaccinations and recording height and weight are appropriate interventions, but are not included during the family assessment process. 4. In order to assess the child and family, the nurse would plan to discuss the child's developmental status, observe interactions among the family members, and discuss any concerns with the parents. Administering age appropriate vaccinations and recording height and weight are appropriate interventions, but are not included during the family assessment process. 5. In order to assess the child and family, the nurse would plan to discuss the child's developmental status, observe interactions among the family members, and discuss any concerns with the parents. Administering age appropriate vaccinations and recording height and weight are appropriate interventions, but are not included during the family assessment process. Page Ref: 168, 169 Cognitive Level: Applying Client Need &Sub: Health Promotion and Maintenance Standards: QSEN Competencies: Patient-centered care | AACN Essential Competencies: Essential VII: Clinical prevention and population health | NLN Competencies: Human flourishing | Nursing/Integrated Concepts: Nursing Process: Planning/Health teaching and health promotion Learning Outcome: LO 8.4 Discuss the importance of family in child health care, and include family assessment in each health supervision visit. MNL LO: Use the nursing process to provide developmentally appropriate care for the pediatric client. 19) The nurse is creating a teaching care plan for the toddler and family. Which nursing diagnoses are normally used at each healthcare visit for this age group? Select all that apply. 1. Knowledge deficit related to growth patterns 2. Risk for injury related to developmental skills 3. Risk for exposure to infectious diseases related to childcare environment 4. Knowledge deficit related to toys that encourage development 5. Risk for loneliness related to lack of siblings Answer: 1, 2, 3, 4 Explanation: 1. The toddler is assessed for height, weight, BMI, head circumference, growth and nutrition, verbal skills, gross and fine motor movement, appropriate toys for developmental age. 2. The toddler is assessed for height, weight, BMI, head circumference, growth and nutrition, verbal skills, gross and fine motor movement, appropriate toys for developmental age. 3. The toddler is assessed for height, weight, BMI, head circumference, growth and nutrition, verbal skills, gross and fine motor movement, appropriate toys for developmental age. 4. The toddler is assessed for height, weight, BMI, head circumference, growth and nutrition, verbal skills, gross and fine motor movement, appropriate toys for developmental age. 5. The toddler is assessed for height, weight, BMI, head circumference, growth and nutrition, verbal skills, gross and fine motor movement, appropriate toys for developmental age. Page Ref: 170 Cognitive Level: Analyzing Client Need &Sub: Safe and Effective Care Environment: Management of Care Standards: QSEN Competencies: Patient-centered care | AACN Essential Competencies: Essential II: Basic organizational and systems leadership for quality care and patient safety | NLN Competencies: Nursing judgement | Nursing/Integrated Concepts: Nursing Process: Diagnosis/Coordination of Care Learning Outcome: LO 8.3 Plan health promotion and health maintenance strategies employed during health supervision visits of toddlers and preschoolers. MNL LO: Use the nursing process to provide developmentally appropriate care for the pediatric client. 20) The nurse is assessing the toddler for early childhood caries. The nurse will teach the family which factors contribute to this condition? Select all that apply. 1. Inadequate activity 2. Inadequate dental care 3. Inadequate diet 4. Inadequate brushing 5. Inadequate pacifiers Answer: 2, 3, 4 Explanation: 1. Early childhood caries is promoted by inadequate preventive care, which can include diet, brushing, feeding habits, and lack of dental care. ECC is serious because young children with the condition are more likely to have continuing dental problems that can influence speech, cause pain, and delay development. 2. Early childhood caries is promoted by inadequate preventive care, which can include diet, brushing, feeding habits, and lack of dental care. ECC is serious because young children with the condition are more likely to have continuing dental problems that can influence speech, cause pain, and delay development. 3. Early childhood caries is promoted by inadequate preventive care, which can include diet, brushing, feeding habits, and lack of dental care. ECC is serious because young children with the condition are more likely to have continuing dental problems that can influence speech, cause pain, and delay development. 4. Early childhood caries is promoted by inadequate preventive care, which can include diet, brushing, feeding habits, and lack of dental care. ECC is serious because young children with the condition are more likely to have continuing dental problems that can influence speech, cause pain, and delay development. 5. Early childhood caries is promoted by inadequate preventive care, which can include diet, brushing, feeding habits, and lack of dental care. ECC is serious because young children with the condition are more likely to have continuing dental problems that can influence speech, cause pain, and delay development. Page Ref: 172 Cognitive Level: Analyzing Client Need &Sub: Health Promotion and Maintenance Standards: QSEN Competencies: Patient-centered care | AACN Essential Competencies: Essential II: Basic organizational and systems leadership for quality care and patient safety | NLN Competencies: Nursing judgement | Nursing/Integrated Concepts: Nursing Process: Health teaching and health promotion Learning Outcome: LO 8.3 Plan health promotion and health maintenance strategies employed during health supervision visits of toddlers and preschoolers. MNL LO: Analyze socioeconomic factors that impact childhood nutrition. Principles of Pediatric Nursing: Caring for Children, 7e (Ball et al.) Chapter 9 Health Promotion and Maintenance for the School-Age Child and Adolescent 1) An adolescent female presents at a nurse practitioner's office and requests a signature for working papers. The nurse reviews her chart and notes that the last physical examination was two years ago. In addition to providing the signature for the working papers, what else should the nurse use this visit? 1. An opportunity to discuss birth-control measures 2. A time to discuss exercise and sports participation 3. A health-supervision opportunity 4. A chance to discuss the importance of pursuing post-secondary education Answer: 3 Explanation: 1. All visits should be used as health-promotion and health-supervision visits. While discussing birth control, exercise, and future plans is important, these can be included in the overall health-supervision protocols. 2. All visits should be used as health-promotion and health-supervision visits. While discussing birth control, exercise, and future plans is important, these can be included in the overall health- supervision protocols. 3. All visits should be used as health-promotion and health-supervision visits. While discussing birth control, exercise, and future plans is important, these can be included in the overall health- supervision protocols. 4. All visits should be used as health-promotion and health-supervision visits. While discussing birth control, exercise, and future plans is important, these can be included in the overall health- supervision protocols. Page Ref: 183 Cognitive Level: Applying Client Need &Sub: Health Promotion and Maintenance Standards: QSEN Competencies: Patient-centered care | AACN Essential Competencies: Essential II: Basic organizational and systems leadership for quality care and patient safety | NLN Competencies: Human flourishing | Nursing/Integrated Concepts: Nursing Process: Planning/Health teaching and health promotion Learning Outcome: LO 9.5 Synthesize data from history and examination of the school-age child and adolescent with knowledge of development to plan interventions appropriate during health supervision visits. MNL LO: Use the nursing process to provide developmentally appropriate care for the pediatric client. 2) An adolescent is accompanied by the mother for an annual physical examination. The nurse is aware of privacy issues related to the adolescent. While the mother is in the room, the nurse should avoid which questions? Select all that apply. 1. Sexual activity 2. Cigarette smoking 3. School performance 4. Use of alcohol 5. Car seatbelt use Answer: 1, 2, 4 Explanation: 1. The nurse must maintain the nurse—client relationship, which is between the nurse and the adolescent, and the nurse must maintain confidentiality. Therefore, the nurse cannot ask any personal questions while the mother is in the room, such as those related to sexual activity, drug and alcohol use, and smoking cigarettes. The nurse can ask general questions about seatbelt use and academic performance without breaching confidentiality. 2. The nurse must maintain the nurse—client relationship, which is between the nurse and the adolescent, and the nurse must maintain confidentiality. Therefore, the nurse cannot ask any personal questions while the mother is in the room, such as those related to sexual activity, drug and alcohol use, and smoking cigarettes. The nurse can ask general questions about seatbelt use and academic performance without breaching confidentiality. 3. The nurse must maintain the nurse—client relationship, which is between the nurse and the adolescent, and the nurse must maintain confidentiality. Therefore, the nurse cannot ask any personal questions while the mother is in the room, such as those related to sexual activity, drug and alcohol use, and smoking cigarettes. The nurse can ask general questions about seatbelt use and academic performance without breaching confidentiality. 4. The nurse must maintain the nurse—client relationship, which is between the nurse and the adolescent, and the nurse must maintain confidentiality. Therefore, the nurse cannot ask any personal questions while the mother is in the room, such as those related to sexual activity, drug and alcohol use, and smoking cigarettes. The nurse can ask general questions about seatbelt use and academic performance without breaching confidentiality. 5. The nurse must maintain the nurse—client relationship, which is between the nurse and the adolescent, and the nurse must maintain confidentiality. Therefore, the nurse cannot ask any personal questions while the mother is in the room, such as those related to sexual activity, drug and alcohol use, and smoking cigarettes. The nurse can ask general questions about seatbelt use and academic performance without breaching confidentiality. Page Ref: 184 Cognitive Level: Applying Client Need &Sub: Health Promotion and Maintenance Standards: QSEN Competencies: Patient-centered care | AACN Essential Competencies: Essential II: Basic organizational and systems leadership for quality care and patient safety | NLN Competencies: Nursing judgement | Nursing/Integrated Concepts: Nursing Process: Implementation/Health teaching and health promotion Learning Outcome: LO 9.3 Apply communication skills to interactions with school-age children, adolescents, and their families. MNL LO: Implement developmentally appropriate communication strategies for pediatric clients and families. 3) The school nurse performs screenings on all students in the middle school. In addition, the nurse will perform selected screenings on individual school-age children. When planning the screenings for the year, which screenings will the nurse include for all school-age children? Select all that apply. 1. Hearing 2. Height and weight 3. Blood-pressure measurement 4. Hepatitis B profile serology 5. Chest x-ray Answer: 1, 2, 3 Explanation: 1. Routine screening for school-age children include hearing, checking for height and weight, and blood-pressure measurements. The hepatitis B profile is only needed once, prior to administration of hepatitis B vaccine; however, this is not a required screening for all school- age children. A chest x-ray is not a routine screening test for school-age children. 2. Routine screening for school-age children include hearing, checking for height and weight, and blood-pressure measurements. The hepatitis B profile is only needed once, prior to administration of hepatitis B vaccine; however, this is not a required screening for all school-age children. A chest x-ray is not a routine screening test for school-age children. 3. Routine screening for school-age children include hearing, checking for height and weight, and blood-pressure measurements. The hepatitis B profile is only needed once, prior to administration of hepatitis B vaccine; however, this is not a required screening for all school-age children. A chest x-ray is not a routine screening test for school-age children. 4. Routine screening for school-age children include hearing, checking for height and weight, and blood-pressure measurements. The hepatitis B profile is only needed once, prior to administration of hepatitis B vaccine; however, this is not a required screening for all school-age children. A chest x-ray is not a routine screening test for school-age children. 5. Routine screening for school-age children include hearing, checking for height and weight, and blood-pressure measurements. The hepatitis B profile is only needed once, prior to administration of hepatitis B vaccine; however, this is not a required screening for all school-age children. A chest x-ray is not a routine screening test for school-age children. Page Ref: 185 Cognitive Level: Applying Client Need &Sub: Health Promotion and Maintenance Standards: QSEN Competencies: Patient-centered care | AACN Essential Competencies: Essential II: Basic organizational and systems leadership for quality care and patient safety | NLN Competencies: Nursing judgement | Nursing/Integrated Concepts: Nursing Process: Planning/Health teaching and health promotion Learning Outcome: LO 9.1 Identify the major health concerns of the school-age and adolescent years. MNL LO: Use the nursing process to provide developmentally appropriate care for the pediatric client. 4) A school nurse is performing annual height and weight screening. The nurse notes that three females who are close friends each lost 15 pounds over the past year. What is the priority nursing action in this situation? 1. Call the respective parents to discuss the eating patterns of each adolescent. 2. Speak with the girls in a group to discuss the problems associated with anorexia nervosa. 3. Refer these adolescents to the school psychologist. 4. Obtain a nutritional history for each of these adolescents. Answer: 4 Explanation: 1. The school nurse must evaluate why these three friends have all lost 15 pounds in one year. The best way to begin this assessment is to obtain a nutritional history for each client. Speaking with the parents would not be appropriate at this time. Discussing anorexia nervosa is too extreme, as is referring the adolescents to a school psychologist without performing a complete nursing assessment. 2. The school nurse must evaluate why these three friends have all lost 15 pounds in one year. The best way to begin this assessment is to obtain a nutritional history for each client. Speaking with the parents would not be appropriate at this time. Discussing anorexia nervosa is too extreme, as is referring the adolescents to a school psychologist without performing a complete nursing assessment. 3. The school nurse must evaluate why these three friends have all lost 15 pounds in one year. The best way to begin this assessment is to obtain a nutritional history for each client. Speaking with the parents would not be appropriate at this time. Discussing anorexia nervosa is too extreme, as is referring the adolescents to a school psychologist without performing a complete nursing assessment. 4. The school nurse must evaluate why these three friends have all lost 15 pounds in one year. The best way to begin this assessment is to obtain a nutritional history for each client. Speaking with the parents would not be appropriate at this time. Discussing anorexia nervosa is too extreme, as is referring the adolescents to a school psychologist without performing a complete nursing assessment. Page Ref: 185 Cognitive Level: Analyzing Client Need &Sub: Health Promotion and Maintenance Standards: QSEN Competencies: Patient-centered care | AACN Essential Competencies: Essential II: Basic organizational and systems leadership for quality care and patient safety | NLN Competencies: Nursing judgement | Nursing/Integrated Concepts: Nursing Process: Assessment/Health teaching and health promotion Learning Outcome: LO 9.4 Apply assessment skills to plan data-gathering methods for nutrition, physical activity, and mental health status of youth. MNL LO: Compare the nutritional needs of the infant, toddler, preschool-age child, school-age child, and adolescent. 5) A school-age client who recently moved to a new school in a different town presents to an ambulatory care center and describes the following: "I have no friends in my new school and I no longer want to play soccer. I know I will be lonely there, too." Which of these takes priority when speaking with the school-age client? 1. Helping the school-age client realize the value of soccer 2. Promoting healthy mental-health outcomes 3. Acknowledging the fact that it takes several months to make new friends at a new school 4. Stressing the importance of remaining in a close parent-child relationship during these stressful times Answer: 2 Explanation: 1. The school-age client is obviously lonely with the move to the new school. The nurse should focus on appropriate coping skills, which will enhance good mental-health outcomes for the child. It would not be appropriate to discuss the importance of soccer at this time, since the school-age client must deal with the loss of friends and developing new friendships first. The parent-child relationship should not be used as a substitute for the development of new peer relationships. 2. The school-age client is obviously lonely with the move to the new school. The nurse should focus on appropriate coping skills, which will enhance good mental-health outcomes for the child. It would not be appropriate to discuss the importance of soccer at this time, since the school-age client must deal with the loss of friends and developing new friendships first. The parent-child relationship should not be used as a substitute for the development of new peer relationships. 3. The school-age client is obviously lonely with the move to the new school. The nurse should focus on appropriate coping skills, which will enhance good mental-health outcomes for the child. It would not be appropriate to discuss the importance of soccer at this time, since the school-age client must deal with the loss of friends and developing new friendships first. The parent-child relationship should not be used as a substitute for the development of new peer relationships. 4. The school-age client is obviously lonely with the move to the new school. The nurse should focus on appropriate coping skills, which will enhance good mental-health outcomes for the child. It would not be appropriate to discuss the importance of soccer at this time, since the school-age client must deal with the loss of friends and developing new friendships first. The parent-child relationship should not be used as a substitute for the development of new peer relationships. Page Ref: 185, 186 Cognitive Level: Applying Client Need &Sub: Health Promotion and Maintenance Standards: QSEN Competencies: Patient-centered care | AACN Essential Competencies: Essential II: Basic organizational and systems leadership for quality care and patient safety | NLN Competencies: Nursing judgement | Nursing/Integrated Concepts: Nursing Process: Evaluation/Health teaching and health promotion Learning Outcome: LO 9.1 Identify the major health concerns of the school-age and adolescent years. MNL LO: Implement developmentally appropriate communication strategies for pediatric clients and families. 6) An adolescent reports participating in an exercise program at school each Wednesday throughout the school year. Further history reveals that the adolescent does not participate in any other physical activities. Which outcome is most appropriate for this adolescent? 1. The adolescent is reporting information consistent with what 60 percent of adolescents report as participation in physical activities. 2. The adolescent is not meeting the recommendations of the Healthy People 2020 initiative. 3. The adolescent should be encouraged to continue this program of exercise, since something is better than nothing. 4. The adolescent should be encouraged to vigorously exercise for at least 5 minutes each day. Answer: 2 Explanation: 1. In this scenario, the adolescent is not receiving the recommended amount of exercise to support good health habits. Encouraging the adolescent to continue as is or to exercise vigorously for 5 minutes each day also is not consistent with current recommendations. Suggesting that "something is better than nothing" is not good practice. 2. In this scenario, the adolescent is not receiving the recommended amount of exercise to support good health habits. Encouraging the adolescent to continue as is or to exercise vigorously for 5 minutes each day also is not consistent with current recommendations. Suggesting that "something is better than nothing" is not good practice. 3. In this scenario, the adolescent is not receiving the recommended amount of exercise to support good health habits. Encouraging the adolescent to continue as is or to exercise vigorously for 5 minutes each day also is not consistent with current recommendations. Suggesting that "something is better than nothing" is not good practice. 4. In this scenario, the adolescent is not receiving the recommended amount of exercise to support good health habits. Encouraging the adolescent to continue as is or to exercise vigorously for 5 minutes each day also is not consistent with current recommendations. Suggesting that "something is better than nothing" is not good practice. Page Ref: 185, 186 Cognitive Level: Applying Client Need &Sub: Health Promotion and Maintenance Standards: QSEN Competencies: Patient-centered care | AACN Essential Competencies: Essential II: Basic organizational and systems leadership for quality care and patient safety | NLN Competencies: Nursing judgement | Nursing/Integrated Concepts: Nursing Process: Evaluation/Health teaching and health promotion Learning Outcome: LO 9.5 Synthesize data from history and examination of the school-age child and adolescent with knowledge of development to plan interventions appropriate during health supervision visits. MNL LO: Evaluate healthcare issues related to pediatric nursing care. 7) An adolescent reports the following: "I get up at 6 a.m., I attend early-morning band classes three times each week, I play sports for 2 hours each day after school, and homework takes me 3 hours each night. I always feel tired." Which question by the nurse is most appropriate based on this information? 1. "How many hours of sleep do you get each night?" 2. "Do you consume foods high in iron?" 3. "Do you think you are doing too much?" 4. "Have you considered talking with your teachers about decreasing your homework, since you have so many extracurricular activities?" Answer: 1 Explanation: 1. The data in this scenario reveals very little time for sleep; therefore, the history should focus on sleep patterns. 2. The data in this scenario reveals very little time for sleep; therefore, the history should focus on sleep patterns. 3. The data in this scenario reveals very little time for sleep; therefore, the history should focus on sleep patterns. 4. The data in this scenario reveals very little time for sleep; therefore, the history should focus on sleep patterns. Page Ref: 185187 Cognitive Level: Analyzing Client Need &Sub: Health Promotion and Maintenance Standards: QSEN Competencies: Patient-centered care | AACN Essential Competencies: Essential II: Basic organizational and systems leadership for quality care and patient safety | NLN Competencies: Nursing judgement | Nursing/Integrated Concepts: Nursing Process: Assessment/Health teaching and health promotion Learning Outcome: LO 9.4 Apply assessment skills to plan data-gathering methods for nutrition, physical activity, and mental health status of youth. MNL LO: Evaluate healthcare issues related to pediatric nursing care. 8) The nurse working in the clinic includes an adolescent history in every client intake interview. Which issue should the nurse address when the parents are not present? 1. Possible domestic violence 2. Teen job responsibilities 3. Activities that are done as a family 4. The adolescent's role in the family Answer: 1 Explanation: 1. If domestic violence is suspected, it would only be appropriate to ask these questions when the teenager is alone with the nurse or healthcare provider. 2. If domestic violence is suspected, it would only be appropriate to ask these questions when the teenager is alone with the nurse or healthcare provider. 3. If domestic violence is suspected, it would only be appropriate to ask these questions when the teenager is alone with the nurse or healthcare provider. 4. If domestic violence is suspected, it would only be appropriate to ask these questions when the teenager is alone with the nurse or healthcare provider. Page Ref: 188, 189 Cognitive Level: Applying Client Need &Sub: Health Promotion and Maintenance Standards: QSEN Competencies: Patient-centered care | AACN Essential Competencies: Essential II: Basic organizational and systems leadership for quality care and patient safety | NLN Competencies: Nursing judgement | Nursing/Integrated Concepts: Nursing Process: Implementation/Health teaching and health promotion Learning Outcome: LO 9.3 Apply communication skills to interactions with school-age children, adolescents, and their families. MNL LO: Examine the role of the nurse in promoting culturally competent family-centered care. 9) The nurse is reviewing the immunization record of an adolescent who will be seen later in the day. Which item in the client's history makes hepatitis B status a priority? 1. Chronic acne 2. Overuse injuries from playing varsity sports 3. Chronic asthma 4. Plans to get a tattoo Answer: 4 Explanation: 1. The adolescent who is most at risk in the scenario presented is the teen who is planning on getting a tattoo. Adolescents with chronic acne or asthma do not have an increased risk for hepatitis B, since transmission has nothing to do with a diagnosis of acne. Overuse of muscles while playing sports is not related to development of hepatitis B. 2. The adolescent who is most at risk in the scenario presented is the teen who is planning on getting a tattoo. Adolescents with chronic acne or asthma do not have an increased risk for hepatitis B, since transmission has nothing to do with a diagnosis of acne. Overuse of muscles while playing sports is not related to development of hepatitis B. 3. The adolescent who is most at risk in the scenario presented is the teen who is planning on getting a tattoo. Adolescents with chronic acne or asthma do not have an increased risk for hepatitis B, since transmission has nothing to do with a diagnosis of acne. Overuse of muscles while playing sports is not related to development of hepatitis B. 4. The adolescent who is most at risk in the scenario presented is the teen who is planning on getting a tattoo. Adolescents with chronic acne or asthma do not have an increased risk for hepatitis B, since transmission has nothing to do with a diagnosis of acne. Overuse of muscles while playing sports is not related to development of hepatitis B. Page Ref: 191, 192 Cognitive Level: Applying Client Need &Sub: Health Promotion and Maintenance Standards: QSEN Competencies: Patient-centered care | AACN Essential Competencies: Essential VII: Clinical prevention and population health | NLN Competencies: Human flourishing | Nursing/Integrated Concepts: Nursing Process: Evaluation/Health teaching and health promotion Learning Outcome: LO 9.1 Identify the major health concerns of the school-age and adolescent years. MNL LO: Demonstrate safe medication administration for the pediatric client. 10) An obese adolescent who adamantly denies sexual activity has a positive pregnancy test, which was performed in the adolescent clinic. Which statement by the nurse is the most appropriate in this situation? 1. "Tell me how you feel about your body image." 2. "When was your last menstrual period (LMP)?" 3. "Let's discuss some activities that you have done within the past few months that could possibly lead to pregnancy." 4. "Were you involved in a date rape and are you hesitant to speak about it?" Answer: 3 Explanation: 1. The nurse must help the adolescent realize that previous behaviors have led to a positive pregnancy test. The only response by the nurse that will accomplish this goal is for the nurse to ask a direct question in which the nurse and client search for an answer. 2. The nurse must help the adolescent realize that previous behaviors have led to a positive pregnancy test. The only response by the nurse that will accomplish this goal is for the nurse to ask a direct question in which the nurse and client search for an answer. 3. The nurse must help the adolescent realize that previous behaviors have led to a positive pregnancy test. The only response by the nurse that will accomplish this goal is for the nurse to ask a direct question in which the nurse and client search for an answer. 4. The nurse must help the adolescent realize that previous behaviors have led to a positive pregnancy test. The only response by the nurse that will accomplish this goal is for the nurse to ask a direct question in which the nurse and client search for an answer. Page Ref: 189 Cognitive Level: Applying Client Need &Sub: Health Promotion and Maintenance Standards: QSEN Competencies: Patient-centered care | AACN Essential Competencies: Essential VII: Clinical prevention and population health | NLN Competencies: Nursing judgement | Nursing/Integrated Concepts: Nursing Process: Implementation/Health teaching and health promotion Learning Outcome: LO 9.4 Apply assessment skills to plan data-gathering methods for nutrition, physical activity, and mental health status of youth. MNL LO: Implement developmentally appropriate communication strategies for pediatric clients and families. 11) A mother reports that her adolescent is always late. The mother states, "She was born late and has been late every day of her life." Which response should the nurse make to this mother? 1. "You need to establish specific time frames for your adolescent and be certain she adheres to them." 2. "You should not expect your adolescent to be an 'on-time' individual unless you set specific alarms and then reinforce the value of being 'on-time.'" 3. "You should not expect your adolescent to be on time. Teenagers are always late." 4. "You have a major problem. There must be a lot of screaming in your home." Answer: 2 Explanation: 1. The best response is to help the mother find a way to help the teen deal with the problem of lateness. The other responses will either create parent-child conflict or make assumptions about household communication. 2. The best response is to help the mother find a way to help the teen deal with the problem of lateness. The other responses will either create parent-child conflict or make assumptions about household communication. 3. The best response is to help the mother find a way to help the teen deal with the problem of lateness. The other responses will either create parent-child conflict or make assumptions about household communication. 4. The best response is to help the mother find a way to help the teen deal with the problem of lateness. The other responses will either create parent-child conflict or make assumptions about household communication. Page Ref: 190, 191 Cognitive Level: Applying Client Need &Sub: Health Promotion and Maintenance Standards: QSEN Competencies: Patient-centered care | AACN Essential Competencies: Essential II: Basic organizational and systems leadership for quality care and patient safety | NLN Competencies: Nursing judgement | Nursing/Integrated Concepts: Nursing Process: Implementation/Coordination of care Learning Outcome: LO 9.3 Apply communication skills to interactions with school-age children, adolescents, and their families. MNL LO: Implement developmentally appropriate communication strategies for pediatric clients and families. 12) When examining a 7-year-old, which action by the nurse would be most appropriate? 1. Allow the child to participate in the exam. 2. Ask the parent what kind of food the child likes to eat. 3. Ask the child whether he plays outside for at least 30 minutes a day. 4. Allow the child to decide whether he is ready for his next immunization. Answer: 1 Explanation: 1. At this age, children have logical thought, and are learning about their bodies. Participating in the physical exam is appropriate for this age. The child can answer the question about food intake himself. Asking whether he plays outside for 30 minutes is fine, but children at this age need at least 60 minutes of activity, so the question will not gather appropriate information. It is not the child's decision whether he is ready for immunization, so do not ask this question. 2. At this age, children have logical thought, and are learning about their bodies. Participating in the physical exam is appropriate for this age. The child can answer the question about food intake himself. Asking whether he plays outside for 30 minutes is fine, but children at this age need at least 60 minutes of activity, so the question will not gather appropriate information. It is not the child's decision whether he is ready for immunization, so do not ask this question. 3. At this age, children have logical thought, and are learning about their bodies. Participating in the physical exam is appropriate for this age. The child can answer the question about food intake himself. Asking whether he plays outside for 30 minutes is fine, but children at this age need at least 60 minutes of activity, so the question will not gather appropriate information. It is not the child's decision whether he is ready for immunization, so do not ask this question. 4. At this age, children have logical thought, and are learning about their bodies. Participating in the physical exam is appropriate for this age. The child can answer the question about food intake himself. Asking whether he plays outside for 30 minutes is fine, but children at this age need at least 60 minutes of activity, so the question will not gather appropriate information. It is not the child's decision whether he is ready for immunization, so do not ask this question. Page Ref: 191, 192 Cognitive Level: Applying Client Need &Sub: Health Promotion and Maintenance Standards: QSEN Competencies: Patient-centered care | AACN Essential Competencies: Essential II: Basic organizational and systems leadership for quality care and patient safety | NLN Competencies: Human flourishing | Nursing/Integrated Concepts: Nursing Process: Implementation/Health teaching and health promotion Learning Outcome: LO 9.5 Synthesize data from history and examination of the school-age child and adolescent with knowledge of development to plan interventions appropriate during health supervision visits. MNL LO: Differentiate developmentally appropriate care environments for the pediatric client and family. 13) The school health nurse recognizes that children who display certain characteristics are at risk for poor school performance. The nurse will, therefore, observe each school-age child for which characteristics? Select all that apply. 1. Decreased ability to perform visual tracking. 2. Decreased auditory stimulation. 3. Decreased muscle tone. 4. Multiple dental caries. 5. Chronic tonsillitis. Answer: 1, 2, 3 Explanation: 1. Children with vision, hearing, and muscle tone problems are at risk for poor school performance, since most school activities involve listening, seeing, and kinetic activity. School performance most likely would not be affected by dental caries and chronic tonsillitis. 2. Children with vision, hearing, and muscle tone problems are at risk for poor school performance, since most school activities involve listening, seeing, and kinetic activity. School performance most likely would not be affected by dental caries and chronic tonsillitis. 3. Children with vision, hearing, and muscle tone problems are at risk for poor school performance, since most school activities involve listening, seeing, and kinetic activity. School performance most likely would not be affected by dental caries and chronic tonsillitis. 4. Children with vision, hearing, and muscle tone problems are at risk for poor school performance, since most school activities involve listening, seeing, and kinetic activity. School performance most likely would not be affected by dental caries and chronic tonsillitis. 5. Children with vision, hearing, and muscle tone problems are at risk for poor school performance, since most school activities involve listening, seeing, and kinetic activity. School performance most likely would not be affected by dental caries and chronic tonsillitis. Page Ref: 189, 190 Cognitive Level: Analyzing Client Need &Sub: Health Promotion and Maintenance Standards: QSEN Competencies: Quality Improvement | AACN Essential Competencies: Essential IX: Baccalaureate generalist nursing practice | NLN Competencies: Professional identity | Nursing/Integrated Concepts: Nursing Process: Assessment/Quality of practice Learning Outcome: LO 9.1 Identify the major health concerns of the school-age and adolescent years. MNL LO: Integumentary and Musculoskeletal Disorders/Examine etiology, risk factors, pathophysiology, and clinical manifestations as seen in children. 14) A nurse obtains a nutritional health history from a 10-year-old child. Which of these food selections, if consumed on a regular basis, should lead the nurse to become concerned about the need for improving oral hygiene? 1. Peanuts and crackers 2. Sorbet and yogurt 3. Gummy bears and licorice 4. Fluoridated water Answer: 3 Explanation: 1. Food items that stick to the teeth lead to dental caries. Items such as gummy bears and licorice all stick to the teeth and lead to dental caries. Foods such as peanut butter, crackers, sorbet, and yogurt do not stick to the teeth and are not considered foods that increase dental caries. Fluoridated water has been shown to decrease the incidence of dental caries. 2. Food items that stick to the teeth lead to dental caries. Items such as gummy bears and licorice all stick to the teeth and lead to dental caries. Foods such as peanut butter, crackers, sorbet, and yogurt do not stick to the teeth and are not considered foods that increase dental caries. Fluoridated water has been shown to decrease the incidence of dental caries. 3. Food items that stick to the teeth lead to dental caries. Items such as gummy bears and licorice all stick to the teeth and lead to dental caries. Foods such as peanut butter, crackers, sorbet, and yogurt do not stick to the teeth and are not considered foods that increase dental caries. Fluoridated water has been shown to decrease the incidence of dental caries. 4. Food items that stick to the teeth lead to dental caries. Items such as gummy bears and licorice all stick to the teeth and lead to dental caries. Foods such as peanut butter, crackers, sorbet, and yogurt do not stick to the teeth and are not considered foods that increase dental caries. Fluoridated water has been shown to decrease the incidence of dental caries. Page Ref: 185 Cognitive Level: Analyzing Client Need &Sub: Health Promotion and Maintenance Standards: QSEN Competencies: Patient-centered care | AACN Essential Competencies: Essential II: Basic organizational and systems leadership for quality care and patient safety | NLN Competencies: Human flourishing | Nursing/Integrated Concepts: Nursing Process: Evaluation/Health teaching and health promotion Learning Outcome: LO 9.4 Apply assessment skills to plan data-gathering methods for nutrition, physical activity, and mental health status of youth. MNL LO: Relate nutrition to the prevention of illness and the maintenance of child health and wellness. 15) The school health nurse is evaluating the home environment of several children as it relates to child safety. The nurse visits the home of each child and gathers the following data. Which activity places a child at greatest risk for bodily harm? 1. The parents are in a methadone program. 2. The parents consume alcohol on a daily basis. 3. The child is permitted to target practice with a revolver, unsupervised. 4. The child is a latchkey child. Answer: 3 Explanation: 1. Of all the activities mentioned, the child who is playing with guns is most at risk for injury. The inappropriate behaviors, such as drug and alcohol use or past use, also place the child at risk, but the use of firearms is more risky. A latchkey child needs special attention but in regard to the situations given is not at the greatest risk of injury. 2. Of all the activities mentioned, the child who is playing with guns is most at risk for injury. The inappropriate behaviors, such as drug and alcohol use or past use, also place the child at risk, but the use of firearms is more risky. A latchkey child needs special attention but in regard to the situations given is not at the greatest risk of injury. 3. Of all the activities mentioned, the child who is playing with guns is most at risk for injury. The inappropriate behaviors, such as drug and alcohol use or past use, also place the child at risk, but the use of firearms is more risky. A latchkey child needs special attention but in regard to the situations given is not at the greatest risk of injury. 4. Of all the activities mentioned, the child who is playing with guns is most at risk for injury. The inappropriate behaviors, such as drug and alcohol use or past use, also place the child at risk, but the use of firearms is more risky. A latchkey child needs special attention but in regard to the situations given is not at the greatest risk of injury. Page Ref: 190, 191 Cognitive Level: Analyzing Client Need &Sub: Health Promotion and Maintenance Standards: QSEN Competencies: Safety | AACN Essential Competencies: Essential II: Basic organizational and systems leadership for quality care and patient safety | NLN Competencies: Nursing judgement | Nursing/Integrated Concepts: Nursing Process: Assessment/Quality of practice Learning Outcome: LO 9.6 Plan with school-age children and adolescents to help them integrate activities to promote health and to prevent disease and injury. MNL LO: Apply age-specific mortality and morbidity data to care of children. 16) The school nurse is teaching a class about safety. The nurse will teach the children that they should wear protective athletic gear when participating in selected activities. Which of these activities require protective athletic gear? Select all that apply. 1. Skateboarding 2. Playing football 3. Swimming 4. Playing lacrosse 5. Performing acrobatic tricks Answer: 1, 2, 4 Explanation: 1. Any sport that includes body contact requires a child to wear protective equipment. These include skateboarding, football, and lacrosse. Swimming and acrobatics do not have any requirements for protective equipment. 2. Any sport that includes body contact requires a child to wear protective equipment. These include skateboarding, football, and lacrosse. Swimming and acrobatics do not have any requirements for protective equipment. 3. Any sport that includes body contact requires a child to wear protective equipment. These include skateboarding, football, and lacrosse. Swimming and acrobatics do not have any requirements for protective equipment. 4. Any sport that includes body contact requires a child to wear protective equipment. These include skateboarding, football, and lacrosse. Swimming and acrobatics do not have any requirements for protective equipment. 5. Any sport that includes body contact requires a child to wear protective equipment. These include skateboarding, football, and lacrosse. Swimming and acrobatics do not have any requirements for protective equipment. Page Ref: 192 Cognitive Level: Applying Client Need &Sub: Health Promotion and Maintenance Standards: QSEN Competencies: Safety | AACN Essential Competencies: Essential II: Basic organizational and systems leadership for quality care and patient safety | NLN Competencies: Nursing judgement | Nursing/Integrated Concepts: Nursing Process: Evaluation/Quality of practice Learning Outcome: LO 9.5 Synthesize data from history and examination of the school-age child and adolescent with knowledge of development to plan interventions appropriate during health supervision visits. MNL LO: Evaluate healthcare issues related to pediatric nursing care. 17) A 9-year-old child who has been followed in the same pediatric home since birth is at the healthcare center for a well-child visit. A nurse who measures the height and weight of the child documents 35th percentile for height and 90th percentile for weight. How should the nurse interpret these data? 1. The child is beginning a growth spurt. 2. The child is obese and needs dietary counseling. 3. The parents are most likely below the 50th percentile for height and weight. 4. As soon as the child begins the adolescent growth spurt, the height and weight measurements will normalize. Answer: 2 Explanation: 1. These data show that the child is disproportionate in height and weight. This child's weight is very high in comparison to height. The child would appear obese. Dietary history and counseling are the first steps. This child may also need an endocrine evaluation. This is not a growth spurt since height is what is referred to as a growth spurt. No assumptions about the parents can be made from the data presented. The statement about the adolescent growth spurt is incorrect for a child of this age. 2. These data show that the child is disproportionate in height and weight. This child's weight is very high in comparison to height. The child would appear obese. Dietary history and counseling are the first steps. This child may also need an endocrine evaluation. This is not a growth spurt since height is what is referred to as a growth spurt. No assumptions about the parents can be made from the data presented. The statement about the adolescent growth spurt is incorrect for a child of this age. 3. These data show that the child is disproportionate in height and weight. This child's weight is very high in comparison to height. The child would appear obese. Dietary history and counseling are the first steps. This child may also need an endocrine evaluation. This is not a growth spurt since height is what is referred to as a growth spurt. No assumptions about the parents can be made from the data presented. The statement about the adolescent growth spurt is incorrect for a child of this age. 4. These data show that the child is disproportionate in height and weight. This child's weight is very high in comparison to height. The child would appear obese. Dietary history and counseling are the first steps. This child may also need an endocrine evaluation. This is not a growth spurt since height is what is referred to as a growth spurt. No assumptions about the parents can be made from the data presented. The statement about the adolescent growth spurt is incorrect for a child of this age. Page Ref: 185, 186 Cognitive Level: Analyzing Client Need &Sub: Health Promotion and Maintenance Standards: QSEN Competencies: Patient-centered care | AACN Essential Competencies: Essential VII: Clinical prevention and population health | NLN Competencies: Nursing judgement | Nursing/Integrated Concepts: Nursing Process: Evaluation/Health teaching and health promotion Learning Outcome: LO 9.5 Synthesize data from history and examination of the school-age child and adolescent with knowledge of development to plan interventions appropriate during health supervision visits. MNL LO: Examine the components of performing a nutritional assessment of the pediatric client. 18) The nurse is preparing to complete a health surveillance appointment with a school-age client and parents. Which observations would necessitate the need for further assessment by the nurse? Select all that apply. 1. Client who does not make eye contact 2. Client with visible bruises in various stages of healing 3. Client holding a video game talking with parent 4. Client playing a card game with sibling 5. Client who appears red in the face while walking to exam room Answer: 1, 2, 5 Explanation: 1. Nursing assessment begins with the first encounter with the client and the family. The nurse would want to further explore a client who does not make eye contact, who has bruises in various stages of healing, and a client who appears red in the face while walking to the exam room. All of these items may be clues to emotional issues, physical violence, and health related issues, such as hypertension. A client who is holding a video game and talking to the parent and a client who is playing a card game with a sibling are not observations that are abnormal for the school-age client. 2. Nursing assessment begins with the first encounter with the client and the family. The nurse would want to further explore a client who does not make eye contact, who has bruises in various stages of healing, and a client who appears red in the face while walking to the exam room. All of these items may be clues to emotional issues, physical violence, and health related issues, such as hypertension. A client who is holding a video game and talking to the parent and a client who is playing a card game with a sibling are not observations that are abnormal for the school-age client. 3. Nursing assessment begins with the first encounter with the client and the family. The nurse would want to further explore a client who does not make eye contact, who has bruises in various stages of healing, and a client who appears red in the face while walking to the exam room. All of these items may be clues to emotional issues, physical violence, and health related issues, such as hypertension. A client who is holding a video game and talking to the parent and a client who is playing a card game with a sibling are not observations that are abnormal for the school-age client. 4. Nursing assessment begins with the first encounter with the client and the family. The nurse would want to further explore a client who does not make eye contact, who has bruises in various stages of healing, and a client who appears red in the face while walking to the exam room. All of these items may be clues to emotional issues, physical violence, and health related issues, such as hypertension. A client who is holding a video game and talking to the parent and a client who is playing a card game with a sibling are not observations that are abnormal for the school-age client. 5. Nursing assessment begins with the first encounter with the client and the family. The nurse would want to further explore a client who does not make eye contact, who has bruises in various stages of healing, and a client who appears red in the face while walking to the exam room. All of these items may be clues to emotional issues, physical violence, and health related issues, such as hypertension. A client who is holding a video game and talking to the parent and a client who is playing a card game with a sibling are not observations that are abnormal for the school-age client. Page Ref: 184, 185 Cognitive Level: Analyzing Client Need &Sub: Health Promotion and Maintenance Standards: QSEN Competencies: Patient-centered care | AACN Essential Competencies: Essential II: Basic organizational and systems leadership for quality care and patient safety | NLN Competencies: Nursing judgement | Nursing/Integrated Concepts: Nursing Process: Assessment/Health teaching and health promotion Learning Outcome: LO 9.2 Describe the general observations made of youth and their families as they come to the pediatric healthcare home for health supervision visits. MNL LO: Examine the role of the nurse in promoting culturally competent family-centered care. 19) The nurse is teaching the adolescent and family about sleep hygiene. What behaviors should the nurse suggest? Select all that apply. 1. Avoid naps in the late afternoon and evening 2. Sleep 12 hours a day 3. Avoid caffeine, tea, coffee, carbonated beverages and energy drinks for several hours before sleep. 4. Avoid setting an alarm clock 5. Go to bed and get up at the same time each day, including weekends Answer: 1, 3, 5 Explanation: 1. General information about sleep includes no drinks or food with stimulants, go to bed and get up at the same time each day, including weekends, and avoid naps in the late afternoon and evening. 2. General information about sleep includes no drinks or food with stimulants, go to bed and get up at the same time each day, including weekends, and avoid naps in the late afternoon and evening. Sleeping 12 hours is not required—sleep is not made up if it is lost. 3. General information about sleep includes no drinks or food with stimulants, go to bed and get up at the same time each day, including weekends, and avoid naps in the late afternoon and evening. 4. General information about sleep includes no drinks or food with stimulants, go to bed and get up at the same time each day, including weekends, and avoid naps in the late afternoon and evening. Avoiding setting an alarm clock does not help the adolescent get and maintain good restful sleep. 5. General information about sleep includes no drinks or food with stimulants, go to bed and get up at the same time each day, including weekends, and avoid naps in the late afternoon and evening. Page Ref: 189 Cognitive Level: Analyzing Client Need &Sub: Health Promotion and Maintenance Standards: QSEN Competencies: Patient-centered care | AACN Essential Competencies: Essential VII: Clinical prevention and population health | NLN Competencies: Nursing judgement | Nursing/Integrated Concepts: Nursing Process: Planning/Health teaching and health promotion Learning Outcome: LO 9.5 Synthesize data from history and examination of the school-age child and adolescent with knowledge of development to plan interventions appropriate during health supervision visits. MNL LO: Develop a family-centered nursing care plan for the child and family. 20) When reviewing the adolescent health record, which immunizations should the nurse encourage? Select all that apply. 1. Varicella 2. Human papillomavirus 3. HIV 4. Cholesterol 5. Hepatitis B Answer: 1, 2, 5 Explanation: 1. When identifying immunizations needed by the adolescent some of the questions to ask would be: When was the last tetanus-diphtheria (Td) booster? Was a second measles-mumps-rubella administered? Is hepatitis A common in your state? Has the youth had hepatitis B vaccine? Did the youth have a documented history of varicella disease? Has the youth received meningococcal vaccine? Have the adolescent female and male received the human papillomavirus vaccine? Has the youth received the annual influenza vaccine? 2. When identifying immunizations needed by the adolescent some of the questions to ask would be: When was the last tetanus-diphtheria (Td) booster? Was a second measles-mumps-rubella administered? Is hepatitis A common in your state? Has the youth had hepatitis B vaccine? Did the youth have a documented history of varicella disease? Has the youth received meningococcal vaccine? Have the adolescent female and male received the human papillomavirus vaccine? Has the youth received the annual influenza vaccine? 3. When identifying immunizations needed by the adolescent some of the questions to ask would be: When was the last tetanus-diphtheria (Td) booster? Was a second measles-mumps-rubella administered? Is hepatitis A common in your state? Has the youth had hepatitis B vaccine? Did the youth have a documented history of varicella disease? Has the youth received meningococcal vaccine? Have the adolescent female and male received the human papillomavirus vaccine? Has the youth received the annual influenza vaccine? HIV is not needed unless the adolescent is sexually active with same sex partner or many partners or has symptoms. 4. When identifying immunizations needed by the adolescent some of the questions to ask would be: When was the last tetanus-diphtheria (Td) booster? Was a second measles-mumps-rubella administered? Is hepatitis A common in your state? Has the youth had hepatitis B vaccine? Did the youth have a documented history of varicella disease? Has the youth received meningococcal vaccine? Have the adolescent female and male received the human papillomavirus vaccine? Has the youth received the annual influenza vaccine? Cholesterol is not needed unless the adolescent eats a high fat diet or there is a family history of high cholesterol. 5. When identifying immunizations needed by the adolescent some of the questions to ask would be: When was the last tetanus-diphtheria (Td) booster? Was a second measles-mumps-rubella administered? Is hepatitis A common in your state? Has the youth had hepatitis B vaccine? Did the youth have a documented history of varicella disease? Has the youth received meningococcal vaccine? Have the adolescent female and male received the human papillomavirus vaccine? Has the youth received the annual influenza vaccine? Page Ref: 203 Cognitive Level: Analyzing Client Need &Sub: Health Promotion and Maintenance Standards: QSEN Competencies: Patient-centered care | AACN Essential Competencies: Essential VII: Clinical prevention and population health | NLN Competencies: Human flourishing | Nursing/Integrated Concepts: Nursing Process: Planning/Coordination of care Learning Outcome: LO 9.5 Synthesize data from history and examination of the school-age child and adolescent with knowledge of development to plan interventions appropriate during health supervision visits. MNL LO: Consider the characteristics and nursing management of immunizations during client teaching. Principles of Pediatric Nursing: Caring for Children, 7e (Ball et al.) Chapter 10 Nursing Considerations for the Child in the Community 1) The community-health nurse visits the child-care center. Which finding indicates the need for staff education? 1. A group of 2-year-olds are eating a snack of Cheerios. 2. Several 4-year-olds are outside playing on a slide. 3. An 18-month-old is pushing a toy truck. 4. A 2-month-old is sleeping in a crib on his stomach. Answer: 4 Explanation: 1. To decrease the incidence of sudden infant death syndrome (SIDS), infants should be placed on their backs to sleep. All of the other examples are developmentally appropriate activities for the specified age group. 2. To decrease the incidence of sudden infant death syndrome (SIDS), infants should be placed on their backs to sleep. All of the other examples are developmentally appropriate activities for the specified age group. 3. To decrease the incidence of sudden infant death syndrome (SIDS), infants should be placed on their backs to sleep. All of the other examples are developmentally appropriate activities for the specified age group. 4. To decrease the incidence of sudden infant death syndrome (SIDS), infants should be placed on their backs to sleep. All of the other examples are developmentally appropriate activities for the specified age group. Page Ref: 210 Cognitive Level: Analyzing Client Need &Sub: Safe and Effective Care Environment: Management of Care Standards: QSEN Competencies: Evidence-based practice | AACN Essential Competencies: Essential VII: Clinical prevention and population health | NLN Competencies: Professional identity | Nursing/Integrated Concepts: Nursing Process: Assessment/Health teaching and health promotion Learning Outcome: LO 10.2 Compare the roles of the nurse in each identified healthcare setting. MNL LO: Explore the practice of pediatric healthcare. 2) An adolescent client has a long leg cast secondary to a fractured femur. Which action by the nurse would effectively facilitate the adolescent's return to school? 1. Meet with teachers and administrators at the school to make sure entrances and classrooms are wheelchair accessible. 2. Develop an individualized health plan (IHP) that focuses on long-term needs of the adolescent. 3. Prior to the student's return to school, meet with all of the other students to emphasize the special needs of the injured teen. 4. Meet with parents of the injured student to encourage homebound schooling until a short leg cast is applied. Answer: 1 Explanation: 1. An adolescent with a long leg cast secondary to a fractured femur will be dependent on a wheelchair for mobility. It is essential that the environment be wheelchair accessible prior to the adolescent's return to school. While an IHP might be developed, short- term needs would be the focus. It is not necessary to meet all of the students to discuss the adolescent's needs. There is no reason to encourage the adolescent to stay at home for schooling if he is ready to return. 2. An adolescent with a long leg cast secondary to a fractured femur will be dependent on a wheelchair for mobility. It is essential that the environment be wheelchair accessible prior to the adolescent's return to school. While an IHP might be developed, short-term needs would be the focus. It is not necessary to meet all of the students to discuss the adolescent's needs. There is no reason to encourage the adolescent to stay at home for schooling if he is ready to return. 3. An adolescent with a long leg cast secondary to a fractured femur will be dependent on a wheelchair for mobility. It is essential that the environment be wheelchair accessible prior to the adolescent's return to school. While an IHP might be developed, short-term needs would be the focus. It is not necessary to meet all of the students to discuss the adolescent's needs. There is no reason to encourage the adolescent to stay at home for schooling if he is ready to return. 4. An adolescent with a long leg cast secondary to a fractured femur will be dependent on a wheelchair for mobility. It is essential that the environment be wheelchair accessible prior to the adolescent's return to school. While an IHP might be developed, short-term needs would be the focus. It is not necessary to meet all of the students to discuss the adolescent's needs. There is no reason to encourage the adolescent to stay at home for schooling if he is ready to return. Page Ref: 212 Cognitive Level: Applying Client Need &Sub: Physiological Integrity Standards: QSEN Competencies: Patient-centered care | AACN Essential Competencies: Essential II: Basic organizational and systems leadership for quality care and patient safety | NLN Competencies: Nursing judgement | Nursing/Integrated Concepts: Nursing Process: Planning/Coordination of care Learning Outcome: LO 10.4 Develop a nursing care plan for a child in the school setting who has short-term mobility limitations. MNL LO: Differentiate treatment options and strategies for continuity of care for the child and family. 3) The community-health nurse is planning an education session for recently hired teachers at a child-care center. Which item is priority for the community-health nurse to include in the educational session? 1. The schedule for immunizations 2. Principles of infection control 3. How to interpret healthcare records 4. How to take a temperature Answer: 2 Explanation: 1. While all of the information is nice to know, it is most essential that teachers know principles of infection control to decrease the spread of germs that can cause disease in young children. 2. While all of the information is nice to know, it is most essential that teachers know principles of infection control to decrease the spread of germs that can cause disease in young children. 3. While all of the information is nice to know, it is most essential that teachers know principles of infection control to decrease the spread of germs that can cause disease in young children. 4. While all of the information is nice to know, it is most essential that teachers know principles of infection control to decrease the spread of germs that can cause disease in young children. Page Ref: 209 Cognitive Level: Analyzing Client Need &Sub: Health Promotion and Maintenance Standards: QSEN Competencies: Teamwork and collaboration | AACN Essential Competencies: Essential VII: Clinical prevention and population health | NLN Competencies: Nursing judgement | Nursing/Integrated Concepts: Nursing Process: Planning/Health teaching and health promotion Learning Outcome: LO 10.2 Compare the roles of the nurse in each identified healthcare setting. MNL LO: Evaluate healthcare issues related to pediatric nursing care. 4) The nurse is providing care for several pediatric clients. Which client would require an Individualized Health Plan (IHP) prior to returning to school? 1. A school-age client who has recently developed a penicillin allergy 2. An adolescent client newly diagnosed with insulin-dependent diabetes mellitus 3. A school-age client who has been treated for head lice 4. An adolescent client who has missed two weeks of school due to mononucleosis Answer: 2 Explanation: 1. An IHP that ensures appropriate management of the child's healthcare needs must be developed for a child newly diagnosed with a chronic illness such as diabetes. A child who is allergic to penicillin will not receive this medication any longer and therefore should not encounter any problems related to it at school. A child who has been treated for head lice can return to school and does not need an IHP. While a child who has missed two weeks of school will need to make arrangements for makeup work, an IHP is not needed. 2. An IHP that ensures appropriate management of the child's healthcare needs must be developed for a child newly diagnosed with a chronic illness such as diabetes. A child who is allergic to penicillin will not receive this medication any longer and therefore should not encounter any problems related to it at school. A child who has been treated for head lice can return to school and does not need an IHP. While a child who has missed two weeks of school will need to make arrangements for makeup work, an IHP is not needed. 3. An IHP that ensures appropriate management of the child's healthcare needs must be developed for a child newly diagnosed with a chronic illness such as diabetes. A child who is allergic to penicillin will not receive this medication any longer and therefore should not encounter any problems related to it at school. A child who has been treated for head lice can return to school and does not need an IHP. While a child who has missed two weeks of school will need to make arrangements for makeup work, an IHP is not needed. 4. An IHP that ensures appropriate management of the child's healthcare needs must be developed for a child newly diagnosed with a chronic illness such as diabetes. A child who is allergic to penicillin will not receive this medication any longer and therefore should not encounter any problems related to it at school. A child who has been treated for head lice can return to school and does not need an IHP. While a child who has missed two weeks of school will need to make arrangements for makeup work, an IHP is not needed. Page Ref: 211 Cognitive Level: Analyzing Client Need &Sub: Safe and Effective Care Environment: Management of Care Standards: QSEN Competencies: Patient-centered care | AACN Essential Competencies: Essential IX: Baccalaureate generalist nursing practice | NLN Competencies: Professional identity | Nursing/Integrated Concepts: Nursing Process: Planning/Coordination of care Learning Outcome: LO 10.1 Discuss the community healthcare settings where nurses provide health services to children. MNL LO: Evaluate healthcare issues related to pediatric nursing care. 5) The school nurse is preparing a plan of care specific to several children in the school who have asthma. What is the initial action on the plan of care? 1. Call 911 to request emergency medical assistance. 2. Call the child's parents to come and pick up the child. 3. Have the child use his or her metered-dose inhaler. 4. Have the child lie down to see if the symptoms subside. Answer: 3 Explanation: 1. A child with a history of asthma may have episodes of wheezing that can be controlled by prompt use of the child's rescue inhaler. An inhaler should be readily available in the school setting for a child previously diagnosed with asthma. This should be tried first. Emergency personnel should be notified if the inhaler does not provide relief and the child is in respiratory distress. Parents may be notified if the child does not feel well, but this is not the initial action. Having the child lie down will likely worsen his condition. 2. A child with a history of asthma may have episodes of wheezing that can be controlled by prompt use of the child's rescue inhaler. An inhaler should be readily available in the school setting for a child previously diagnosed with asthma. This should be tried first. Emergency personnel should be notified if the inhaler does not provide relief and the child is in respiratory distress. Parents may be notified if the child does not feel well, but this is not the initial action. Having the child lie down will likely worsen his condition. 3. A child with a history of asthma may have episodes of wheezing that can be controlled by prompt use of the child's rescue inhaler. An inhaler should be readily available in the school setting for a child previously diagnosed with asthma. This should be tried first. Emergency personnel should be notified if the inhaler does not provide relief and the child is in respiratory distress. Parents may be notified if the child does not feel well, but this is not the initial action. Having the child lie down will likely worsen his condition. 4. A child with a history of asthma may have episodes of wheezing that can be controlled by prompt use of the child's rescue inhaler. An inhaler should be readily available in the school setting for a child previously diagnosed with asthma. This should be tried first. Emergency personnel should be notified if the inhaler does not provide relief and the child is in respiratory distress. Parents may be notified if the child does not feel well, but this is not the initial action. Having the child lie down will likely worsen his condition. Page Ref: 211, 212 Cognitive Level: Analyzing Client Need &Sub: Physiological Integrity: Reduction of Risk Potential Standards: QSEN Competencies: Patient-centered care | AACN Essential Competencies: Essential IX: Baccalaureate generalist nursing practice | NLN Competencies: Nursing judgement | Nursing/Integrated Concepts: Nursing Process: Implementation/Coordination of care Learning Outcome: LO 10.2 Compare the roles of the nurse in each identified healthcare setting. MNL LO: Apply the general concepts related to caring for a child with a chronic illness. 6) The telephone triage nurse receives a call from a parent who states that her 18-month-old is making a crowing sound when he breathes and is hard to wake up. Which action by the nurse is the most appropriate? 1. Obtain the history of the illness from the parent. 2. Advise the parent to hang up and call 9-1-1. 3. Make an appointment for the child to see the healthcare provider. 4. Reassure the parent and provide instructions on home care for the child. Answer: 2 Explanation: 1. The nurse should immediately recognize the symptoms of severe upper respiratory distress and advise the parent to call 9-1-1. Crowing is heard when there is severe narrowing of the airway. The other actions would be appropriate in nonemergency situations. 2. The nurse should immediately recognize the symptoms of severe upper respiratory distress and advise the parent to call 9-1-1. Crowing is heard when there is severe narrowing of the airway. The other actions would be appropriate in nonemergency situations. 3. The nurse should immediately recognize the symptoms of severe upper respiratory distress and advise the parent to call 9-1-1. Crowing is heard when there is severe narrowing of the airway. The other actions would be appropriate in nonemergency situations. 4. The nurse should immediately recognize the symptoms of severe upper respiratory distress and advise the parent to call 9-1-1. Crowing is heard when there is severe narrowing of the airway. The other actions would be appropriate in nonemergency situations. Page Ref: 209 Cognitive Level: Analyzing Client Need &Sub: Physiological Integrity: Reduction of Risk Potential Standards: QSEN Competencies: Patient-centered care | AACN Essential Competencies: Essential IX: Baccalaureate generalist nursing practice | NLN Competencies: Professional identity | Nursing/Integrated Concepts: Nursing Process: Assessment/Quality of practice Learning Outcome: LO 10.1 Discuss the community healthcare settings where nurses provide health services to children. MNL LO: Respiratory Disorders/Examine etiology, risk factors, pathophysiology, and clinical manifestations as seen in children. 7) A young school-age client who has had a tracheostomy for several years is scheduled to begin school in the fall. The teacher is concerned about this child's being in her class and consults the school nurse. Which action by the nurse is the most appropriate? 1. Make arrangements for the child to go to a special school. 2. Ask the parents of the child to provide a caregiver during school hours. 3. Recommend that the child be home schooled. 4. Teach the teacher how to care for the child in the classroom. Answer: 4 Explanation: 1. Section 504 of the Rehabilitation Act of 1973 guarantees access for children with disabilities to federally funded programs, including public schools. The child may need little extra attention while in the school setting, since he has had the tracheostomy for several years. The teacher should be taught how to care for the child if needed and the signs of distress. If needed, a health aide may be assigned to the child, but this is not the responsibility of the parents. 2. Section 504 of the Rehabilitation Act of 1973 guarantees access for children with disabilities to federally funded programs, including public schools. The child may need little extra attention while in the school setting, since he has had the tracheostomy for several years. The teacher should be taught how to care for the child if needed and the signs of distress. If needed, a health aide may be assigned to the child, but this is not the responsibility of the parents. 3. Section 504 of the Rehabilitation Act of 1973 guarantees access for children with disabilities to federally funded programs, including public schools. The child may need little extra attention while in the school setting, since he has had the tracheostomy for several years. The teacher should be taught how to care for the child if needed and the signs of distress. If needed, a health aide may be assigned to the child, but this is not the responsibility of the parents. 4. Section 504 of the Rehabilitation Act of 1973 guarantees access for children with disabilities to federally funded programs, including public schools. The child may need little extra attention while in the school setting, since he has had the tracheostomy for several years. The teacher should be taught how to care for the child if needed and the signs of distress. If needed, a health aide may be assigned to the child, but this is not the responsibility of the parents. Page Ref: 211, 212 Cognitive Level: Analyzing Client Need &Sub: Safe and Effective Care Environment: Management of Care Standards: QSEN Competencies: Teamwork and collaboration | AACN Essential Competencies: Essential IX: Baccalaureate generalist nursing practice | NLN Competencies: Professional identity | Nursing/Integrated Concepts: Nursing Process: Planning/Health teaching and health promotion Learning Outcome: LO 10.2 Compare the roles of the nurse in each identified healthcare setting. MNL LO: Respiratory Disorders/Examine etiology, risk factors, pathophysiology, and clinical manifestations as seen in children. 8) A 2-month-old infant with bronchopulmonary dysplasia (BPD) is being prepared for discharge from the neonatal intensive-care unit (NICU). The infant will continue to receive oxygen via nasal cannula at home. Prior to discharge, the home-health nurse assesses the home. Which finding poses the greatest risk to this infant? 1. Small toys strewn on the floor 2. A woodstove used for heating 3. A sibling who has an ear infection 4. Paint peeling on the walls Answer: 2 Explanation: 1. Assessment of the home environment is essential prior to discharge of a medically fragile infant. The use of a woodstove poses great risk to the infant who already has fragile lungs. Oxygen and woodstove heat will produce a flammable reaction. Small toy pieces and paint peeling from the wall will pose a choking risk to the older infant who is crawling. Ear infections are not contagious. 2. Assessment of the home environment is essential prior to discharge of a medically fragile infant. The use of a woodstove poses great risk to the infant who already has fragile lungs. Oxygen and woodstove heat will produce a flammable reaction. Small toy pieces and paint peeling from the wall will pose a choking risk to the older infant who is crawling. Ear infections are not contagious. 3. Assessment of the home environment is essential prior to discharge of a medically fragile infant. The use of a woodstove poses great risk to the infant who already has fragile lungs. Oxygen and woodstove heat will produce a flammable reaction. Small toy pieces and paint peeling from the wall will pose a choking risk to the older infant who is crawling. Ear infections are not contagious. 4. Assessment of the home environment is essential prior to discharge of a medically fragile infant. The use of a woodstove poses great risk to the infant who already has fragile lungs. Oxygen and woodstove heat will produce a flammable reaction. Small toy pieces and paint peeling from the wall will pose a choking risk to the older infant who is crawling. Ear infections are not contagious. Page Ref: 214, 215 Cognitive Level: Analyzing Client Need &Sub: Safe and Effective Care Environment: Safety and Infection Control Standards: QSEN Competencies: Patient-centered care | AACN Essential Competencies: Essential IX: Baccalaureate generalist nursing practice | NLN Competencies: Nursing judgement | Nursing/Integrated Concepts: Nursing Process: Assessment/Health teaching and health promotion Learning Outcome: LO 10.5 Examine five ways in which nurses assist families in the home healthcare setting. MNL LO: Respiratory Disorders/Differentiate treatment options and strategies for continuity of care for the child and family. 9) A child who is dependent on a ventilator is being discharged from the hospital. Prior to discharge, the home-health nurse discusses development of an emergency plan of care with the family. Which is the most essential part of the plan? 1. Acquisition of a backup generator 2. Designation of an emergency shelter site 3. Provision for an alternate heating source if power is lost 4. Notifying the power company that the child is on life support Answer: 1 Explanation: 1. Prior to discharge to home, it is essential that the family acquire a generator so that the child's life support will continue to function effectively should power be lost. While all other actions are very important, it is most essential that the ventilator has power to continue to function at all times. 2. Prior to discharge to home, it is essential that the family acquire a generator so that the child's life support will continue to function effectively should power be lost. While all other actions are very important, it is most essential that the ventilator has power to continue to function at all times. 3. Prior to discharge to home, it is essential that the family acquire a generator so that the child's life support will continue to function effectively should power be lost. While all other actions are very important, it is most essential that the ventilator has power to continue to function at all times. 4. Prior to discharge to home, it is essential that the family acquire a generator so that the child's life support will continue to function effectively should power be lost. While all other actions are very important, it is most essential that the ventilator has power to continue to function at all times. Page Ref: 214 Cognitive Level: Analyzing Client Need &Sub: Safe and Effective Care Environment: Safety and Infection Control Standards: QSEN Competencies: Patient-centered care | AACN Essential Competencies: Essential II: Basic organizational and systems leadership for quality care and patient safety | NLN Competencies: Professional identity | Nursing/Integrated Concepts: Nursing Process: Planning/Health teaching and health promotion Learning Outcome: LO 10.6 Summarize the special developmental needs of children to consider in disaster preparedness planning. MNL LO: Respiratory Disorders/Differentiate treatment options and strategies for continuity of care for the child and family. 10) What must a home-health nurse realize prior to accepting an assignment? 1. All decisions will be made by the healthcare provider. 2. The family will adapt their lifestyle to the needs of the nurse. 3. Independent decisions regarding emergency care of the child will be made by the nurse. 4. The family is in charge. Answer: 4 Explanation: 1. The home-health nurse must realize that the family is in charge. The nurse must be flexible and adaptable to the lifestyle of the family. The family must provide informed consent for emergency care. 2. The home-health nurse must realize that the family is in charge. The nurse must be flexible and adaptable to the lifestyle of the family. The family must provide informed consent for emergency care. 3. The home-health nurse must realize that the family is in charge. The nurse must be flexible and adaptable to the lifestyle of the family. The family must provide informed consent for emergency care. 4. The home-health nurse must realize that the family is in charge. The nurse must be flexible and adaptable to the lifestyle of the family. The family must provide informed consent for emergency care. Page Ref: 214, 215 Cognitive Level: Applying Client Need &Sub: Safe and Effective Care Environment: Management of Care Standards: QSEN Competencies: Patient-centered care | AACN Essential Competencies: Essential IX: Baccalaureate generalist nursing practice | NLN Competencies: Professional identity | Nursing/Integrated Concepts: Nursing Process: Planning/Quality of practice Learning Outcome: LO 10.5 Examine five ways in which nurses assist families in the home healthcare setting. MNL LO: Explore the practice of pediatric healthcare. 11) Which aspect of an Emergency Medical Services (EMS) system is most indicative that EMS providers are prepared to provide emergency care to children? 1. Placement of small stretchers in emergency vehicles 2. Lists of hospitals in the area that treat children 3. Staff education related to assessment and treatment of children of all ages 4. Pediatric-sized equipment and supplies Answer: 3 Explanation: 1. While size-appropriate equipment and lists of hospitals that treat children are essential parts of an EMS system, the aspect that is most indicative that EMS providers are actually prepared to take care of children is evidence of education related to assessment and emergency treatment. 2. While size-appropriate equipment and lists of hospitals that treat children are essential parts of an EMS system, the aspect that is most indicative that EMS providers are actually prepared to take care of children is evidence of education related to assessment and emergency treatment. 3. While size-appropriate equipment and lists of hospitals that treat children are essential parts of an EMS system, the aspect that is most indicative that EMS providers are actually prepared to take care of children is evidence of education related to assessment and emergency treatment. 4. While size-appropriate equipment and lists of hospitals that treat children are essential parts of an EMS system, the aspect that is most indicative that EMS providers are actually prepared to take care of children is evidence of education related to assessment and emergency treatment. Page Ref: 212 Cognitive Level: Analyzing Client Need &Sub: Safe and Effective Care Environment: Safety and Infection Control Standards: QSEN Competencies: Patient-centered care | AACN Essential Competencies: Essential II: Basic organizational and systems leadership for quality care and patient safety | NLN Competencies: Nursing judgement | Nursing/Integrated Concepts: Nursing Process: Evaluation/Education Learning Outcome: LO 10.6 Summarize the special developmental needs of children to consider in disaster preparedness planning. MNL LO: Explore the practice of pediatric healthcare. 12) The number of serious injuries in children has doubled in the past year. Based on this information, which is the most appropriate community nursing diagnosis? 1. Noncompliance Related to Inappropriate Use of Child Safety Seats 2. Risk for Injury Related to Inadequate Use of Bicycle Helmets 3. Altered Family Processes Related to Hospitalization of an Injured Child 4. Knowledge Deficit Related to Injury Prevention in Children Answer: 4 Explanation: 1. All of these diagnoses might be appropriate in specific situations, but Knowledge Deficit Related to Injury Prevention in Children is the only one that is general to the problem as a whole and is therefore the most appropriate community nursing diagnosis. 2. All of these diagnoses might be appropriate in specific situations, but Knowledge Deficit Related to Injury Prevention in Children is the only one that is general to the problem as a whole and is therefore the most appropriate community nursing diagnosis. 3. All of these diagnoses might be appropriate in specific situations, but Knowledge Deficit Related to Injury Prevention in Children is the only one that is general to the problem as a whole and is therefore the most appropriate community nursing diagnosis. 4. All of these diagnoses might be appropriate in specific situations, but Knowledge Deficit Related to Injury Prevention in Children is the only one that is general to the problem as a whole and is therefore the most appropriate community nursing diagnosis. Page Ref: 210 Cognitive Level: Analyzing Client Need &Sub: Safe and Effective Care Environment: Safety and Infection Control Standards: QSEN Competencies: Safety | AACN Essential Competencies: Essential IX: Baccalaureate generalist nursing practice | NLN Competencies: Nursing judgement | Nursing/Integrated Concepts: Nursing Process: Diagnosis/Quality of practice Learning Outcome: LO 10.2 Compare the roles of the nurse in each identified healthcare setting. MNL LO: Evaluate healthcare issues related to pediatric nursing care. 13) Some nursing students are discussing job options. One of the student states that a position as a school nurse sounds interesting. What is an important role of the school nurse? 1. Screening for congenital heart disease 2. Prescribing antibiotics for streptococcal pharyngitis 3. Developing a plan for emergency care of injured children 4. Diagnosing an ear infection Answer: 3 Explanation: 1. Screening of students for certain conditions; educating students, teachers, and staff; and developing emergency plans are all roles of the school nurse. Diagnosing acute illness and prescribing medication for a new illness are beyond the scope of practice for the school nurse unless the nurse is licensed as an advance-practice nurse. 2. Screening of students for certain conditions; educating students, teachers, and staff; and developing emergency plans are all roles of the school nurse. Diagnosing acute illness and prescribing medication for a new illness are beyond the scope of practice for the school nurse unless the nurse is licensed as an advance-practice nurse. 3. Screening of students for certain conditions; educating students, teachers, and staff; and developing emergency plans are all roles of the school nurse. Diagnosing acute illness and prescribing medication for a new illness are beyond the scope of practice for the school nurse unless the nurse is licensed as an advance-practice nurse. 4. Screening of students for certain conditions; educating students, teachers, and staff; and developing emergency plans are all roles of the school nurse. Diagnosing acute illness and prescribing medication for a new illness are beyond the scope of practice for the school nurse unless the nurse is licensed as an advance-practice nurse. Page Ref: 211, 212 Cognitive Level: Analyzing Client Need &Sub: Safe and Effective Care Environment: Management of Care Standards: QSEN Competencies: Patient-centered care | AACN Essential Competencies: Essential IX: Baccalaureate generalist nursing practice | NLN Competencies: Nursing judgement | Nursing/Integrated Concepts: Nursing Process: Implementation/Quality of practice Learning Outcome: LO 10.2 Compare the roles of the nurse in each identified healthcare setting. MNL LO: Explore the practice of pediatric healthcare. 14) Which would be an acceptable community-health diagnosis? 1. Risk for Injury Related to Lack of Safe Bicycle Paths in High-Traffic Areas 2. Ineffective Family Coping Related to Lack of Time Together 3. Alterations in Nutrition Related to Use of Fast Food Restaurants 4. Ineffective Communication Related to Lack of Community Newsletter Answer: 1 Explanation: 1. The lack of safe bicycle paths in high-traffic areas is a community hazard affecting a large population of people. Ineffective family coping is appropriate for one family; alterations in nutrition and ineffective communication are not appropriate for the community as a whole. 2. The lack of safe bicycle paths in high-traffic areas is a community hazard affecting a large population of people. Ineffective family coping is appropriate for one family; alterations in nutrition and ineffective communication are not appropriate for the community as a whole. 3. The lack of safe bicycle paths in high-traffic areas is a community hazard affecting a large population of people. Ineffective family coping is appropriate for one family; alterations in nutrition and ineffective communication are not appropriate for the community as a whole. 4. The lack of safe bicycle paths in high-traffic areas is a community hazard affecting a large population of people. Ineffective family coping is appropriate for one family; alterations in nutrition and ineffective communication are not appropriate for the community as a whole. Page Ref: 209, 210 Cognitive Level: Analyzing Client Need &Sub: Safe and Effective Care Environment: Safety and Infection Control Standards: QSEN Competencies: Safety | AACN Essential Competencies: Essential VII: Clinical prevention and population health | NLN Competencies: Professional identity | Nursing/Integrated Concepts: Nursing Process: Planning/Quality of practice Learning Outcome: LO 10.2 Compare the roles of the nurse in each identified healthcare setting. MNL LO: Evaluate healthcare issues related to pediatric nursing care. 15) The nurse is providing care to a school-age client and family. The family, which consists of two parents and 4 children, live in a one-bedroom apartment. The father recently lost his job and the mother stays at home with the children. Which community resources would most benefit this family? Select all that apply. 1. Play groups 2. Parenting programs 3. Social services programs 4. Job skills training 5. Respite care Answer: 3, 4 Explanation: 1. This family is currently living in a one-bedroom apartment and the sole income earner recently lost his job. This family would most benefit from social services programs for monetary assistance and job skills training which would allow the parents to learn a trade and become employed. Play groups, parenting programs, and respite care are not applicable to this family's situation. 2. This family is currently living in a one-bedroom apartment and the sole income earner recently lost his job. This family would most benefit from social services programs for monetary assistance and job skills training which would allow the parents to learn a trade and become employed. Play groups, parenting programs, and respite care are not applicable to this family's situation. 3. This family is currently living in a one-bedroom apartment and the sole income earner recently lost his job. This family would most benefit from social services programs for monetary assistance and job skills training which would allow the parents to learn a trade and become employed. Play groups, parenting programs, and respite care are not applicable to this family's situation. 4. This family is currently living in a one-bedroom apartment and the sole income earner recently lost his job. This family would most benefit from social services programs for monetary assistance and job skills training which would allow the parents to learn a trade and become employed. Play groups, parenting programs, and respite care are not applicable to this family's situation. 5. This family is currently living in a one-bedroom apartment and the sole income earner recently lost his job. This family would most benefit from social services programs for monetary assistance and job skills training which would allow the parents to learn a trade and become employed. Play groups, parenting programs, and respite care are not applicable to this family's situation. Page Ref: 210 Cognitive Level: Applying Client Need &Sub: Psychosocial Integrity Standards: QSEN Competencies: Quality Improvement | AACN Essential Competencies: Essential II: Basic organizational and systems leadership for quality care and patient safety | NLN Competencies: Professional identity | Nursing/Integrated Concepts: Nursing Process: Planning/Coordination of care Learning Outcome: LO 10.3 Assemble a list of family support services that might be available in a community. MNL LO: Examine the role of the nurse in promoting culturally competent family-centered care. 16) The nurse manager is assisting the organization to open a healthcare center. What items must the manager include in pediatric inventory? Select all that apply. 1. Preprinted drug dosage chart 2. Oxygen face masks 3. Pediatric chairs and litters 4. Length-based resuscitation tape 5. Oral and NG airways and laryngoscope blades Answer: 1, 2, 4, 5 Explanation: 1. Essential equipment: child/neonate sized, proper weight dosage medications, smaller bags of IV fluids and special IV tubing, pediatric external defibrillator, pediatric oxygen masks/oral/NG airways and laryngoscope blades, length based resuscitation tapes and preprinted dosage chart to quickly identify equipment sizes and drug dosages by the length or weight of the child, essential emergency pediatric drugs and equipment. 2. Essential equipment: be child/neonate sized, proper weight dosage medications, smaller bags of IV fluids and special IV tubing, pediatric external defibrillator, pediatric oxygen masks/oral/NG airways and laryngoscope blades, length based resuscitation tapes and preprinted dosage chart to quickly identify equipment sizes and drug dosages by the length or weight of the child, essential emergency pediatric drugs and equipment. 3. Essential equipment: child/neonate sized, proper weight dosage medications, smaller bags of IV fluids and special IV tubing, pediatric external defibrillator, pediatric oxygen masks/oral/NG airways and laryngoscope blades, length based resuscitation tapes and preprinted dosage chart to quickly identify equipment sizes and drug dosages by the length or weight of the child, essential emergency pediatric drugs and equipment. Pediatric chairs and litters would be nice, but not essential. 4. Essential equipment: child/neonate sized, proper weight dosage medications, smaller bags of IV fluids and special IV tubing, pediatric external defibrillator, pediatric oxygen masks/oral/NG airways and laryngoscope blades, length based resuscitation tapes and preprinted dosage chart to quickly identify equipment sizes and drug dosages by the length or weight of the child, essential emergency pediatric drugs and equipment. 5. Essential equipment: child/neonate sized, proper weight dosage medications, smaller bags of IV fluids and special IV tubing, pediatric external defibrillator, pediatric oxygen masks/oral/NG airways and laryngoscope blades, length based resuscitation tapes and preprinted dosage chart to quickly identify equipment sizes and drug dosages by the length or weight of the child, essential emergency pediatric drugs and equipment. Page Ref: 210 Cognitive Level: Analyzing Client Need &Sub: Safe and Effective Care Environment: Safety and Infection Control Standards: QSEN Competencies: Patient-centered care | AACN Essential Competencies: Essential IX: Baccalaureate generalist nursing practice | NLN Competencies: Professional identity | Nursing/Integrated Concepts: Nursing Process: Planning/Quality of practice Learning Outcome: LO 10.1 Discuss the community healthcare settings where nurses provide health services to children. MNL LO: Differentiate developmentally appropriate care environments for the pediatric client and family. 17) The school nurse plans, develops, manages, and evaluates healthcare services to all children while they are in the educational setting. With which healthcare providers will the nurse be collaborating? Select all that apply. 1. School physician 2. Teachers 3. Cafeteria staff 4. Primary physician 5. Bus driver Answer: 1, 4 Explanation: 1. Partnering with the school physician consultant to discuss and update standing orders for the care of children; these standing orders usually address urgent and emergency care potentially needed by students. 2. The school nurse may need to educate the teachers, cafeteria staff, and bus drivers regarding detecting complications and alerting emergency personnel. 3. The school nurse may need to educate the teachers, cafeteria staff, and bus drivers regarding detecting complications and alerting emergency personnel. 4. Communicating with the child's primary healthcare provider or pediatric specialist about a child's specific health condition that needs to be effectively managed in the school setting. 5. The school nurse may need to educate the teachers, cafeteria staff, and bus drivers regarding detecting complications and alerting emergency personnel. Page Ref: 211, 212 Cognitive Level: Analyzing Client Need &Sub: Safe and Effective Care Environment: Safety and Infection Control Standards: QSEN Competencies: Patient-centered care | AACN Essential Competencies: Essential VI: Interprofessional communication and collaboration for improving patient health outcomes | NLN Competencies: Professional identity | Nursing/Integrated Concepts: Nursing Process: Planning/Collaboration Learning Outcome: LO 10.2 Compare the roles of the nurse in each identified healthcare setting. MNL LO: Analyze the role of the nurse and the role of the family in pediatric care. Principles of Pediatric Nursing: Caring for Children, 7e (Ball et al.) Chapter 11 Nursing Considerations for the Hospitalized Child 1) The parents of a critically injured child wish to stay in the room while the child is receiving emergency care. Which action by the nurse is the most appropriate? 1. Escort the parents to the waiting room and assure them that they can see their child soon. 2. Allow the parents to stay with the child. 3. Ask the physician if the parents can stay with the child. 4. Tell the parents that they do not need to stay with the child. Answer: 2 Explanation: 1. Parents should be allowed to stay with their child if they wish to do so. This position is supported by the Emergency Nurses Association and is a key aspect of family- centered care. 2. Parents should be allowed to stay with their child if they wish to do so. This position is supported by the Emergency Nurses Association and is a key aspect of family-centered care. 3. Parents should be allowed to stay with their child if they wish to do so. This position is supported by the Emergency Nurses Association and is a key aspect of family-centered care. 4. Parents should be allowed to stay with their child if they wish to do so. This position is supported by the Emergency Nurses Association and is a key aspect of family-centered care. Page Ref: 227-228 Cognitive Level: Applying Client Need &Sub: Psychosocial Integrity Standards: QSEN Competencies: Patient-centered care | AACN Essential Competencies: Essential II: Basic organizational and systems leadership for quality care and patient safety | NLN Competencies: Nursing judgement | Nursing/Integrated Concepts: Nursing Process: Implementation/Coordination of care Learning Outcome: LO 11.6 Integrate the concept of family presence during procedures and nursing strategies used to prepare the family. MNL LO: Apply key concepts of family-centered care. 2) The charge nurse on a hospital unit is developing plans of care related to separation anxiety. The charge nurse recognizes that which hospitalized child at highest risk to experience separation anxiety when parents cannot stay? 1. 6-month-old 2. 18-month-old 3. 3-year-old 4. 4-year-old Answer: 2 Explanation: 1. While all of these children can experience separation anxiety, the young toddler is at highest risk. Toddlers are the group most at risk for a stressful experience when hospitalized. Separation from parents increases this risk greatly. 2. While all of these children can experience separation anxiety, the young toddler is at highest risk. Toddlers are the group most at risk for a stressful experience when hospitalized. Separation from parents increases this risk greatly. 3. While all of these children can experience separation anxiety, the young toddler is at highest risk. Toddlers are the group most at risk for a stressful experience when hospitalized. Separation from parents increases this risk greatly. 4. While all of these children can experience separation anxiety, the young toddler is at highest risk. Toddlers are the group most at risk for a stressful experience when hospitalized. Separation from parents increases this risk greatly. Page Ref: 224 Cognitive Level: Applying Client Need &Sub: Psychosocial Integrity Standards: QSEN Competencies: Patient-centered care | AACN Essential Competencies: Essential II: Basic organizational and systems leadership for quality care and patient safety | NLN Competencies: Nursing judgement | Nursing/Integrated Concepts: Nursing Process: Diagnosis/Coordination of care Learning Outcome: LO 11.5 Identify nursing strategies to minimize the stressors related to hospitalization. MNL LO: Differentiate developmentally appropriate care environments for the pediatric client and family. 3) A group of children on one hospital unit are all suffering separation anxiety. Which child is experiencing the despair stage of separation anxiety? 1. Does not cry if parents return and leave again 2. Screams and cries when parents leave 3. Appears to be happy and content with staff 4. Lies quietly in bed Answer: 4 Explanation: 1. Children in the despair stage appear sad, depressed, or withdrawn. A child who is lying in bed might be exhibiting any of these. Screaming and crying are components of the protest stage. The young child who appears to be happy and content with everyone is in the denial stage, as is the child who does not cry if parents return and leave again. 2. Children in the despair stage appear sad, depressed, or withdrawn. A child who is lying in bed might be exhibiting any of these. Screaming and crying are components of the protest stage. The young child who appears to be happy and content with everyone is in the denial stage, as is the child who does not cry if parents return and leave again. 3. Children in the despair stage appear sad, depressed, or withdrawn. A child who is lying in bed might be exhibiting any of these. Screaming and crying are components of the protest stage. The young child who appears to be happy and content with everyone is in the denial stage, as is the child who does not cry if parents return and leave again. 4. Children in the despair stage appear sad, depressed, or withdrawn. A child who is lying in bed might be exhibiting any of these. Screaming and crying are components of the protest stage. The young child who appears to be happy and content with everyone is in the denial stage, as is the child who does not cry if parents return and leave again. Page Ref: 225-226 Cognitive Level: Applying Client Need &Sub: Psychosocial Integrity Standards: QSEN Competencies: Patient-centered care | AACN Essential Competencies: Essential II: Basic organizational and systems leadership for quality care and patient safety | NLN Competencies: Human flourishing | Nursing/Integrated Concepts: Nursing Process: Assessment/Coordination of care Learning Outcome: LO 11.2 Explain the effect of hospitalization on the child and family. MNL LO: Compare the developmental stages for pediatric clients. 4) A preschool-age client is seen in the clinic for a sore throat. In this child's mind, what is the most likely causative agent for the sore throat? 1. Was exposed to someone else with a sore throat. 2. Did not eat the right foods. 3. Yelled at his brother. 4. Did not take his vitamins. Answer: 3 Explanation: 1. Preschool-age children understand some concepts of being sick but not the cause of illness. They are likely to think that they are sick as a result of something that they have done. They will frequently view illness as punishment. A child of this age does not yet understand that he can become sick from exposure to someone else who is sick. The other two answers, while not causes of sore throat, can be factors in some illnesses but are beyond the thinking of a 4-year- old. 2. Preschool-age children understand some concepts of being sick but not the cause of illness. They are likely to think that they are sick as a result of something that they have done. They will frequently view illness as punishment. A child of this age does not yet understand that he can become sick from exposure to someone else who is sick. The other two answers, while not causes of sore throat, can be factors in some illnesses but are beyond the thinking of a 4-year-old. 3. Preschool-age children understand some concepts of being sick but not the cause of illness. They are likely to think that they are sick as a result of something that they have done. They will frequently view illness as punishment. A child of this age does not yet understand that he can become sick from exposure to someone else who is sick. The other two answers, while not causes of sore throat, can be factors in some illnesses but are beyond the thinking of a 4-year-old. 4. Preschool-age children understand some concepts of being sick but not the cause of illness. They are likely to think that they are sick as a result of something that they have done. They will frequently view illness as punishment. A child of this age does not yet understand that he can become sick from exposure to someone else who is sick. The other two answers, while not causes of sore throat, can be factors in some illnesses but are beyond the thinking of a 4-year-old. Page Ref: 226—227 Cognitive Level: Analyzing Client Need &Sub: Psychosocial Integrity Standards: QSEN Competencies: Patient-centered care | AACN Essential Competencies: Essential I: Liberal education for Baccalaureate generalist nursing practice | NLN Competencies: Human flourishing | Nursing/Integrated Concepts: Nursing Process: Assessment/Quality of practice Learning Outcome: LO 11.1 Compare and contrast the child's understanding of health and illness according to the child's developmental level. MNL LO: Use the nursing process to provide developmentally appropriate care for the pediatric client. 5) The charge nurse is concerned with reducing the stressors of hospitalization. Which nursing intervention is most helpful in decreasing the stressors for the toddler-age client? 1. Assign the same nurse to the toddler as much as possible. 2. Let the child listen to an audiotape of the mother's voice. 3. Place a picture of the family at the bedside. 4. Encourage a parent to stay with the child. Answer: 4 Explanation: 1. While all of the interventions are appropriate for the hospitalized toddler, presence of a parent is most important. Separation from parents is the major stressor for the hospitalized toddler. 2. While all of the interventions are appropriate for the hospitalized toddler, presence of a parent is most important. Separation from parents is the major stressor for the hospitalized toddler. 3. While all of the interventions are appropriate for the hospitalized toddler, presence of a parent is most important. Separation from parents is the major stressor for the hospitalized toddler. 4. While all of the interventions are appropriate for the hospitalized toddler, presence of a parent is most important. Separation from parents is the major stressor for the hospitalized toddler. Page Ref: 224-225 Cognitive Level: Analyzing Client Need &Sub: Psychosocial Integrity Standards: QSEN Competencies: Patient-centered care | AACN Essential Competencies: Essential II: Basic organizational and systems leadership for quality care and patient safety | NLN Competencies: Nursing judgement | Nursing/Integrated Concepts: Nursing Process: Implementation/Coordination of care Learning Outcome: LO 11.5 Identify nursing strategies to minimize the stressors related to hospitalization. MNL LO: Use the nursing process to provide developmentally appropriate care for the pediatric client. 6) The nurse is working with a school-age child who is hospitalized. Which action by the nurse will promote a sense of industry in this child? 1. Allow the child to assist with her care. 2. Encourage parents to participate in the child's care. 3. Give the child a detailed scientific explanation of the illness. 4. Speak to the child in a high-pitched voice. Answer: 1 Explanation: 1. Allowing the child to participate in her care will decrease the sense of loss of control and increase a sense of industry. While parents can certainly participate in their child's care, it does not increase the child's sense of control. School-age children in general will not understand detailed scientific explanations. Change in voice tone is appropriate when talking to very young children. 2. Allowing the child to participate in her care will decrease the sense of loss of control and increase a sense of industry. While parents can certainly participate in their child's care, it does not increase the child's sense of control. School-age children in general will not understand detailed scientific explanations. Change in voice tone is appropriate when talking to very young children. 3. Allowing the child to participate in her care will decrease the sense of loss of control and increase a sense of industry. While parents can certainly participate in their child's care, it does not increase the child's sense of control. School-age children in general will not understand detailed scientific explanations. Change in voice tone is appropriate when talking to very young children. 4. Allowing the child to participate in her care will decrease the sense of loss of control and increase a sense of industry. While parents can certainly participate in their child's care, it does not increase the child's sense of control. School-age children in general will not understand detailed scientific explanations. Change in voice tone is appropriate when talking to very young children. Page Ref: 225-227 Cognitive Level: Analyzing Client Need &Sub: Psychosocial Integrity Standards: QSEN Competencies: Patient-centered care | AACN Essential Competencies: Essential II: Basic organizational and systems leadership for quality care and patient safety | NLN Competencies: Nursing judgement | Nursing/Integrated Concepts: Nursing Process: Implementation/Coordination of care Learning Outcome: LO 11.1 Compare and contrast the child's understanding of health and illness according to the child's developmental level. MNL LO: Use the nursing process to provide developmentally appropriate care for the pediatric client. 7) The nurse is caring for a client in the pediatric intensive-care unit (PICU). The parents have expressed anger over the nursing care their child is receiving. Which nursing intervention is most appropriate based on the situation? 1. Ask the physician to talk with the family. 2. Explain to the parents that their anger is affecting their child so they will not be allowed to visit the child until they calm down. 3. Acknowledge the parents' concerns and collaborate with them regarding the care of their child. 4. Call the chaplain to sit with the family. Answer: 3 Explanation: 1. Hospitalization of the child in a pediatric intensive-care unit is a great stressor for parents. If the parents feel that they are not informed or involved in the care of their child, they may become angry and upset. Calling the physician or chaplain may be appropriate at some point, but the nurse must assume the role of supporter in this situation to promote a sense of trust. Telling the parents that they cannot visit their child will only increase their anger. 2. Hospitalization of the child in a pediatric intensive-care unit is a great stressor for parents. If the parents feel that they are not informed or involved in the care of their child, they may become angry and upset. Calling the physician or chaplain may be appropriate at some point, but the nurse must assume the role of supporter in this situation to promote a sense of trust. Telling the parents that they cannot visit their child will only increase their anger. 3. Hospitalization of the child in a pediatric intensive-care unit is a great stressor for parents. If the parents feel that they are not informed or involved in the care of their child, they may become angry and upset. Calling the physician or chaplain may be appropriate at some point, but the nurse must assume the role of supporter in this situation to promote a sense of trust. Telling the parents that they cannot visit their child will only increase their anger. 4. Hospitalization of the child in a pediatric intensive-care unit is a great stressor for parents. If the parents feel that they are not informed or involved in the care of their child, they may become angry and upset. Calling the physician or chaplain may be appropriate at some point, but the nurse must assume the role of supporter in this situation to promote a sense of trust. Telling the parents that they cannot visit their child will only increase their anger. Page Ref: 227-229 Cognitive Level: Analyzing Client Need &Sub: Psychosocial Integrity Standards: QSEN Competencies: Patient-centered care | AACN Essential Competencies: Essential IX: Baccalaureate generalist nursing practice | NLN Competencies: Nursing judgement | Nursing/Integrated Concepts: Nursing Process: Implementation/Coordination of care Learning Outcome: LO 11.2 Explain the effect of hospitalization on the child and family. MNL LO: Examine health promotion, stress reduction therapies, and safety for hospitalized child and family. 8) A toddler recently diagnosed with a seizure disorder will be discharged home on an anticonvulsant. Which action by the mother best demonstrates understanding of how to give the medication? 1. Verbalizing how to give the medication 2. Acknowledging understanding of written instructions 3. Drawing up the medication correctly in an oral syringe and administering it to the child 4. Observing the nurse draw up the medication and administering it to the child. Answer: 3 Explanation: 1. Verbalization of how to give the medication and acknowledging understanding of written instructions are methods that might be used, but they do not actually demonstrate understanding. Observing the nurse draw up and administer the medication may be used in the teaching process. The best way for the mother to demonstrate understanding is to actually draw up and give the medication. 2. Verbalization of how to give the medication and acknowledging understanding of written instructions are methods that might be used, but they do not actually demonstrate understanding. Observing the nurse draw up and administer the medication may be used in the teaching process. The best way for the mother to demonstrate understanding is to actually draw up and give the medication. 3. Verbalization of how to give the medication and acknowledging understanding of written instructions are methods that might be used, but they do not actually demonstrate understanding. Observing the nurse draw up and administer the medication may be used in the teaching process. The best way for the mother to demonstrate understanding is to actually draw up and give the medication. 4. Verbalization of how to give the medication and acknowledging understanding of written instructions are methods that might be used, but they do not actually demonstrate understanding. Observing the nurse draw up and administer the medication may be used in the teaching process. The best way for the mother to demonstrate understanding is to actually draw up and give the medication. Page Ref: 234 Cognitive Level: Applying Client Need &Sub: Safe and Effective Care Environment Standards: QSEN Competencies: Patient-centered care | AACN Essential Competencies: Essential VII: Clinical prevention and population health | NLN Competencies: Nursing judgement | Nursing/Integrated Concepts: Nursing Process: Evaluation/Health teaching and health promotion Learning Outcome: LO 11.7 Summarize strategies for preparing children and families for discharge from the hospital setting. MNL LO: Demonstrate safe medication administration for the pediatric client. 9) The nurse must perform a procedure on a toddler. Which technique is the most appropriate when performing the procedure? 1. Ask the mother to restrain the child during the procedure. 2. Ask the child if it is okay to start the procedure. 3. Perform the procedure in the child's hospital bed. 4. Allow the child to cry or scream. Answer: 4 Explanation: 1. While the toddler will need to be restrained, the parent should not be the one to do this. The nurse should avoid giving the child a choice if there is no choice. The treatment room should be utilized for the procedure so that the hospital bed remains a safe place. The child should be allowed to cry or scream during the procedure. 2. While the toddler will need to be restrained, the parent should not be the one to do this. The nurse should avoid giving the child a choice if there is no choice. The treatment room should be utilized for the procedure so that the hospital bed remains a safe place. The child should be allowed to cry or scream during the procedure. 3. While the toddler will need to be restrained, the parent should not be the one to do this. The nurse should avoid giving the child a choice if there is no choice. The treatment room should be utilized for the procedure so that the hospital bed remains a safe place. The child should be allowed to cry or scream during the procedure. 4. While the toddler will need to be restrained, the parent should not be the one to do this. The nurse should avoid giving the child a choice if there is no choice. The treatment room should be utilized for the procedure so that the hospital bed remains a safe place. The child should be allowed to cry or scream during the procedure. Page Ref: 235 Cognitive Level: Applying Client Need &Sub: Psychosocial Integrity Standards: QSEN Competencies: Patient-centered care | AACN Essential Competencies: Essential IX: Baccalaureate generalist nursing practice | NLN Competencies: Professional identity | Nursing/Integrated Concepts: Nursing Process: Planning/Coordination of care Learning Outcome: LO 11.5 Identify nursing strategies to minimize the stressors related to hospitalization. MNL LO: Utilize play and communication strategies when preparing and administering medications to children. 10) A young school-age client is in the playroom when the respiratory therapist arrives on the pediatric unit to give the child a scheduled breathing treatment. Which action by the nurse is the most appropriate? 1. Reschedule the treatment for a later time. 2. Show the respiratory therapist to the playroom so the treatment may be performed. 3. Escort the child to his room and ask the child-life specialist to bring toys to the bedside. 4. Assist the child back to his room for the treatment but reassure the child that he may return when the procedure is completed. Answer: 4 Explanation: 1. Procedures should not be performed in the playroom. Scheduled respiratory treatments should be performed on time; however, the child should be allowed to return to the playroom as soon as the procedure is completed. 2. Procedures should not be performed in the playroom. Scheduled respiratory treatments should be performed on time; however, the child should be allowed to return to the playroom as soon as the procedure is completed. 3. Procedures should not be performed in the playroom. Scheduled respiratory treatments should be performed on time; however, the child should be allowed to return to the playroom as soon as the procedure is completed. 4. Procedures should not be performed in the playroom. Scheduled respiratory treatments should be performed on time; however, the child should be allowed to return to the playroom as soon as the procedure is completed. Page Ref: 235 Cognitive Level: Applying Client Need &Sub: Psychosocial Integrity Standards: QSEN Competencies: Patient-centered care | AACN Essential Competencies: Essential IX: Baccalaureate generalist nursing practice | NLN Competencies: Nursing judgement | Nursing/Integrated Concepts: Nursing Process: Implementation/Coordination of care Learning Outcome: LO 11.5 Identify nursing strategies to minimize the stressors related to hospitalization. MNL LO: Use the nursing process to provide developmentally appropriate care for the pediatric client. 11) The nurse is working with a hospitalized preschool-age child. The nurse is planning activities to reduce anxiety in this child. Which action by the nurse is the most appropriate? 1. Provide the child with a doll and safe medical equipment. 2. Read a story to the child. 3. Use an anatomically correct doll to teach the child about the illness. 4. Talk to the child about the hospitalization. Answer: 1 Explanation: 1. Therapeutic play is a means of anxiety reduction in the hospitalized child. Allowing the child to play with safe medical equipment is an age-appropriate method through which the child can express her feelings, thereby reducing anxiety. Anatomically correct dolls are not age appropriate. Reading a story to the child does not allow for expression of feelings. Talking to the child may be beneficial, but it does not allow for active release of frustration and anxiety as active play does. 2. Therapeutic play is a means of anxiety reduction in the hospitalized child. Allowing the child to play with safe medical equipment is an age-appropriate method through which the child can express her feelings, thereby reducing anxiety. Anatomically correct dolls are not age appropriate. Reading a story to the child does not allow for expression of feelings. Talking to the child may be beneficial, but it does not allow for active release of frustration and anxiety as active play does. 3. Therapeutic play is a means of anxiety reduction in the hospitalized child. Allowing the child to play with safe medical equipment is an age-appropriate method through which the child can express her feelings, thereby reducing anxiety. Anatomically correct dolls are not age appropriate. Reading a story to the child does not allow for expression of feelings. Talking to the child may be beneficial, but it does not allow for active release of frustration and anxiety as active play does. 4. Therapeutic play is a means of anxiety reduction in the hospitalized child. Allowing the child to play with safe medical equipment is an age-appropriate method through which the child can express her feelings, thereby reducing anxiety. Anatomically correct dolls are not age appropriate. Reading a story to the child does not allow for expression of feelings. Talking to the child may be beneficial, but it does not allow for active release of frustration and anxiety as active play does. Page Ref: 235 Cognitive Level: Applying Client Need &Sub: Psychosocial Integrity Standards: QSEN Competencies: Patient-centered care | AACN Essential Competencies: Essential IX: Baccalaureate generalist nursing practice | NLN Competencies: Nursing judgement | Nursing/Integrated Concepts: Nursing Process: Implementation/Coordination of care Learning Outcome: LO 11.5 Identify nursing strategies to minimize the stressors related to hospitalization. MNL LO: Use the nursing process to provide developmentally appropriate care for the pediatric client. 12) The nurse needs to administer a medication to a preschool-age child. The medication is only available in tablet form. Which action by the nurse is the most appropriate? 1. Place the tablet on the child's tongue and give the child a drink of water. 2. Break the tablet in small pieces and ask the child to swallow the pieces one by one. 3. Crush the tablet and mix it in a teaspoon of applesauce. 4. Crush the table and mix it in a cup of juice. Answer: 3 Explanation: 1. A 4-year-old is not mature enough to swallow a pill or pieces of a pill. The medication should be crushed and mixed with a very small amount of food, not juice. 2. A 4-year-old is not mature enough to swallow a pill or pieces of a pill. The medication should be crushed and mixed with a very small amount of food, not juice. 3. A 4-year-old is not mature enough to swallow a pill or pieces of a pill. The medication should be crushed and mixed with a very small amount of food, not juice. 4. A 4-year-old is not mature enough to swallow a pill or pieces of a pill. The medication should be crushed and mixed with a very small amount of food, not juice. Page Ref: 234 Cognitive Level: Analyzing Client Need &Sub: Physiological Integrity Standards: QSEN Competencies: Patient-centered care | AACN Essential Competencies: Essential IX: Baccalaureate generalist nursing practice | NLN Competencies: Nursing judgement | Nursing/Integrated Concepts: Nursing Process: Implementation/Coordination of care Learning Outcome: LO 11.2 Explain the effect of hospitalization on the child and family. MNL LO: Demonstrate safe medication administration for the pediatric client. 13) A child is being discharged from the hospital after a 3-week stay following a motor vehicle accident. The mother expresses concern about caring for the child's wounds at home. She has demonstrated appropriate technique with medication administration and wound care. Which nursing diagnosis is the priority in this situation? 1. Knowledge Deficit of Home Care 2. Altered Family Processes Related to Hospitalization 3. Parental Anxiety Related to Care of the Child at Home 4. Risk for Infection Related to Presence of Healing Wounds Answer: 3 Explanation: 1. While all of the diagnoses might have been appropriate at some point, the current focus is the mother's anxiety about caring for the child at home. The priority of the nurse is relieving this anxiety. 2. While all of the diagnoses might have been appropriate at some point, the current focus is the mother's anxiety about caring for the child at home. The priority of the nurse is relieving this anxiety. 3. While all of the diagnoses might have been appropriate at some point, the current focus is the mother's anxiety about caring for the child at home. The priority of the nurse is relieving this anxiety. 4. While all of the diagnoses might have been appropriate at some point, the current focus is the mother's anxiety about caring for the child at home. The priority of the nurse is relieving this anxiety. Page Ref: 244-245 Cognitive Level: Analyzing Client Need &Sub: Psychosocial Integrity Standards: QSEN Competencies: Patient-centered care | AACN Essential Competencies: Essential II: Basic organizational and systems leadership for quality care and patient safety | NLN Competencies: Human flourishing | Nursing/Integrated Concepts: Nursing Process: Diagnosis/Health teaching and health promotion Learning Outcome: LO 11.7 Summarize strategies for preparing children and families for discharge from the hospital setting. MNL LO: Develop a family-centered nursing care plan for the child and family. 14) An infant has been NPO for surgery for 4 hours and does not have an intravenous line. The nurse receives a call from the operating room with the information that the surgery has been postponed due to an emergency. Which action by the nurse is the most appropriate? 1. Feed the infant 4 ounces of formula. 2. Reassure the parents that it will not be much longer before surgery. 3. Allow the parents to feed the infant an ounce of oral rehydration solution. 4. Call the physician to see if the infant needs to have an intravenous line started. Answer: 4 Explanation: 1. The infant who is NPO is at high risk for dehydration. The nurse does not know how much longer it will be before surgery. The nurse cannot independently make the decision to feed the infant. Feeding the infant could further postpone the surgery, should an operating room become available sooner than expected. It is best to keep the infant NPO and consult the physician to see if an intravenous line is needed. 2. The infant who is NPO is at high risk for dehydration. The nurse does not know how much longer it will be before surgery. The nurse cannot independently make the decision to feed the infant. Feeding the infant could further postpone the surgery, should an operating room become available sooner than expected. It is best to keep the infant NPO and consult the physician to see if an intravenous line is needed. 3. The infant who is NPO is at high risk for dehydration. The nurse does not know how much longer it will be before surgery. The nurse cannot independently make the decision to feed the infant. Feeding the infant could further postpone the surgery, should an operating room become available sooner than expected. It is best to keep the infant NPO and consult the physician to see if an intravenous line is needed. 4. The infant who is NPO is at high risk for dehydration. The nurse does not know how much longer it will be before surgery. The nurse cannot independently make the decision to feed the infant. Feeding the infant could further postpone the surgery, should an operating room become available sooner than expected. It is best to keep the infant NPO and consult the physician to see if an intravenous line is needed. Page Ref: 235-238 Cognitive Level: Analyzing Client Need &Sub: Physiological Integrity Standards: QSEN Competencies: Patient-centered care | AACN Essential Competencies: Essential VI: Interprofessional communication and collaboration for improving patient health outcomes | NLN Competencies: Professional identity | Nursing/Integrated Concepts: Nursing Process: Assessment/Coordination of care Learning Outcome: LO 11.2 Explain the effect of hospitalization on the child and family. MNL LO: Examine the concepts related to the hospitalization of a child. 15) The nurse is working with an adolescent client who will be admitted to the hospital in two days. Which nursing approach is most appropriate to prepare this client for hospitalization? 1. Have teens who have had similar experiences talk to the adolescent about hospitalization. 2. Provide an opportunity for the child to talk with an adult who has had a similar experience. 3. Teach parents what to expect so the information can be shared with the adolescent. 4. Provide an opportunity for the teen to try on surgical attire. Answer: 1 Explanation: 1. Adolescents benefit from a different approach than younger children when being prepared for hospitalization. Written materials, anatomically correct dolls, and talking to peers who have had similar experiences are all appropriate for the adolescent. The adolescent should be taught first-hand what to expect during the hospitalization. Dressing up in surgical attire is appropriate for the younger child. 2. Adolescents benefit from a different approach than younger children when being prepared for hospitalization. Written materials, anatomically correct dolls, and talking to peers who have had similar experiences are all appropriate for the adolescent. The adolescent should be taught first- hand what to expect during the hospitalization. Dressing up in surgical attire is appropriate for the younger child. 3. Adolescents benefit from a different approach than younger children when being prepared for hospitalization. Written materials, anatomically correct dolls, and talking to peers who have had similar experiences are all appropriate for the adolescent. The adolescent should be taught first- hand what to expect during the hospitalization. Dressing up in surgical attire is appropriate for the younger child. 4. Adolescents benefit from a different approach than younger children when being prepared for hospitalization. Written materials, anatomically correct dolls, and talking to peers who have had similar experiences are all appropriate for the adolescent. The adolescent should be taught first- hand what to expect during the hospitalization. Dressing up in surgical attire is appropriate for the younger child. Page Ref: 233 Cognitive Level: Applying Client Need &Sub: Psychosocial Integrity Standards: QSEN Competencies: Patient-centered care | AACN Essential Competencies: Essential VII: Clinical prevention and population health | NLN Competencies: Nursing judgement | Nursing/Integrated Concepts: Nursing Process: Planning/Coordination of care Learning Outcome: LO 11.5 Identify nursing strategies to minimize the stressors related to hospitalization. MNL LO: Examine health promotion, stress reduction therapies, and safety for hospitalized child and family. 16) The nurse is providing care to a school-age client who is admitted to the hospital after a motor vehicle accident. Which interventions are appropriate to prepare this client and family for their hospital stay? Select all that apply. 1. A hospital tour 2. A health fair brochure 3. An orientation to the unit 4. An age-appropriate explanation of procedures 5. A child life program consultation Answer: 3, 4, 5 Explanation: 1. Interventions that are appropriate for this client and family are those that occur as the result of an unplanned hospital admission. The nurse would orient the client and family to the unit and provide age-appropriate explanation for all procedures. It is also appropriate for the nurse to consult with the child life program. A hospital tour and a health fair brochure are appropriate interventions for a planned hospitalization. 2. Interventions that are appropriate for this client and family are those that occur as the result of an unplanned hospital admission. The nurse would orient the client and family to the unit and provide age-appropriate explanation for all procedures. It is also appropriate for the nurse to consult with the child life program. A hospital tour and a health fair brochure are appropriate interventions for a planned hospitalization. 3. Interventions that are appropriate for this client and family are those that occur as the result of an unplanned hospital admission. The nurse would orient the client and family to the unit and provide age-appropriate explanation for all procedures. It is also appropriate for the nurse to consult with the child life program. A hospital tour and a health fair brochure are appropriate interventions for a planned hospitalization. 4. Interventions that are appropriate for this client and family are those that occur as the result of an unplanned hospital admission. The nurse would orient the client and family to the unit and provide age-appropriate explanation for all procedures. It is also appropriate for the nurse to consult with the child life program. A hospital tour and a health fair brochure are appropriate interventions for a planned hospitalization. 5. Interventions that are appropriate for this client and family are those that occur as the result of an unplanned hospital admission. The nurse would orient the client and family to the unit and provide age-appropriate explanation for all procedures. It is also appropriate for the nurse to consult with the child life program. A hospital tour and a health fair brochure are appropriate interventions for a planned hospitalization. Page Ref: 230-231 Cognitive Level: Applying Client Need &Sub: Psychosocial Integrity Standards: QSEN Competencies: Patient-centered care | AACN Essential Competencies: Essential II: Basic organizational and systems leadership for quality care and patient safety | NLN Competencies: Nursing judgement | Nursing/Integrated Concepts: Nursing Process: Implementation/Coordination of care Learning Outcome: LO 11.3 Describe the child's and family's adaptation to hospitalization. MNL LO: Apply the nursing process to care for the needs of the hospitalized child. 17) The nurse is providing care to a preschool-age client who was admitted to the medical- surgical unit after an acute asthma attack. Which interventions foster a family-centered focus to client care? Select all that apply. 1. Discussing rooming in with the parents of the client 2. Allowing the client to "cry it out" after the parents leave for the evening 3. Providing comfort items from home, such as a blanket 4. Maintaining strict visitation for the family 5. Discussing what to expect during the hospital stay Answer: 1, 3, 5 Explanation: 1. Family-centered care principles that are used in the hospital setting include rooming in, providing comfort items from home, and discussing what to expect. Allowing the child to "cry it out" and maintaining strict visitation for the family are not family-centered principles. 2. Family-centered care principles that are used in the hospital setting include rooming in, providing comfort items from home, and discussing what to expect. Allowing the child to "cry it out" and maintaining strict visitation for the family are not family-centered principles. 3. Family-centered care principles that are used in the hospital setting include rooming in, providing comfort items from home, and discussing what to expect. Allowing the child to "cry it out" and maintaining strict visitation for the family are not family-centered principles. 4. Family-centered care principles that are used in the hospital setting include rooming in, providing comfort items from home, and discussing what to expect. Allowing the child to "cry it out" and maintaining strict visitation for the family are not family-centered principles. 5. Family-centered care principles that are used in the hospital setting include rooming in, providing comfort items from home, and discussing what to expect. Allowing the child to "cry it out" and maintaining strict visitation for the family are not family-centered principles. Page Ref: 230-231 Cognitive Level: Applying Client Need &Sub: Psychosocial Integrity Standards: QSEN Competencies: Patient-centered care | AACN Essential Competencies: Essential II: Basic organizational and systems leadership for quality care and patient safety | NLN Competencies: Nursing judgement | Nursing/Integrated Concepts: Nursing Process: Planning/Coordination of care Learning Outcome: LO 11.4 Apply family-centered care principles to the hospital setting. MNL LO: Apply key concepts of family-centered care. 18) Match the behaviors with its stage of separation anxiety the child may exhibit. A. Protest B. Despair C. Denial 1. Withdrawal or compliant behavior 2. Appearance of being happy and content with everyone 3. Clinging to parents 4. Lack of protest when parents leave 5. Screaming and crying 6. Sadness Answer: 1/B, 2/C, 3/A, 4/C, 5/A, 6/B 1. Despair 2. Denial 3. Protest 4. Denial 5. Protest 6. Despair Explanation: The stages of separation anxiety include: Protest–Screaming, crying, clinging to parents, and may resist attempts by other adults to comfort them. Despair–Sadness, quiet, appear to have "settled in," withdrawal or compliant behavior, and crying when parents return. Denial– Lack of protest when parents leave, appearance of being happy and content with everyone, show interest in surroundings, and close relationships not established. Page Ref: 227 Cognitive Level: Analyzing Client Need &Sub: Psychosocial Integrity Standards: QSEN Competencies: Patient-centered care | AACN Essential Competencies: Essential VII: Clinical prevention and population health | NLN Competencies: Nursing judgement | Nursing/Integrated Concepts: Nursing Process: Diagnosing/Coordination of care Learning Outcome: LO 11.1 Compare and contrast the child's understanding of health and illness according to the child's developmental level. MNL LO: Examine health promotion, stress reduction therapies, and safety for hospitalized child and family. 19) The child was just transferred to the postanesthesia unit (PACU) and report given. The nurse has performed baseline vital signs, the child is stable and pain is under control. What should the nurse do next? 1. Document 2. Allow the parents to visit the child 3. Discharge the child 4. Look for signs of infection 5. Offer clear liquids Answer: 2 Explanation: 1. This is not the next task the nurse should perform. 2. If the child is stable and pain is under control, the nurse should allow the parents to visit with the child. 3. The child has just come to the PACU, the normal amount of time in the PACU is at least one hour. 4. The vital signs and operative area will be monitored throughout the child's time in PACU. 5. The child has just been transferred to PACU, the child will be offered liquids when fully awake. Page Ref: 224 Cognitive Level: Analyzing Client Need &Sub: Health Promotion and Maintenance Standards: QSEN Competencies: Patient-centered care | AACN Essential Competencies: Essential II: Basic organizational and systems leadership for quality care and patient safety | NLN Competencies: Nursing judgement | Nursing/Integrated Concepts: Nursing Process: Planning/Coordination of care Learning Outcome: LO 11.6 Integrate the concept of family presence during procedures and nursing strategies used to prepare the family. MNL LO: Apply key concepts of family-centered care. Principles of Pediatric Nursing: Caring for Children, 7e (Ball et al.) Chapter 12 The Child with a Chronic Condition 1) The nurse in the long-term care clinic is reviewing the charts of a group of children with chronic physical, psychological, functional, and social limitations. Which conditions are most likely to lead to chronic limitations? Select all that apply. 1. Near drowning 2. Congenital heart defect 3. Sinusitis 4. Fetal insult when the mother contracted rubella in the first trimester of pregnancy 5. Sepsis contracted as a neonate Answer: 1, 2, 4, 5 Explanation: 1. All of these conditions or events except sinusitis can leave a child with a permanent chronic condition. 2. All of these conditions or events except sinusitis can leave a child with a permanent chronic condition. 3. All of these conditions or events except sinusitis can leave a child with a permanent chronic condition. 4. All of these conditions or events except sinusitis can leave a child with a permanent chronic condition. 5. All of these conditions or events except sinusitis can leave a child with a permanent chronic condition. Page Ref: 247-248 Cognitive Level: Analyzing Client Need &Sub: Health Promotion and Maintenance Standards: QSEN Competencies: Quality improvement | AACN Essential Competencies: Essential IV: Information management and application of patient care technology | NLN Competencies: Nursing judgement | Nursing/Integrated Concepts: Nursing Process: Assessment/Professional practice evaluation Learning Outcome: LO 12.1 Explain the causes of chronic conditions in children. MNL LO: Analyze the impact the diagnosis of a chronic illness has on the pediatric client and family. 2) The school nurse completes an assessment of a school-age client to determine the services this child will need in the classroom. The client is a newly diagnosed with type I diabetes mellitus. Based on this information, which special healthcare need category is the most appropriate? 1. Dependent on medication or special diet 2. Dependent on medical technology 3. Increase use of healthcare services 4. Functional limitations Answer: 1 Explanation: 1. A child recently diagnosed with type I diabetes mellitus with no other medical diagnoses would be placed in the dependent on medication or special diet category. The other categories of care are not appropriate for this client. 2. A child recently diagnosed with type I diabetes mellitus with no other medical diagnoses would be placed in the dependent on medication or special diet category. The other categories of care are not appropriate for this client. 3. A child recently diagnosed with type I diabetes mellitus with no other medical diagnoses would be placed in the dependent on medication or special diet category. The other categories of care are not appropriate for this client. 4. A child recently diagnosed with type I diabetes mellitus with no other medical diagnoses would be placed in the dependent on medication or special diet category. The other categories of care are not appropriate for this client. Page Ref: 248 Cognitive Level: Applying Client Need &Sub: Physiological Integrity Standards: QSEN Competencies: Patient-centered care | AACN Essential Competencies: Essential IX: Baccalaureate generalist nursing practice | NLN Competencies: Nursing judgement | Nursing/Integrated Concepts: Nursing Process: Planning/Coordination of care Learning Outcome: LO 12.2 Identify the categories of chronic conditions in children. MNL LO: Apply the general concepts related to caring for a child with a chronic illness. 3) The nurse can instruct parents to expect children in which age group to begin to assume more independent responsibility for their own management of a chronic condition, such as blood- glucose monitoring, insulin administration, intermittent self-catheterization, and appropriate inhaler use? 1. Toddlers 2. Preschool-age 3. School-age 4. Adolescents Answer: 3 Explanation: 1. School-age children are developing a sense of industry and can begin assuming responsibility for self-care. Toddlers and preschool-age children do not have the cognitive and psychomotor skills for these tasks. Adolescents should already be well accomplished at self-care. 2. School-age children are developing a sense of industry and can begin assuming responsibility for self-care. Toddlers and preschool-age children do not have the cognitive and psychomotor skills for these tasks. Adolescents should already be well accomplished at self-care. 3. School-age children are developing a sense of industry and can begin assuming responsibility for self-care. Toddlers and preschool-age children do not have the cognitive and psychomotor skills for these tasks. Adolescents should already be well accomplished at self-care. 4. School-age children are developing a sense of industry and can begin assuming responsibility for self-care. Toddlers and preschool-age children do not have the cognitive and psychomotor skills for these tasks. Adolescents should already be well accomplished at self-care. Page Ref: 252 Cognitive Level: Applying Client Need &Sub: Health Promotion and Maintenance Standards: QSEN Competencies: Quality improvement | AACN Essential Competencies: Essential VII: Clinical prevention and population health | NLN Competencies: Nursing judgement | Nursing/Integrated Concepts: Nursing Process: Implementation/Health teaching and health promotion Learning Outcome: LO 12.5 Prepare the family of a child with a chronic condition to effectively care for the child in the home. MNL LO: Apply the general concepts related to caring for a child with a chronic illness. 4) The nurse is working in an adolescent medical clinic. What can the nurse anticipate when comparing adolescents in the clinic with chronic conditions to their peers? 1. A high level self-esteem 2. A concern for their parents 3. An altered body image 4. A decreased concern about their appearance Answer: 3 Explanation: 1. As adolescents develop a sense of identity, they are focused on themselves and the present. They have a heightened concern about their appearance but may have inaccurate assessments of their body image and low self-esteem when comparing their bodies with those of their peers. 2. As adolescents develop a sense of identity, they are focused on themselves and the present. They have a heightened concern about their appearance but may have inaccurate assessments of their body image and low self-esteem when comparing their bodies with those of their peers. 3. As adolescents develop a sense of identity, they are focused on themselves and the present. They have a heightened concern about their appearance but may have inaccurate assessments of their body image and low self-esteem when comparing their bodies with those of their peers. 4. As adolescents develop a sense of identity, they are focused on themselves and the present. They have a heightened concern about their appearance but may have inaccurate assessments of their body image and low self-esteem when comparing their bodies with those of their peers. Page Ref: 252 Cognitive Level: Applying Client Need &Sub: Health Promotion and Maintenance Standards: QSEN Competencies: Patient-centered care | AACN Essential Competencies: Essential IX: Baccalaureate generalist nursing practice | NLN Competencies: Human flourishing | Nursing/Integrated Concepts: Nursing Process: Assessment/Quality of practice Learning Outcome: LO 12.4 Assess the family of a child with a chronic condition. MNL LO: Evaluate the developmental considerations when caring for a child with a chronic illness. 5) The nurse is working with a child with a chronic condition. The nurse observes that over time, the parents have experienced a pattern of periodic grieving alternating with denial. What are the parents currently experiencing based on this assessment finding? 1. Chronic sorrow 2. Compassion fatigue 3. Dysfunctional parenting 4. Pathological grieving Answer: 1 Explanation: 1. Parents experience chronic sorrow as they grieve when their child does not meet developmental milestones or participate in activities of "normal" children. The time between periods of grieving may be times of parental denial, which allows the family to function. Compassion fatigue is experienced by caregivers as their ability to feel compassion is exhausted. Dysfunctional parenting involves inadequately meeting the needs of children. Pathological grieving results when persons do not move through the stages of grief to resolution. 2. Parents experience chronic sorrow as they grieve when their child does not meet developmental milestones or participate in activities of "normal" children. The time between periods of grieving may be times of parental denial, which allows the family to function. Compassion fatigue is experienced by caregivers as their ability to feel compassion is exhausted. Dysfunctional parenting involves inadequately meeting the needs of children. Pathological grieving results when persons do not move through the stages of grief to resolution. 3. Parents experience chronic sorrow as they grieve when their child does not meet developmental milestones or participate in activities of "normal" children. The time between periods of grieving may be times of parental denial, which allows the family to function. Compassion fatigue is experienced by caregivers as their ability to feel compassion is exhausted. Dysfunctional parenting involves inadequately meeting the needs of children. Pathological grieving results when persons do not move through the stages of grief to resolution. 4. Parents experience chronic sorrow as they grieve when their child does not meet developmental milestones or participate in activities of "normal" children. The time between periods of grieving may be times of parental denial, which allows the family to function. Compassion fatigue is experienced by caregivers as their ability to feel compassion is exhausted. Dysfunctional parenting involves inadequately meeting the needs of children. Pathological grieving results when persons do not move through the stages of grief to resolution. Page Ref: 258 Cognitive Level: Analyzing Client Need &Sub: Psychosocial Integrity Standards: QSEN Competencies: Patient-centered care | AACN Essential Competencies: Essential IX: Baccalaureate generalist nursing practice | NLN Competencies: Human flourishing | Nursing/Integrated Concepts: Nursing Process: Diagnosis/Quality of practice Learning Outcome: LO 12.5 Prepare the family of a child with a chronic condition to effectively care for the child in the home. MNL LO: Analyze the impact the diagnosis of a chronic illness has on the pediatric client and family. 6) In working with parents of children with chronic diseases, the nurse is concerned with helping the parents to protect themselves from compassion fatigue. Which activities are appropriate for the nurse to encourage? Select all that apply. 1. Sleeping more than 9 hours per 24-hour period 2. Exercising 3. Fostering social relationships 4. Developing a hobby 5. Moving away Answer: 2, 3, 4 Explanation: 1. Exercising, fostering social relationships, and developing a hobby all contribute to physical, spiritual, social, and mental rest and restoration. Sleeping more than the body requires and moving away are avoidance behaviors that do not address exhaustion from overwhelming caregiving responsibilities. 2. Exercising, fostering social relationships, and developing a hobby all contribute to physical, spiritual, social, and mental rest and restoration. Sleeping more than the body requires and moving away are avoidance behaviors that do not address exhaustion from overwhelming caregiving responsibilities. 3. Exercising, fostering social relationships, and developing a hobby all contribute to physical, spiritual, social, and mental rest and restoration. Sleeping more than the body requires and moving away are avoidance behaviors that do not address exhaustion from overwhelming caregiving responsibilities. 4. Exercising, fostering social relationships, and developing a hobby all contribute to physical, spiritual, social, and mental rest and restoration. Sleeping more than the body requires and moving away are avoidance behaviors that do not address exhaustion from overwhelming caregiving responsibilities. 5. Exercising, fostering social relationships, and developing a hobby all contribute to physical, spiritual, social, and mental rest and restoration. Sleeping more than the body requires and moving away are avoidance behaviors that do not address exhaustion from overwhelming caregiving responsibilities. Page Ref: 258 Cognitive Level: Applying Client Need &Sub: Psychosocial Integrity Standards: QSEN Competencies: Patient-centered care | AACN Essential Competencies: Essential IX: Baccalaureate generalist nursing practice | NLN Competencies: Human flourishing | Nursing/Integrated Concepts: Nursing Process: Implementation/Quality of care Learning Outcome: LO 12.5 Prepare the family of a child with a chronic condition to effectively care for the child in the home. MNL LO: Apply the nursing process to care for the needs of a child with a chronic illness. 7) The nurse is working with a group of parents who have children with chronic conditions. Which statement by a parent would indicate a risk for a caregiver burden that could become overwhelming? 1. "My mother moved in and helped us take our quadruplets home." 2. "Our health insurance sent us a rejection letter for my child's brand-name medication, and we must fill out forms to get the generic." 3. "I chose to quit my job to be home with my child, and my husband helps in the evening when he can." 4. "I have to care for my child day and night, which leaves little time for me." Answer: 4 Explanation: 1. No respite time from caregiving responsibilities may lead to overwhelming caregiver burden. The family's pitching in to help indicates family support. Substituting generic for brand-name medications will not result in caregiver burden. The mother's choosing to care for the child and receiving help from the husband indicates family support. 2. No respite time from caregiving responsibilities may lead to overwhelming caregiver burden. The family's pitching in to help indicates family support. Substituting generic for brand-name medications will not result in caregiver burden. The mother's choosing to care for the child and receiving help from the husband indicates family support. 3. No respite time from caregiving responsibilities may lead to overwhelming caregiver burden. The family's pitching in to help indicates family support. Substituting generic for brand-name medications will not result in caregiver burden. The mother's choosing to care for the child and receiving help from the husband indicates family support. 4. No respite time from caregiving responsibilities may lead to overwhelming caregiver burden. The family's pitching in to help indicates family support. Substituting generic for brand-name medications will not result in caregiver burden. The mother's choosing to care for the child and receiving help from the husband indicates family support. Page Ref: 258 Cognitive Level: Applying Client Need &Sub: Psychosocial Integrity Standards: QSEN Competencies: Patient-centered care | AACN Essential Competencies: Essential IX: Baccalaureate generalist nursing practice | NLN Competencies: Nursing judgement | Nursing/Integrated Concepts: Nursing Process: Evaluation/Quality of practice Learning Outcome: LO 12.7 Discuss the family's role in care coordination. MNL LO: Analyze the impact the diagnosis of a chronic illness has on the pediatric client and family. 8) The clinic nurse is working with a child with multiple disabilities. The parents have asked the nurse to help them in meeting with the school board to develop an Individualized Education Plan (IEP) and an Individualized Health Plan (IHP). Which nursing intervention is most appropriate? 1. Providing a written list of the child's medical diagnoses for the IEP meeting. 2. Offering to wait with the child while the parents attend the IEP meeting. 3. Listening to the parents' concerns and complaints about the school district. 4. Presenting verbally the child's cognitive, physical, and social skills to school officials at the IEP meeting. Answer: 4 Explanation: 1. As an advocate for the child and a partner with the family, the nurse attends the IEP meeting and presents the child's functional skills to develop a comprehensive IEP. A list of medical diagnoses does not accurately inform school officials about the child's skills or needs. Waiting with the child and listening to parents' concerns may be kind and empathetic but does not contribute to an action plan for the child's educational needs. 2. As an advocate for the child and a partner with the family, the nurse attends the IEP meeting and presents the child's functional skills to develop a comprehensive IEP. A list of medical diagnoses does not accurately inform school officials about the child's skills or needs. Waiting with the child and listening to parents' concerns may be kind and empathetic but does not contribute to an action plan for the child's educational needs. 3. As an advocate for the child and a partner with the family, the nurse attends the IEP meeting and presents the child's functional skills to develop a comprehensive IEP. A list of medical diagnoses does not accurately inform school officials about the child's skills or needs. Waiting with the child and listening to parents' concerns may be kind and empathetic but does not contribute to an action plan for the child's educational needs. 4. As an advocate for the child and a partner with the family, the nurse attends the IEP meeting and presents the child's functional skills to develop a comprehensive IEP. A list of medical diagnoses does not accurately inform school officials about the child's skills or needs. Waiting with the child and listening to parents' concerns may be kind and empathetic but does not contribute to an action plan for the child's educational needs. Page Ref: 256-257 Cognitive Level: Applying Client Need &Sub: Psychosocial Integrity Standards: QSEN Competencies: Patient-centered care | AACN Essential Competencies: Essential IX: Baccalaureate generalist nursing practice | NLN Competencies: Professional identity | Nursing/Integrated Concepts: Nursing Process: Implementation/Coordination of care Learning Outcome: LO 12.6 Summarize nursing management for the child with a chronic condition to support transition to school and adult living. MNL LO: Apply the nursing process to care for the needs of the hospitalized child. 9) The nurse is planning activities for a toddler with a birth injury of a torn brachial plexus that resulted in muscle atrophy and weakness of his right arm. Which nursing intervention is most appropriate for this client? 1. Offering the toddler a choice of clothing 2. Asking the toddler if he would like to take his medicine 3. Dressing the toddler 4. Feeding the toddler Answer: 1 Explanation: 1. Toddlers are developing autonomy, self-control, and independence. Offering the toddler a choice contributes to their sense of autonomy. However, taking medicine is not within the toddler's realm of choice. Dressing and feeding the toddler does not encourage independence and will eventually cause frustration for both parent and toddler. The toddler must learn how to do these activities despite the physical limitations of the right arm. 2. Toddlers are developing autonomy, self-control, and independence. Offering the toddler a choice contributes to their sense of autonomy. However, taking medicine is not within the toddler's realm of choice. Dressing and feeding the toddler does not encourage independence and will eventually cause frustration for both parent and toddler. The toddler must learn how to do these activities despite the physical limitations of the right arm. 3. Toddlers are developing autonomy, self-control, and independence. Offering the toddler a choice contributes to their sense of autonomy. However, taking medicine is not within the toddler's realm of choice. Dressing and feeding the toddler does not encourage independence and will eventually cause frustration for both parent and toddler. The toddler must learn how to do these activities despite the physical limitations of the right arm. 4. Toddlers are developing autonomy, self-control, and independence. Offering the toddler a choice contributes to their sense of autonomy. However, taking medicine is not within the toddler's realm of choice. Dressing and feeding the toddler does not encourage independence and will eventually cause frustration for both parent and toddler. The toddler must learn how to do these activities despite the physical limitations of the right arm. Page Ref: 252 Cognitive Level: Applying Client Need &Sub: Health Promotion and Maintenance Standards: QSEN Competencies: Patient-centered care | AACN Essential Competencies: Essential IX: Baccalaureate generalist nursing practice | NLN Competencies: Nursing judgement | Nursing/Integrated Concepts: Nursing Process: Implementation/Coordination of care Learning Outcome: LO 12.4 Assess the family of a child with a chronic condition. MNL LO: Apply the nursing process to care for the needs of a child with a chronic illness. 10) The nurse has set up a group discussion for several families with chronically ill children. The nurse informs these parents that they may face which ethical issue? 1. Normalization 2. Withholding and refusal of treatment 3. Repeated hospital admissions 4. Lack of proper dietary needs Answer: 2 Explanation: 1. Withholding and refusal of treatment is an ethical issue involving the life and quality of life of the child. Normalization is a family process of adaptation as the family members cope with daily life with their child. Lack of dietary needs is not an ethical issue, nor is repeated hospital admissions. 2. Withholding and refusal of treatment is an ethical issue involving the life and quality of life of the child. Normalization is a family process of adaptation as the family members cope with daily life with their child. Lack of dietary needs is not an ethical issue, nor is repeated hospital admissions. 3. Withholding and refusal of treatment is an ethical issue involving the life and quality of life of the child. Normalization is a family process of adaptation as the family members cope with daily life with their child. Lack of dietary needs is not an ethical issue, nor is repeated hospital admissions. 4. Withholding and refusal of treatment is an ethical issue involving the life and quality of life of the child. Normalization is a family process of adaptation as the family members cope with daily life with their child. Lack of dietary needs is not an ethical issue, nor is repeated hospital admissions. Page Ref: 253 Cognitive Level: Applying Client Need &Sub: Psychosocial Integrity Standards: QSEN Competencies: Patient-centered care | AACN Essential Competencies: Essential VII: Clinical prevention and population health | NLN Competencies: Professional identity | Nursing/Integrated Concepts: Nursing Process: Implementation/Health teaching and health promotion Learning Outcome: LO 12.5 Prepare the family of a child with a chronic condition to effectively care for the child in the home. MNL LO: Analyze the impact the diagnosis of a chronic illness has on the pediatric client and family. 11) At the conclusion of teaching parents about cerebral palsy, the nurse asks, "What is your hope for your toddler with cerebral palsy?" Which reply from a parent best indicates an understanding of a realistic achievement for the child? 1. "I hope my child qualifies for the Winter Olympics like I did." 2. "I hope my child just enjoys life." 3. "I hope my child will attend our neighborhood school." 4. "I hope my child is liked and accepted by other children." Answer: 3 Explanation: 1. Expecting a child with cerebral palsy to do well in the local school is a realistic hope that the child can possibly achieve. A child with cerebral palsy does not have the gross motor skills to qualify for the Olympics; thus, this is unrealistic. A hope for the child to enjoy life is realistic, but is not an achievement for the child. A hope that the child is liked and accepted by other children is realistic, but this hope is also dependent on other children. 2. Expecting a child with cerebral palsy to do well in the local school is a realistic hope that the child can possibly achieve. A child with cerebral palsy does not have the gross motor skills to qualify for the Olympics; thus, this is unrealistic. A hope for the child to enjoy life is realistic, but is not an achievement for the child. A hope that the child is liked and accepted by other children is realistic, but this hope is also dependent on other children. 3. Expecting a child with cerebral palsy to do well in the local school is a realistic hope that the child can possibly achieve. A child with cerebral palsy does not have the gross motor skills to qualify for the Olympics; thus, this is unrealistic. A hope for the child to enjoy life is realistic, but is not an achievement for the child. A hope that the child is liked and accepted by other children is realistic, but this hope is also dependent on other children. 4. Expecting a child with cerebral palsy to do well in the local school is a realistic hope that the child can possibly achieve. A child with cerebral palsy does not have the gross motor skills to qualify for the Olympics; thus, this is unrealistic. A hope for the child to enjoy life is realistic, but is not an achievement for the child. A hope that the child is liked and accepted by other children is realistic, but this hope is also dependent on other children. Page Ref: 253 Cognitive Level: Applying Client Need &Sub: Health Promotion and Maintenance Standards: QSEN Competencies: Quality improvement | AACN Essential Competencies: Essential IX: Baccalaureate generalist nursing practice | NLN Competencies: Professional identity | Nursing/Integrated Concepts: Nursing Process: Evaluation/Quality of practice Learning Outcome: LO 12.5 Prepare the family of a child with a chronic condition to effectively care for the child in the home. MNL LO: Apply the general concepts related to caring for a child with a chronic illness. 12) A family actively participates in school functions. One of the children is paraplegic and requires a wheelchair for mobility. Which process does the nurse determine the family is working on based on these assessment findings? 1. Stagnation 2. Normalization 3. Isolation 4. Interaction Answer: 2 Explanation: 1. The family is normalizing life with the children through activities. The family is not staying at home because one member cannot walk; rather, the family is moving on to full participation in life. The family is interacting with others through the process of normalization. 2. The family is normalizing life with the children through activities. The family is not staying at home because one member cannot walk; rather, the family is moving on to full participation in life. The family is interacting with others through the process of normalization. 3. The family is normalizing life with the children through activities. The family is not staying at home because one member cannot walk; rather, the family is moving on to full participation in life. The family is interacting with others through the process of normalization. 4. The family is normalizing life with the children through activities. The family is not staying at home because one member cannot walk; rather, the family is moving on to full participation in life. The family is interacting with others through the process of normalization. Page Ref: 255 Cognitive Level: Applying Client Need &Sub: Psychosocial Integrity Standards: QSEN Competencies: Quality improvement | AACN Essential Competencies: Essential IX: Baccalaureate generalist nursing practice | NLN Competencies: Human flourishing | Nursing/Integrated Concepts: Nursing Process: Evaluation/Quality of practice Learning Outcome: LO 12.7 Discuss the family's role in care coordination. MNL LO: Analyze the role of the family in pediatric care. 13) A 5-year-old sibling of a 9-year-old child with cystic fibrosis tells the nurse, "I wish I had a breathing disease, too." The nurse knows the parents strive to spend quality time with each child and with both children together. What is the sibling currently experiencing? 1. Jealousy 2. Isolation 3. Loneliness 4. Anger Answer: 1 Explanation: 1. The child with cystic fibrosis has something the younger child does not have. Cystic fibrosis brings the affected child more attention from others. Even if parents strive to spend more time with siblings of ill children, the well-child will be jealous because the situation can never be equal. The 5-year-old child does not understand the complications of the disease and only sees the 9-year-old child treated differently. Siblings of ill children may experience loneliness, isolation, or anger; but the child's comment does not support these feelings. 2. The child with cystic fibrosis has something the younger child does not have. Cystic fibrosis brings the affected child more attention from others. Even if parents strive to spend more time with siblings of ill children, the well-child will be jealous because the situation can never be equal. The 5-year-old child does not understand the complications of the disease and only sees the 9-year-old child treated differently. Siblings of ill children may experience loneliness, isolation, or anger; but the child's comment does not support these feelings. 3. The child with cystic fibrosis has something the younger child does not have. Cystic fibrosis brings the affected child more attention from others. Even if parents strive to spend more time with siblings of ill children, the well-child will be jealous because the situation can never be equal. The 5-year-old child does not understand the complications of the disease and only sees the 9-year-old child treated differently. Siblings of ill children may experience loneliness, isolation, or anger; but the child's comment does not support these feelings. 4. The child with cystic fibrosis has something the younger child does not have. Cystic fibrosis brings the affected child more attention from others. Even if parents strive to spend more time with siblings of ill children, the well-child will be jealous because the situation can never be equal. The 5-year-old child does not understand the complications of the disease and only sees the 9-year-old child treated differently. Siblings of ill children may experience loneliness, isolation, or anger; but the child's comment does not support these feelings. Page Ref: 248-249 Cognitive Level: Applying Client Need &Sub: Psychosocial Integrity Standards: QSEN Competencies: Patient-centered care | AACN Essential Competencies: Essential VII: Clinical prevention and population health | NLN Competencies: Human flourishing | Nursing/Integrated Concepts: Nursing Process: Diagnosis/Health teaching and health promotion Learning Outcome: LO 12.5 Prepare the family of a child with a chronic condition to effectively care for the child in the home. MNL LO: Analyze the impact the diagnosis of a chronic illness has on the pediatric client and family. 14) There are many healthcare needs of children with chronic conditions. What nursing strategy would best help parents with continuity of care? 1. Include the family and older child in decision making. 2. Assist the family in gaining transportation to healthcare appointments. 3. Provide the family with resources such as social services. 4. Recognize and respect the cultural needs of the family. Answer: 1 Explanation: 1. Continuity of care involves the family and child's participation in their health care. Access to transportation involves access to care, not continuity. Providing resources such as social services is related to comprehensiveness of care, not to continuity. Recognizing and respecting cultural needs are part of the degree to which healthcare services, not continuity of care, are provided. 2. Continuity of care involves the family and child's participation in their health care. Access to transportation involves access to care, not continuity. Providing resources such as social services is related to comprehensiveness of care, not to continuity. Recognizing and respecting cultural needs are part of the degree to which healthcare services, not continuity of care, are provided. 3. Continuity of care involves the family and child's participation in their health care. Access to transportation involves access to care, not continuity. Providing resources such as social services is related to comprehensiveness of care, not to continuity. Recognizing and respecting cultural needs are part of the degree to which healthcare services, not continuity of care, are provided. 4. Continuity of care involves the family and child's participation in their health care. Access to transportation involves access to care, not continuity. Providing resources such as social services is related to comprehensiveness of care, not to continuity. Recognizing and respecting cultural needs are part of the degree to which healthcare services, not continuity of care, are provided. Page Ref: 249 Cognitive Level: Applying Client Need &Sub: Health Promotion and Maintenance Standards: QSEN Competencies: Patient-centered care | AACN Essential Competencies: Essential IX: Baccalaureate generalist nursing practice | NLN Competencies: Nursing judgement | Nursing/Integrated Concepts: Nursing Process: Implementation/Coordination of care Learning Outcome: LO 12.5 Prepare the family of a child with a chronic condition to effectively care for the child in the home. MNL LO: Apply the nursing process to care for the needs of a child with a chronic illness. 15) It is important that parents of adolescents with special needs transition care of the adolescent so they can learn to make good decisions on their own. Which items are considered transitional needs? Select all that apply. 1. Attending school 2. Discussing sexual matters 3. Letting most friends know of the medical condition 4. Socialization beyond the family 5. To write his or her own individualized healthcare plan Answer: 1, 2, 4 Explanation: 1. Transitional needs toward independence include attending school, discussion of sexual matters, and socialization beyond the family. The other areas are not transitional needs. 2. Transitional needs toward independence include attending school, discussion of sexual matters, and socialization beyond the family. The other areas are not transitional needs. 3. Transitional needs toward independence include attending school, discussion of sexual matters, and socialization beyond the family. The other areas are not transitional needs. 4. Transitional needs toward independence include attending school, discussion of sexual matters, and socialization beyond the family. The other areas are not transitional needs. 5. Transitional needs toward independence include attending school, discussion of sexual matters, and socialization beyond the family. The other areas are not transitional needs. Page Ref: 257-258 Cognitive Level: Applying Client Need &Sub: Psychosocial Integrity Standards: QSEN Competencies: Patient-centered care | AACN Essential Competencies: Essential VII: Clinical prevention and population health | NLN Competencies: Human flourishing | Nursing/Integrated Concepts: Nursing Process: Planning/Health teaching and health promotion Learning Outcome: LO 12.6 Summarize nursing management for the child with a chronic condition to support transition to school and adult living. MNL LO: Evaluate the developmental considerations when caring for a child with a chronic illness. 16) The nurse is planning care for a preschool-age client who has cerebral palsy (CP). Which interventions are appropriate for this client? Select all that apply. 1. Providing heath supervision 2. Collaborating with physical therapy 3. Assisting with planning educational services 4. Prescribing medication for spasticity 5. Promoting growth and development Answer: 1, 2, 3, 5 Explanation: 1. Appropriate interventions for the nurse who is providing care to a client with a chronic condition include providing health supervision, collaborating with other specialties, assisting with planning educational services, and promoting growth and development. It is outside the scope of nursing practice to prescribe medication. The nurse could, however, administer prescribed medications if appropriate. 2. Appropriate interventions for the nurse who is providing care to a client with a chronic condition include providing health supervision, collaborating with other specialties, assisting with planning educational services, and promoting growth and development. It is outside the scope of nursing practice to prescribe medication. The nurse could, however, administer prescribed medications if appropriate. 3. Appropriate interventions for the nurse who is providing care to a client with a chronic condition include providing health supervision, collaborating with other specialties, assisting with planning educational services, and promoting growth and development. It is outside the scope of nursing practice to prescribe medication. The nurse could, however, administer prescribed medications if appropriate. 4. Appropriate interventions for the nurse who is providing care to a client with a chronic condition include providing health supervision, collaborating with other specialties, assisting with planning educational services, and promoting growth and development. It is outside the scope of nursing practice to prescribe medication. The nurse could, however, administer prescribed medications if appropriate. 5. Appropriate interventions for the nurse who is providing care to a client with a chronic condition include providing health supervision, collaborating with other specialties, assisting with planning educational services, and promoting growth and development. It is outside the scope of nursing practice to prescribe medication. The nurse could, however, administer prescribed medications if appropriate. Page Ref: 249 Cognitive Level: Applying Client Need &Sub: Health Promotion and Maintenance Standards: QSEN Competencies: Patient-centered care | AACN Essential Competencies: Essential IX: Baccalaureate generalist nursing practice | NLN Competencies: Human flourishing | Nursing/Integrated Concepts: Nursing Process: Implementation/Coordination of care Learning Outcome: LO 12.3 Describe the nurse's role in caring for a child with a chronic condition. MNL LO: Apply the nursing process to care for the needs of a child with a chronic illness. 17) Match the formalized plan for the child with a chronic condition with its description. A. Individualized family service plan (IFSP) B. Individualized education plan (IEP) C. Individualized health plan (IHP) D. Individualized transition plan (ITP) 1. Helps individuals receive vocational training and move successfully from the home into other community settings. 2. Developed for a child with cognitive, motor, social, and communication impairment who needs special education services. 3. Developed for the early intervention process for infants with special healthcare needs and their families. 4. Developed for the child with medical conditions that need to be managed within the school setting. Answer: 1/D, 2/B, 3/A, 4/C 1. Individualized transition plan (ITP) 2. Individualized education plan (IEP) 3. Individualized family service plan (IFSP) 4. Individualized health plan (IHP) Explanation: Because some children need medications or other therapies during school hours, the parents and child, school nurse, teacher, and school administrators develop a plan to manage the child's condition during school hours. ITP: Helps individuals receive vocational training and move successfully from the home into other community settings. IEP: Developed for a child with cognitive, motor, social, and communication impairment who needs special education services. IFSP: Developed for the early intervention process for infants with special healthcare needs and their families. IHP: Developed for the child with medical conditions that need to be managed within the school setting. Page Ref: 255-256 Cognitive Level: Analyzing Client Need &Sub: Health Promotion and Maintenance Standards: QSEN Competencies: Patient-centered care | AACN Essential Competencies: Essential VI: Interprofessional communication and collaboration for improving patient health outcomes | NLN Competencies: Human flourishing | Nursing/Integrated Concepts: Nursing Process: Assessment/Coordination of care Learning Outcome: LO 12.6 Summarize nursing management for the child with a chronic condition to support transition to school and adult living. MNL LO: Examine the role of the nurse in promoting culturally competent family-centered care. 18) Which stressor is common in the hospitalized toddler with a chronic disorder? Select all that apply. 1. Fear of painful procedures 2. Self-concept 3. Interruption of normal routines 4. Unfamiliarity of caregivers 5. Isolation Answer: 1, 3, 4 Explanation: 1. This is a stressor common in the hospitalized toddler with a chronic disorder. 2. This is a stressor common in the hospitalized adolescent with a chronic disorder. 3. This is a stressor common in the hospitalized toddler with a chronic disorder. 4. This is a stressor common in the hospitalized toddler with a chronic disorder. 5. This is a stressor common in the hospitalized adolescent with a chronic disorder. Page Ref: 252 Cognitive Level: Analyzing Client Need &Sub: Health Promotion and Maintenance Standards: QSEN Competencies: Patient-centered care | AACN Essential Competencies: Essential IX: Baccalaureate generalist nursing practice | NLN Competencies: Human flourishing | Nursing/Integrated Concepts: Nursing Process: Assessment/Coordination of care Learning Outcome: LO 12.4 Assess the family of a child with a chronic condition. MNL LO: Differentiate developmentally appropriate care environments for the pediatric client and family. Principles of Pediatric Nursing: Caring for Children, 7e (Ball et al.) Chapter 13 The Child with a Life-Threatening Condition and End-of-Life Care 1) The parents of a toddler-age child who sustained severe head trauma from falling out a second-story window are arguing in the pediatric intensive-care unit (PICU) and blaming each other for the child's accident. Which nursing diagnosis is most appropriate for this family? 1. Parental Role Conflict Related to Protecting the Child 2. Hopelessness Related to the Child's Deteriorating Condition 3. Anxiety Related to the Critical-Care-Unit Environment 4. Family Coping: Compromised, Related to the Child's Critical Injury Answer: 4 Explanation: 1. The parents are displaying ineffective coping behaviors as a family. Parental role conflict does not refer to the parents' argument in the PICU, but means a parent is conflicted or confused about some aspect of the parental role. Each parent may be experiencing hopelessness, frustration, and anxiety, but they are not coping well as a family unit. 2. The parents are displaying ineffective coping behaviors as a family. Parental role conflict does not refer to the parents' argument in the PICU, but means a parent is conflicted or confused about some aspect of the parental role. Each parent may be experiencing hopelessness, frustration, and anxiety, but they are not coping well as a family unit. 3. The parents are displaying ineffective coping behaviors as a family. Parental role conflict does not refer to the parents' argument in the PICU, but means a parent is conflicted or confused about some aspect of the parental role. Each parent may be experiencing hopelessness, frustration, and anxiety, but they are not coping well as a family unit. 4. The parents are displaying ineffective coping behaviors as a family. Parental role conflict does not refer to the parents' argument in the PICU, but means a parent is conflicted or confused about some aspect of the parental role. Each parent may be experiencing hopelessness, frustration, and anxiety, but they are not coping well as a family unit. Page Ref: 266 Cognitive Level: Analyzing Client Need &Sub: Psychosocial Integrity Standards: QSEN Competencies: Patient-centered care | AACN Essential Competencies: Essential IX: Baccalaureate generalist nursing practice | NLN Competencies: Nursing judgement | Nursing/Integrated Concepts: Nursing Process: Diagnosis/Coordination of care Learning Outcome: LO 13.3 Identify the coping mechanisms used by the child and family in response to stress. MNL LO: Examine health promotion, stress reduction therapies, and safety for hospitalized child and family. 2) The emergency-room nurse receives a preschool-age child who was hit by a car. Which nursing interventions are a priority for this child? Select all that apply. 1. Performing a rapid head-to-toe assessment 2. Recording the parents' insurance information 3. Assessing airway, breathing, and circulation 4. Asking the parents about organ donation 5. Asking the parents if anyone witnessed the accident Answer: 1, 3 Explanation: 1. Assessing airway, breathing, and circulation and performing a rapid head-to-toe assessment are the priority nursing interventions. Asking the parents about organ donation is insensitive until the extent of the child's injuries is known. Recording insurance information is necessary but should never come before lifesaving assessment and intervention. Detailed information about the accident is helpful in determining the child's point of impact with the car and mechanism of injury, but this is not the initial priority. 2. Assessing airway, breathing, and circulation and performing a rapid head-to-toe assessment are the priority nursing interventions. Asking the parents about organ donation is insensitive until the extent of the child's injuries is known. Recording insurance information is necessary but should never come before lifesaving assessment and intervention. Detailed information about the accident is helpful in determining the child's point of impact with the car and mechanism of injury, but this is not the initial priority. 3. Assessing airway, breathing, and circulation and performing a rapid head-to-toe assessment are the priority nursing interventions. Asking the parents about organ donation is insensitive until the extent of the child's injuries is known. Recording insurance information is necessary but should never come before lifesaving assessment and intervention. Detailed information about the accident is helpful in determining the child's point of impact with the car and mechanism of injury, but this is not the initial priority. 4. Assessing airway, breathing, and circulation and performing a rapid head-to-toe assessment are the priority nursing interventions. Asking the parents about organ donation is insensitive until the extent of the child's injuries is known. Recording insurance information is necessary but should never come before lifesaving assessment and intervention. Detailed information about the accident is helpful in determining the child's point of impact with the car and mechanism of injury, but this is not the initial priority. 5. Assessing airway, breathing, and circulation and performing a rapid head-to-toe assessment are the priority nursing interventions. Asking the parents about organ donation is insensitive until the extent of the child's injuries is known. Recording insurance information is necessary but should never come before lifesaving assessment and intervention. Detailed information about the accident is helpful in determining the child's point of impact with the car and mechanism of injury, but this is not the initial priority. Page Ref: 262 Cognitive Level: Analyzing Client Need &Sub: Physiological Integrity Standards: QSEN Competencies: Patient-centered care | AACN Essential Competencies: Essential IX: Baccalaureate generalist nursing practice | NLN Competencies: Nursing judgement | Nursing/Integrated Concepts: Nursing Process: Implementation/Coordination of care Learning Outcome: LO 13.4 Develop a nursing care plan for the child with a life-threatening illness or injury. MNL LO: Apply the nursing process to care for the needs of the hospitalized child. 3) A young school-age child is in the pediatric intensive-care unit (PICU) with a fractured femur and head trauma. The child was not wearing a helmet while riding his new bicycle on the highway and collided with a car. Which nursing diagnoses may be appropriate for this family? Select all that apply. 1. Guilt Related to Lack of Child Supervision and Safety Precautions 2. Family Coping: Compromised, Related to the Critical Injury of the Child 3. Parental Role Conflict Related to Child's Injuries and PICU Policies 4. Knowledge Deficit Related to Home Care of Fractured Femur 5. Anger Related to Feelings of Helplessness Answer: 1, 2, 3, 5 Explanation: 1. All of these nursing diagnoses except Knowledge Deficit are possible in this situation. Although planning for discharge begins with admission, it is too early to begin teaching the parents about home care. The astute and experienced PICU nurse is prepared to recognize current problems and intervene appropriately. 2. All of these nursing diagnoses except Knowledge Deficit are possible in this situation. Although planning for discharge begins with admission, it is too early to begin teaching the parents about home care. The astute and experienced PICU nurse is prepared to recognize current problems and intervene appropriately. 3. All of these nursing diagnoses except Knowledge Deficit are possible in this situation. Although planning for discharge begins with admission, it is too early to begin teaching the parents about home care. The astute and experienced PICU nurse is prepared to recognize current problems and intervene appropriately. 4. All of these nursing diagnoses except Knowledge Deficit are possible in this situation. Although planning for discharge begins with admission, it is too early to begin teaching the parents about home care. The astute and experienced PICU nurse is prepared to recognize current problems and intervene appropriately. 5. All of these nursing diagnoses except Knowledge Deficit are possible in this situation. Although planning for discharge begins with admission, it is too early to begin teaching the parents about home care. The astute and experienced PICU nurse is prepared to recognize current problems and intervene appropriately. Page Ref: 266 Cognitive Level: Analyzing Client Need &Sub: Psychosocial Integrity Standards: QSEN Competencies: Patient-centered care | AACN Essential Competencies: Essential IX: Baccalaureate generalist nursing practice | NLN Competencies: Nursing judgement | Nursing/Integrated Concepts: Nursing Process: Diagnosis/Coordination of care Learning Outcome: LO 13.2 Examine the family's experience and reactions to having a child with a life-threatening illness or injury. MNL LO: Apply the nursing process to care for the needs of the hospitalized child. 4) The nurse must prepare parents to see their adolescent daughter in the pediatric intensive-care unit (PICU). The child arrived by life flight after experiencing multiple traumas in a car accident involving a suspected drunk driver. At this time, which statement by the nurse to the family is the most appropriate? 1. "Don't worry; everything will be okay. We will take excellent care of your child." 2. "You should press charges against the drunk driver." 3. "Your child's leg was crushed and may have to be amputated." 4. "Your child's condition is very critical; her face is swollen, and she may not look like herself." Answer: 4 Explanation: 1. The priority is to prepare the parents for the child's changed appearance. The nurse must not offer false reassurance nor project future stressful events. Truthful statements about the child's condition can be introduced after the parents have seen the child and grasped the situation. The nurse supports the family but remains nonjudgmental about accident details. 2. The priority is to prepare the parents for the child's changed appearance. The nurse must not offer false reassurance nor project future stressful events. Truthful statements about the child's condition can be introduced after the parents have seen the child and grasped the situation. The nurse supports the family but remains nonjudgmental about accident details. 3. The priority is to prepare the parents for the child's changed appearance. The nurse must not offer false reassurance nor project future stressful events. Truthful statements about the child's condition can be introduced after the parents have seen the child and grasped the situation. The nurse supports the family but remains nonjudgmental about accident details. 4. The priority is to prepare the parents for the child's changed appearance. The nurse must not offer false reassurance nor project future stressful events. Truthful statements about the child's condition can be introduced after the parents have seen the child and grasped the situation. The nurse supports the family but remains nonjudgmental about accident details. Page Ref: 267-268 Cognitive Level: Applying Client Need &Sub: Psychosocial Integrity Standards: QSEN Competencies: Patient-centered care | AACN Essential Competencies: Essential IX: Baccalaureate generalist nursing practice | NLN Competencies: Professional identity | Nursing/Integrated Concepts: Nursing Process: Implementation/Quality of practice Learning Outcome: LO 13.2 Examine the family's experience and reactions to having a child with a life-threatening illness or injury. MNL LO: Analyze the reactions by the child, parents, and siblings related to life-threatening illnesses. 5) An adolescent with cystic fibrosis is intubated with an endotracheal tube. Which nursing diagnosis is most appropriate for this adolescent? 1. Potential for Imbalanced Nutrition, More Than Body Requirements Related to Inactivity 2. Anxiety Related to Leaving Chores Undone at Home 3. Potential for Fear of Future Pain Related to Medical Procedures 4. Powerlessness (Moderate) Related to Inability to Speak to or Communicate with Friends Answer: 4 Explanation: 1. The adolescent values communication with peers and may feel frustrated that he cannot speak to them while intubated. The adolescent is present-oriented and is unlikely to worry about household chores or future unknown procedures. The adolescent with cystic fibrosis is likely to be underweight and is unlikely to take in more calories than needed while intubated. 2. The adolescent values communication with peers and may feel frustrated that he cannot speak to them while intubated. The adolescent is present-oriented and is unlikely to worry about household chores or future unknown procedures. The adolescent with cystic fibrosis is likely to be underweight and is unlikely to take in more calories than needed while intubated. 3. The adolescent values communication with peers and may feel frustrated that he cannot speak to them while intubated. The adolescent is present-oriented and is unlikely to worry about household chores or future unknown procedures. The adolescent with cystic fibrosis is likely to be underweight and is unlikely to take in more calories than needed while intubated. 4. The adolescent values communication with peers and may feel frustrated that he cannot speak to them while intubated. The adolescent is present-oriented and is unlikely to worry about household chores or future unknown procedures. The adolescent with cystic fibrosis is likely to be underweight and is unlikely to take in more calories than needed while intubated. Page Ref: 264 Cognitive Level: Analyzing Client Need &Sub: Psychosocial Integrity Standards: QSEN Competencies: Patient-centered care | AACN Essential Competencies: Essential II: Basic organizational and systems leadership for quality care and patient safety | NLN Competencies: Professional identity | Nursing/Integrated Concepts: Nursing Process: Diagnosis Learning Outcome: LO 13.4 Develop a nursing care plan for the child with a life-threatening illness or injury. MNL LO: Apply the nursing process to care for child and family with life-threatening illness/at end of life. 6) An adolescent client has a stiff neck, a headache, a fever of 103 degrees Fahrenheit, and purpuric lesions noted on the legs. Although the adolescent's physical needs take priority at the present time, the nurse can expect which to be the most significant psychological stressor for this adolescent? 1. Separation from parents and home 2. Separation from friends and permanent changes in appearance 3. Fear of painful procedures and bodily mutilation 4. Fear of getting behind in schoolwork Answer: 2 Explanation: 1. Adolescents are developing their identity and rely most on their friends. They are concerned about their appearance and how they look compared to their peers. Separation from parents and home is the main psychological stressor for infants and toddlers. Preschool-age children fear pain and bodily mutilation. School-age children are developing a sense of industry and fear getting behind in schoolwork. 2. Adolescents are developing their identity and rely most on their friends. They are concerned about their appearance and how they look compared to their peers. Separation from parents and home is the main psychological stressor for infants and toddlers. Preschool-age children fear pain and bodily mutilation. School-age children are developing a sense of industry and fear getting behind in schoolwork. 3. Adolescents are developing their identity and rely most on their friends. They are concerned about their appearance and how they look compared to their peers. Separation from parents and home is the main psychological stressor for infants and toddlers. Preschool-age children fear pain and bodily mutilation. School-age children are developing a sense of industry and fear getting behind in schoolwork. 4. Adolescents are developing their identity and rely most on their friends. They are concerned about their appearance and how they look compared to their peers. Separation from parents and home is the main psychological stressor for infants and toddlers. Preschool-age children fear pain and bodily mutilation. School-age children are developing a sense of industry and fear getting behind in schoolwork. Page Ref: 264 Cognitive Level: Applying Client Need &Sub: Psychosocial Integrity Standards: QSEN Competencies: Patient-centered care | AACN Essential Competencies: Essential IX: Baccalaureate generalist nursing practice | NLN Competencies: Nursing judgement | Nursing/Integrated Concepts: Nursing Process: Assessment/Quality of practice Learning Outcome: LO 13.3 Identify the coping mechanisms used by the child and family in response to stress. MNL LO: Examine health promotion, stress reduction therapies, and safety for hospitalized child and family. 7) A school bus carrying children in grades K-12 crashed into a ravine. The critically injured children were transported by ambulance and admitted to the pediatric intensive-care unit (PICU). The nurse is concerned about calming the frightened children. Which nursing intervention is most appropriate to achieve the goal of calming the frightened children? 1. Tell the children that the physicians are competent. 2. Assure the children that the nurses are caring. 3. Explain that the PICU equipment is state of the art. 4. Call the children's parents to come into the PICU. Answer: 4 Explanation: 1. A sense of physical and psychological security is best achieved by the presence of parents. Children at all developmental levels look first to their parents or whoever acts as their parents for safety and security. Healthcare providers, no matter how competent or caring, cannot substitute for parents. Children often neither recognize nor care about state-of-the-art equipment. 2. A sense of physical and psychological security is best achieved by the presence of parents. Children at all developmental levels look first to their parents or whoever acts as their parents for safety and security. Healthcare providers, no matter how competent or caring, cannot substitute for parents. Children often neither recognize nor care about state-of-the-art equipment. 3. A sense of physical and psychological security is best achieved by the presence of parents. Children at all developmental levels look first to their parents or whoever acts as their parents for safety and security. Healthcare providers, no matter how competent or caring, cannot substitute for parents. Children often neither recognize nor care about state-of-the-art equipment. 4. A sense of physical and psychological security is best achieved by the presence of parents. Children at all developmental levels look first to their parents or whoever acts as their parents for safety and security. Healthcare providers, no matter how competent or caring, cannot substitute for parents. Children often neither recognize nor care about state-of-the-art equipment. Page Ref: 263 Cognitive Level: Applying Client Need &Sub: Psychosocial Integrity Standards: QSEN Competencies: Patient-centered care | AACN Essential Competencies: Essential IX: Baccalaureate generalist nursing practice | NLN Competencies: Nursing judgement | Nursing/Integrated Concepts: Nursing Process: Implementation/Coordination of care Learning Outcome: LO 13.1 Summarize the effects of a life-threatening illness or injury on children. MNL LO: Analyze the reactions by the child, parents, and siblings related to life-threatening illnesses. 8) A school-age child with congenital heart block codes in the emergency department (ED). The parents witness this and stare at the resuscitation scene unfolding before them. Which nursing intervention is most appropriate in this situation? 1. Ask the parents to leave until the child has stabilized. 2. Ask the parents to call the family to come into watch the resuscitation. 3. Ask the parents to sit near the child's face and hold her hand. 4. Ask the parents to stand at the foot of the cart to watch. Answer: 3 Explanation: 1. Parents should be helped to support their child through emergency procedures, if they are able. Parents should never be asked to take part in emergency efforts unless absolutely necessary. Merely watching the resuscitation serves no purpose for the child. If the parents interfere with resuscitation efforts or they are unable to tolerate the situation, they can be asked to leave later. 2. Parents should be helped to support their child through emergency procedures, if they are able. Parents should never be asked to take part in emergency efforts unless absolutely necessary. Merely watching the resuscitation serves no purpose for the child. If the parents interfere with resuscitation efforts or they are unable to tolerate the situation, they can be asked to leave later. 3. Parents should be helped to support their child through emergency procedures, if they are able. Parents should never be asked to take part in emergency efforts unless absolutely necessary. Merely watching the resuscitation serves no purpose for the child. If the parents interfere with resuscitation efforts or they are unable to tolerate the situation, they can be asked to leave later. 4. Parents should be helped to support their child through emergency procedures, if they are able. Parents should never be asked to take part in emergency efforts unless absolutely necessary. Merely watching the resuscitation serves no purpose for the child. If the parents interfere with resuscitation efforts or they are unable to tolerate the situation, they can be asked to leave later. Page Ref: 268 Cognitive Level: Applying Client Need &Sub: Psychosocial Integrity Standards: QSEN Competencies: Patient-centered care | AACN Essential Competencies: Essential IX: Baccalaureate generalist nursing practice | NLN Competencies: Human flourishing | Nursing/Integrated Concepts: Nursing Process: Implementation/Coordination of care Learning Outcome: LO 13.6 Develop a nursing care plan to provide family-centered care for the dying child and family. MNL LO: Apply the nursing process to care for child and family with life-threatening illness/at end of life. 9) A child is admitted to the neonatal intensive care unit (NICU). The parents are concerned because they cannot stay for long hours to visit. Which statement made by the nurse is most appropriate? 1. "One of you might take a leave of absence to be here more." 2. "Parents often feel this way; would you be interested in talking with others who have experienced having a child in the NICU?" 3. "Perhaps the grandparents can make the visits for you." 4. "Why can't you visit after work every day?" Answer: 2 Explanation: 1. "Parents often feel this way; would you be interested in talking with others who have experienced having a child in the NICU?" is therapeutic; it focuses on feelings and offers support to the parents. The other options do not focus on how the parents feel and attempt to solve the issue rather than allow for the parents to deal with their feelings and form solutions. 2. "Parents often feel this way; would you be interested in talking with others who have experienced having a child in the NICU?" is therapeutic; it focuses on feelings and offers support to the parents. The other options do not focus on how the parents feel and attempt to solve the issue rather than allow for the parents to deal with their feelings and form solutions. 3. "Parents often feel this way; would you be interested in talking with others who have experienced having a child in the NICU?" is therapeutic; it focuses on feelings and offers support to the parents. The other options do not focus on how the parents feel and attempt to solve the issue rather than allow for the parents to deal with their feelings and form solutions. 4. "Parents often feel this way; would you be interested in talking with others who have experienced having a child in the NICU?" is therapeutic; it focuses on feelings and offers support to the parents. The other options do not focus on how the parents feel and attempt to solve the issue rather than allow for the parents to deal with their feelings and form solutions. Page Ref: 268 Cognitive Level: Applying Client Need &Sub: Psychosocial Integrity Standards: QSEN Competencies: Patient-centered care | AACN Essential Competencies: Essential IX: Baccalaureate generalist nursing practice | NLN Competencies: Human flourishing | Nursing/Integrated Concepts: Nursing Process: Implementation/Coordination of care Learning Outcome: LO 13.6 Develop a nursing care plan to provide family-centered care for the dying child and family. MNL LO: Apply the nursing process to care for child and family with life-threatening illness/at end of life. 10) A school-age client is admitted to the pediatric intensive care unit (PICU) in critical condition after a motor vehicle accident. Which intervention should be implemented at this time? 1. Maintain consistent caregivers. 2. Turn the lights off at night. 3. Keep alarm levels low. 4. Consult the hospital play therapist. Answer: 1 Explanation: 1. The intensive care environment is fast-paced, overwhelming, and frightening. Maintaining consistent caregivers is invaluable in developing a familiar and trusting relationship with the child. Turning off the lights in an intensive care environment is not feasible. Keeping alarm levels low could increase risk of injury if an alarm is not heard by staff. Consulting the play therapist is not appropriate at this time. 2. The intensive care environment is fast-paced, overwhelming, and frightening. Maintaining consistent caregivers is invaluable in developing a familiar and trusting relationship with the child. Turning off the lights in an intensive care environment is not feasible. Keeping alarm levels low could increase risk of injury if an alarm is not heard by staff. Consulting the play therapist is not appropriate at this time. 3. The intensive care environment is fast-paced, overwhelming, and frightening. Maintaining consistent caregivers is invaluable in developing a familiar and trusting relationship with the child. Turning off the lights in an intensive care environment is not feasible. Keeping alarm levels low could increase risk of injury if an alarm is not heard by staff. Consulting the play therapist is not appropriate at this time. 4. The intensive care environment is fast-paced, overwhelming, and frightening. Maintaining consistent caregivers is invaluable in developing a familiar and trusting relationship with the child. Turning off the lights in an intensive care environment is not feasible. Keeping alarm levels low could increase risk of injury if an alarm is not heard by staff. Consulting the play therapist is not appropriate at this time. Page Ref: 268 Cognitive Level: Applying Client Need &Sub: Physiological Integrity Standards: QSEN Competencies: Patient-centered care | AACN Essential Competencies: Essential IX: Baccalaureate generalist nursing practice | NLN Competencies: Nursing judgement | Nursing/Integrated Concepts: Nursing Process: Assessment Learning Outcome: LO 13.4 Develop a nursing care plan for the child with a life-threatening illness or injury. MNL LO: Apply the nursing process to care for child and family with life-threatening illness/at end of life. 11) Parents of a child in the pediatric intensive care unit (PICU) have been experiencing shock and disbelief regarding their situation. Which statement by the parents indicates they are moving forward into the next stage of coping? 1. "Why not me instead of my child?" 2. "It is hard for me to have others take care of my child." 3. "I feel like life is suspended in time." 4. "I am glad I can help with his care." Answer: 1 Explanation: 1. The parents initially enter the stage of shock and disbelief. Asking "Why not me instead of my child?" shows they are moving into the next stage, which is anger and disbelief. Having feelings about others caring for their child is the third stage of deprivation and loss. The feeling of being suspended in time is the fourth stage, which is anticipatory guidance. 2. The parents initially enter the stage of shock and disbelief. Asking "Why not me instead of my child?" shows they are moving into the next stage, which is anger and disbelief. Having feelings about others caring for their child is the third stage of deprivation and loss. The feeling of being suspended in time is the fourth stage, which is anticipatory guidance. 3. The parents initially enter the stage of shock and disbelief. Asking "Why not me instead of my child?" shows they are moving into the next stage, which is anger and disbelief. Having feelings about others caring for their child is the third stage of deprivation and loss. The feeling of being suspended in time is the fourth stage, which is anticipatory guidance. 4. The parents initially enter the stage of shock and disbelief. Asking "Why not me instead of my child?" shows they are moving into the next stage, which is anger and disbelief. Having feelings about others caring for their child is the third stage of deprivation and loss. The feeling of being suspended in time is the fourth stage, which is anticipatory guidance. Page Ref: 266 Cognitive Level: Analyzing Client Need &Sub: Psychosocial Integrity Standards: QSEN Competencies: Patient-centered care | AACN Essential Competencies: Essential IX: Baccalaureate generalist nursing practice | NLN Competencies: Human flourishing | Nursing/Integrated Concepts: Nursing Process: Diagnosis/Quality of practice Learning Outcome: LO 13.7 Plan bereavement support for the parents and siblings after the death of a child. MNL LO: Apply the concepts related to life-threatening illnesses and end-of-life care. 12) A child is on a ventilator in the pediatric intensive care unit (PICU). Which nursing intervention would best meet the psychosocial needs of this child? 1. Allow the parents to remain at the bedside. 2. Touch and talk to the child often. 3. Provide the child with a blanket from home. 4. Provide consistent caregivers. Answer: 2 Explanation: 1. Touch and verbal exchanges will aid in psychosocial support. The other responses provide a sense of security. 2. Touch and verbal exchanges will aid in psychosocial support. The other responses provide a sense of security. 3. Touch and verbal exchanges will aid in psychosocial support. The other responses provide a sense of security. 4. Touch and verbal exchanges will aid in psychosocial support. The other responses provide a sense of security. Page Ref: 267-269 Cognitive Level: Applying Client Need &Sub: Psychosocial Integrity Standards: QSEN Competencies: Patient-centered care | AACN Essential Competencies: Essential IX: Baccalaureate generalist nursing practice | NLN Competencies: Nursing judgement | Nursing/Integrated Concepts: Nursing Process: Implementation/Coordination of care Learning Outcome: LO 13.4 Develop a nursing care plan for the child with a life-threatening illness or injury. MNL LO: Apply the nursing process to care for child and family with life-threatening illness/at end of life. 13) Which client in the pediatric intensive care unit (PICU) would most benefit from palliative care? 1. A child with end-stage leukemia 2. A child with a broken arm after a motor vehicle accident 3. A child with burn injuries to the legs 4. A child with recurrent asthma Answer: 3 Explanation: 1. A child with burn injuries to the legs will benefit most from palliative care to help control pain, anxiety, sleep disturbances, and so on. The child with end-stage leukemia will benefit from hospice care. The child with a broken arm or recurrent asthma will not need palliative care. 2. A child with burn injuries to the legs will benefit most from palliative care to help control pain, anxiety, sleep disturbances, and so on. The child with end-stage leukemia will benefit from hospice care. The child with a broken arm or recurrent asthma will not need palliative care. 3. A child with burn injuries to the legs will benefit most from palliative care to help control pain, anxiety, sleep disturbances, and so on. The child with end-stage leukemia will benefit from hospice care. The child with a broken arm or recurrent asthma will not need palliative care. 4. A child with burn injuries to the legs will benefit most from palliative care to help control pain, anxiety, sleep disturbances, and so on. The child with end-stage leukemia will benefit from hospice care. The child with a broken arm or recurrent asthma will not need palliative care. Page Ref: 263-264 Cognitive Level: Applying Client Need &Sub: Psychosocial Integrity Standards: QSEN Competencies: Patient-centered care | AACN Essential Competencies: Essential IX: Baccalaureate generalist nursing practice | NLN Competencies: Nursing judgement | Nursing/Integrated Concepts: Nursing Process: Implementation/Coordination of care Learning Outcome: LO 13.4 Develop a nursing care plan for the child with a life-threatening illness or injury. MNL LO: Apply the nursing process to care for child and family with life-threatening illness/at end of life. 14) Siblings of a client in pediatric intensive care unit (PICU) are preparing to visit their brother, who was hit by a car while riding his bike. Which intervention by the nurse will assist the siblings in preparing for the visit? 1. Spend time developing a relationship with the siblings. 2. Have the parents go with the siblings when they visit. 3. Encourage the siblings to talk to a social worker before seeing their brother. 4. Explain what the siblings will hear and see when they visit. Answer: 4 Explanation: 1. Explaining what the siblings will hear and see when they visit will best prepare them for the visit with their brother. The other responses are good ways to help alleviate stress but won't help prepare the siblings for the visit. 2. Explaining what the siblings will hear and see when they visit will best prepare them for the visit with their brother. The other responses are good ways to help alleviate stress but won't help prepare the siblings for the visit. 3. Explaining what the siblings will hear and see when they visit will best prepare them for the visit with their brother. The other responses are good ways to help alleviate stress but won't help prepare the siblings for the visit. 4. Explaining what the siblings will hear and see when they visit will best prepare them for the visit with their brother. The other responses are good ways to help alleviate stress but won't help prepare the siblings for the visit. Page Ref: 269-270 Cognitive Level: Analyzing Client Need &Sub: Health Promotion and Maintenance Standards: QSEN Competencies: Patient-centered care | AACN Essential Competencies: Essential IX: Baccalaureate generalist nursing practice | NLN Competencies: Human flourishing | Nursing/Integrated Concepts: Nursing Process: Planning/Communication Learning Outcome: LO 13.2 Examine the family's experience and reactions to having a child with a life-threatening illness or injury. MNL LO: Analyze the reactions by the child, parents, and siblings related to life-threatening illnesses. 15) A toddler-age client is in end-stage renal failure. Which nursing intervention will assist this child most? 1. Maintain the child's normal routines. 2. Explain body changes that will take place. 3. Encourage friends to visit. 4. Allow the child to talk about the illness. Answer: 1 Explanation: 1. A toddler has no real concept of death, but does sense changes in routine and parent behavior. Maintaining normal routines is the best intervention to assist this child. A toddler will not understand the body changes; this approach would be more appropriate for a school-age child. Encouraging friends to visit and allowing the child to talk about the illness are more appropriate for older children. 2. A toddler has no real concept of death, but does sense changes in routine and parent behavior. Maintaining normal routines is the best intervention to assist this child. A toddler will not understand the body changes; this approach would be more appropriate for a school-age child. Encouraging friends to visit and allowing the child to talk about the illness are more appropriate for older children. 3. A toddler has no real concept of death, but does sense changes in routine and parent behavior. Maintaining normal routines is the best intervention to assist this child. A toddler will not understand the body changes; this approach would be more appropriate for a school-age child. Encouraging friends to visit and allowing the child to talk about the illness are more appropriate for older children. 4. A toddler has no real concept of death, but does sense changes in routine and parent behavior. Maintaining normal routines is the best intervention to assist this child. A toddler will not understand the body changes; this approach would be more appropriate for a school-age child. Encouraging friends to visit and allowing the child to talk about the illness are more appropriate for older children. Page Ref: 274-275 Cognitive Level: Analyzing Client Need &Sub: Psychosocial Integrity Standards: QSEN Competencies: Patient-centered care | AACN Essential Competencies: Essential IX: Baccalaureate generalist nursing practice | NLN Competencies: Nursing judgement | Nursing/Integrated Concepts: Nursing Process: Implementation/Coordination of care Learning Outcome: LO 13.6 Develop a nursing care plan to provide family-centered care for the dying child and family. MNL LO: Apply the nursing process to care for child and family with life-threatening illness/at end of life. 16) Which nursing interventions would be best for the nursing diagnosis of Powerlessness Related to Relinquishing Control to the Healthcare Team? Select all that apply. 1. Provide a primary nursing care model. 2. Prepare the child in advance for procedures. 3. Provide optimal pain relief. 4. Explain procedures in developmentally appropriate terms. 5. Incorporate home rituals when possible. Answer: 2, 4, 5 Explanation: 1. Preparation in advance—and in terms that are developmentally appropriate— and incorporating home rituals provide some degree of control, and might reduce the feeling of powerlessness. Providing a primary nursing care model will help decrease anxiety, and providing pain relief will decrease pain. 2. Preparation in advance—and in terms that are developmentally appropriate—and incorporating home rituals provide some degree of control, and might reduce the feeling of powerlessness. Providing a primary nursing care model will help decrease anxiety, and providing pain relief will decrease pain. 3. Preparation in advance—and in terms that are developmentally appropriate—and incorporating home rituals provide some degree of control, and might reduce the feeling of powerlessness. Providing a primary nursing care model will help decrease anxiety, and providing pain relief will decrease pain. 4. Preparation in advance—and in terms that are developmentally appropriate—and incorporating home rituals provide some degree of control, and might reduce the feeling of powerlessness. Providing a primary nursing care model will help decrease anxiety, and providing pain relief will decrease pain. 5. Preparation in advance—and in terms that are developmentally appropriate—and incorporating home rituals provide some degree of control, and might reduce the feeling of powerlessness. Providing a primary nursing care model will help decrease anxiety, and providing pain relief will decrease pain. Page Ref: 273-274 Cognitive Level: Applying Client Need &Sub: Physiological Integrity Standards: QSEN Competencies: Patient-centered care | AACN Essential Competencies: Essential IX: Baccalaureate generalist nursing practice | NLN Competencies: Nursing judgement | Nursing/Integrated Concepts: Nursing Process: Implementation/Coordination of care Learning Outcome: LO 13.4 Develop a nursing care plan for the child with a life-threatening illness or injury. MNL LO: Apply the nursing process to care for child and family with life-threatening illness/at end of life. 17) Which intervention is considered supportive care for a family whose infant has died from sudden infant death syndrome (SIDS)? 1. Interviewing parents to determine the cause of the SIDS incident 2. Allowing parents to hold, touch, and rock the infant 3. Sheltering parents from the grief by not giving them any personal items of the infant, such as footprints 4. Advising parents that an autopsy is not necessary Answer: 2 Explanation: 1. The parents should be allowed to hold, touch, and rock the infant, giving them a chance to say good-bye to their baby. The other options are nontherapeutic. The death of an infant without a known medical condition is an indication for an autopsy. 2. The parents should be allowed to hold, touch, and rock the infant, giving them a chance to say good-bye to their baby. The other options are nontherapeutic. The death of an infant without a known medical condition is an indication for an autopsy. 3. The parents should be allowed to hold, touch, and rock the infant, giving them a chance to say good-bye to their baby. The other options are nontherapeutic. The death of an infant without a known medical condition is an indication for an autopsy. 4. The parents should be allowed to hold, touch, and rock the infant, giving them a chance to say good-bye to their baby. The other options are nontherapeutic. The death of an infant without a known medical condition is an indication for an autopsy. Page Ref: 277 Cognitive Level: Applying Client Need &Sub: Psychosocial Integrity Standards: QSEN Competencies: Patient-centered care | AACN Essential Competencies: Essential IX: Baccalaureate generalist nursing practice | NLN Competencies: Human flourishing | Nursing/Integrated Concepts: Nursing Process: Implementation/Coordination of care Learning Outcome: LO 13.7 Plan bereavement support for the parents and siblings after the death of a child. MNL LO: Evaluate the developmental aspects and family reactions related to end-of-life care. 18) The nurse is providing care to an adolescent client who is dying. Which assessment findings indicate the client is experiencing a decrease in peripheral circulation? Select all that apply. 1. Cool skin 2. Mottled appearance 3. Cheyne-Stokes respirations 4. Increased agitation 5. Increased urine output Answer: 1, 2 Explanation: 1. A client who is experiencing decreased peripheral circulation will have cool, mottled skin. While Cheyne-Stokes respirations may indicate death is approaching, this is not indicative of a decrease in peripheral circulation. Increased agitation indicates decreased perfusion to the brain. A client will not experience increased urine output near the end life. 2. A client who is experiencing decreased peripheral circulation will have cool, mottled skin. While Cheyne-Stokes respirations may indicate death is approaching, this is not indicative of a decrease in peripheral circulation. Increased agitation indicates decreased perfusion to the brain. A client will not experience increased urine output near the end life. 3. A client who is experiencing decreased peripheral circulation will have cool, mottled skin. While Cheyne-Stokes respirations may indicate death is approaching, this is not indicative of a decrease in peripheral circulation. Increased agitation indicates decreased perfusion to the brain. A client will not experience increased urine output near the end life. 4. A client who is experiencing decreased peripheral circulation will have cool, mottled skin. While Cheyne-Stokes respirations may indicate death is approaching, this is not indicative of a decrease in peripheral circulation. Increased agitation indicates decreased perfusion to the brain. A client will not experience increased urine output near the end life. 5. A client who is experiencing decreased peripheral circulation will have cool, mottled skin. While Cheyne-Stokes respirations may indicate death is approaching, this is not indicative of a decrease in peripheral circulation. Increased agitation indicates decreased perfusion to the brain. A client will not experience increased urine output near the end life. Page Ref: 273 Cognitive Level: Applying Client Need &Sub: Physiological Integrity Standards: QSEN Competencies: Patient-centered care | AACN Essential Competencies: Essential IX: Baccalaureate generalist nursing practice | NLN Competencies: Nursing judgement | Nursing/Integrated Concepts: Nursing Process: Assessment/Coordination of care Learning Outcome: LO 13.5 Apply assessment skills to identify the physiologic changes that occur in the dying child. MNL LO: Apply the concepts related to life-threatening illnesses and end-of-life care. 19) A novice nurse in the newborn intensive care unit (NICU) has just performed postmortem care on a premature infant who passed away. The novice nurse asks to be excused near the end of the shift. Which interventions can be implemented to support this nurse? Select all that apply. 1. Schedule additional education on bereavement care 2. Ask a seasoned nurse to talk with the novice nurse 3. Tell the nurse it is OK to grieve with the family 4. Recommend that the nurse transfer to another unit 5. Assign the nurse to stable clients only Answer: 1, 2, 3 Explanation: 1. Appropriate interventions for this nurse include scheduling additional education on bereavement care, asking a seasoned nurse to talk about the situation with the novice nurse, and telling the nurse it is OK to grieve with the family. Recommending a transfer and assigning the nurse to only stable clients are not appropriate interventions to support the novice nurse. 2. Appropriate interventions for this nurse include scheduling additional education on bereavement care, asking a seasoned nurse to talk about the situation with the novice nurse, and telling the nurse it is OK to grieve with the family. Recommending a transfer and assigning the nurse to only stable clients are not appropriate interventions to support the novice nurse. 3. Appropriate interventions for this nurse include scheduling additional education on bereavement care, asking a seasoned nurse to talk about the situation with the novice nurse, and telling the nurse it is OK to grieve with the family. Recommending a transfer and assigning the nurse to only stable clients are not appropriate interventions to support the novice nurse. 4. Appropriate interventions for this nurse include scheduling additional education on bereavement care, asking a seasoned nurse to talk about the situation with the novice nurse, and telling the nurse it is OK to grieve with the family. Recommending a transfer and assigning the nurse to only stable clients are not appropriate interventions to support the novice nurse. 5. Appropriate interventions for this nurse include scheduling additional education on bereavement care, asking a seasoned nurse to talk about the situation with the novice nurse, and telling the nurse it is OK to grieve with the family. Recommending a transfer and assigning the nurse to only stable clients are not appropriate interventions to support the novice nurse. Page Ref: 279 Cognitive Level: Applying Client Need &Sub: Psychosocial Integrity Standards: QSEN Competencies: Quality improvement | AACN Essential Competencies: Essential II: Basic organizational and systems leadership for quality care and patient safety | NLN Competencies: Nursing judgement | Nursing/Integrated Concepts: Nursing Process: Implementation/Quality of practice Learning Outcome: LO 13.8 Evaluate strategies to support nurses who care for children who die. MNL LO: Apply the concepts related to life-threatening illnesses and end-of-life care. 20) A school-aged child is admitted with pneumococcal meningitis. The child weighs 44 pounds. The physician orders: ceftriaxone (Rocephin) 50 mg/kg/dose IV every 12 hours three times and then every 24 hours. Calculate how many mg/dose of ceftriaxone the child will receive and then calculate mL/hr to infuse via pump. Supply on hand is: a premix of ceftriaxone 1 g/50 mL, administer over 30 minutes. Answer: 1000 mg/dose; 100 mL/hr Explanation: The child will receive 1000 mg/dose of ceftriaxone, then 100 mL/hr to infuse via pump. Page Ref: 262-263 Cognitive Level: Analyzing Client Need &Sub: Health Promotion and Maintenance Standards: QSEN Competencies: Patient-centered care | AACN Essential Competencies: Essential IX: Baccalaureate generalist nursing practice | NLN Competencies: Human flourishing | Nursing/Integrated Concepts: Nursing Process: Assessment/Coordination of care Learning Outcome: LO 13.4 Develop a nursing care plan for the child with a life-threatening illness or injury. MNL LO: Demonstrate safe medication administration for the pediatric client. 21) Match the child's concept of death with their behavioral response. A. Infant B. Toddler C. Preschool-age child D. School-age child E. Adolescent 1. Understands difference between temporary separation and death. 2. Senses emotions of caregivers and altered routines. 3. Capable of understanding death, recognizes all people and self will die. 4. No understanding of true concept of death. 5. Believes death is temporary and the person will return. Answer: 1/D, 2/A, 3/E, 4/B, 5/C 1. School-age child 2. Infant 3. Adolescent 4. Toddler 5. Preschool-age child Explanation: School-age child—Understands difference between temporary separation and death. Infant—Senses emotions of caregivers, and altered routines. Adolescent—Capable of understanding death, recognizes all people and self will die. Toddler—No understanding of true concept of death. Preschool-age child— Believes death is temporary and the person will return. Page Ref: 278 Cognitive Level: Analyzing Client Need &Sub: Health Promotion and Maintenance Standards: QSEN Competencies: Patient-centered care | AACN Essential Competencies: Essential IX: Baccalaureate generalist nursing practice | NLN Competencies: Human flourishing | Nursing/Integrated Concepts: Nursing Process: Evaluation/Coordination of care Learning Outcome: LO 13.3 Identify the coping mechanisms used by the child and family in response to stress. MNL LO: Differentiate the developmental aspects of the hospitalized child. Principles of Pediatric Nursing: Caring for Children, 7e (Ball et al.) Chapter 14 Infant, Child, and Adolescent Nutrition 1) A school-age client, recently diagnosed with asthma, also has a peanut allergy. The nurse instructs the family to not only avoid peanuts but also to carefully check food label ingredients for peanut products and to make sure dishes and utensils are adequately washed prior to food preparation. The mother asks why this is specific for her child. Based on the client's history, the nurse knows that this client is at an increased risk for which complication? 1. Urticaria 2. Diarrhea 3. Anaphylaxis 4. Headache Answer: 3 Explanation: 1. Children with food allergies may experience all of the above reactions to a particular food, but the child who also has asthma is most at risk for death secondary to anaphylaxis caused by a food allergy. 2. Children with food allergies may experience all of the above reactions to a particular food, but the child who also has asthma is most at risk for death secondary to anaphylaxis caused by a food allergy. 3. Children with food allergies may experience all of the above reactions to a particular food, but the child who also has asthma is most at risk for death secondary to anaphylaxis caused by a food allergy. 4. Children with food allergies may experience all of the above reactions to a particular food, but the child who also has asthma is most at risk for death secondary to anaphylaxis caused by a food allergy. Page Ref: 305 Cognitive Level: Applying Client Need &Sub: Physiological Integrity Standards: QSEN Competencies: Patient-centered care | AACN Essential Competencies: Essential VII: Clinical prevention and population health | NLN Competencies: Human flourishing | Nursing/Integrated Concepts: Nursing Process: Implementation/Health teaching and health promotion Learning Outcome: LO 14.5 Develop nursing interventions for children with nutritional disorders. MNL LO: Examine the components of performing a nutritional assessment of the pediatric client. 2) While teaching the parents of a newborn about infant care and feeding, which instruction by the nurse is the most appropriate? 1. Delay supplemental foods until the infant is 4 to 6 months old. 2. Delay supplemental foods until the infant reaches 15 pounds or greater. 3. Begin diluted fruit juice at 2 months of age, but wait 3 to 5 days before trying a new food. 4. Add rice cereal to the nighttime feeding if the infant is having difficulty sleeping after 2 months of age. Answer: 1 Explanation: 1. Four to six months is the optimal age to begin supplemental feedings because earlier feeding of nonformula foods is not needed by the infant and does not promote sleep. Earlier feeding of nonformula foods, regardless of the infant's weight, is more likely to cause the development of food allergies. Also, early feeding is not well tolerated by infants because the necessary tongue control is not well developed and they lack the digestive enzymes to take in and metabolize many food products. 2. Four to six months is the optimal age to begin supplemental feedings because earlier feeding of nonformula foods is not needed by the infant and does not promote sleep. Earlier feeding of nonformula foods, regardless of the infant's weight, is more likely to cause the development of food allergies. Also, early feeding is not well tolerated by infants because the necessary tongue control is not well developed and they lack the digestive enzymes to take in and metabolize many food products. 3. Four to six months is the optimal age to begin supplemental feedings because earlier feeding of nonformula foods is not needed by the infant and does not promote sleep. Earlier feeding of nonformula foods, regardless of the infant's weight, is more likely to cause the development of food allergies. Also, early feeding is not well tolerated by infants because the necessary tongue control is not well developed and they lack the digestive enzymes to take in and metabolize many food products. 4. Four to six months is the optimal age to begin supplemental feedings because earlier feeding of nonformula foods is not needed by the infant and does not promote sleep. Earlier feeding of nonformula foods, regardless of the infant's weight, is more likely to cause the development of food allergies. Also, early feeding is not well tolerated by infants because the necessary tongue control is not well developed and they lack the digestive enzymes to take in and metabolize many food products. Page Ref: 285-286 Cognitive Level: Applying Client Need &Sub: Health Promotion and Maintenance Standards: QSEN Competencies: Evidence-based practice | AACN Essential Competencies: Essential VII: Clinical prevention and population health | NLN Competencies: Nursing judgement | Nursing/Integrated Concepts: Nursing Process: Implementation/Health teaching and health promotion Learning Outcome: LO 14.1 Discuss major nutritional concepts pertaining to the growth and development of children. MNL LO: Relate nutrition to the prevention of illness and the maintenance of child health and wellness. 3) During a 4-month-old's well-child check, the nurse discusses introduction of solid foods into the infant's diet and concerns for foods commonly associated with food allergies. Due to allergies, which foods will the nurse instruction the parents to avoid until after 1 year of age? 1. Strawberries, eggs, and wheat 2. Peas, tomatoes, and spinach 3. Carrots, beets, and spinach 4. Squash, pork, and tomatoes Answer: 1 Explanation: 1. Strawberries, eggs, and wheat, along with corn, fish, and nut products, are all foods that have commonly been associated with food allergies. Carrots, beets, and spinach contain nitrates and should not be given before the age of 4 months. Squash, peas, and tomatoes are acceptable to try after an infant is 4 to 6 months old but should be given one at a time and 3 to 5 days after starting a new food. Pork can be tried after the infant is 8 to 10 months old, as meats are harder to digest and have a high protein load. 2. Strawberries, eggs, and wheat, along with corn, fish, and nut products, are all foods that have commonly been associated with food allergies. Carrots, beets, and spinach contain nitrates and should not be given before the age of 4 months. Squash, peas, and tomatoes are acceptable to try after an infant is 4 to 6 months old but should be given one at a time and 3 to 5 days after starting a new food. Pork can be tried after the infant is 8 to 10 months old, as meats are harder to digest and have a high protein load. 3. Strawberries, eggs, and wheat, along with corn, fish, and nut products, are all foods that have commonly been associated with food allergies. Carrots, beets, and spinach contain nitrates and should not be given before the age of 4 months. Squash, peas, and tomatoes are acceptable to try after an infant is 4 to 6 months old but should be given one at a time and 3 to 5 days after starting a new food. Pork can be tried after the infant is 8 to 10 months old, as meats are harder to digest and have a high protein load. 4. Strawberries, eggs, and wheat, along with corn, fish, and nut products, are all foods that have commonly been associated with food allergies. Carrots, beets, and spinach contain nitrates and should not be given before the age of 4 months. Squash, peas, and tomatoes are acceptable to try after an infant is 4 to 6 months old but should be given one at a time and 3 to 5 days after starting a new food. Pork can be tried after the infant is 8 to 10 months old, as meats are harder to digest and have a high protein load. Page Ref: 285-287 Cognitive Level: Applying Client Need &Sub: Health Promotion and Maintenance Standards: QSEN Competencies: Evidence-based practice | AACN Essential Competencies: Essential VII: Clinical prevention and population health | NLN Competencies: Human flourishing | Nursing/Integrated Concepts: Nursing Process: Implementation/Health teaching and health promotion Learning Outcome: LO 14.1 Discuss major nutritional concepts pertaining to the growth and development of children. MNL LO: Relate nutrition to the prevention of illness and the maintenance of child health and wellness. 4) The mother of a toddler is concerned because her child does not seem interested in eating. The child is drinking 5 to 6 cups of whole milk per day and one cup of fruit juice. When the weight- to-height percentile is calculated, the child is in the 90th to 95th percentile. What is the best advice the nurse can provide to the mother? 1. Eliminate the fruit juice from the child's diet. 2. Offer healthy snacks, presented in a creative manner, and let the child choose what he wants to eat without pressure from the parents. 3. Change from whole milk to 2 percent milk and decrease milk consumption to three to four cups per day and the fruit juice to a half cup per day, offering water if the child is still thirsty in between. 4. Make sure that the child is getting adequate opportunities for exercise, as this will increase his appetite and help lower the child's weight-to-height percentile. Answer: 3 Explanation: 1. Toddlers require a maximum of about 1 L of milk per day. This toddler is consuming most of his or her calories from the milk and thus is not hungry. The high fat content of the milk and the high sugar content of the fruit juice are also contributing to the child's higher weight-to-height percentile. Decreasing the amount and fat content of the milk and decreasing the intake of fruit juice will decrease calories and thus make the child hungry for other foods. The other advice is also appropriate but did not address the problem of excessive milk consumption. 2. Toddlers require a maximum of about 1 L of milk per day. This toddler is consuming most of his or her calories from the milk and thus is not hungry. The high fat content of the milk and the high sugar content of the fruit juice are also contributing to the child's higher weight-to-height percentile. Decreasing the amount and fat content of the milk and decreasing the intake of fruit juice will decrease calories and thus make the child hungry for other foods. The other advice is also appropriate but did not address the problem of excessive milk consumption. 3. Toddlers require a maximum of about 1 L of milk per day. This toddler is consuming most of his or her calories from the milk and thus is not hungry. The high fat content of the milk and the high sugar content of the fruit juice are also contributing to the child's higher weight-to-height percentile. Decreasing the amount and fat content of the milk and decreasing the intake of fruit juice will decrease calories and thus make the child hungry for other foods. The other advice is also appropriate but did not address the problem of excessive milk consumption. 4. Toddlers require a maximum of about 1 L of milk per day. This toddler is consuming most of his or her calories from the milk and thus is not hungry. The high fat content of the milk and the high sugar content of the fruit juice are also contributing to the child's higher weight-to-height percentile. Decreasing the amount and fat content of the milk and decreasing the intake of fruit juice will decrease calories and thus make the child hungry for other foods. The other advice is also appropriate but did not address the problem of excessive milk consumption. Page Ref: 287 Cognitive Level: Analyzing Client Need &Sub: Health Promotion and Maintenance Standards: QSEN Competencies: Patient-centered care | AACN Essential Competencies: Essential VII: Clinical prevention and population health | NLN Competencies: Nursing judgement | Nursing/Integrated Concepts: Nursing Process: Implementation/Health teaching and health promotion Learning Outcome: LO 14.2 Describe and plan nursing interventions to meet nutritional needs for all age groups from infancy through adolescence. MNL LO: Relate nutrition to the prevention of illness and the maintenance of child health and wellness. 5) A nurse is talking to the mother of an exclusively breastfed African American 3-month-old infant who was born in late fall. Which supplement will the nurse recommend for this infant? 1. Iron 2. Vitamin D 3. Fluoride 4. Calcium Answer: 2 Explanation: 1. An infant's iron stores are usually adequate until about 4 to 6 months of age. The infant should be receiving sufficient amounts of calcium from breast milk, and fluoride supplementation, if needed, does not begin until the child is approximately 6 months old. This infant will have limited exposure to sunlight and thus vitamin D because of the infant's dark skin and decreased sun exposure in the fall and winter months. 2. An infant's iron stores are usually adequate until about 4 to 6 months of age. The infant should be receiving sufficient amounts of calcium from breast milk, and fluoride supplementation, if needed, does not begin until the child is approximately 6 months old. This infant will have limited exposure to sunlight and thus vitamin D because of the infant's dark skin and decreased sun exposure in the fall and winter months. 3. An infant's iron stores are usually adequate until about 4 to 6 months of age. The infant should be receiving sufficient amounts of calcium from breast milk, and fluoride supplementation, if needed, does not begin until the child is approximately 6 months old. This infant will have limited exposure to sunlight and thus vitamin D because of the infant's dark skin and decreased sun exposure in the fall and winter months. 4. An infant's iron stores are usually adequate until about 4 to 6 months of age. The infant should be receiving sufficient amounts of calcium from breast milk, and fluoride supplementation, if needed, does not begin until the child is approximately 6 months old. This infant will have limited exposure to sunlight and thus vitamin D because of the infant's dark skin and decreased sun exposure in the fall and winter months. Page Ref: 284 Cognitive Level: Analyzing Client Need &Sub: Health Promotion and Maintenance Standards: QSEN Competencies: Patient-centered care | AACN Essential Competencies: Essential VII: Clinical prevention and population health | NLN Competencies: Professional identity | Nursing/Integrated Concepts: Nursing Process: Planning/Health teaching and health promotion Learning Outcome: LO 14.2 Describe and plan nursing interventions to meet nutritional needs for all age groups from infancy through adolescence. MNL LO: Relate nutrition to the prevention of illness and the maintenance of child health and wellness. 6) The nurse is teaching the parents of a 4-month-old infant about good feeding habits. The nurse emphasizes the importance of holding the baby during feeding and not letting the infant go to sleep with the bottle. Which disorder is associated with propped feedings and going to sleep with the bottle? 1. Otitis media 2. Aspiration 3. Malocclusion problems 4. Sleeping disorders Answer: 1 Explanation: 1. It has been shown in numerous studies that allowing an infant to fall asleep with a bottle in his or her mouth causes pooling of the formula in the mouth, which increases the risk of both dental caries and otitis media. There has been limited data to date showing a positive correlation between bottle propping and increased risk of aspiration, malocclusions, and sleeping disorders. 2. It has been shown in numerous studies that allowing an infant to fall asleep with a bottle in his or her mouth causes pooling of the formula in the mouth, which increases the risk of both dental caries and otitis media. There has been limited data to date showing a positive correlation between bottle propping and increased risk of aspiration, malocclusions, and sleeping disorders. 3. It has been shown in numerous studies that allowing an infant to fall asleep with a bottle in his or her mouth causes pooling of the formula in the mouth, which increases the risk of both dental caries and otitis media. There has been limited data to date showing a positive correlation between bottle propping and increased risk of aspiration, malocclusions, and sleeping disorders. 4. It has been shown in numerous studies that allowing an infant to fall asleep with a bottle in his or her mouth causes pooling of the formula in the mouth, which increases the risk of both dental caries and otitis media. There has been limited data to date showing a positive correlation between bottle propping and increased risk of aspiration, malocclusions, and sleeping disorders. Page Ref: 284 Cognitive Level: Applying Client Need &Sub: Health Promotion and Maintenance Standards: QSEN Competencies: Patient-centered care | AACN Essential Competencies: Essential VII: Clinical prevention and population health | NLN Competencies: Nursing judgement | Nursing/Integrated Concepts: Nursing Process: Implementation/Health teaching and health promotion Learning Outcome: LO 14.4 Identify and explain common nutritional problems of children. MNL LO: Analyze socioeconomic factors that impact childhood nutrition. 7) The parents of a toddler are concerned about their child's finicky eating habits. While counseling the parents, which statements by the nurse are the most appropriate? Select all that apply. 1. "The child is experiencing physiologic anorexia, which is normal for this age group." 2. "A general guideline for food quantity at a meal is one-quarter cup of each food per year of age." 3. "It is more appropriate to assess a toddler's nutritional demands over a 1-week period rather than a 24-hour one." 4. "Nutritious foods should be made available at all times of the day so that she is able to 'graze' whenever she is hungry." 5. "The toddler should drink 16 to 24 ounces of milk daily." Answer: 1, 3, 5 Explanation: 1. Physiologic anorexia is caused when the extremely high metabolic demands of infancy slow to keep pace with the slower growth of toddlerhood, and it is a very normal finding at this age. It is not unusual for toddlers to have food jags during which they only want one or two food items for that day. So it is more helpful to look at what their intake has been over a week instead of a day. Two to three cups of milk per day are sufficient for a toddler, and more than that can decrease the child's desire for other foods and lead to dietary deficiencies. The correct general guideline for food quantity is one tablespoon of each food per year of age. Food should only be offered at meal and snack times, and children should sit at the table while eating to encourage their socialization skills. 2. Physiologic anorexia is caused when the extremely high metabolic demands of infancy slow to keep pace with the slower growth of toddlerhood, and it is a very normal finding at this age. It is not unusual for toddlers to have food jags during which they only want one or two food items for that day. So it is more helpful to look at what their intake has been over a week instead of a day. Two to three cups of milk per day are sufficient for a toddler, and more than that can decrease the child's desire for other foods and lead to dietary deficiencies. The correct general guideline for food quantity is one tablespoon of each food per year of age. Food should only be offered at meal and snack times, and children should sit at the table while eating to encourage their socialization skills. 3. Physiologic anorexia is caused when the extremely high metabolic demands of infancy slow to keep pace with the slower growth of toddlerhood, and it is a very normal finding at this age. It is not unusual for toddlers to have food jags during which they only want one or two food items for that day. So it is more helpful to look at what their intake has been over a week instead of a day. Two to three cups of milk per day are sufficient for a toddler, and more than that can decrease the child's desire for other foods and lead to dietary deficiencies. The correct general guideline for food quantity is one tablespoon of each food per year of age. Food should only be offered at meal and snack times, and children should sit at the table while eating to encourage their socialization skills. 4. Physiologic anorexia is caused when the extremely high metabolic demands of infancy slow to keep pace with the slower growth of toddlerhood, and it is a very normal finding at this age. It is not unusual for toddlers to have food jags during which they only want one or two food items for that day. So it is more helpful to look at what their intake has been over a week instead of a day. Two to three cups of milk per day are sufficient for a toddler, and more than that can decrease the child's desire for other foods and lead to dietary deficiencies. The correct general guideline for food quantity is one tablespoon of each food per year of age. Food should only be offered at meal and snack times, and children should sit at the table while eating to encourage their socialization skills. 5. Physiologic anorexia is caused when the extremely high metabolic demands of infancy slow to keep pace with the slower growth of toddlerhood, and it is a very normal finding at this age. It is not unusual for toddlers to have food jags during which they only want one or two food items for that day. So it is more helpful to look at what their intake has been over a week instead of a day. Two to three cups of milk per day are sufficient for a toddler, and more than that can decrease the child's desire for other foods and lead to dietary deficiencies. The correct general guideline for food quantity is one tablespoon of each food per year of age. Food should only be offered at meal and snack times, and children should sit at the table while eating to encourage their socialization skills. Page Ref: 287 Cognitive Level: Applying Client Need &Sub: Health Promotion and Maintenance Standards: QSEN Competencies: Patient-centered care | AACN Essential Competencies: Essential II: Basic organizational and systems leadership for quality care and patient safety | NLN Competencies: Nursing judgement | Nursing/Integrated Concepts: Nursing Process: Implementation/Health teaching and health promotion Learning Outcome: LO 14.2 Describe and plan nursing interventions to meet nutritional needs for all age groups from infancy through adolescence. MNL LO: Compare the nutritional needs of the infant, toddler, preschool-age child, school-age child, and adolescent. 8) The nurse is providing nutritional guidance to the parents of a toddler. Which comment by the parent would prompt the nurse to provide additional education? 1. "I should not give my child raw oysters." 2. "It is safe to leave my meat red in the center as long as there are no juices running." 3. "We always wash our hands well before any food preparation." 4. "We use separate utensils for preparing raw meat and preparing fruits, vegetables, and other foods." Answer: 2 Explanation: 1. Meats should be cooked thoroughly before eating. Meat that is red in the center, with or without running juices, is insufficiently cooked and increases the risk of food-borne illness. Washing hands and using separate utensils help to prevent infection with food-borne pathogens. Raw oysters should be avoided. 2. Meats should be cooked thoroughly before eating. Meat that is red in the center, with or without running juices, is insufficiently cooked and increases the risk of food-borne illness. Washing hands and using separate utensils help to prevent infection with food-borne pathogens. Raw oysters should be avoided. 3. Meats should be cooked thoroughly before eating. Meat that is red in the center, with or without running juices, is insufficiently cooked and increases the risk of food-borne illness. Washing hands and using separate utensils help to prevent infection with food-borne pathogens. Raw oysters should be avoided. 4. Meats should be cooked thoroughly before eating. Meat that is red in the center, with or without running juices, is insufficiently cooked and increases the risk of food-borne illness. Washing hands and using separate utensils help to prevent infection with food-borne pathogens. Raw oysters should be avoided. Page Ref: 296-297 Cognitive Level: Analyzing Client Need &Sub: Health Promotion and Maintenance Standards: QSEN Competencies: Evidence-based practice | AACN Essential Competencies: Essential II: Basic organizational and systems leadership for quality care and patient safety | NLN Competencies: Nursing judgement | Nursing/Integrated Concepts: Nursing Process: Evaluation/Health teaching and health promotion Learning Outcome: LO 14.2 Describe and plan nursing interventions to meet nutritional needs for all age groups from infancy through adolescence. MNL LO: Compare the nutritional needs of the infant, toddler, preschool-age child, school-age child, and adolescent. 9) During a well-child physical, an adolescent female has a normal history and physical except for an excessive amount of tooth enamel erosion, a greater-than-normal number of filled cavities, and calluses on the back of her hand. Her body mass index is in the 50th to 75th percentile for her age. Which disorder is the nurse concerned about based on the assessment findings? 1. Anorexia nervosa 2. Kwashiorkor 3. Bulimia nervosa 4. Marasmus. Answer: 3 Explanation: 1. The erosion of tooth enamel, dental caries, and calluses on the back of her hand all most likely are due to frequent vomiting of gastric acids, which is common with bulimia nervosa as part of a binge-purge cycle. Anorexia nervosa is an eating disorder where adolescents literally starve themselves to prevent weight gain; they also exercise excessively and use laxatives and diuretics to lose weight. Anorexia usually manifests as extreme weight loss and an obsession with food. Kwashiorkor is a protein deficiency, usually from malnutrition, that manifests as generalized edema. Marasmus is a lack of energy-producing calories that can be seen in anorexia, and this causes emaciation, decreased energy levels, and retarded development. 2. The erosion of tooth enamel, dental caries, and calluses on the back of her hand all most likely are due to frequent vomiting of gastric acids, which is common with bulimia nervosa as part of a binge-purge cycle. Anorexia nervosa is an eating disorder where adolescents literally starve themselves to prevent weight gain; they also exercise excessively and use laxatives and diuretics to lose weight. Anorexia usually manifests as extreme weight loss and an obsession with food. Kwashiorkor is a protein deficiency, usually from malnutrition, that manifests as generalized edema. Marasmus is a lack of energy-producing calories that can be seen in anorexia, and this causes emaciation, decreased energy levels, and retarded development. 3. The erosion of tooth enamel, dental caries, and calluses on the back of her hand all most likely are due to frequent vomiting of gastric acids, which is common with bulimia nervosa as part of a binge-purge cycle. Anorexia nervosa is an eating disorder where adolescents literally starve themselves to prevent weight gain; they also exercise excessively and use laxatives and diuretics to lose weight. Anorexia usually manifests as extreme weight loss and an obsession with food. Kwashiorkor is a protein deficiency, usually from malnutrition, that manifests as generalized edema. Marasmus is a lack of energy-producing calories that can be seen in anorexia, and this causes emaciation, decreased energy levels, and retarded development. 4. The erosion of tooth enamel, dental caries, and calluses on the back of her hand all most likely are due to frequent vomiting of gastric acids, which is common with bulimia nervosa as part of a binge-purge cycle. Anorexia nervosa is an eating disorder where adolescents literally starve themselves to prevent weight gain; they also exercise excessively and use laxatives and diuretics to lose weight. Anorexia usually manifests as extreme weight loss and an obsession with food. Kwashiorkor is a protein deficiency, usually from malnutrition, that manifests as generalized edema. Marasmus is a lack of energy-producing calories that can be seen in anorexia, and this causes emaciation, decreased energy levels, and retarded development. Page Ref: 303-304 Cognitive Level: Applying Client Need &Sub: Physiological Integrity Standards: QSEN Competencies: Patient-centered care | AACN Essential Competencies: Essential IX: Baccalaureate generalist nursing practice | NLN Competencies: Nursing judgement | Nursing/Integrated Concepts: Nursing Process: Assessment/Coordination of care Learning Outcome: LO 14.4 Identify and explain common nutritional problems of children. MNL LO: Analyze socioeconomic factors that impact childhood nutrition. 10) The nurse is instructing a parent of a newborn on the foods that are to be started based on age. The nurse instructs the parent that the first food given to a newborn is rice cereal. What statement by the parent suggests appropriate understanding of the next food that can be introduced? 1. "Chicken can be given next." 2. "Eggs can be given next." 3. "Fruits should be given next." 4. "Whole milk should be started." Answer: 3 Explanation: 1. Chicken is not given until 8 to 10 months, eggs are not given until 12 months, whole milk is given at 12 months. Fruits are given after rice cereal. 2. Chicken is not given until 8 to 10 months, eggs are not given until 12 months, whole milk is given at 12 months. Fruits are given after rice cereal. 3. Chicken is not given until 8 to 10 months, eggs are not given until 12 months, whole milk is given at 12 months. Fruits are given after rice cereal. 4. Chicken is not given until 8 to 10 months, eggs are not given until 12 months, whole milk is given at 12 months. Fruits are given after rice cereal. Page Ref: 286 Cognitive Level: Applying Client Need &Sub: Physiological Integrity Standards: QSEN Competencies: Evidence-based practice | AACN Essential Competencies: Essential VII: Clinical prevention and population health | NLN Competencies: Human flourishing | Nursing/Integrated Concepts: Nursing Process: Evaluation/Health teaching and health promotion Learning Outcome: LO 14.2 Describe and plan nursing interventions to meet nutritional needs for all age groups from infancy through adolescence. MNL LO: Compare the nutritional needs of the infant, toddler, preschool-age child, school-age child, and adolescent. 11) The mother of an infant born prematurely at 32 weeks expresses the desire to breastfeed her child. The nurse correctly responds with which statement when the mother asks how long she should breastfeed her baby? 1. "Until the child begins solid foods." 2. "Many breastfeed for 2 years." 3. "It is recommended that mothers of preterm infants breastfeed at least a month." 4. "Breast milk should be the only food for the first 6 months." Answer: 4 Explanation: 1. Breast milk should be the only food for the first 6 months, and should continue until 12 months even after solid foods are introduced. 2. Breast milk should be the only food for the first 6 months, and should continue until 12 months even after solid foods are introduced. 3. Breast milk should be the only food for the first 6 months, and should continue until 12 months even after solid foods are introduced. 4. Breast milk should be the only food for the first 6 months, and should continue until 12 months even after solid foods are introduced. Page Ref: 283-284 Cognitive Level: Applying Client Need &Sub: Health Promotion and Maintenance Standards: QSEN Competencies: Evidence-based practice | AACN Essential Competencies: Essential VII: Clinical prevention and population health | NLN Competencies: Human flourishing | Nursing/Integrated Concepts: Nursing Process: Implementation/Health teaching and health promotion Learning Outcome: LO 14.2 Describe and plan nursing interventions to meet nutritional needs for all age groups from infancy through adolescence. MNL LO: Compare the nutritional needs of the infant, toddler, preschool-age child, school-age child, and adolescent. 12) Celiac disease presents many challenges for a family. What should the nurse emphasize when educating the parents of a newly diagnosed child? 1. Ice cream is a safe dessert on a gluten-free diet. 2. The child's weight and height should reach normal levels in about 1 year. 3. Processed foods are usually gluten-free. 4. Insurance pays only a small amount of the cost of celiac diets. Answer: 2 Explanation: 1. Ice cream and many processed foods contain gluten. Payment by insurance is dependent on the plan the family has. Once on a gluten-free diet, the child's height and weight will reach normal range in about 1 year. 2. Ice cream and many processed foods contain gluten. Payment by insurance is dependent on the plan the family has. Once on a gluten-free diet, the child's height and weight will reach normal range in about 1 year. 3. Ice cream and many processed foods contain gluten. Payment by insurance is dependent on the plan the family has. Once on a gluten-free diet, the child's height and weight will reach normal range in about 1 year. 4. Ice cream and many processed foods contain gluten. Payment by insurance is dependent on the plan the family has. Once on a gluten-free diet, the child's height and weight will reach normal range in about 1 year. Page Ref: 298-299 Cognitive Level: Applying Client Need &Sub: Physiological Integrity Standards: QSEN Competencies: Patient-centered care | AACN Essential Competencies: Essential VII: Clinical prevention and population health | NLN Competencies: Human flourishing | Nursing/Integrated Concepts: Nursing Process: Implementation/Health teaching and health promotion Learning Outcome: LO 14.5 Develop nursing interventions for children with nutritional disorders. MNL LO: Analyze socioeconomic factors that impact childhood nutrition. 13) While teaching a health promotion class to a group of parents of children in a Head Start class, which information should the nurse include to help decrease the risk of dental caries? 1. Delay introducing cow's milk until at least 1 year of age. 2. Offer drinking cups only at meal and snack times. 3. Encourage use of homemade baby food without preservatives. 4. Offer juices diluted 50 percent with water. Answer: 2 Explanation: 1. Offering drinking cups only at meal and snack times encourages drinking when thirsty rather than carrying a cup around. This reduces the risk of dental caries. Delaying the introduction of cow's milk, making homemade baby food, or diluting juice does not decrease dental caries. 2. Offering drinking cups only at meal and snack times encourages drinking when thirsty rather than carrying a cup around. This reduces the risk of dental caries. Delaying the introduction of cow's milk, making homemade baby food, or diluting juice does not decrease dental caries. 3. Offering drinking cups only at meal and snack times encourages drinking when thirsty rather than carrying a cup around. This reduces the risk of dental caries. Delaying the introduction of cow's milk, making homemade baby food, or diluting juice does not decrease dental caries. 4. Offering drinking cups only at meal and snack times encourages drinking when thirsty rather than carrying a cup around. This reduces the risk of dental caries. Delaying the introduction of cow's milk, making homemade baby food, or diluting juice does not decrease dental caries. Page Ref: 284-285 Cognitive Level: Applying Client Need &Sub: Health Promotion and Maintenance Standards: QSEN Competencies: Evidence-based practice | AACN Essential Competencies: Essential VII: Clinical prevention and population health | NLN Competencies: Human flourishing | Nursing/Integrated Concepts: Nursing Process: Implementation/Health teaching and health promotion Learning Outcome: LO 14.4 Identify and explain common nutritional problems of children. MNL LO: Relate nutrition to the prevention of illness and the maintenance of child health and wellness. 14) The nurse is planning a class for school-age children on prevention of obesity through exercise. It is important to encourage the children to exercise a minimum of how many minutes a day to meet current recommendations? 1. 20 minutes 2. 30 minutes 3. 60 minutes 4. 90 minutes Answer: 3 Explanation: 1. The current recommendation is 60 minutes of exercise daily. 2. The current recommendation is 60 minutes of exercise daily. 3. The current recommendation is 60 minutes of exercise daily. 4. The current recommendation is 60 minutes of exercise daily. Page Ref: 294-296 Cognitive Level: Applying Client Need &Sub: Health Promotion and Maintenance Standards: QSEN Competencies: Patient-centered care | AACN Essential Competencies: Essential VII: Clinical prevention and population health | NLN Competencies: Human flourishing | Nursing/Integrated Concepts: Nursing Process: Implementation/Health teaching and health promotion Learning Outcome: LO 14.4 Identify and explain common nutritional problems of children. MNL LO: Analyze socioeconomic factors that impact childhood nutrition. 15) Parents of a child who will begin enteral feedings ask the nurse what advantage this type of feeding has over other methods. Which responses by the nurse are the most appropriate? Select all that apply. 1. "Enteral feeding is the closest to natural feeding methods." 2. "The child must be able to absorb nutrients." 3. "Enteral feeding is complex to administer." 4. "Enteral feeding requires a central venous catheter." 5. "Enteral feeding has a high success rate." Answer: 1, 2, 5 Explanation: 1. Enteral feedings are the closest to natural feeding methods. The child must be able to absorb nutrients. Enteral feeding has a high success rate. It is not complex to administer, and does not require a central venous catheter. 2. Enteral feedings are the closest to natural feeding methods. The child must be able to absorb nutrients. Enteral feeding has a high success rate. It is not complex to administer, and does not require a central venous catheter. 3. Enteral feedings are the closest to natural feeding methods. The child must be able to absorb nutrients. Enteral feeding has a high success rate. It is not complex to administer, and does not require a central venous catheter. 4. Enteral feedings are the closest to natural feeding methods. The child must be able to absorb nutrients. Enteral feeding has a high success rate. It is not complex to administer, and does not require a central venous catheter. 5. Enteral feedings are the closest to natural feeding methods. The child must be able to absorb nutrients. Enteral feeding has a high success rate. It is not complex to administer, and does not require a central venous catheter. Page Ref: 307-308 Cognitive Level: Applying Client Need &Sub: Physiological Integrity Standards: QSEN Competencies: Patient-centered care | AACN Essential Competencies: Essential VII: Clinical prevention and population health | NLN Competencies: Human flourishing | Nursing/Integrated Concepts: Nursing Process: Planning/Health teaching and health promotion Learning Outcome: LO 14.5 Develop nursing interventions for children with nutritional disorders. MNL LO: Analyze socioeconomic factors that impact childhood nutrition. 16) The nurse is providing care to a pediatric client recently diagnosed with celiac disease. Which food choice indicates appropriate understanding of the material presented? 1. Pizza with milk 2. Spaghetti and meat sauce with juice 3. Hot dog on a bun with a shake 4. Fruit plate with Gatorade Answer: 4 Explanation: 1. A child with celiac disease needs a gluten-free diet. Included on the list are fruits, meats, rice, and vegetables, including corn. Excluded are bread, cake, doughnuts, cookies, crackers, and many processed foods that may contain hidden gluten. Therefore, the child would be allowed to have the fruit plate with Gatorade. 2. A child with celiac disease needs a gluten-free diet. Included on the list are fruits, meats, rice, and vegetables, including corn. Excluded are bread, cake, doughnuts, cookies, crackers, and many processed foods that may contain hidden gluten. Therefore, the child would be allowed to have the fruit plate with Gatorade. 3. A child with celiac disease needs a gluten-free diet. Included on the list are fruits, meats, rice, and vegetables, including corn. Excluded are bread, cake, doughnuts, cookies, crackers, and many processed foods that may contain hidden gluten. Therefore, the child would be allowed to have the fruit plate with Gatorade. 4. A child with celiac disease needs a gluten-free diet. Included on the list are fruits, meats, rice, and vegetables, including corn. Excluded are bread, cake, doughnuts, cookies, crackers, and many processed foods that may contain hidden gluten. Therefore, the child would be allowed to have the fruit plate with Gatorade. Page Ref: 299 Cognitive Level: Applying Client Need &Sub: Health Promotion and Maintenance Standards: QSEN Competencies: Patient-centered care | AACN Essential Competencies: Essential IX: Baccalaureate generalist nursing practice | NLN Competencies: Nursing judgement | Nursing/Integrated Concepts: Nursing Process: Evaluation/Coordination of care Learning Outcome: LO 14.5 Develop nursing interventions for children with nutritional disorders. MNL LO: Analyze socioeconomic factors that impact childhood nutrition. 17) The nurse is conducting a nutritional assessment for a toddler client who is diagnosed with failure to thrive (FTT). Which parameters will the nurse include in the assessment process for this toddler and family? Select all that apply. 1. Height 2. Weight 3. Hemoglobin and hematocrit 4. Twenty-four-hour food diary 5. Maternal dietary intake during pregnancy Answer: 1, 2, 3, 4 Explanation: 1. In order to adequately assess the toddler client's FTT, the nurse would plan to measure height and weight; obtain a hemoglobin and hematocrit; and ask the family for a 24- hour food diary. Information regarding maternal dietary intake during pregnancy is not information that is necessary to assess for a toddler diagnosed with FTT. 2. In order to adequately assess the toddler client's FTT, the nurse would plan to measure height and weight; obtain a hemoglobin and hematocrit; and ask the family for a 24-hour food diary. Information regarding maternal dietary intake during pregnancy is not information that is necessary to assess for a toddler diagnosed with FTT. 3. In order to adequately assess the toddler client's FTT, the nurse would plan to measure height and weight; obtain a hemoglobin and hematocrit; and ask the family for a 24-hour food diary. Information regarding maternal dietary intake during pregnancy is not information that is necessary to assess for a toddler diagnosed with FTT. 4. In order to adequately assess the toddler client's FTT, the nurse would plan to measure height and weight; obtain a hemoglobin and hematocrit; and ask the family for a 24-hour food diary. Information regarding maternal dietary intake during pregnancy is not information that is necessary to assess for a toddler diagnosed with FTT. 5. In order to adequately assess the toddler client's FTT, the nurse would plan to measure height and weight; obtain a hemoglobin and hematocrit; and ask the family for a 24-hour food diary. Information regarding maternal dietary intake during pregnancy is not information that is necessary to assess for a toddler diagnosed with FTT. Page Ref: 300 Cognitive Level: Applying Client Need &Sub: Health Promotion and Maintenance Standards: QSEN Competencies: Patient-centered care | AACN Essential Competencies: Essential IX: Baccalaureate generalist nursing practice | NLN Competencies: Human flourishing | Nursing/Integrated Concepts: Nursing Process: Assessment/Coordination of care Learning Outcome: LO 14.3 Integrate methods of nutritional assessment into nursing care of infants, children, and adolescents. MNL LO: Examine the components of performing a nutritional assessment of the pediatric client. 18) The nurse is assessing a 14-year-old and notes signs and symptoms of bulimia nervosa. Which assessments led the nurse to this conclusion? Select all that apply. 1. Pale skin 2. Dry, splitting hair 3. Erosion of tooth enamel 4. Calluses on back of hand 5. Gum recession Answer: 3, 4, 5 Explanation: 1. Pale skin is not a sign and symptom of bulimia nervosa. 2. Dry, splitting hair is not a sign and symptom of bulimia nervosa. 3. Erosion of tooth enamel is a sign and symptom of bulimia nervosa. 4. Calluses on back of hand is a sign and symptom of bulimia nervosa. 5. Gum recession is a sign and symptom of bulimia nervosa. Page Ref: 303-304 Cognitive Level: Analyzing Client Need &Sub: Health Promotion and Maintenance Standards: QSEN Competencies: Patient-centered care | AACN Essential Competencies: Essential IX: Baccalaureate generalist nursing practice | NLN Competencies: Nursing judgement | Nursing/Integrated Concepts: Nursing Process: Assessment/Coordination of care Learning Outcome: LO 14.3 Integrate methods of nutritional assessment into nursing care of infants, children, and adolescents. MNL LO: Examine the components of performing a nutritional assessment of the pediatric client. 19) The nurse is assessing an adolescent and notes signs and symptoms of anorexia nervosa. Which signs and symptoms led the nurse to believe the adolescent has this condition? Select all that apply. 1. Extreme weight loss 2. Depression 3. Irregular menses 4. Sedentary lifestyle 5. Bradycardia Answer: 1, 2, 3, 5 Explanation: 1. Extreme weight loss is a sign and symptom of anorexia nervosa. 2. Depression is a sign and symptom of anorexia nervosa. 3. Irregular menses is a sign and symptom of anorexia nervosa. 4. Sedentary lifestyle is not a sign and symptom of anorexia nervosa. 5. Bradycardia is a sign and symptom of anorexia nervosa. Page Ref: 301-303 Cognitive Level: Analyzing Client Need &Sub: Health Promotion and Maintenance Standards: QSEN Competencies: Patient-centered care | AACN Essential Competencies: Essential IX: Baccalaureate generalist nursing practice | NLN Competencies: Nursing judgement | Nursing/Integrated Concepts: Nursing Process: Assessment/Coordination of care Learning Outcome: LO 14.3 Integrate methods of nutritional assessment into nursing care of infants, children, and adolescents. MNL LO: Examine the components of performing a nutritional assessment of the pediatric client. Principles of Pediatric Nursing: Caring for Children, 7e (Ball et al.) Chapter 15 Pain Assessment and Management in Children 1) The nurse is admitting a school-age Vietnamese client who hit a parked car while riding a bike. The child has a fracture of the left radius and femur in addition to a fractured orbit. The child is stoic and denies pain. Which nursing actions are most appropriate in this situation? Select all that apply. 1. Use the FLACC scale to determine the child's pain level. 2. Tell the child to ring the call bell if the leg starts hurting. 3. Administer pain medication now and continue on a regular basis. 4. Ask the child's parents to notify the nurse if the child complains of pain. 5. Use the NIPS scale to determine the child's pain level. Answer: 1, 3, 4 Explanation: 1. Based on the type of injuries the child has, pain will be present. Analgesics should be given on a scheduled basis so that the pain does not get out of control. The FLACC scale is the most appropriate tool to use with an 8-year-old. The child's stoic expression is likely to be culturally related, and the child may not admit hurting. While asking the parents to call the nurse is not inappropriate, it is not the most appropriate initial action. The NIPS scale is appropriate for a newborn, not a school-age, client. 2. Based on the type of injuries the child has, pain will be present. Analgesics should be given on a scheduled basis so that the pain does not get out of control. The FLACC scale is the most appropriate tool to use with an 8-year-old. The child's stoic expression is likely to be culturally related, and the child may not admit hurting. While asking the parents to call the nurse is not inappropriate, it is not the most appropriate initial action. The NIPS scale is appropriate for a newborn, not a school-age, client. 3. Based on the type of injuries the child has, pain will be present. Analgesics should be given on a scheduled basis so that the pain does not get out of control. The FLACC scale is the most appropriate tool to use with an 8-year-old. The child's stoic expression is likely to be culturally related, and the child may not admit hurting. While asking the parents to call the nurse is not inappropriate, it is not the most appropriate initial action. The NIPS scale is appropriate for a newborn, not a school-age, client. 4. Based on the type of injuries the child has, pain will be present. Analgesics should be given on a scheduled basis so that the pain does not get out of control. The FLACC scale is the most appropriate tool to use with an 8-year-old. The child's stoic expression is likely to be culturally related, and the child may not admit hurting. While asking the parents to call the nurse is not inappropriate, it is not the most appropriate initial action. The NIPS scale is appropriate for a newborn, not a school-age, client. 5. Based on the type of injuries the child has, pain will be present. Analgesics should be given on a scheduled basis so that the pain does not get out of control. The FLACC scale is the most appropriate tool to use with an 8-year-old. The child's stoic expression is likely to be culturally related, and the child may not admit hurting. While asking the parents to call the nurse is not inappropriate, it is not the most appropriate initial action. The NIPS scale is appropriate for a newborn, not a school-age, client. Page Ref: 313-317 Cognitive Level: Analyzing Client Need &Sub: Physiological Integrity Standards: QSEN Competencies: Patient-centered care | AACN Essential Competencies: Essential IX: Baccalaureate generalist nursing practice | NLN Competencies: Professional identity | Nursing/Integrated Concepts: Nursing Process: Implementation/Coordination of care Learning Outcome: LO 15.2 Analyze the behaviors of an infant or a child to assess for pain. MNL LO: Examine the pathophysiology of pain perception and associated concepts seen in children. 2) During the nurse's initial assessment of a school-age child, the child reports a pain level of 6 out of 10. The child is lying quietly in bed watching television. Which action by the nurse is most appropriate? 1. Administer prescribed analgesic. 2. Ask the child's parents if they think the child is hurting. 3. Reassess the child in 15 minutes to see if the pain rating has changed. 4. Do nothing, since the child appears to be resting. Answer: 1 Explanation: 1. School-age children are old enough to accurately report their pain level. A pain score of 6 is an indication for prompt administration of pain medication. The child may be trying to be brave or may be lying still because movement is painful. 2. School-age children are old enough to accurately report their pain level. A pain score of 6 is an indication for prompt administration of pain medication. The child may be trying to be brave or may be lying still because movement is painful. 3. School-age children are old enough to accurately report their pain level. A pain score of 6 is an indication for prompt administration of pain medication. The child may be trying to be brave or may be lying still because movement is painful. 4. School-age children are old enough to accurately report their pain level. A pain score of 6 is an indication for prompt administration of pain medication. The child may be trying to be brave or may be lying still because movement is painful. Page Ref: 313 Cognitive Level: Applying Client Need &Sub: Physiological Integrity Standards: QSEN Competencies: Patient-centered care | AACN Essential Competencies: Essential IX: Baccalaureate generalist nursing practice | NLN Competencies: Nursing judgement | Nursing/Integrated Concepts: Nursing Process: Assessment/Coordination of care Learning Outcome: LO 15.3 Assess the developmental abilities of children to perform a self- assessment of pain intensity. MNL LO: Determine developmental and cultural factors and tools used to assess pain in the pediatric client. 3) A school-age client has been receiving morphine every two hours for postoperative pain as ordered. The medication relieves the pain for approximately 90 minutes, and then the pain returns. Which action by the nurse is the most appropriate? 1. Tell the child that pain medication cannot be administered more frequently than every two hours. 2. Reposition the child and quietly leave the room. 3. Inform the parents that the child is dependent on the medication. 4. Call the healthcare provider to see if the child's orders for pain medication can be changed. Answer: 4 Explanation: 1. The nurse has the responsibility of relieving the child's pain. The child has been receiving the prescribed medication on a regular basis. The healthcare provider should be called to see if the child's orders can be changed. This child might do well with patient-controlled analgesia (PCA). Oral medications such as acetaminophen and NSAIDs can be given with morphine to provide optimum pain relief. 2. The nurse has the responsibility of relieving the child's pain. The child has been receiving the prescribed medication on a regular basis. The healthcare provider should be called to see if the child's orders can be changed. This child might do well with patient-controlled analgesia (PCA). Oral medications such as acetaminophen and NSAIDs can be given with morphine to provide optimum pain relief. 3. The nurse has the responsibility of relieving the child's pain. The child has been receiving the prescribed medication on a regular basis. The healthcare provider should be called to see if the child's orders can be changed. This child might do well with patient-controlled analgesia (PCA). Oral medications such as acetaminophen and NSAIDs can be given with morphine to provide optimum pain relief. 4. The nurse has the responsibility of relieving the child's pain. The child has been receiving the prescribed medication on a regular basis. The healthcare provider should be called to see if the child's orders can be changed. This child might do well with patient-controlled analgesia (PCA). Oral medications such as acetaminophen and NSAIDs can be given with morphine to provide optimum pain relief. Page Ref: 319 Cognitive Level: Applying Client Need &Sub: Physiological Integrity Standards: QSEN Competencies: Patient-centered care | AACN Essential Competencies: Essential IX: Baccalaureate generalist nursing practice | NLN Competencies: Nursing judgement | Nursing/Integrated Concepts: Nursing Process: Implementation/Coordination of care Learning Outcome: LO 15.4 Plan the nursing care for a child receiving an opioid analgesic. MNL LO: Apply the nursing process to provide age-appropriate care for pediatric clients experiencing pain. 4) The nurse is working in a pediatric surgical unit. In discussing patient-controlled analgesia (PCA) in a preoperative parental meeting, which client would be a candidate for PCA? 1. Developmentally delayed 16-year-old, postoperative bone surgery 2. A 5-year-old, postoperative tonsillectomy 3. A 10-year-old who has a fractured femur and concussion from a bike accident 4. A 12-year-old, postoperative spinal fusion for scoliosis Answer: 4 Explanation: 1. Patient-controlled analgesia (PCA) is most appropriate in children 5 years and over. The child must be able to press the button and understand that she will receive pain medicine by pushing the button. PCA is generally prescribed for clients who will be hospitalized for at least 48 hours. Children who are developmentally delayed or have suffered head trauma are not candidates for PCA. 2. Patient-controlled analgesia (PCA) is most appropriate in children 5 years and over. The child must be able to press the button and understand that she will receive pain medicine by pushing the button. PCA is generally prescribed for clients who will be hospitalized for at least 48 hours. Children who are developmentally delayed or have suffered head trauma are not candidates for PCA. 3. Patient-controlled analgesia (PCA) is most appropriate in children 5 years and over. The child must be able to press the button and understand that she will receive pain medicine by pushing the button. PCA is generally prescribed for clients who will be hospitalized for at least 48 hours. Children who are developmentally delayed or have suffered head trauma are not candidates for PCA. 4. Patient-controlled analgesia (PCA) is most appropriate in children 5 years and over. The child must be able to press the button and understand that she will receive pain medicine by pushing the button. PCA is generally prescribed for clients who will be hospitalized for at least 48 hours. Children who are developmentally delayed or have suffered head trauma are not candidates for PCA. Page Ref: 322 Cognitive Level: Analyzing Client Need &Sub: Physiological Integrity Standards: QSEN Competencies: Patient-centered care | AACN Essential Competencies: Essential II: Basic organizational and systems leadership for quality care and patient safety | NLN Competencies: Nursing judgement | Nursing/Integrated Concepts: Nursing Process: Implementation/Coordination of care Learning Outcome: LO 15.7 Develop a nursing care plan for the child with a chronic painful condition. MNL LO: Differentiate the developmentally appropriate therapies used to treat pain in the pediatric client. 5) A toddler is hospitalized with a fractured femur. In addition to pain medication, which will best provide pain relief for this child? 1. Parents' presence at the bedside 2. Age-appropriate toys 3. Deep-breathing exercises 4. Videos for the child to watch Answer: 1 Explanation: 1. Parents' presence at the bedside reduces anxiety and subsequently reduces pain. Although play and other methods of distraction might be somewhat effective, they do not equal the comfort that parents' presence provides, especially in a 2-year-old, who is also at high risk for separation anxiety. 2. Parents' presence at the bedside reduces anxiety and subsequently reduces pain. Although play and other methods of distraction might be somewhat effective, they do not equal the comfort that parents' presence provides, especially in a 2-year-old, who is also at high risk for separation anxiety. 3. Parents' presence at the bedside reduces anxiety and subsequently reduces pain. Although play and other methods of distraction might be somewhat effective, they do not equal the comfort that parents' presence provides, especially in a 2-year-old, who is also at high risk for separation anxiety. 4. Parents' presence at the bedside reduces anxiety and subsequently reduces pain. Although play and other methods of distraction might be somewhat effective, they do not equal the comfort that parents' presence provides, especially in a 2-year-old, who is also at high risk for separation anxiety. Page Ref: 327 Cognitive Level: Applying Client Need &Sub: Physiological Integrity Standards: QSEN Competencies: Patient-centered care | AACN Essential Competencies: Essential IX: Baccalaureate generalist nursing practice | NLN Competencies: Nursing judgement | Nursing/Integrated Concepts: Nursing Process: Planning/Coordination of care Learning Outcome: LO 15.7 Develop a nursing care plan for the child with a chronic painful condition. MNL LO: Determine developmental and cultural factors and tools used to assess pain in the pediatric client. 6) The nurse is caring for a toddler client in the postoperative period. Which pain assessment tool is most appropriate for this client? 1. FLACC Behavioral Pain Assessment Scale 2. FACES pain scale 3. Oucher scale 4. Poker-chip tool Answer: 1 Explanation: 1. The FLACC scale is an appropriate tool for infants and young children who cannot self-report pain. The FACES Scale, Oucher scale, and poker-chip tool are all self-report scales. 2. The FLACC scale is an appropriate tool for infants and young children who cannot self-report pain. The FACES Scale, Oucher scale, and poker-chip tool are all self-report scales. 3. The FLACC scale is an appropriate tool for infants and young children who cannot self-report pain. The FACES Scale, Oucher scale, and poker-chip tool are all self-report scales. 4. The FLACC scale is an appropriate tool for infants and young children who cannot self-report pain. The FACES Scale, Oucher scale, and poker-chip tool are all self-report scales. Page Ref: 316-318 Cognitive Level: Applying Client Need &Sub: Physiological Integrity Standards: QSEN Competencies: Patient-centered care | AACN Essential Competencies: Essential IX: Baccalaureate generalist nursing practice | NLN Competencies: Nursing judgement | Nursing/Integrated Concepts: Nursing Process: Assessment/Coordination of care Learning Outcome: LO 15.2 Analyze the behaviors of an infant or a child to assess for pain. MNL LO: Examine the pathophysiology of pain perception and associated concepts seen in children. 7) A hospitalized toddler-age client needs to have an IV restarted. The child begins to cry when carried into the treatment room by the mother. Which nursing diagnosis is most appropriate? 1. Ineffective Individual Coping Related to an Invasive Procedure 2. Anxiety Related to Anticipated Painful Procedure 3. Fear Related to the Unfamiliar Environment 4. Knowledge Deficit of the Procedure Answer: 2 Explanation: 1. At this age, the child is not old enough to understand the need for an IV infusion. The stem indicates that the child has been through this painful procedure before, and his reaction to entering the treatment room is based on anticipation of repeat discomfort. The child's behavior is appropriate for a child of this age. 2. At this age, the child is not old enough to understand the need for an IV infusion. The stem indicates that the child has been through this painful procedure before, and his reaction to entering the treatment room is based on anticipation of repeat discomfort. The child's behavior is appropriate for a child of this age. 3. At this age, the child is not old enough to understand the need for an IV infusion. The stem indicates that the child has been through this painful procedure before, and his reaction to entering the treatment room is based on anticipation of repeat discomfort. The child's behavior is appropriate for a child of this age. 4. At this age, the child is not old enough to understand the need for an IV infusion. The stem indicates that the child has been through this painful procedure before, and his reaction to entering the treatment room is based on anticipation of repeat discomfort. The child's behavior is appropriate for a child of this age. Page Ref: 319 Cognitive Level: Analyzing Client Need &Sub: Psychosocial Integrity Standards: QSEN Competencies: Patient-centered care | AACN Essential Competencies: Essential IX: Baccalaureate generalist nursing practice | NLN Competencies: Nursing judgement | Nursing/Integrated Concepts: Nursing Process: Diagnosis/Coordination of care Learning Outcome: LO 15.1 Summarize the physiologic and behavioral consequences of pain in infants and children. MNL LO: Determine developmental and cultural factors and tools used to assess pain in the pediatric client. 8) A parent asks the nurse if there is anything that can be done to reduce the pain that his 3-year- old experiences each morning when blood is drawn for lab studies. Which intervention would the nurse implement- based on the parent's concern? 1. Intravenous sedation 15 minutes prior to the procedure 2. EMLA cream (lidocaine 2.5% and prilocaine 2.5%) applied to skin at least one hour prior to the procedure 3. Use of guided imagery during the procedure 4. Use of muscle-relaxation techniques Answer: 2 Explanation: 1. Sedation is not generally used with quick minor procedures such as venipuncture. A 3-year-old is too young to participate in techniques such as muscle relaxation and guided imagery. EMLA cream is shown to be effective in providing topical anesthesia if applied at least one hour prior to the procedure. 2. Sedation is not generally used with quick minor procedures such as venipuncture. A 3-year- old is too young to participate in techniques such as muscle relaxation and guided imagery. EMLA cream is shown to be effective in providing topical anesthesia if applied at least one hour prior to the procedure. 3. Sedation is not generally used with quick minor procedures such as venipuncture. A 3-year- old is too young to participate in techniques such as muscle relaxation and guided imagery. EMLA cream is shown to be effective in providing topical anesthesia if applied at least one hour prior to the procedure. 4. Sedation is not generally used with quick minor procedures such as venipuncture. A 3-year- old is too young to participate in techniques such as muscle relaxation and guided imagery. EMLA cream is shown to be effective in providing topical anesthesia if applied at least one hour prior to the procedure. Page Ref: 327-328 Cognitive Level: Applying Client Need &Sub: Physiological Integrity Standards: QSEN Competencies: Patient-centered care | AACN Essential Competencies: Essential IX: Baccalaureate generalist nursing practice | NLN Competencies: Nursing judgement | Nursing/Integrated Concepts: Nursing Process: Implementation/Coordination of care Learning Outcome: LO 15.6 Plan nursing care for a child in acute pain that integrates pharmacologic interventions and developmentally appropriate nonpharmacologic (complementary) therapies. MNL LO: Differentiate the developmentally appropriate therapies used to treat pain in the pediatric client. 9) As an advocate for the child undergoing bone-marrow aspiration, which intervention would the nurse suggest to decrease the pain experienced due to the procedure? 1. General anesthesia 2. Conscious sedation 3. Intravenous narcotics ten minutes before the procedure 4. Oral pain medication for discomfort after the procedure Answer: 2 Explanation: 1. For the child undergoing repeated procedures, it is important for the child to be sedated prior to and during the initial procedure. General anesthesia is not necessary for bone- marrow aspiration. Narcotics alone will not provide appropriate sedation to keep the child from remembering the procedure. While oral pain medication postprocedure is not inappropriate if discomfort exists, it is not the best answer. The child will have great anxiety and discomfort during the procedures and prior to future procedures. 2. For the child undergoing repeated procedures, it is important for the child to be sedated prior to and during the initial procedure. General anesthesia is not necessary for bone-marrow aspiration. Narcotics alone will not provide appropriate sedation to keep the child from remembering the procedure. While oral pain medication postprocedure is not inappropriate if discomfort exists, it is not the best answer. The child will have great anxiety and discomfort during the procedures and prior to future procedures. 3. For the child undergoing repeated procedures, it is important for the child to be sedated prior to and during the initial procedure. General anesthesia is not necessary for bone-marrow aspiration. Narcotics alone will not provide appropriate sedation to keep the child from remembering the procedure. While oral pain medication postprocedure is not inappropriate if discomfort exists, it is not the best answer. The child will have great anxiety and discomfort during the procedures and prior to future procedures. 4. For the child undergoing repeated procedures, it is important for the child to be sedated prior to and during the initial procedure. General anesthesia is not necessary for bone-marrow aspiration. Narcotics alone will not provide appropriate sedation to keep the child from remembering the procedure. While oral pain medication postprocedure is not inappropriate if discomfort exists, it is not the best answer. The child will have great anxiety and discomfort during the procedures and prior to future procedures. Page Ref: 330-331 Cognitive Level: Applying Client Need &Sub: Psychosocial Integrity Standards: QSEN Competencies: Patient-centered care | AACN Essential Competencies: Essential IX: Baccalaureate generalist nursing practice | NLN Competencies: Nursing judgement | Nursing/Integrated Concepts: Nursing Process: Implementation/Coordination of care Learning Outcome: LO 15.8 Develop a nursing care plan for assessing and monitoring the child having sedation and analgesia for a medical procedure. MNL LO: Apply the nursing process to provide age-appropriate care for pediatric clients experiencing pain. 10) A school-age client is being discharged from the outpatient surgical center. Which statement by the parent would indicate the need for further teaching? 1. "I can expect my child to have some pain for the next few days." 2. "I will plan to give my child pain medicine around the clock for the next day or so." 3. "Since my child just had surgery today, I can expect the pain level to be higher tomorrow." 4. "I will call the office tomorrow if the pain medicine is not relieving the pain." Answer: 3 Explanation: 1. Increasing pain can be a sign of complication and should be reported to the physician; therefore, if the parent expects the pain to be higher the next day, the nurse should clarify expectations for pain control. The child is expected to have some pain for a few days after surgery and should receive pain medication on a scheduled basis. If prescribed medication is not relieving the pain to a satisfactory level, the physician should be notified. 2. Increasing pain can be a sign of complication and should be reported to the physician; therefore, if the parent expects the pain to be higher the next day, the nurse should clarify expectations for pain control. The child is expected to have some pain for a few days after surgery and should receive pain medication on a scheduled basis. If prescribed medication is not relieving the pain to a satisfactory level, the physician should be notified. 3. Increasing pain can be a sign of complication and should be reported to the physician; therefore, if the parent expects the pain to be higher the next day, the nurse should clarify expectations for pain control. The child is expected to have some pain for a few days after surgery and should receive pain medication on a scheduled basis. If prescribed medication is not relieving the pain to a satisfactory level, the physician should be notified. 4. Increasing pain can be a sign of complication and should be reported to the physician; therefore, if the parent expects the pain to be higher the next day, the nurse should clarify expectations for pain control. The child is expected to have some pain for a few days after surgery and should receive pain medication on a scheduled basis. If prescribed medication is not relieving the pain to a satisfactory level, the physician should be notified. Page Ref: 328-329 Cognitive Level: Analyzing Client Need &Sub: Physiological Integrity Standards: QSEN Competencies: Patient-centered care | AACN Essential Competencies: Essential VII: Clinical prevention and population health | NLN Competencies: Nursing judgement | Nursing/Integrated Concepts: Nursing Process: Evaluation/Health teaching and health promotion Learning Outcome: LO 15.7 Develop a nursing care plan for the child with a chronic painful condition. MNL LO: Differentiate the developmentally appropriate therapies used to treat pain in the pediatric client. 11) The nurse is caring for a child who has been sedated for a painful procedure. Which nursing activity is the priority for this child? 1. Allow parents to stay with the child. 2. Monitor pulse oximetry. 3. Assess the child's respiratory effort. 4. Place the child on a cardiac monitor. Answer: 3 Explanation: 1. When the child is sedated for a procedure, it is very important for the nurse to actually visualize the child and his effort of breathing. Although equipment is important and is used routinely during sedation, it does not replace the need for visual assessment. Parents may be allowed to stay with the child, but assessment of breathing effort must take priority. 2. When the child is sedated for a procedure, it is very important for the nurse to actually visualize the child and his effort of breathing. Although equipment is important and is used routinely during sedation, it does not replace the need for visual assessment. Parents may be allowed to stay with the child, but assessment of breathing effort must take priority. 3. When the child is sedated for a procedure, it is very important for the nurse to actually visualize the child and his effort of breathing. Although equipment is important and is used routinely during sedation, it does not replace the need for visual assessment. Parents may be allowed to stay with the child, but assessment of breathing effort must take priority. 4. When the child is sedated for a procedure, it is very important for the nurse to actually visualize the child and his effort of breathing. Although equipment is important and is used routinely during sedation, it does not replace the need for visual assessment. Parents may be allowed to stay with the child, but assessment of breathing effort must take priority. Page Ref: 331 Cognitive Level: Analyzing Client Need &Sub: Physiological Integrity Standards: QSEN Competencies: Patient-centered care | AACN Essential Competencies: Essential IX: Baccalaureate generalist nursing practice NLN Competencies: Nursing judgement | Nursing/Integrated Concepts: Nursing Process: Implementation/Quality of practice Learning Outcome: LO 15.8 Develop a nursing care plan for assessing and monitoring the child having sedation and analgesia for a medical procedure. MNL LO: Apply the nursing process to provide age-appropriate care for pediatric clients experiencing pain. 12) A young school-age client is hospitalized with a fractured femur. Which assessment tools are appropriate for this client? Select all that apply. 1. FACES pain scale 2. Oucher scale 3. Visual Analog Scale 4. CRIES Scale 5. Poker-chip tool Answer: 1, 2, 5 Explanation: 1. A young school-age client should be able to use the FACES Scale and Oucher scale to choose which face best matches the child's pain level. The child should also be able to count and understand the concepts of the poker-chip tool. The CRIES Scale was developed for preterm and full-term neonates. A young school-age client is not old enough to use the Visual Analog Scale. 2. A young school-age client should be able to use the FACES Scale and Oucher scale to choose which face best matches the child's pain level. The child should also be able to count and understand the concepts of the poker-chip tool. The CRIES Scale was developed for preterm and full-term neonates. A young school-age client is not old enough to use the Visual Analog Scale. 3. A young school-age client should be able to use the FACES Scale and Oucher scale to choose which face best matches the child's pain level. The child should also be able to count and understand the concepts of the poker-chip tool. The CRIES Scale was developed for preterm and full-term neonates. A young school-age client is not old enough to use the Visual Analog Scale. 4. A young school-age client should be able to use the FACES Scale and Oucher scale to choose which face best matches the child's pain level. The child should also be able to count and understand the concepts of the poker-chip tool. The CRIES Scale was developed for preterm and full-term neonates. A young school-age client is not old enough to use the Visual Analog Scale. 5. A young school-age client should be able to use the FACES Scale and Oucher scale to choose which face best matches the child's pain level. The child should also be able to count and understand the concepts of the poker-chip tool. The CRIES Scale was developed for preterm and full-term neonates. A young school-age client is not old enough to use the Visual Analog Scale. Page Ref: 316-318 Cognitive Level: Applying Client Need &Sub: Physiological Integrity Standards: QSEN Competencies: Patient-centered care | AACN Essential Competencies: Essential IX: Baccalaureate generalist nursing practice | NLN Competencies: Nursing judgement | Nursing/Integrated Concepts: Nursing Process: Assessment/Coordination of care Learning Outcome: LO 15.3 Assess the developmental abilities of children to perform a self- assessment of pain intensity. MNL LO: Determine developmental and cultural factors and tools used to assess pain in the pediatric client. 13) The nurse is caring for a child who has a long leg cast. The child complains of increasing pain in the toes of the casted foot. Which initial action by the nurse is the most appropriate? 1. Call the healthcare provider to report increasing pain 2. Administer pain medication 3. Reposition the child in bed 4. Check to see if the cast is too tight Answer: 4 Explanation: 1. While all of the actions are appropriate, the nurse's initial action is to assess for external factors that might be causing pain. 2. While all of the actions are appropriate, the nurse's initial action is to assess for external factors that might be causing pain. 3. While all of the actions are appropriate, the nurse's initial action is to assess for external factors that might be causing pain. 4. While all of the actions are appropriate, the nurse's initial action is to assess for external factors that might be causing pain. Page Ref: 324 Cognitive Level: Analyzing Client Need &Sub: Physiological Integrity Standards: QSEN Competencies: Patient-centered care | AACN Essential Competencies: Essential IX: Baccalaureate generalist nursing practice | NLN Competencies: Nursing judgement | Nursing/Integrated Concepts: Nursing Process: Assessment/Coordination of care Learning Outcome: LO 15.6 Plan nursing care for a child in acute pain that integrates pharmacologic interventions and developmentally appropriate nonpharmacologic (complementary) therapies. MNL LO: Differentiate the developmentally appropriate therapies used to treat pain in the pediatric client. 14) The nurse is preparing to perform a heel stick on a neonate. Which complementary therapy is appropriate for the nurse to use decrease pain during this quick but painful procedure? 1. Swaddling 2. Sucrose pacifier 3. Massage 4. Holding the infant Answer: 2 Explanation: 1. Sucrose provides short-term natural pain relief and is most appropriate for use in neonates to decrease pain associated with a quick procedure. The other measures are more appropriate following the procedure or as an adjunct to pain medication for ongoing pain or distress. 2. Sucrose provides short-term natural pain relief and is most appropriate for use in neonates to decrease pain associated with a quick procedure. The other measures are more appropriate following the procedure or as an adjunct to pain medication for ongoing pain or distress. 3. Sucrose provides short-term natural pain relief and is most appropriate for use in neonates to decrease pain associated with a quick procedure. The other measures are more appropriate following the procedure or as an adjunct to pain medication for ongoing pain or distress. 4. Sucrose provides short-term natural pain relief and is most appropriate for use in neonates to decrease pain associated with a quick procedure. The other measures are more appropriate following the procedure or as an adjunct to pain medication for ongoing pain or distress. Page Ref: 323 Cognitive Level: Applying Client Need &Sub: Physiological Integrity Standards: QSEN Competencies: Patient-centered care | AACN Essential Competencies: Essential IX: Baccalaureate generalist nursing practice | NLN Competencies: Nursing judgement | Nursing/Integrated Concepts: Nursing Process: Planning/Coordination of care Learning Outcome: LO 15.5 Examine the role of nonpharmacologic (complementary) interventions in effective pain management. MNL LO: Differentiate the developmentally appropriate therapies used to treat pain in the pediatric client. 15) A preschool-age client is hospitalized following surgery for a ruptured appendix. During assessment of the child, the nurse notes that the child is sleeping. Vital signs are as follows: temperature 97.8 degrees F axillary, pulse 90, respirations 12, and blood pressure 100/60. Which conclusion by the nurse is appropriate based on the assessment findings? 1. The client is comfortable and the pain is controlled. 2. The client is in shock secondary to blood loss during surgery. 3. The client is experiencing respiratory depression secondary to opioid administration for postoperative pain. 4. The client is sleeping to avoid pain associated with surgery. Answer: 3 Explanation: 1. Respiratory depression secondary to opioid use is most likely to occur when the child is sleeping. A respiratory rate of 12 is well below normal for a preschool-age client. The other vital signs are within normal limits for a sleeping preschool-age client. 2. Respiratory depression secondary to opioid use is most likely to occur when the child is sleeping. A respiratory rate of 12 is well below normal for a preschool-age client. The other vital signs are within normal limits for a sleeping preschool-age client. 3. Respiratory depression secondary to opioid use is most likely to occur when the child is sleeping. A respiratory rate of 12 is well below normal for a preschool-age client. The other vital signs are within normal limits for a sleeping preschool-age client. 4. Respiratory depression secondary to opioid use is most likely to occur when the child is sleeping. A respiratory rate of 12 is well below normal for a preschool-age client. The other vital signs are within normal limits for a sleeping preschool-age client. Page Ref: 321 Cognitive Level: Analyzing Client Need &Sub: Physiological Integrity Standards: QSEN Competencies: Patient-centered care | AACN Essential Competencies: Essential IX: Baccalaureate generalist nursing practice | NLN Competencies: Nursing judgement | Nursing/Integrated Concepts: Nursing Process: Evaluation/Coordination of care Learning Outcome: LO 15.6 Plan nursing care for a child in acute pain that integrates pharmacologic interventions and developmentally appropriate nonpharmacologic (complementary) therapies. MNL LO: Differentiate the developmentally appropriate therapies used to treat pain in the pediatric client. 16) The nurse is working with a preschool-age client in Bryant traction for a fractured femur. Why is the Oucher Scale useful to the nurse caring for this child? 1. It provides continuity and consistency in assessing and monitoring the child's pain. 2. It decreases anxiety in the child. 3. It increases the child's comfort level. 4. It reduces the child's fear of painful procedures. Answer: 1 Explanation: 1. Pain assessment scales are used to assess and monitor pain. Using an assessment scale cannot reduce the child's anxiety or fear, nor can it increase the child's comfort level. The nurse can reduce anxiety or fear and increase the child's comfort level by implementing appropriate nursing interventions based on assessment scale data. 2. Pain assessment scales are used to assess and monitor pain. Using an assessment scale cannot reduce the child's anxiety or fear, nor can it increase the child's comfort level. The nurse can reduce anxiety or fear and increase the child's comfort level by implementing appropriate nursing interventions based on assessment scale data. 3. Pain assessment scales are used to assess and monitor pain. Using an assessment scale cannot reduce the child's anxiety or fear, nor can it increase the child's comfort level. The nurse can reduce anxiety or fear and increase the child's comfort level by implementing appropriate nursing interventions based on assessment scale data. 4. Pain assessment scales are used to assess and monitor pain. Using an assessment scale cannot reduce the child's anxiety or fear, nor can it increase the child's comfort level. The nurse can reduce anxiety or fear and increase the child's comfort level by implementing appropriate nursing interventions based on assessment scale data. Page Ref: 317 Cognitive Level: Analyzing Client Need &Sub: Psychosocial Integrity Standards: QSEN Competencies: Patient-centered care | AACN Essential Competencies: Essential IX: Baccalaureate generalist nursing practice | NLN Competencies: Nursing judgement | Nursing/Integrated Concepts: Nursing Process: Implementation/Coordination of care Learning Outcome: LO 15.3 Assess the developmental abilities of children to perform a self- assessment of pain intensity. MNL LO: Determine developmental and cultural factors and tools used to assess pain in the pediatric client. 17) The preschool-age child has been back from surgery for removal of a Wilm's tumor for 6 hours, the nurse anticipates the preschooler will need pain medication very soon. The nurse is aware that the preschool-age child may not complain of pain because Select all that apply. 1. The preschooler cannot give a description of his pain. 2. The preschooler may assume the nurse knows he has pain. 3. The preschooler may be afraid it may hurt more to have the pain treated. 4. The preschooler believes he must be brave. 5. The preschooler uses sleeping to deal with pain. Answer: 1, 2, 3, 4 Explanation: 1. This is why the preschooler may not complain of pain. Children may not complain of pain for several reasons: young children cannot give a description of their pain because of a limited vocabulary or few pain experiences; some children believe they need to be brave and not worry their parents; preschoolers may assume the nurse knows they have pain, and some children are afraid that it will hurt more to have the pain treated. 2. This is why the preschooler may not complain of pain. Children may not complain of pain for several reasons: young children cannot give a description of their pain because of a limited vocabulary or few pain experiences; some children believe they need to be brave and not worry their parents; preschoolers may assume the nurse knows they have pain, and some children are afraid that it will hurt more to have the pain treated. 3. This is why the preschooler may not complain of pain. Children may not complain of pain for several reasons: young children cannot give a description of their pain because of a limited vocabulary or few pain experiences; some children believe they need to be brave and not worry their parents; preschoolers may assume the nurse knows they have pain, and some children are afraid that it will hurt more to have the pain treated. 4. This is why the preschooler may not complain of pain. Children may not complain of pain for several reasons: young children cannot give a description of their pain because of a limited vocabulary or few pain experiences; some children believe they need to be brave and not worry their parents; preschoolers may assume the nurse knows they have pain, and some children are afraid that it will hurt more to have the pain treated. 5. This is not why the preschooler may not complain of pain. Children may not complain of pain for several reasons: young children cannot give a description of their pain because of a limited vocabulary or few pain experiences; some children believe they need to be brave and not worry their parents; preschoolers may assume the nurse knows they have pain, and some children are afraid that it will hurt more to have the pain treated. Page Ref: 313 Cognitive Level: Analyzing Client Need &Sub: Psychosocial Integrity Standards: QSEN Competencies: Patient-centered care | AACN Essential Competencies: Essential IX: Baccalaureate generalist nursing practice | NLN Competencies: Nursing judgement | Nursing/Integrated Concepts: Nursing Process: Evaluation/Coordination of care Learning Outcome: LO 15.3 Assess the developmental abilities of children to perform a self- assessment of pain intensity. MNL LO: Determine developmental and cultural factors and tools used to assess pain in the pediatric client. 18) The adolescent is 6-hours postappendectomy and refuses pain medications. The nurse would like to walk the child in the hall but is concerned that the child has unrelieved pain. The nurse knows that unrelieved pain causes physiologic consequences such as (Select all that apply.) 1. Atelectasis 2. Pneumonia 3. Ileus 4. Lethargy 5. Hypoactive bowel sounds Answer: 1, 2, 3 Explanation: 1. Unrelieved pain causes physiologic consequences, such as atelectasis. 2. Unrelieved pain causes physiologic consequences, such as pneumonia. 3. Unrelieved pain causes physiologic consequences, such as ileus. 4. Unrelieved pain does not cause lethargy. Unrelieved pain causes physiologic consequences, such as alkalosis, decreased O2 saturation, atelectasis, retention of secretions, pneumonia, tachycardia, increased blood pressure, increased intracranial pressure, change in sleep patterns, irritability, fluid and electrolyte losses, altered nutritional intake, hypoglycemia, increased risk of infection, delayed wound healing, impaired GI functioning, poor nutritional intake, ileus, hyperalgesia, decreased pain threshold, and exaggerated memory of painful experiences. 5. Unrelieved pain does not cause hypoactive bowel sounds. Unrelieved pain causes physiologic consequences, such as alkalosis, decreased O2 saturation, atelectasis, retention of secretions, pneumonia, tachycardia, increased blood pressure, increased intracranial pressure, change in sleep patterns, irritability, fluid and electrolyte losses, altered nutritional intake, hypoglycemia, increased risk of infection, delayed wound healing, impaired GI functioning, poor nutritional intake, ileus, hyperalgesia, decreased pain threshold, and exaggerated memory of painful experiences. Page Ref: 315 Cognitive Level: Analyzing Client Need &Sub: Psychosocial Integrity Standards: QSEN Competencies: Patient-centered care | AACN Essential Competencies: Essential IX: Baccalaureate generalist nursing practice | NLN Competencies: Nursing judgement | Nursing/Integrated Concepts: Nursing Process: Evaluation/Coordination of care Learning Outcome: LO 15.1 Summarize the physiologic and behavioral consequences of pain in infants and children. MNL LO: Determine developmental and cultural factors and tools used to assess pain in the pediatric client. Principles of Pediatric Nursing: Caring for Children, 7e (Ball et al.) Chapter 16 Immunizations and Communicable Diseases 1) A nurse is providing information to a group of new mothers. Which statement best explains why newborns and young infants are more susceptible to infection? 1. "They have high levels of maternal antibodies to diseases to which the mother has been exposed." 2. "They have passive transplacental immunity from maternal immunoglobulin G." 3. "They have immune systems that are not fully mature at birth." 4. "They have been exposed to microorganisms during the birth process." Answer: 3 Explanation: 1. Newborns have a limited storage pool of neutrophils and plasma proteins to defend against infection. Newborns' and young infants' high levels of maternal antibodies, passive transplacental immunity, and exposure to microorganisms during the birth process are all true but are incorrect answers because they do not explain the susceptibility of newborns and young infants to infection. 2. Newborns have a limited storage pool of neutrophils and plasma proteins to defend against infection. Newborns' and young infants' high levels of maternal antibodies, passive transplacental immunity, and exposure to microorganisms during the birth process are all true but are incorrect answers because they do not explain the susceptibility of newborns and young infants to infection. 3. Newborns have a limited storage pool of neutrophils and plasma proteins to defend against infection. Newborns' and young infants' high levels of maternal antibodies, passive transplacental immunity, and exposure to microorganisms during the birth process are all true but are incorrect answers because they do not explain the susceptibility of newborns and young infants to infection. 4. Newborns have a limited storage pool of neutrophils and plasma proteins to defend against infection. Newborns' and young infants' high levels of maternal antibodies, passive transplacental immunity, and exposure to microorganisms during the birth process are all true but are incorrect answers because they do not explain the susceptibility of newborns and young infants to infection. Page Ref: 334-335 Cognitive Level: Analyzing Client Need &Sub: Physiological Integrity Standards: QSEN Competencies: Patient-centered care | AACN Essential Competencies: Essential VII: Clinical prevention and population health | NLN Competencies: Nursing judgement | Nursing/Integrated Concepts: Nursing Process: Implementation/Health teaching and health promotion Learning Outcome: LO 16.1 Compare the vulnerability of young children and adults to communicable diseases. MNL LO: Immunity, Infection, and Communicable Disease/Examine etiology, risk factors, pathophysiology, and clinical manifestations as seen in children. 2) The nurse is discussing ways to treat fever in the home environment to a group of parents in the community. Which statement is appropriate for the nurse to include in the presentation? 1. "Ibuprofen is the only effective means to reduce fever." 2. "If the child requires more than one dose of acetaminophen antibiotics are needed." 3. "Purchase a new bottle of acetaminophen for your newborn because it will have recommended medication concentration." 4. "It is not necessary to follow the recommendations on the bottle of ibuprofen as this will not prevent an overdose for your child." Answer: 3 Explanation: 1. The recommendation to purchase a new bottle of acetaminophen due to recommended medication concentrations is an appropriate statement for the nurse to include in the teaching session. The other statements are inaccurate or inappropriate for the nurse to include in the teaching session. 2. The recommendation to purchase a new bottle of acetaminophen due to recommended medication concentrations is an appropriate statement for the nurse to include in the teaching session. The other statements are inaccurate or inappropriate for the nurse to include in the teaching session. 3. The recommendation to purchase a new bottle of acetaminophen due to recommended medication concentrations is an appropriate statement for the nurse to include in the teaching session. The other statements are inaccurate or inappropriate for the nurse to include in the teaching session. 4. The recommendation to purchase a new bottle of acetaminophen due to recommended medication concentrations is an appropriate statement for the nurse to include in the teaching session. The other statements are inaccurate or inappropriate for the nurse to include in the teaching session. Page Ref: 349 Cognitive Level: Analyzing Client Need &Sub: Physiological Integrity Standards: QSEN Competencies: Patient-centered care | AACN Essential Competencies: Essential VII: Clinical prevention and population health | NLN Competencies: Nursing judgement | Nursing/Integrated Concepts: Nursing Process: Implementation/Health teaching and health promotion Learning Outcome: LO 16.7 Create a parent education session that focuses on the care of infants and children with a fever. MNL LO: Immunity, Infection, and Communicable Disease/Differentiate treatment options and strategies for continuity of care for the child and family. 3) The nurse prepares the second diphtheria, tetanus toxoid, and acellular pertussis (DTaP) and second inactivated polio vaccine (IPV) immunization injections for an infant who is 4 months old. The nurse may also give which of immunizations during the same well-child-care appointment? 1. Var (varicella) 2. TIV (influenza) 3. MMR (measles, mumps, rubella) 4. Haemophilus influenza type B (HIB) Answer: 4 Explanation: 1. Haemophilus influenza type B (HIB) vaccine is given at 2, 4, 6, and 12 to 15 months of age (four doses). None of the other vaccines can be given to a 4-month-old infant. Influenza (TIV) vaccine may be given yearly to infants between 6 months and 3 years of age. Measles, mumps, and rubella (MMR) vaccine is given at 12 to 15 months and 4 to 6 years of age (two doses). Varicella (Var) is given at 12 to 18 months or any time up to 12 years for one dose; for 13 years and older two doses are given, 4 to 8 weeks apart. 2. Haemophilus influenza type B (HIB) vaccine is given at 2, 4, 6, and 12 to 15 months of age (four doses). None of the other vaccines can be given to a 4-month-old infant. Influenza (TIV) vaccine may be given yearly to infants between 6 months and 3 years of age. Measles, mumps, and rubella (MMR) vaccine is given at 12 to 15 months and 4 to 6 years of age (two doses). Varicella (Var) is given at 12 to 18 months or any time up to 12 years for one dose; for 13 years and older two doses are given, 4 to 8 weeks apart. 3. Haemophilus influenza type B (HIB) vaccine is given at 2, 4, 6, and 12 to 15 months of age (four doses). None of the other vaccines can be given to a 4-month-old infant. Influenza (TIV) vaccine may be given yearly to infants between 6 months and 3 years of age. Measles, mumps, and rubella (MMR) vaccine is given at 12 to 15 months and 4 to 6 years of age (two doses). Varicella (Var) is given at 12 to 18 months or any time up to 12 years for one dose; for 13 years and older two doses are given, 4 to 8 weeks apart. 4. Haemophilus influenza type B (HIB) vaccine is given at 2, 4, 6, and 12 to 15 months of age (four doses). None of the other vaccines can be given to a 4-month-old infant. Influenza (TIV) vaccine may be given yearly to infants between 6 months and 3 years of age. Measles, mumps, and rubella (MMR) vaccine is given at 12 to 15 months and 4 to 6 years of age (two doses). Varicella (Var) is given at 12 to 18 months or any time up to 12 years for one dose; for 13 years and older two doses are given, 4 to 8 weeks apart. Page Ref: 346 Cognitive Level: Applying Client Need &Sub: Health Promotion and Maintenance Standards: QSEN Competencies: Patient-centered care | AACN Essential Competencies: Essential IX: Baccalaureate generalist nursing practice | NLN Competencies: Nursing judgement | Nursing/Integrated Concepts: Nursing Process: Implementation/Coordination of care Learning Outcome: LO 16.5 Plan the nursing care for children of all ages needing immunizations. MNL LO: Immunity, Infection, and Communicable Disease/Consider the characteristics and nursing management of immunizations during client teaching. 4) A mother refuses to have her child be immunized with measles, mumps, and rubella (MMR) vaccine because she believes that letting her infant get these diseases will help him fight off other diseases later in life. Which response by the nurse is most appropriate? 1. Honor her request because she is the parent. 2. Explain that antibodies can fight many diseases. 3. Tell her that not immunizing her infant may protect pregnant women. 4. Explain that if her child contracts measles, mumps, or rubella, there could be very serious and permanent complications from these diseases. Answer: 4 Explanation: 1. Explaining that if her child contracts measles, mumps, or rubella, he could have very serious and permanent complications from these diseases is correct because measles, mumps, and rubella all have potentially serious sequelae, such as encephalitis, brain damage, and deafness. Honoring her request is not correct because the nurse has a professional duty to explain that the mother's belief about immunizations is erroneous and may result in harm to her infant. Explaining that antibodies can fight many diseases is not correct because the body makes antibodies that are specific to antigens of each disease. Antibodies for one disease cannot fight another disease. Telling her that not immunizing her infant may protect pregnant women is not correct because immunizing the infant with MMR vaccine will help protect pregnant women from contracting rubella by decreasing the transmission. If a pregnant woman contracts rubella, her fetus can be severely damaged with congenital rubella syndrome. 2. Explaining that if her child contracts measles, mumps, or rubella, he could have very serious and permanent complications from these diseases is correct because measles, mumps, and rubella all have potentially serious sequelae, such as encephalitis, brain damage, and deafness. Honoring her request is not correct because the nurse has a professional duty to explain that the mother's belief about immunizations is erroneous and may result in harm to her infant. Explaining that antibodies can fight many diseases is not correct because the body makes antibodies that are specific to antigens of each disease. Antibodies for one disease cannot fight another disease. Telling her that not immunizing her infant may protect pregnant women is not correct because immunizing the infant with MMR vaccine will help protect pregnant women from contracting rubella by decreasing the transmission. If a pregnant woman contracts rubella, her fetus can be severely damaged with congenital rubella syndrome. 3. Explaining that if her child contracts measles, mumps, or rubella, he could have very serious and permanent complications from these diseases is correct because measles, mumps, and rubella all have potentially serious sequelae, such as encephalitis, brain damage, and deafness. Honoring her request is not correct because the nurse has a professional duty to explain that the mother's belief about immunizations is erroneous and may result in harm to her infant. Explaining that antibodies can fight many diseases is not correct because the body makes antibodies that are specific to antigens of each disease. Antibodies for one disease cannot fight another disease. Telling her that not immunizing her infant may protect pregnant women is not correct because immunizing the infant with MMR vaccine will help protect pregnant women from contracting rubella by decreasing the transmission. If a pregnant woman contracts rubella, her fetus can be severely damaged with congenital rubella syndrome. 4. Explaining that if her child contracts measles, mumps, or rubella, he could have very serious and permanent complications from these diseases is correct because measles, mumps, and rubella all have potentially serious sequelae, such as encephalitis, brain damage, and deafness. Honoring her request is not correct because the nurse has a professional duty to explain that the mother's belief about immunizations is erroneous and may result in harm to her infant. Explaining that antibodies can fight many diseases is not correct because the body makes antibodies that are specific to antigens of each disease. Antibodies for one disease cannot fight another disease. Telling her that not immunizing her infant may protect pregnant women is not correct because immunizing the infant with MMR vaccine will help protect pregnant women from contracting rubella by decreasing the transmission. If a pregnant woman contracts rubella, her fetus can be severely damaged with congenital rubella syndrome. Page Ref: 341 Cognitive Level: Applying Client Need &Sub: Health Promotion and Maintenance Standards: QSEN Competencies: Patient-centered care | AACN Essential Competencies: Essential IX: Baccalaureate generalist nursing practice | NLN Competencies: Nursing judgement | Nursing/Integrated Concepts: Nursing Process: Implementation Learning Outcome: LO 16.3 Examine the role that vaccines play in reducing and eliminating communicable diseases. MNL LO: Immunity, Infection, and Communicable Disease/Consider the characteristics and nursing management of immunizations during client teaching. 5) A parent reports that her school-age child, who has had all recommended immunizations, had a mild fever one week ago and now has bright red cheeks and a lacy red maculopapular rash on the trunk and arms. Which disease process does the nurse suspect based on the parent's description? 1. Chicken pox (varicella) 2. German measles (rubella) 3. Roseola (exanthem subitum) 4. Fifth disease (erythema infectiosum) Answer: 4 Explanation: 1. Fifth disease manifests first with a flulike illness, followed by a red "slapped- cheek" sign. Then a lacy maculopapular erythematous rash spreads symmetrically from the trunk to the extremities, sparing the soles and palms. Varicella (chicken pox) and rubella (German measles) are unlikely if the child has had all recommended immunizations. The rash of varicella progresses from papules to vesicles to pustules. The rash of rubella is a pink maculopapular rash that begins on the face and progresses downward to the trunk and extremities. Roseola typically occurs in infants and begins abruptly with a high fever followed by a pale pink rash starting on the trunk and spreading to the face, neck, and extremities. 2. Fifth disease manifests first with a flulike illness, followed by a red "slapped-cheek" sign. Then a lacy maculopapular erythematous rash spreads symmetrically from the trunk to the extremities, sparing the soles and palms. Varicella (chicken pox) and rubella (German measles) are unlikely if the child has had all recommended immunizations. The rash of varicella progresses from papules to vesicles to pustules. The rash of rubella is a pink maculopapular rash that begins on the face and progresses downward to the trunk and extremities. Roseola typically occurs in infants and begins abruptly with a high fever followed by a pale pink rash starting on the trunk and spreading to the face, neck, and extremities. 3. Fifth disease manifests first with a flulike illness, followed by a red "slapped-cheek" sign. Then a lacy maculopapular erythematous rash spreads symmetrically from the trunk to the extremities, sparing the soles and palms. Varicella (chicken pox) and rubella (German measles) are unlikely if the child has had all recommended immunizations. The rash of varicella progresses from papules to vesicles to pustules. The rash of rubella is a pink maculopapular rash that begins on the face and progresses downward to the trunk and extremities. Roseola typically occurs in infants and begins abruptly with a high fever followed by a pale pink rash starting on the trunk and spreading to the face, neck, and extremities. 4. Fifth disease manifests first with a flulike illness, followed by a red "slapped-cheek" sign. Then a lacy maculopapular erythematous rash spreads symmetrically from the trunk to the extremities, sparing the soles and palms. Varicella (chicken pox) and rubella (German measles) are unlikely if the child has had all recommended immunizations. The rash of varicella progresses from papules to vesicles to pustules. The rash of rubella is a pink maculopapular rash that begins on the face and progresses downward to the trunk and extremities. Roseola typically occurs in infants and begins abruptly with a high fever followed by a pale pink rash starting on the trunk and spreading to the face, neck, and extremities. Page Ref: 352 Cognitive Level: Analyzing Client Need &Sub: Physiological Integrity Standards: QSEN Competencies: Patient-centered care | AACN Essential Competencies: Essential IX: Baccalaureate generalist nursing practice | NLN Competencies: Nursing judgement | Nursing/Integrated Concepts: Nursing Process: Assessment/Coordination of care Learning Outcome: LO 16.8 Recognize common infectious and communicable diseases. MNL LO: Immunity, Infection, and Communicable Disease/Examine etiology, risk factors, pathophysiology, and clinical manifestations as seen in children. 6) The nurse prepares a DTaP (diphtheria, tetanus toxoid, and acellular pertussis) immunization for a 6-month-old infant. To administer this injection safely, the nurse chooses which needle, size and length, injection type, and injection site? 1. 25-gauge, 5/8-inch needle; IM (intramuscular); anterolateral thigh. 2. 22-gauge, 1/2-inch needle; IM (intramuscular); ventrogluteal. 3. 25-gauge, 5/8-inch needle; ID (intradermal); deltoid. 4. 25-gauge, 3/4-inch needle; SQ (subcutaneous); anterolateral thigh. Answer: 1 Explanation: 1. The dose of DTaP is 0.5 cc or 0.5 mL, to be given with a 22 to 25-gauge, 5/8- to 3/4-inch needle; IM (intramuscularly). The only safe intramuscular injection site for a 6-month- old infant is the anterolateral thigh. 2. The dose of DTaP is 0.5 cc or 0.5 mL, to be given with a 22 to 25-gauge, 5/8- to 3/4-inch needle; IM (intramuscularly). The only safe intramuscular injection site for a 6-month-old infant is the anterolateral thigh. 3. The dose of DTaP is 0.5 cc or 0.5 mL, to be given with a 22 to 25-gauge, 5/8- to 3/4-inch needle; IM (intramuscularly). The only safe intramuscular injection site for a 6-month-old infant is the anterolateral thigh. 4. The dose of DTaP is 0.5 cc or 0.5 mL, to be given with a 22 to 25-gauge, 5/8- to 3/4-inch needle; IM (intramuscularly). The only safe intramuscular injection site for a 6-month-old infant is the anterolateral thigh. Page Ref: 348 Cognitive Level: Analyzing Client Need &Sub: Physiological Integrity Standards: QSEN Competencies: Patient-centered care | AACN Essential Competencies: Essential IX: Baccalaureate generalist nursing practice | NLN Competencies: Nursing judgement | Nursing/Integrated Concepts: Nursing Process: Planning/Coordination of care Learning Outcome: LO 16.5 Plan the nursing care for children of all ages needing immunizations. MNL LO: Immunity, Infection, and Communicable Disease/Differentiate treatment options and strategies for continuity of care for the child and family. 7) Reducing the number of preventable childhood illnesses is a major national goal in Healthy People 2020. What will the school nurse teach families regarding immunizations in order to reach this goal? 1. A minor illness with a low-grade fever is a contraindication to receiving an immunization according to Healthy People 2020. 2. Vaccines should be given one at a time for optimum active immunity in the prevention of illness and disease. 3. Premature infants and low-birth-weight infants should receive half doses of vaccines for protection from communicable diseases. 4. It is important to maintain vaccination coverage for recommended vaccines in early childhood and to maintain them through kindergarten. Answer: 4 Explanation: 1. The benefits of vaccines far outweigh the risks from communicable diseases and resulting complications. A minor illness is not a contraindication to immunization. Giving vaccines one at a time will result in many missed opportunities. Half doses of vaccines should not be given routinely to premature and low-birth-weight infants. 2. The benefits of vaccines far outweigh the risks from communicable diseases and resulting complications. A minor illness is not a contraindication to immunization. Giving vaccines one at a time will result in many missed opportunities. Half doses of vaccines should not be given routinely to premature and low-birth-weight infants. 3. The benefits of vaccines far outweigh the risks from communicable diseases and resulting complications. A minor illness is not a contraindication to immunization. Giving vaccines one at a time will result in many missed opportunities. Half doses of vaccines should not be given routinely to premature and low-birth-weight infants. 4. The benefits of vaccines far outweigh the risks from communicable diseases and resulting complications. A minor illness is not a contraindication to immunization. Giving vaccines one at a time will result in many missed opportunities. Half doses of vaccines should not be given routinely to premature and low-birth-weight infants. Page Ref: 341 Cognitive Level: Applying Client Need &Sub: Health Promotion and Maintenance Standards: QSEN Competencies: Patient-centered care | AACN Essential Competencies: Essential VII: Clinical prevention and population health | NLN Competencies: Nursing judgement | Nursing/Integrated Concepts: Nursing Process: Implementation/Health teaching and health promotion Learning Outcome: LO 16.3 Examine the role that vaccines play in reducing and eliminating communicable diseases. MNL LO: Immunity, Infection, and Communicable Disease/Examine etiology, risk factors, pathophysiology, and clinical manifestations as seen in children. 8) A toddler client with a fever is prescribed amoxicillin clavulanate 250 mg/5 cc three times daily by mouth × 10 days for otitis media. Which teaching point will guard against antibiotic resistance to the disease process? 1. Administer a loading dose for the first dose. 2. Measure the prescribed dose in a household teaspoon. 3. Give the antibiotic for the full 10 days. 4. Stop the antibiotic if the child is afebrile. Answer: 3 Explanation: 1. Antibiotics must be administered for the full number of days ordered to prevent mutation of resistant strains of bacteria. A loading dose was not ordered. A household teaspoon may contain less than 5 cc, and the full dose must be given. Stopping the antibiotic before the prescribed time will permit remaining bacteria to reproduce, and the otitis media will return, possibly with antibiotic-resistant organisms. The absence of a fever is not an indication that all bacteria are killed or not reproducing. 2. Antibiotics must be administered for the full number of days ordered to prevent mutation of resistant strains of bacteria. A loading dose was not ordered. A household teaspoon may contain less than 5 cc, and the full dose must be given. Stopping the antibiotic before the prescribed time will permit remaining bacteria to reproduce, and the otitis media will return, possibly with antibiotic-resistant organisms. The absence of a fever is not an indication that all bacteria are killed or not reproducing. 3. Antibiotics must be administered for the full number of days ordered to prevent mutation of resistant strains of bacteria. A loading dose was not ordered. A household teaspoon may contain less than 5 cc, and the full dose must be given. Stopping the antibiotic before the prescribed time will permit remaining bacteria to reproduce, and the otitis media will return, possibly with antibiotic-resistant organisms. The absence of a fever is not an indication that all bacteria are killed or not reproducing. 4. Antibiotics must be administered for the full number of days ordered to prevent mutation of resistant strains of bacteria. A loading dose was not ordered. A household teaspoon may contain less than 5 cc, and the full dose must be given. Stopping the antibiotic before the prescribed time will permit remaining bacteria to reproduce, and the otitis media will return, possibly with antibiotic-resistant organisms. The absence of a fever is not an indication that all bacteria are killed or not reproducing. Page Ref: 362-364 Cognitive Level: Applying Client Need &Sub: Physiological Integrity Standards: QSEN Competencies: Patient-centered care | AACN Essential Competencies: Essential VII: Clinical prevention and population health | NLN Competencies: Nursing judgement | Nursing/Integrated Concepts: Nursing Process: Implementation/Health teaching and health promotion Learning Outcome: LO 16.2 Propose strategies to control the spread of infection in healthcare and community settings and the home. MNL LO: Demonstrate safe medication administration for the pediatric client. 9) The hospital has just provided its nurses with information about biologic threats and terrorism. After completing the course, a group of nurses is discussing its responsibility in relation to bioterrorism. Which statement by the nurse indicates a correct understanding of the concepts presented? 1. "It is important to separate clients according to age and illness to prevent the spread of disease." 2. "It is important to dispose blood-contaminated needles in the lead-lined container." 3. "I will notify the Centers for Disease Control (CDC) if a large number of persons with the same life-threatening infection present to the emergency room." 4. "I will initiate isolation precautions for a hospitalized client with methicillin-resistant staphylococcus aureus (MRSA)." Answer: 3 Explanation: 1. The CDC must be contacted to investigate the source of serious infections and to determine if a bioterrorist threat exists. Separating clients according to age and illness to prevent the spread of disease will do nothing to stop terrorism. Proper disposal of blood-contaminated needles in the sharps container and initiating isolation precautions for a hospitalized client with methicillin-resistant staphylococcus aureus (MRSA) are appropriate nursing actions but do not relate to bioterrorism. 2. The CDC must be contacted to investigate the source of serious infections and to determine if a bioterrorist threat exists. Separating clients according to age and illness to prevent the spread of disease will do nothing to stop terrorism. Proper disposal of blood-contaminated needles in the sharps container and initiating isolation precautions for a hospitalized client with methicillin- resistant staphylococcus aureus (MRSA) are appropriate nursing actions but do not relate to bioterrorism. 3. The CDC must be contacted to investigate the source of serious infections and to determine if a bioterrorist threat exists. Separating clients according to age and illness to prevent the spread of disease will do nothing to stop terrorism. Proper disposal of blood-contaminated needles in the sharps container and initiating isolation precautions for a hospitalized client with methicillin- resistant staphylococcus aureus (MRSA) are appropriate nursing actions but do not relate to bioterrorism. 4. The CDC must be contacted to investigate the source of serious infections and to determine if a bioterrorist threat exists. Separating clients according to age and illness to prevent the spread of disease will do nothing to stop terrorism. Proper disposal of blood-contaminated needles in the sharps container and initiating isolation precautions for a hospitalized client with methicillin- resistant staphylococcus aureus (MRSA) are appropriate nursing actions but do not relate to bioterrorism. Page Ref: 365 Cognitive Level: Applying Client Need &Sub: Health Promotion and Maintenance Standards: QSEN Competencies: Patient-centered care | AACN Essential Competencies: Essential VII: Clinical prevention and population health | NLN Competencies: Nursing judgement | Nursing/Integrated Concepts: Nursing Process: Implementation/Health teaching and health promotion Learning Outcome: LO 16.9 Develop a nursing care plan for the child with a common communicable disease. MNL LO: Immunity, Infection, and Communicable Disease/Differentiate treatment options and strategies for continuity of care for the child and family. 10) The school nurse is trying to prevent the spread of a flu virus through the school. Which infection-control strategies can be employed to prevent the spread of the flu virus? Select all that apply. 1. Teaching parents safe food preparation and storage 2. Withholding immunizations for children with compromised immune systems 3. Sanitizing toys, telephones, and door knobs to kill pathogens 4. Separating children with infections from children who are well 5. Teaching children to wash their hands after using the bathroom Answer: 3, 4, 5 Explanation: 1. To prevent the spread of communicable diseases, microorganisms must be killed or their growth controlled. Sanitizing toys and all contact surfaces, separating children with infections, and teaching children to wash their hands all control the growth and spread of microorganisms. Teaching parents safe food preparation and storage is another tool to prevent the spread of microorganisms but is not related to the flu virus. Immunizations should not be withheld from immunocompromised children; this is not an infection-control strategy. 2. To prevent the spread of communicable diseases, microorganisms must be killed or their growth controlled. Sanitizing toys and all contact surfaces, separating children with infections, and teaching children to wash their hands all control the growth and spread of microorganisms. Teaching parents safe food preparation and storage is another tool to prevent the spread of microorganisms but is not related to the flu virus. Immunizations should not be withheld from immunocompromised children; this is not an infection-control strategy. 3. To prevent the spread of communicable diseases, microorganisms must be killed or their growth controlled. Sanitizing toys and all contact surfaces, separating children with infections, and teaching children to wash their hands all control the growth and spread of microorganisms. Teaching parents safe food preparation and storage is another tool to prevent the spread of microorganisms but is not related to the flu virus. Immunizations should not be withheld from immunocompromised children; this is not an infection-control strategy. 4. To prevent the spread of communicable diseases, microorganisms must be killed or their growth controlled. Sanitizing toys and all contact surfaces, separating children with infections, and teaching children to wash their hands all control the growth and spread of microorganisms. Teaching parents safe food preparation and storage is another tool to prevent the spread of microorganisms but is not related to the flu virus. Immunizations should not be withheld from immunocompromised children; this is not an infection-control strategy. 5. To prevent the spread of communicable diseases, microorganisms must be killed or their growth controlled. Sanitizing toys and all contact surfaces, separating children with infections, and teaching children to wash their hands all control the growth and spread of microorganisms. Teaching parents safe food preparation and storage is another tool to prevent the spread of microorganisms but is not related to the flu virus. Immunizations should not be withheld from immunocompromised children; this is not an infection-control strategy. Page Ref: 363 Cognitive Level: Applying Client Need &Sub: Safe and Effective Care Environment Standards: QSEN Competencies: Patient-centered care | AACN Essential Competencies: Essential IX: Baccalaureate generalist nursing practice | NLN Competencies: Nursing judgement | Nursing/Integrated Concepts: Nursing Process: Implementation/Communication Learning Outcome: LO 16.2 Propose strategies to control the spread of infection in healthcare and community settings and the home. MNL LO: Immunity, Infection, and Communicable Disease/Differentiate treatment options and strategies for continuity of care for the child and family. 11) The nurse is teaching a prenatal class about infant care. Under which circumstances should the nurse emphasize that parents should call their healthcare provider immediately? Select all that apply. 1. Child 4 months old, received a DTaP immunization yesterday, and has a temperature of 38.0°C (100.4°F) 2. Child under 3 months old and has a temperature over 40.1°C (104.2°F) 3. Child difficult to awaken and has a pulsing fontanel 4. Child has purple spots on the skin and is lethargic. 5. Child has a stiff neck and has been irritable for three days. Answer: 2, 3, 4, 5 Explanation: 1. Infants under 3 months of age have limited ability to develop antibodies to fight infection, and a fever as high as 40.1°C indicates a serious infection. Difficulty to awaken and a pulsing fontanel, purple spots on the skin and lethargy, a stiff neck and irritability for 3 days in infants and children of any age may indicate meningitis. A mild fever of 38.0°C (100.4°F) in the 4-month-old who received a DTaP immunization yesterday is incorrect because the mild fever is expected as the body develops antibodies in response to antigens in the immunization. 2. Infants under 3 months of age have limited ability to develop antibodies to fight infection, and a fever as high as 40.1°C indicates a serious infection. Difficulty to awaken and a pulsing fontanel, purple spots on the skin and lethargy, a stiff neck and irritability for 3 days in infants and children of any age may indicate meningitis. A mild fever of 38.0°C (100.4°F) in the 4- month-old who received a DTaP immunization yesterday is incorrect because the mild fever is expected as the body develops antibodies in response to antigens in the immunization. 3. Infants under 3 months of age have limited ability to develop antibodies to fight infection, and a fever as high as 40.1°C indicates a serious infection. Difficulty to awaken and a pulsing fontanel, purple spots on the skin and lethargy, a stiff neck and irritability for 3 days in infants and children of any age may indicate meningitis. A mild fever of 38.0°C (100.4°F) in the 4- month-old who received a DTaP immunization yesterday is incorrect because the mild fever is expected as the body develops antibodies in response to antigens in the immunization. 4. Infants under 3 months of age have limited ability to develop antibodies to fight infection, and a fever as high as 40.1°C indicates a serious infection. Difficulty to awaken and a pulsing fontanel, purple spots on the skin and lethargy, a stiff neck and irritability for 3 days in infants and children of any age may indicate meningitis. A mild fever of 38.0°C (100.4°F) in the 4- month-old who received a DTaP immunization yesterday is incorrect because the mild fever is expected as the body develops antibodies in response to antigens in the immunization. 5. Infants under 3 months of age have limited ability to develop antibodies to fight infection, and a fever as high as 40.1°C indicates a serious infection. Difficulty to awaken and a pulsing fontanel, purple spots on the skin and lethargy, a stiff neck and irritability for 3 days in infants and children of any age may indicate meningitis. A mild fever of 38.0°C (100.4°F) in the 4- month-old who received a DTaP immunization yesterday is incorrect because the mild fever is expected as the body develops antibodies in response to antigens in the immunization. Page Ref: 362 Cognitive Level: Applying Client Need &Sub: Health Promotion and Maintenance Standards: #QSEN Competencies: Patient-centered care | AACN Essential Competencies: Nursing judgement | NLN Competencies: Nursing judgement | Nursing/Integrated Concepts: Nursing Process: Implementation/Health teaching and health promotion Learning Outcome: LO 16.8 Recognize common infectious and communicable diseases. MNL LO: Immunity, Infection, and Communicable Disease/Examine etiology, risk factors, pathophysiology, and clinical manifestations as seen in children. 12) The hospital admitting nurse is taking a history of a child's illness from the parents. The nurse concludes that the parents treated their 6-year-old child appropriately for a fever related to otitis media. Which action by the parents brought the nurse to this conclusion? 1. Used aspirin every four hours to reduce the fever 2. Alternated acetaminophen with ibuprofen every two hours 3. Put the child in a tub of cold water to reduce the fever 4. Offered generous amounts of fluids frequently Answer: 4 Explanation: 1. The body's need for fluids increases during a febrile illness. Aspirin has been associated with Reye syndrome and should not be given to children with a febrile illness. Alternating acetaminophen with ibuprofen every two hours may result in an overdose. Pediatric medication doses are more accurately calculated using the child's weight, not age. Putting the child in a tub of cold water will chill the child and cause shivering, a response that will increase body temperature. 2. The body's need for fluids increases during a febrile illness. Aspirin has been associated with Reye syndrome and should not be given to children with a febrile illness. Alternating acetaminophen with ibuprofen every two hours may result in an overdose. Pediatric medication doses are more accurately calculated using the child's weight, not age. Putting the child in a tub of cold water will chill the child and cause shivering, a response that will increase body temperature. 3. The body's need for fluids increases during a febrile illness. Aspirin has been associated with Reye syndrome and should not be given to children with a febrile illness. Alternating acetaminophen with ibuprofen every two hours may result in an overdose. Pediatric medication doses are more accurately calculated using the child's weight, not age. Putting the child in a tub of cold water will chill the child and cause shivering, a response that will increase body temperature. 4. The body's need for fluids increases during a febrile illness. Aspirin has been associated with Reye syndrome and should not be given to children with a febrile illness. Alternating acetaminophen with ibuprofen every two hours may result in an overdose. Pediatric medication doses are more accurately calculated using the child's weight, not age. Putting the child in a tub of cold water will chill the child and cause shivering, a response that will increase body temperature. Page Ref: 364 Cognitive Level: Applying Client Need &Sub: Physiological Integrity Standards: QSEN Competencies: Patient-centered care | AACN Essential Competencies: Essential IX: Baccalaureate generalist nursing practice | NLN Competencies: Nursing judgement | Nursing/Integrated Concepts: Nursing Process: Evaluation/Coordination of care Learning Outcome: LO 16.8 Recognize common infectious and communicable diseases. MNL LO: Immunity, Infection, and Communicable Disease/Apply the nursing process in providing care for the child and family. 13) A mother brings her 4-month-old infant in for a routine checkup and vaccinations. The mother reports that the infant was exposed to a brother who has the flu. Which action by the nurse is most appropriate based on these assessment findings? 1. Withhold the vaccinations. 2. Give the vaccinations as scheduled. 3. Withhold the DTaP vaccination but give the others as scheduled. 4. Give the infant the flu vaccination but withhold the others. Answer: 2 Explanation: 1. Recent exposure to an infectious disease is not a reason to defer a vaccine. There is no reason to withhold any of the vaccinations due at this time. The flu vaccination would not routinely be given to a 4-month-old. 2. Recent exposure to an infectious disease is not a reason to defer a vaccine. There is no reason to withhold any of the vaccinations due at this time. The flu vaccination would not routinely be given to a 4-month-old. 3. Recent exposure to an infectious disease is not a reason to defer a vaccine. There is no reason to withhold any of the vaccinations due at this time. The flu vaccination would not routinely be given to a 4-month-old. 4. Recent exposure to an infectious disease is not a reason to defer a vaccine. There is no reason to withhold any of the vaccinations due at this time. The flu vaccination would not routinely be given to a 4-month-old. Page Ref: 349 Cognitive Level: Applying Client Need &Sub: Health Promotion and Maintenance Standards: QSEN Competencies: Patient-centered care | AACN Essential Competencies: Essential IX: Baccalaureate generalist nursing practice | NLN Competencies: Nursing judgement | Nursing/Integrated Concepts: Nursing Process: Implementation Learning Outcome: LO 16.5 Plan the nursing care for children of all ages needing immunizations. MNL LO: Immunity, Infection, and Communicable Disease/Consider the characteristics and nursing management of immunizations during client teaching. 14) A parent brings her school-age child to the clinic because the child has a temperature of 100.2°F. The child remains active without other symptoms. Which statement by the nurse to the parents is most appropriate? 1. "Take the child's temperature every 2 hours and call the clinic if it reaches 102°F or above." 2. "Unless the fever bothers the child, it is best to let the natural body defenses respond to the infection." 3. "Keep the child warm, because shivering often occurs with fever." 4. "Alternate acetaminophen and ibuprofen to help keep the fever down and keep the child comfortable." Answer: 2 Explanation: 1. Fever is the body's response to an infection, and is not a disease. Allowing the body's natural defenses (fever) to fight the infection is best. The fever is treated if the child is uncomfortable from effects of the fever, such as body aches, headache, and so on. Taking the child's temperature more than every 4 to 6 hours is unnecessary. The child should be dressed for comfort. Light clothing is recommended. Alternating acetaminophen and ibuprofen is not recommended. 2. Fever is the body's response to an infection, and is not a disease. Allowing the body's natural defenses (fever) to fight the infection is best. The fever is treated if the child is uncomfortable from effects of the fever, such as body aches, headache, etc. Taking the child's temperature more than every 4 to 6 hours is unnecessary. The child should be dressed for comfort. Light clothing is recommended. Alternating acetaminophen and ibuprofen is not recommended. 3. Fever is the body's response to an infection, and is not a disease. Allowing the body's natural defenses (fever) to fight the infection is best. The fever is treated if the child is uncomfortable from effects of the fever, such as body aches, headache, and so on. Taking the child's temperature more than every 4 to 6 hours is unnecessary. The child should be dressed for comfort. Light clothing is recommended. Alternating acetaminophen and ibuprofen is not recommended. 4. Fever is the body's response to an infection, and is not a disease. Allowing the body's natural defenses (fever) to fight the infection is best. The fever is treated if the child is uncomfortable from effects of the fever, such as body aches, headache, and so on. Taking the child's temperature more than every 4 to 6 hours is unnecessary. The child should be dressed for comfort. Light clothing is recommended. Alternating acetaminophen and ibuprofen is not recommended. Page Ref: 349 Cognitive Level: Applying Client Need &Sub: Health Promotion and Maintenance Standards: QSEN Competencies: Patient-centered care | AACN Essential Competencies: Essential IX: Baccalaureate generalist nursing practice | NLN Competencies: Nursing judgement | Nursing/Integrated Concepts: Nursing Process: Implementation/Coordination of care Learning Outcome: LO 16.2 Propose strategies to control the spread of infection in healthcare and community settings and the home. MNL LO: Immunity, Infection, and Communicable Disease/Differentiate treatment options and strategies for continuity of care for the child and family. 15) A child is admitted with a diagnosis of early localized Lyme disease. Which clinical manifestations would the nurse expect to find on the initial assessment of this client? Select all that apply. 1. Erythema 5 to 15 cm in diameter 2. Hyperactivity 3. Cranial nerve palsies 4. Fever 5. Headache Answer: 1, 4, 5 Explanation: 1. Erythema, fever, and headache are signs/symptoms in the early localized stage of Lyme disease. Cranial nerve palsies are seen in the early disseminated stage of the disease. Malaise, rather than hyperactivity, is seen with this disease. 2. Erythema, fever, and headache are signs/symptoms in the early localized stage of Lyme disease. Cranial nerve palsies are seen in the early disseminated stage of the disease. Malaise, rather than hyperactivity, is seen with this disease. 3. Erythema, fever, and headache are signs/symptoms in the early localized stage of Lyme disease. Cranial nerve palsies are seen in the early disseminated stage of the disease. Malaise, rather than hyperactivity, is seen with this disease. 4. Erythema, fever, and headache are signs/symptoms in the early localized stage of Lyme disease. Cranial nerve palsies are seen in the early disseminated stage of the disease. Malaise, rather than hyperactivity, is seen with this disease. 5. Erythema, fever, and headache are signs/symptoms in the early localized stage of Lyme disease. Cranial nerve palsies are seen in the early disseminated stage of the disease. Malaise, rather than hyperactivity, is seen with this disease. Page Ref: 360 Cognitive Level: Applying Client Need &Sub: Health Promotion and Maintenance Standards: QSEN Competencies: Patient-centered care | AACN Essential Competencies: Essential IX: Baccalaureate generalist nursing practice | NLN Competencies: Nursing judgement | Nursing/Integrated Concepts: Nursing Process: Implementation/Coordination of care Learning Outcome: LO 16.8 Recognize common infectious and communicable diseases. MNL LO: Immunity, Infection, and Communicable Disease/Examine etiology, risk factors, pathophysiology, and clinical manifestations as seen in children. 16) A nurse working in a pediatric clinic is responsible for monitoring and maintaining the vaccinations on site. Which actions are appropriate for this nurse to implement? Select all that apply. 1. Fluctuate refrigerator and freezer temperatures each day. 2. Store vaccines in the center of the unit. 3. Check and record the temperature of the unit twice each day. 4. Have a plan for power outages. 5. Place bottles of water in each unit to help keep temperatures consistent. Answer: 2, 3, 4, 5 Explanation: 1. Appropriate interventions for the nurse to implement in order to maintain the potency of vaccines include storing the vaccines in the center of the unit, checking and recording the temperature of the storage unit twice a day, having a plan for power outages, and placing bottles of water in each unit to help keep temperatures consistent. The temperature of the refrigerator and freezer should be consistent and not fluctuate. 2. Appropriate interventions for the nurse to implement in order to maintain the potency of vaccines include storing the vaccines in the center of the unit, checking and recording the temperature of the storage unit twice a day, having a plan for power outages, and placing bottles of water in each unit to help keep temperatures consistent. The temperature of the refrigerator and freezer should be consistent and not fluctuate. 3. Appropriate interventions for the nurse to implement in order to maintain the potency of vaccines include storing the vaccines in the center of the unit, checking and recording the temperature of the storage unit twice a day, having a plan for power outages, and placing bottles of water in each unit to help keep temperatures consistent. The temperature of the refrigerator and freezer should be consistent and not fluctuate. 4. Appropriate interventions for the nurse to implement in order to maintain the potency of vaccines include storing the vaccines in the center of the unit, checking and recording the temperature of the storage unit twice a day, having a plan for power outages, and placing bottles of water in each unit to help keep temperatures consistent. The temperature of the refrigerator and freezer should be consistent and not fluctuate. 5. Appropriate interventions for the nurse to implement in order to maintain the potency of vaccines include storing the vaccines in the center of the unit, checking and recording the temperature of the storage unit twice a day, having a plan for power outages, and placing bottles of water in each unit to help keep temperatures consistent. The temperature of the refrigerator and freezer should be consistent and not fluctuate. Page Ref: 346 Cognitive Level: Applying Client Need &Sub: Safe and Effective Care Environment Standards: QSEN Competencies: Patient-centered care | AACN Essential Competencies: Essential IX: Baccalaureate generalist nursing practice | NLN Competencies: Nursing judgement | Nursing/Integrated Concepts: Nursing Process: Implementation/Coordination of care Learning Outcome: LO 16.6 Outline a plan to maintain the potency of vaccines. MNL LO: Immunity, Infection, and Communicable Disease/Consider the characteristics and nursing management of immunizations during client teaching. 17) The student nurse is learning a lesson about communicable diseases and how they are spread. On a quiz the next day the nurse uses the information learned in this lesson and demonstrates learning. For a communicable disease to occur what factors must be in place? Select all that apply. 1. Antibodies 2. Toxoid 3. Pathogen 4. Transmission 5. Host Answer: 3, 4, 5 Explanation: 1. For a communicable disease to occur, three factors need to be in place: an infectious agent or pathogen, means of transmission, and a host. This is not a factor needed for communicable disease to occur. 2. For a communicable disease to occur, three factors need to be in place: an infectious agent or pathogen, means of transmission, and a host. This is not a factor needed for communicable disease to occur. 3. For a communicable disease to occur, three factors need to be in place: an infectious agent or pathogen, means of transmission, and a host. This is a factor needed for communicable disease to occur. 4. For a communicable disease to occur, three factors need to be in place: an infectious agent or pathogen, means of transmission, and a host. This is a factor needed for communicable disease to occur. 5. For a communicable disease to occur, three factors need to be in place: an infectious agent or pathogen, means of transmission, and a host. This is a factor needed for communicable disease to occur. Page Ref: 335 Cognitive Level: Analyzing Client Need &Sub: Health Promotion and Maintenance Standards: QSEN Competencies: Patient-centered care | AACN Essential Competencies: Essential IX: Baccalaureate generalist nursing practice | NLN Competencies: Nursing judgement | Nursing/Integrated Concepts: Nursing Process: Evaluation/ Coordination of care Learning Outcome: LO 16.2 Propose strategies to control the spread of infection in healthcare and community settings and the home. MNL LO: Immunity, Infection, and Communicable Disease/Consider the characteristics and nursing management of immunizations during client teaching. 18) The family and school-age child are at the healthcare clinic for immunizations. The nurse takes the time to talk with the child and family about reducing the transmission of infection. What practices should the nurse suggest for the family? Select all that apply. 1. Do not share dishes, utensils, and cups. 2. Sanitize toys every week with Lysol. 3. Use alcohol-based hand sanitizer with the child after eating and toileting. 4. Cough or sneeze into cloth tissue 5. Dispose of diapers in a closed container. Answer: 1, 5 Explanation: 1. Teach families to reduce transmission of infection among family members with the following practices: use disposable tissues and dispose immediately after using, wash hands thoroughly with soap/water after all contact with diapers/tissues/mucous, sneeze/cough into elbow, wash hands with soap/water after eating and toileting, do not share dishes/utensils/cups, wash hands thoroughly before preparing food and again several times during the preparation process, use soapy warm water to wash dishes/cutting boards, wipe counters/surfaces that are used for diaper changes or that the child touches with disinfectant, make sure diaper changing area is well away from food prep areas, dispose of diapers in closed containers. This is a practice that the nurse should suggest for the family. 2. Teach families to reduce transmission of infection among family members with the following practices: use disposable tissues and dispose immediately after using, wash hands thoroughly with soap/water after all contact with diapers/tissues/mucous, sneeze/cough into elbow, wash hands with soap/water after eating and toileting, do not share dishes/utensils/cups, wash hands thoroughly before preparing food and again several times during the preparation process, use soapy warm water to wash dishes/cutting boards, wipe counters/surfaces that are used for diaper changes or that the child touches with disinfectant, make sure diaper changing area is well away from food prep areas, dispose of diapers in closed containers. This is not a practice that the nurse should suggest for the family. 3. Teach families to reduce transmission of infection among family members with the following practices: use disposable tissues and dispose immediately after using, wash hands thoroughly with soap/water after all contact with diapers/tissues/mucous, sneeze/cough into elbow, wash hands with soap/water after eating and toileting, do not share dishes/utensils/cups, wash hands thoroughly before preparing food and again several times during the preparation process, use soapy warm water to wash dishes/cutting boards, wipe counters/surfaces that are used for diaper changes or that the child touches with disinfectant, make sure diaper changing area is well away from food prep areas, dispose of diapers in closed containers. This is not a practice that the nurse should suggest for the family. 4. Teach families to reduce transmission of infection among family members with the following practices: use disposable tissues and dispose immediately after using, wash hands thoroughly with soap/water after all contact with diapers/tissues/mucous, sneeze/cough into elbow, wash hands with soap/water after eating and toileting, do not share dishes/utensils/cups, wash hands thoroughly before preparing food and again several times during the preparation process, use soapy warm water to wash dishes/cutting boards, wipe counters/surfaces that are used for diaper changes or that the child touches with disinfectant, make sure diaper changing area is well away from food prep areas, dispose of diapers in closed containers. This is not a practice that the nurse should suggest for the family. 5. Teach families to reduce transmission of infection among family members with the following practices: use disposable tissues and dispose immediately after using, wash hands thoroughly with soap/water after all contact with diapers/tissues/mucous, sneeze/cough into elbow, wash hands with soap/water after eating and toileting, do not share dishes/utensils/cups, wash hands thoroughly before preparing food and again several times during the preparation process, use soapy warm water to wash dishes/cutting boards, wipe counters/surfaces that are used for diaper changes or that the child touches with disinfectant, make sure diaper changing area is well away from food prep areas, dispose of diapers in closed containers. This is a practice that the nurse should suggest for the family. Page Ref: 336 Cognitive Level: Analyzing Client Need &Sub: Health Promotion and Maintenance Standards: QSEN Competencies: Patient-centered care | AACN Essential Competencies: Essential IX: Baccalaureate generalist nursing practice | NLN Competencies: Nursing judgement | Nursing/Integrated Concepts: Nursing Process: Planning/Coordination of care Learning Outcome: LO 16.2 Propose strategies to control the spread of infection in healthcare and community settings and the home. MNL LO: Immunity, Infection, and Communicable Disease/Consider the characteristics and nursing management of immunizations during client teaching. Principles of Pediatric Nursing: Caring for Children, 7e (Ball et al.) Chapter 17 Social and Environmental Influences on the Child 1) Concerned parents call the school nurse because of changes in their 15-year-old adolescent's behavior. Which behavior would the nurse indicate as indicative of adolescent substance abuse? 1. Buying baggy, oversized clothing at thrift shops and dying her hair black 2. Becoming very involved with friends and in activities related to the basketball team that she is on; never seeming to be home; and, when she is home, preferring to be in her room with the door shut 3. Receiving numerous detentions lately from teachers for sleeping in class 4. Becoming very moody, dramatically crying and weeping one minute and then being cheerful and excited the next Answer: 3 Explanation: 1. Mood swings, experimenting with clothes and hair, periodically distancing themselves from their parents, and preferring involvement with their peers are all normal adolescent behaviors. Even though most teens do prefer staying up late, they are not usually so tired that they would fall asleep during the day, especially while engaged in classroom activities. This behavior is abnormal and may indicate involvement with substance abuse or an underlying pathology. 2. Mood swings, experimenting with clothes and hair, periodically distancing themselves from their parents, and preferring involvement with their peers are all normal adolescent behaviors. Even though most teens do prefer staying up late, they are not usually so tired that they would fall asleep during the day, especially while engaged in classroom activities. This behavior is abnormal and may indicate involvement with substance abuse or an underlying pathology. 3. Mood swings, experimenting with clothes and hair, periodically distancing themselves from their parents, and preferring involvement with their peers are all normal adolescent behaviors. Even though most teens do prefer staying up late, they are not usually so tired that they would fall asleep during the day, especially while engaged in classroom activities. This behavior is abnormal and may indicate involvement with substance abuse or an underlying pathology. 4. Mood swings, experimenting with clothes and hair, periodically distancing themselves from their parents, and preferring involvement with their peers are all normal adolescent behaviors. Even though most teens do prefer staying up late, they are not usually so tired that they would fall asleep during the day, especially while engaged in classroom activities. This behavior is abnormal and may indicate involvement with substance abuse or an underlying pathology. Page Ref: 378 Cognitive Level: Analyzing Client Need &Sub: Psychosocial Integrity Standards: QSEN Competencies: Patient-centered care | AACN Essential Competencies: Essential II: Basic organizational and systems leadership for quality care and patient safety | NLN Competencies: Nursing judgement | Nursing/Integrated Concepts: Nursing Process: Assessment/Coordination of care Learning Outcome: LO 17.3 Examine the effects of substance use, physical activity, and other lifestyle patterns on health. MNL LO: Compare and contrast known family theories and assessment strategies. 2) During a well-child exam, the parents of a preschool-age child inform the nurse that they are thinking of buying a television for their child's bedroom and ask for advice as to whether this is appropriate. Which response by the nurse is the most appropriate? 1. "Research has shown that children with a television in their bedroom spend significantly less time playing outside than other children, and physical inactivity in children has been linked to many chronic diseases such as obesity and type 2 diabetes." 2. "Research has shown that watching educational television shows improves a child's performance in school." 3. "Don't buy a television for your child's room; he is much too young for that." 4. "It is okay for children to have a television in their room as long as you limit the amount of time they watch it to less than two hours per day." Answer: 1 Explanation: 1. Young children need to be physically active at this age. "Research has shown that children with a television in their bedroom spend significantly less time playing outside than other children, and physical inactivity in children has been linked to many chronic diseases such as obesity and type 2 diabetes" is the best response because it gives the parents an evidence- based rationale for not placing a television in the child's room. "Don't buy a television for your child's room; he is much too young for that" does not give parents a rationale and may seem opinionated to them. While there may be some truth in the comment "Research has shown that watching educational television shows improves a child's performance in school," this statement may encourage increased television watching by the child, and the child's developmental need for physical activity is greater than the benefit that he may obtain by watching educational programs. "It is okay for children to have a television in their room as long as you limit the amount of time they watch it to less than two hours per day" is correct in that limiting television viewing to less than two hours per day is appropriate, but the probability of this occurring with a television in the child's room is low, and the child will most likely be watching much more than two hours per day. 2. Young children need to be physically active at this age. "Research has shown that children with a television in their bedroom spend significantly less time playing outside than other children, and physical inactivity in children has been linked to many chronic diseases such as obesity and type 2 diabetes" is the best response because it gives the parents an evidence-based rationale for not placing a television in the child's room. "Don't buy a television for your child's room; he is much too young for that" does not give parents a rationale and may seem opinionated to them. While there may be some truth in the comment "Research has shown that watching educational television shows improves a child's performance in school," this statement may encourage increased television watching by the child, and the child's developmental need for physical activity is greater than the benefit that he may obtain by watching educational programs. "It is okay for children to have a television in their room as long as you limit the amount of time they watch it to less than two hours per day" is correct in that limiting television viewing to less than two hours per day is appropriate, but the probability of this occurring with a television in the child's room is low, and the child will most likely be watching much more than two hours per day. 3. Young children need to be physically active at this age. "Research has shown that children with a television in their bedroom spend significantly less time playing outside than other children, and physical inactivity in children has been linked to many chronic diseases such as obesity and type 2 diabetes" is the best response because it gives the parents an evidence-based rationale for not placing a television in the child's room. "Don't buy a television for your child's room; he is much too young for that" does not give parents a rationale and may seem opinionated to them. While there may be some truth in the comment "Research has shown that watching educational television shows improves a child's performance in school," this statement may encourage increased television watching by the child, and the child's developmental need for physical activity is greater than the benefit that he may obtain by watching educational programs. "It is okay for children to have a television in their room as long as you limit the amount of time they watch it to less than two hours per day" is correct in that limiting television viewing to less than two hours per day is appropriate, but the probability of this occurring with a television in the child's room is low, and the child will most likely be watching much more than two hours per day. 4. Young children need to be physically active at this age. "Research has shown that children with a television in their bedroom spend significantly less time playing outside than other children, and physical inactivity in children has been linked to many chronic diseases such as obesity and type 2 diabetes" is the best response because it gives the parents an evidence-based rationale for not placing a television in the child's room. "Don't buy a television for your child's room; he is much too young for that" does not give parents a rationale and may seem opinionated to them. While there may be some truth in the comment "Research has shown that watching educational television shows improves a child's performance in school," this statement may encourage increased television watching by the child, and the child's developmental need for physical activity is greater than the benefit that he may obtain by watching educational programs. "It is okay for children to have a television in their room as long as you limit the amount of time they watch it to less than two hours per day" is correct in that limiting television viewing to less than two hours per day is appropriate, but the probability of this occurring with a television in the child's room is low, and the child will most likely be watching much more than two hours per day. Page Ref: 381 Cognitive Level: Applying Client Need &Sub: Health Promotion and Maintenance Standards: QSEN Competencies: Patient-centered care | AACN Essential Competencies: Essential II: Basic organizational and systems leadership for quality care and patient safety | NLN Competencies: Nursing judgement | Nursing/Integrated Concepts: Nursing Process: Implementation/Coordination of care Learning Outcome: LO 17.1 Identify major social and environmental factors that influence the health of children and adolescents. MNL LO: Evaluate healthcare issues related to pediatric nursing care. 3) A high school student calls to ask the nurse for advice on how to care for a new navel piercing. Which response by the nurse is the most appropriate? 1. "Apply warm soaks to the area for the first two days to minimize swelling." 2. "Do not move or turn the jewelry for the first 3 days." 3. "Avoid contact with another person's bodily fluids until the area is well healed." 4. "Apply lotion to the area, rubbing gently, to prevent skin from becoming dry and irritated." Answer: 3 Explanation: 1. Until the piercing has healed, it is a nonintact area of skin that has potential for infection, especially from contact with bodily fluids from someone else. Ice, not warm soaks, should be applied to the area for the first two days to minimize the swelling. The jewelry needs to be gently rotated several times per day to aid with healing. Lotion can provide a medium for bacteria, and rubbing at the site can cause irritation to the area. 2. Until the piercing has healed, it is a nonintact area of skin that has potential for infection, especially from contact with bodily fluids from someone else. Ice, not warm soaks, should be applied to the area for the first two days to minimize the swelling. The jewelry needs to be gently rotated several times per day to aid with healing. Lotion can provide a medium for bacteria, and rubbing at the site can cause irritation to the area. 3. Until the piercing has healed, it is a nonintact area of skin that has potential for infection, especially from contact with bodily fluids from someone else. Ice, not warm soaks, should be applied to the area for the first two days to minimize the swelling. The jewelry needs to be gently rotated several times per day to aid with healing. Lotion can provide a medium for bacteria, and rubbing at the site can cause irritation to the area. 4. Until the piercing has healed, it is a nonintact area of skin that has potential for infection, especially from contact with bodily fluids from someone else. Ice, not warm soaks, should be applied to the area for the first two days to minimize the swelling. The jewelry needs to be gently rotated several times per day to aid with healing. Lotion can provide a medium for bacteria, and rubbing at the site can cause irritation to the area. Page Ref: 383-384 Cognitive Level: Applying Client Need &Sub: Health Promotion and Maintenance Standards: QSEN Competencies: Patient-centered care | AACN Essential Competencies: Essential II: Basic organizational and systems leadership for quality care and patient safety | NLN Competencies: Nursing judgement | Nursing/Integrated Concepts: Nursing Process: Implementation/Coordination of care Learning Outcome: LO 17.2 Apply the ecologic model and resiliency theory to assessment of the social and environmental factors in children's lives. MNL LO: Examine the role of the nurse in promoting culturally competent family-centered care. 4) While working at a weekend "free clinic," the nurse is assessing a toddler when the mother of the child confides that it has been very difficult providing for her family of four children on her limited budget. She is not sure that she has enough money to buy food for the rest of the month and the antibiotic that is needed for the child's ear infection. Which intervention would be the most beneficial for this family? 1. Giving the mother enough free samples of the antibiotic for the recommended course of treatment 2. Putting the mother in contact with a local agency that provides food on a regular basis to needy families and helps them access other resources in the community 3. Talking with the mother about the factors that increase a child's risk of acquiring an ear infection 4. Talking with the mother about the importance of a balanced diet in the growth and development of children and providing her with a list of inexpensive, nutritious foods Answer: 2 Explanation: 1. While most of these are good interventions, putting the mother in contact with a local agency is most likely to meet the family's basic need for food and possibly connect the mother to a resource that could supply her with the antibiotic for her child. Also, many uncomplicated ear infections can resolve without antibiotic treatment. Giving antibiotics will only benefit the child, and the question asks what will benefit the child and family. 2. While most of these are good interventions, putting the mother in contact with a local agency is most likely to meet the family's basic need for food and possibly connect the mother to a resource that could supply her with the antibiotic for her child. Also, many uncomplicated ear infections can resolve without antibiotic treatment. Giving antibiotics will only benefit the child, and the question asks what will benefit the child and family. 3. While most of these are good interventions, putting the mother in contact with a local agency is most likely to meet the family's basic need for food and possibly connect the mother to a resource that could supply her with the antibiotic for her child. Also, many uncomplicated ear infections can resolve without antibiotic treatment. Giving antibiotics will only benefit the child, and the question asks what will benefit the child and family. 4. While most of these are good interventions, putting the mother in contact with a local agency is most likely to meet the family's basic need for food and possibly connect the mother to a resource that could supply her with the antibiotic for her child. Also, many uncomplicated ear infections can resolve without antibiotic treatment. Giving antibiotics will only benefit the child, and the question asks what will benefit the child and family. Page Ref: 371 Cognitive Level: Analyzing Client Need &Sub: Health Promotion and Maintenance Standards: QSEN Competencies: Patient-centered care | AACN Essential Competencies: Essential IX: Baccalaureate generalist nursing practice | NLN Competencies: Nursing judgement | Nursing/Integrated Concepts: Nursing Process: Planning/Coordination of care Learning Outcome: LO 17.7 Plan nursing interventions for children related to social and environmental situations. MNL LO: Develop a family-centered nursing care plan for the child and family. 5) A school-age child has been seen in the pediatric clinic three times in the last two months for complaints of abdominal pain. Physical exam and all ordered lab work have been normal. Which question by the nurse would most likely help determine the etiology of the child's abdominal pain? 1. "Have there been any changes in your child's school or home life recently?" 2. "How many meals does your child eat each day?" 3. "Are your child's immunizations up to date?" 4. "Has your child had any fevers or viral illnesses in the last three months?" Answer: 1 Explanation: 1. With a normal exam and lab work there is a high probability that this child's abdominal pain is stress related, and it is most important to identify the possible stressors in this child's life to aid in diagnosis and treatment. The other questions are also important to ask but are not as relevant to this child's symptoms as "Have there been any changes in your child's school or home life recently?" 2. With a normal exam and lab work there is a high probability that this child's abdominal pain is stress related, and it is most important to identify the possible stressors in this child's life to aid in diagnosis and treatment. The other questions are also important to ask but are not as relevant to this child's symptoms as "Have there been any changes in your child's school or home life recently?" 3. With a normal exam and lab work there is a high probability that this child's abdominal pain is stress related, and it is most important to identify the possible stressors in this child's life to aid in diagnosis and treatment. The other questions are also important to ask but are not as relevant to this child's symptoms as "Have there been any changes in your child's school or home life recently?" 4. With a normal exam and lab work there is a high probability that this child's abdominal pain is stress related, and it is most important to identify the possible stressors in this child's life to aid in diagnosis and treatment. The other questions are also important to ask but are not as relevant to this child's symptoms as "Have there been any changes in your child's school or home life recently?" Page Ref: 371 Cognitive Level: Analyzing Client Need &Sub: Psychosocial Integrity Standards: QSEN Competencies: Patient-centered care | AACN Essential Competencies: Essential IX: Baccalaureate generalist nursing practice | NLN Competencies: Nursing judgement | Nursing/Integrated Concepts: Nursing Process: Planning/Coordination of care Learning Outcome: LO 17.1 Identify major social and environmental factors that influence the health of children and adolescents. MNL LO: Develop a family-centered nursing care plan for the child and family. 6) A recently divorced mother who must return to work is concerned about the effects of placing her child in day care full time. In counseling the mother, which factor does the nurse share as the most influential in determining whether or not day care has a positive or negative effect on the child? 1. The ratio of day-care workers to children 2. The closeness of the parent-child relationship 3. The amount of time that the children spend playing outside 4. The cleanliness of the facility Answer: 2 Explanation: 1. While the ratio of child-care workers to the children, the cleanliness of the facility, and how much time the children are able to spend playing outdoors all can contribute to whether or not child care is a positive or negative experience, the closeness of the parent-child relationship is more likely to impact how resilient the child is, and this has a greater impact on the effects of the child-care experience. 2. While the ratio of child-care workers to the children, the cleanliness of the facility, and how much time the children are able to spend playing outdoors all can contribute to whether or not child care is a positive or negative experience, the closeness of the parent-child relationship is more likely to impact how resilient the child is, and this has a greater impact on the effects of the child-care experience. 3. While the ratio of child-care workers to the children, the cleanliness of the facility, and how much time the children are able to spend playing outdoors all can contribute to whether or not child care is a positive or negative experience, the closeness of the parent-child relationship is more likely to impact how resilient the child is, and this has a greater impact on the effects of the child-care experience. 4. While the ratio of child-care workers to the children, the cleanliness of the facility, and how much time the children are able to spend playing outdoors all can contribute to whether or not child care is a positive or negative experience, the closeness of the parent-child relationship is more likely to impact how resilient the child is, and this has a greater impact on the effects of the child-care experience. Page Ref: 372 Cognitive Level: Analyzing Client Need &Sub: Psychosocial Integrity Standards: QSEN Competencies: Patient-centered care | AACN Essential Competencies: Essential IX: Baccalaureate generalist nursing practice | NLN Competencies: Nursing judgement | Nursing/Integrated Concepts: Nursing Process: Implementation/Coordination of care Learning Outcome: LO 17.1 Identify major social and environmental factors that influence the health of children and adolescents. MNL LO: Use the nursing process to provide developmentally appropriate care for the pediatric client. 7) When examining a toddler-age child during a well-child physical, which assessment is the priority? 1. Visual acuity 2. Helmet use 3. Risk of lead exposure 4. Whether household drinking water contains fluorine Answer: 3 Explanation: 1. Elevated lead levels are neurotoxic to young children and, if untreated, can cause irreparable neurological damage. Visual acuity may be difficult to accurately assess at this age secondary to the child's compliance and ability to understand the directions for the screening test. While teaching helmet use at an early age is important, it is unlikely that this child is riding a bicycle yet, and although early exposure to fluorine is important for good dental health, lack of fluorinated drinking water will not be as harmful to the child as toxic lead levels. 2. Elevated lead levels are neurotoxic to young children and, if untreated, can cause irreparable neurological damage. Visual acuity may be difficult to accurately assess at this age secondary to the child's compliance and ability to understand the directions for the screening test. While teaching helmet use at an early age is important, it is unlikely that this child is riding a bicycle yet, and although early exposure to fluorine is important for good dental health, lack of fluorinated drinking water will not be as harmful to the child as toxic lead levels. 3. Elevated lead levels are neurotoxic to young children and, if untreated, can cause irreparable neurological damage. Visual acuity may be difficult to accurately assess at this age secondary to the child's compliance and ability to understand the directions for the screening test. While teaching helmet use at an early age is important, it is unlikely that this child is riding a bicycle yet, and although early exposure to fluorine is important for good dental health, lack of fluorinated drinking water will not be as harmful to the child as toxic lead levels. 4. Elevated lead levels are neurotoxic to young children and, if untreated, can cause irreparable neurological damage. Visual acuity may be difficult to accurately assess at this age secondary to the child's compliance and ability to understand the directions for the screening test. While teaching helmet use at an early age is important, it is unlikely that this child is riding a bicycle yet, and although early exposure to fluorine is important for good dental health, lack of fluorinated drinking water will not be as harmful to the child as toxic lead levels. Page Ref: 400-401 Cognitive Level: Applying Client Need &Sub: Health Promotion and Maintenance Standards: QSEN Competencies: Patient-centered care | AACN Essential Competencies: Essential IX: Baccalaureate generalist nursing practice | NLN Competencies: Nursing judgement | Nursing/Integrated Concepts: Nursing Process: Assessment/Coordination of care Learning Outcome: LO 17.5 Evaluate the environment for hazards to children, such as exposure to harmful substances and potential for poisoning. MNL LO: Differentiate developmentally appropriate care environments for the pediatric client and family. 8) The nurse is conducting a health promotion class for adolescents. In counseling an adolescent about lifestyle choices, what should the adolescent eliminate in order to decrease the risk of the most preventable cause of adult death? 1. Alcohol use 2. Obesity 3. Tobacco use 4. Cocaine use Answer: 3 Explanation: 1. Although all of these factors are preventable causes of mortality in the United States, tobacco use accounts for 438,000 deaths annually and is the most preventable cause of adult death. 2. Although all of these factors are preventable causes of mortality in the United States, tobacco use accounts for 438,000 deaths annually and is the most preventable cause of adult death. 3. Although all of these factors are preventable causes of mortality in the United States, tobacco use accounts for 438,000 deaths annually and is the most preventable cause of adult death. 4. Although all of these factors are preventable causes of mortality in the United States, tobacco use accounts for 438,000 deaths annually and is the most preventable cause of adult death. Page Ref: 375-376 Cognitive Level: Applying Client Need &Sub: Health Promotion and Maintenance Standards: QSEN Competencies: Patient-centered care | AACN Essential Competencies: Essential VII: Clinical prevention and population health | NLN Competencies: Nursing judgement | Nursing/Integrated Concepts: Nursing Process: Implementation/Health teaching and health promotion Learning Outcome: LO 17.3 Examine the effects of substance use, physical activity, and other lifestyle patterns on health. MNL LO: Apply age-specific mortality and morbidity data to care of children. 9) The school nurse is planning a smoking-prevention program for middle school students. Which intervention is most likely to be effective in preventing middle school children from smoking? 1. Having a local high school basketball star come to talk to the students about the importance of not smoking 2. Having the school's biology teacher demonstrate the pathophysiology of the effects of smoking tobacco on the body 3. Developing colorful posters with catchy slogans and placing them all over the school 4. Having a pledge campaign with prizes awarded, during which students sign contracts saying that they will not use tobacco products Answer: 1 Explanation: 1. While all of the strategies are good, the most effective tip would be to have a local high school basketball star come to talk to the students about the importance of not smoking, because students at this age are more likely to listen to and attempt to emulate someone of their own peer group whom they respect and look up to. Information from adults, posters, and signed contracts are not as likely to influence children of this age more than the pressure of their peers. 2. While all of the strategies are good, the most effective tip would be to have a local high school basketball star come to talk to the students about the importance of not smoking, because students at this age are more likely to listen to and attempt to emulate someone of their own peer group whom they respect and look up to. Information from adults, posters, and signed contracts are not as likely to influence children of this age more than the pressure of their peers. 3. While all of the strategies are good, the most effective tip would be to have a local high school basketball star come to talk to the students about the importance of not smoking, because students at this age are more likely to listen to and attempt to emulate someone of their own peer group whom they respect and look up to. Information from adults, posters, and signed contracts are not as likely to influence children of this age more than the pressure of their peers. 4. While all of the strategies are good, the most effective tip would be to have a local high school basketball star come to talk to the students about the importance of not smoking, because students at this age are more likely to listen to and attempt to emulate someone of their own peer group whom they respect and look up to. Information from adults, posters, and signed contracts are not as likely to influence children of this age more than the pressure of their peers. Page Ref: 375—376 Cognitive Level: Analyzing Client Need &Sub: Health Promotion and Maintenance Standards: QSEN Competencies: Patient-centered care | AACN Essential Competencies: Essential IX: Baccalaureate generalist nursing practice | NLN Competencies: Nursing judgement | Nursing/Integrated Concepts: Nursing Process: Implementation/Coordination of care Learning Outcome: LO 17.7 Plan nursing interventions for children related to social and environmental situations. MNL LO: Differentiate developmentally appropriate care environments for the pediatric client and family. 10) While the nurse is conducting the history of a school-age child, the parents admit to owning firearms. Which safety measures are appropriate to include in the teaching plan for this family? Select all that apply. 1. Using a gun lock on all firearms in the house 2. Taking the child to a shooting range for lessons on how to use a gun properly 3. Storing the guns and ammunition in separate places 4. Keeping all the guns in a locked cabinet 5. Explaining the dangers of a gun to the child and telling her explicitly to never touch it Answer: 1, 3, 4 Explanation: 1. Over 4000 youth from 10 to 19 years old die from firearm homicides annually, and approximately 1500 additional youth die from firearm suicide. Firearm homicide is the second leading cause of injury death for youth, and firearm suicide is the fifth leading cause of injury death for youth (CDC, 2011c). The safety measures of using a gun lock, keeping the gun and ammunition separate, and putting the guns in a locked cabinet will at least make the guns less accessible. Telling a child that a gun is "dangerous" and not to be touched will probably make it more fascinating. Even with knowledge of the proper use of a firearm, a 10-year-old child's judgment may not be mature enough to prevent misuse of it. 2. Over 4000 youth from 10 to 19 years old die from firearm homicides annually, and approximately 1500 additional youth die from firearm suicide. Firearm homicide is the second leading cause of injury death for youth, and firearm suicide is the fifth leading cause of injury death for youth (CDC, 2011c). The safety measures of using a gun lock, keeping the gun and ammunition separate, and putting the guns in a locked cabinet will at least make the guns less accessible. Telling a child that a gun is "dangerous" and not to be touched will probably make it more fascinating. Even with knowledge of the proper use of a firearm, a 10-year-old child's judgment may not be mature enough to prevent misuse of it. 3. Over 4000 youth from 10 to 19 years old die from firearm homicides annually, and approximately 1500 additional youth die from firearm suicide. Firearm homicide is the second leading cause of injury death for youth, and firearm suicide is the fifth leading cause of injury death for youth (CDC, 2011c). The safety measures of using a gun lock, keeping the gun and ammunition separate, and putting the guns in a locked cabinet will at least make the guns less accessible. Telling a child that a gun is "dangerous" and not to be touched will probably make it more fascinating. Even with knowledge of the proper use of a firearm, a 10-year-old child's judgment may not be mature enough to prevent misuse of it. 4. Over 4000 youth from 10 to 19 years old die from firearm homicides annually, and approximately 1500 additional youth die from firearm suicide. Firearm homicide is the second leading cause of injury death for youth, and firearm suicide is the fifth leading cause of injury death for youth (CDC, 2011c). The safety measures of using a gun lock, keeping the gun and ammunition separate, and putting the guns in a locked cabinet will at least make the guns less accessible. Telling a child that a gun is "dangerous" and not to be touched will probably make it more fascinating. Even with knowledge of the proper use of a firearm, a 10-year-old child's judgment may not be mature enough to prevent misuse of it. 5. Over 4000 youth from 10 to 19 years old die from firearm homicides annually, and approximately 1500 additional youth die from firearm suicide. Firearm homicide is the second leading cause of injury death for youth, and firearm suicide is the fifth leading cause of injury death for youth (CDC, 2011c). The safety measures of using a gun lock, keeping the gun and ammunition separate, and putting the guns in a locked cabinet will at least make the guns less accessible. Telling a child that a gun is "dangerous" and not to be touched will probably make it more fascinating. Even with knowledge of the proper use of a firearm, a 10-year-old child's judgment may not be mature enough to prevent misuse of it. Page Ref: 385 Cognitive Level: Applying Client Need &Sub: Health Promotion and Maintenance Standards: QSEN Competencies: Patient-centered care | AACN Essential Competencies: Essential VII: Clinical prevention and population health | NLN Competencies: Nursing judgement | Nursing/Integrated Concepts: Nursing Process: Implementation/Health teaching and health promotion Learning Outcome: LO 17.6 Explore the nursing role in prevention and treatment of child abuse and neglect and other forms of violence. MNL LO: Apply key concepts of family-centered care. 11) In counseling an adolescent female about safe sex practices, which question is the most appropriate for the nurse to ask? 1. "Do you and your boyfriend use a condom every time you have sex?" 2. "Do you have a boyfriend, and if so, are you sexually active?" 3. "Do you have one or more sexual partners?" 4. "Have you and your boyfriend ever had unprotected sex?" Answer: 3 Explanation: 1. "Do you have one or more sexual partners?" provides the adolescent an opportunity to discuss sexual practices in a homosexual or alternative relationship if this applies to her. All of the other questions assume that the adolescent is involved in heterosexual relationships. 2. "Do you have one or more sexual partners?" provides the adolescent an opportunity to discuss sexual practices in a homosexual or alternative relationship if this applies to her. All of the other questions assume that the adolescent is involved in heterosexual relationships. 3. "Do you have one or more sexual partners?" provides the adolescent an opportunity to discuss sexual practices in a homosexual or alternative relationship if this applies to her. All of the other questions assume that the adolescent is involved in heterosexual relationships. 4. "Do you have one or more sexual partners?" provides the adolescent an opportunity to discuss sexual practices in a homosexual or alternative relationship if this applies to her. All of the other questions assume that the adolescent is involved in heterosexual relationships. Page Ref: 370 Cognitive Level: Analyzing Client Need &Sub: Health Promotion and Maintenance Standards: QSEN Competencies: Patient-centered care | AACN Essential Competencies: Essential IX: Baccalaureate generalist nursing practice | NLN Competencies: Nursing judgement | Nursing/Integrated Concepts: Nursing Process: Assessment/Coordination of care Learning Outcome: LO 17.3 Examine the effects of substance use, physical activity, and other lifestyle patterns on health. MNL LO: Implement developmentally appropriate communication strategies for pediatric clients and families. 12) A mother of two school-age children tells the nurse that her husband has recently been deployed overseas. The mother is concerned about the children's constant interest in watching TV news coverage of military activities overseas. Which suggestion from the nurse is the most appropriate? 1. "Allow the children to watch as much television as they want. This is how they are coping with their father's absence." 2. "It will just take some time to adjust to their father's absence, then everything will return to normal." 3. "The less that you discuss this, the quicker the children will adjust to their father's absence. Try to keep them busy, and use distractions to keep their mind off of it." 4. "Spend time with your children and take cues from them about how much they want to discuss." Answer: 4 Explanation: 1. Constant viewing of the TV coverage of the war may increase the children's anxiety and fear for their father's safety. The mother should be aware that even though the children may appear to have adjusted, there may be delayed reactions or regressions in behavior. Children need to be able to discuss their feelings and concerns with an adult; otherwise, their emotional distress may increase. 2. Constant viewing of the TV coverage of the war may increase the children's anxiety and fear for their father's safety. The mother should be aware that even though the children may appear to have adjusted, there may be delayed reactions or regressions in behavior. Children need to be able to discuss their feelings and concerns with an adult; otherwise, their emotional distress may increase. 3. Constant viewing of the TV coverage of the war may increase the children's anxiety and fear for their father's safety. The mother should be aware that even though the children may appear to have adjusted, there may be delayed reactions or regressions in behavior. Children need to be able to discuss their feelings and concerns with an adult; otherwise, their emotional distress may increase. 4. Constant viewing of the TV coverage of the war may increase the children's anxiety and fear for their father's safety. The mother should be aware that even though the children may appear to have adjusted, there may be delayed reactions or regressions in behavior. Children need to be able to discuss their feelings and concerns with an adult; otherwise, their emotional distress may increase. Page Ref: 371-372 Cognitive Level: Analyzing Client Need &Sub: Psychosocial Integrity Standards: QSEN Competencies: Patient-centered care | AACN Essential Competencies: Essential IX: Baccalaureate generalist nursing practice | NLN Competencies: Nursing judgement | Nursing/Integrated Concepts: Nursing Process: Implementation/Coordination of care Learning Outcome: LO 17.2 Apply the ecologic model and resiliency theory to assessment of the social and environmental factors in children's lives. MNL LO: Differentiate developmentally appropriate care environments for the pediatric client and family. 13) The pediatric nurse is working with a parent who is suspected of Münchausen Syndrome by Proxy. Which action by the nurse is the priority? 1. Confront the parent with concerns of possible abuse. 2. Carefully document parent-child interactions. 3. Try to keep the parent separated from the child as much as possible. 4. Explain to the child that the parent is causing the illness and that the health team will prevent the child from being harmed. Answer: 2 Explanation: 1. Münchausen Syndrome by Proxy is very difficult to prove, and evidence provided by the careful documentation of the nursing staff can be very influential. Care must be taken not to make the parent suspicious and to keep the child in the hospital until enough evidence is collected. Confronting the parent or separating the parent from the child may alienate the parent and cause him or her to leave with the child. Talking to the child about the health-care team's suspicions may be confusing and frightening for the child. 2. Münchausen Syndrome by Proxy is very difficult to prove, and evidence provided by the careful documentation of the nursing staff can be very influential. Care must be taken not to make the parent suspicious and to keep the child in the hospital until enough evidence is collected. Confronting the parent or separating the parent from the child may alienate the parent and cause him or her to leave with the child. Talking to the child about the health-care team's suspicions may be confusing and frightening for the child. 3. Münchausen Syndrome by Proxy is very difficult to prove, and evidence provided by the careful documentation of the nursing staff can be very influential. Care must be taken not to make the parent suspicious and to keep the child in the hospital until enough evidence is collected. Confronting the parent or separating the parent from the child may alienate the parent and cause him or her to leave with the child. Talking to the child about the health-care team's suspicions may be confusing and frightening for the child. 4. Münchausen Syndrome by Proxy is very difficult to prove, and evidence provided by the careful documentation of the nursing staff can be very influential. Care must be taken not to make the parent suspicious and to keep the child in the hospital until enough evidence is collected. Confronting the parent or separating the parent from the child may alienate the parent and cause him or her to leave with the child. Talking to the child about the health-care team's suspicions may be confusing and frightening for the child. Page Ref: 394-395 Cognitive Level: Applying Client Need &Sub: Safe and Effective Care Environment: Safety and Infection Control Standards: QSEN Competencies: Ethics | AACN Essential Competencies: Essential IX: Baccalaureate generalist nursing practice | NLN Competencies: Nursing judgement | Nursing/Integrated Concepts: Nursing Process: Implementation/Coordination of care Learning Outcome: LO 17.6 Explore the nursing role in prevention and treatment of child abuse and neglect and other forms of violence. MNL LO: Compare and contrast known family theories and assessment strategies. 14) A child is admitted to the hospital unit with physical injuries. The nurse is taking the child's history. Which statement by the parent would arouse suspicion of abuse? 1. "I did not realize that my baby was able to roll over yet, and I was just gone a minute to check on dinner when the baby rolled off of the couch and onto our tile floor." 2. "The baby's 18-month-old brother was trying to pull the baby out of the crib and dropped the baby on the floor." 3. "I placed the baby in the infant swing. His 6-year-old brother was running through the house and tripped over the swing, causing it to fall." 4. "I was walking up the steps and slipped on the ice, falling while carrying my baby." Answer: 2 Explanation: 1. All of the statements made by the parent are plausible from a developmental perspective except the statement "The baby's 18-month-old brother was trying to pull the baby out of the crib and dropped the baby on the floor." Developmentally, it would be very difficult for an 18-month-old child to pull an infant out of a crib. 2. All of the statements made by the parent are plausible from a developmental perspective except the statement "The baby's 18-month-old brother was trying to pull the baby out of the crib and dropped the baby on the floor." Developmentally, it would be very difficult for an 18-month- old child to pull an infant out of a crib. 3. All of the statements made by the parent are plausible from a developmental perspective except the statement "The baby's 18-month-old brother was trying to pull the baby out of the crib and dropped the baby on the floor." Developmentally, it would be very difficult for an 18-month- old child to pull an infant out of a crib. 4. All of the statements made by the parent are plausible from a developmental perspective except the statement "The baby's 18-month-old brother was trying to pull the baby out of the crib and dropped the baby on the floor." Developmentally, it would be very difficult for an 18-month- old child to pull an infant out of a crib. Page Ref: 389-390 Cognitive Level: Analyzing Client Need &Sub: Psychosocial Integrity Standards: QSEN Competencies: Patient-centered care | AACN Essential Competencies: Essential IX: Baccalaureate generalist nursing practice | NLN Competencies: Nursing judgement | Nursing/Integrated Concepts: Nursing Process: Diagnosis/Coordination of care Learning Outcome: LO 17.6 Explore the nursing role in prevention and treatment of child abuse and neglect and other forms of violence. MNL LO: Compare the developmental stages for pediatric clients. 15) The nurse is providing care to a toddler-age child. Which assessment finding is indicative of abuse? 1. Parents indicating that they did not see the event occur 2. Inconsistency of stories between caregivers 3. Bruising noted on the knees and shins 4. Acting out behavior of the child Answer: 2 Explanation: 1. Inconsistency of stories is a red flag for abuse. All other answers are logical explanations for this age group. 2. Inconsistency of stories is a red flag for abuse. All other answers are logical explanations for this age group. 3. Inconsistency of stories is a red flag for abuse. All other answers are logical explanations for this age group. 4. Inconsistency of stories is a red flag for abuse. All other answers are logical explanations for this age group. Page Ref: 389-390 Cognitive Level: Analyzing Client Need &Sub: Safe and Effective Care Environment: Safety and Infection Control Standards: QSEN Competencies: Patient-centered care | AACN Essential Competencies: Essential IX: Baccalaureate generalist nursing practice | NLN Competencies: Nursing judgement | Nursing/Integrated Concepts: Nursing Process: Assessment/Coordination of care Learning Outcome: LO 17.6 Explore the nursing role in prevention and treatment of child abuse and neglect and other forms of violence. MNL LO: Analyze the role of the nurse and the role of the family in pediatric care. 16) The nurse teaches a group of parents' strategies to reduce the risk of lead exposure for their children. Which statements indicate an appropriate understanding of the content presented? Select all that apply. 1. "We will provide our child with frequent snacks high in iron and calcium." 2. "We will wash any surfaces that have peeling paint." 3. "We will store leftovers in a ceramic pot." 4. "We can continue to use our traditional-medicine treatment, Azarcon, for any GI upset." 5. "We will sand the windowsills to remove the lead-based paint." Answer: 1, 2 Explanation: 1. Snacks and meals high in iron and calcium should be encouraged. Lead is absorbed more readily on an empty stomach. Any surface with peeling paint should be washed with a damp sponge. Ceramic pots, if fired improperly, could contain lead. Food should not be prepared or stored in them. Azarcon, a traditional medicine used to treat a colic-like illness, may contain large amounts of lead. Sanding the windowsills will cause the lead to be dispersed in the air, leading to lead poisoning. 2. Snacks and meals high in iron and calcium should be encouraged. Lead is absorbed more readily on an empty stomach. Any surface with peeling paint should be washed with a damp sponge. Ceramic pots, if fired improperly, could contain lead. Food should not be prepared or stored in them. Azarcon, a traditional medicine used to treat a colic-like illness, may contain large amounts of lead. Sanding the windowsills will cause the lead to be dispersed in the air, leading to lead poisoning. 3. Snacks and meals high in iron and calcium should be encouraged. Lead is absorbed more readily on an empty stomach. Any surface with peeling paint should be washed with a damp sponge. Ceramic pots, if fired improperly, could contain lead. Food should not be prepared or stored in them. Azarcon, a traditional medicine used to treat a colic-like illness, may contain large amounts of lead. Sanding the windowsills will cause the lead to be dispersed in the air, leading to lead poisoning. 4. Snacks and meals high in iron and calcium should be encouraged. Lead is absorbed more readily on an empty stomach. Any surface with peeling paint should be washed with a damp sponge. Ceramic pots, if fired improperly, could contain lead. Food should not be prepared or stored in them. Azarcon, a traditional medicine used to treat a colic-like illness, may contain large amounts of lead. Sanding the windowsills will cause the lead to be dispersed in the air, leading to lead poisoning. 5. Snacks and meals high in iron and calcium should be encouraged. Lead is absorbed more readily on an empty stomach. Any surface with peeling paint should be washed with a damp sponge. Ceramic pots, if fired improperly, could contain lead. Food should not be prepared or stored in them. Azarcon, a traditional medicine used to treat a colic-like illness, may contain large amounts of lead. Sanding the windowsills will cause the lead to be dispersed in the air, leading to lead poisoning. Page Ref: 400-401 Cognitive Level: Analyzing Client Need &Sub: Health Promotion and Maintenance Standards: QSEN Competencies: Patient-centered care | AACN Essential Competencies: Essential VII: Clinical prevention and population health | NLN Competencies: Nursing judgement | Nursing/Integrated Concepts: Nursing Process: Evaluation/Health teaching and health promotion Learning Outcome: LO 17.5 Evaluate the environment for hazards to children, such as exposure to harmful substances and potential for poisoning. MNL LO: Evaluate healthcare issues related to pediatric nursing care. 17) The school nurse is implementing a program to decrease bullying. Which interventions are appropriate for the school nurse to implement? Select all that apply. 1. Train teachers about the behaviors 2. Ensure adult supervision in the hallways 3. Teach children to report behaviors 4. Ensure that immunizations are up-to-date 5. Set up anti-hazing policies Answer: 1, 2, 3 Explanation: 1. Appropriate interventions for the school nurse to implement when dealing with bullying in a school include training the teachers on the signs of bullying; ensuring adult supervision in the hallways, as this is where bullying tends to take place; and teach children to report bullying behaviors. Ensuring that immunizations are up to date is not an intervention aimed at decreasing bullying. Hazing and bullying are two separate problems. 2. Appropriate interventions for the school nurse to implement when dealing with bullying in a school include training the teachers on the signs of bullying; ensuring adult supervision in the hallways, as this is where bullying tends to take place; and teach children to report bullying behaviors. Ensuring that immunizations are up to date is not an intervention aimed at decreasing bullying. Hazing and bullying are two separate problems. 3. Appropriate interventions for the school nurse to implement when dealing with bullying in a school include training the teachers on the signs of bullying; ensuring adult supervision in the hallways, as this is where bullying tends to take place; and teach children to report bullying behaviors. Ensuring that immunizations are up to date is not an intervention aimed at decreasing bullying. Hazing and bullying are two separate problems. 4. Appropriate interventions for the school nurse to implement when dealing with bullying in a school include training the teachers on the signs of bullying; ensuring adult supervision in the hallways, as this is where bullying tends to take place; and teach children to report bullying behaviors. Ensuring that immunizations are up to date is not an intervention aimed at decreasing bullying. Hazing and bullying are two separate problems. 5. Appropriate interventions for the school nurse to implement when dealing with bullying in a school include training the teachers on the signs of bullying; ensuring adult supervision in the hallways, as this is where bullying tends to take place; and teach children to report bullying behaviors. Ensuring that immunizations are up to date is not an intervention aimed at decreasing bullying. Hazing and bullying are two separate problems. Page Ref: 385-386 Cognitive Level: Applying Client Need &Sub: Safe and Effective Care Environment: Safety and Infection Control Standards: QSEN Competencies: Patient-centered care | AACN Essential Competencies: Essential II: Basic organizational and systems leadership for quality care and patient safety | NLN Competencies: Nursing judgement | Nursing/Integrated Concepts: Nursing Process: Implementation/Coordination of care Learning Outcome: LO 17.4 Plan nursing interventions for children who experience violence. MNL LO: Use the nursing process to provide developmentally appropriate care for the pediatric client. 18) The nurse is conducting an admission assessment for a preschool-age client in the emergency department. When using the resiliency theory, which findings place this client at risk? Select all that apply. 1. Loss of health insurance 2. No primary care provider 3. Incomplete immunizations 4. A grandmother who is able to room-in 5. High level language skills from the child Answer: 1, 2, 3 Explanation: 1. When using the resiliency theory, a child and family will have both protective and risk factors. Risk factors include lack of health insurance, not having a consistent care provider, and incomplete immunizations. Protective factors include a parent or family member being able to room-in with the child, a family who is able to stay with the other children in the family, and a child with the ability to communicate needs to the hospital staff. 2. When using the resiliency theory, a child and family will have both protective and risk factors. Risk factors include lack of health insurance, not having a consistent care provider, and incomplete immunizations. Protective factors include a parent or family member being able to room-in with the child, a family who is able to stay with the other children in the family, and a child with the ability to communicate needs to the hospital staff. 3. When using the resiliency theory, a child and family will have both protective and risk factors. Risk factors include lack of health insurance, not having a consistent care provider, and incomplete immunizations. Protective factors include a parent or family member being able to room-in with the child, a family who is able to stay with the other children in the family, and a child with the ability to communicate needs to the hospital staff. 4. When using the resiliency theory, a child and family will have both protective and risk factors. Risk factors include lack of health insurance, not having a consistent care provider, and incomplete immunizations. Protective factors include a parent or family member being able to room-in with the child, a family who is able to stay with the other children in the family, and a child with the ability to communicate needs to the hospital staff. 5. When using the resiliency theory, a child and family will have both protective and risk factors. Risk factors include lack of health insurance, not having a consistent care provider, and incomplete immunizations. Protective factors include a parent or family member being able to room-in with the child, a family who is able to stay with the other children in the family, and a child with the ability to communicate needs to the hospital staff. Page Ref: 369 Cognitive Level: Applying Client Need &Sub: Safe and Effective Care Environment: Safety and Infection Control Standards: QSEN Competencies: Patient-centered care | AACN Essential Competencies: Essential IX: Baccalaureate generalist nursing practice | NLN Competencies: Nursing judgement | Nursing/Integrated Concepts: Nursing Process: Assessment/Coordination of care Learning Outcome: LO 17.2 Apply the ecologic model and resiliency theory to assessment of the social and environmental factors in children's lives. MNL LO: Compare and contrast known family theories and assessment strategies. 19) A social service coordinator is consulted to arrange for a phototherapy blanket at discharge for an infant/family with multiple social difficulties. Which social difficulty is more than likely to have the greatest influence on discharge? 1. Cultural practices and rituals 2. Financial difficulties 3. The family is homeless 4. The family does not have a healthcare provider 5. Religious beliefs Answer: 3 Explanation: 1. This is not the greatest influence on discharge. The greatest impact on discharge is the fact that the family is homeless, all other aspects of care depend on homelessness. 2. This is not the greatest influence on discharge. The greatest impact on discharge is the fact that the family is homeless, all other aspects of care depend on homelessness. 3. The greatest impact on discharge is the fact that the family is homeless, all other aspects of care depend on homelessness. 4. This is not the greatest influence on discharge. The greatest impact on discharge is the fact that the family is homeless, all other aspects of care depend on homelessness. 5. This is not the greatest influence on discharge. The greatest impact on discharge is the fact that the family is homeless, all other aspects of care depend on homelessness. Page Ref: 369-372 Cognitive Level: Analyzing Client Need &Sub: Psychosocial Integrity Standards: QSEN Competencies: Patient-centered care | AACN Essential Competencies: Essential IX: Baccalaureate generalist nursing practice | NLN Competencies: Nursing judgement | Nursing/Integrated Concepts: Nursing Process: Assessment/Coordination of care Learning Outcome: LO 17.1 Identify major social and environmental factors that influence the health of children and adolescents. MNL LO: Examine the role of the nurse in promoting culturally competent family-centered care. 20) What are some common health problems associated with the poor and/or homeless child? Select all that apply. 1. Asthma 2. Sexually transmitted infections 3. Good dentition 4. Mental illness 5. Tuberculosis Answer: 2, 4, 5 Explanation: 1. This is not a common health problem in children who are poor and/or homeless. Common health problems among children who are poor and/or homeless include: lack of immunizations, common infectious diseases, sleep deficits, vision and hearing deficits, nutritional deficits, dental care problems, injuries, pregnancy, sexually transmitted infections, and mental illness. 2. This is a common health problem in children who are poor and/or homeless. Common health problems among children who are poor and/or homeless include: lack of immunizations, common infectious diseases, sleep deficits, vision and hearing deficits, nutritional deficits, dental care problems, injuries, pregnancy, sexually transmitted infections, and mental illness. 3. This is not a common health problem in children who are poor and/or homeless. Common health problems among children who are poor and/or homeless include: lack of immunizations, common infectious diseases, sleep deficits, vision and hearing deficits, nutritional deficits, dental care problems, injuries, pregnancy, sexually transmitted infections, and mental illness. 4. This is a common health problem in children who are poor and/or homeless. Common health problems among children who are poor and/or homeless include: lack of immunizations, common infectious diseases, sleep deficits, vision and hearing deficits, nutritional deficits, dental care problems, injuries, pregnancy, sexually transmitted infections, and mental illness. 5. This is a common health problem in children who are poor and/or homeless. Common health problems among children who are poor and/or homeless include: lack of immunizations, common infectious diseases, sleep deficits, vision and hearing deficits, nutritional deficits, dental care problems, injuries, pregnancy, sexually transmitted infections, and mental illness. Page Ref: 371 Cognitive Level: Analyzing Client Need &Sub: Psychosocial Integrity Standards: QSEN Competencies: Patient-centered care | AACN Essential Competencies: Essential IX: Baccalaureate generalist nursing practice | NLN Competencies: Nursing judgement | Nursing/Integrated Concepts: Nursing Process: Assessment/Coordination of care Learning Outcome: LO 17.1 Identify major social and environmental factors that influence the health of children and adolescents. MNL LO: Examine the role of the nurse in promoting culturally competent family-centered care. Principles of Pediatric Nursing: Caring for Children, 7e (Ball et al.) Chapter 18 Alterations in Fluid, Electrolyte, and Acid-Base Balance 1) A nurse is taking care of four different pediatric clients. Which client poses the great risk for dehydration? 1. A 15-year-old working out in a weight room for an hour before football practice 2. A 10-year-old playing baseball outdoors in 85-degree heat 3. A 5-year-old refusing to eat because of a virus 4. A newborn under a radiant warmer for an hour after the first bath Answer: 2 Explanation: 1. A condition that increases the risk of insensible fluid loss places the child at risk for dehydration. Any of these situations can place the child at risk for dehydration but the child at greatest risk is the child playing baseball in direct heat, which will increase utilization of extracellular fluids more rapidly than the other situations. 2. A condition that increases the risk of insensible fluid loss places the child at risk for dehydration. Any of these situations can place the child at risk for dehydration but the child at greatest risk is the child playing baseball in direct heat, which will increase utilization of extracellular fluids more rapidly than the other situations. 3. A condition that increases the risk of insensible fluid loss places the child at risk for dehydration. Any of these situations can place the child at risk for dehydration but the child at greatest risk is the child playing baseball in direct heat, which will increase utilization of extracellular fluids more rapidly than the other situations. 4. A condition that increases the risk of insensible fluid loss places the child at risk for dehydration. Any of these situations can place the child at risk for dehydration but the child at greatest risk is the child playing baseball in direct heat, which will increase utilization of extracellular fluids more rapidly than the other situations. Page Ref: 408 Cognitive Level: Analyzing Client Need &Sub: Safe and Effective Care Environment: Safety and Infection Control Standards: QSEN Competencies: Patient-centered care | AACN Essential Competencies: Essential IX: Baccalaureate generalist nursing practice | NLN Competencies: Nursing judgement | Nursing/Integrated Concepts: Nursing Process: Assessment/Coordination of care Learning Outcome: LO 18.3 Interpret threats to fluid and electrolyte balance in children. MNL LO: Gastrointestinal Disorders/Recognize feeding disorders in the pediatric population. 2) The nurse is assessing an infant brought to the clinic with diarrhea. The infant is alert but has dry mucous membranes. Which other sign indicates the infant is still in the early or mild stage of dehydration? 1. Tachycardia 2. Bradycardia 3. Increased blood pressure 4. Decreased blood pressure Answer: 1 Explanation: 1. Tachycardia is a sign that indicates mild dehydration. Bradycardia and increased blood pressure are not signs of dehydration. Decreased blood pressure is not a sign of mild dehydration. Decreased blood pressure indicates moderate to severe dehydration. 2. Tachycardia is a sign that indicates mild dehydration. Bradycardia and increased blood pressure are not signs of dehydration. Decreased blood pressure is not a sign of mild dehydration. Decreased blood pressure indicates moderate to severe dehydration. 3. Tachycardia is a sign that indicates mild dehydration. Bradycardia and increased blood pressure are not signs of dehydration. Decreased blood pressure is not a sign of mild dehydration. Decreased blood pressure indicates moderate to severe dehydration. 4. Tachycardia is a sign that indicates mild dehydration. Bradycardia and increased blood pressure are not signs of dehydration. Decreased blood pressure is not a sign of mild dehydration. Decreased blood pressure indicates moderate to severe dehydration. Page Ref: 411 Cognitive Level: Analyzing Client Need &Sub: Physiological Integrity: Physiological Adaptation Standards: QSEN Competencies: Patient-centered care | AACN Essential Competencies: Essential IX: Baccalaureate generalist nursing practice | NLN Competencies: Nursing judgement | Nursing/Integrated Concepts: Nursing Process: Assessment/Coordination of care Learning Outcome: LO 18.5 Analyze assessment findings to recognize fluid-electrolyte problems and acid—base imbalance in children. MNL LO: Gastrointestinal Disorders/Examine etiology, risk factors, pathophysiology, and clinical manifestations as seen in children. 3) A 1-month-old client is admitted to the emergency room with severe diarrhea. Which assessment suggests the client is severely dehydrated? 1. Skin moist and flushed; mucous membranes dry 2. Low specific gravity of urine; skin color pale 3. Fontanels depressed; capillary refill greater than three seconds 4. High specific gravity of urine; moist mucous membranes Answer: 3 Explanation: 1. Two signs of severe dehydration are depressed fontanels and capillary refill time greater than three seconds. Moist, flushed skin; moist mucous membranes; and low specific gravity of urine are not signs of dehydration. Dry mucous membranes and pale skin color are signs of mild dehydration, not severe. 2. Two signs of severe dehydration are depressed fontanels and capillary refill time greater than three seconds. Moist, flushed skin; moist mucous membranes; and low specific gravity of urine are not signs of dehydration. Dry mucous membranes and pale skin color are signs of mild dehydration, not severe. 3. Two signs of severe dehydration are depressed fontanels and capillary refill time greater than three seconds. Moist, flushed skin; moist mucous membranes; and low specific gravity of urine are not signs of dehydration. Dry mucous membranes and pale skin color are signs of mild dehydration, not severe. 4. Two signs of severe dehydration are depressed fontanels and capillary refill time greater than three seconds. Moist, flushed skin; moist mucous membranes; and low specific gravity of urine are not signs of dehydration. Dry mucous membranes and pale skin color are signs of mild dehydration, not severe. Page Ref: 410 Cognitive Level: Analyzing Client Need &Sub: Physiological Integrity: Physiological Adaptation Standards: QSEN Competencies: Patient-centered care | AACN Essential Competencies: Essential IX: Baccalaureate generalist nursing practice | NLN Competencies: Nursing judgement | Nursing/Integrated Concepts: Nursing Process: Assessment/Coordination of care Learning Outcome: LO 18.5 Analyze assessment findings to recognize fluid-electrolyte problems and acid—base imbalance in children. MNL LO: Gastrointestinal Disorders/Examine etiology, risk factors, pathophysiology, and clinical manifestations as seen in children. 4) The nurse is expecting the admission of a child with severe isotonic dehydration. Which intravenous fluid should the nurse anticipate the practitioner to order initially to replace fluids? 1. D5W 2. 0.9 percent Normal Saline (NS) 3. Albumin 4. D5 0.2 percent (1/4) Normal Saline Answer: 2 Explanation: 1. 0.9 percent Normal Saline (NS) maintains Na and chloride at present levels. D5W can lower sodium levels so would not be used to initially replace fluids in severe isotonic dehydration. Albumin is used to restore plasma proteins. D5 0.2 percent (1/4) Normal Saline would not be used initially but later, as maintenance fluids. 2. 0.9 percent Normal Saline (NS) maintains Na and chloride at present levels. D5W can lower sodium levels so would not be used to initially replace fluids in severe isotonic dehydration. Albumin is used to restore plasma proteins. D5 0.2 percent (1/4) Normal Saline would not be used initially but later, as maintenance fluids. 3. 0.9 percent Normal Saline (NS) maintains Na and chloride at present levels. D5W can lower sodium levels so would not be used to initially replace fluids in severe isotonic dehydration. Albumin is used to restore plasma proteins. D5 0.2 percent (1/4) Normal Saline would not be used initially but later, as maintenance fluids. 4. 0.9 percent Normal Saline (NS) maintains Na and chloride at present levels. D5W can lower sodium levels so would not be used to initially replace fluids in severe isotonic dehydration. Albumin is used to restore plasma proteins. D5 0.2 percent (1/4) Normal Saline would not be used initially but later, as maintenance fluids. Page Ref: 413 Cognitive Level: Applying Client Need &Sub: Physiological Integrity: Pharmacological and Parenteral Therapies Standards: QSEN Competencies: Patient-centered care | AACN Essential Competencies: Essential IX: Baccalaureate generalist nursing practice | NLN Competencies: Nursing judgement | Nursing/Integrated Concepts: Nursing Process: Implementation/Coordination of care Learning Outcome: LO 18.6 Plan appropriate nursing interventions for children experiencing fluid-electrolyte problems and acid—base imbalance. MNL LO: Gastrointestinal Disorders/Apply the nursing process in providing care for the child and family. 5) Parents of an infant with slow weight gain ask the nurse if they can feed their baby a highly concentrated formula. Which response by the nurse is the most appropriate? 1. "A higher-concentrated formula could lead to dehydration because of high sodium content; let's discuss other strategies." 2. "An undiluted formula concentrate could be given to help the child gain weight; let's look at brands." 3. "Evaporated milk could be given to the infant instead of the current formula you're using." 4. "A higher-concentrated formula could be given for daytime feedings; let's work on a schedule." Answer: 1 Explanation: 1. Parents and caregivers of bottle-fed babies should be taught never to give undiluted formula concentrate or evaporated milk due to the high sodium content. 2. Parents and caregivers of bottle-fed babies should be taught never to give undiluted formula concentrate or evaporated milk due to the high sodium content. 3. Parents and caregivers of bottle-fed babies should be taught never to give undiluted formula concentrate or evaporated milk due to the high sodium content. 4. Parents and caregivers of bottle-fed babies should be taught never to give undiluted formula concentrate or evaporated milk due to the high sodium content. Page Ref: 416 Cognitive Level: Applying Client Need &Sub: Physiological Integrity: Basic Care and Comfort Standards: QSEN Competencies: Patient-centered care | AACN Essential Competencies: Essential VII: Clinical prevention and population health | NLN Competencies: Nursing judgement | Nursing/Integrated Concepts: Nursing Process: Implementation/Health teaching and health promotion Learning Outcome: LO 18.3 Interpret threats to fluid and electrolyte balance in children. MNL LO: Gastrointestinal Disorders/Recognize feeding disorders in the pediatric population. 6) The nurse finishes a parent-teaching session on preventing heat-related illnesses for children who exercise. Which statement by a parent indicates understanding of preventive techniques taught? 1. Hydration should occur at the end of an exercise session. 2. Water is the drink of choice to replenish fluids. 3. Wearing dark clothing during exercise is recommended. 4. During activity, stop for fluids every 15 to 20 minutes. Answer: 4 Explanation: 1. During activity, stopping for fluids every 15 to 20 minutes is recommended. Hydration should occur before and during the activity, not just at the end. A combination of water and sports drinks is best to replace fluids during exercise. Light-colored, light clothing is best to wear during exercise activities; wearing of dark colors can increase sweating. 2. During activity, stopping for fluids every 15 to 20 minutes is recommended. Hydration should occur before and during the activity, not just at the end. A combination of water and sports drinks is best to replace fluids during exercise. Light-colored, light clothing is best to wear during exercise activities; wearing of dark colors can increase sweating. 3. During activity, stopping for fluids every 15 to 20 minutes is recommended. Hydration should occur before and during the activity, not just at the end. A combination of water and sports drinks is best to replace fluids during exercise. Light-colored, light clothing is best to wear during exercise activities; wearing of dark colors can increase sweating. 4. During activity, stopping for fluids every 15 to 20 minutes is recommended. Hydration should occur before and during the activity, not just at the end. A combination of water and sports drinks is best to replace fluids during exercise. Light-colored, light clothing is best to wear during exercise activities; wearing of dark colors can increase sweating. Page Ref: 415 Cognitive Level: Analyzing Client Need &Sub: Health Promotion and Maintenance Standards: QSEN Competencies: Patient-centered care | AACN Essential Competencies: Essential VII: Clinical prevention and population health | NLN Competencies: Nursing judgement | Nursing/Integrated Concepts: Nursing Process: Evaluation/Health teaching and health promotion Learning Outcome: LO 18.6 Plan appropriate nursing interventions for children experiencing fluid-electrolyte problems and acid—base imbalance. MNL LO: Gastrointestinal Disorders/Apply the nursing process in providing care for the child and family. 7) A child is being treated for dehydration with intravenous fluids. The child currently weighs 13 kg and is estimated to have lost 7 percent of the normal body weight. The nurse is double- checking the IV rate the practitioner has ordered. The formula the practitioner used was for maintenance fluids: 1000 mL for 10 kg of body weight plus 50 cc for every kg over 10 for 24 hours. Replacement fluid is the percentage of lost body weight × 10 per kg of body weight. According to the calculation for maintenance plus replacement fluid, this child's hourly IV rate for 24 hours should be _ mL. Round the answer to the nearest whole number. Answer: 86 Explanation: Maintenance need for 13 kg is 1000 + (50 × 3), or 1150 mL/24 hours. Add to this the replacement-fluid loss = 7 (percent of total body weight lost) × 10 = 70 mL/kg/24 hours (70 × 13 = 910). 1150 + 910 = 2060 for 24 hours. 2060/24 = 86 mL per hour. Page Ref: 416 Cognitive Level: Applying Client Need &Sub: Physiological Integrity: Pharmacological and Parenteral Therapies Standards: QSEN Competencies: Patient-centered care | AACN Essential Competencies: Essential IX: Baccalaureate generalist nursing practice | NLN Competencies: Nursing judgement | Nursing/Integrated Concepts: Nursing Process: Implementation/Coordination of care Learning Outcome: LO 18.6 Plan appropriate nursing interventions for children experiencing fluid-electrolyte problems and acid—base imbalance. MNL LO: Gastrointestinal Disorders/Apply the nursing process in providing care for the child and family. 8) In the morning, a nurse receives a report on four pediatric clients who have some form of fluid-volume excess. Which client should the nurse assess first? 1. A client with periorbital edema, normal respiratory rate 2. A client with tachypnea and pulmonary congestion 3. A client with dependent and sacral edema, regular pulse 4. A client with hepatomegaly, normal respiratory rate Answer: 2 Explanation: 1. A child with respiratory distress should be the first client the nurse checks after receiving report. The child with periorbital edema and normal respiratory rate, the child with dependent and sacral edema and regular pulse, and the child with hepatomegaly and normal respiratory rate are all more stable than the child with tachypnea and pulmonary congestion. 2. A child with respiratory distress should be the first client the nurse checks after receiving report. The child with periorbital edema and normal respiratory rate, the child with dependent and sacral edema and regular pulse, and the child with hepatomegaly and normal respiratory rate are all more stable than the child with tachypnea and pulmonary congestion. 3. A child with respiratory distress should be the first client the nurse checks after receiving report. The child with periorbital edema and normal respiratory rate, the child with dependent and sacral edema and regular pulse, and the child with hepatomegaly and normal respiratory rate are all more stable than the child with tachypnea and pulmonary congestion. 4. A child with respiratory distress should be the first client the nurse checks after receiving report. The child with periorbital edema and normal respiratory rate, the child with dependent and sacral edema and regular pulse, and the child with hepatomegaly and normal respiratory rate are all more stable than the child with tachypnea and pulmonary congestion. Page Ref: 418 Cognitive Level: Analyzing Client Need &Sub: Safe and Effective Care Environment: Safety and Infection Control Standards: QSEN Competencies: Patient-centered care | AACN Essential Competencies: Essential IX: Baccalaureate generalist nursing practice | NLN Competencies: Nursing judgement | Nursing/Integrated Concepts: Nursing Process: Implementation/Coordination of care Learning Outcome: LO 18.3 Interpret threats to fluid and electrolyte balance in children. MNL LO: Gastrointestinal Disorders/Recognize feeding disorders in the pediatric population. 9) The nurse is caring for a child on bed rest who has severe edema in a left lower leg due to blocked lymphatic drainage. Which is the priority diagnosis for this child? 1. Risk for Imbalanced Nutrition: Less Than Body Requirements 2. Risk for Impaired Skin Integrity 3. Risk for Altered Body Image 4. Risk for Activity Intolerance Answer: 2 Explanation: 1. The highest priority problem is skin integrity. Nutrition, body image, and activity intolerance would not take priority over the integrity of the skin for this scenario. 2. The highest priority problem is skin integrity. Nutrition, body image, and activity intolerance would not take priority over the integrity of the skin for this scenario. 3. The highest priority problem is skin integrity. Nutrition, body image, and activity intolerance would not take priority over the integrity of the skin for this scenario. 4. The highest priority problem is skin integrity. Nutrition, body image, and activity intolerance would not take priority over the integrity of the skin for this scenario. Page Ref: 418 Cognitive Level: Applying Client Need &Sub: Safe and Effective Care Environment: Safety and Infection Control Standards: QSEN Competencies: Patient-centered care | AACN Essential Competencies: Essential IX: Baccalaureate generalist nursing practice | NLN Competencies: Nursing judgement | Nursing/Integrated Concepts: Nursing Process: Diagnosis/Coordination of care Learning Outcome: LO 18.5 Analyze assessment findings to recognize fluid-electrolyte problems and acid—base imbalance in children. MNL LO: Gastrointestinal Disorders/Apply the nursing process in providing care for the child and family. 10) A nurse is planning care for a child with hyponatremia. The nurse, delegating care of this child to a new RN on the pediatric unit, cautions the new nurse to be especially alert for which condition in the child? 1. Seizures 2. Bradycardia 3. Respiratory distress 4. Hyperthermia Answer: 1 Explanation: 1. A child with hyponatremia is at risk for seizures. Bradycardia, respiratory distress, and hyperthermia are not risks of hyponatremia. 2. A child with hyponatremia is at risk for seizures. Bradycardia, respiratory distress, and hyperthermia are not risks of hyponatremia. 3. A child with hyponatremia is at risk for seizures. Bradycardia, respiratory distress, and hyperthermia are not risks of hyponatremia. 4. A child with hyponatremia is at risk for seizures. Bradycardia, respiratory distress, and hyperthermia are not risks of hyponatremia. Page Ref: 423 Cognitive Level: Analyzing Client Need &Sub: Safe and Effective Care Environment: Safety and Infection Control Standards: QSEN Competencies: Patient-centered care | AACN Essential Competencies: Essential IX: Baccalaureate generalist nursing practice | NLN Competencies: Nursing judgement | Nursing/Integrated Concepts: Nursing Process: Planning/Coordination of care Learning Outcome: LO 18.5 Analyze assessment findings to recognize fluid-electrolyte problems and acid—base imbalance in children. MNL LO: Gastrointestinal Disorders/Apply the nursing process in providing care for the child and family. 11) A nurse is planning care for a child with hyperkalemia. Which clinical manifestation will the nurse plan to assessment this child for based on the diagnosis? 1. Seizures 2. Bradycardia 3. Respiratory distress 4. Hyperthermia Answer: 2 Explanation: 1. A child with hyperkalemia is at risk for cardiac issues. Seizures, respiratory distress, and hyperthermia are not risks of hyperkalemia. 2. A child with hyperkalemia is at risk for cardiac issues. Seizures, respiratory distress, and hyperthermia are not risks of hyperkalemia. 3. A child with hyperkalemia is at risk for cardiac issues. Seizures, respiratory distress, and hyperthermia are not risks of hyperkalemia. 4. A child with hyperkalemia is at risk for cardiac issues. Seizures, respiratory distress, and hyperthermia are not risks of hyperkalemia. Page Ref: 424-425 Cognitive Level: Analyzing Client Need &Sub: Physiological Integrity: Physiological Adaptation Standards: QSEN Competencies: Patient-centered care | AACN Essential Competencies: Essential IX: Baccalaureate generalist nursing practice | NLN Competencies: Nursing judgement | Nursing/Integrated Concepts: Nursing Process: Planning/Coordination of care Learning Outcome: LO 18.2 Identify regulatory mechanisms for fluid and electrolyte balance. MNL LO: Gastrointestinal Disorders/Examine etiology, risk factors, pathophysiology, and clinical manifestations as seen in children. 12) A school-age client is hypokalemic. The nurse is helping the client complete her menu. Which food selection will the nurse encourage for this client? 1. A hamburger with French fries 2. Pizza with a fruit plate 3. Chicken strips with chips 4. A fajita with rice Answer: 2 Explanation: 1. Pizza with the fruit plate should be encouraged because fruits (bananas, apricots, cantaloupe, cherries, peaches, and strawberries) have high amounts of potassium, and a child is likely to eat this combination. 2. Pizza with the fruit plate should be encouraged because fruits (bananas, apricots, cantaloupe, cherries, peaches, and strawberries) have high amounts of potassium, and a child is likely to eat this combination. 3. Pizza with the fruit plate should be encouraged because fruits (bananas, apricots, cantaloupe, cherries, peaches, and strawberries) have high amounts of potassium, and a child is likely to eat this combination. 4. Pizza with the fruit plate should be encouraged because fruits (bananas, apricots, cantaloupe, cherries, peaches, and strawberries) have high amounts of potassium, and a child is likely to eat this combination. Page Ref: 426 Cognitive Level: Applying Client Need &Sub: Physiological Integrity: Reduction of Risk Potential Standards: QSEN Competencies: Patient-centered care | AACN Essential Competencies: Essential IX: Baccalaureate generalist nursing practice | NLN Competencies: Nursing judgement | Nursing/Integrated Concepts: Nursing Process: Implementation/Coordination of care Learning Outcome: LO 18.6 Plan appropriate nursing interventions for children experiencing fluid-electrolyte problems and acid—base imbalance. MNL LO: Gastrointestinal Disorders/Apply the nursing process in providing care for the child and family. 13) A child is admitted to the hospital for hypercalcemia and is placed on diuretic therapy. Which diuretic would the nurse expect to give? 1. Hydrochlorothiazide (Aquazide) 2. Spironolactone (Aldactone) 3. Furosemide (Lasix) 4. Mannitol (Osmitrol) Answer: 3 Explanation: 1. Furosemide (Lasix) is the diuretic used to aid in excretion of calcium. Thiazide diuretics (hydrochlorothiazide) decrease calcium excretion and should not be given to the hypercalcemic client. Mannitol (Osmitrol) is a diuretic used to decrease cerebral edema and is not routinely used to aid in excretion of calcium. Spironolactone (Aldactone) is a potassium- sparing diuretic and would not be effective for excretion of calcium. 2. Furosemide (Lasix) is the diuretic used to aid in excretion of calcium. Thiazide diuretics (hydrochlorothiazide) decrease calcium excretion and should not be given to the hypercalcemic client. Mannitol (Osmitrol) is a diuretic used to decrease cerebral edema and is not routinely used to aid in excretion of calcium. Spironolactone (Aldactone) is a potassium-sparing diuretic and would not be effective for excretion of calcium. 3. Furosemide (Lasix) is the diuretic used to aid in excretion of calcium. Thiazide diuretics (hydrochlorothiazide) decrease calcium excretion and should not be given to the hypercalcemic client. Mannitol (Osmitrol) is a diuretic used to decrease cerebral edema and is not routinely used to aid in excretion of calcium. Spironolactone (Aldactone) is a potassium-sparing diuretic and would not be effective for excretion of calcium. 4. Furosemide (Lasix) is the diuretic used to aid in excretion of calcium. Thiazide diuretics (hydrochlorothiazide) decrease calcium excretion and should not be given to the hypercalcemic client. Mannitol (Osmitrol) is a diuretic used to decrease cerebral edema and is not routinely used to aid in excretion of calcium. Spironolactone (Aldactone) is a potassium-sparing diuretic and would not be effective for excretion of calcium. Page Ref: 428-429 Cognitive Level: Applying Client Need &Sub: Physiological Integrity: Pharmacological and Parenteral Therapies Standards: QSEN Competencies: Patient-centered care | AACN Essential Competencies: Essential IX: Baccalaureate generalist nursing practice | NLN Competencies: Nursing judgement | Nursing/Integrated Concepts: Nursing Process: Planning/Coordination of care Learning Outcome: LO 18.6 Plan appropriate nursing interventions for children experiencing fluid-electrolyte problems and acid—base imbalance. MNL LO: Demonstrate safe medication administration for the pediatric client. 14) A preschool-aged client, diagnosed with croup, has an increased pCO2, a decreased pH, and a normal HCO3 blood-gas value. Which documentation in the medical record is the most appropriate? 1. Uncompensated respiratory acidosis 2. Uncompensated respiratory alkalosis 3. Uncompensated metabolic acidosis 4. Uncompensated metabolic alkalosis Answer: 1 Explanation: 1. If the pH is decreased and the pCO2 is increased with a normal HCO3, it is uncompensated respiratory acidosis. Also, croup can be a disease process that causes respiratory acidosis. Uncompensated respiratory alkalosis has an increased pH, decreased pCO2 and normal HCO3; uncompensated metabolic acidosis has a decreased pH, normal pCO2 and normal HCO3; and uncompensated metabolic alkalosis has an increased pH, normal pCO2, and increased HCO3. 2. If the pH is decreased and the pCO2 is increased with a normal HCO3, it is uncompensated respiratory acidosis. Also, croup can be a disease process that causes respiratory acidosis. Uncompensated respiratory alkalosis has an increased pH, decreased pCO2 and normal HCO3; uncompensated metabolic acidosis has a decreased pH, normal pCO2 and normal HCO3; and uncompensated metabolic alkalosis has an increased pH, normal pCO2, and increased HCO3. 3. If the pH is decreased and the pCO2 is increased with a normal HCO3, it is uncompensated respiratory acidosis. Also, croup can be a disease process that causes respiratory acidosis. Uncompensated respiratory alkalosis has an increased pH, decreased pCO2 and normal HCO3; uncompensated metabolic acidosis has a decreased pH, normal pCO2 and normal HCO3; and uncompensated metabolic alkalosis has an increased pH, normal pCO2, and increased HCO3. 4. If the pH is decreased and the pCO2 is increased with a normal HCO3, it is uncompensated respiratory acidosis. Also, croup can be a disease process that causes respiratory acidosis. Uncompensated respiratory alkalosis has an increased pH, decreased pCO2 and normal HCO3; uncompensated metabolic acidosis has a decreased pH, normal pCO2 and normal HCO3; and uncompensated metabolic alkalosis has an increased pH, normal pCO2, and increased HCO3. Page Ref: 434-435 Cognitive Level: Analyzing Client Need &Sub: Physiological Integrity: Physiological Adaptation Standards: QSEN Competencies: Patient-centered care | AACN Essential Competencies: Essential IX: Baccalaureate generalist nursing practice | NLN Competencies: Nursing judgement | Nursing/Integrated Concepts: Nursing Process: Planning/Coordination of care Learning Outcome: LO 18.5 Analyze assessment findings to recognize fluid-electrolyte problems and acid—base imbalance in children. MNL LO: Endocrine and Metabolic Disorders/Apply the nursing process in providing care for the child and family. 15) The nurse is completing the intake and output record for a preschool-age client admitted for fluid volume deficit. The client has had the following intake and output during the shift: Intake: 4 oz of Pedialyte 1/2 of an 8-oz cup of clear orange Jell-O 2 graham crackers 200 mL of D 5-1/2 sodium chloride IV Output: 345 mL of urine 50 mL of loose stool The nurse documents the client's intake as milliliters. Round the answer to the nearest whole number. Answer: 440 Explanation: Pedialyte, Jell-O and IV fluid would be calculated for intake. The child has had 240 mL orally and 200 mL intravenously for a total of 440. Page Ref: 406-412 Cognitive Level: Applying Client Need &Sub: Physiological Integrity: Pharmacological and Parenteral Therapies Standards: QSEN Competencies: Patient-centered care | AACN Essential Competencies: Essential IX: Baccalaureate generalist nursing practice | NLN Competencies: Nursing judgement | Nursing/Integrated Concepts: Nursing Process: Implementation/Coordination of care Learning Outcome: LO 18.6 Plan appropriate nursing interventions for children experiencing fluid-electrolyte problems and acid—base imbalance. MNL LO: Endocrine and Metabolic Disorders/Differentiate treatment options and strategies for continuity of care for the child and family. 16) The nurse educator is preparing an in-service for new RNs hired on a general pediatric unit regarding normal fluid and electrolyte status for children at various ages. Which statements will the educator include about normal fluid and electrolyte status of an infant? Select all that apply. 1. The infant has 75 percent total body water. 2. The extracellular fluid accounts for 25 percent of total body water in the infant. 3. A high metabolic rate requires generous fluid intake for the infant. 4. The infant's kidneys are mature and able to conserve water and electrolytes. 5. The infant's high body surface area promotes fluid loss. Answer: 2, 3, 5 Explanation: 1. The nurse educator would include the following statements in the in-service: the extracellular fluid accounts for 25 percent of total body water in the infant; a high metabolic rate requires generous fluid intake for the infant; and the infant's high body surface area promotes fluid loss. All of these statements are true and accurate. The newborn, not the infant, has 75 percent total body water. All clients under the age of two years have immature kidney and are unable to conserve water and electrolytes. 2. The nurse educator would include the following statements in the in-service: the extracellular fluid accounts for 25 percent of total body water in the infant; a high metabolic rate requires generous fluid intake for the infant; and the infant's high body surface area promotes fluid loss. All of these statements are true and accurate. The newborn, not the infant, has 75 percent total body water. All clients under the age of two years have immature kidney and are unable to conserve water and electrolytes. 3. The nurse educator would include the following statements in the in-service: the extracellular fluid accounts for 25 percent of total body water in the infant; a high metabolic rate requires generous fluid intake for the infant; and the infant's high body surface area promotes fluid loss. All of these statements are true and accurate. The newborn, not the infant, has 75 percent total body water. All clients under the age of two years have immature kidney and are unable to conserve water and electrolytes. 4. The nurse educator would include the following statements in the in-service: the extracellular fluid accounts for 25 percent of total body water in the infant; a high metabolic rate requires generous fluid intake for the infant; and the infant's high body surface area promotes fluid loss. All of these statements are true and accurate. The newborn, not the infant, has 75 percent total body water. All clients under the age of two years have immature kidney and are unable to conserve water and electrolytes. 5. The nurse educator would include the following statements in the in-service: the extracellular fluid accounts for 25 percent of total body water in the infant; a high metabolic rate requires generous fluid intake for the infant; and the infant's high body surface area promotes fluid loss. All of these statements are true and accurate. The newborn, not the infant, has 75 percent total body water. All clients under the age of two years have immature kidney and are unable to conserve water and electrolytes. Page Ref: 409-410 Cognitive Level: Applying Client Need &Sub: Physiological Integrity: Physiological Adaptation Standards: QSEN Competencies: Patient-centered care | AACN Essential Competencies: Essential VII: Clinical prevention and population health | NLN Competencies: Nursing judgement | Nursing/Integrated Concepts: Nursing Process: Implementation/Health teaching and health promotion Learning Outcome: LO 18.1 Describe normal fluid and electrolyte status for children at various ages. MNL LO: Endocrine and Metabolic Disorders/Examine etiology, risk factors, pathophysiology, and clinical manifestations as seen in children. 17) The nurse is planning an in-service for new RNs who will be working on a general pediatric unit. Which statements are appropriate to include when discussing normal acid-base balance? Select all that apply. 1. The lungs are responsible for excreting excess carbonic acid from body. 2. The lungs reabsorb filtered bicarbonate. 3. The kidneys form bicarbonate if needed to restore balance. 4. The liver forms bicarbonate if needed to restore balance. 5. The liver synthesizes proteins needed to maintain osmotic pressure in the fluid compartments. Answer: 1, 3, 5 Explanation: 1. Statements that the nurse educator will include in the in-service include: the lungs are responsible for excreting excess carbonic acid from body; the kidneys form bicarbonate if needed to restore balance; and the liver synthesizes proteins needed to maintain osmotic pressure in the fluid compartments. The kidneys, not the lungs, reabsorb filtered bicarbonate. The kidneys, not the liver, form bicarbonate to restore balance, if needed. 2. Statements that the nurse educator will include in the in-service include: the lungs are responsible for excreting excess carbonic acid from body; the kidneys form bicarbonate if needed to restore balance; and the liver synthesizes proteins needed to maintain osmotic pressure in the fluid compartments. The kidneys, not the lungs, reabsorb filtered bicarbonate. The kidneys, not the liver, form bicarbonate to restore balance, if needed. 3. Statements that the nurse educator will include in the in-service include: the lungs are responsible for excreting excess carbonic acid from body; the kidneys form bicarbonate if needed to restore balance; and the liver synthesizes proteins needed to maintain osmotic pressure in the fluid compartments. The kidneys, not the lungs, reabsorb filtered bicarbonate. The kidneys, not the liver, form bicarbonate to restore balance, if needed. 4. Statements that the nurse educator will include in the in-service include: the lungs are responsible for excreting excess carbonic acid from body; the kidneys form bicarbonate if needed to restore balance; and the liver synthesizes proteins needed to maintain osmotic pressure in the fluid compartments. The kidneys, not the lungs, reabsorb filtered bicarbonate. The kidneys, not the liver, form bicarbonate to restore balance, if needed. 5. Statements that the nurse educator will include in the in-service include: the lungs are responsible for excreting excess carbonic acid from body; the kidneys form bicarbonate if needed to restore balance; and the liver synthesizes proteins needed to maintain osmotic pressure in the fluid compartments. The kidneys, not the lungs, reabsorb filtered bicarbonate. The kidneys, not the liver, form bicarbonate to restore balance, if needed. Page Ref: 433-435 Cognitive Level: Applying Client Need &Sub: Physiological Integrity: Physiological Adaptation Standards: QSEN Competencies: Patient-centered care | AACN Essential Competencies: Essential VII: Clinical prevention and population health | NLN Competencies: Nursing judgement | Nursing/Integrated Concepts: Nursing Process: Implementation/Health teaching and health promotion Learning Outcome: LO 18.4 Describe acid-base balance and recognize disruptions common in children. MNL LO: Endocrine and Metabolic Disorders/Examine etiology, risk factors, pathophysiology, and clinical manifestations as seen in children. 18) The school-age child is admitted to the hospital with dehydration. The child weighs 30 pounds. The physician orders: 50 mL/kg 0.9 percent NSS with 5 percent dextrose IV over 4 hours. Calculate the IV pump to infuse 50 mL/kg/4hrs. Supply on hand: 1000 mL 0.9 percent NSS/2.5 percent dextrose Answer: 170.4 mL/hr Explanation: Infuse 170.4 mL/hr Page Ref: 412-413 Cognitive Level: Analyzing Client Need &Sub: Physiological Integrity: Physiological Adaptation Standards: QSEN Competencies: Patient-centered care | AACN Essential Competencies: Essential VII: Clinical prevention and population health | NLN Competencies: Nursing judgement | Nursing/Integrated Concepts: Nursing Process: Implementation/Coordination of care Learning Outcome: LO 18.6 Plan appropriate nursing interventions for children experiencing fluid-electrolyte problems and acid—base imbalance. MNL LO: Demonstrate safe medication administration for the pediatric client. 19) Match the types of dehydration with their description. A. Isotonic dehydration B. Hypotonic dehydration C. Hypertonic dehydration 1. Occurs when fluid loss is characterized by a proportionately greater loss of sodium than water. 2. Occurs when fluid loss is characterized by a proportionately greater loss of water than sodium. 3. Occurs when fluid loss is not balanced by intake, and the loss of water and sodium are in proportion. Answer: 1/B, 2/C, 3/A 1. Hypotonic dehydration 2. Hypertonic dehydration 3. Isotonic dehydration Explanation: Isotonic dehydration: occurs when fluid loss is not balanced by intake, and the loss of water and sodium are in proportion. Hypotonic dehydration: occurs when fluid loss is characterized by a proportionately greater loss of sodium than water. Hypertonic dehydration: occurs when fluid loss is characterized by a proportionately greater loss of water than sodium. Page Ref: 406 Cognitive Level: Analyzing Client Need &Sub: Physiological Integrity: Physiological Adaptation Standards: QSEN Competencies: Patient-centered care | AACN Essential Competencies: Essential VII: Clinical prevention and population health | NLN Competencies: Nursing judgement | Nursing/Integrated Concepts: Nursing Process: Evaluation/Coordination of care Learning Outcome: LO 18.5 Analyze assessment findings to recognize fluid-electrolyte problems and acid—base imbalance in children. MNL LO: Educate the child and family on the care of the child during hospitalization and upon discharge. Principles of Pediatric Nursing: Caring for Children, 7e (Ball et al.) Chapter 19 Alterations in Eye, Ear, Nose, and Throat Function 1) A nurse is assessing infants for visually related developmental milestones. Which infant is showing a delay in meeting an expected milestone? 1. A 4-month-old who has a social smile 2. An 8-month-old who has just begun to inspect her own hand 3. A 12-month-old who stacks blocks 4. A 7-month-old who picks up a raisin by raking Answer: 2 Explanation: 1. An 8-month-old who has just begun to inspect her own hand is delayed. The infant usually inspects her own hand beginning at 3 months. A 4-month-old with a social smile, a 12-month-old who stacks blocks, and a 7-month-old who picks up a raisin by raking are all showing appropriate visually related milestones. 2. An 8-month-old who has just begun to inspect her own hand is delayed. The infant usually inspects her own hand beginning at 3 months. A 4-month-old with a social smile, a 12-month-old who stacks blocks, and a 7-month-old who picks up a raisin by raking are all showing appropriate visually related milestones. 3. An 8-month-old who has just begun to inspect her own hand is delayed. The infant usually inspects her own hand beginning at 3 months. A 4-month-old with a social smile, a 12-month-old who stacks blocks, and a 7-month-old who picks up a raisin by raking are all showing appropriate visually related milestones. 4. An 8-month-old who has just begun to inspect her own hand is delayed. The infant usually inspects her own hand beginning at 3 months. A 4-month-old with a social smile, a 12-month-old who stacks blocks, and a 7-month-old who picks up a raisin by raking are all showing appropriate visually related milestones. Page Ref: 444 Cognitive Level: Analyzing Client Need &Sub: Health Promotion and Maintenance Standards: QSEN Competencies: Patient-centered care | AACN Essential Competencies: Essential IX: Baccalaureate generalist nursing practice | NLN Competencies: Human flourishing | Nursing/Integrated Concepts: Nursing Process: Assessment/Coordination of care Learning Outcome: LO 19.1 Identify anatomy, physiology, and pediatric differences in the eye, ear, nose, and throat of children and adolescents. MNL LO: Neurological and Sensory Disorders/Examine etiology, risk factors, pathophysiology, and clinical manifestations as seen in children. 2) A neonate has been diagnosed with a herpes simplex viral infection of the eye. Which medication will the nurse prepare to administer? 1. Fluoroquinolone eye drops or ointment 2. Intravenous penicillin 3. Oral erythromycin 4. Parenteral acyclovir (Zovirax) and vidarabine (VIRA-A) ophthalmic ointment Answer: 4 Explanation: 1. Neonatal herpes simplex virus is treated vigorously with parenteral acyclovir for 14 days or longer and topical ophthalmic medication (trifluridine, iododeoxyuridine, or vidarabine). Fluoroquinolone eye drops are used to treat bacterial eye infections. Intravenous penicillin treats selected bacterial infections. Oral erythromycin is used to treat chlamydial eye infections. 2. Neonatal herpes simplex virus is treated vigorously with parenteral acyclovir for 14 days or longer and topical ophthalmic medication (trifluridine, iododeoxyuridine, or vidarabine). Fluoroquinolone eye drops are used to treat bacterial eye infections. Intravenous penicillin treats selected bacterial infections. Oral erythromycin is used to treat chlamydial eye infections. 3. Neonatal herpes simplex virus is treated vigorously with parenteral acyclovir for 14 days or longer and topical ophthalmic medication (trifluridine, iododeoxyuridine, or vidarabine). Fluoroquinolone eye drops are used to treat bacterial eye infections. Intravenous penicillin treats selected bacterial infections. Oral erythromycin is used to treat chlamydial eye infections. 4. Neonatal herpes simplex virus is treated vigorously with parenteral acyclovir for 14 days or longer and topical ophthalmic medication (trifluridine, iododeoxyuridine, or vidarabine). Fluoroquinolone eye drops are used to treat bacterial eye infections. Intravenous penicillin treats selected bacterial infections. Oral erythromycin is used to treat chlamydial eye infections. Page Ref: 442 Cognitive Level: Applying Client Need &Sub: Physiological Integrity: Pharmacological and Parenteral Therapies Standards: QSEN Competencies: Patient-centered care | AACN Essential Competencies: Essential IX: Baccalaureate generalist nursing practice | NLN Competencies: Nursing judgement | Nursing/Integrated Concepts: Nursing Process: Planning/Coordination of care Learning Outcome: LO 19.6 Integrate preventive and treatment principles when implementing health promotion for children related to eyes, ears, nose, and throat. MNL LO: Demonstrate safe medication administration for the pediatric client. 3) The nurse suspects that an infant has a visual disorder caused by abnormal musculature. Which test will the nurse perform to detect this disorder? 1. A cover/uncover test 2. An ophthalmologic exam 3. A vision-acuity exam 4. A pupil-reaction-to-light test Answer: 1 Explanation: 1. The cover/uncover test can detect abnormal musculature of the eye that can lead to asymmetric eye movement. An ophthalmologic eye exam allows the practitioner to view the internal structures of the eye, not abnormal musculature. A vision acuity test is used to test for myopia. A pupil-reaction-to-light test evaluates neurological status. 2. The cover/uncover test can detect abnormal musculature of the eye that can lead to asymmetric eye movement. An ophthalmologic eye exam allows the practitioner to view the internal structures of the eye, not abnormal musculature. A vision acuity test is used to test for myopia. A pupil-reaction-to-light test evaluates neurological status. 3. The cover/uncover test can detect abnormal musculature of the eye that can lead to asymmetric eye movement. An ophthalmologic eye exam allows the practitioner to view the internal structures of the eye, not abnormal musculature. A vision acuity test is used to test for myopia. A pupil-reaction-to-light test evaluates neurological status. 4. The cover/uncover test can detect abnormal musculature of the eye that can lead to asymmetric eye movement. An ophthalmologic eye exam allows the practitioner to view the internal structures of the eye, not abnormal musculature. A vision acuity test is used to test for myopia. A pupil-reaction-to-light test evaluates neurological status. Page Ref: 447 Cognitive Level: Applying Client Need &Sub: Health Promotion and Maintenance Standards: QSEN Competencies: Patient-centered care | AACN Essential Competencies: Essential IX: Baccalaureate generalist nursing practice | NLN Competencies: Nursing judgement | Nursing/Integrated Concepts: Nursing Process: Assessment/Coordination of care Learning Outcome: LO 19.3 Carry out screening programs to identify children with vision and hearing abnormalities. MNL LO: Neurological and Sensory Disorders/Examine etiology, risk factors, pathophysiology, and clinical manifestations as seen in children. 4) The nurse is caring for four clients. Which client has the highest risk of developing retinopathy of prematurity? 1. 30-week-gestation infant who was in an Oxy-Hood for 12 hours and weighed 1800 g. 2. 32-week-gestation infant who needed no oxygen and weighed 1850 g. 3. 28-week-gestation infant who has been on long-term oxygen and weighed 1400 g. 4. 28-week-gestation infant who was on short-term oxygen and weighed 1420 g. Answer: 3 Explanation: 1. The 28-week-gestation infant on oxygen weighing 1400 g has the highest risk of retinopathy of prematurity because of gestational age (28 weeks or less), weight (less than 1500 g), and oxygen therapy. The other neonates have fewer risk factors. 2. The 28-week-gestation infant on oxygen weighing 1400 g has the highest risk of retinopathy of prematurity because of gestational age (28 weeks or less), weight (less than 1500 g), and oxygen therapy. The other neonates have fewer risk factors. 3. The 28-week-gestation infant on oxygen weighing 1400 g has the highest risk of retinopathy of prematurity because of gestational age (28 weeks or less), weight (less than 1500 g), and oxygen therapy. The other neonates have fewer risk factors. 4. The 28-week-gestation infant on oxygen weighing 1400 g has the highest risk of retinopathy of prematurity because of gestational age (28 weeks or less), weight (less than 1500 g), and oxygen therapy. The other neonates have fewer risk factors. Page Ref: 449-451 Cognitive Level: Analyzing Client Need &Sub: Physiological Integrity: Physiological Adaptation Standards: QSEN Competencies: Patient-centered care | AACN Essential Competencies: Essential IX: Baccalaureate generalist nursing practice | NLN Competencies: Human flourishing | Nursing/Integrated Concepts: Nursing Process: Evaluation/Coordination of care Learning Outcome: LO 19.2 Describe abnormalities of the eyes, ears, nose, throat, and mouth in children. MNL LO: Neurological and Sensory Disorders/Examine etiology, risk factors, pathophysiology, and clinical manifestations as seen in children. 5) A nurse is caring for a visually impaired 20-month-old who has not begun to walk. Which nursing diagnosis is the most appropriate for this client? 1. Delayed growth and development 2. Impaired physical mobility 3. Self-care deficit 4. Impaired home maintenance Answer: 1 Explanation: 1. A 20-month-old child who is not walking is delayed in growth and development. The child's mobility is not due to a physiological problem, so impaired mobility is not appropriate. Self-care deficit does not apply to this age of child. There is not enough data to determine if home maintenance is impaired. 2. A 20-month-old child who is not walking is delayed in growth and development. The child's mobility is not due to a physiological problem, so impaired mobility is not appropriate. Self-care deficit does not apply to this age of child. There is not enough data to determine if home maintenance is impaired. 3. A 20-month-old child who is not walking is delayed in growth and development. The child's mobility is not due to a physiological problem, so impaired mobility is not appropriate. Self-care deficit does not apply to this age of child. There is not enough data to determine if home maintenance is impaired. 4. A 20-month-old child who is not walking is delayed in growth and development. The child's mobility is not due to a physiological problem, so impaired mobility is not appropriate. Self-care deficit does not apply to this age of child. There is not enough data to determine if home maintenance is impaired. Page Ref: 454 Cognitive Level: Applying Client Need &Sub: Physiological Integrity: Basic Care and Comfort Standards: QSEN Competencies: Patient-centered care | AACN Essential Competencies: Essential IX: Baccalaureate generalist nursing practice | NLN Competencies: Human flourishing | Nursing/Integrated Concepts: Nursing Process: Diagnosis/Coordination of care Learning Outcome: LO 19.1 Identify anatomy, physiology, and pediatric differences in the eye, ear, nose, and throat of children and adolescents. MNL LO: Neurological and Sensory Disorders/Examine etiology, risk factors, pathophysiology, and clinical manifestations as seen in children. 6) A nurse is caring for a visually impaired school-age child. Which nursing intervention is the highest priority for this child during the admission process? 1. Explaining playroom policies 2. Orienting the child to where furniture is placed in the room 3. Letting the child touch equipment that will be used during the hospitalization 4. Taking the child on a tour of the unit Answer: 2 Explanation: 1. The priority intervention is to orient the child to furniture placement in the room. This is priority because it addresses basic safety for a client with a visual impairment. Policies, handling equipment, and tours can be done at a later time. 2. The priority intervention is to orient the child to furniture placement in the room. This is priority because it addresses basic safety for a client with a visual impairment. Policies, handling equipment, and tours can be done at a later time. 3. The priority intervention is to orient the child to furniture placement in the room. This is priority because it addresses basic safety for a client with a visual impairment. Policies, handling equipment, and tours can be done at a later time. 4. The priority intervention is to orient the child to furniture placement in the room. This is priority because it addresses basic safety for a client with a visual impairment. Policies, handling equipment, and tours can be done at a later time. Page Ref: 455 Cognitive Level: Analyzing Client Need &Sub: Safe and Effective Care Environment: Safety and Infection Control Standards: QSEN Competencies: Patient-centered care | AACN Essential Competencies: Essential IX: Baccalaureate generalist nursing practice | NLN Competencies: Human flourishing | Nursing/Integrated Concepts: Nursing Process: Implementation/Coordination of care Learning Outcome: LO 19.6 Integrate preventive and treatment principles when implementing health promotion for children related to eyes, ears, nose, and throat. MNL LO: Neurological and Sensory Disorders/Educate the child and family on the care of the child during hospitalization and upon discharge. 7) An infant is diagnosed with acute otitis media. Which intervention is most appropriate for the nurse to teach the infant's parents? 1. Keep the baby in a flat lying position during sleep. 2. Administer acetaminophen (Tylenol) to relieve discomfort. 3. Administer a decongestant. 4. Place baby to sleep with a pacifier. Answer: 2 Explanation: 1. An infant with a bulging tympanic membrane because of acute otitis media will have pain. Parents are taught to administer acetaminophen (Tylenol) to relieve the discomfort associated with acute otitis media. A flat lying position may exacerbate the discomfort. Elevating the head slightly is recommended. Decongestants are not recommended for treatment of acute otitis media. Placing infants to sleep with a pacifier may increase the incidence of otitis media. 2. An infant with a bulging tympanic membrane because of acute otitis media will have pain. Parents are taught to administer acetaminophen (Tylenol) to relieve the discomfort associated with acute otitis media. A flat lying position may exacerbate the discomfort. Elevating the head slightly is recommended. Decongestants are not recommended for treatment of acute otitis media. Placing infants to sleep with a pacifier may increase the incidence of otitis media. 3. An infant with a bulging tympanic membrane because of acute otitis media will have pain. Parents are taught to administer acetaminophen (Tylenol) to relieve the discomfort associated with acute otitis media. A flat lying position may exacerbate the discomfort. Elevating the head slightly is recommended. Decongestants are not recommended for treatment of acute otitis media. Placing infants to sleep with a pacifier may increase the incidence of otitis media. 4. An infant with a bulging tympanic membrane because of acute otitis media will have pain. Parents are taught to administer acetaminophen (Tylenol) to relieve the discomfort associated with acute otitis media. A flat lying position may exacerbate the discomfort. Elevating the head slightly is recommended. Decongestants are not recommended for treatment of acute otitis media. Placing infants to sleep with a pacifier may increase the incidence of otitis media. Page Ref: 456-459 Cognitive Level: Applying Client Need &Sub: Physiological Integrity: Basic Care and Comfort Standards: QSEN Competencies: Patient-centered care | AACN Essential Competencies: Essential VII: Clinical prevention and population health | NLN Competencies: Nursing judgement | Nursing/Integrated Concepts: Nursing Process: Implementation/Health teaching and health promotion Learning Outcome: LO 19.5 Select and apply latest recommendations when implementing care and teaching for children with abnormalities of eyes, ears, nose, throat, and mouth. MNL LO: Neurological and Sensory Disorders/Differentiate treatment options and strategies for continuity of care for the child and family. 8) The nurse teaches parents how to care for their child who has tympanostomy tubes inserted. Which actions by the parents indicate appropriate understanding of the teaching session? Select all that apply. 1. Encouraging the child to drink generous amounts of fluids 2. Administering a decongestant for 1 to 2 weeks following surgery 3. Restricting the child to quiet activities after surgery 4. Limiting diet to soft, bland foods 5. Avoiding getting water in ears during bath time Answer: 1, 3, 5 Explanation: 1. The correct responses include encouraging the child to drink generous amount of water, restricting the child to quiet activities after surgery, and avoiding water in the child's ears at bath time. Incorrect responses include administering a decongestant for 1 to 2 weeks following surgery and limiting diet to soft, bland foods–decongestants are not needed after surgery, and a regular diet should be resumed. 2. The correct responses include encouraging the child to drink generous amount of water, restricting the child to quiet activities after surgery, and avoiding water in the child's ears at bath time. Incorrect responses include administering a decongestant for 1 to 2 weeks following surgery and limiting diet to soft, bland foods–decongestants are not needed after surgery, and a regular diet should be resumed. 3. The correct responses include encouraging the child to drink generous amount of water, restricting the child to quiet activities after surgery, and avoiding water in the child's ears at bath time. Incorrect responses include administering a decongestant for 1 to 2 weeks following surgery and limiting diet to soft, bland foods–decongestants are not needed after surgery, and a regular diet should be resumed. 4. The correct responses include encouraging the child to drink generous amount of water, restricting the child to quiet activities after surgery, and avoiding water in the child's ears at bath time. Incorrect responses include administering a decongestant for 1 to 2 weeks following surgery and limiting diet to soft, bland foods–decongestants are not needed after surgery, and a regular diet should be resumed. 5. The correct responses include encouraging the child to drink generous amount of water, restricting the child to quiet activities after surgery, and avoiding water in the child's ears at bath time. Incorrect responses include administering a decongestant for 1 to 2 weeks following surgery and limiting diet to soft, bland foods–decongestants are not needed after surgery, and a regular diet should be resumed. Page Ref: 458-459 Cognitive Level: Analyzing Client Need &Sub: Health Promotion and Maintenance Standards: QSEN Competencies: Patient-centered care | AACN Essential Competencies: Essential VII: Clinical prevention and population health | NLN Competencies: Nursing judgement | Nursing/Integrated Concepts: Nursing Process: Evaluation/Health teaching and health promotion Learning Outcome: LO 19.5 Select and apply latest recommendations when implementing care and teaching for children with abnormalities of eyes, ears, nose, throat, and mouth. MNL LO: Neurological and Sensory Disorders/Differentiate treatment options and strategies for continuity of care for the child and family. 9) Which action by the nurse can assist a child who has a mild hearing loss and reads lips to adapt to hospitalization? 1. Speaking directly to the parents for communication 2. Speaking in a loud voice while facing the child 3. Using a picture board as the main means of communication 4. Touching the child lightly before speaking Answer: 4 Explanation: 1. The nurse can facilitate hospital adaptation of a child who has a hearing loss and can lip-read by obtaining the child's visual attention by lightly touching the child before communicating. Speaking to the parents only does not help the child with the hospitalization. Speaking in a loud voice may not promote hearing in the child, and a picture board, while useful, should not be the primary means of communication for a child who reads lips. 2. The nurse can facilitate hospital adaptation of a child who has a hearing loss and can lip-read by obtaining the child's visual attention by lightly touching the child before communicating. Speaking to the parents only does not help the child with the hospitalization. Speaking in a loud voice may not promote hearing in the child, and a picture board, while useful, should not be the primary means of communication for a child who reads lips. 3. The nurse can facilitate hospital adaptation of a child who has a hearing loss and can lip-read by obtaining the child's visual attention by lightly touching the child before communicating. Speaking to the parents only does not help the child with the hospitalization. Speaking in a loud voice may not promote hearing in the child, and a picture board, while useful, should not be the primary means of communication for a child who reads lips. 4. The nurse can facilitate hospital adaptation of a child who has a hearing loss and can lip-read by obtaining the child's visual attention by lightly touching the child before communicating. Speaking to the parents only does not help the child with the hospitalization. Speaking in a loud voice may not promote hearing in the child, and a picture board, while useful, should not be the primary means of communication for a child who reads lips. Page Ref: 464-465 Cognitive Level: Applying Client Need &Sub: Psychosocial Integrity Standards: QSEN Competencies: Patient-centered care | AACN Essential Competencies: Essential IX: Baccalaureate generalist nursing practice | NLN Competencies: Human flourishing | Nursing/Integrated Concepts: Nursing Process: Implementation/Coordination of care Learning Outcome: LO 19.4 Plan nursing care for children with vision or hearing impairments. MNL LO: Neurological and Sensory Disorders/Apply the nursing process in providing care for the child and family. 10) A school-age child has epistaxis. Which intervention by the school nurse is the most appropriate? 1. Tilting the child's head forward, squeezing the nares below the nasal bone, and applying ice to the nose 2. Tilting the child's head back, squeezing the bridge of the nose, and applying a warm, moist pack to the nose 3. Lying the child down and applying no pressure, ice, or warm pack 4. Immediately packing the nares with a cotton ball soaked with Neo-Synephrine Answer: 1 Explanation: 1. The correct initial treatment for a nosebleed is to tilt the head forward, squeeze the nares below the nasal bone for 10 to 15 minutes, and apply ice to the nose or back of the head. Tilting the child's head back may cause the blood to trickle down the throat. Warmth can cause an increase in bleeding because of vasodilation. Lying the child down without application of pressure to the nares may not stop the bleeding. A cotton ball soaked with Neo-Synephrine would only be used if the bleeding does not stop with pressure and ice. 2. The correct initial treatment for a nosebleed is to tilt the head forward, squeeze the nares below the nasal bone for 10 to 15 minutes, and apply ice to the nose or back of the head. Tilting the child's head back may cause the blood to trickle down the throat. Warmth can cause an increase in bleeding because of vasodilation. Lying the child down without application of pressure to the nares may not stop the bleeding. A cotton ball soaked with Neo-Synephrine would only be used if the bleeding does not stop with pressure and ice. 3. The correct initial treatment for a nosebleed is to tilt the head forward, squeeze the nares below the nasal bone for 10 to 15 minutes, and apply ice to the nose or back of the head. Tilting the child's head back may cause the blood to trickle down the throat. Warmth can cause an increase in bleeding because of vasodilation. Lying the child down without application of pressure to the nares may not stop the bleeding. A cotton ball soaked with Neo-Synephrine would only be used if the bleeding does not stop with pressure and ice. 4. The correct initial treatment for a nosebleed is to tilt the head forward, squeeze the nares below the nasal bone for 10 to 15 minutes, and apply ice to the nose or back of the head. Tilting the child's head back may cause the blood to trickle down the throat. Warmth can cause an increase in bleeding because of vasodilation. Lying the child down without application of pressure to the nares may not stop the bleeding. A cotton ball soaked with Neo-Synephrine would only be used if the bleeding does not stop with pressure and ice. Page Ref: 466-467 Cognitive Level: Applying Client Need &Sub: Physiological Integrity: Basic Care and Comfort Standards: QSEN Competencies: Patient-centered care | AACN Essential Competencies: Essential IX: Baccalaureate generalist nursing practice | NLN Competencies: Nursing judgement | Nursing/Integrated Concepts: Nursing Process: Implementation/Coordination of care Learning Outcome: LO 19.6 Integrate preventive and treatment principles when implementing health promotion for children related to eyes, ears, nose, and throat. MNL LO: Neurological and Sensory Disorders/Differentiate treatment options and strategies for continuity of care for the child and family. 11) A nurse is planning to teach school-age children about the common cold. Which information should the nurse include in the teaching session? 1. Vaccinations can prevent contraction of a nasopharyngitis virus. 2. Antibiotics will eliminate the nasopharyngitis virus. 3. Proper handwashing can prevent the spread of the infection. 4. Aspirin should be taken for alleviation of fever if the "common cold" is contracted. Answer: 3 Explanation: 1. Proper handwashing should be taught to school-age children to reduce the spread of the "common cold" virus. No vaccine can prevent the common cold. Antibiotics are not used to treat viral infections. Aspirin should not be taken for fever because of its association with Reye syndrome. 2. Proper handwashing should be taught to school-age children to reduce the spread of the "common cold" virus. No vaccine can prevent the common cold. Antibiotics are not used to treat viral infections. Aspirin should not be taken for fever because of its association with Reye syndrome. 3. Proper handwashing should be taught to school-age children to reduce the spread of the "common cold" virus. No vaccine can prevent the common cold. Antibiotics are not used to treat viral infections. Aspirin should not be taken for fever because of its association with Reye syndrome. 4. Proper handwashing should be taught to school-age children to reduce the spread of the "common cold" virus. No vaccine can prevent the common cold. Antibiotics are not used to treat viral infections. Aspirin should not be taken for fever because of its association with Reye syndrome. Page Ref: 468-469 Cognitive Level: Analyzing Client Need &Sub: Health Promotion and Maintenance Standards: QSEN Competencies: Patient-centered care | AACN Essential Competencies: Essential VII: Clinical prevention and population health | NLN Competencies: Human flourishing | Nursing/Integrated Concepts: Nursing Process: Planning/Health teaching and health promotion Learning Outcome: LO 19.6 Integrate preventive and treatment principles when implementing health promotion for children related to eyes, ears, nose, and throat. MNL LO: Neurological and Sensory Disorders/Differentiate treatment options and strategies for continuity of care for the child and family. 12) A child is diagnosed with group A beta-hemolytic streptococcus (GABHS) infection of the throat. Which item will the nurse include in the teaching plan for the parents? 1. Complete the entire course of antibiotics. 2. Keep the child NPO (nothing by mouth). 3. Continue normal activities. 4. Do not allow the child to gargle with saltwater. Answer: 1 Explanation: 1. It is important for parents to complete the entire course of antibiotics for GABHS infections. Nothing-by-mouth, or NPO, status is not recommended because the child needs to stay hydrated. The child should rest, and use of warm saltwater gargles is recommended. 2. It is important for parents to complete the entire course of antibiotics for GABHS infections. Nothing-by-mouth, or NPO, status is not recommended because the child needs to stay hydrated. The child should rest, and use of warm saltwater gargles is recommended. 3. It is important for parents to complete the entire course of antibiotics for GABHS infections. Nothing-by-mouth, or NPO, status is not recommended because the child needs to stay hydrated. The child should rest, and use of warm saltwater gargles is recommended. 4. It is important for parents to complete the entire course of antibiotics for GABHS infections. Nothing-by-mouth, or NPO, status is not recommended because the child needs to stay hydrated. The child should rest, and use of warm saltwater gargles is recommended. Page Ref: 469-470 Cognitive Level: Applying Client Need &Sub: Physiological Integrity: Pharmacological and Parenteral Therapies Standards: QSEN Competencies: Patient-centered care | AACN Essential Competencies: Essential VII: Clinical prevention and population health | NLN Competencies: Human flourishing | Nursing/Integrated Concepts: Nursing Process: Implementation/Health teaching and health promotion Learning Outcome: LO 19.6 Integrate preventive and treatment principles when implementing health promotion for children related to eyes, ears, nose, and throat. MNL LO: Neurological and Sensory Disorders/Differentiate treatment options and strategies for continuity of care for the child and family. 13) The nurse completes postoperative discharge teaching to the parents of a child who had a tonsillectomy. Which statement by the parents indicates correct understanding of the teaching session? 1. "We will call the physician for any indication of ear pain." 2. "We will plan on administering acetaminophen (Tylenol) for pain." 3. "We will be sure to give our child adequate amounts of citrus juices." 4. "We will keep our child on bed rest for 10 days after the surgery." Answer: 2 Explanation: 1. Acetaminophen (Tylenol) is recommended for pain after a tonsillectomy. Citrus juices should be avoided for the first week because highly acidic foods and beverages can cause irritation. Ear pain 4 to 8 days after a tonsillectomy may be experienced and does not indicate an ear infection. Children do not need to be confined to bed. They can return to school in 10 days. 2. Acetaminophen (Tylenol) is recommended for pain after a tonsillectomy. Citrus juices should be avoided for the first week because highly acidic foods and beverages can cause irritation. Ear pain 4 to 8 days after a tonsillectomy may be experienced and does not indicate an ear infection. Children do not need to be confined to bed. They can return to school in 10 days. 3. Acetaminophen (Tylenol) is recommended for pain after a tonsillectomy. Citrus juices should be avoided for the first week because highly acidic foods and beverages can cause irritation. Ear pain 4 to 8 days after a tonsillectomy may be experienced and does not indicate an ear infection. Children do not need to be confined to bed. They can return to school in 10 days. 4. Acetaminophen (Tylenol) is recommended for pain after a tonsillectomy. Citrus juices should be avoided for the first week because highly acidic foods and beverages can cause irritation. Ear pain 4 to 8 days after a tonsillectomy may be experienced and does not indicate an ear infection. Children do not need to be confined to bed. They can return to school in 10 days. Page Ref: 471 Cognitive Level: Analyzing Client Need &Sub: Physiological Integrity: Pharmacological and Parenteral Therapies Standards: QSEN Competencies: Patient-centered care | AACN Essential Competencies: Essential VII: Clinical prevention and population health | NLN Competencies: Nursing judgement | Nursing/Integrated Concepts: Nursing Process: Evaluation/Health teaching and health promotion Learning Outcome: LO 19.5 Select and apply latest recommendations when implementing care and teaching for children with abnormalities of eyes, ears, nose, throat, and mouth. MNL LO: Neurological and Sensory Disorders/Differentiate treatment options and strategies for continuity of care for the child and family. 14) The child who had a tonsillectomy earlier today is now awake and tolerating fluids. The child asks for something to eat. Which food choice is most appropriate for this client? 1. Orange slices 2. Lemonade 3. Grapefruit juice 4. Applesauce Answer: 4 Explanation: 1. Soft foods such as applesauce can be added as tolerated to a diet following a tonsillectomy. Citric juices or citric fruits should be avoided because they may cause a burning sensation in the throat. 2. Soft foods such as applesauce can be added as tolerated to a diet following a tonsillectomy. Citric juices or citric fruits should be avoided because they may cause a burning sensation in the throat. 3. Soft foods such as applesauce can be added as tolerated to a diet following a tonsillectomy. Citric juices or citric fruits should be avoided because they may cause a burning sensation in the throat. 4. Soft foods such as applesauce can be added as tolerated to a diet following a tonsillectomy. Citric juices or citric fruits should be avoided because they may cause a burning sensation in the throat. Page Ref: 472 Cognitive Level: Applying Client Need &Sub: Physiological Integrity: Basic Care and Comfort Standards: QSEN Competencies: Patient-centered care | AACN Essential Competencies: Essential IX: Baccalaureate generalist nursing practice | NLN Competencies: Nursing judgement | Nursing/Integrated Concepts: Nursing Process: Implementation/Coordination of care Learning Outcome: LO 19.6 Integrate preventive and treatment principles when implementing health promotion for children related to eyes, ears, nose, and throat. MNL LO: Neurological and Sensory Disorders/Differentiate treatment options and strategies for continuity of care for the child and family. 15) During an admission assessment, the nurse notes that the child has impaired oral mucous membranes. Which intervention is most appropriate for the nurse to implement for this child? 1. Administering topical analgesics 2. Promoting an adequate intake of nutrients 3. Administering antibiotics as ordered 4. Using lemon and glycerin for oral hygiene Answer: 2 Explanation: 1. Adequate intake of fluids and nutrients promotes the intactness of the oral mucosal membrane tissue, which is the desired outcome for an impaired oral mucous membrane problem. Lemon and glycerin may dry the oral mucous membrane, which is not desirable. Administration of antibiotics or topical analgesics are medical interventions that might be performed but do not ensure that impaired tissue will be resolved. 2. Adequate intake of fluids and nutrients promotes the intactness of the oral mucosal membrane tissue, which is the desired outcome for an impaired oral mucous membrane problem. Lemon and glycerin may dry the oral mucous membrane, which is not desirable. Administration of antibiotics or topical analgesics are medical interventions that might be performed but do not ensure that impaired tissue will be resolved. 3. Adequate intake of fluids and nutrients promotes the intactness of the oral mucosal membrane tissue, which is the desired outcome for an impaired oral mucous membrane problem. Lemon and glycerin may dry the oral mucous membrane, which is not desirable. Administration of antibiotics or topical analgesics are medical interventions that might be performed but do not ensure that impaired tissue will be resolved. 4. Adequate intake of fluids and nutrients promotes the intactness of the oral mucosal membrane tissue, which is the desired outcome for an impaired oral mucous membrane problem. Lemon and glycerin may dry the oral mucous membrane, which is not desirable. Administration of antibiotics or topical analgesics are medical interventions that might be performed but do not ensure that impaired tissue will be resolved. Page Ref: 470-471 Cognitive Level: Applying Client Need &Sub: Physiological Integrity: Basic Care and Comfort Standards: QSEN Competencies: Patient-centered care | AACN Essential Competencies: Essential IX: Baccalaureate generalist nursing practice | NLN Competencies: Human flourishing | Nursing/Integrated Concepts: Nursing Process: Implementation/Coordination of care Learning Outcome: LO 19.6 Integrate preventive and treatment principles when implementing health promotion for children related to eyes, ears, nose, and throat. MNL LO: Neurological and Sensory Disorders/Differentiate treatment options and strategies for continuity of care for the child and family. 16) A 2-year-old child is seen in the clinic with swelling in the eyelid, mattering and difficulty opening the eye in the morning, the healthcare provider is ordering an antibiotic for bacterial conjunctivitis. What organisms could be causing this infection? Select all that apply. 1. Staphylococcus aureus 2. Pneumococcal pneumoniae 3. Haemophilus influenza 4. Streptococcus pneumoniae 5. Moraxella catarrhalis Answer: 1, 3, 4, 5 Explanation: 1. Common infectious organisms in bacterial conjunctivitis include: S. aureus, H. influenza, S. pneumonia, and M. catarrhalis. 2. Common infectious organisms in bacterial conjunctivitis include: S. aureus, H. influenza, S. pneumonia, and M. catarrhalis. 3. This is a common cause of bacterial conjunctivitis. 4. Common infectious organisms in bacterial conjunctivitis include: S. aureus, H. influenza, S. pneumonia, and M. catarrhalis. 5. Common infectious organisms in bacterial conjunctivitis include: S. aureus, H. influenza, S. pneumonia, and M. catarrhalis. Page Ref: 442 Cognitive Level: Analyzing Client Need &Sub: Safe and Effective Care Environment: Management of Care Standards: QSEN Competencies: Patient-centered care | AACN Essential Competencies: Essential IX: Baccalaureate generalist nursing practice | NLN Competencies: Human flourishing | Nursing/Integrated Concepts: Nursing Process: Diagnosis/Coordination of care Learning Outcome: LO 19.2 Describe abnormalities of the eyes, ears, nose, throat, and mouth in children. MNL LO: Immunity, Infection, and Communicable Disease/Examine etiology, risk factors, pathophysiology, and clinical manifestations as seen in children. 17) A 22-month-old child is seen in the office for reoccurring otitis media. The child weighs 25 pounds. The healthcare provider orders: Amoxicillin 45 mg/kg/day by mouth in divided doses every 12 hours. Medication on hand: Amoxicillin 125 mg/5 mL Calculate how many mL/dose of amoxicillin will be given by mouth. Answer: 10.2 or 10 mL/dose Explanation: 10.2 or 10 mL/dose Page Ref: 456-457 Cognitive Level: Analyzing Client Need &Sub: Physiological Integrity: Pharmacological and Parenteral Therapies Standards: QSEN Competencies: Patient-centered care | AACN Essential Competencies: Essential IX: Baccalaureate generalist nursing practice | NLN Competencies: Human flourishing | Nursing/Integrated Concepts: Nursing Process: Planning/Coordination of care Learning Outcome: LO 19.2 Describe abnormalities of the eyes, ears, nose, throat, and mouth in children. MNL LO: Immunity, Infection, and Communicable Disease/Examine etiology, risk factors, pathophysiology, and clinical manifestations as seen in children. Principles of Pediatric Nursing: Caring for Children, 7e (Ball et al.) Chapter 20 Alterations in Respiratory Function 1) A child is showing signs of acute respiratory distress. Which position will the nurse place this child? 1. Upright 2. Side-lying 3. Flat 4. In semi-Fowler's Answer: 1 Explanation: 1. Upright is correct because it allows for optimal chest expansion. Side-lying, flat, and semi-Fowler's (head up slightly) do not allow for as optimal chest expansion as the upright position. 2. Upright is correct because it allows for optimal chest expansion. Side-lying, flat, and semi- Fowler's (head up slightly) do not allow for as optimal chest expansion as the upright position. 3. Upright is correct because it allows for optimal chest expansion. Side-lying, flat, and semi- Fowler's (head up slightly) do not allow for as optimal chest expansion as the upright position. 4. Upright is correct because it allows for optimal chest expansion. Side-lying, flat, and semi- Fowler's (head up slightly) do not allow for as optimal chest expansion as the upright position. Page Ref: 481 Cognitive Level: Applying Client Need &Sub: Physiological Integrity: Physiological Adaptation Standards: QSEN Competencies: Patient-centered care | AACN Essential Competencies: Essential IX: Baccalaureate generalist nursing practice | NLN Competencies: Human flourishing | Nursing/Integrated Concepts: Nursing Process: Implementation/Coordination of care Learning Outcome: LO 20.6 Create a nursing care plan for a child with a common acute respiratory condition. MNL LO: Respiratory Disorders/Apply the nursing process in providing care for the child and family. 2) A nurse delegates the task of neonatal vital-sign assessment to a nurse technician. Which instruction will the nurse give to the technician prior to assign care? 1. Report any neonate using abdominal muscles to breathe. 2. Report any neonate with apnea for 10 seconds. 3. Count respirations for 15 seconds and multiply by 4 to get the rate for 1 minute. 4. Report any neonate with a breathing pause that lasts 20 seconds or longer. Answer: 4 Explanation: 1. The abnormal assessment finding for vital signs that the nurse should instruct a nurse technician to report is any breathing pause by a neonate lasting longer than 20 seconds. This can indicate apnea and could lead to an apparent life-threatening event (ALTE). A breathing pause of 10 seconds or less is called periodic breathing and is a normal pattern for a neonate. Respirations should be counted for 1 minute, not 15 seconds. It is normal for neonates to use abdominal muscles for breathing. 2. The abnormal assessment finding for vital signs that the nurse should instruct a nurse technician to report is any breathing pause by a neonate lasting longer than 20 seconds. This can indicate apnea and could lead to an apparent life-threatening event (ALTE). A breathing pause of 10 seconds or less is called periodic breathing and is a normal pattern for a neonate. Respirations should be counted for 1 minute, not 15 seconds. It is normal for neonates to use abdominal muscles for breathing. 3. The abnormal assessment finding for vital signs that the nurse should instruct a nurse technician to report is any breathing pause by a neonate lasting longer than 20 seconds. This can indicate apnea and could lead to an apparent life-threatening event (ALTE). A breathing pause of 10 seconds or less is called periodic breathing and is a normal pattern for a neonate. Respirations should be counted for 1 minute, not 15 seconds. It is normal for neonates to use abdominal muscles for breathing. 4. The abnormal assessment finding for vital signs that the nurse should instruct a nurse technician to report is any breathing pause by a neonate lasting longer than 20 seconds. This can indicate apnea and could lead to an apparent life-threatening event (ALTE). A breathing pause of 10 seconds or less is called periodic breathing and is a normal pattern for a neonate. Respirations should be counted for 1 minute, not 15 seconds. It is normal for neonates to use abdominal muscles for breathing. Page Ref: 482-483 Cognitive Level: Analyzing Client Need &Sub: Safe and Effective Care Environment: Management of Care Standards: QSEN Competencies: Teamwork and collaboration | AACN Essential Competencies: Essential VI: Interprofessional communication and collaboration for improving patient health outcomes | NLN Competencies: Professional identity | Nursing/Integrated Concepts: Nursing Process: Assessment/Collaboration Learning Outcome: LO 20.2 Contrast the different respiratory medical conditions that can cause respiratory distress in infants and children. MNL LO: Respiratory Disorders/Examine etiology, risk factors, pathophysiology, and clinical manifestations as seen in children. 3) A child is admitted to the hospital with the diagnosis of laryngotracheobronchitis (LTB). Which nursing intervention is the priority for this child? 1. Administer nebulized epinephrine and oral or IM dexamethasone. 2. Administer antibiotics and assist with possible intubation. 3. Swab the throat for a throat culture. 4. Obtain a sputum specimen. Answer: 1 Explanation: 1. Nebulized epinephrine and dexamethasone are given for LTB. Antibiotic administration and possible intubation are associated with epiglottitis. Throat cultures are not obtained for LTB because it is viral and swabbing the throat could cause complete obstruction to occur. Sputum specimens will not assist in the diagnosis of LTB. 2. Nebulized epinephrine and dexamethasone are given for LTB. Antibiotic administration and possible intubation are associated with epiglottitis. Throat cultures are not obtained for LTB because it is viral and swabbing the throat could cause complete obstruction to occur. Sputum specimens will not assist in the diagnosis of LTB. 3. Nebulized epinephrine and dexamethasone are given for LTB. Antibiotic administration and possible intubation are associated with epiglottitis. Throat cultures are not obtained for LTB because it is viral and swabbing the throat could cause complete obstruction to occur. Sputum specimens will not assist in the diagnosis of LTB. 4. Nebulized epinephrine and dexamethasone are given for LTB. Antibiotic administration and possible intubation are associated with epiglottitis. Throat cultures are not obtained for LTB because it is viral and swabbing the throat could cause complete obstruction to occur. Sputum specimens will not assist in the diagnosis of LTB. Page Ref: 487 Cognitive Level: Analyzing Client Need &Sub: Physiological Integrity: Pharmacological and Parenteral Therapies Standards: QSEN Competencies: Patient-centered care | AACN Essential Competencies: Essential IX: Baccalaureate generalist nursing practice | NLN Competencies: Nursing judgement | Nursing/Integrated Concepts: Nursing Process: Planning/Coordination of care Learning Outcome: LO 20.6 Create a nursing care plan for a child with a common acute respiratory condition. MNL LO: Respiratory Disorders/Apply the nursing process in providing care for the child and family. 4) The nurse is teaching a group of mothers of infants about the benefits of immunization. Which immunization will the nurse teach to the mothers that can assist in preventing the life-threatening disease epiglottitis? 1. Measles, mumps, and rubella (MMR) 2. Haemophilus influenzae type B (HIB) 3. Hepatitis B 4. Polio Answer: 2 Explanation: 1. The Haemophilus influenzae type B (HIB) immunization can assist in prevention of epiglottitis. Hepatitis B, measles, mumps, rubella, and the polio virus are not causative agents for epiglottitis. 2. The Haemophilus influenzae type B (HIB) immunization can assist in prevention of epiglottitis. Hepatitis B, measles, mumps, rubella, and the polio virus are not causative agents for epiglottitis. 3. The Haemophilus influenzae type B (HIB) immunization can assist in prevention of epiglottitis. Hepatitis B, measles, mumps, rubella, and the polio virus are not causative agents for epiglottitis. 4. The Haemophilus influenzae type B (HIB) immunization can assist in prevention of epiglottitis. Hepatitis B, measles, mumps, rubella, and the polio virus are not causative agents for epiglottitis. Page Ref: 486-487 Cognitive Level: Analyzing Client Need &Sub: Health Promotion and Maintenance Standards: QSEN Competencies: Patient-centered care | AACN Essential Competencies: Essential VII: Clinical prevention and population health | NLN Competencies: Nursing judgement | Nursing/Integrated Concepts: Nursing Process: Planning/Health teaching and health promotion Learning Outcome: LO 20.2 Contrast the different respiratory medical conditions that can cause respiratory distress in infants and children. MNL LO: Respiratory Disorders/Examine etiology, risk factors, pathophysiology, and clinical manifestations as seen in children. 5) A nurse is assessing a neonate. Which assessment finding indicates that the neonate's respiratory status is worsening? 1. Acrocyanosis 2. Arterial CO2 of 40 3. Periorbital edema 4. Grunting respirations with nasal flaring Answer: 4 Explanation: 1. Grunting respirations with nasal flaring indicates respiratory status is becoming worse. Acrocyanosis (cyanosis of the extremities) is a normal finding in a neonate. CO2 of 40 is within a normal range. Periorbital edema does not necessarily mean deterioration in respiratory status. 2. Grunting respirations with nasal flaring indicates respiratory status is becoming worse. Acrocyanosis (cyanosis of the extremities) is a normal finding in a neonate. CO2 of 40 is within a normal range. Periorbital edema does not necessarily mean deterioration in respiratory status. 3. Grunting respirations with nasal flaring indicates respiratory status is becoming worse. Acrocyanosis (cyanosis of the extremities) is a normal finding in a neonate. CO2 of 40 is within a normal range. Periorbital edema does not necessarily mean deterioration in respiratory status. 4. Grunting respirations with nasal flaring indicates respiratory status is becoming worse. Acrocyanosis (cyanosis of the extremities) is a normal finding in a neonate. CO2 of 40 is within a normal range. Periorbital edema does not necessarily mean deterioration in respiratory status. Page Ref: 487-488 Cognitive Level: Analyzing Client Need &Sub: Physiological Integrity: Physiological Adaptation Standards: QSEN Competencies: Patient-centered care | AACN Essential Competencies: Essential IX: Baccalaureate generalist nursing practice | NLN Competencies: Nursing judgement | Nursing/Integrated Concepts: Nursing Process: Assessment/Coordination of care Learning Outcome: LO 20.9 Contrast the signs of different injuries to the respiratory system. MNL LO: Respiratory Disorders/Examine etiology, risk factors, pathophysiology, and clinical manifestations as seen in children. 6) Which nursing diagnosis is most appropriate for an infant with acute bronchiolitis due to respiratory syncytial virus (RSV)? 1. Activity Intolerance 2. Decreased Cardiac Output 3. Pain, Acute 4. Tissue Perfusion, Ineffective (peripheral) Answer: 1 Explanation: 1. Activity intolerance is a problem because of the imbalance between oxygen supply and demand. Cardiac output is not compromised during an acute phase of bronchiolitis. Pain is not usually associated with acute bronchiolitis. Tissue perfusion (peripheral) is not affected by this respiratory-disease process. 2. Activity intolerance is a problem because of the imbalance between oxygen supply and demand. Cardiac output is not compromised during an acute phase of bronchiolitis. Pain is not usually associated with acute bronchiolitis. Tissue perfusion (peripheral) is not affected by this respiratory-disease process. 3. Activity intolerance is a problem because of the imbalance between oxygen supply and demand. Cardiac output is not compromised during an acute phase of bronchiolitis. Pain is not usually associated with acute bronchiolitis. Tissue perfusion (peripheral) is not affected by this respiratory-disease process. 4. Activity intolerance is a problem because of the imbalance between oxygen supply and demand. Cardiac output is not compromised during an acute phase of bronchiolitis. Pain is not usually associated with acute bronchiolitis. Tissue perfusion (peripheral) is not affected by this respiratory-disease process. Page Ref: 490 Cognitive Level: Analyzing Client Need &Sub: Physiological Integrity: Basic Care and Comfort Standards: QSEN Competencies: Patient-centered care | AACN Essential Competencies: Essential IX: Baccalaureate generalist nursing practice | NLN Competencies: Nursing judgement | Nursing/Integrated Concepts: Nursing Process: Diagnosis/Coordination of care Learning Outcome: LO 20.2 Contrast the different respiratory medical conditions that can cause respiratory distress in infants and children. MNL LO: Respiratory Disorders/Examine etiology, risk factors, pathophysiology, and clinical manifestations as seen in children. 7) A child is admitted to the hospital with pneumonia. The child's oximetry reading is 88 percent upon admission to the pediatric floor. Which is the priority nursing intervention for this child? 1. Obtain a blood sample to send to the lab for electrolyte analysis. 2. Begin oxygen per nasal cannula. 3. Medicate for pain. 4. Begin administration of intravenous fluids. Answer: 2 Explanation: 1. Pulse oximetry reading should be 92 or greater. Oxygen by nasal cannula should be started initially. Medicating for pain, administering IV fluids, and sending lab specimens can be done once the child's oxygenation status has been addressed. 2. Pulse oximetry reading should be 92 or greater. Oxygen by nasal cannula should be started initially. Medicating for pain, administering IV fluids, and sending lab specimens can be done once the child's oxygenation status has been addressed. 3. Pulse oximetry reading should be 92 or greater. Oxygen by nasal cannula should be started initially. Medicating for pain, administering IV fluids, and sending lab specimens can be done once the child's oxygenation status has been addressed. 4. Pulse oximetry reading should be 92 or greater. Oxygen by nasal cannula should be started initially. Medicating for pain, administering IV fluids, and sending lab specimens can be done once the child's oxygenation status has been addressed. Page Ref: 493 Cognitive Level: Applying Client Need &Sub: Safe and Effective Care Environment: Safety and Infection Control Standards: QSEN Competencies: Patient-centered care | AACN Essential Competencies: Essential IX: Baccalaureate generalist nursing practice | NLN Competencies: Nursing judgement | Nursing/Integrated Concepts: Nursing Process: Implementation/Coordination of care Learning Outcome: LO 20.4 Assess the child's respiratory status and analyze the need for oxygen supplementation. MNL LO: Respiratory Disorders/Apply the nursing process in providing care for the child and family. 8) The practitioner changes the medications for the child with asthma to salmeterol (Serevent). The mother asks the nurse what this drug will do. The nurse explains that salmeterol (Serevent) is used to treat asthma because the drug produces which characteristic? 1. Decreases inflammation 2. Decreases mucous production 3. Controls allergic rhinitis 4. Dilates the bronchioles Answer: 4 Explanation: 1. Salmeterol (Serevent) is a long-acting beta2-agonist that acts by bronchodilating. Steroids are anti-inflammatory, anticholinergics decrease mucous production, and antihistamines control allergic rhinitis. 2. Salmeterol (Serevent) is a long-acting beta2-agonist that acts by bronchodilating. Steroids are anti-inflammatory, anticholinergics decrease mucous production, and antihistamines control allergic rhinitis. 3. Salmeterol (Serevent) is a long-acting beta2-agonist that acts by bronchodilating. Steroids are anti-inflammatory, anticholinergics decrease mucous production, and antihistamines control allergic rhinitis. 4. Salmeterol (Serevent) is a long-acting beta2-agonist that acts by bronchodilating. Steroids are anti-inflammatory, anticholinergics decrease mucous production, and antihistamines control allergic rhinitis. Page Ref: 499-501 Cognitive Level: Analyzing Client Need &Sub: Physiological Integrity: Pharmacological and Parenteral Therapies Standards: QSEN Competencies: Patient-centered care | AACN Essential Competencies: Essential IX: Baccalaureate generalist nursing practice | NLN Competencies: Nursing judgement | Nursing/Integrated Concepts: Nursing Process: Implementation/Coordination of care Learning Outcome: LO 20.6 Create a nursing care plan for a child with a common acute respiratory condition. MNL LO: Respiratory Disorders/Apply the nursing process in providing care for the child and family. 9) Following parental teaching, the nurse is evaluating the parents' understanding of environmental control for their child's asthma management. Which statement by the parents indicates appropriate understanding of the teaching? 1. "We will replace the carpet in our child's bedroom with tile." 2. "We're glad the dog can continue to sleep in our child's room." 3. "We'll be sure to use the fireplace often to keep the house warm in the winter." 4. "We'll keep the plants in our child's room dusted." Answer: 1 Explanation: 1. Control of dust in the child's bedroom is an important aspect of environmental control for asthma management. When possible, pets and plants should not be kept in the home. Smoke from fireplaces should be eliminated. 2. Control of dust in the child's bedroom is an important aspect of environmental control for asthma management. When possible, pets and plants should not be kept in the home. Smoke from fireplaces should be eliminated. 3. Control of dust in the child's bedroom is an important aspect of environmental control for asthma management. When possible, pets and plants should not be kept in the home. Smoke from fireplaces should be eliminated. 4. Control of dust in the child's bedroom is an important aspect of environmental control for asthma management. When possible, pets and plants should not be kept in the home. Smoke from fireplaces should be eliminated. Page Ref: 506-507 Cognitive Level: Analyzing Client Need &Sub: Health Promotion and Maintenance Standards: QSEN Competencies: Patient-centered care | AACN Essential Competencies: Essential VII: Clinical prevention and population health | NLN Competencies: Nursing judgement | Nursing/Integrated Concepts: Nursing Process: Evaluation/Health teaching and health promotion Learning Outcome: LO 20.6 Create a nursing care plan for a child with a common acute respiratory condition. MNL LO: Respiratory Disorders/Apply the nursing process in providing care for the child and family. 10) A newborn is suspected of having cystic fibrosis. As the child is being prepared for transfer to a pediatric hospital, the mother asks the nurse which symptoms made the practitioner suspect cystic fibrosis. Which response by the nurse is the most appropriate? 1. Steatorrheic stools 2. Constipation 3. Meconium ileus 4. Rectal prolapse Answer: 3 Explanation: 1. Newborns with cystic fibrosis may present in the first 48 hours with meconium ileus. Steatorrhea, constipation, and rectal prolapse may be signs of cystic fibrosis seen in an older infant or child. 2. Newborns with cystic fibrosis may present in the first 48 hours with meconium ileus. Steatorrhea, constipation, and rectal prolapse may be signs of cystic fibrosis seen in an older infant or child. 3. Newborns with cystic fibrosis may present in the first 48 hours with meconium ileus. Steatorrhea, constipation, and rectal prolapse may be signs of cystic fibrosis seen in an older infant or child. 4. Newborns with cystic fibrosis may present in the first 48 hours with meconium ileus. Steatorrhea, constipation, and rectal prolapse may be signs of cystic fibrosis seen in an older infant or child. Page Ref: 508 Cognitive Level: Analyzing Client Need &Sub: Physiological Integrity: Physiological Adaptation Standards: QSEN Competencies: Patient-centered care | AACN Essential Competencies: Essential VII: Clinical prevention and population health | NLN Competencies: Nursing judgement | Nursing/Integrated Concepts: Nursing Process: Assessment/Health teaching and health promotion Learning Outcome: LO 20.2 Contrast the different respiratory medical conditions that can cause respiratory distress in infants and children. MNL LO: Respiratory Disorders/Examine etiology, risk factors, pathophysiology, and clinical manifestations as seen in children. 11) The nurse is teaching the parents of a newly diagnosed cystic fibrosis patient how to administer the pancreatic enzymes. How often will the nurse teach the parents to administer the enzymes? 1. Two times per day 2. With meals and snacks 3. Every 6 hours around the clock 4. Four times per day Answer: 2 Explanation: 1. Pancreatic enzymes are administered with meals and large snacks. A scheduled time would not be appropriate because the enzymes are used to assist in digestion of nutrients. 2. Pancreatic enzymes are administered with meals and large snacks. A scheduled time would not be appropriate because the enzymes are used to assist in digestion of nutrients. 3. Pancreatic enzymes are administered with meals and large snacks. A scheduled time would not be appropriate because the enzymes are used to assist in digestion of nutrients. 4. Pancreatic enzymes are administered with meals and large snacks. A scheduled time would not be appropriate because the enzymes are used to assist in digestion of nutrients. Page Ref: 509 Cognitive Level: Applying Client Need &Sub: Physiological Integrity: Pharmacological and Parenteral Therapies Standards: QSEN Competencies: Patient-centered care | AACN Essential Competencies: Essential VII: Clinical prevention and population health | NLN Competencies: Nursing judgement | Nursing/Integrated Concepts: Nursing Process: Implementation/Health teaching and health promotion Learning Outcome: LO 20.6 Create a nursing care plan for a child with a common acute respiratory condition. MNL LO: Respiratory Disorders/Apply the nursing process in providing care for the child and family. 12) A child with asthma will be receiving an oral dose of prednisone. The order reads prednisone 2 mg/kg per day. The child weighs 50 lbs. The child will receive _ milligrams daily. (Round the answer.) Round the answer to the nearest whole number. Answer: 45.5 = 46 Explanation: 22.7 × 2 = 45.5 (46) Page Ref: 500 Cognitive Level: Analyzing Client Need &Sub: Physiological Integrity: Pharmacological and Parenteral Therapies Standards: QSEN Competencies: Patient-centered care | AACN Essential Competencies: Essential IX: Baccalaureate generalist nursing practice | NLN Competencies: Human flourishing | Nursing/Integrated Concepts: Nursing Process: Implementation/Coordination of care Learning Outcome: LO 20.7 Develop a school-based nursing care plan for the child with asthma. MNL LO: Respiratory Disorders/Differentiate treatment options and strategies for continuity of care for the child and family. 13) Parents of a child admitted with respiratory distress are concerned because the child won't lie down and wants to sit in a chair leaning forward. Which response by the nurse is the most appropriate? 1. "This helps the child feel in control of his situation." 2. "The child needs to be encouraged to lie flat in bed." 3. "This position helps keep the airway open." 4. "This confirms the child has asthma." Answer: 3 Explanation: 1. Leaning forward helps keep the airway open. The child is not in control just because he is leaning forward. Lying flat in bed will increase the respiratory distress. This position does not confirm asthma. 2. Leaning forward helps keep the airway open. The child is not in control just because he is leaning forward. Lying flat in bed will increase the respiratory distress. This position does not confirm asthma. 3. Leaning forward helps keep the airway open. The child is not in control just because he is leaning forward. Lying flat in bed will increase the respiratory distress. This position does not confirm asthma. 4. Leaning forward helps keep the airway open. The child is not in control just because he is leaning forward. Lying flat in bed will increase the respiratory distress. This position does not confirm asthma. Page Ref: 479 Cognitive Level: Applying Client Need &Sub: Safe and Effective Care Environment: Safety and Infection Control Standards: QSEN Competencies: Patient-centered care | AACN Essential Competencies: Essential VII: Clinical prevention and population health | NLN Competencies: Nursing judgement | Nursing/Integrated Concepts: Nursing Process: Implementation/Health teaching and health promotion Learning Outcome: LO 20.6 Create a nursing care plan for a child with a common acute respiratory condition. MNL LO: Respiratory Disorders/Apply the nursing process in providing care for the child and family. 14) A child is prescribed rifampicin for treatment of tuberculosis. For which length of time will the nurse tell the parents that this child must remain on the medication? 1. 2 months 2. 4 months 3. 6 months 4. 8 months Answer: 3 Explanation: 1. Active and latent TB are treated with isoniazid, rifampicin, pyrazinamide, and ethambutol. Therapy for active TB usually involves a 6-month regimen consisting of isoniazid, rifampicin, pyrazinamide, and ethambutol for the first 2 months and isoniazid and rifampicin for the remaining 4 months. Therefore, the child will remain on rifampicin for a total of 6 months. 2. Active and latent TB are treated with isoniazid, rifampicin, pyrazinamide, and ethambutol. Therapy for active TB usually involves a 6-month regimen consisting of isoniazid, rifampicin, pyrazinamide, and ethambutol for the first 2 months and isoniazid and rifampicin for the remaining 4 months. Therefore, the child will remain on rifampicin for a total of 6 months. 3. Active and latent TB are treated with isoniazid, rifampicin, pyrazinamide, and ethambutol. Therapy for active TB usually involves a 6-month regimen consisting of isoniazid, rifampicin, pyrazinamide, and ethambutol for the first 2 months and isoniazid and rifampicin for the remaining 4 months. Therefore, the child will remain on rifampicin for a total of 6 months. 4. Active and latent TB are treated with isoniazid, rifampicin, pyrazinamide, and ethambutol. Therapy for active TB usually involves a 6-month regimen consisting of isoniazid, rifampicin, pyrazinamide, and ethambutol for the first 2 months and isoniazid and rifampicin for the remaining 4 months. Therefore, the child will remain on rifampicin for a total of 6 months. Page Ref: 493 Cognitive Level: Applying Client Need &Sub: Physiological Integrity: Pharmacological and Parenteral Therapies Standards: QSEN Competencies: Patient-centered care | AACN Essential Competencies: Essential IX: Baccalaureate generalist nursing practice | NLN Competencies: Human flourishing | Nursing/Integrated Concepts: Nursing Process: Implementation/Coordination of care Learning Outcome: LO 20.6 Create a nursing care plan for a child with a common acute respiratory condition. MNL LO: Respiratory Disorders/Differentiate treatment options and strategies for continuity of care for the child and family. 15) The nurse is providing care to an infant who is diagnosed with bronchiolitis. Which breath sounds indicate the infant is experiencing respiratory distress? Select all that apply. 1. Tachypnea 2. Wheezing 3. Grunting 4. Retractions 5. Eupnea Answer: 1, 2, 3 Explanation: 1. Wheezing and grunting are adventitious respiratory sounds that indicate respiratory distress in the neonate. Tachypnea is the term used to indicate a respiratory rate of greater than 60 breaths per minute in an infant. While this does indicate respiratory distress, tachypnea is not a type of breath sound. Retractions, or the use of accessory muscles, are indicative of respiratory distress in the neonate, but this is not a type of breath sound. Eupnea is the medical term for "normal breathing." 2. Wheezing and grunting are adventitious respiratory sounds that indicate respiratory distress in the neonate. Tachypnea is the term used to indicate a respiratory rate of greater than 60 breaths per minute in an infant. While this does indicate respiratory distress, tachypnea is not a type of breath sound. Retractions, or the use of accessory muscles, are indicative of respiratory distress in the neonate, but this is not a type of breath sound. Eupnea is the medical term for "normal breathing." 3. Wheezing and grunting are adventitious respiratory sounds that indicate respiratory distress in the neonate. Tachypnea is the term used to indicate a respiratory rate of greater than 60 breaths per minute in an infant. While this does indicate respiratory distress, tachypnea is not a type of breath sound. Retractions, or the use of accessory muscles, are indicative of respiratory distress in the neonate, but this is not a type of breath sound. Eupnea is the medical term for "normal breathing." 4. Wheezing and grunting are adventitious respiratory sounds that indicate respiratory distress in the neonate. Tachypnea is the term used to indicate a respiratory rate of greater than 60 breaths per minute in an infant. While this does indicate respiratory distress, tachypnea is not a type of breath sound. Retractions, or the use of accessory muscles, are indicative of respiratory distress in the neonate, but this is not a type of breath sound. Eupnea is the medical term for "normal breathing." 5. Wheezing and grunting are adventitious respiratory sounds that indicate respiratory distress in the neonate. Tachypnea is the term used to indicate a respiratory rate of greater than 60 breaths per minute in an infant. While this does indicate respiratory distress, tachypnea is not a type of breath sound. Retractions, or the use of accessory muscles, are indicative of respiratory distress in the neonate, but this is not a type of breath sound. Eupnea is the medical term for "normal breathing." Page Ref: 489 Cognitive Level: Applying Client Need &Sub: Physiological Integrity: Physiological Adaptation Standards: QSEN Competencies: Patient-centered care | AACN Essential Competencies: Essential IX: Baccalaureate generalist nursing practice | NLN Competencies: Human flourishing | Nursing/Integrated Concepts: Nursing Process: Assessment/Coordination of care Learning Outcome: LO 20.3 Explain the visual and auditory observations made to assess a child's respiratory effort or work of breathing. MNL LO: Respiratory Disorders/Examine etiology, risk factors, pathophysiology, and clinical manifestations as seen in children. 16) The nurse educator is teaching a group of nursing students how to perform a respiratory assessment for a newborn in the newborn intensive care unit (NICU) diagnosed with respiratory distress syndrome (RDS). Which normal characteristics of the newborn's respiratory system increase the risk for obstruction? Select all that apply. 1. Shorter and narrower airway 2. Higher trachea 3. Bronchial branching at different angles 4. Inadequate smooth muscle bundles 5. Diaphragmatic breather Answer: 1, 2, 3 Explanation: 1. Normal characteristics of the pediatric respiratory system that increase the risk for obstruction include a shorter and narrower airway, a higher trachea, and a different angle for bronchial branching. Inadequate smooth muscle bundles and being diaphragmatic breathers are characteristics that do not increase the risk of obstruction. 2. Normal characteristics of the pediatric respiratory system that increase the risk for obstruction include a shorter and narrower airway, a higher trachea, and a different angle for bronchial branching. Inadequate smooth muscle bundles and being diaphragmatic breathers are characteristics that do not increase the risk of obstruction. 3. Normal characteristics of the pediatric respiratory system that increase the risk for obstruction include a shorter and narrower airway, a higher trachea, and a different angle for bronchial branching. Inadequate smooth muscle bundles and being diaphragmatic breathers are characteristics that do not increase the risk of obstruction. 4. Normal characteristics of the pediatric respiratory system that increase the risk for obstruction include a shorter and narrower airway, a higher trachea, and a different angle for bronchial branching. Inadequate smooth muscle bundles and being diaphragmatic breathers are characteristics that do not increase the risk of obstruction. 5. Normal characteristics of the pediatric respiratory system that increase the risk for obstruction include a shorter and narrower airway, a higher trachea, and a different angle for bronchial branching. Inadequate smooth muscle bundles and being diaphragmatic breathers are characteristics that do not increase the risk of obstruction. Page Ref: 478 Cognitive Level: Applying Client Need &Sub: Physiological Integrity: Physiological Adaptation Standards: QSEN Competencies: Patient-centered care | AACN Essential Competencies: Essential VII: Clinical prevention and population health | NLN Competencies: Human flourishing | Nursing/Integrated Concepts: Nursing Process: Assessment/Health teaching and health promotion Learning Outcome: LO 20.1 Describe unique characteristics of the pediatric respiratory system anatomy and physiology and apply that information to the care of children with respiratory conditions. MNL LO: Respiratory Disorders/Examine etiology, risk factors, pathophysiology, and clinical manifestations as seen in children. 17) A school nurse is planning care for a school-age child recently diagnosed with asthma. Which items will the school nurse include in the plan of care at the school? Select all that apply. 1. Maintain a log of quick-relief medication administration. 2. Call the parents if quick-relief medications work appropriately. 3. Assess for symptoms of exercise-induced bronchospasm. 4. Coordinate education of the child's teachers. 5. Conduct a support group for all children with asthma. Answer: 1, 3, 4, 5 Explanation: 1. Appropriate interventions for the school nurse to include in the plan of care include: keeping a log of the quick-relief medications administered; assessing the child for exercise-induced bronchospasms and reporting, if needed; coordinating education of the child's teachers; and conducting a support group for all children in the school with asthma. The nurse would only call the parents if the quick-relief mediation was not effective in treating the child's symptoms. 2. Appropriate interventions for the school nurse to include in the plan of care include: keeping a log of the quick-relief medications administered; assessing the child for exercise-induced bronchospasms and reporting, if needed; coordinating education of the child's teachers; and conducting a support group for all children in the school with asthma. The nurse would only call the parents if the quick-relief mediation was not effective in treating the child's symptoms. 3. Appropriate interventions for the school nurse to include in the plan of care include: keeping a log of the quick-relief medications administered; assessing the child for exercise-induced bronchospasms and reporting, if needed; coordinating education of the child's teachers; and conducting a support group for all children in the school with asthma. The nurse would only call the parents if the quick-relief mediation was not effective in treating the child's symptoms. 4. Appropriate interventions for the school nurse to include in the plan of care include: keeping a log of the quick-relief medications administered; assessing the child for exercise-induced bronchospasms and reporting, if needed; coordinating education of the child's teachers; and conducting a support group for all children in the school with asthma. The nurse would only call the parents if the quick-relief mediation was not effective in treating the child's symptoms. 5. Appropriate interventions for the school nurse to include in the plan of care include: keeping a log of the quick-relief medications administered; assessing the child for exercise-induced bronchospasms and reporting, if needed; coordinating education of the child's teachers; and conducting a support group for all children in the school with asthma. The nurse would only call the parents if the quick-relief mediation was not effective in treating the child's symptoms. Page Ref: 507 Cognitive Level: Applying Client Need &Sub: Physiological Integrity: Physiological Adaptation Standards: QSEN Competencies: Patient-centered care | AACN Essential Competencies: Essential IX: Baccalaureate generalist nursing practice | NLN Competencies: Nursing judgement | Nursing/Integrated Concepts: Nursing Process: Planning/Coordination of care Learning Outcome: LO 20.7 Develop a school-based nursing care plan for the child with asthma. MNL LO: Respiratory Disorders/Apply the nursing process in providing care for the child and family. 18) The nurse is providing care to an infant in the emergency department. Upon assessment, the infant is noted to have to be experiencing tachypnea, wheezing, retractions, and nasal flaring. The infant is irritability and the parents state the infant has had poor fluid intake for two days. Pulse ox reading is currently at 85 percent on room air. The infant's blood gas is pending. Which diagnosis does the nurse anticipate for this infant? 1. Bronchitis 2. Bronchiolitis 3. Pneumonia 4. Active pulmonary tuberculosis Answer: 2 Explanation: 1. The nurse anticipates the infant will be diagnosed with bronchiolitis. Symptoms of bronchiolitis include mild respiratory symptoms that progress to tachypnea, wheezing, retractions, nasal flaring, irritability, poor fluid intake, hypoxia, cyanosis, and decreased mental status. Symptoms of bronchitis include a dry hacking cough, increases in severity at night, painful chest and ribs. Symptoms of pneumonia include initial rhinitis and cough, followed by fever, crackles, wheezes, dyspnea, tachypnea, restlessness, diminished breath sounds. Symptoms of active pulmonary tuberculosis include persistent cough, decreased appetite, weight loss or failure to gain weight, low-grade fever, night sweats, chills, enlarged lymph nodes. 2. The nurse anticipates the infant will be diagnosed with bronchiolitis. Symptoms of bronchiolitis include mild respiratory symptoms that progress to tachypnea, wheezing, retractions, nasal flaring, irritability, poor fluid intake, hypoxia, cyanosis, and decreased mental status. Symptoms of bronchitis include a dry hacking cough, increases in severity at night, painful chest and ribs. Symptoms of pneumonia include initial rhinitis and cough, followed by fever, crackles, wheezes, dyspnea, tachypnea, restlessness, diminished breath sounds. Symptoms of active pulmonary tuberculosis include persistent cough, decreased appetite, weight loss or failure to gain weight, low-grade fever, night sweats, chills, enlarged lymph nodes. 3. The nurse anticipates the infant will be diagnosed with bronchiolitis. Symptoms of bronchiolitis include mild respiratory symptoms that progress to tachypnea, wheezing, retractions, nasal flaring, irritability, poor fluid intake, hypoxia, cyanosis, and decreased mental status. Symptoms of bronchitis include a dry hacking cough, increases in severity at night, painful chest and ribs. Symptoms of pneumonia include initial rhinitis and cough, followed by fever, crackles, wheezes, dyspnea, tachypnea, restlessness, diminished breath sounds. Symptoms of active pulmonary tuberculosis include persistent cough, decreased appetite, weight loss or failure to gain weight, low-grade fever, night sweats, chills, enlarged lymph nodes. 4. The nurse anticipates the infant will be diagnosed with bronchiolitis. Symptoms of bronchiolitis include mild respiratory symptoms that progress to tachypnea, wheezing, retractions, nasal flaring, irritability, poor fluid intake, hypoxia, cyanosis, and decreased mental status. Symptoms of bronchitis include a dry hacking cough, increases in severity at night, painful chest and ribs. Symptoms of pneumonia include initial rhinitis and cough, followed by fever, crackles, wheezes, dyspnea, tachypnea, restlessness, diminished breath sounds. Symptoms of active pulmonary tuberculosis include persistent cough, decreased appetite, weight loss or failure to gain weight, low-grade fever, night sweats, chills, enlarged lymph nodes. Page Ref: 489 Cognitive Level: Applying Client Need &Sub: Physiological Integrity: Physiological Adaptation Standards: QSEN Competencies: Patient-centered care | AACN Essential Competencies: Essential IX: Baccalaureate generalist nursing practice | NLN Competencies: Human flourishing | Nursing/Integrated Concepts: Nursing Process: Assessment/Coordination of care Learning Outcome: LO 20.5 Distinguish between conditions of the lower respiratory tract that cause illness in children. MNL LO: Respiratory Disorders/Apply the nursing process in providing care for the child and family. 19) The nurse is assessing a school-age client who experienced blunt force trauma to the chest when an airbag deployed following a motor vehicle crash. Which areas of assessment are essential for this client? Select all that apply. 1. Monitor responsiveness and behavior. 2. Monitor SpO2. 3. Auscultate the lungs for crackles, wheezes, decreased breath sounds. 4. Document input and output. 5. Note changes in voice quality or coughing. Answer: 1, 2, 3, 4 Explanation: 1. The areas of assessment that are essential for this client include: monitoring for responsive and behavior in order to detect hypoxia and the potential for airway obstruction; monitoring SpO2 frequently to identify changes indicating deterioration in condition; auscultating the lungs for crackles, wheezes, decreased breath sound; and noting changes in voice quality or coughing. Documenting input and output is not a priority for this client. 2. The areas of assessment that are essential for this client include: monitoring for responsive and behavior in order to detect hypoxia and the potential for airway obstruction; monitoring SpO2 frequently to identify changes indicating deterioration in condition; auscultating the lungs for crackles, wheezes, decreased breath sound; and noting changes in voice quality or coughing. Documenting input and output is not a priority for this client. 3. The areas of assessment that are essential for this client include: monitoring for responsive and behavior in order to detect hypoxia and the potential for airway obstruction; monitoring SpO2 frequently to identify changes indicating deterioration in condition; auscultating the lungs for crackles, wheezes, decreased breath sound; and noting changes in voice quality or coughing. Documenting input and output is not a priority for this client. 4. The areas of assessment that are essential for this client include: monitoring for responsive and behavior in order to detect hypoxia and the potential for airway obstruction; monitoring SpO2 frequently to identify changes indicating deterioration in condition; auscultating the lungs for crackles, wheezes, decreased breath sound; and noting changes in voice quality or coughing. Documenting input and output is not a priority for this client. 5. The areas of assessment that are essential for this client include: monitoring for responsive and behavior in order to detect hypoxia and the potential for airway obstruction; monitoring SpO2 frequently to identify changes indicating deterioration in condition; auscultating the lungs for crackles, wheezes, decreased breath sound; and noting changes in voice quality or coughing. Documenting input and output is not a priority for this client. Page Ref: 514-515 Cognitive Level: Applying Client Need &Sub: Physiological Integrity: Physiological Adaptation Standards: QSEN Competencies: Patient-centered care | AACN Essential Competencies: Essential IX: Baccalaureate generalist nursing practice | NLN Competencies: Human flourishing | Nursing/Integrated Concepts: Nursing Process: Assessment/Coordination of care Learning Outcome: LO 20.8 Perform a nursing assessment of the child with an acute lung injury. MNL LO: Respiratory Disorders/Apply the nursing process in providing care for the child and family. 20) The toddler is admitted to the hospital during an acute asthma attack. The physician orders: methylprednisolone 80 mg infused IV push every 3 hours. Medication on hand: methylprednisolone 125 mg/2 mL Calculate how many ml of methylprednisolone the patient will receive. Answer: 1.28 mL Explanation: 1.28 mL Page Ref: 502 Cognitive Level: Analyzing Client Need &Sub: Physiological Integrity: Pharmacological and Parenteral Therapies Standards: QSEN Competencies: Patient-centered care | AACN Essential Competencies: Essential IX: Baccalaureate generalist nursing practice | NLN Competencies: Human flourishing | Nursing/Integrated Concepts: Nursing Process: Planning/Coordination of care Learning Outcome: LO 20.2 Contrast the different respiratory medical conditions that can cause respiratory distress in infants and children. MNL LO: Apply the general concepts related to caring for a child with a chronic illness. 21) A 3-year-old has been diagnosed with cystic fibrosis. The guardians asked the nurse what respiratory symptoms they should expect to see. What will the nurse tell the guardians? Select all that apply. 1. Purulent nasal discharge 2. Frequent infections 3. Mottled nail beds 4. Chronic moist, productive cough 5. Increased fertility Answer: 1, 2, 4 Explanation: 1. Respiratory symptoms the guardians will see are: nasal polyps, chronic sinusitis, frontal headaches, purulent nasal discharge, postnasal discharge, cough (chronic, moist, productive), wheezing, coarse crackles, frequent infections, shortness of breath, decreased exercise tolerance, barrel chest, and clubbing of fingers and toes. 2. Respiratory symptoms the guardians will see are: nasal polyps, chronic sinusitis, frontal headaches, purulent nasal discharge, postnasal discharge, cough (chronic, moist, productive), wheezing, coarse crackles, frequent infections, shortness of breath, decreased exercise tolerance, barrel chest, and clubbing of fingers and toes. 3. Respiratory symptoms the guardians will see are: nasal polyps, chronic sinusitis, frontal headaches, purulent nasal discharge, postnasal discharge, cough (chronic, moist, productive), wheezing, coarse crackles, frequent infections, shortness of breath, decreased exercise tolerance, barrel chest, and clubbing of fingers and toes. 4. Respiratory symptoms the guardians will see are: nasal polyps, chronic sinusitis, frontal headaches, purulent nasal discharge, postnasal discharge, cough (chronic, moist, productive), wheezing, coarse crackles, frequent infections, shortness of breath, decreased exercise tolerance, barrel chest, and clubbing of fingers and toes. 5. Respiratory symptoms the guardians will see are: nasal polyps, chronic sinusitis, frontal headaches, purulent nasal discharge, postnasal discharge, cough (chronic, moist, productive), wheezing, coarse crackles, frequent infections, shortness of breath, decreased exercise tolerance, barrel chest, and clubbing of fingers and toes. Page Ref: 508 Cognitive Level: Analyzing Client Need &Sub: Health Promotion and Maintenance Standards: QSEN Competencies: Patient-centered care | AACN Essential Competencies: Essential VII: Clinical prevention and population health | NLN Competencies: Human flourishing | Nursing/Integrated Concepts: Nursing Process: Planning/Health teaching and health promotion Learning Outcome: LO 20.3 Explain the visual and auditory observations made to assess a child's respiratory effort or work of breathing. MNL LO: Respiratory Disorders/Examine etiology, risk factors, pathophysiology, and clinical manifestations as seen in children. Principles of Pediatric Nursing: Caring for Children, 7e (Ball et al.) Chapter 21 Alterations in Cardiovascular Function 1) The nurse has admitted a child with tricuspid atresia. The nurse would expect which initial lab result? 1. A high hemoglobin 2. A low hematocrit 3. A high WBC count 4. A low platelet count Answer: 1 Explanation: 1. The child's bone marrow responds to chronic hypoxemia by producing more RBCs to increase the amount of hemoglobin available to carry oxygen to the tissues. This occurs in cases of cyanotic heart defects such as tricuspid atresia. Therefore, the hematocrit would not be low, the WBC count would not be high (unless an infection were present), and the platelets would be normal. 2. The child's bone marrow responds to chronic hypoxemia by producing more RBCs to increase the amount of hemoglobin available to carry oxygen to the tissues. This occurs in cases of cyanotic heart defects such as tricuspid atresia. Therefore, the hematocrit would not be low, the WBC count would not be high (unless an infection were present), and the platelets would be normal. 3. The child's bone marrow responds to chronic hypoxemia by producing more RBCs to increase the amount of hemoglobin available to carry oxygen to the tissues. This occurs in cases of cyanotic heart defects such as tricuspid atresia. Therefore, the hematocrit would not be low, the WBC count would not be high (unless an infection were present), and the platelets would be normal. 4. The child's bone marrow responds to chronic hypoxemia by producing more RBCs to increase the amount of hemoglobin available to carry oxygen to the tissues. This occurs in cases of cyanotic heart defects such as tricuspid atresia. Therefore, the hematocrit would not be low, the WBC count would not be high (unless an infection were present), and the platelets would be normal. Page Ref: 534 Cognitive Level: Analyzing Client Need &Sub: Physiological Integrity: Physiological Adaptation Standards: QSEN Competencies: Patient-centered care | AACN Essential Competencies: Essential IX: Baccalaureate generalist nursing practice | NLN Competencies: Nursing judgement | Nursing/Integrated Concepts: Nursing Process: Assessment/Coordination of care Learning Outcome: LO 21.2 Describe the pathophysiology associated with congenital heart defects with increased pulmonary circulation, decreased pulmonary circulation, mixed defects, and obstructed systemic blood flow. MNL LO: Cardiovascular and Hematological Disorders/Examine etiology, risk factors, pathophysiology, and clinical manifestations as seen in children. 2) A child has been admitted to the hospital unit in congestive heart failure (CHF). Which symptom would the nurse anticipate upon assessment of the child? 1. Weight loss 2. Bradycardia 3. Tachycardia 4. Increased blood pressure Answer: 3 Explanation: 1. Tachycardia is a sign of congestive heart failure because the heart attempts to improve cardiac output by beating faster. Bradycardia is a serious sign and can indicate impending cardiac arrest. Blood pressure does not increase in CHF, and the weight, instead of decreasing, increases because of retention of fluids. 2. Tachycardia is a sign of congestive heart failure because the heart attempts to improve cardiac output by beating faster. Bradycardia is a serious sign and can indicate impending cardiac arrest. Blood pressure does not increase in CHF, and the weight, instead of decreasing, increases because of retention of fluids. 3. Tachycardia is a sign of congestive heart failure because the heart attempts to improve cardiac output by beating faster. Bradycardia is a serious sign and can indicate impending cardiac arrest. Blood pressure does not increase in CHF, and the weight, instead of decreasing, increases because of retention of fluids. 4. Tachycardia is a sign of congestive heart failure because the heart attempts to improve cardiac output by beating faster. Bradycardia is a serious sign and can indicate impending cardiac arrest. Blood pressure does not increase in CHF, and the weight, instead of decreasing, increases because of retention of fluids. Page Ref: 544 Cognitive Level: Analyzing Client Need &Sub: Physiological Integrity: Physiological Adaptation Standards: QSEN Competencies: Patient-centered care | AACN Essential Competencies: Essential IX: Baccalaureate generalist nursing practice | NLN Competencies: Human flourishing | Nursing/Integrated Concepts: Nursing Process: Assessment/Coordination of care Learning Outcome: LO 21.6 Develop a nursing care plan for a child with congestive heart failure. MNL LO: Cardiovascular and Hematological Disorders/Apply the nursing process in providing care for the child and family. 3) A toddler is started on digoxin (Lanoxin) for cardiac failure. Which is the initial symptom the nurse would assess if the child develops digoxin (Lanoxin) toxicity? 1. Lowered blood pressure 2. Tinnitus 3. Ataxia 4. A change in heart rhythm Answer: 4 Explanation: 1. An early sign of digoxin (Lanoxin) toxicity is a change in heart rhythm. Digoxin (Lanoxin) toxicity does not cause lowered blood pressure, tinnitus (ringing in the ears), or ataxia (unsteady gait). 2. An early sign of digoxin (Lanoxin) toxicity is a change in heart rhythm. Digoxin (Lanoxin) toxicity does not cause lowered blood pressure, tinnitus (ringing in the ears), or ataxia (unsteady gait). 3. An early sign of digoxin (Lanoxin) toxicity is a change in heart rhythm. Digoxin (Lanoxin) toxicity does not cause lowered blood pressure, tinnitus (ringing in the ears), or ataxia (unsteady gait). 4. An early sign of digoxin (Lanoxin) toxicity is a change in heart rhythm. Digoxin (Lanoxin) toxicity does not cause lowered blood pressure, tinnitus (ringing in the ears), or ataxia (unsteady gait). Page Ref: 545 Cognitive Level: Analyzing Client Need &Sub: Physiological Integrity: Physiological Adaptation Standards: QSEN Competencies: Patient-centered care | AACN Essential Competencies: Essential IX: Baccalaureate generalist nursing practice | NLN Competencies: Human flourishing | Nursing/Integrated Concepts: Nursing Process: Assessment/Coordination of care Learning Outcome: LO 21.5 Recognize the signs and symptoms of congestive heart failure in an infant and child. MNL LO: Cardiovascular and Hematological Disorders/Examine etiology, risk factors, pathophysiology, and clinical manifestations as seen in children. 4) The nurse is checking peripheral perfusion to a child's extremity following a cardiac catheterization. Which assessment finding indicates adequate peripheral circulation to the affected extremity? 1. A capillary refill of greater than three seconds 2. A palpable dorsalis pedis pulse but a weak posterior tibial pulse 3. A decrease in sensation with a weakened dorsalis pedis pulse 4. A capillary refill of less than three seconds with palpable warmth Answer: 4 Explanation: 1. The nurse checks the extremity to determine adequacy of circulation following a cardiac catheterization. An extremity that is warm with capillary refill of less than three seconds has adequate circulation. Other indicators of adequate circulation include palpable pedal (dorsalis and posterior tibial) pulses, adequate sensation, and pinkness of skin color. If the capillary refill is over three seconds; if any of the pedal pulses are absent and/or weakened; or if the extremity is cool, cyanotic, or lacking sensation, circulation may not be adequate. 2. The nurse checks the extremity to determine adequacy of circulation following a cardiac catheterization. An extremity that is warm with capillary refill of less than three seconds has adequate circulation. Other indicators of adequate circulation include palpable pedal (dorsalis and posterior tibial) pulses, adequate sensation, and pinkness of skin color. If the capillary refill is over three seconds; if any of the pedal pulses are absent and/or weakened; or if the extremity is cool, cyanotic, or lacking sensation, circulation may not be adequate. 3. The nurse checks the extremity to determine adequacy of circulation following a cardiac catheterization. An extremity that is warm with capillary refill of less than three seconds has adequate circulation. Other indicators of adequate circulation include palpable pedal (dorsalis and posterior tibial) pulses, adequate sensation, and pinkness of skin color. If the capillary refill is over three seconds; if any of the pedal pulses are absent and/or weakened; or if the extremity is cool, cyanotic, or lacking sensation, circulation may not be adequate. 4. The nurse checks the extremity to determine adequacy of circulation following a cardiac catheterization. An extremity that is warm with capillary refill of less than three seconds has adequate circulation. Other indicators of adequate circulation include palpable pedal (dorsalis and posterior tibial) pulses, adequate sensation, and pinkness of skin color. If the capillary refill is over three seconds; if any of the pedal pulses are absent and/or weakened; or if the extremity is cool, cyanotic, or lacking sensation, circulation may not be adequate. Page Ref: 526 Cognitive Level: Analyzing Client Need &Sub: Physiological Integrity: Physiological Adaptation Standards: QSEN Competencies: Patient-centered care | AACN Essential Competencies: Essential IX: Baccalaureate generalist nursing practice | NLN Competencies: Human flourishing | Nursing/Integrated Concepts: Nursing Process: Assessment/Coordination of care Learning Outcome: LO 21.1 Describe the anatomy and physiology of the cardiovascular system, focusing on the flow of blood and the action of heart valves. MNL LO: Cardiovascular and Hematological Disorders/Examine etiology, risk factors, pathophysiology, and clinical manifestations as seen in children. 5) The nurse admits a child with a ventricular septal defect (VSD) to the unit. Which nursing diagnosis for this child is the most appropriate? 1. Impaired Gas Exchange Related to Pulmonary Congestion Secondary to the Increased Pulmonary Blood Flow 2. Deficient Fluid Volume Related to Hyperthermia Secondary to the Congenital Heart Defect 3. Acute Pain Related to the Effects of a Congenital Heart Defect 4. Hypothermia Related to Decreased Metabolic State Answer: 1 Explanation: 1. Because of the increased pulmonary congestion, Impaired Gas Exchange would be an appropriate nursing diagnosis. Ventricular septal defects do not cause pain, fever, or deficient fluid volume. 2. Because of the increased pulmonary congestion, Impaired Gas Exchange would be an appropriate nursing diagnosis. Ventricular septal defects do not cause pain, fever, or deficient fluid volume. 3. Because of the increased pulmonary congestion, Impaired Gas Exchange would be an appropriate nursing diagnosis. Ventricular septal defects do not cause pain, fever, or deficient fluid volume. 4. Because of the increased pulmonary congestion, Impaired Gas Exchange would be an appropriate nursing diagnosis. Ventricular septal defects do not cause pain, fever, or deficient fluid volume. Page Ref: 528-529 Cognitive Level: Analyzing Client Need &Sub: Physiological Integrity: Basic Care and Comfort Standards: QSEN Competencies: Patient-centered care | AACN Essential Competencies: Essential IX: Baccalaureate generalist nursing practice | NLN Competencies: Human flourishing | Nursing/Integrated Concepts: Nursing Process: Diagnosis/Coordination of care Learning Outcome: LO 21.2 Describe the pathophysiology associated with congenital heart defects with increased pulmonary circulation, decreased pulmonary circulation, mixed defects, and obstructed systemic blood flow. MNL LO: Cardiovascular and Hematological Disorders/Examine etiology, risk factors, pathophysiology, and clinical manifestations as seen in children. 6) The nurse is teaching the parents of a group of cardiac patients. Which teaching guideline will the nurse include for any child who has undergone cardiac surgery? 1. The child should be restricted from most play activities. 2. The child should be evaluated to determine if prophylactic antibiotics for dental, oral, or upper-respiratory-tract procedures are necessary. 3. The child should not receive routine immunizations. 4. The child can be expected to have a fever for several weeks following the surgery. Answer: 2 Explanation: 1. Parents should be taught that the child may need prophylactic antibiotics for some dental procedures, according to the American Heart Association, to prevent endocarditis. The child should live a normal and active life following repair of a cardiac defect. Immunizations should be provided according to the schedule, and any unexplained fever should be reported. 2. Parents should be taught that the child may need prophylactic antibiotics for some dental procedures, according to the American Heart Association, to prevent endocarditis. The child should live a normal and active life following repair of a cardiac defect. Immunizations should be provided according to the schedule, and any unexplained fever should be reported. 3. Parents should be taught that the child may need prophylactic antibiotics for some dental procedures, according to the American Heart Association, to prevent endocarditis. The child should live a normal and active life following repair of a cardiac defect. Immunizations should be provided according to the schedule, and any unexplained fever should be reported. 4. Parents should be taught that the child may need prophylactic antibiotics for some dental procedures, according to the American Heart Association, to prevent endocarditis. The child should live a normal and active life following repair of a cardiac defect. Immunizations should be provided according to the schedule, and any unexplained fever should be reported. Page Ref: 540 Cognitive Level: Applying Client Need &Sub: Health Promotion and Maintenance Standards: QSEN Competencies: Patient-centered care | AACN Essential Competencies: Essential VII: Clinical prevention and population health | NLN Competencies: Human flourishing | Nursing/Integrated Concepts: Nursing Process: Implementation/Health teaching and health promotion Learning Outcome: LO 21.4 Create a nursing care plan for the child undergoing open heart surgery. MNL LO: Cardiovascular and Hematological Disorders/Educate the child and family on the care of the child during hospitalization and upon discharge. 7) An infant with tetralogy of Fallot is having a hypercyanotic episode ("tet" spell). Which nursing interventions are appropriate for the nurse to implement for this infant? Select all that apply. 1. Place the child in knee-chest position. 2. Draw blood for a serum hemoglobin. 3. Administer oxygen. 4. Administer morphine and propranolol intravenously as ordered. 5. Administer Benadryl as ordered. Answer: 1, 3, 4 Explanation: 1. When an infant with tetralogy of Fallot has a hypercyanotic episode, interventions should be geared toward decreasing the pulmonary vascular resistance. Therefore, the nurse would place the infant in knee-chest position (to decrease venous blood return from the lower extremities), and administer oxygen, morphine, and propranolol (to decrease the pulmonary vascular resistance). The nurse would not draw blood until the episode had subsided, because unpleasant procedures are postponed. Benadryl is not appropriate for this child. 2. When an infant with tetralogy of Fallot has a hypercyanotic episode, interventions should be geared toward decreasing the pulmonary vascular resistance. Therefore, the nurse would place the infant in knee-chest position (to decrease venous blood return from the lower extremities), and administer oxygen, morphine, and propranolol (to decrease the pulmonary vascular resistance). The nurse would not draw blood until the episode had subsided, because unpleasant procedures are postponed. Benadryl is not appropriate for this child. 3. When an infant with tetralogy of Fallot has a hypercyanotic episode, interventions should be geared toward decreasing the pulmonary vascular resistance. Therefore, the nurse would place the infant in knee-chest position (to decrease venous blood return from the lower extremities), and administer oxygen, morphine, and propranolol (to decrease the pulmonary vascular resistance). The nurse would not draw blood until the episode had subsided, because unpleasant procedures are postponed. Benadryl is not appropriate for this child. 4. When an infant with tetralogy of Fallot has a hypercyanotic episode, interventions should be geared toward decreasing the pulmonary vascular resistance. Therefore, the nurse would place the infant in knee-chest position (to decrease venous blood return from the lower extremities), and administer oxygen, morphine, and propranolol (to decrease the pulmonary vascular resistance). The nurse would not draw blood until the episode had subsided, because unpleasant procedures are postponed. Benadryl is not appropriate for this child. 5. When an infant with tetralogy of Fallot has a hypercyanotic episode, interventions should be geared toward decreasing the pulmonary vascular resistance. Therefore, the nurse would place the infant in knee-chest position (to decrease venous blood return from the lower extremities), and administer oxygen, morphine, and propranolol (to decrease the pulmonary vascular resistance). The nurse would not draw blood until the episode had subsided, because unpleasant procedures are postponed. Benadryl is not appropriate for this child. Page Ref: 535 Cognitive Level: Applying Client Need &Sub: Safe and Effective Care Environment: Management of Care Standards: QSEN Competencies: Patient-centered care | AACN Essential Competencies: Essential IX: Baccalaureate generalist nursing practice | NLN Competencies: Nursing judgement | Nursing/Integrated Concepts: Nursing Process: Implementation/Coordination of care Learning Outcome: LO 21.2 Describe the pathophysiology associated with congenital heart defects with increased pulmonary circulation, decreased pulmonary circulation, mixed defects, and obstructed systemic blood flow. MNL LO: Cardiovascular and Hematological Disorders/Examine etiology, risk factors, pathophysiology, and clinical manifestations as seen in children. 8) A child recently had a heart transplant and the nurse teaches the parents the importance of administering cyclosporine A. Which statement by the parents indicates an appropriate understanding of the teaching session? 1. "Cyclosporin A reduces serum-cholesterol level." 2. "Cyclosporin A prevents rejection." 3. "Cyclosporin A treats hypertension." 4. "Cyclosporin A treats infections." Answer: 2 Explanation: 1. Cyclosporin A is given to prevent rejection. Lovastatin is given to reduce serum- cholesterol level, calcium channel blockers may be used to treat hypertension, and an antibiotic may be given to treat an infection. 2. Cyclosporin A is given to prevent rejection. Lovastatin is given to reduce serum-cholesterol level, calcium channel blockers may be used to treat hypertension, and an antibiotic may be given to treat an infection. 3. Cyclosporin A is given to prevent rejection. Lovastatin is given to reduce serum-cholesterol level, calcium channel blockers may be used to treat hypertension, and an antibiotic may be given to treat an infection. 4. Cyclosporin A is given to prevent rejection. Lovastatin is given to reduce serum-cholesterol level, calcium channel blockers may be used to treat hypertension, and an antibiotic may be given to treat an infection. Page Ref: 550-551 Cognitive Level: Analyzing Client Need &Sub: Physiological Integrity: Pharmacological and Parenteral Therapies Standards: QSEN Competencies: Patient-centered care | AACN Essential Competencies: Essential VII: Clinical prevention and population health | NLN Competencies: Human flourishing | Nursing/Integrated Concepts: Nursing Process: Evaluation/Health teaching and health promotion Learning Outcome: LO 21.4 Create a nursing care plan for the child undergoing open heart surgery. MNL LO: Cardiovascular and Hematological Disorders/Apply the nursing process in providing care for the child and family. 9) Which athletic activity can the nurse recommend for a school-age client with pulmonary- artery hypertension? 1. Cross-country running 2. Soccer 3. Golf 4. Basketball Answer: 3 Explanation: 1. A child with pulmonary-artery hypertension should have exercise tailored to avoid dyspnea. Golf would require less exertion than soccer, basketball, or cross-country running. 2. A child with pulmonary-artery hypertension should have exercise tailored to avoid dyspnea. Golf would require less exertion than soccer, basketball, or cross-country running. 3. A child with pulmonary-artery hypertension should have exercise tailored to avoid dyspnea. Golf would require less exertion than soccer, basketball, or cross-country running. 4. A child with pulmonary-artery hypertension should have exercise tailored to avoid dyspnea. Golf would require less exertion than soccer, basketball, or cross-country running. Page Ref: 551 Cognitive Level: Applying Client Need &Sub: Health Promotion and Maintenance Standards: QSEN Competencies: Patient-centered care | AACN Essential Competencies: Essential IX: Baccalaureate generalist nursing practice | NLN Competencies: Human flourishing | Nursing/Integrated Concepts: Nursing Process: Implementation/Coordination of care Learning Outcome: LO 21.7 Differentiate among the heart diseases that are acquired during childhood. MNL LO: Cardiovascular and Hematological Disorders/Examine etiology, risk factors, pathophysiology, and clinical manifestations as seen in children. 10) A child is admitted with infective endocarditis. Which nursing intervention is most appropriate for this child? 1. Start an intravenous line. 2. Place the child in contact isolation. 3. Place the child on seizure precautions. 4. Assist with a lumbar puncture. Answer: 1 Explanation: 1. Infective endocarditis is treated with intravenous antibiotics for 2 to 8 weeks. It is not contagious, so the child is not placed in contact isolation. Seizures are not a risk of infective endocarditis. A lumbar puncture is not a diagnostic test done for infective endocarditis. 2. Infective endocarditis is treated with intravenous antibiotics for 2 to 8 weeks. It is not contagious, so the child is not placed in contact isolation. Seizures are not a risk of infective endocarditis. A lumbar puncture is not a diagnostic test done for infective endocarditis. 3. Infective endocarditis is treated with intravenous antibiotics for 2 to 8 weeks. It is not contagious, so the child is not placed in contact isolation. Seizures are not a risk of infective endocarditis. A lumbar puncture is not a diagnostic test done for infective endocarditis. 4. Infective endocarditis is treated with intravenous antibiotics for 2 to 8 weeks. It is not contagious, so the child is not placed in contact isolation. Seizures are not a risk of infective endocarditis. A lumbar puncture is not a diagnostic test done for infective endocarditis. Page Ref: 551-552 Cognitive Level: Applying Client Need &Sub: Physiological Integrity: Pharmacological and Parenteral Therapies Standards: QSEN Competencies: Patient-centered care | AACN Essential Competencies: Essential IX: Baccalaureate generalist nursing practice | NLN Competencies: Human flourishing | Nursing/Integrated Concepts: Nursing Process: Planning/Coordination of care Learning Outcome: LO 21.1 Describe the anatomy and physiology of the cardiovascular system, focusing on the flow of blood and the action of heart valves. MNL LO: Cardiovascular and Hematological Disorders/Examine etiology, risk factors, pathophysiology, and clinical manifestations as seen in children. 11) The mother of a child with a heart defect is questioning the nurse about the child's diuretic. When teaching the mother about the medication, what should the emphasis from the nurse? 1. Close monitoring of output 2. The digitalization process 3. The possibility that pulses in the child might be weak 4. The child's increased appetite Answer: 1 Explanation: 1. It is important to monitor the output of the child on a diuretic to determine effectiveness of the drug. Digitalization pulses are not associated with diuretics. The child will usually have a decreased appetite. 2. It is important to monitor the output of the child on a diuretic to determine effectiveness of the drug. Digitalization pulses are not associated with diuretics. The child will usually have a decreased appetite. 3. It is important to monitor the output of the child on a diuretic to determine effectiveness of the drug. Digitalization pulses are not associated with diuretics. The child will usually have a decreased appetite. 4. It is important to monitor the output of the child on a diuretic to determine effectiveness of the drug. Digitalization pulses are not associated with diuretics. The child will usually have a decreased appetite. Page Ref: 547 Cognitive Level: Applying Client Need &Sub: Physiological Integrity: Pharmacological and Parenteral Therapies Standards: QSEN Competencies: Patient-centered care | AACN Essential Competencies: Essential VII: Clinical prevention and population health | NLN Competencies: Nursing judgement | Nursing/Integrated Concepts: Nursing Process: Implementation/Health teaching and health promotion Learning Outcome: LO 21.2 Describe the pathophysiology associated with congenital heart defects with increased pulmonary circulation, decreased pulmonary circulation, mixed defects, and obstructed systemic blood flow. MNL LO: Cardiovascular and Hematological Disorders/Examine etiology, risk factors, pathophysiology, and clinical manifestations as seen in children. 12) The nurse is performing the initial assessment of a child newly diagnosed Kawasaki disease. Which symptoms would the nurse expect to assess with this child? 1. Dry, swollen, fissured lips 2. Nonpalpable lymph nodes 3. Conjunctivitis with exudates 4. Cyanosis of the hands and feet Answer: 1 Explanation: 1. Dry, swollen, fissured lips are symptoms of Kawasaki disease. Lymph nodes can be palpable, conjunctivitis is present but without exudates, and hands and feet are typically erythematous. 2. Dry, swollen, fissured lips are symptoms of Kawasaki disease. Lymph nodes can be palpable, conjunctivitis is present but without exudates, and hands and feet are typically erythematous. 3. Dry, swollen, fissured lips are symptoms of Kawasaki disease. Lymph nodes can be palpable, conjunctivitis is present but without exudates, and hands and feet are typically erythematous. 4. Dry, swollen, fissured lips are symptoms of Kawasaki disease. Lymph nodes can be palpable, conjunctivitis is present but without exudates, and hands and feet are typically erythematous. Page Ref: 553-554 Cognitive Level: Applying Client Need &Sub: Physiological Integrity: Physiological Adaptation Standards: QSEN Competencies: Patient-centered care | AACN Essential Competencies: Essential IX: Baccalaureate generalist nursing practice | NLN Competencies: Nursing judgement | Nursing/Integrated Concepts: Nursing Process: Assessment/Coordination of care Learning Outcome: LO 21.7 Differentiate among the heart diseases that are acquired during childhood. MNL LO: Cardiovascular and Hematological Disorders/Examine etiology, risk factors, pathophysiology, and clinical manifestations as seen in children. 13) The nurse finds that an infant has stronger pulses in the upper extremities than in the lower extremities, and higher blood pressure readings in the arms than in the legs. Which assessment will the nurse perform next on this infant? 1. Pedal pulses 2. Pulse oximetry level 3. Hemoglobin and hematocrit values 4. Blood pressure of the four extremities Answer: 4 Explanation: 1. Coarctation of the aorta can present with stronger pulses in the upper extremities than in the lower extremities and higher blood pressure readings in the arms than in the legs because of obstruction of circulation to the lower extremities. Blood pressure values of the four limbs should be the next assessment data collected. Pedal pulses, pulse oximetry, and labs themselves will not provide the data needed. 2. Coarctation of the aorta can present with stronger pulses in the upper extremities than in the lower extremities and higher blood pressure readings in the arms than in the legs because of obstruction of circulation to the lower extremities. Blood pressure values of the four limbs should be the next assessment data collected. Pedal pulses, pulse oximetry, and labs themselves will not provide the data needed. 3. Coarctation of the aorta can present with stronger pulses in the upper extremities than in the lower extremities and higher blood pressure readings in the arms than in the legs because of obstruction of circulation to the lower extremities. Blood pressure values of the four limbs should be the next assessment data collected. Pedal pulses, pulse oximetry, and labs themselves will not provide the data needed. 4. Coarctation of the aorta can present with stronger pulses in the upper extremities than in the lower extremities and higher blood pressure readings in the arms than in the legs because of obstruction of circulation to the lower extremities. Blood pressure values of the four limbs should be the next assessment data collected. Pedal pulses, pulse oximetry, and labs themselves will not provide the data needed. Page Ref: 540-543 Cognitive Level: Analyzing Client Need &Sub: Physiological Integrity: Physiological Adaptation Standards: QSEN Competencies: Patient-centered care | AACN Essential Competencies: Essential IX: Baccalaureate generalist nursing practice | NLN Competencies: Human flourishing | Nursing/Integrated Concepts: Nursing Process: Assessment/Coordination of care Learning Outcome: LO 21.2 Describe the pathophysiology associated with congenital heart defects with increased pulmonary circulation, decreased pulmonary circulation, mixed defects, and obstructed systemic blood flow. MNL LO: Cardiovascular and Hematological Disorders/Examine etiology, risk factors, pathophysiology, and clinical manifestations as seen in children. 14) The nurse is admitting an infant diagnosed with supraventricular tachycardia. Which intervention is the priority for this infant? 1. Apply ice to the face. 2. Perform Valsalva's maneuver. 3. Administer a beta blocker. 4. Prepare for cardioversion. Answer: 1 Explanation: 1. Supraventricular tachycardia episodes are initially treated with vagal maneuvers to slow the heart rate when the infant is stable. In stable infants, the application of ice or iced saline solution to the face can reduce the heart rate. The infant is not capable of performing Valsalva's maneuver. Calcium channel blockers, not beta blockers, are the drugs of choice. Cardioversion is used in an urgent situation, but is not typically the initial treatment. 2. Supraventricular tachycardia episodes are initially treated with vagal maneuvers to slow the heart rate when the infant is stable. In stable infants, the application of ice or iced saline solution to the face can reduce the heart rate. The infant is not capable of performing Valsalva's maneuver. Calcium channel blockers, not beta blockers, are the drugs of choice. Cardioversion is used in an urgent situation, but is not typically the initial treatment. 3. Supraventricular tachycardia episodes are initially treated with vagal maneuvers to slow the heart rate when the infant is stable. In stable infants, the application of ice or iced saline solution to the face can reduce the heart rate. The infant is not capable of performing Valsalva's maneuver. Calcium channel blockers, not beta blockers, are the drugs of choice. Cardioversion is used in an urgent situation, but is not typically the initial treatment. 4. Supraventricular tachycardia episodes are initially treated with vagal maneuvers to slow the heart rate when the infant is stable. In stable infants, the application of ice or iced saline solution to the face can reduce the heart rate. The infant is not capable of performing Valsalva's maneuver. Calcium channel blockers, not beta blockers, are the drugs of choice. Cardioversion is used in an urgent situation, but is not typically the initial treatment. Page Ref: 555 Cognitive Level: Analyzing Client Need &Sub: Physiological Integrity: Physiological Adaptation Standards: QSEN Competencies: Patient-centered care | AACN Essential Competencies: Essential IX: Baccalaureate generalist nursing practice | NLN Competencies: Human flourishing | Nursing/Integrated Concepts: Nursing Process: Implementation/Coordination of care Learning Outcome: LO 21.2 Describe the pathophysiology associated with congenital heart defects with increased pulmonary circulation, decreased pulmonary circulation, mixed defects, and obstructed systemic blood flow. MNL LO: Cardiovascular and Hematological Disorders/Examine etiology, risk factors, pathophysiology, and clinical manifestations as seen in children. 15) The nurse is preparing to discharge an infant with a congenital heart defect. The infant will be cared for at home by the parents until surgery. Which items will the nurse include in the discharge teaching for this infant and family? Select all that apply. 1. Allow the infant to feed for 60 minutes. 2. Hold the infant at a 45-degree angle. 3. Encourage frequent hand hygiene. 4. Notify the health care provider for fever. 5. Pump the breasts and feed with a bottle if weight gain is an issue. Answer: 2, 3, 4, 5 Explanation: 1. Children are often managed at home until surgery. The parents should hold the infant at a 45-degree angle to decrease tachypnea. The parents should also encourage frequent hand hygiene to decrease the risk of infection. It is important to notify the health care provider for a fever, as the infant will be at risk for dehydration and digoxin toxicity. If the mother is breastfeeding and the infant is losing weight, the mother should be encouraged to pump the milk and feed the infant from a bottle, but each feeding should be limited to 30 minutes. Tube feedings may be needed for this infant to conserve calories expenditure. 2. Children are often managed at home until surgery. The parents should hold the infant at a 45- degree angle to decrease tachypnea. The parents should also encourage frequent hand hygiene to decrease the risk of infection. It is important to notify the health care provider for a fever, as the infant will be at risk for dehydration and digoxin toxicity. If the mother is breastfeeding and the infant is losing weight, the mother should be encouraged to pump the milk and feed the infant from a bottle, but each feeding should be limited to 30 minutes. Tube feedings may be needed for this infant to conserve calories expenditure. 3. Children are often managed at home until surgery. The parents should hold the infant at a 45- degree angle to decrease tachypnea. The parents should also encourage frequent hand hygiene to decrease the risk of infection. It is important to notify the health care provider for a fever, as the infant will be at risk for dehydration and digoxin toxicity. If the mother is breastfeeding and the infant is losing weight, the mother should be encouraged to pump the milk and feed the infant from a bottle, but each feeding should be limited to 30 minutes. Tube feedings may be needed for this infant to conserve calories expenditure. 4. Children are often managed at home until surgery. The parents should hold the infant at a 45- degree angle to decrease tachypnea. The parents should also encourage frequent hand hygiene to decrease the risk of infection. It is important to notify the health care provider for a fever, as the infant will be at risk for dehydration and digoxin toxicity. If the mother is breastfeeding and the infant is losing weight, the mother should be encouraged to pump the milk and feed the infant from a bottle, but each feeding should be limited to 30 minutes. Tube feedings may be needed for this infant to conserve calories expenditure. 5. Children are often managed at home until surgery. The parents should hold the infant at a 45- degree angle to decrease tachypnea. The parents should also encourage frequent hand hygiene to decrease the risk of infection. It is important to notify the health care provider for a fever, as the infant will be at risk for dehydration and digoxin toxicity. If the mother is breastfeeding and the infant is losing weight, the mother should be encouraged to pump the milk and feed the infant from a bottle, but each feeding should be limited to 30 minutes. Tube feedings may be needed for this infant to conserve calories expenditure. Page Ref: 539 Cognitive Level: Applying Client Need &Sub: Safe and Effective Care Environment: Management of Care Standards: QSEN Competencies: Patient-centered care | AACN Essential Competencies: Essential IX: Baccalaureate generalist nursing practice | NLN Competencies: Human flourishing | Nursing/Integrated Concepts: Nursing Process: Implementation/Coordination of care Learning Outcome: LO 21.3 Develop a nursing care plan for the infant with a congenital heart defect cared for at home prior to corrective surgery. MNL LO: Cardiovascular and Hematological Disorders/Educate the child and family on the care of the child during hospitalization and upon discharge. 16) The nurse is providing care to an adolescent child who is at risk for developing adult-onset cardiovascular disease. Which teaching points will decrease the adolescent's risk? Select all that apply. 1. Encourage a decrease in smoking. 2. Limit fat intake to 20 to 35 percent of intake. 3. Encourage participation in vigorous exercise for at least 30 minutes. 4. Maintain a normal weight. 5. Include high-fat dairy products in the daily diet. Answer: 2, 3, 4 Explanation: 1. Teaching points that will decrease the adolescent's risk of developing adult- onset cardiovascular disease include: limiting fat intake to 20 to 35 percent of total daily intake; encouraging the participation in vigorous exercise at least 30 minutes each day; and maintaining a normal weight. The adolescent and family members should be encouraged to stop smoking, not just to decrease smoking. The family should be educated to include low-fat dairy products in the daily diet. 2. Teaching points that will decrease the adolescent's risk of developing adult-onset cardiovascular disease include: limiting fat intake to 20 to 35 percent of total daily intake; encouraging the participation in vigorous exercise at least 30 minutes each day; and maintaining a normal weight. The adolescent and family members should be encouraged to stop smoking, not just to decrease smoking. The family should be educated to include low-fat dairy products in the daily diet. 3. Teaching points that will decrease the adolescent's risk of developing adult-onset cardiovascular disease include: limiting fat intake to 20 to 35 percent of total daily intake; encouraging the participation in vigorous exercise at least 30 minutes each day; and maintaining a normal weight. The adolescent and family members should be encouraged to stop smoking, not just to decrease smoking. The family should be educated to include low-fat dairy products in the daily diet. 4. Teaching points that will decrease the adolescent's risk of developing adult-onset cardiovascular disease include: limiting fat intake to 20 to 35 percent of total daily intake; encouraging the participation in vigorous exercise at least 30 minutes each day; and maintaining a normal weight. The adolescent and family members should be encouraged to stop smoking, not just to decrease smoking. The family should be educated to include low-fat dairy products in the daily diet. 5. Teaching points that will decrease the adolescent's risk of developing adult-onset cardiovascular disease include: limiting fat intake to 20 to 35 percent of total daily intake; encouraging the participation in vigorous exercise at least 30 minutes each day; and maintaining a normal weight. The adolescent and family members should be encouraged to stop smoking, not just to decrease smoking. The family should be educated to include low-fat dairy products in the daily diet. Page Ref: 556 Cognitive Level: Applying Client Need &Sub: Health Promotion and Maintenance Standards: QSEN Competencies: Patient-centered care | AACN Essential Competencies: Essential VII: Clinical prevention and population health | NLN Competencies: Nursing judgement | Nursing/Integrated Concepts: Nursing Process: Implementation/Health teaching and health promotion Learning Outcome: LO 21.8 List strategies to reduce a child's risk of adult-onset cardiovascular disease. MNL LO: Cardiovascular and Hematological Disorders/Examine etiology, risk factors, pathophysiology, and clinical manifestations as seen in children. 17) The nurse is providing care to a school-age client admitted to the emergency department following a motor vehicle crash. The client is exhibiting symptoms of hypovolemic shock. Which nursing interventions are appropriate for this client? Select all that apply. 1. Monitor hemoglobin and hematocrit. 2. Monitor liver enzymes. 3. Administer oxygen, as needed. 4. Administer a dextrose solution. 5. Monitor blood glucose. Answer: 1, 3, 5 Explanation: 1. Nursing care for a client experiencing hypovolemic shock is aimed at monitoring the child's condition and response to clinical therapy. It is appropriate for the nurse to monitor hemoglobin, hematocrit, and blood glucose. The nurse will also administer oxygen. The nurse will administer large volumes of crystalloid fluids (normal saline or lactated Ringer's), not dextrose. It is not necessary to monitor liver enzymes for this client. 2. Nursing care for a client experiencing hypovolemic shock is aimed at monitoring the child's condition and response to clinical therapy. It is appropriate for the nurse to monitor hemoglobin, hematocrit, and blood glucose. The nurse will also administer oxygen. The nurse will administer large volumes of crystalloid fluids (normal saline or lactated Ringer's), not dextrose. It is not necessary to monitor liver enzymes for this client. 3. Nursing care for a client experiencing hypovolemic shock is aimed at monitoring the child's condition and response to clinical therapy. It is appropriate for the nurse to monitor hemoglobin, hematocrit, and blood glucose. The nurse will also administer oxygen. The nurse will administer large volumes of crystalloid fluids (normal saline or lactated Ringer's), not dextrose. It is not necessary to monitor liver enzymes for this client. 4. Nursing care for a client experiencing hypovolemic shock is aimed at monitoring the child's condition and response to clinical therapy. It is appropriate for the nurse to monitor hemoglobin, hematocrit, and blood glucose. The nurse will also administer oxygen. The nurse will administer large volumes of crystalloid fluids (normal saline or lactated Ringer's), not dextrose. It is not necessary to monitor liver enzymes for this client. 5. Nursing care for a client experiencing hypovolemic shock is aimed at monitoring the child's condition and response to clinical therapy. It is appropriate for the nurse to monitor hemoglobin, hematocrit, and blood glucose. The nurse will also administer oxygen. The nurse will administer large volumes of crystalloid fluids (normal saline or lactated Ringer's), not dextrose. It is not necessary to monitor liver enzymes for this client. Page Ref: 558 Cognitive Level: Applying Client Need &Sub: Safe and Effective Care Environment: Management of Care Standards: QSEN Competencies: Patient-centered care | AACN Essential Competencies: Essential IX: Baccalaureate generalist nursing practice | NLN Competencies: Human flourishing | Nursing/Integrated Concepts: Nursing Process: Implementation/Coordination of care Learning Outcome: LO 21.10 Plan the nursing management of hypovolemic shock. MNL LO: Cardiovascular and Hematological Disorders/Differentiate treatment options and strategies for continuity of care for the child and family. 18) The child and family come to the clinic requesting information about causes of cardiac defects. The father has high incidence of cardiac defects in his family, and the child is frequently cyanotic around the lips. What causes should the nurse tell the family about? Select all that apply. 1. Decreased maternal age 2. Chromosomal abnormalities 3. Fetal exposure to maternal drugs 4. Maternal viral infections 5. Maternal metabolic disorders Answer: 2, 3, 4, 5 Explanation: 1. Cardiac defects may result from fetal exposure to maternal drugs, increased maternal age, chromosomal abnormalities, maternal viral infections, maternal metabolic disorders, and multifactorial genetic factors. 2. Cardiac defects may result from fetal exposure to maternal drugs, increased maternal age, chromosomal abnormalities, maternal viral infections, maternal metabolic disorders, and multifactorial genetic factors. 3. Cardiac defects may result from fetal exposure to maternal drugs, increased maternal age, chromosomal abnormalities, maternal viral infections, maternal metabolic disorders, and multifactorial genetic factors. 4. Cardiac defects may result from fetal exposure to maternal drugs, increased maternal age, chromosomal abnormalities, maternal viral infections, maternal metabolic disorders, and multifactorial genetic factors. 5. Cardiac defects may result from fetal exposure to maternal drugs, increased maternal age, chromosomal abnormalities, maternal viral infections, maternal metabolic disorders, and multifactorial genetic factors. Page Ref: 523 Cognitive Level: Analyzing Client Need &Sub: Health Promotion and Maintenance Standards: QSEN Competencies: Patient-centered care | AACN Essential Competencies: Essential VII: Clinical prevention and population health | NLN Competencies: Human flourishing | Nursing/Integrated Concepts: Nursing Process: Planning/Health teaching and health promotion Learning Outcome: LO 21.2 Describe the pathophysiology associated with congenital heart defects with increased pulmonary circulation, decreased pulmonary circulation, mixed defects, and obstructed systemic blood flow. MNL LO: Cardiovascular and Hematological Disorders/Examine etiology, risk factors, pathophysiology, and clinical manifestations as seen in children. 19) The family has just been informed by the healthcare provider that their newborn is diagnosed with a congenital heart defect, Tetralogy of Fallot (TOF). The family tells the nurse that the healthcare provider told them that TOF is comprised of several defects, and they ask the nurse what the defects are. What will the nurse tell the family? Select all that apply. 1. Pulmonary stenosis 2. Coarctation of the aorta 3. Right ventricular hypertrophy 4. Ventral septal defect 5. Overriding aorta Answer: 1, 3, 4, 5 Explanation: 1. Four defects are involved with TOF include: pulmonary stenosis, right ventricular hypertrophy, ventral septal defect, and overriding aorta. 2. Four defects are involved with TOF include: pulmonary stenosis, right ventricular hypertrophy, ventral septal defect, and overriding aorta. 3. Four defects are involved with TOF include: pulmonary stenosis, right ventricular hypertrophy, ventral septal defect, and overriding aorta. 4. Four defects are involved with TOF include: pulmonary stenosis, right ventricular hypertrophy, ventral septal defect, and overriding aorta. 5. Four defects are involved with TOF include: pulmonary stenosis, right ventricular hypertrophy, ventral septal defect, and overriding aorta. Page Ref: 533-534 Cognitive Level: Analyzing Client Need &Sub: Health Promotion and Maintenance Standards: QSEN Competencies: Patient-centered care | AACN Essential Competencies: Essential VII: Clinical prevention and population health | NLN Competencies: Human flourishing | Nursing/Integrated Concepts: Nursing Process: Planning/Health teaching and health promotion Learning Outcome: LO 21.2 Describe the pathophysiology associated with congenital heart defects with increased pulmonary circulation, decreased pulmonary circulation, mixed defects, and obstructed systemic blood flow. MNL LO: Cardiovascular and Hematological Disorders/Examine etiology, risk factors, pathophysiology, and clinical manifestations as seen in children. Principles of Pediatric Nursing: Caring for Children, 7e (Ball et al.) Chapter 22 Alterations in Immune Function 1) A nurse begins an infusion of intravenous immune globulin (IVIG) to a child who has combined immunodeficiency disease. Which assessment finding indicates that the nurse should stop the infusion? 1. A mild headache 2. Clear yellow urine 3. Severe shaking, chills, and fever 4. Complaints of being "thirsty" Answer: 3 Explanation: 1. Hypersensitivity reaction can be seen with IVIG. The infusion should be started slowly and increased if there is no reaction. Shaking, chills, and fever can indicate a reaction. A mild headache is an adverse side effect of IVIG but not a severe reaction. Thirst is not an indication of a reaction. Voiding clear yellow urine is a normal finding. 2. Hypersensitivity reaction can be seen with IVIG. The infusion should be started slowly and increased if there is no reaction. Shaking, chills, and fever can indicate a reaction. A mild headache is an adverse side effect of IVIG but not a severe reaction. Thirst is not an indication of a reaction. Voiding clear yellow urine is a normal finding. 3. Hypersensitivity reaction can be seen with IVIG. The infusion should be started slowly and increased if there is no reaction. Shaking, chills, and fever can indicate a reaction. A mild headache is an adverse side effect of IVIG but not a severe reaction. Thirst is not an indication of a reaction. Voiding clear yellow urine is a normal finding. 4. Hypersensitivity reaction can be seen with IVIG. The infusion should be started slowly and increased if there is no reaction. Shaking, chills, and fever can indicate a reaction. A mild headache is an adverse side effect of IVIG but not a severe reaction. Thirst is not an indication of a reaction. Voiding clear yellow urine is a normal finding. Page Ref: 568 Cognitive Level: Applying Client Need &Sub: Physiological Integrity: Pharmacological and Parenteral Therapies Standards: QSEN Competencies: Patient-centered care | AACN Essential Competencies: Essential IX: Baccalaureate generalist nursing practice | NLN Competencies: Human flourishing | Nursing/Integrated Concepts: Nursing Process: Implementation/Coordination of care Learning Outcome: LO 22.1 Describe the structure and function of the immune system and apply that knowledge to the care of children with immunologic disorders. MNL LO: Immunity, Infection, and Communicable Disease/Examine etiology, risk factors, pathophysiology, and clinical manifestations as seen in children. 2) A nurse is administering an intramuscular vaccination to an infant diagnosed with Wiskott- Aldrich syndrome (WAS). Which reaction is the infant more at risk for due to the diagnosis of WAS? 1. Pain at injection site 2. Bleeding at injection site 3. Redness and swelling at injection site 4. Mild rash at injection site Answer: 2 Explanation: 1. Wiskott-Aldrich syndrome is characterized by thrombocytopenia, with bleeding tendencies appearing during the neonatal period. The syndrome would not put the child at higher risk for pain, redness, swelling, or rash at the injection site. 2. Wiskott-Aldrich syndrome is characterized by thrombocytopenia, with bleeding tendencies appearing during the neonatal period. The syndrome would not put the child at higher risk for pain, redness, swelling, or rash at the injection site. 3. Wiskott-Aldrich syndrome is characterized by thrombocytopenia, with bleeding tendencies appearing during the neonatal period. The syndrome would not put the child at higher risk for pain, redness, swelling, or rash at the injection site. 4. Wiskott-Aldrich syndrome is characterized by thrombocytopenia, with bleeding tendencies appearing during the neonatal period. The syndrome would not put the child at higher risk for pain, redness, swelling, or rash at the injection site. Page Ref: 570 Cognitive Level: Analyzing Client Need &Sub: Physiological Integrity: Pharmacological and Parenteral Therapies Standards: QSEN Competencies: Patient-centered care | AACN Essential Competencies: Essential IX: Baccalaureate generalist nursing practice | NLN Competencies: Human flourishing | Nursing/Integrated Concepts: Nursing Process: Assessment/Coordination of care Learning Outcome: LO 22.1 Describe the structure and function of the immune system and apply that knowledge to the care of children with immunologic disorders. MNL LO: Immunity, Infection, and Communicable Disease/Examine etiology, risk factors, pathophysiology, and clinical manifestations as seen in children. 3) A nurse is planning care for a child with human immunodeficiency virus (HIV). Which nursing diagnosis is the highest priority for this child? 1. Risk for Infection 2. Risk for Fluid-Volume Deficit 3. Ineffective Thermoregulation 4. Ineffective Tissue Perfusion, Peripheral Answer: 1 Explanation: 1. A child with HIV is at risk for a myriad of bacterial, viral, fungal, and opportunistic infections because of the effect of the virus on the immune system. Risk for Fluid- Volume Deficit, Ineffective Thermoregulation, and Ineffective Tissue Perfusion, Peripheral would not be priority problems with this disease process. 2. A child with HIV is at risk for a myriad of bacterial, viral, fungal, and opportunistic infections because of the effect of the virus on the immune system. Risk for Fluid-Volume Deficit, Ineffective Thermoregulation, and Ineffective Tissue Perfusion, Peripheral would not be priority problems with this disease process. 3. A child with HIV is at risk for a myriad of bacterial, viral, fungal, and opportunistic infections because of the effect of the virus on the immune system. Risk for Fluid-Volume Deficit, Ineffective Thermoregulation, and Ineffective Tissue Perfusion, Peripheral would not be priority problems with this disease process. 4. A child with HIV is at risk for a myriad of bacterial, viral, fungal, and opportunistic infections because of the effect of the virus on the immune system. Risk for Fluid-Volume Deficit, Ineffective Thermoregulation, and Ineffective Tissue Perfusion, Peripheral would not be priority problems with this disease process. Page Ref: 570-571 Cognitive Level: Applying Client Need &Sub: Safe and Effective Care Environment: Safety and Infection Control Standards: QSEN Competencies: Patient-centered care | AACN Essential Competencies: Essential IX: Baccalaureate generalist nursing practice | NLN Competencies: Human flourishing | Nursing/Integrated Concepts: Nursing Process: Diagnosis/Coordination of care Learning Outcome: LO 22.3 Develop a nursing care plan in partnership with the family for a child with human immunodeficiency virus (HIV infection). MNL LO: Immunity, Infection, and Communicable Disease/Apply the nursing process in providing care for the child and family. 4) A child is prescribed Didanosine (Videx), a nucleoside reverse transcriptase inhibitor, for human immunodeficiency virus (HIV). Which lab value will the nurse monitor closely for this child? 1. Potassium 2. Sodium 3. RBC count 4. Glucose Answer: 3 Explanation: 1. Didanosine (Videx) causes bone-marrow suppression with resulting anemia. RBC counts are monitored at least monthly for changes. Potassium and sodium are electrolytes, and glucose is a laboratory test for checking diabetes. Didanosine (Videx) does not affect these values. 2. Didanosine (Videx) causes bone-marrow suppression with resulting anemia. RBC counts are monitored at least monthly for changes. Potassium and sodium are electrolytes, and glucose is a laboratory test for checking diabetes. Didanosine (Videx) does not affect these values. 3. Didanosine (Videx) causes bone-marrow suppression with resulting anemia. RBC counts are monitored at least monthly for changes. Potassium and sodium are electrolytes, and glucose is a laboratory test for checking diabetes. Didanosine (Videx) does not affect these values. 4. Didanosine (Videx) causes bone-marrow suppression with resulting anemia. RBC counts are monitored at least monthly for changes. Potassium and sodium are electrolytes, and glucose is a laboratory test for checking diabetes. Didanosine (Videx) does not affect these values. Page Ref: 571-573 Cognitive Level: Applying Client Need &Sub: Physiological Integrity: Pharmacological and Parenteral Therapies Standards: QSEN Competencies: Patient-centered care | AACN Essential Competencies: Essential IX: Baccalaureate generalist nursing practice | NLN Competencies: Human flourishing | Nursing/Integrated Concepts: Nursing Process: Assessment/Coordination of care Learning Outcome: LO 22.3 Develop a nursing care plan in partnership with the family for a child with human immunodeficiency virus (HIV infection). MNL LO: Immunity, Infection, and Communicable Disease/Apply the nursing process in providing care for the child and family. 5) A child with human immunodeficiency virus is started on sulfamethoxazole and trimethoprim (Bactrim) for Pneumocystis carinii pneumonia (PCP) prophylaxis. The recommended dose is based on the trimethoprim (TMP) component and is 15 to 20 mg TMP/kg/day in divided doses every 6 to 8 hours. The child weighs 6.8 kg. The highest dose of TMP the child can receive a day is . Round your answer to the nearest whole number. Answer: 136 Explanation: 6.8 kg (the child's weight) is multiplied by 20 mg. This yields the answer, which is 136 mg a day. Page Ref: 571 Cognitive Level: Applying Client Need &Sub: Physiological Integrity: Pharmacological and Parenteral Therapies Standards: QSEN Competencies: Patient-centered care | AACN Essential Competencies: Essential IX: Baccalaureate generalist nursing practice | NLN Competencies: Human flourishing | Nursing/Integrated Concepts: Nursing Process: Implementation/Coordination of care Learning Outcome: LO 22.3 Develop a nursing care plan in partnership with the family for a child with human immunodeficiency virus (HIV infection). MNL LO: Immunity, Infection, and Communicable Disease/Apply the nursing process in providing care for the child and family. 6) The nurse is providing care to a preschool-age client who is diagnosed with acquired immune deficiency syndrome (AIDS). In planning the client's care, which vaccine is inappropriate for the client to receive? 1. Diphtheria and tetanus toxoids and acellular pertussis vaccine (DTaP) 2. Haemophilus influenzae type B (HIB conjugate vaccine) 3. Varicella vaccine 4. Hepatitis B vaccine (Hep B) Answer: 3 Explanation: 1. A child with an immune disorder should not be immunized with a live varicella vaccine because of the risk of contracting the disease. DTaP, HIB, and hepatitis B vaccinations are not live vaccines and should be given on schedule. 2. A child with an immune disorder should not be immunized with a live varicella vaccine because of the risk of contracting the disease. DTaP, HIB, and hepatitis B vaccinations are not live vaccines and should be given on schedule. 3. A child with an immune disorder should not be immunized with a live varicella vaccine because of the risk of contracting the disease. DTaP, HIB, and hepatitis B vaccinations are not live vaccines and should be given on schedule. 4. A child with an immune disorder should not be immunized with a live varicella vaccine because of the risk of contracting the disease. DTaP, HIB, and hepatitis B vaccinations are not live vaccines and should be given on schedule. Page Ref: 578 Cognitive Level: Applying Client Need &Sub: Health Promotion and Maintenance Standards: QSEN Competencies: Patient-centered care | AACN Essential Competencies: Essential IX: Baccalaureate generalist nursing practice | NLN Competencies: Human flourishing | Nursing/Integrated Concepts: Nursing Process: Implementation/Coordination of care Learning Outcome: LO 22.3 Develop a nursing care plan in partnership with the family for a child with human immunodeficiency virus (HIV infection). MNL LO: Immunity, Infection, and Communicable Disease/Apply the nursing process in providing care for the child and family. 7) A child with human immunodeficiency virus (HIV) also has oral candidiasis. Which type of mouth care solution will the nurse teach the child to use? 1. Normal saline 2. Listerine 3. Scope 4. Viscous lidocaine Answer: 1 Explanation: 1. The mouth care should be with a non-alcohol base. Normal saline can keep the child's lips and mouth moist. Listerine and Scope are commercial mouth rinses that can have an alcohol base and cause drying of the membranes. Viscous lidocaine causes numbing and could depress the gag reflex in a younger child. 2. The mouth care should be with a non-alcohol base. Normal saline can keep the child's lips and mouth moist. Listerine and Scope are commercial mouth rinses that can have an alcohol base and cause drying of the membranes. Viscous lidocaine causes numbing and could depress the gag reflex in a younger child. 3. The mouth care should be with a non-alcohol base. Normal saline can keep the child's lips and mouth moist. Listerine and Scope are commercial mouth rinses that can have an alcohol base and cause drying of the membranes. Viscous lidocaine causes numbing and could depress the gag reflex in a younger child. 4. The mouth care should be with a non-alcohol base. Normal saline can keep the child's lips and mouth moist. Listerine and Scope are commercial mouth rinses that can have an alcohol base and cause drying of the membranes. Viscous lidocaine causes numbing and could depress the gag reflex in a younger child. Page Ref: 577 Cognitive Level: Applying Client Need &Sub: Physiological Integrity: Pharmacological and Parenteral Therapies Standards: QSEN Competencies: Patient-centered care | AACN Essential Competencies: Essential VII: Clinical prevention and population health | NLN Competencies: Human flourishing | Nursing/Integrated Concepts: Nursing Process: Implementation/Health teaching and health promotion Learning Outcome: LO 22.3 Develop a nursing care plan in partnership with the family for a child with human immunodeficiency virus (HIV infection). MNL LO: Immunity, Infection, and Communicable Disease/Apply the nursing process in providing care for the child and family. 8) The nurse is providing care to an adolescent client diagnosed with systemic lupus erythematosus (SLE). Which action by the client indicates acceptance of body changes associated with SLE? 1. She refuses to attend school. 2. She doesn't want to attend any social functions. 3. She discusses the body changes with a peer. 4. She discusses the body changes with healthcare personnel only. Answer: 3 Explanation: 1. Peer interaction is important to the teen. Being able to discuss the changes to her body with a peer indicates acceptance of the change in body image. Discussing changes only with healthcare personnel does not indicate the teen has adjusted to body-image changes. Refusing to go to school or not going to social functions indicates nonacceptance of the changes to body image. 2. Peer interaction is important to the teen. Being able to discuss the changes to her body with a peer indicates acceptance of the change in body image. Discussing changes only with healthcare personnel does not indicate the teen has adjusted to body-image changes. Refusing to go to school or not going to social functions indicates nonacceptance of the changes to body image. 3. Peer interaction is important to the teen. Being able to discuss the changes to her body with a peer indicates acceptance of the change in body image. Discussing changes only with healthcare personnel does not indicate the teen has adjusted to body-image changes. Refusing to go to school or not going to social functions indicates nonacceptance of the changes to body image. 4. Peer interaction is important to the teen. Being able to discuss the changes to her body with a peer indicates acceptance of the change in body image. Discussing changes only with healthcare personnel does not indicate the teen has adjusted to body-image changes. Refusing to go to school or not going to social functions indicates nonacceptance of the changes to body image. Page Ref: 581 Cognitive Level: Analyzing Client Need &Sub: Psychosocial Integrity Standards: QSEN Competencies: Patient-centered care | AACN Essential Competencies: Essential IX: Baccalaureate generalist nursing practice | NLN Competencies: Human flourishing | Nursing/Integrated Concepts: Nursing Process: Evaluation/Coordination of care Learning Outcome: LO 22.4 Plan nursing care for the child with an autoimmune condition such as systemic lupus erythematosus or juvenile arthritis. MNL LO: Immunity, Infection, and Communicable Disease/Apply the nursing process in providing care for the child and family. 9) A school-age child diagnosed with rheumatoid arthritis asks the nurse to recommend an exercise activity. Which activity is most appropriate for this child? 1. Softball 2. Football 3. Swimming 4. Basketball Answer: 3 Explanation: 1. Swimming helps to exercise all of the extremities without putting undue stress on joints. Softball, football, and basketball could exacerbate joint discomfort. 2. Swimming helps to exercise all of the extremities without putting undue stress on joints. Softball, football, and basketball could exacerbate joint discomfort. 3. Swimming helps to exercise all of the extremities without putting undue stress on joints. Softball, football, and basketball could exacerbate joint discomfort. 4. Swimming helps to exercise all of the extremities without putting undue stress on joints. Softball, football, and basketball could exacerbate joint discomfort. Page Ref: 583 Cognitive Level: Applying Client Need &Sub: Health Promotion and Maintenance Standards: QSEN Competencies: Patient-centered care | AACN Essential Competencies: Essential IX: Baccalaureate generalist nursing practice | NLN Competencies: Human flourishing | Nursing/Integrated Concepts: Nursing Process: Implementation/Coordination of care Learning Outcome: LO 22.4 Plan nursing care for the child with an autoimmune condition such as systemic lupus erythematosus or juvenile arthritis. MNL LO: Immunity, Infection, and Communicable Disease/Apply the nursing process in providing care for the child and family. 10) The nurse is caring for an adolescent client diagnosed with rheumatoid arthritis. Which nonpharmacological measure to reduce joint pain is most appropriate for the nurse to recommend to this client? 1. Moist heat 2. Elevation of extremity 3. Massage 4. Immobilization Answer: 1 Explanation: 1. Moist heat can promote relief of pain and decrease joint stiffness. Elevation of extremity would not have an effect on reducing pain in rheumatoid arthritis. Massage of extremities should be avoided because of a potential risk for emboli. Immobilization can lead to contractures, and range of motion to the involved joint should be maintained. 2. Moist heat can promote relief of pain and decrease joint stiffness. Elevation of extremity would not have an effect on reducing pain in rheumatoid arthritis. Massage of extremities should be avoided because of a potential risk for emboli. Immobilization can lead to contractures, and range of motion to the involved joint should be maintained. 3. Moist heat can promote relief of pain and decrease joint stiffness. Elevation of extremity would not have an effect on reducing pain in rheumatoid arthritis. Massage of extremities should be avoided because of a potential risk for emboli. Immobilization can lead to contractures, and range of motion to the involved joint should be maintained. 4. Moist heat can promote relief of pain and decrease joint stiffness. Elevation of extremity would not have an effect on reducing pain in rheumatoid arthritis. Massage of extremities should be avoided because of a potential risk for emboli. Immobilization can lead to contractures, and range of motion to the involved joint should be maintained. Page Ref: 583 Cognitive Level: Analyzing Client Need &Sub: Physiological Integrity: Reduction of Risk Potential Standards: QSEN Competencies: Patient-centered care | AACN Essential Competencies: Essential IX: Baccalaureate generalist nursing practice | NLN Competencies: Human flourishing | Nursing/Integrated Concepts: Nursing Process: Implementation/Coordination of care Learning Outcome: LO 22.4 Plan nursing care for the child with an autoimmune condition such as systemic lupus erythematosus or juvenile arthritis. MNL LO: Immunity, Infection, and Communicable Disease/Apply the nursing process in providing care for the child and family. 11) The nurse is providing discharge teaching to a school-age client who was recently diagnosed with a latex allergy. Which product will the nurse educate the client and family to avoid? 1. Plastic bottles 2. Footballs 3. Chewing gum 4. Paper bags Answer: 3 Explanation: 1. When a child is diagnosed with a latex allergy, it is essential for the nurse to educate both the child and the family regarding sources of latex within the home and the community. The child and family should be educated to avoid chewing gum as it contains latex. The other items do not contain latex and do not pose a risk for this child in the community. 2. When a child is diagnosed with a latex allergy, it is essential for the nurse to educate both the child and the family regarding sources of latex within the home and the community. The child and family should be educated to avoid chewing gum as it contains latex. The other items do not contain latex and do not pose a risk for this child in the community. 3. When a child is diagnosed with a latex allergy, it is essential for the nurse to educate both the child and the family regarding sources of latex within the home and the community. The child and family should be educated to avoid chewing gum as it contains latex. The other items do not contain latex and do not pose a risk for this child in the community. 4. When a child is diagnosed with a latex allergy, it is essential for the nurse to educate both the child and the family regarding sources of latex within the home and the community. The child and family should be educated to avoid chewing gum as it contains latex. The other items do not contain latex and do not pose a risk for this child in the community. Page Ref: 586 Cognitive Level: Applying Client Need &Sub: Physiological Integrity: Reduction of Risk Potential Standards: QSEN Competencies: Patient-centered care | AACN Essential Competencies: Essential IX: Baccalaureate generalist nursing practice | NLN Competencies: Human flourishing | Nursing/Integrated Concepts: Nursing Process: Implementation/Coordination of care Learning Outcome: LO 22.5 Identify exposure prevention measures for the child with latex allergy. MNL LO: Immunity, Infection, and Communicable Disease/Differentiate treatment options and strategies for continuity of care for the child and family. 12) An HIV-positive mother states she is relieved after the birth of her child to hear that the child is HIV-negative. Which response by the nurse is the most appropriate? 1. "Symptoms could still appear over the next 2 years." 2. "You took good care of yourself, so your child did not get HIV." 3. "We will assess for signs of pneumonia to be sure." 4. "The test will be repeated in 1 week to verify the negative status." Answer: 1 Explanation: 1. Symptoms of HIV could still manifest within the first 2 years. An infant is retested 1 to 2 months after the initial negative result. The HIV-positive mother can infect the newborn regardless of how well she takes care of herself once she is HIV-positive. There is no reason to assess for signs of pneumonia if the newborn is HIV-negative. 2. Symptoms of HIV could still manifest within the first 2 years. An infant is retested in 1 to 2 months after the initial negative result. The HIV-positive mother can infect the newborn regardless of how well she takes care of herself once she is HIV-positive. There is no reason to assess for signs of pneumonia if the newborn is HIV-negative. 3. Symptoms of HIV could still manifest within the first 2 years. An infant is retested in 1 to 2 months after the initial negative result. The HIV-positive mother can infect the newborn regardless of how well she takes care of herself once she is HIV-positive. There is no reason to assess for signs of pneumonia if the newborn is HIV-negative. 4. Symptoms of HIV could still manifest within the first 2 years. An infant is retested in 1 to 2 months after the initial negative result. The HIV-positive mother can infect the newborn regardless of how well she takes care of herself once she is HIV-positive. There is no reason to assess for signs of pneumonia if the newborn is HIV-negative. Page Ref: 571 Cognitive Level: Applying Client Need &Sub: Psychosocial Integrity Standards: QSEN Competencies: Patient-centered care | AACN Essential Competencies: Essential IX: Baccalaureate generalist nursing practice | NLN Competencies: Human flourishing | Nursing/Integrated Concepts: Nursing Process: Diagnosis/Coordination of care Learning Outcome: LO 22.3 Develop a nursing care plan in partnership with the family for a child with human immunodeficiency virus (HIV infection). MNL LO: Immunity, Infection, and Communicable Disease/Apply the nursing process in providing care for the child and family. 13) Parents of a child who experienced a moderately severe allergic reaction after eating peanuts ask the nurse what they can do to help if it happens again. Which response by the nurse is the most appropriate? 1. "If it happens again, I will teach you what to do." 2. "You should have an antihistamine like Benadryl with you at all times." 3. "We can start a desensitization process to take the allergy away." 4. "I will teach you how to use an Epi-Pen." Answer: 4 Explanation: 1. An Epi-Pen is the appropriate treatment if this reaction occurs again. Benadryl is fine, but most likely is not strong enough in light of the serious reaction the child had. Desensitization is not the appropriate instruction at this time. Telling the parents that they will be taught if it happens again is brushing off the seriousness of the situation. 2. An Epi-Pen is the appropriate treatment if this reaction occurs again. Benadryl is fine, but most likely is not strong enough in light of the serious reaction the child had. Desensitization is not the appropriate instruction at this time. Telling the parents that they will be taught if it happens again is brushing off the seriousness of the situation. 3. An Epi-Pen is the appropriate treatment if this reaction occurs again. Benadryl is fine, but most likely is not strong enough in light of the serious reaction the child had. Desensitization is not the appropriate instruction at this time. Telling the parents that they will be taught if it happens again is brushing off the seriousness of the situation. 4. An Epi-Pen is the appropriate treatment if this reaction occurs again. Benadryl is fine, but most likely is not strong enough in light of the serious reaction the child had. Desensitization is not the appropriate instruction at this time. Telling the parents that they will be taught if it happens again is brushing off the seriousness of the situation. Page Ref: 586 Cognitive Level: Applying Client Need &Sub: Physiological Integrity: Pharmacological and Parenteral Therapies Standards: QSEN Competencies: Patient-centered care | AACN Essential Competencies: Essential IX: Baccalaureate generalist nursing practice | NLN Competencies: Human flourishing | Nursing/Integrated Concepts: Nursing Process: Implementation/Health teaching and health promotion Learning Outcome: LO 22.6 Determine nursing interventions and prevention measures for the child experiencing hypersensitivity reactions. MNL LO: Immunity, Infection, and Communicable Disease/Differentiate treatment options and strategies for continuity of care for the child and family. 14) A preschool-age child has just had a moderate reaction to latex. When teaching the parents about latex allergy, the nurse should inform the parents of what common household items that contain latex? Select all that apply. 1. Rubber bands 2. Sneakers 3. Toothbrushes 4. Big Wheel® tricycle 5. Water toys Answer: 1, 2, 3, 5 Explanation: 1. Rubber bands, sneakers, toothbrushes, and water toys are household items that might contain latex. A Big Wheel® tricycle is plastic and does not contain latex. 2. Rubber bands, sneakers, toothbrushes, and water toys are household items that might contain latex. A Big Wheel® tricycle is plastic and does not contain latex. 3. Rubber bands, sneakers, toothbrushes, and water toys are household items that might contain latex. A Big Wheel® tricycle is plastic and does not contain latex. 4. Rubber bands, sneakers, toothbrushes, and water toys are household items that might contain latex. A Big Wheel® tricycle is plastic and does not contain latex. 5. Rubber bands, sneakers, toothbrushes, and water toys are household items that might contain latex. A Big Wheel® tricycle is plastic and does not contain latex. Page Ref: 586 Cognitive Level: Applying Client Need &Sub: Physiological Integrity: Reduction of Risk Potential Standards: QSEN Competencies: Patient-centered care | AACN Essential Competencies: Essential VII: Clinical prevention and population health | NLN Competencies: Human flourishing | Nursing/Integrated Concepts: Nursing Process: Implementation/Health teaching and health promotion Learning Outcome: LO 22.5 Identify exposure prevention measures for the child with latex allergy. MNL LO: Immunity, Infection, and Communicable Disease/Differentiate treatment options and strategies for continuity of care for the child and family. 15) A child comes to the clinic for an assessment 20 days post-bone marrow transplant. Which system should receive the highest priority during the nursing assessment? 1. Integumentary 2. Gastrointestinal 3. Respiratory 4. Cardiovascular Answer: 1 Explanation: 1. The skin is most commonly affected in graft-versus-host disease after a transplant. A pruritic, macular papular rash and a blistering, burning sensation can occur. The other systems are important to assess, but are not the highest priority. 2. The skin is most commonly affected in graft-versus-host disease after a transplant. A pruritic, macular papular rash and a blistering, burning sensation can occur. The other systems are important to assess, but are not the highest priority. 3. The skin is most commonly affected in graft-versus-host disease after a transplant. A pruritic, macular papular rash and a blistering, burning sensation can occur. The other systems are important to assess, but are not the highest priority. 4. The skin is most commonly affected in graft-versus-host disease after a transplant. A pruritic, macular papular rash and a blistering, burning sensation can occur. The other systems are important to assess, but are not the highest priority. Page Ref: 587 Cognitive Level: Analyzing Client Need &Sub: Physiological Integrity: Physiological Adaptation Standards: QSEN Competencies: Patient-centered care | AACN Essential Competencies: Essential IX: Baccalaureate generalist nursing practice | NLN Competencies: Human flourishing | Nursing/Integrated Concepts: Nursing Process: Assessment/Coordination of care Learning Outcome: LO 22.1 Describe the structure and function of the immune system and apply that knowledge to the care of children with immunologic disorders. MNL LO: Immunity, Infection, and Communicable Disease/Examine etiology, risk factors, pathophysiology, and clinical manifestations as seen in children. 16) The nurse is providing care to a school-age client with a documented immunodeficiency who is admitted to the general pediatric unit for intravenous medication administration. Which interventions are appropriate for this client? Select all that apply. 1. Institute droplet precautions. 2. Place in a positive-pressure room. 3. Avoid live vaccines. 4. Perform frequent handwashing. 5. Recommend fresh fruits brought in by the family. Answer: 2, 3, 4 Explanation: 1. Pediatric clients with documented immunodeficiency require specific interventions to decrease their risk for developing infections while in the hospital environment. Appropriate interventions for this client include a positive-pressure room, avoiding live vaccines, and meticulous handwashing from staff and visitors. This client would require standard precautions, not droplet precautions. Because of the risk of infection with fresh fruit, the family would not be allowed to bring this to the client during their hospital stay. 2. Pediatric clients with documented immunodeficiency require specific interventions to decrease their risk for developing infections while in the hospital environment. Appropriate interventions for this client include a positive-pressure room, avoiding live vaccines, and meticulous handwashing from staff and visitors. This client would require standard precautions, not droplet precautions. Because of the risk of infection with fresh fruit, the family would not be allowed to bring this to the client during their hospital stay. 3. Pediatric clients with documented immunodeficiency require specific interventions to decrease their risk for developing infections while in the hospital environment. Appropriate interventions for this client include a positive-pressure room, avoiding live vaccines, and meticulous handwashing from staff and visitors. This client would require standard precautions, not droplet precautions. Because of the risk of infection with fresh fruit, the family would not be allowed to bring this to the client during their hospital stay. 4. Pediatric clients with documented immunodeficiency require specific interventions to decrease their risk for developing infections while in the hospital environment. Appropriate interventions for this client include a positive-pressure room, avoiding live vaccines, and meticulous handwashing from staff and visitors. This client would require standard precautions, not droplet precautions. Because of the risk of infection with fresh fruit, the family would not be allowed to bring this to the client during their hospital stay. 5. Pediatric clients with documented immunodeficiency require specific interventions to decrease their risk for developing infections while in the hospital environment. Appropriate interventions for this client include a positive-pressure room, avoiding live vaccines, and meticulous handwashing from staff and visitors. This client would require standard precautions, not droplet precautions. Because of the risk of infection with fresh fruit, the family would not be allowed to bring this to the client during their hospital stay. Page Ref: 574 Cognitive Level: Applying Client Need &Sub: Safe and Effective Care Environment: Safety and Infection Control Standards: QSEN Competencies: Patient-centered care | AACN Essential Competencies: Essential IX: Baccalaureate generalist nursing practice | NLN Competencies: Human flourishing | Nursing/Integrated Concepts: Nursing Process: Implementation/Coordination of care Learning Outcome: LO 22.2 Summarize infection control measures needed for children with an immunodeficiency. MNL LO: Immunity, Infection, and Communicable Disease/Educate the child and family on the care of the child during hospitalization and upon discharge. 17) The nurse is caring for the adolescent with systemic lupus erythematosus (SLE). What nursing diagnoses would the nurse address? Select all that apply. 1. Activity intolerance 2. Risk for impaired skin integrity 3. Body image disturbed 4. Ineffective breathing pattern 5. Risk for infection Answer: 2, 3, 5 Explanation: 1. Nursing diagnoses that may apply to the adolescent with SLE are: risk for impaired skin integrity, risk for activity intolerance, disturbed body image, risk for infection, acute pain, and ineffective family therapeutic regimen management. 2. Nursing diagnoses that may apply to the adolescent with SLE are: risk for impaired skin integrity, risk for activity intolerance, disturbed body image, risk for infection, acute pain, and ineffective family therapeutic regimen management. 3. Nursing diagnoses that may apply to the adolescent with SLE are: risk for impaired skin integrity, risk for activity intolerance, disturbed body image, risk for infection, acute pain, and ineffective family therapeutic regimen management. 4. Nursing diagnoses that may apply to the adolescent with SLE are: risk for impaired skin integrity, risk for activity intolerance, disturbed body image, risk for infection, acute pain, and ineffective family therapeutic regimen management. 5. Nursing diagnoses that may apply to the adolescent with SLE are: risk for impaired skin integrity, risk for activity intolerance, disturbed body image, risk for infection, acute pain, and ineffective family therapeutic regimen management. Page Ref: 581 Cognitive Level: Applying Client Need &Sub: Safe and Effective Care Environment: Management of Care Standards: QSEN Competencies: Patient-centered care | AACN Essential Competencies: Essential IX: Baccalaureate generalist nursing practice | NLN Competencies: Human flourishing | Nursing/Integrated Concepts: Nursing Process: Diagnosis/Coordination of care Learning Outcome: LO 22.4 Plan nursing care for the child with an autoimmune condition such as systemic lupus erythematosus or juvenile arthritis. MNL LO: Immunity, Infection, and Communicable Disease/Examine etiology, risk factors, pathophysiology, and clinical manifestations as seen in children. 18) A child with the diagnosis of Wiskott-Aldrich syndrome has been ordered an IV infusion of gamma globulin. The child weighs 20 pounds. The healthcare provider orders: gamma globulin 2 g/kg IV over 12 hours. Calculate how many grams of gamma globulin will be given IV. Answer: 18 g Explanation: 18 g Page Ref: 570 Cognitive Level: Analyzing Client Need &Sub: Physiological Integrity: Pharmacological and Parenteral Therapies Standards: QSEN Competencies: Patient-centered care | AACN Essential Competencies: Essential IX: Baccalaureate generalist nursing practice | NLN Competencies: Human flourishing | Nursing/Integrated Concepts: Nursing Process: Planning/Coordination of care Learning Outcome: LO 22.4 Plan nursing care for the child with an autoimmune condition such as systemic lupus erythematosus or juvenile arthritis. MNL LO: Immunity, Infection, and Communicable Disease/Examine etiology, risk factors, pathophysiology, and clinical manifestations as seen in children. Principles of Pediatric Nursing: Caring for Children, 7e (Ball et al.) Chapter 23 Alterations in Hematologic Function 1) The nurse is evaluating the activity tolerance of a 9-month-old with iron deficiency anemia. Which finding indicates that the infant is not tolerating activity? 1. Heart rate of 138 2. Increased alertness 3. Respiratory rate less than 40 with activity 4. Muscle weakness Answer: 4 Explanation: 1. Iron deficiency anemia can result in less oxygen reaching the cells and tissues, causing activity intolerance. An indication that a 9-month-old child is not tolerating activity and that iron deficiency anemia is worsening would be the presence of muscle weakness during activity. A heart rate of 138, increased alertness, and a respiratory rate of less than 40 with activity are all signs that iron deficiency anemia is resolving and activity tolerance is improving. 2. Iron deficiency anemia can result in less oxygen reaching the cells and tissues, causing activity intolerance. An indication that a 9-month-old child is not tolerating activity and that iron deficiency anemia is worsening would be the presence of muscle weakness during activity. A heart rate of 138, increased alertness, and a respiratory rate of less than 40 with activity are all signs that iron deficiency anemia is resolving and activity tolerance is improving. 3. Iron deficiency anemia can result in less oxygen reaching the cells and tissues, causing activity intolerance. An indication that a 9-month-old child is not tolerating activity and that iron deficiency anemia is worsening would be the presence of muscle weakness during activity. A heart rate of 138, increased alertness, and a respiratory rate of less than 40 with activity are all signs that iron deficiency anemia is resolving and activity tolerance is improving. 4. Iron deficiency anemia can result in less oxygen reaching the cells and tissues, causing activity intolerance. An indication that a 9-month-old child is not tolerating activity and that iron deficiency anemia is worsening would be the presence of muscle weakness during activity. A heart rate of 138, increased alertness, and a respiratory rate of less than 40 with activity are all signs that iron deficiency anemia is resolving and activity tolerance is improving. Page Ref: 592 Cognitive Level: Analyzing Client Need &Sub: Physiological Integrity: Physiological Adaptation Standards: QSEN Competencies: Patient-centered care | AACN Essential Competencies: Essential IX: Baccalaureate generalist nursing practice | NLN Competencies: Human flourishing | Nursing/Integrated Concepts: Nursing Process: Evaluation/Coordination of care Learning Outcome: LO 23.2 Discuss the pathophysiology and clinical manifestations of the major disorders of RBCs affecting the pediatric population. MNL LO: Cardiovascular and Hematological Disorders/Examine etiology, risk factors, pathophysiology, and clinical manifestations as seen in children. 2) Which action by the parents demonstrates an understanding of the nurse's teaching with regard to prevention of iron-deficient anemia? 1. Feeding their infant with a formula that is not iron fortified 2. Starting iron-fortified infant cereal at 4 to 6 months of age 3. Introducing cow's milk at 6 months of age 4. Limiting vitamin C consumption after 1 year of age Answer: 2 Explanation: 1. Starting iron-fortified infant cereal at 4 to 6 months of age is recommended for prevention of iron deficiency in children. Infants who are not breast-fed should get iron-fortified formula. Cow's milk should not be introduced until 12 months of age. Vitamin C should be started at 6 to 9 months of age and continued, because foods rich in vitamin C improve iron absorption. 2. Starting iron-fortified infant cereal at 4 to 6 months of age is recommended for prevention of iron deficiency in children. Infants who are not breast-fed should get iron-fortified formula. Cow's milk should not be introduced until 12 months of age. Vitamin C should be started at 6 to 9 months of age and continued, because foods rich in vitamin C improve iron absorption. 3. Starting iron-fortified infant cereal at 4 to 6 months of age is recommended for prevention of iron deficiency in children. Infants who are not breast-fed should get iron-fortified formula. Cow's milk should not be introduced until 12 months of age. Vitamin C should be started at 6 to 9 months of age and continued, because foods rich in vitamin C improve iron absorption. 4. Starting iron-fortified infant cereal at 4 to 6 months of age is recommended for prevention of iron deficiency in children. Infants who are not breast-fed should get iron-fortified formula. Cow's milk should not be introduced until 12 months of age. Vitamin C should be started at 6 to 9 months of age and continued, because foods rich in vitamin C improve iron absorption. Page Ref: 593 Cognitive Level: Analyzing Client Need &Sub: Health Promotion and Maintenance Standards: QSEN Competencies: Patient-centered care | AACN Essential Competencies: Essential VII: Clinical prevention and population health | NLN Competencies: Human flourishing | Nursing/Integrated Concepts: Nursing Process: Evaluation/Health teaching and health promotion Learning Outcome: LO 23.5 Plan the nursing management and collaborative care of a child with a hematologic disorder. MNL LO: Cardiovascular and Hematological Disorders/Differentiate treatment options and strategies for continuity of care for the child and family. 3) A child is diagnosed with sickle cell disease. The parents are unsure how their child contracted the disease. Which explanation by the nurse is the most appropriate? 1. "Both the mother and the father have the sickle cell trait." 2. "The mother has the trait, but the father doesn't." 3. "The father has the trait, but the mother doesn't." 4. "The mother has sickle cell disease, but the father doesn't have the disease or the trait." Answer: 1 Explanation: 1. Sickle cell disease is an autosomal recessive disorder; both parents must have the trait in order for a child to have the disease. 2. Sickle cell disease is an autosomal recessive disorder; both parents must have the trait in order for a child to have the disease. 3. Sickle cell disease is an autosomal recessive disorder; both parents must have the trait in order for a child to have the disease. 4. Sickle cell disease is an autosomal recessive disorder; both parents must have the trait in order for a child to have the disease. Page Ref: 594 Cognitive Level: Applying Client Need &Sub: Health Promotion and Maintenance Standards: QSEN Competencies: Patient-centered care | AACN Essential Competencies: Essential VII: Clinical prevention and population health | NLN Competencies: Human flourishing | Nursing/Integrated Concepts: Nursing Process: Assessment/Health teaching and health promotion Learning Outcome: LO 23.2 Discuss the pathophysiology and clinical manifestations of the major disorders of RBCs affecting the pediatric population. MNL LO: Cardiovascular and Hematological Disorders/Examine etiology, risk factors, pathophysiology, and clinical manifestations as seen in children. 4) The charge nurse on a pediatric unit is making a room assignment for a school-age child diagnosed with sickle cell disease, who is in splenic sequestration crisis. Which room assignment is most appropriate for this client? 1. Semiprivate room 2. Reverse-isolation room 3. Contact-isolation room 4. Private room Answer: 4 Explanation: 1. Splenic sequestration can be life-threatening, and there is profound anemia. The child does not need an isolation room but should not be placed in a room with any child who may have an infectious illness. The private room is appropriate for this child. 2. Splenic sequestration can be life-threatening, and there is profound anemia. The child does not need an isolation room but should not be placed in a room with any child who may have an infectious illness. The private room is appropriate for this child. 3. Splenic sequestration can be life-threatening, and there is profound anemia. The child does not need an isolation room but should not be placed in a room with any child who may have an infectious illness. The private room is appropriate for this child. 4. Splenic sequestration can be life-threatening, and there is profound anemia. The child does not need an isolation room but should not be placed in a room with any child who may have an infectious illness. The private room is appropriate for this child. Page Ref: 596 Cognitive Level: Applying Client Need &Sub: Safe and Effective Care Environment: Safety and Infection Control Standards: QSEN Competencies: Patient-centered care | AACN Essential Competencies: Essential VII: Clinical prevention and population health | NLN Competencies: Human flourishing | Nursing/Integrated Concepts: Nursing Process: Implementation/Coordination of care Learning Outcome: LO 23.5 Plan the nursing management and collaborative care of a child with a hematologic disorder. MNL LO: Cardiovascular and Hematological Disorders/Apply the nursing process in providing care for the child and family. 5) The nurse is providing care for an adolescent client who is experiencing pain related to a sickle cell crisis. Which medication does the nurse prepare to administer to this client? 1. Morphine sulfate 2. Meperidine 3. Acetaminophen 4. Ibuprofen Answer: 1 Explanation: 1. The pain during a sickling crisis is severe, and morphine is needed for pain control around the clock or by patient-controlled analgesia (PCA). Meperidine is not used for pain control for clients with sickle cell pain crisis because it could cause seizures. Acetaminophen or ibuprofen is used for mild pain and would not be effective for the severe pain experienced by a child in sickle cell pain crisis. 2. The pain during a sickling crisis is severe, and morphine is needed for pain control around the clock or by patient-controlled analgesia (PCA). Meperidine is not used for pain control for clients with sickle cell pain crisis because it could cause seizures. Acetaminophen or ibuprofen is used for mild pain and would not be effective for the severe pain experienced by a child in sickle cell pain crisis. 3. The pain during a sickling crisis is severe, and morphine is needed for pain control around the clock or by patient-controlled analgesia (PCA). Meperidine is not used for pain control for clients with sickle cell pain crisis because it could cause seizures. Acetaminophen or ibuprofen is used for mild pain and would not be effective for the severe pain experienced by a child in sickle cell pain crisis. 4. The pain during a sickling crisis is severe, and morphine is needed for pain control around the clock or by patient-controlled analgesia (PCA). Meperidine is not used for pain control for clients with sickle cell pain crisis because it could cause seizures. Acetaminophen or ibuprofen is used for mild pain and would not be effective for the severe pain experienced by a child in sickle cell pain crisis. Page Ref: 598 Cognitive Level: Applying Client Need &Sub: Physiological Integrity: Pharmacological and Parenteral Therapies Standards: QSEN Competencies: Patient-centered care | AACN Essential Competencies: Essential IX: Baccalaureate generalist nursing practice | NLN Competencies: Nursing judgement | Nursing/Integrated Concepts: Nursing Process: Implementation/Coordination of care Learning Outcome: LO 23.5 Plan the nursing management and collaborative care of a child with a hematologic disorder. MNL LO: Cardiovascular and Hematological Disorders/Apply the nursing process in providing care for the child and family. 6) The nurse is teaching parents how to prevent a sickle cell crisis in the child with sickle cell disease. Which precipitating factors to a sickle cell crisis will the nurse include in the explanation? Select all that apply. 1. Fever 2. Dehydration 3. Regular exercise 4. Altitude 5. Increased fluid intake Answer: 1, 2, 4 Explanation: 1. Fever, dehydration, and altitude are precipitating factors contributing to a sickle cell crisis. Regular exercise and increased fluid intake are recommended activities for a child with sickle cell disease and will not contribute to a sickle cell crisis. 2. Fever, dehydration, and altitude are precipitating factors contributing to a sickle cell crisis. Regular exercise and increased fluid intake are recommended activities for a child with sickle cell disease and will not contribute to a sickle cell crisis. 3. Fever, dehydration, and altitude are precipitating factors contributing to a sickle cell crisis. Regular exercise and increased fluid intake are recommended activities for a child with sickle cell disease and will not contribute to a sickle cell crisis. 4. Fever, dehydration, and altitude are precipitating factors contributing to a sickle cell crisis. Regular exercise and increased fluid intake are recommended activities for a child with sickle cell disease and will not contribute to a sickle cell crisis. 5. Fever, dehydration, and altitude are precipitating factors contributing to a sickle cell crisis. Regular exercise and increased fluid intake are recommended activities for a child with sickle cell disease and will not contribute to a sickle cell crisis. Page Ref: 598 Cognitive Level: Analyzing Client Need &Sub: Health Promotion and Maintenance Standards: QSEN Competencies: Patient-centered care | AACN Essential Competencies: Essential VII: Clinical prevention and population health | NLN Competencies: Human flourishing | Nursing/Integrated Concepts: Nursing Process: Assessment/Health teaching and health promotion Learning Outcome: LO 23.2 Discuss the pathophysiology and clinical manifestations of the major disorders of RBCs affecting the pediatric population. MNL LO: Cardiovascular and Hematological Disorders/Examine etiology, risk factors, pathophysiology, and clinical manifestations as seen in children. 7) The nurse is administering packed RBCs to a child with sickle cell disease (SCD). The nurse is monitoring for a transfusion reaction and knows it is most likely to occur during which time frame? 1. Six hours after the transfusion is given 2. Within the first 20 minutes of administration of the transfusion 3. At the end of the administration of the transfusion 4. Never; children with SCD do not have reactions. Answer: 2 Explanation: 1. Blood reactions can occur as soon as the blood transfusion begins or within the first 20 minutes. The nurse should remain with the child for the first 20 minutes of the transfusion. 2. Blood reactions can occur as soon as the blood transfusion begins or within the first 20 minutes. The nurse should remain with the child for the first 20 minutes of the transfusion. 3. Blood reactions can occur as soon as the blood transfusion begins or within the first 20 minutes. The nurse should remain with the child for the first 20 minutes of the transfusion. 4. Blood reactions can occur as soon as the blood transfusion begins or within the first 20 minutes. The nurse should remain with the child for the first 20 minutes of the transfusion. Page Ref: 598 Cognitive Level: Analyzing Client Need &Sub: Physiological Integrity: Pharmacological and Parenteral Therapies Standards: QSEN Competencies: Patient-centered care | AACN Essential Competencies: Essential IX: Baccalaureate generalist nursing practice | NLN Competencies: Human flourishing | Nursing/Integrated Concepts: Nursing Process: Assessment/Coordination of care Learning Outcome: LO 23.5 Plan the nursing management and collaborative care of a child with a hematologic disorder. MNL LO: Cardiovascular and Hematological Disorders/Apply the nursing process in providing care for the child and family. 8) A child who has beta-thalassemia is receiving numerous blood transfusions. The child is also receiving deferoxamine (Desferal) therapy. The parents ask how the deferoxamine will help their child. Which rationale does the nurse use when responding to the parents? 1. It prevents blood transfusion reactions. 2. It stimulates RBC production. 3. It provides vitamin supplementation. 4. It prevents iron overload. Answer: 4 Explanation: 1. Iron overload can be a side effect of a hypertransfusion therapy. Deferoxamine (Desferal) is an iron-chelating drug, which binds excess iron so it can be excreted by the kidneys. It does not prevent blood-transfusion reactions, stimulate RBC production, or provide vitamin supplementation. 2. Iron overload can be a side effect of a hypertransfusion therapy. Deferoxamine (Desferal) is an iron-chelating drug, which binds excess iron so it can be excreted by the kidneys. It does not prevent blood-transfusion reactions, stimulate RBC production, or provide vitamin supplementation. 3. Iron overload can be a side effect of a hypertransfusion therapy. Deferoxamine (Desferal) is an iron-chelating drug, which binds excess iron so it can be excreted by the kidneys. It does not prevent blood-transfusion reactions, stimulate RBC production, or provide vitamin supplementation. 4. Iron overload can be a side effect of a hypertransfusion therapy. Deferoxamine (Desferal) is an iron-chelating drug, which binds excess iron so it can be excreted by the kidneys. It does not prevent blood-transfusion reactions, stimulate RBC production, or provide vitamin supplementation. Page Ref: 602 Cognitive Level: Analyzing Client Need &Sub: Physiological Integrity: Pharmacological and Parenteral Therapies Standards: QSEN Competencies: Patient-centered care | AACN Essential Competencies: Essential VII: Clinical prevention and population health | NLN Competencies: Human flourishing | Nursing/Integrated Concepts: Nursing Process: Planning/Health teaching and health promotion Learning Outcome: LO 23.5 Plan the nursing management and collaborative care of a child with a hematologic disorder. MNL LO: Cardiovascular and Hematological Disorders/Apply the nursing process in providing care for the child and family. 9) A child recently diagnosed with aplastic anemia is being prepared for discharge. When planning support for the family, which service should the nurse plan to include in the discharge plan? 1. Referrals to support groups and social services 2. Short-term support 3. Genetic counseling 4. Nutrition counseling Answer: 1 Explanation: 1. Families require support in dealing with a child who has a life-threatening disease. They should be referred to support groups for counseling, if indicated, and to social services. The support will be long term in nature. Aplastic anemia is not a genetically transmitted disease. Nutrition counseling is not a priority and may or may not be needed with aplastic anemia. 2. Families require support in dealing with a child who has a life-threatening disease. They should be referred to support groups for counseling, if indicated, and to social services. The support will be long term in nature. Aplastic anemia is not a genetically transmitted disease. Nutrition counseling is not a priority and may or may not be needed with aplastic anemia. 3. Families require support in dealing with a child who has a life-threatening disease. They should be referred to support groups for counseling, if indicated, and to social services. The support will be long term in nature. Aplastic anemia is not a genetically transmitted disease. Nutrition counseling is not a priority and may or may not be needed with aplastic anemia. 4. Families require support in dealing with a child who has a life-threatening disease. They should be referred to support groups for counseling, if indicated, and to social services. The support will be long term in nature. Aplastic anemia is not a genetically transmitted disease. Nutrition counseling is not a priority and may or may not be needed with aplastic anemia. Page Ref: 603 Cognitive Level: Applying Client Need &Sub: Psychosocial Integrity Standards: QSEN Competencies: Patient-centered care | AACN Essential Competencies: Essential IX: Baccalaureate generalist nursing practice | NLN Competencies: Nursing judgement | Nursing/Integrated Concepts: Nursing Process: Planning/Coordination of care Learning Outcome: LO 23.5 Plan the nursing management and collaborative care of a child with a hematologic disorder. MNL LO: Cardiovascular and Hematological Disorders/Educate the child and family on the care of the child during hospitalization and upon discharge. 10) A school-age child with hemophilia falls on the playground and goes to the nurse's office with superficial bleeding above the knee. Which action by the nurse is the most appropriate? 1. Apply a warm, moist pack to the area. 2. Perform some passive range of motion to the affected leg. 3. Apply pressure to the area for at least 15 minutes. 4. Keep the affected extremity in a dependent position. Answer: 3 Explanation: 1. If a hemophiliac child experiences a bleeding episode, superficial bleeding should be controlled by applying pressure to the area for at least 15 minutes. Ice should be applied, not heat. The extremity should be immobilized and elevated, so passive range of motion and keeping the extremity in a dependent position would not be appropriate interventions at this time. 2. If a hemophiliac child experiences a bleeding episode, superficial bleeding should be controlled by applying pressure to the area for at least 15 minutes. Ice should be applied, not heat. The extremity should be immobilized and elevated, so passive range of motion and keeping the extremity in a dependent position would not be appropriate interventions at this time. 3. If a hemophiliac child experiences a bleeding episode, superficial bleeding should be controlled by applying pressure to the area for at least 15 minutes. Ice should be applied, not heat. The extremity should be immobilized and elevated, so passive range of motion and keeping the extremity in a dependent position would not be appropriate interventions at this time. 4. If a hemophiliac child experiences a bleeding episode, superficial bleeding should be controlled by applying pressure to the area for at least 15 minutes. Ice should be applied, not heat. The extremity should be immobilized and elevated, so passive range of motion and keeping the extremity in a dependent position would not be appropriate interventions at this time. Page Ref: 604 Cognitive Level: Applying Client Need &Sub: Physiological Integrity: Basic Care and Comfort Standards: QSEN Competencies: Patient-centered care | AACN Essential Competencies: Essential IX: Baccalaureate generalist nursing practice | NLN Competencies: Nursing judgement | Nursing/Integrated Concepts: Nursing Process: Implementation/Coordination of care Learning Outcome: LO 23.5 Plan the nursing management and collaborative care of a child with a hematologic disorder. MNL LO: Cardiovascular and Hematological Disorders/Apply the nursing process in providing care for the child and family. 11) A child diagnosed with hemophilia plans on participating in a bicycling club. Which recommendation by the nurse is the most appropriate? 1. Consider a swim club instead of the bicycling club. 2. Wear kneepads, elbow pads, and a helmet while bicycling. 3. Participate only in the social activities of the club. 4. Not join the club. Answer: 2 Explanation: 1. Children with hemophilia should be encouraged to participate in noncontact sports activities. Bicycling is an excellent option and is recommended along with swimming. The child should always use kneepads, elbow pads, and a helmet when participating in a physical sport. Participating only in the social aspects of the club would not encourage physical activity. Discouraging a child from joining a club would not foster growth and development. 2. Children with hemophilia should be encouraged to participate in noncontact sports activities. Bicycling is an excellent option and is recommended along with swimming. The child should always use kneepads, elbow pads, and a helmet when participating in a physical sport. Participating only in the social aspects of the club would not encourage physical activity. Discouraging a child from joining a club would not foster growth and development. 3. Children with hemophilia should be encouraged to participate in noncontact sports activities. Bicycling is an excellent option and is recommended along with swimming. The child should always use kneepads, elbow pads, and a helmet when participating in a physical sport. Participating only in the social aspects of the club would not encourage physical activity. Discouraging a child from joining a club would not foster growth and development. 4. Children with hemophilia should be encouraged to participate in noncontact sports activities. Bicycling is an excellent option and is recommended along with swimming. The child should always use kneepads, elbow pads, and a helmet when participating in a physical sport. Participating only in the social aspects of the club would not encourage physical activity. Discouraging a child from joining a club would not foster growth and development. Page Ref: 604-605 Cognitive Level: Applying Client Need &Sub: Health Promotion and Maintenance Standards: QSEN Competencies: Patient-centered care | AACN Essential Competencies: Essential IX: Baccalaureate generalist nursing practice | NLN Competencies: Human flourishing | Nursing/Integrated Concepts: Nursing Process: Implementation/Coordination of care Learning Outcome: LO 23.5 Plan the nursing management and collaborative care of a child with a hematologic disorder. MNL LO: Cardiovascular and Hematological Disorders/Differentiate treatment options and strategies for continuity of care for the child and family. 12) The nurse is caring for a child with disseminated intravascular coagulation (DIC). Which nursing intervention is a priority for this child? 1. Frequent ambulation 2. Maintenance of skin integrity 3. Monitoring of fluid restriction 4. Preparation for x-ray procedures Answer: 2 Explanation: 1. Impairment of skin integrity can lead to bleeding in DIC. The child with DIC should be placed on bed rest. Fluids need to be monitored but will not be restricted, and DIC is not diagnosed with x-ray examination but by serum lab studies. 2. Impairment of skin integrity can lead to bleeding in DIC. The child with DIC should be placed on bed rest. Fluids need to be monitored but will not be restricted, and DIC is not diagnosed with x-ray examination but by serum lab studies. 3. Impairment of skin integrity can lead to bleeding in DIC. The child with DIC should be placed on bed rest. Fluids need to be monitored but will not be restricted, and DIC is not diagnosed with x-ray examination but by serum lab studies. 4. Impairment of skin integrity can lead to bleeding in DIC. The child with DIC should be placed on bed rest. Fluids need to be monitored but will not be restricted, and DIC is not diagnosed with x-ray examination but by serum lab studies. Page Ref: 606 Cognitive Level: Analyzing Client Need &Sub: Physiological Integrity: Reduction of Risk Potential Standards: QSEN Competencies: Patient-centered care | AACN Essential Competencies: Essential IX: Baccalaureate generalist nursing practice | NLN Competencies: Human flourishing | Nursing/Integrated Concepts: Nursing Process: Implementation/Coordination of care Learning Outcome: LO 23.5 Plan the nursing management and collaborative care of a child with a hematologic disorder. MNL LO: Cardiovascular and Hematological Disorders/Apply the nursing process in providing care for the child and family. 13) The nurse is providing care to a school-age client diagnosed with idiopathic thrombocytopenic purpura (ITP). Which nursing diagnosis is the priority for this client? 1. Risk for Injury 2. Ineffective Breathing Pattern 3. Nausea 4. Fluid-Volume Deficit. Answer: 1 Explanation: 1. ITP is the most common bleeding disorder in children, so risk for injury (bleeding) is the priority nursing diagnosis. The disease process does not usually cause ineffective breathing patterns, nausea, or fluid-volume deficits. 2. ITP is the most common bleeding disorder in children, so risk for injury (bleeding) is the priority nursing diagnosis. The disease process does not usually cause ineffective breathing patterns, nausea, or fluid-volume deficits. 3. ITP is the most common bleeding disorder in children, so risk for injury (bleeding) is the priority nursing diagnosis. The disease process does not usually cause ineffective breathing patterns, nausea, or fluid-volume deficits. 4. ITP is the most common bleeding disorder in children, so risk for injury (bleeding) is the priority nursing diagnosis. The disease process does not usually cause ineffective breathing patterns, nausea, or fluid-volume deficits. Page Ref: 606 Cognitive Level: Analyzing Client Need &Sub: Physiological Integrity: Basic Care and Comfort Standards: QSEN Competencies: Patient-centered care | AACN Essential Competencies: Essential IX: Baccalaureate generalist nursing practice | NLN Competencies: Human flourishing | Nursing/Integrated Concepts: Nursing Process: Diagnosis/Coordination of care Learning Outcome: LO 23.4 Discuss the pathophysiology and clinical manifestations of the major bleeding disorders affecting the pediatric population. MNL LO: Cardiovascular and Hematological Disorders/Examine etiology, risk factors, pathophysiology, and clinical manifestations as seen in children. 14) A child with meningococcemia is being admitted to the pediatric intensive-care unit. Which room assignment is the most appropriate for this child? 1. Semiprivate room 2. Private room, but not in isolation 3. Private room, in protective isolation 4. Private room, in respiratory isolation Answer: 4 Explanation: 1. Meningococcemia follows an infection with Neisseria meningitidis. N. meningitidis is transmitted through airborne droplets; thus, the child should be placed in a private room in respiratory isolation. A private room with protective isolation (child is essentially kept in a "bubble") would not be appropriate. 2. Meningococcemia follows an infection with Neisseria meningitidis. N. meningitidis is transmitted through airborne droplets; thus, the child should be placed in a private room in respiratory isolation. A private room with protective isolation (child is essentially kept in a "bubble") would not be appropriate. 3. Meningococcemia follows an infection with Neisseria meningitidis. N. meningitidis is transmitted through airborne droplets; thus, the child should be placed in a private room in respiratory isolation. A private room with protective isolation (child is essentially kept in a "bubble") would not be appropriate. 4. Meningococcemia follows an infection with Neisseria meningitidis. N. meningitidis is transmitted through airborne droplets; thus, the child should be placed in a private room in respiratory isolation. A private room with protective isolation (child is essentially kept in a "bubble") would not be appropriate. Page Ref: 606 Cognitive Level: Applying Client Need &Sub: Safe and Effective Care Environment: Safety and Infection Control Standards: QSEN Competencies: Patient-centered care | AACN Essential Competencies: Essential IX: Baccalaureate generalist nursing practice | NLN Competencies: Human flourishing | Nursing/Integrated Concepts: Nursing Process: Planning/Coordination of care Learning Outcome: LO 23.5 Plan the nursing management and collaborative care of a child with a hematologic disorder. MNL LO: Cardiovascular and Hematological Disorders/Apply the nursing process in providing care for the child and family. 15) A child who has undergone a hematopoietic stem cell transplantation (HSCT) is ready for discharge. Which items will the nurse include in the discharge teaching for this child and family? Select all that apply. 1. Recognize the signs of graft-versus-host disease. 2. Return the child to school within six weeks. 3. Practice good handwashing. 4. Avoid obtaining influenza vaccinations. 5. Avoid live plants and fresh vegetables. Answer: 1, 3, 5 Explanation: 1. A child who is preparing for discharge after a HSCT will require specific interventions to decrease the risk of contracting communicable illnesses. Appropriate teaching points include: recognizing the signs of graft-versus-host disease; practicing good handwashing; and avoiding live plants and fresh vegetables. The child will require home schooling for 6 to 12 months. The child and family members should be encouraged to obtain yearly influenza vaccinations. 2. A child who is preparing for discharge after a HSCT will require specific interventions to decrease the risk of contracting communicable illnesses. Appropriate teaching points include: recognizing the signs of graft-versus-host disease; practicing good handwashing; and avoiding live plants and fresh vegetables. The child will require home schooling for 6 to 12 months. The child and family members should be encouraged to obtain yearly influenza vaccinations. 3. A child who is preparing for discharge after a HSCT will require specific interventions to decrease the risk of contracting communicable illnesses. Appropriate teaching points include: recognizing the signs of graft-versus-host disease; practicing good handwashing; and avoiding live plants and fresh vegetables. The child will require home schooling for 6 to 12 months. The child and family members should be encouraged to obtain yearly influenza vaccinations. 4. A child who is preparing for discharge after a HSCT will require specific interventions to decrease the risk of contracting communicable illnesses. Appropriate teaching points include: recognizing the signs of graft-versus-host disease; practicing good handwashing; and avoiding live plants and fresh vegetables. The child will require home schooling for 6 to 12 months. The child and family members should be encouraged to obtain yearly influenza vaccinations. 5. A child who is preparing for discharge after a HSCT will require specific interventions to decrease the risk of contracting communicable illnesses. Appropriate teaching points include: recognizing the signs of graft-versus-host disease; practicing good handwashing; and avoiding live plants and fresh vegetables. The child will require home schooling for 6 to 12 months. The child and family members should be encouraged to obtain yearly influenza vaccinations. Page Ref: 607 Cognitive Level: Applying Client Need &Sub: Health Promotion and Maintenance Standards: QSEN Competencies: Patient-centered care | AACN Essential Competencies: Essential VII: Clinical prevention and population health | NLN Competencies: Human flourishing | Nursing/Integrated Concepts: Nursing Process: Implementation/Health teaching and health promotion Learning Outcome: LO 23.6 Prioritize nursing interventions for a child receiving hematopoietic stem cell transplantation (HSCT). MNL LO: Cardiovascular and Hematological Disorders/Educate the child and family on the care of the child during hospitalization and upon discharge. 16) The nurse is caring for a child who is in a sickle cell crisis and has severe pain. Which nursing intervention is the most appropriate for this child? 1. Giving comfort measures, such as back rubs 2. Suggesting diversional activities, such as coloring 3. Administering pain medication 4. Preparing the child for painful procedures Answer: 3 Explanation: 1. Severe pain requires administration of pain medication for pain relief. Comfort measures and diversional activities are not effective against severe pain in children. Comfort measures should be given to every child and can be used after pain medication is given. A child in severe pain is not capable of participating in or enjoying diversional activities. Preparing the child for painful procedures is not appropriate when the child is already in pain. 2. Severe pain requires administration of pain medication for pain relief. Comfort measures and diversional activities are not effective against severe pain in children. Comfort measures should be given to every child and can be used after pain medication is given. A child in severe pain is not capable of participating in or enjoying diversional activities. Preparing the child for painful procedures is not appropriate when the child is already in pain. 3. Severe pain requires administration of pain medication for pain relief. Comfort measures and diversional activities are not effective against severe pain in children. Comfort measures should be given to every child and can be used after pain medication is given. A child in severe pain is not capable of participating in or enjoying diversional activities. Preparing the child for painful procedures is not appropriate when the child is already in pain. 4. Severe pain requires administration of pain medication for pain relief. Comfort measures and diversional activities are not effective against severe pain in children. Comfort measures should be given to every child and can be used after pain medication is given. A child in severe pain is not capable of participating in or enjoying diversional activities. Preparing the child for painful procedures is not appropriate when the child is already in pain. Page Ref: 598 Cognitive Level: Applying Client Need &Sub: Psychosocial Integrity Standards: QSEN Competencies: Patient-centered care | AACN Essential Competencies: Essential IX: Baccalaureate generalist nursing practice | NLN Competencies: Human flourishing | Nursing/Integrated Concepts: Nursing Process: Implementation/Coordination of care Learning Outcome: LO 23.5 Plan the nursing management and collaborative care of a child with a hematologic disorder. MNL LO: Cardiovascular and Hematological Disorders/Differentiate treatment options and strategies for continuity of care for the child and family. 17) The nurse is providing an educational session for parents with children diagnosed with iron deficiency anemia. Which statements will the nurse include educate about the normal functions of RBCs? Select all that apply. 1. "RBCs transport oxygen from the lungs to the tissue." 2. "RBCs transport carbon dioxide to the lungs." 3. "RBCs protect the body against bacterial invaders." 4. "RBCs form hemostatic plugs to stop bleeding." 5. "RBCs are responsible for psychosocial development." Answer: 1, 2 Explanation: 1. The normal function of RBCs includes transporting oxygen from the lungs to the tissue and transporting carbon dioxide to the lungs. WBCs protect the body against bacterial invaders. Platelets form hemostatic plugs to stop bleeding. RBCs are not directly responsible for psychosocial development. 2. The normal function of RBCs includes transporting oxygen from the lungs to the tissue and transporting carbon dioxide to the lungs. WBCs protect the body against bacterial invaders. Platelets form hemostatic plugs to stop bleeding. RBCs are not directly responsible for psychosocial development. 3. The normal function of RBCs includes transporting oxygen from the lungs to the tissue and transporting carbon dioxide to the lungs. WBCs protect the body against bacterial invaders. Platelets form hemostatic plugs to stop bleeding. RBCs are not directly responsible for psychosocial development. 4. The normal function of RBCs includes transporting oxygen from the lungs to the tissue and transporting carbon dioxide to the lungs. WBCs protect the body against bacterial invaders. Platelets form hemostatic plugs to stop bleeding. RBCs are not directly responsible for psychosocial development. 5. The normal function of RBCs includes transporting oxygen from the lungs to the tissue and transporting carbon dioxide to the lungs. WBCs protect the body against bacterial invaders. Platelets form hemostatic plugs to stop bleeding. RBCs are not directly responsible for psychosocial development. Page Ref: 591 Cognitive Level: Applying Client Need &Sub: Health Promotion and Maintenance Standards: QSEN Competencies: Patient-centered care | AACN Essential Competencies: Essential VII: Clinical prevention and population health | NLN Competencies: Human flourishing | Nursing/Integrated Concepts: Nursing Process: Implementation/Health teaching and health promotion Learning Outcome: LO 23.1 Describe the function of RBCs, WBCs, and platelets. MNL LO: Cardiovascular and Hematological Disorders/Examine etiology, risk factors, pathophysiology, and clinical manifestations as seen in children. 18) The nurse is providing care to a school-age client with neutropenia. Which clinical manifestations does the nurse anticipate when assessing this client? Select all that apply. 1. Fever 2. Fatigue 3. Tachycardia 4. Hypertension 5. Tachypnea Answer: 1, 2, 3, 5 Explanation: 1. A school-age client who is diagnosed with neutropenia, or a decrease in WBCs, will likely exhibit fever, fatigue, tachycardia, and tachypnea (as a result of congestive heart failure). The nurse would not anticipate that the client will exhibit hypertension as a result of the diagnosis. 2. A school-age client who is diagnosed with neutropenia, or a decrease in WBCs, will likely exhibit fever, fatigue, tachycardia, and tachypnea (as a result of congestive heart failure). The nurse would not anticipate that the client will exhibit hypertension as a result of the diagnosis. 3. A school-age client who is diagnosed with neutropenia, or a decrease in WBCs, will likely exhibit fever, fatigue, tachycardia, and tachypnea (as a result of congestive heart failure). The nurse would not anticipate that the client will exhibit hypertension as a result of the diagnosis. 4. A school-age client who is diagnosed with neutropenia, or a decrease in WBCs, will likely exhibit fever, fatigue, tachycardia, and tachypnea (as a result of congestive heart failure). The nurse would not anticipate that the client will exhibit hypertension as a result of the diagnosis. 5. A school-age client who is diagnosed with neutropenia, or a decrease in WBCs, will likely exhibit fever, fatigue, tachycardia, and tachypnea (as a result of congestive heart failure). The nurse would not anticipate that the client will exhibit hypertension as a result of the diagnosis. Page Ref: 591 Cognitive Level: Applying Client Need &Sub: Physiological Integrity: Physiological Adaptation Standards: QSEN Competencies: Patient-centered care | AACN Essential Competencies: Essential IX: Baccalaureate generalist nursing practice | NLN Competencies: Nursing judgment | Nursing/Integrated Concepts: Nursing Process: Assessment/Coordination of care Learning Outcome: LO 23.3 Discuss the pathophysiology and clinical manifestations of the selected disorders of WBCs affecting the pediatric population. MNL LO: Cardiovascular and Hematological Disorders/Examine etiology, risk factors, pathophysiology, and clinical manifestations as seen in children. 19) The adolescent is admitted to the hospital in sickle cell crisis with a pain level of 10/10. The physician orders: Morphine sulfate 5 mg IV q 2 hr prn Medication on hand: morphine sulfate 10 mg/mL Calculate how many ml of morphine sulfate will be given IV. Answer: 0.5 mL Explanation: 0.5 mL Page Ref: 597-598 Cognitive Level: Analyzing Client Need &Sub: Physiological Integrity: Pharmacological and Parenteral Therapies Standards: QSEN Competencies: Patient-centered care | AACN Essential Competencies: Essential IX: Baccalaureate generalist nursing practice | NLN Competencies: Human flourishing | Nursing/Integrated Concepts: Nursing Process: Planning/Coordination of care Learning Outcome: LO 23.5 Plan the nursing management and collaborative care of a child with a hematologic disorder. MNL LO: Cardiovascular and Hematological Disorders/Examine etiology, risk factors, pathophysiology, and clinical manifestations as seen in children. 20) The nurse is caring for the 5-year-old just diagnosed with von Willebrand disease after a tooth extraction with increased bleeding. The family asks the nurse how the signs and symptoms of von Willebrand disease are manifested. What will the nurse tell the family? Select all that apply. 1. Decreased partial thromboplastin time 2. Factor VI deficiency 3. Frequent nosebleeds 4. Bleeding from mucous membranes 5. Frequent bruising Answer: 3, 4, 5 Explanation: 1. Characteristic manifestations are prolonged and excessive mucocutaneous bleeding, in children this is exhibited through gingival bleeding, epistaxis, menorrhagia, bruising, and minor wounds or lacerations. 2. Characteristic manifestations are prolonged and excessive mucocutaneous bleeding, in children this is exhibited through gingival bleeding, epistaxis, menorrhagia, bruising, and minor wounds or lacerations. 3. Characteristic manifestations are prolonged and excessive mucocutaneous bleeding, in children this is exhibited through gingival bleeding, epistaxis, menorrhagia, bruising, and minor wounds or lacerations. 4. Characteristic manifestations are prolonged and excessive mucocutaneous bleeding, in children this is exhibited through gingival bleeding, epistaxis, menorrhagia, bruising, and minor wounds or lacerations. 5. Characteristic manifestations are prolonged and excessive mucocutaneous bleeding, in children this is exhibited through gingival bleeding, epistaxis, menorrhagia, bruising, and minor wounds or lacerations. Page Ref: 605 Cognitive Level: Analyzing Client Need &Sub: Health Promotion and Maintenance Standards: QSEN Competencies: Patient-centered care | AACN Essential Competencies: Essential VII: Clinical prevention and population health | NLN Competencies: Human flourishing | Nursing/Integrated Concepts: Nursing Process: Planning/Health teaching and health promotion Learning Outcome: LO 23.4 Discuss the pathophysiology and clinical manifestations of the major bleeding disorders affecting the pediatric population. MNL LO: Cardiovascular and Hematological Disorders/Examine etiology, risk factors, pathophysiology, and clinical manifestations as seen in children. Principles of Pediatric Nursing: Caring for Children, 7e (Ball et al.) Chapter 24 The Child with Cancer 1) A child diagnosed with a Wilms tumor is prescribed chemotherapy. Which laboratory test will the nurse monitor prior to administering the chemotherapy to determine the child's infection- fighting capability? 1. Hemoglobin 2. RBC count 3. Absolute neutrophil count (ANC) 4. Platelets Answer: 3 Explanation: 1. The absolute neutrophil count uses both the segmented (mature) and bands (immature) neutrophils as a measure of the body's infection-fighting capability. RBC count, hemoglobin, and platelets cannot determine infection-fighting capabilities. 2. The absolute neutrophil count uses both the segmented (mature) and bands (immature) neutrophils as a measure of the body's infection-fighting capability. RBC count, hemoglobin, and platelets cannot determine infection-fighting capabilities. 3. The absolute neutrophil count uses both the segmented (mature) and bands (immature) neutrophils as a measure of the body's infection-fighting capability. RBC count, hemoglobin, and platelets cannot determine infection-fighting capabilities. 4. The absolute neutrophil count uses both the segmented (mature) and bands (immature) neutrophils as a measure of the body's infection-fighting capability. RBC count, hemoglobin, and platelets cannot determine infection-fighting capabilities. Page Ref: 643 Cognitive Level: Analyzing Client Need &Sub: Physiological Integrity: Pharmacological and Parenteral Therapies Standards: QSEN Competencies: Patient-centered care | AACN Essential Competencies: Essential IX: Baccalaureate generalist nursing practice | NLN Competencies: Human flourishing | Nursing/Integrated Concepts: Nursing Process: Assessment/Coordination of care Learning Outcome: LO 24.2 Synthesize information about diagnostic tests and clinical therapy for cancer to plan comprehensive care for children undergoing these procedures. MNL LO: Cellular Alterations/Examine etiology, risk factors, pathophysiology, and clinical manifestations as seen in children. 2) A child diagnosed with cancer is prescribed chemotherapy. The latest lab value indicates the WBC count is very low. Which medication order does the nurse anticipate? 1. Filgrastim (Neupogen) 2. Ondansetron (Zofran) 3. Oprelvekin (Neumega) 4. Epoetin alfa (human recombinant erythropoietin) Answer: 1 Explanation: 1. Filgrastim (Neupogen) increases production of neutrophils by the bone marrow. Ondansetron (Zofran) is an antiemetic, oprelvekin (Neumega) increases platelets, and epoetin alfa (human recombinant erythropoietin) stimulates RBC production. 2. Filgrastim (Neupogen) increases production of neutrophils by the bone marrow. Ondansetron (Zofran) is an antiemetic, oprelvekin (Neumega) increases platelets, and epoetin alfa (human recombinant erythropoietin) stimulates RBC production. 3. Filgrastim (Neupogen) increases production of neutrophils by the bone marrow. Ondansetron (Zofran) is an antiemetic, oprelvekin (Neumega) increases platelets, and epoetin alfa (human recombinant erythropoietin) stimulates RBC production. 4. Filgrastim (Neupogen) increases production of neutrophils by the bone marrow. Ondansetron (Zofran) is an antiemetic, oprelvekin (Neumega) increases platelets, and epoetin alfa (human recombinant erythropoietin) stimulates RBC production. Page Ref: 622 Cognitive Level: Analyzing Client Need &Sub: Physiological Integrity: Pharmacological and Parenteral Therapies Standards: QSEN Competencies: Patient-centered care | AACN Essential Competencies: Essential IX: Baccalaureate generalist nursing practice | NLN Competencies: Human flourishing | Nursing/Integrated Concepts: Nursing Process: Implementation/Coordination of care Learning Outcome: LO 24.2 Synthesize information about diagnostic tests and clinical therapy for cancer to plan comprehensive care for children undergoing these procedures. MNL LO: Cellular Alterations/Examine etiology, risk factors, pathophysiology, and clinical manifestations as seen in children. 3) A preschool child is seen in the clinic, and the nurse anticipates a diagnosis of leukemia. Which reaction does the nurse anticipate this child will exhibit upon diagnosis? 1. Acceptance, especially if able to discuss the disease with children their own age 2. Thoughts that they caused their illness and are being punished 3. Understanding of what cancer is and how it is treated 4. Unawareness of the illness and its severity Answer: 2 Explanation: 1. Preschool-age children may think they caused their illness. Adolescents find contact with others who have gone through their experience helpful. School-age children can understand a diagnosis of cancer. Infants and toddlers are unaware of the severity of the disease. 2. Preschool-age children may think they caused their illness. Adolescents find contact with others who have gone through their experience helpful. School-age children can understand a diagnosis of cancer. Infants and toddlers are unaware of the severity of the disease. 3. Preschool-age children may think they caused their illness. Adolescents find contact with others who have gone through their experience helpful. School-age children can understand a diagnosis of cancer. Infants and toddlers are unaware of the severity of the disease. 4. Preschool-age children may think they caused their illness. Adolescents find contact with others who have gone through their experience helpful. School-age children can understand a diagnosis of cancer. Infants and toddlers are unaware of the severity of the disease. Page Ref: 626 Cognitive Level: Analyzing Client Need &Sub: Psychosocial Integrity Standards: QSEN Competencies: Patient-centered care | AACN Essential Competencies: Essential IX: Baccalaureate generalist nursing practice | NLN Competencies: Human flourishing | Nursing/Integrated Concepts: Nursing Process: Assessment/Coordination of care Learning Outcome: LO 24.5 Plan care for children and adolescents of all ages who have a diagnosis of leukemia. MNL LO: Cellular Alterations/Apply the nursing process in providing care for the child and family. 4) The nurse is monitoring the urine specific gravity and pH on a child receiving chemotherapy. Which urinalysis result is the goal for this child? 1. Spec gravity 1.030; pH 6 2. Spec gravity 1.030; pH 7.5 3. Spec gravity 1.005; pH 6 4. Spec gravity 1.005; pH 7.5 Answer: 4 Explanation: 1. Because the breakdown of malignant cells releases intracellular components into the blood and electrolyte imbalance causes metabolic acidosis, the urine specific gravity should remain at less than 1.010 and the pH at 7 to 7.5. A specific gravity higher than 1.010 can mean fluid intake is not high enough, and a pH of less than 7 means acidosis. 2. Because the breakdown of malignant cells releases intracellular components into the blood and electrolyte imbalance causes metabolic acidosis, the urine specific gravity should remain at less than 1.010 and the pH at 7 to 7.5. A specific gravity higher than 1.010 can mean fluid intake is not high enough, and a pH of less than 7 means acidosis. 3. Because the breakdown of malignant cells releases intracellular components into the blood and electrolyte imbalance causes metabolic acidosis, the urine specific gravity should remain at less than 1.010 and the pH at 7 to 7.5. A specific gravity higher than 1.010 can mean fluid intake is not high enough, and a pH of less than 7 means acidosis. 4. Because the breakdown of malignant cells releases intracellular components into the blood and electrolyte imbalance causes metabolic acidosis, the urine specific gravity should remain at less than 1.010 and the pH at 7 to 7.5. A specific gravity higher than 1.010 can mean fluid intake is not high enough, and a pH of less than 7 means acidosis. Page Ref: 630 Cognitive Level: Analyzing Client Need &Sub: Physiological Integrity: Physiological Adaptation Standards: QSEN Competencies: Patient-centered care | AACN Essential Competencies: Essential IX: Baccalaureate generalist nursing practice | NLN Competencies: Nursing judgement | Nursing/Integrated Concepts: Nursing Process: Assessment/Coordination of care Learning Outcome: LO 24.2 Synthesize information about diagnostic tests and clinical therapy for cancer to plan comprehensive care for children undergoing these procedures. MNL LO: Cellular Alterations/Apply the nursing process in providing care for the child and family. 5) The antiemetic drug ondansetron (Zofran) is administered to a child receiving chemotherapy. When should the nurse administer this medication? 1. Only if the child experiences nausea 2. After the chemotherapy has been administered 3. Before chemotherapy administration as a prophylactic measure 4. Never; this antiemetic is not effective for controlling nausea and vomiting associated with chemotherapy Answer: 3 Explanation: 1. The antiemetic ondansetron (Zofran) should be administered before chemotherapy as a prophylactic measure. Giving it after the child has nausea or at the end of chemotherapy treatment does not help with preventing nausea. It is the drug of choice for controlling nausea caused by chemotherapy agents. 2. The antiemetic ondansetron (Zofran) should be administered before chemotherapy as a prophylactic measure. Giving it after the child has nausea or at the end of chemotherapy treatment does not help with preventing nausea. It is the drug of choice for controlling nausea caused by chemotherapy agents. 3. The antiemetic ondansetron (Zofran) should be administered before chemotherapy as a prophylactic measure. Giving it after the child has nausea or at the end of chemotherapy treatment does not help with preventing nausea. It is the drug of choice for controlling nausea caused by chemotherapy agents. 4. The antiemetic ondansetron (Zofran) should be administered before chemotherapy as a prophylactic measure. Giving it after the child has nausea or at the end of chemotherapy treatment does not help with preventing nausea. It is the drug of choice for controlling nausea caused by chemotherapy agents. Page Ref: 633 Cognitive Level: Applying Client Need &Sub: Physiological Integrity: Pharmacological and Parenteral Therapies Standards: QSEN Competencies: Patient-centered care | AACN Essential Competencies: Essential IX: Baccalaureate generalist nursing practice | NLN Competencies: Human flourishing | Nursing/Integrated Concepts: Nursing Process: Implementation/Coordination of care Learning Outcome: LO 24.2 Synthesize information about diagnostic tests and clinical therapy for cancer to plan comprehensive care for children undergoing these procedures. MNL LO: Cellular Alterations/Apply the nursing process in providing care for the child and family. 6) A child is diagnosed with thrombocytopenia secondary to chemotherapy treatments. Which action by the nurse is the most appropriate? 1. Refrain from administering any intramuscular injections (IM). 2. Perform oral hygiene. 3. Monitor intake and output. 4. Use palpation as a component of assessment. Answer: 1 Explanation: 1. When the child is thrombocytopenic (decreased platelets) from chemotherapy, the nurse should not administer IM injections because of the risk of bleeding. Oral hygiene care should be done with a soft toothbrush and intake and output monitored for any abnormalities. Gentle palpation should still be included in physical assessments. 2. When the child is thrombocytopenic (decreased platelets) from chemotherapy, the nurse should not administer IM injections because of the risk of bleeding. Oral hygiene care should be done with a soft toothbrush and intake and output monitored for any abnormalities. Gentle palpation should still be included in physical assessments. 3. When the child is thrombocytopenic (decreased platelets) from chemotherapy, the nurse should not administer IM injections because of the risk of bleeding. Oral hygiene care should be done with a soft toothbrush and intake and output monitored for any abnormalities. Gentle palpation should still be included in physical assessments. 4. When the child is thrombocytopenic (decreased platelets) from chemotherapy, the nurse should not administer IM injections because of the risk of bleeding. Oral hygiene care should be done with a soft toothbrush and intake and output monitored for any abnormalities. Gentle palpation should still be included in physical assessments. Page Ref: 633 Cognitive Level: Applying Client Need &Sub: Physiological Integrity: Basic Care and Comfort Standards: QSEN Competencies: Patient-centered care | AACN Essential Competencies: Essential IX: Baccalaureate generalist nursing practice | NLN Competencies: Human flourishing | Nursing/Integrated Concepts: Nursing Process: Assessment/Coordination of care Learning Outcome: LO 24.2 Synthesize information about diagnostic tests and clinical therapy for cancer to plan comprehensive care for children undergoing these procedures. MNL LO: Cellular Alterations/Apply the nursing process in providing care for the child and family. 7) A child undergoing chemotherapeutic treatment for cancer is being admitted to the hospital for fever of 102 degrees F and possible sepsis. Cultures, antibiotics, and acetaminophen (Tylenol) along with bed rest have been ordered for this child. Place the following steps in order from first to last. Response 1 Administer the antibiotics. Response 2 Administer the acetaminophen (Tylenol). Response 3 Obtain the cultures. Response 4 Ensure the child has bed rest. Answer: 2, 3, 1, 4 1. Response 2 Administer the acetaminophen (Tylenol). 2. Response 3 Obtain the cultures. 3. Response 1 Administer the antibiotics. 4. Response 4 Ensure the child has bed rest. Explanation: Give acetaminophen (Tylenol) first to decrease discomfort and reduce fever. Obtain the cultures next because management of infections is critical, and since a child on chemotherapy has lowered immune status, unusual agents can be identified. Cultures can help identify the causative agents before treatment is started. Give the antibiotics next, as an infection can seriously impact the child who is receiving chemotherapy. Finally, provide comfort followed by bed rest to allow the child to rest. Page Ref: 634 Cognitive Level: Applying Client Need &Sub: Safe and Effective Care Environment: Management of Care Standards: QSEN Competencies: Patient-centered care | AACN Essential Competencies: Essential IX: Baccalaureate generalist nursing practice | NLN Competencies: Nursing judgement | Nursing/Integrated Concepts: Nursing Process: Implementation/Coordination of care Learning Outcome: LO 24.2 Synthesize information about diagnostic tests and clinical therapy for cancer to plan comprehensive care for children undergoing these procedures. MNL LO: Cellular Alterations/Apply the nursing process in providing care for the child and family. 8) A 24-hour urine collection for vanillylmandelic acid (VMA) has been ordered on a child suspected of having neuroblastoma. When is the most appropriate time for the nurse to begin the collection? 1. At 0700 2. After the next time the child voids 3. At bedtime 4. When the order is noted Answer: 2 Explanation: 1. A 24-hour urine collection is started after the child voids. That specimen is not saved, but all subsequent specimens in that 24-hour period should be collected. It would not be an accurate collection of 24 hours of urine if the collection began at 0700, bedtime, or when the order is noted. 2. A 24-hour urine collection is started after the child voids. That specimen is not saved, but all subsequent specimens in that 24-hour period should be collected. It would not be an accurate collection of 24 hours of urine if the collection began at 0700, bedtime, or when the order is noted. 3. A 24-hour urine collection is started after the child voids. That specimen is not saved, but all subsequent specimens in that 24-hour period should be collected. It would not be an accurate collection of 24 hours of urine if the collection began at 0700, bedtime, or when the order is noted. 4. A 24-hour urine collection is started after the child voids. That specimen is not saved, but all subsequent specimens in that 24-hour period should be collected. It would not be an accurate collection of 24 hours of urine if the collection began at 0700, bedtime, or when the order is noted. Page Ref: 641 Cognitive Level: Applying Client Need &Sub: Physiological Integrity: Basic Care and Comfort Standards: QSEN Competencies: Patient-centered care | AACN Essential Competencies: Essential IX: Baccalaureate generalist nursing practice | NLN Competencies: Nursing judgement | Nursing/Integrated Concepts: Nursing Process: Planning/Coordination of care Learning Outcome: LO 24.4 Recognize the most common solid tumors in children, describe their treatment, and plan comprehensive nursing care. MNL LO: Cellular Alterations/Differentiate treatment options and strategies for continuity of care for the child and family. 9) A child is diagnosed with a Wilms tumor. Which nursing action is most appropriate prior to surgery? 1. Careful bathing and handling 2. Monitoring of behavioral status 3. Maintenance of strict isolation 4. Administration of packed RBCs Answer: 1 Explanation: 1. The tumor should never be palpated; careful bathing and handling are an important nursing consideration. Palpating the tumor can cause a piece of the tumor to dislodge. The child's behavior will not be affected with a Wilms tumor. The tumor does not cause excessive lowering of WBCs or RBCs, so strict isolation or administration of packed RBCs is not usually a nursing intervention. 2. The tumor should never be palpated; careful bathing and handling are an important nursing consideration. Palpating the tumor can cause a piece of the tumor to dislodge. The child's behavior will not be affected with a Wilms tumor. The tumor does not cause excessive lowering of WBCs or RBCs, so strict isolation or administration of packed RBCs is not usually a nursing intervention. 3. The tumor should never be palpated; careful bathing and handling are an important nursing consideration. Palpating the tumor can cause a piece of the tumor to dislodge. The child's behavior will not be affected with a Wilms tumor. The tumor does not cause excessive lowering of WBCs or RBCs, so strict isolation or administration of packed RBCs is not usually a nursing intervention. 4. The tumor should never be palpated; careful bathing and handling are an important nursing consideration. Palpating the tumor can cause a piece of the tumor to dislodge. The child's behavior will not be affected with a Wilms tumor. The tumor does not cause excessive lowering of WBCs or RBCs, so strict isolation or administration of packed RBCs is not usually a nursing intervention. Page Ref: 643 Cognitive Level: Applying Client Need &Sub: Safe and Effective Care Environment: Safety and Infection Control Standards: QSEN Competencies: Patient-centered care | AACN Essential Competencies: Essential IX: Baccalaureate generalist nursing practice | NLN Competencies: Human flourishing | Nursing/Integrated Concepts: Nursing Process: Implementation/Coordination of care Learning Outcome: LO 24.4 Recognize the most common solid tumors in children, describe their treatment, and plan comprehensive nursing care. MNL LO: Cellular Alterations/Examine etiology, risk factors, pathophysiology, and clinical manifestations as seen in children. 10) An adolescent is receiving methotrexate chemotherapy after undergoing limb-salvage surgery for osteogenic sarcoma. Which statement by the adolescent indicates understanding of the purpose of leucovorin therapy after the methotrexate? 1. "I'm glad I only need one dose of the leucovorin." 2. "I don't have any pain so I won't need to take the leucovorin this time." 3. "I know I will be taking the leucovorin every 6 hours for about the next 3 days." 4. "I don't have any nausea so I won't need the leucovorin." Answer: 3 Explanation: 1. Leucovorin (citrovorum factor) is a form of folic acid that helps to protect normal cells from the destructive action of methotrexate. It is started within 24 hours of methotrexate administration and is given along with hydration therapy. Usual administration is every 6 hours for 72 hours or until serum methotrexate is at the desired level. 2. Leucovorin (citrovorum factor) is a form of folic acid that helps to protect normal cells from the destructive action of methotrexate. It is started within 24 hours of methotrexate administration and is given along with hydration therapy. Usual administration is every 6 hours for 72 hours or until serum methotrexate is at the desired level. 3. Leucovorin (citrovorum factor) is a form of folic acid that helps to protect normal cells from the destructive action of methotrexate. It is started within 24 hours of methotrexate administration and is given along with hydration therapy. Usual administration is every 6 hours for 72 hours or until serum methotrexate is at the desired level. 4. Leucovorin (citrovorum factor) is a form of folic acid that helps to protect normal cells from the destructive action of methotrexate. It is started within 24 hours of methotrexate administration and is given along with hydration therapy. Usual administration is every 6 hours for 72 hours or until serum methotrexate is at the desired level. Page Ref: 644 Cognitive Level: Analyzing Client Need &Sub: Physiological Integrity: Pharmacological and Parenteral Therapies Standards: QSEN Competencies: Patient-centered care | AACN Essential Competencies: Essential VII: Clinical prevention and population health | NLN Competencies: Human flourishing | Nursing/Integrated Concepts: Nursing Process: Evaluation/Health teaching and health promotion Learning Outcome: LO 24.4 Recognize the most common solid tumors in children, describe their treatment, and plan comprehensive nursing care. MNL LO: Cellular Alterations/Examine etiology, risk factors, pathophysiology, and clinical manifestations as seen in children. 11) A child who is diagnosed with leukemia has a sibling who is expressing feelings of anger and guilt. How would the nurse characterize this reaction by the sibling? 1. Abnormal; the sibling should be referred to a psychologist. 2. Normal; the illness doesn't affect the sibling. 3. Unexpected; the cancer is easily treated. 4. Normal; the sibling is affected too, and anger and guilt are expected feelings. Answer: 4 Explanation: 1. A diagnosis of cancer affects the whole family, and initial feelings experienced by the sibling may be anger and guilt. Seldom will the sibling be unaffected; however, the response is not abnormal. 2. A diagnosis of cancer affects the whole family, and initial feelings experienced by the sibling may be anger and guilt. Seldom will the sibling be unaffected; however, the response is not abnormal. 3. A diagnosis of cancer affects the whole family, and initial feelings experienced by the sibling may be anger and guilt. Seldom will the sibling be unaffected; however, the response is not abnormal. 4. A diagnosis of cancer affects the whole family, and initial feelings experienced by the sibling may be anger and guilt. Seldom will the sibling be unaffected; however, the response is not abnormal. Page Ref: 624 Cognitive Level: Analyzing Client Need &Sub: Psychosocial Integrity Standards: QSEN Competencies: Patient-centered care | AACN Essential Competencies: Essential IX: Baccalaureate generalist nursing practice | NLN Competencies: Human flourishing | Nursing/Integrated Concepts: Nursing Process: Assessment/Coordination of care Learning Outcome: LO 24.5 Plan care for children and adolescents of all ages who have a diagnosis of leukemia. MNL LO: Cellular Alterations/Apply the nursing process in providing care for the child and family. 12) The child is admitted to the hospital after being diagnosed with retinoblastoma. Which assessment finding does the nurse anticipate for this child? 1. A red reflex 2. Yellow sclera 3. A white pupil 4. Blue-tinged sclera Answer: 3 Explanation: 1. The first sign of retinoblastoma is a white pupil. The red reflex is absent. Yellow sclera is a sign of jaundice, not retinoblastoma. Blue-tinged sclera is a sign of osteogenesis imperfecta, not retinoblastoma. 2. The first sign of retinoblastoma is a white pupil. The red reflex is absent. Yellow sclera is a sign of jaundice, not retinoblastoma. Blue-tinged sclera is a sign of osteogenesis imperfecta, not retinoblastoma. 3. The first sign of retinoblastoma is a white pupil. The red reflex is absent. Yellow sclera is a sign of jaundice, not retinoblastoma. Blue-tinged sclera is a sign of osteogenesis imperfecta, not retinoblastoma. 4. The first sign of retinoblastoma is a white pupil. The red reflex is absent. Yellow sclera is a sign of jaundice, not retinoblastoma. Blue-tinged sclera is a sign of osteogenesis imperfecta, not retinoblastoma. Page Ref: 651 Cognitive Level: Analyzing Client Need &Sub: Physiological Integrity: Physiological Adaptation Standards: QSEN Competencies: Patient-centered care | AACN Essential Competencies: Essential IX: Baccalaureate generalist nursing practice | NLN Competencies: Human flourishing | Nursing/Integrated Concepts: Nursing Process: Assessment/Coordination of care Learning Outcome: LO 24.4 Recognize the most common solid tumors in children, describe their treatment, and plan comprehensive nursing care. MNL LO: Cellular Alterations/Examine etiology, risk factors, pathophysiology, and clinical manifestations as seen in children. 13) A preschool-age child is brought to the clinic by the mother, who says the child has been lethargic and anorexic lately and complains of bone pain. On exam, the nurse notes petechiae, joint pain, and an enlarged liver. Which diagnosis does the nurse anticipate for this child? 1. Hodgkin disease 2. Leukemia 3. Rhabdomyosarcoma 4. Ewing sarcoma Answer: 2 Explanation: 1. Hodgkin disease, rhabdomyosarcoma, and Ewing sarcoma are all childhood cancers, but they do not have the clinical manifestations listed. Leukemia is one of the most common childhood cancers, and has those clinical symptoms. 2. Hodgkin disease, rhabdomyosarcoma, and Ewing sarcoma are all childhood cancers, but they do not have the clinical manifestations listed. Leukemia is one of the most common childhood cancers, and has those clinical symptoms. 3. Hodgkin disease, rhabdomyosarcoma, and Ewing sarcoma are all childhood cancers, but they do not have the clinical manifestations listed. Leukemia is one of the most common childhood cancers, and has those clinical symptoms. 4. Hodgkin disease, rhabdomyosarcoma, and Ewing sarcoma are all childhood cancers, but they do not have the clinical manifestations listed. Leukemia is one of the most common childhood cancers, and has those clinical symptoms. Page Ref: 646-647 Cognitive Level: Analyzing Client Need &Sub: Physiological Integrity: Physiological Adaptation Standards: QSEN Competencies: Patient-centered care | AACN Essential Competencies: Essential IX: Baccalaureate generalist nursing practice | NLN Competencies: Human flourishing | Nursing/Integrated Concepts: Nursing Process: Assessment/Coordination of care Learning Outcome: LO 24.5 Plan care for children and adolescents of all ages who have a diagnosis of leukemia. MNL LO: Cellular Alterations/Apply the nursing process in providing care for the child and family. 14) A child with a brain tumor is admitted to the pediatric intensive care unit (PICU) after brain surgery to remove the tumor. Which postoperative order would the nurse question? 1. Antibiotics 2. Sodium levels every 24 hours 3. Anticonvulsants 4. Hourly intake and output Answer: 2 Explanation: 1. Antibiotics, anticonvulsants, and hourly intake and output are appropriate orders. Serum sodium levels should be done every 4 to 6 hours, not every 24 hours. 2. Antibiotics, anticonvulsants, and hourly intake and output are appropriate orders. Serum sodium levels should be done every 4 to 6 hours, not every 24 hours. 3. Antibiotics, anticonvulsants, and hourly intake and output are appropriate orders. Serum sodium levels should be done every 4 to 6 hours, not every 24 hours. 4. Antibiotics, anticonvulsants, and hourly intake and output are appropriate orders. Serum sodium levels should be done every 4 to 6 hours, not every 24 hours. Page Ref: 639-640 Cognitive Level: Applying Client Need &Sub: Physiological Integrity: Basic Care and Comfort Standards: QSEN Competencies: Patient-centered care | AACN Essential Competencies: Essential IX: Baccalaureate generalist nursing practice | NLN Competencies: Human flourishing | Nursing/Integrated Concepts: Nursing Process: Assessment/Coordination of care Learning Outcome: LO 24.4 Recognize the most common solid tumors in children, describe their treatment, and plan comprehensive nursing care. MNL LO: Cellular Alterations/Examine etiology, risk factors, pathophysiology, and clinical manifestations as seen in children. 15) A child is diagnosed with rhabdomyosarcoma. Which nursing intervention is most appropriate for this child? 1. Position the child with the head elevated 2. Monitor for hematuria 3. Demonstrate the use of a conformer 4. Administer oxygen Answer: 1 Explanation: 1. The most common area of the body affected by rhabdomyosarcoma is the bladder. The nursing intervention that is most appropriate is to monitor the child's urine for hematuria. Positioning the child with the head elevated and administering oxygen is appropriate for a child diagnosed with lymphoma. Demonstrating the use of a conformer is appropriate for a child diagnosed with retinoblastoma. 2. The most common area of the body affected by rhabdomyosarcoma is the bladder. The nursing intervention that is most appropriate is to monitor the child's urine for hematuria. Positioning the child with the head elevated and administering oxygen is appropriate for a child diagnosed with lymphoma. Demonstrating the use of a conformer is appropriate for a child diagnosed with retinoblastoma. 3. The most common area of the body affected by rhabdomyosarcoma is the bladder. The nursing intervention that is most appropriate is to monitor the child's urine for hematuria. Positioning the child with the head elevated and administering oxygen is appropriate for a child diagnosed with lymphoma. Demonstrating the use of a conformer is appropriate for a child diagnosed with retinoblastoma. 4. The most common area of the body affected by rhabdomyosarcoma is the bladder. The nursing intervention that is most appropriate is to monitor the child's urine for hematuria. Positioning the child with the head elevated and administering oxygen is appropriate for a child diagnosed with lymphoma. Demonstrating the use of a conformer is appropriate for a child diagnosed with retinoblastoma. Page Ref: 650-651 Cognitive Level: Applying Client Need &Sub: Physiological Integrity: Physiological Adaptation Standards: QSEN Competencies: Patient-centered care | AACN Essential Competencies: Essential IX: Baccalaureate generalist nursing practice | NLN Competencies: Nursing judgement | Nursing/Integrated Concepts: Nursing Process: Assessment/Coordination of care Learning Outcome: LO 24.6 Prioritize elements of comprehensive care planning for children with soft-tissue tumors. MNL LO: Cellular Alterations/Apply the nursing process in providing care for the child and family. 16) The pediatric nurse educator is conducting an in-service for novice nurses who will begin working on the pediatric oncology unit. The educator wants to include the common clinical manifestations of cancer. Which manifestation will the educator include in the presentation? Select all that apply. 1. Cachexia 2. Anemia 3. Gene abnormalities 4. Palpable mass 5. Chromosomal abnormalities Answer: 1, 2, 4 Explanation: 1. Common clinical manifestations of childhood cancer include cachexia, anemia, and a palpable mass. Gene abnormalities and chromosomal abnormalities are common etiologies to childhood cancer, not clinical manifestations. 2. Common clinical manifestations of childhood cancer include cachexia, anemia, and a palpable mass. Gene abnormalities and chromosomal abnormalities are common etiologies to childhood cancer, not clinical manifestations. 3. Common clinical manifestations of childhood cancer include cachexia, anemia, and a palpable mass. Gene abnormalities and chromosomal abnormalities are common etiologies to childhood cancer, not clinical manifestations. 4. Common clinical manifestations of childhood cancer include cachexia, anemia, and a palpable mass. Gene abnormalities and chromosomal abnormalities are common etiologies to childhood cancer, not clinical manifestations. 5. Common clinical manifestations of childhood cancer include cachexia, anemia, and a palpable mass. Gene abnormalities and chromosomal abnormalities are common etiologies to childhood cancer, not clinical manifestations. Page Ref: 616 Cognitive Level: Applying Client Need &Sub: Physiological Integrity: Physiological Adaptation Standards: QSEN Competencies: Patient-centered care | AACN Essential Competencies: Essential VII: Clinical prevention and population health | NLN Competencies: Human flourishing | Nursing/Integrated Concepts: Nursing Process: Assessment/Health teaching and health promotion Learning Outcome: LO 24.1 Describe the incidence, known etiologies, and common clinical manifestations of cancer. MNL LO: Cellular Alterations/Examine etiology, risk factors, pathophysiology, and clinical manifestations as seen in children. 17) The pediatric nurse is providing care to a school-age child receiving chemotherapy to treat cancer. Which interventions are appropriate to include in the plan of care in order to monitor for oncologic emergencies? Select all that apply. 1. Monitor complete blood count (CBC). 2. Document intake and output. 3. Observe for behavioral changes. 4. Refer for psychosocial support. 5. Implement neutropenic precautions. Answer: 1, 2, 3 Explanation: 1. Oncologic emergencies can be organized into three groups: metabolic, hematologic, and those involving space-occupying lesions. Appropriate interventions for the nurse to include in the plan of care to monitor for these emergencies include monitoring the CBC to prevent sepsis and hemorrhage; monitoring intake and output by encouraging hydration to prevent hypercalcemia and observing for signs of water intoxication; and observing for behavioral changes as space-occupying lesions may cause seizures or increased intracranial pressure. Referring for psychosocial support and implementing neutropenia precautions may be appropriate, but these interventions do not address oncologic emergencies. 2. Oncologic emergencies can be organized into three groups: metabolic, hematologic, and those involving space-occupying lesions. Appropriate interventions for the nurse to include in the plan of care to monitor for these emergencies include monitoring the CBC to prevent sepsis and hemorrhage; monitoring intake and output by encouraging hydration to prevent hypercalcemia and observing for signs of water intoxication; and observing for behavioral changes as space- occupying lesions may cause seizures or increased intracranial pressure. Referring for psychosocial support and implementing neutropenia precautions may be appropriate, but these interventions do not address oncologic emergencies. 3. Oncologic emergencies can be organized into three groups: metabolic, hematologic, and those involving space-occupying lesions. Appropriate interventions for the nurse to include in the plan of care to monitor for these emergencies include monitoring the CBC to prevent sepsis and hemorrhage; monitoring intake and output by encouraging hydration to prevent hypercalcemia and observing for signs of water intoxication; and observing for behavioral changes as space- occupying lesions may cause seizures or increased intracranial pressure. Referring for psychosocial support and implementing neutropenia precautions may be appropriate, but these interventions do not address oncologic emergencies. 4. Oncologic emergencies can be organized into three groups: metabolic, hematologic, and those involving space-occupying lesions. Appropriate interventions for the nurse to include in the plan of care to monitor for these emergencies include monitoring the CBC to prevent sepsis and hemorrhage; monitoring intake and output by encouraging hydration to prevent hypercalcemia and observing for signs of water intoxication; and observing for behavioral changes as space- occupying lesions may cause seizures or increased intracranial pressure. Referring for psychosocial support and implementing neutropenia precautions may be appropriate, but these interventions do not address oncologic emergencies. 5. Oncologic emergencies can be organized into three groups: metabolic, hematologic, and those involving space-occupying lesions. Appropriate interventions for the nurse to include in the plan of care to monitor for these emergencies include monitoring the CBC to prevent sepsis and hemorrhage; monitoring intake and output by encouraging hydration to prevent hypercalcemia and observing for signs of water intoxication; and observing for behavioral changes as space- occupying lesions may cause seizures or increased intracranial pressure. Referring for psychosocial support and implementing neutropenia precautions may be appropriate, but these interventions do not address oncologic emergencies. Page Ref: 623 Cognitive Level: Applying Client Need &Sub: Physiological Integrity: Reduction of Risk Potential Standards: QSEN Competencies: Patient-centered care | AACN Essential Competencies: Essential IX: Baccalaureate generalist nursing practice | NLN Competencies: Nursing judgement | Nursing/Integrated Concepts: Nursing Process: Implementation/Coordination of care Learning Outcome: LO 24.3 Integrate information about oncologic emergencies into plans for monitoring all children with cancer. MNL LO: Cellular Alterations/Differentiate treatment options and strategies for continuity of care for the child and family. 18) A seasoned nurse is precepting a novice nurse on a pediatric oncology unit. The seasoned nurse would like to review the ongoing physiologic and psychosocial care of the children who survive cancer. Which topics will the seasoned nurse include in the discussion with the novice nurse? Select all that apply. 1. Developing other cancers 2. Recommending regular office visits 3. Encouraging school-age clients to manage their own care 4. Needing weekly laboratory tests 5. Providing educational and psychosocial support Answer: 1, 2, 5 Explanation: 1. Appropriate topics include discussing the increased risk for these children to develop other cancers; recommending regular office visits for monitoring purposes; and providing educational and psychosocial support. It would be appropriate to encourage the adolescent and young adult clients to manage their own care, not a school-age child. While these clients need regular laboratory examinations, weekly laboratory tests are not appropriate. 2. Appropriate topics include discussing the increased risk for these children to develop other cancers; recommending regular office visits for monitoring purposes; and providing educational and psychosocial support. It would be appropriate to encourage the adolescent and young adult clients to manage their own care, not a school-age child. While these clients need regular laboratory examinations, weekly laboratory tests are not appropriate. 3. Appropriate topics include discussing the increased risk for these children to develop other cancers; recommending regular office visits for monitoring purposes; and providing educational and psychosocial support. It would be appropriate to encourage the adolescent and young adult clients to manage their own care, not a school-age child. While these clients need regular laboratory examinations, weekly laboratory tests are not appropriate. 4. Appropriate topics include discussing the increased risk for these children to develop other cancers; recommending regular office visits for monitoring purposes; and providing educational and psychosocial support. It would be appropriate to encourage the adolescent and young adult clients to manage their own care, not a school-age child. While these clients need regular laboratory examinations, weekly laboratory tests are not appropriate. 5. Appropriate topics include discussing the increased risk for these children to develop other cancers; recommending regular office visits for monitoring purposes; and providing educational and psychosocial support. It would be appropriate to encourage the adolescent and young adult clients to manage their own care, not a school-age child. While these clients need regular laboratory examinations, weekly laboratory tests are not appropriate. Page Ref: 646 Cognitive Level: Applying Client Need &Sub: Health Promotion and Maintenance Standards: QSEN Competencies: Quality Improvement | AACN Essential Competencies: Essential II: Basic organizational and systems leadership for quality care and patient safety | NLN Competencies: Professional identity | Nursing/Integrated Concepts: Nursing Process: Implementation/Leadership Learning Outcome: LO 24.7 Analyze the impact of cancer survival on children and use this information to plan for ongoing physiologic and psychosocial care in the children's futures. MNL LO: Cellular Alterations/Differentiate treatment options and strategies for continuity of care for the child and family. 19) The child has just been diagnosed with osteosarcoma, and the nurse is teaching the family regarding this type of cancer. The nurse knows that instruction has been successful when the family states that osteosarcoma is common in which age group? 1. Infants 2. Toddlers 3. Preschool-age children 4. School-age children 5. Adolescents Answer: 5 Explanation: 1. Osteosarcoma's peak incidence is during the rapid growth years, at age 13 for girls and 14 for boys. 2. Osteosarcoma's peak incidence is during the rapid growth years, at age 13 for girls and 14 for boys. 3. Osteosarcoma's peak incidence is during the rapid growth years, at age 13 for girls and 14 for boys. 4. Osteosarcoma's peak incidence is during the rapid growth years, at age 13 for girls and 14 for boys. 5. Osteosarcoma's peak incidence is during the rapid growth years, at age 13 for girls and 14 for boys. Page Ref: 644 Cognitive Level: Analyzing Client Need &Sub: Health Promotion and Maintenance Standards: QSEN Competencies: Patient-centered care | AACN Essential Competencies: Essential VII: Clinical prevention and population health | NLN Competencies: Human flourishing | Nursing/Integrated Concepts: Nursing Process: Intervention/Health teaching and health promotion Learning Outcome: LO 24.1 Describe the incidence, known etiologies, and common clinical manifestations of cancer. MNL LO: Integumentary and Musculoskeletal Disorders/Examine etiology, risk factors, pathophysiology, and clinical manifestations as seen in children. 20) The nurse is teaching a 10-year-old and family about the diagnosis of Ewing sarcoma. The nurse knows that instruction has been successful when the child and family indicate which is a common site? 1. Bone marrow 2. Head 3. Shaft 4. Growth plate 5. Bursae Answer: 3 Explanation: 1. Ewing sarcoma is a malignant, tumor involving the diaphyseal (shaft) portion of the long bones. Common sites include: femur, pelvis, tibia, fibula, ribs, humerus, scapula, and clavicle. 2. Ewing sarcoma is a malignant, tumor involving the diaphyseal (shaft) portion of the long bones. Common sites include: femur, pelvis, tibia, fibula, ribs, humerus, scapula, and clavicle. 3. Ewing sarcoma is a malignant, tumor involving the diaphyseal (shaft) portion of the long bones. Common sites include: femur, pelvis, tibia, fibula, ribs, humerus, scapula, and clavicle. 4. Ewing sarcoma is a malignant, tumor involving the diaphyseal (shaft) portion of the long bones. Common sites include: femur, pelvis, tibia, fibula, ribs, humerus, scapula, and clavicle. 5. Ewing sarcoma is a malignant, tumor involving the diaphyseal (shaft) portion of the long bones. Common sites include: femur, pelvis, tibia, fibula, ribs, humerus, scapula, and clavicle. Page Ref: 644-645 Cognitive Level: Analyzing Client Need &Sub: Health Promotion and Maintenance Standards: QSEN Competencies: Patient-centered care | AACN Essential Competencies: Essential VII: Clinical prevention and population health | NLN Competencies: Human flourishing | Nursing/Integrated Concepts: Nursing Process: Intervention/Health teaching and health promotion Learning Outcome: LO 24.1 Describe the incidence, known etiologies, and common clinical manifestations of cancer. MNL LO: Integumentary and Musculoskeletal Disorders/Examine etiology, risk factors, pathophysiology, and clinical manifestations as seen in children. Principles of Pediatric Nursing: Caring for Children, 7e (Ball et al.) Chapter 25 Alterations in Gastrointestinal Function 1) The nurse is planning postoperative care for an infant after a cleft-lip repair. Which nursing intervention is most appropriate for this infant? 1. Prone positioning 2. Suctioning with a Yankauer device 3. Supine or side-lying positioning 4. Avoidance of soft elbow restraints Answer: 3 Explanation: 1. Integrity of the suture line is essential for postoperative care of cleft-lip repair. The infant should be placed in a supine or side-lying position to avoid rubbing the suture line on the bedding. The prone position should be avoided. A Yankauer suction device is made of hard plastic and, if used, could cause trauma to the suture line. Suctioning should be done with a small, soft suction catheter. Soft elbow restraints may be used to prevent the infant from touching the incisional area. 2. Integrity of the suture line is essential for postoperative care of cleft-lip repair. The infant should be placed in a supine or side-lying position to avoid rubbing the suture line on the bedding. The prone position should be avoided. A Yankauer suction device is made of hard plastic and, if used, could cause trauma to the suture line. Suctioning should be done with a small, soft suction catheter. Soft elbow restraints may be used to prevent the infant from touching the incisional area. 3. Integrity of the suture line is essential for postoperative care of cleft-lip repair. The infant should be placed in a supine or side-lying position to avoid rubbing the suture line on the bedding. The prone position should be avoided. A Yankauer suction device is made of hard plastic and, if used, could cause trauma to the suture line. Suctioning should be done with a small, soft suction catheter. Soft elbow restraints may be used to prevent the infant from touching the incisional area. 4. Integrity of the suture line is essential for postoperative care of cleft-lip repair. The infant should be placed in a supine or side-lying position to avoid rubbing the suture line on the bedding. The prone position should be avoided. A Yankauer suction device is made of hard plastic and, if used, could cause trauma to the suture line. Suctioning should be done with a small, soft suction catheter. Soft elbow restraints may be used to prevent the infant from touching the incisional area. Page Ref: 664 Cognitive Level: Applying Client Need &Sub: Safe and Effective Care Environment: Safety and Infection Control Standards: QSEN Competencies: Patient-centered care | AACN Essential Competencies: Essential IX: Baccalaureate generalist nursing practice | NLN Competencies: Human flourishing | Nursing/Integrated Concepts: Nursing Process: Planning/Coordination of care Learning Outcome: LO 25.5 Analyze developmentally appropriate approaches for nursing management of gastrointestinal disorders in the pediatric population. MNL LO: Gastrointestinal Disorders/Differentiate treatment options and strategies for continuity of care for the child and family. 2) An infant is born with an esophageal atresia and tracheoesophageal fistula. Which preoperative nursing diagnosis is the priority for this infant? 1. Risk for Aspiration Related to Regurgitation 2. Acute Pain Related to Esophageal Defect 3. Ineffective Infant Feeding Pattern Related to Uncoordinated Suck and Swallow 4. Ineffective Tissue Perfusion: Gastrointestinal, Related to Decreased Circulation Answer: 1 Explanation: 1. With the most common type of esophageal atresia and tracheoesophageal fistula, the upper segment of the esophagus ends in a blind pouch and a fistula connects the lower segment to the trachea. Preoperatively, there is a risk of aspiration of gastric secretions from the stomach into the trachea because of the fistula that connects the lower segment of the esophagus to the trachea. Pain is not usually experienced preoperatively with this condition. The infant is always kept NPO (nothing by mouth) preoperatively, so ineffective feeding pattern would not apply. Tissue perfusion is not a problem with this condition. 2. With the most common type of esophageal atresia and tracheoesophageal fistula, the upper segment of the esophagus ends in a blind pouch and a fistula connects the lower segment to the trachea. Preoperatively, there is a risk of aspiration of gastric secretions from the stomach into the trachea because of the fistula that connects the lower segment of the esophagus to the trachea. Pain is not usually experienced preoperatively with this condition. The infant is always kept NPO (nothing by mouth) preoperatively, so ineffective feeding pattern would not apply. Tissue perfusion is not a problem with this condition. 3. With the most common type of esophageal atresia and tracheoesophageal fistula, the upper segment of the esophagus ends in a blind pouch and a fistula connects the lower segment to the trachea. Preoperatively, there is a risk of aspiration of gastric secretions from the stomach into the trachea because of the fistula that connects the lower segment of the esophagus to the trachea. Pain is not usually experienced preoperatively with this condition. The infant is always kept NPO (nothing by mouth) preoperatively, so ineffective feeding pattern would not apply. Tissue perfusion is not a problem with this condition. 4. With the most common type of esophageal atresia and tracheoesophageal fistula, the upper segment of the esophagus ends in a blind pouch and a fistula connects the lower segment to the trachea. Preoperatively, there is a risk of aspiration of gastric secretions from the stomach into the trachea because of the fistula that connects the lower segment of the esophagus to the trachea. Pain is not usually experienced preoperatively with this condition. The infant is always kept NPO (nothing by mouth) preoperatively, so ineffective feeding pattern would not apply. Tissue perfusion is not a problem with this condition. Page Ref: 666 Cognitive Level: Analyzing Client Need &Sub: Physiological Integrity: Basic Care and Comfort Standards: QSEN Competencies: Patient-centered care | AACN Essential Competencies: Essential IX: Baccalaureate generalist nursing practice | NLN Competencies: Human flourishing | Nursing/Integrated Concepts: Nursing Process: Diagnosis/Coordination of care Learning Outcome: LO 25.2 Discuss the pathophysiologic processes associated with specific gastrointestinal disorders in the pediatric population. MNL LO: Gastrointestinal Disorders/Examine etiology, risk factors, pathophysiology, and clinical manifestations as seen in children. 3) The nurse is evaluating an infant's tolerance of feedings after a pyloromyotomy. Which finding indicates that the infant is not tolerating the feeding? 1. Need for frequent burping 2. Irritability during feeding 3. The passing of gas 4. Emesis after two feedings Answer: 4 Explanation: 1. An infant is not tolerating feedings after a pyloromyotomy if emesis is present. Frequent burping, irritability, and the passing of gas would be expected findings following a pyloromyotomy and would indicate tolerance of the feeding. 2. An infant is not tolerating feedings after a pyloromyotomy if emesis is present. Frequent burping, irritability, and the passing of gas would be expected findings following a pyloromyotomy and would indicate tolerance of the feeding. 3. An infant is not tolerating feedings after a pyloromyotomy if emesis is present. Frequent burping, irritability, and the passing of gas would be expected findings following a pyloromyotomy and would indicate tolerance of the feeding. 4. An infant is not tolerating feedings after a pyloromyotomy if emesis is present. Frequent burping, irritability, and the passing of gas would be expected findings following a pyloromyotomy and would indicate tolerance of the feeding. Page Ref: 668 Cognitive Level: Analyzing Client Need &Sub: Physiological Integrity: Physiological Adaptation Standards: QSEN Competencies: Patient-centered care | AACN Essential Competencies: Essential IX: Baccalaureate generalist nursing practice | NLN Competencies: Nursing judgement | Nursing/Integrated Concepts: Nursing Process: Evaluation/Coordination of care Learning Outcome: LO 25.4 Contrast nursing management and plan care for disorders of the gastrointestinal system for the child needing abdominal surgery versus the child needing nonoperative management. MNL LO: Gastrointestinal Disorders/Apply the nursing process in providing care for the child and family. 4) An infant born with an omphalocele defect is admitted to the intensive-care nursery. Which instruction from the nurse manager to the unlicensed assistive personnel is most appropriate? 1. Prepare a warmer. 2. Prepare a crib. 3. Prepare a feeding of formula. 4. Prepare the bilirubin light. Answer: 1 Explanation: 1. Omphalocele is a congenital malformation in which intra-abdominal contents herniate through the umbilical cord. The infant many lose heat through the viscera; a warmer is indicated to prevent hypothermia. The crib would not provide adequate maintenance of temperature control. The infant is NPO (nothing by mouth) preoperatively and may or may not need a bilirubin light before surgery. 2. Omphalocele is a congenital malformation in which intra-abdominal contents herniate through the umbilical cord. The infant many lose heat through the viscera; a warmer is indicated to prevent hypothermia. The crib would not provide adequate maintenance of temperature control. The infant is NPO (nothing by mouth) preoperatively and may or may not need a bilirubin light before surgery. 3. Omphalocele is a congenital malformation in which intra-abdominal contents herniate through the umbilical cord. The infant many lose heat through the viscera; a warmer is indicated to prevent hypothermia. The crib would not provide adequate maintenance of temperature control. The infant is NPO (nothing by mouth) preoperatively and may or may not need a bilirubin light before surgery. 4. Omphalocele is a congenital malformation in which intra-abdominal contents herniate through the umbilical cord. The infant many lose heat through the viscera; a warmer is indicated to prevent hypothermia. The crib would not provide adequate maintenance of temperature control. The infant is NPO (nothing by mouth) preoperatively and may or may not need a bilirubin light before surgery. Page Ref: 671 Cognitive Level: Applying Client Need &Sub: Safe and Effective Care Environment: Management of Care Standards: QSEN Competencies: Patient-centered care | AACN Essential Competencies: Essential IX: Baccalaureate generalist nursing practice | NLN Competencies: Nursing judgement | Nursing/Integrated Concepts: Nursing Process: Implementation/Coordination of care Learning Outcome: LO 25.4 Contrast nursing management and plan care for disorders of the gastrointestinal system for the child needing abdominal surgery versus the child needing nonoperative management. MNL LO: Gastrointestinal Disorders/Differentiate treatment options and strategies for continuity of care for the child and family. 5) The nurse is providing instruction to the parents of an infant with a colostomy. Which statement by the parents indicates appropriate understanding of the teaching session? 1. "We will change the colostomy bag with each wet diaper." 2. "We will use adhesive enhancers when we change the bag." 3. "We will watch for skin irritation around the stoma." 4. "We will expect a moderate amount of bleeding after cleansing the area around the stoma." Answer: 3 Explanation: 1. Skin irritation around the stoma should be assessed; it may indicate leakage. Physical or chemical skin irritation may occur if the appliance is changed too frequently or with each wet diaper. Adhesive enhancers should be avoided on the skin of newborns. Their skin layers are thin, and removal of the appliance can strip off the skin. Also, adhesive contains latex, and its constant use is not advised due to risk of latex allergy development. Bleeding is usually attributable to excessive cleaning. 2. Skin irritation around the stoma should be assessed; it may indicate leakage. Physical or chemical skin irritation may occur if the appliance is changed too frequently or with each wet diaper. Adhesive enhancers should be avoided on the skin of newborns. Their skin layers are thin, and removal of the appliance can strip off the skin. Also, adhesive contains latex, and its constant use is not advised due to risk of latex allergy development. Bleeding is usually attributable to excessive cleaning. 3. Skin irritation around the stoma should be assessed; it may indicate leakage. Physical or chemical skin irritation may occur if the appliance is changed too frequently or with each wet diaper. Adhesive enhancers should be avoided on the skin of newborns. Their skin layers are thin, and removal of the appliance can strip off the skin. Also, adhesive contains latex, and its constant use is not advised due to risk of latex allergy development. Bleeding is usually attributable to excessive cleaning. 4. Skin irritation around the stoma should be assessed; it may indicate leakage. Physical or chemical skin irritation may occur if the appliance is changed too frequently or with each wet diaper. Adhesive enhancers should be avoided on the skin of newborns. Their skin layers are thin, and removal of the appliance can strip off the skin. Also, adhesive contains latex, and its constant use is not advised due to risk of latex allergy development. Bleeding is usually attributable to excessive cleaning. Page Ref: 673 Cognitive Level: Analyzing Client Need &Sub: Health Promotion and Maintenance Standards: QSEN Competencies: Patient-centered care | AACN Essential Competencies: Essential VII: Clinical prevention and population health | NLN Competencies: Human flourishing | Nursing/Integrated Concepts: Nursing Process: Evaluation/Health teaching and health promotion Learning Outcome: LO 25.4 Contrast nursing management and plan care for disorders of the gastrointestinal system for the child needing abdominal surgery versus the child needing nonoperative management. MNL LO: Gastrointestinal Disorders/Apply the nursing process in providing care for the child and family. 6) A nurse is preparing for the delivery of a newborn with a known diaphragmatic hernia defect. Which equipment does the nurse ensure is prepared at the bedside? 1. Intubation setup 2. Appropriate bag and mask 3. Sterile gauze and saline 4. Soft arm restraints Answer: 1 Explanation: 1. A diaphragmatic hernia (protrusion of abdominal contents into the chest cavity through a defect in the diaphragm) is a life-threatening condition. Intubation is required immediately so the newborn's respiratory status can be stabilized. A bag and mask will not be adequate to ventilate a newborn with this condition. The defect is not external, so sterile gauze and saline are not needed. Soft arm restraints are not immediately necessary. 2. A diaphragmatic hernia (protrusion of abdominal contents into the chest cavity through a defect in the diaphragm) is a life-threatening condition. Intubation is required immediately so the newborn's respiratory status can be stabilized. A bag and mask will not be adequate to ventilate a newborn with this condition. The defect is not external, so sterile gauze and saline are not needed. Soft arm restraints are not immediately necessary. 3. A diaphragmatic hernia (protrusion of abdominal contents into the chest cavity through a defect in the diaphragm) is a life-threatening condition. Intubation is required immediately so the newborn's respiratory status can be stabilized. A bag and mask will not be adequate to ventilate a newborn with this condition. The defect is not external, so sterile gauze and saline are not needed. Soft arm restraints are not immediately necessary. 4. A diaphragmatic hernia (protrusion of abdominal contents into the chest cavity through a defect in the diaphragm) is a life-threatening condition. Intubation is required immediately so the newborn's respiratory status can be stabilized. A bag and mask will not be adequate to ventilate a newborn with this condition. The defect is not external, so sterile gauze and saline are not needed. Soft arm restraints are not immediately necessary. Page Ref: 675 Cognitive Level: Applying Client Need &Sub: Physiological Integrity: Reduction of Risk Potential Standards: QSEN Competencies: Patient-centered care | AACN Essential Competencies: Essential IX: Baccalaureate generalist nursing practice | NLN Competencies: Human flourishing | Nursing/Integrated Concepts: Nursing Process: Implementation/Coordination of care Learning Outcome: LO 25.4 Contrast nursing management and plan care for disorders of the gastrointestinal system for the child needing abdominal surgery versus the child needing nonoperative management. MNL LO: Gastrointestinal Disorders/Apply the nursing process in providing care for the child and family. 7) The nurse is administering several medications to an infant with neurologic impairment and delay. Which medication is a proton pump inhibitor that is administered for gastroesophageal reflux? 1. Omeprazole 2. Ranitidine 3. Phenytoin 4. Glycopyrrolate Answer: 1 Explanation: 1. Omeprazole is the proton pump inhibitor that blocks the action of acid- producing cells and is used to treat gastroesophageal reflux. Ranitidine causes the stomach to produce less acid and may be used to treat gastroesophageal reflux, but it is a histamine-2 receptor blocker. Phenytoin is an anticonvulsant used to treat seizures, and glycopyrrolate is an anticholinergic agent used to inhibit excessive salivation. 2. Omeprazole is the proton pump inhibitor that blocks the action of acid-producing cells and is used to treat gastroesophageal reflux. Ranitidine causes the stomach to produce less acid and may be used to treat gastroesophageal reflux, but it is a histamine-2 receptor blocker. Phenytoin is an anticonvulsant used to treat seizures, and glycopyrrolate is an anticholinergic agent used to inhibit excessive salivation. 3. Omeprazole is the proton pump inhibitor that blocks the action of acid-producing cells and is used to treat gastroesophageal reflux. Ranitidine causes the stomach to produce less acid and may be used to treat gastroesophageal reflux, but it is a histamine-2 receptor blocker. Phenytoin is an anticonvulsant used to treat seizures, and glycopyrrolate is an anticholinergic agent used to inhibit excessive salivation. 4. Omeprazole is the proton pump inhibitor that blocks the action of acid-producing cells and is used to treat gastroesophageal reflux. Ranitidine causes the stomach to produce less acid and may be used to treat gastroesophageal reflux, but it is a histamine-2 receptor blocker. Phenytoin is an anticonvulsant used to treat seizures, and glycopyrrolate is an anticholinergic agent used to inhibit excessive salivation. Page Ref: 670 Cognitive Level: Understanding Client Need &Sub: Physiological Integrity: Pharmacological and Parenteral Therapies Standards: QSEN Competencies: Patient-centered care | AACN Essential Competencies: Essential IX: Baccalaureate generalist nursing practice | NLN Competencies: Human flourishing | Nursing/Integrated Concepts: Nursing Process: Implementation/Coordination of care Learning Outcome: LO 25.2 Discuss the pathophysiologic processes associated with specific gastrointestinal disorders in the pediatric population. MNL LO: Gastrointestinal Disorders/Examine etiology, risk factors, pathophysiology, and clinical manifestations as seen in children. 8) A newborn is diagnosed with Hirschsprung disease. Which clinical manifestations found on assessment support this newborn's diagnosis? 1. Acute diarrhea; dehydration 2. Failure to pass meconium; abdominal distension 3. Currant jelly; gelatinous stools; pain 4. Projectile vomiting; altered electrolytes Answer: 2 Explanation: 1. Hirschsprung disease is the absence of autonomic parasympathetic ganglion cells in the colon that prevent peristalsis at that portion of the intestine. In newborns, the symptoms include failure to pass meconium and abdominal distension. Acute diarrhea and dehydration are symptoms characteristic of gastroenteritis. Currant jelly, gelatinous stools, and pain are symptoms of intussusception, and projectile vomiting and altered electrolytes are symptoms of pyloric stenosis. 2. Hirschsprung disease is the absence of autonomic parasympathetic ganglion cells in the colon that prevent peristalsis at that portion of the intestine. In newborns, the symptoms include failure to pass meconium and abdominal distension. Acute diarrhea and dehydration are symptoms characteristic of gastroenteritis. Currant jelly, gelatinous stools, and pain are symptoms of intussusception, and projectile vomiting and altered electrolytes are symptoms of pyloric stenosis. 3. Hirschsprung disease is the absence of autonomic parasympathetic ganglion cells in the colon that prevent peristalsis at that portion of the intestine. In newborns, the symptoms include failure to pass meconium and abdominal distension. Acute diarrhea and dehydration are symptoms characteristic of gastroenteritis. Currant jelly, gelatinous stools, and pain are symptoms of intussusception, and projectile vomiting and altered electrolytes are symptoms of pyloric stenosis. 4. Hirschsprung disease is the absence of autonomic parasympathetic ganglion cells in the colon that prevent peristalsis at that portion of the intestine. In newborns, the symptoms include failure to pass meconium and abdominal distension. Acute diarrhea and dehydration are symptoms characteristic of gastroenteritis. Currant jelly, gelatinous stools, and pain are symptoms of intussusception, and projectile vomiting and altered electrolytes are symptoms of pyloric stenosis. Page Ref: 673 Cognitive Level: Analyzing Client Need &Sub: Physiological Integrity: Physiological Adaptation Standards: QSEN Competencies: Patient-centered care | AACN Essential Competencies: Essential IX: Baccalaureate generalist nursing practice | NLN Competencies: Human flourishing | Nursing/Integrated Concepts: Nursing Process: Assessment/Coordination of care Learning Outcome: LO 25.3 Identify signs and symptoms that may indicate a disorder of the gastrointestinal system. MNL LO: Gastrointestinal Disorders/Examine etiology, risk factors, pathophysiology, and clinical manifestations as seen in children. 9) A child with severe gastroenteritis is admitted to a semiprivate room on the pediatric unit. The charge nurse should place this client with which roommate? 1. An infant with meningitis 2. A child with fever and neutropenia 3. Another child with gastroenteritis 4. A child recovering from an appendectomy Answer: 3 Explanation: 1. Gastroenteritis may be viral or bacterial and can be infectious. It is best to cohort children with this infectious process. Good handwashing is essential to prevent the spread. An infant with meningitis, a child with fever and neutropenia, and a child recovering from an appendectomy should not be placed with another child with an infectious process. 2. Gastroenteritis may be viral or bacterial and can be infectious. It is best to cohort children with this infectious process. Good handwashing is essential to prevent the spread. An infant with meningitis, a child with fever and neutropenia, and a child recovering from an appendectomy should not be placed with another child with an infectious process. 3. Gastroenteritis may be viral or bacterial and can be infectious. It is best to cohort children with this infectious process. Good handwashing is essential to prevent the spread. An infant with meningitis, a child with fever and neutropenia, and a child recovering from an appendectomy should not be placed with another child with an infectious process. 4. Gastroenteritis may be viral or bacterial and can be infectious. It is best to cohort children with this infectious process. Good handwashing is essential to prevent the spread. An infant with meningitis, a child with fever and neutropenia, and a child recovering from an appendectomy should not be placed with another child with an infectious process. Page Ref: 684 Cognitive Level: Applying Client Need &Sub: Safe and Effective Care Environment: Safety and Infection Control Standards: QSEN Competencies: Patient-centered care | AACN Essential Competencies: Essential IX: Baccalaureate generalist nursing practice | NLN Competencies: Nursing judgement | Nursing/Integrated Concepts: Nursing Process: Implementation/Coordination of care Learning Outcome: LO 25.2 Discuss the pathophysiologic processes associated with specific gastrointestinal disorders in the pediatric population. MNL LO: Gastrointestinal Disorders/Examine etiology, risk factors, pathophysiology, and clinical manifestations as seen in children. 10) The nurse is preparing to ambulate a school-age client who had an appendectomy. In addition to pharmacological pain management, the nurse can use which nonpharmacological pain-management strategy for this client? 1. A heating pad 2. A warm, moist pack 3. A pillow on the abdomen 4. An ice pack Answer: 3 Explanation: 1. A pillow placed on the abdomen can be a nonpharmacological strategy to decrease discomfort after an appendectomy. Heat and ice are not used on the incisional area as they can impair the healing process of the wound. 2. A pillow placed on the abdomen can be a nonpharmacological strategy to decrease discomfort after an appendectomy. Heat and ice are not used on the incisional area as they can impair the healing process of the wound. 3. A pillow placed on the abdomen can be a nonpharmacological strategy to decrease discomfort after an appendectomy. Heat and ice are not used on the incisional area as they can impair the healing process of the wound. 4. A pillow placed on the abdomen can be a nonpharmacological strategy to decrease discomfort after an appendectomy. Heat and ice are not used on the incisional area as they can impair the healing process of the wound. Page Ref: 679 Cognitive Level: Applying Client Need &Sub: Physiological Integrity: Basic Care and Comfort Standards: QSEN Competencies: Patient-centered care | AACN Essential Competencies: Essential IX: Baccalaureate generalist nursing practice | NLN Competencies: Human flourishing | Nursing/Integrated Concepts: Nursing Process: Implementation/Coordination of care Learning Outcome: LO 25.4 Contrast nursing management and plan care for disorders of the gastrointestinal system for the child needing abdominal surgery versus the child needing nonoperative management. MNL LO: Gastrointestinal Disorders/Apply the nursing process in providing care for the child and family. 11) A neonate is fed 20 mL of formula every three hours by orogastric lavage. At the beginning of this feeding, the nurse aspirates 15 mL of gastric residual. Which action by the nurse is the most appropriate? 1. Withhold the feeding and notify the healthcare provider. 2. Replace the residual and continue with the full feeding. 3. Replace the residual but only give 5 mL of the feeding. 4. Withhold the feeding and check the residual in three hours. Answer: 1 Explanation: 1. Residual of more than half the amount of feeding indicates a feeding intolerance and could be a sign of necrotizing enterocolitis. Early detection of enterocolitis is essential, and aggressive management is required. Therefore, the healthcare provider should be notified of this finding. The amount of residual is too much to replace and continue with the feeding, and waiting for 3 hours to recheck the residual could delay treatment of a serious condition. 2. Residual of more than half the amount of feeding indicates a feeding intolerance and could be a sign of necrotizing enterocolitis. Early detection of enterocolitis is essential, and aggressive management is required. Therefore, the healthcare provider should be notified of this finding. The amount of residual is too much to replace and continue with the feeding, and waiting for 3 hours to recheck the residual could delay treatment of a serious condition. 3. Residual of more than half the amount of feeding indicates a feeding intolerance and could be a sign of necrotizing enterocolitis. Early detection of enterocolitis is essential, and aggressive management is required. Therefore, the healthcare provider should be notified of this finding. The amount of residual is too much to replace and continue with the feeding, and waiting for 3 hours to recheck the residual could delay treatment of a serious condition. 4. Residual of more than half the amount of feeding indicates a feeding intolerance and could be a sign of necrotizing enterocolitis. Early detection of enterocolitis is essential, and aggressive management is required. Therefore, the healthcare provider should be notified of this finding. The amount of residual is too much to replace and continue with the feeding, and waiting for 3 hours to recheck the residual could delay treatment of a serious condition. Page Ref: 688 Cognitive Level: Applying Client Need &Sub: Physiological Integrity: Physiological Adaptation Standards: QSEN Competencies: Teamwork and collaboration | AACN Essential Competencies: Essential VI: Interprofessional communication and collaboration for improving patient health outcomes | NLN Competencies: Nursing judgement | Nursing/Integrated Concepts: Nursing Process: Implementation/Collaboration Learning Outcome: LO 25.3 Identify signs and symptoms that may indicate a disorder of the gastrointestinal system. MNL LO: Gastrointestinal Disorders/Examine etiology, risk factors, pathophysiology, and clinical manifestations as seen in children. 12) A child with inflammatory bowel disease is prescribed prednisone daily. At which time is it most appropriate for the family to administer the prednisone? 1. Between meals 2. One hour before meals 3. At bedtime 4. With meals Answer: 4 Explanation: 1. Prednisone, a corticosteroid, can cause gastric irritation. It should be administered with meals to reduce the gastric irritation. 2. Prednisone, a corticosteroid, can cause gastric irritation. It should be administered with meals to reduce the gastric irritation. 3. Prednisone, a corticosteroid, can cause gastric irritation. It should be administered with meals to reduce the gastric irritation. 4. Prednisone, a corticosteroid, can cause gastric irritation. It should be administered with meals to reduce the gastric irritation. Page Ref: 682 Cognitive Level: Applying Client Need &Sub: Physiological Integrity: Pharmacological and Parenteral Therapies Standards: QSEN Competencies: Patient-centered care | AACN Essential Competencies: Essential IX: Baccalaureate generalist nursing practice | NLN Competencies: Nursing judgement | Nursing/Integrated Concepts: Nursing Process: Implementation/Coordination of care Learning Outcome: LO 25.5 Analyze developmentally appropriate approaches for nursing management of gastrointestinal disorders in the pediatric population. MNL LO: Gastrointestinal Disorders/Apply the nursing process in providing care for the child and family. 13) The nurse is caring for a school-age client who had an appendectomy after a ruptured appendix. Which orders does the nurse anticipate for this client? Select all that apply. 1. Antibiotics 2. A clear liquid diet 3. NG tube 4. Vital signs every 4 hours 5. Frequent monitoring of bowel sounds Answer: 1, 3, 4, 5 Explanation: 1. Antibiotics, an NG tube, vital signs every 4 hours, and frequent monitoring of bowel sounds are appropriate interventions following a ruptured appendix. The client is NPO until bowel sounds return. 2. Antibiotics, an NG tube, vital signs every 4 hours, and frequent monitoring of bowel sounds are appropriate interventions following a ruptured appendix. The client is NPO until bowel sounds return. 3. Antibiotics, an NG tube, vital signs every 4 hours, and frequent monitoring of bowel sounds are appropriate interventions following a ruptured appendix. The client is NPO until bowel sounds return. 4. Antibiotics, an NG tube, vital signs every 4 hours, and frequent monitoring of bowel sounds are appropriate interventions following a ruptured appendix. The client is NPO until bowel sounds return. 5. Antibiotics, an NG tube, vital signs every 4 hours, and frequent monitoring of bowel sounds are appropriate interventions following a ruptured appendix. The client is NPO until bowel sounds return. Page Ref: 678-679 Cognitive Level: Applying Client Need &Sub: Physiological Integrity: Basic Care and Comfort Standards: QSEN Competencies: Patient-centered care | AACN Essential Competencies: Essential IX: Baccalaureate generalist nursing practice | NLN Competencies: Nursing judgement | Nursing/Integrated Concepts: Nursing Process: Implementation/Coordination of care Learning Outcome: LO 25.4 Contrast nursing management and plan care for disorders of the gastrointestinal system for the child needing abdominal surgery versus the child needing nonoperative management. MNL LO: Gastrointestinal Disorders/Apply the nursing process in providing care for the child and family. 14) A 3-day-old preterm infant is diagnosed with necrotizing enterocolitis. The nurse plans care around the frequent radiographs. How frequently should the nurse anticipate that the radiology staff will bring the portable machine to the nursery? 1. Every 6 hours 2. Every 12 hours 3. Every 24 hours 4. Every 48 hours Answer: 1 Explanation: 1. Radiographs are done every 6 hours to evaluate for perforation. 2. Radiographs are done every 6 hours to evaluate for perforation. 3. Radiographs are done every 6 hours to evaluate for perforation. 4. Radiographs are done every 6 hours to evaluate for perforation. Page Ref: 680 Cognitive Level: Applying Client Need &Sub: Physiological Integrity: Basic Care and Comfort Standards: QSEN Competencies: Teamwork and collaboration | AACN Essential Competencies: Essential VI: Interprofessional communication and collaboration for improving patient health outcomes | NLN Competencies: Human flourishing | Nursing/Integrated Concepts: Nursing Process: Implementation/Collaboration Learning Outcome: LO 25.4 Contrast nursing management and plan care for disorders of the gastrointestinal system for the child needing abdominal surgery versus the child needing nonoperative management. MNL LO: Gastrointestinal Disorders/Apply the nursing process in providing care for the child and family. 15) A school-age client is recovering after abdominal surgery. The nurse is planning care for the return of bowel function. Which intervention should be included in the client's plan of care? 1. Fowler's position 3 times per day for 30 minutes each time 2. Assist the child in choosing a low-fat diet. 3. Commode at bedside 4. Ambulate 3 to 4 times a day. Answer: 4 Explanation: 1. The best data that indicate return of bowel sounds are flatus and passage of stool. Ambulation is the primary intervention to assist with both. A Fowler's position, bedside commode, and a low-fat diet will not assist with bowel function. 2. The best data that indicate return of bowel sounds are flatus and passage of stool. Ambulation is the primary intervention to assist with both. A Fowler's position, bedside commode, and a low- fat diet will not assist with bowel function. 3. The best data that indicate return of bowel sounds are flatus and passage of stool. Ambulation is the primary intervention to assist with both. A Fowler's position, bedside commode, and a low- fat diet will not assist with bowel function. 4. The best data that indicate return of bowel sounds are flatus and passage of stool. Ambulation is the primary intervention to assist with both. A Fowler's position, bedside commode, and a low- fat diet will not assist with bowel function. Page Ref: 679 Cognitive Level: Applying Client Need &Sub: Health Promotion and Maintenance Standards: QSEN Competencies: Patient-centered care | AACN Essential Competencies: Essential IX: Baccalaureate generalist nursing practice : NLN Competencies: Nursing judgement | Nursing/Integrated Concepts: Nursing Process: Implementation/Coordination of care Learning Outcome: LO 25.4 Contrast nursing management and plan care for disorders of the gastrointestinal system for the child needing abdominal surgery versus the child needing nonoperative management. MNL LO: Gastrointestinal Disorders/Apply the nursing process in providing care for the child and family. 16) A child experienced a lacerated spleen in a motor vehicle accident. Which is the highest- priority nursing intervention on admission to the pediatric intensive care unit (PICU) following surgery? 1. Observing for signs of hypovolemic shock 2. Maintaining IV fluids 3. Implementing strict bedrest 4. Administering blood products as ordered Answer: 1 Explanation: 1. The priority nursing intervention is observing for signs of hypovolemic shock due to bleeding from the lacerated spleen. The other interventions are appropriate but not the highest priority. 2. The priority nursing intervention is observing for signs of hypovolemic shock due to bleeding from the lacerated spleen. The other interventions are appropriate but not the highest priority. 3. The priority nursing intervention is observing for signs of hypovolemic shock due to bleeding from the lacerated spleen. The other interventions are appropriate but not the highest priority. 4. The priority nursing intervention is observing for signs of hypovolemic shock due to bleeding from the lacerated spleen. The other interventions are appropriate but not the highest priority. Page Ref: 696-697 Cognitive Level: Analyzing Client Need &Sub: Health Promotion and Maintenance Standards: QSEN Competencies: Patient-centered care | AACN Essential Competencies: Essential IX: Baccalaureate generalist nursing practice | NLN Competencies: Nursing judgement | Nursing/Integrated Concepts: Nursing Process: Evaluation/Coordination of care Learning Outcome: LO 25.6 Plan nursing care for the child with an injury to the gastrointestinal system. MNL LO: Gastrointestinal Disorders/Apply the nursing process in providing care for the child and family. 17) The nurse educator is preparing an in-service on the basic functions of the gastrointestinal (GI) system. Which statements will the nurse educator include in the in-service? Select all that apply. 1. "The GI system is responsible for the ingestion of fluids and nutrients." 2. "The GI system is responsible for the excretion of fluids and nutrients." 3. "The GI system is responsible for the metabolism of nutrients." 4. "As infants grow, their stomach capacity increases, decreasing the frequency with which they need to be fed." 5. "By the second year of life, digestive processes are still developing." Answer: 1, 3, 4 Explanation: 1. The GI system is responsible for the ingestion of fluids and nutrients as well as the metabolism of nutrients. As infants grow, their stomach capacity increases, which does decrease the frequency with which they need to be fed. The GI system is responsible for the excretion of waste products. By the second year of life, digestive processes are fairly complete. 2. The GI system is responsible for the ingestion of fluids and nutrients as well as the metabolism of nutrients. As infants grow, their stomach capacity increases, which does decrease the frequency with which they need to be fed. The GI system is responsible for the excretion of waste products. By the second year of life, digestive processes are fairly complete. 3. The GI system is responsible for the ingestion of fluids and nutrients as well as the metabolism of nutrients. As infants grow, their stomach capacity increases, which does decrease the frequency with which they need to be fed. The GI system is responsible for the excretion of waste products. By the second year of life, digestive processes are fairly complete. 4. The GI system is responsible for the ingestion of fluids and nutrients as well as the metabolism of nutrients. As infants grow, their stomach capacity increases, which does decrease the frequency with which they need to be fed. The GI system is responsible for the excretion of waste products. By the second year of life, digestive processes are fairly complete. 5. The GI system is responsible for the ingestion of fluids and nutrients as well as the metabolism of nutrients. As infants grow, their stomach capacity increases, which does decrease the frequency with which they need to be fed. The GI system is responsible for the excretion of waste products. By the second year of life, digestive processes are fairly complete. Page Ref: 657-659 Cognitive Level: Applying Client Need &Sub: Physiological Integrity: Physiological Adaptation Standards: QSEN Competencies: Patient-centered care | AACN Essential Competencies: Essential VII: Clinical prevention and population health | NLN Competencies: Nursing judgement | Nursing/Integrated Concepts: Nursing Process: Implementation/Health teaching and health promotion Learning Outcome: LO 25.1 Describe the general function of the gastrointestinal system. MNL LO: Gastrointestinal Disorders/Examine etiology, risk factors, pathophysiology, and clinical manifestations as seen in children. 18) The nurse is caring for a 5-month-old with biliary atresia. The mother asks why the healthcare provider wants her child to take the medication, cholestyramine. What would the nurse's response be? 1. Decrease itching 2. Increase WBCs 3. Decrease use of antibiotics 4. Increase appetite Answer: 1 Explanation: 1. Cholestyramine is taken to decrease itching. 2. Cholestyramine is taken to decrease itching. 3. Cholestyramine is taken to decrease itching. 4. Cholestyramine is taken to decrease itching. Page Ref: 693 Cognitive Level: Analyzing Client Need &Sub: Health Promotion and Maintenance Standards: QSEN Competencies: Patient-centered care | AACN Essential Competencies: Essential VII: Clinical prevention and population health | NLN Competencies: Human flourishing | Nursing/Integrated Concepts: Nursing Process: Assessment/Coordination of care Learning Outcome: LO 25.3 Identify signs and symptoms that may indicate a disorder of the gastrointestinal system. MNL LO: Gastrointestinal Disorders/Differentiate treatment options and strategies for continuity of care for the child and family. 19) The nurse is caring for the newborn with hyperbilirubinemia. What nursing diagnoses would the nurse address? Select all that apply. 1. Activity intolerance 2. Deficient fluid volume 3. Risk for impaired attachment 4. Ineffective breathing pattern 5. Risk for imbalanced body temperature Answer: 2, 3, 5 Explanation: 1. Nursing diagnoses that may apply to the newborn with hyperbilirubinemia are deficient fluid volume, risk for impaired attachment, risk for imbalanced body temperature, and risk for injury. 2. Nursing diagnoses that may apply to the newborn with hyperbilirubinemia are deficient fluid volume, risk for impaired attachment, risk for imbalanced body temperature, and risk for injury. 3. Nursing diagnoses that may apply to the newborn with hyperbilirubinemia are deficient fluid volume, risk for impaired attachment, risk for imbalanced body temperature, and risk for injury. 4. Nursing diagnoses that may apply to the newborn with hyperbilirubinemia are deficient fluid volume, risk for impaired attachment, risk for imbalanced body temperature, and risk for injury. 5. Nursing diagnoses that may apply to the newborn with hyperbilirubinemia are deficient fluid volume, risk for impaired attachment, risk for imbalanced body temperature, and risk for injury. Page Ref: 691-692 Cognitive Level: Applying Client Need &Sub: Safe and Effective Care Environment: Management of Care Standards: QSEN Competencies: Patient-centered care | AACN Essential Competencies: Essential IX: Baccalaureate generalist nursing practice | NLN Competencies: Human flourishing | Nursing/Integrated Concepts: Nursing Process: Diagnosis/Coordination of care Learning Outcome: LO 25.6 Plan nursing care for the child with an injury to the gastrointestinal system. MNL LO: Gastrointestinal Disorders/Apply the nursing process in providing care for the child and family. Principles of Pediatric Nursing: Caring for Children, 7e (Ball et al.) Chapter 26 Alterations in Genitourinary Function 1) The nurse is providing care to a male infant who is diagnosed with hypospadias. Which clinical manifestation does the nurse anticipate when assessing this infant? 1. A urethral meatus that is located on the ventral surface of the penis 2. The presence of foreskin 3. A small opening or a fissure that extends the entire length of the penis 4. An opening on the dorsal surface of the penis Answer: 1 Explanation: 1. For an infant diagnosed with hypospadias, the nurse would anticipate a urethral meatus that is located on the ventral surface of the penis. Infants diagnosed with hypospadias may also have a partial absence of the foreskin. A small opening or a fissure that extends the entire length of the penis or an opening on the dorsal side of the penis would be expected for an infant diagnosed with epispadias. 2. For an infant diagnosed with hypospadias, the nurse would anticipate a urethral meatus that is located on the ventral surface of the penis. Infants diagnosed with hypospadias may also have a partial absence of the foreskin. A small opening or a fissure that extends the entire length of the penis or an opening on the dorsal side of the penis would be expected for an infant diagnosed with epispadias. 3. For an infant diagnosed with hypospadias, the nurse would anticipate a urethral meatus that is located on the ventral surface of the penis. Infants diagnosed with hypospadias may also have a partial absence of the foreskin. A small opening or a fissure that extends the entire length of the penis or an opening on the dorsal side of the penis would be expected for an infant diagnosed with epispadias. 4. For an infant diagnosed with hypospadias, the nurse would anticipate a urethral meatus that is located on the ventral surface of the penis. Infants diagnosed with hypospadias may also have a partial absence of the foreskin. A small opening or a fissure that extends the entire length of the penis or an opening on the dorsal side of the penis would be expected for an infant diagnosed with epispadias. Page Ref: 708-709 Cognitive Level: Applying Client Need &Sub: Physiological Integrity: Physiological Adaptation Standards: QSEN Competencies: Patient-centered care | AACN Essential Competencies: Essential IX: Baccalaureate generalist nursing practice | NLN Competencies: Human flourishing | Nursing/Integrated Concepts: Nursing Process: Assessment/Coordination of care Learning Outcome: LO 26.3 Discuss the nursing management of a child with a structural defect of the genitourinary system. MNL LO: Renal and Genitourinary Disorders/Apply the nursing process in providing care for the child and family. 2) A nurse is preparing to admit a child with possible obstructive uropathy. Which laboratory test should the nurse expect to draw on this child? 1. Platelet count 2. Blood urea nitrogen (BUN) and creatinine 3. Partial thromboplastin time (PTT) 4. Blood culture Answer: 2 Explanation: 1. The blood urea nitrogen (BUN) and creatinine are serum lab tests for kidney function. Obstructive uropathy is a structural or functional abnormality of the urinary system that interferes with urine flow and results in urine backflow into the kidneys; therefore, the BUN and creatinine will be elevated. Platelet count and partial thromboplastin time (PTT) are drawn when a bleeding disorder is suspected. A blood culture is done when an infectious process is suspected. 2. The blood urea nitrogen (BUN) and creatinine are serum lab tests for kidney function. Obstructive uropathy is a structural or functional abnormality of the urinary system that interferes with urine flow and results in urine backflow into the kidneys; therefore, the BUN and creatinine will be elevated. Platelet count and partial thromboplastin time (PTT) are drawn when a bleeding disorder is suspected. A blood culture is done when an infectious process is suspected. 3. The blood urea nitrogen (BUN) and creatinine are serum lab tests for kidney function. Obstructive uropathy is a structural or functional abnormality of the urinary system that interferes with urine flow and results in urine backflow into the kidneys; therefore, the BUN and creatinine will be elevated. Platelet count and partial thromboplastin time (PTT) are drawn when a bleeding disorder is suspected. A blood culture is done when an infectious process is suspected. 4. The blood urea nitrogen (BUN) and creatinine are serum lab tests for kidney function. Obstructive uropathy is a structural or functional abnormality of the urinary system that interferes with urine flow and results in urine backflow into the kidneys; therefore, the BUN and creatinine will be elevated. Platelet count and partial thromboplastin time (PTT) are drawn when a bleeding disorder is suspected. A blood culture is done when an infectious process is suspected. Page Ref: 710-711 Cognitive Level: Applying Client Need &Sub: Physiological Integrity: Reduction of Risk Potential Standards: QSEN Competencies: Patient-centered care | AACN Essential Competencies: Essential IX: Baccalaureate generalist nursing practice | NLN Competencies: Human flourishing | Nursing/Integrated Concepts: Nursing Process: Planning/Coordination of care Learning Outcome: LO 26.3 Discuss the nursing management of a child with a structural defect of the genitourinary system. MNL LO: Renal and Genitourinary Disorders/Apply the nursing process in providing care for the child and family. 3) The nurse is preparing to discharge a toddler-age client who just had an orchiopexy. Which discharge instruction is appropriate for this client? 1. Information to the parents about the child's resuming normal vigorous activities 2. Discussion with the parents about the low incidence of testicular malignancy and no further need for any follow-up 3. Explanation to the parents about the need for loose, nonrestrictive clothing 4. Reassurance to the parents that infertility is not a future risk Answer: 3 Explanation: 1. Orchiopexy is the surgical correction of cryptorchidism (failure of the testes to descend into the scrotal sac). Discharge instructions should include information about the need for loose, nonrestrictive clothing to avoid pressure on the postoperative site. The risk of testicular cancer is 35 to 50 times greater in men with a history of cryptorchidism. Long-term planning includes teaching the child to perform monthly testicular examinations once puberty has been reached. Vigorous activities such as straddling toys, riding bicycles, or rough play should be avoided for up to two weeks following surgery to promote healing and prevent injury. A discussion of fertility and the possible need for fertility testing is important, since cryptorchidism increases the risk of infertility. 2. Orchiopexy is the surgical correction of cryptorchidism (failure of the testes to descend into the scrotal sac). Discharge instructions should include information about the need for loose, nonrestrictive clothing to avoid pressure on the postoperative site. The risk of testicular cancer is 35 to 50 times greater in men with a history of cryptorchidism. Long-term planning includes teaching the child to perform monthly testicular examinations once puberty has been reached. Vigorous activities such as straddling toys, riding bicycles, or rough play should be avoided for up to two weeks following surgery to promote healing and prevent injury. A discussion of fertility and the possible need for fertility testing is important, since cryptorchidism increases the risk of infertility. 3. Orchiopexy is the surgical correction of cryptorchidism (failure of the testes to descend into the scrotal sac). Discharge instructions should include information about the need for loose, nonrestrictive clothing to avoid pressure on the postoperative site. The risk of testicular cancer is 35 to 50 times greater in men with a history of cryptorchidism. Long-term planning includes teaching the child to perform monthly testicular examinations once puberty has been reached. Vigorous activities such as straddling toys, riding bicycles, or rough play should be avoided for up to two weeks following surgery to promote healing and prevent injury. A discussion of fertility and the possible need for fertility testing is important, since cryptorchidism increases the risk of infertility. 4. Orchiopexy is the surgical correction of cryptorchidism (failure of the testes to descend into the scrotal sac). Discharge instructions should include information about the need for loose, nonrestrictive clothing to avoid pressure on the postoperative site. The risk of testicular cancer is 35 to 50 times greater in men with a history of cryptorchidism. Long-term planning includes teaching the child to perform monthly testicular examinations once puberty has been reached. Vigorous activities such as straddling toys, riding bicycles, or rough play should be avoided for up to two weeks following surgery to promote healing and prevent injury. A discussion of fertility and the possible need for fertility testing is important, since cryptorchidism increases the risk of infertility. Page Ref: 732 Cognitive Level: Applying Client Need &Sub: Health Promotion and Maintenance Standards: QSEN Competencies: Patient-centered care | AACN Essential Competencies: Essential VII: Clinical prevention and population health | NLN Competencies: Human flourishing | Nursing/Integrated Concepts: Nursing Process: Planning/Health teaching and health promotion Learning Outcome: LO 26.3 Discuss the nursing management of a child with a structural defect of the genitourinary system. MNL LO: Renal and Genitourinary Disorders/Apply the nursing process in providing care for the child and family. 4) Which symptoms are characteristic of a preschool-age client who is diagnosed with a urinary tract infection? 1. Foul-smelling urine, elevated blood pressure, and hematuria 2. Severe flank pain, nausea, headache 3. Headache, hematuria, vertigo 4. Urgency, dysuria, fever Answer: 4 Explanation: 1. Clinical manifestations of a urinary tract infection (UTI) in a preschool-age client include fever, urgency, and dysuria. While hematuria may be present, there is no elevated blood pressure, headache, or vertigo. 2. Clinical manifestations of a urinary tract infection (UTI) in a preschool-age client include fever, urgency, and dysuria. While hematuria may be present, there is no elevated blood pressure, headache, or vertigo. 3. Clinical manifestations of a urinary tract infection (UTI) in a preschool-age client include fever, urgency, and dysuria. While hematuria may be present, there is no elevated blood pressure, headache, or vertigo. 4. Clinical manifestations of a urinary tract infection (UTI) in a preschool-age client include fever, urgency, and dysuria. While hematuria may be present, there is no elevated blood pressure, headache, or vertigo. Page Ref: 706 Cognitive Level: Analyzing Client Need &Sub: Physiological Integrity: Physiological Adaptation Standards: QSEN Competencies: Patient-centered care | AACN Essential Competencies: Essential IX: Baccalaureate generalist nursing practice | NLN Competencies: Human flourishing | Nursing/Integrated Concepts: Nursing Process: Assessment/Coordination of care Learning Outcome: LO 26.2 Develop a nursing care plan for the child with a urinary tract infection. MNL LO: Renal and Genitourinary Disorders/Apply the nursing process in providing care for the child and family. 5) A child is admitted to the hospital unit with a diagnosis of minimal-change nephrotic syndrome (MCNS). Which clinical manifestations does the nurse anticipate when conducting the admission assessment? 1. Hematuria, bacteriuria, weight gain 2. Gross hematuria, albuminuria, fever 3. Massive proteinuria, hypoalbuminemia, edema 4. Hypertension, weight loss, proteinuria Answer: 3 Explanation: 1. Nephrotic syndrome is an alteration in kidney function secondary to increased glomerular basement membrane permeability to plasma protein. It is characterized by massive proteinuria, hypoalbuminemia, and edema. While hematuria and hypertension may be present, they are not pronounced. Gross hematuria and hypertension are associated with glomerulonephritis. Bacteriuria and fever are associated with a urinary tract infection. Because of the edema, a weight gain, not a weight loss, would be seen. 2. Nephrotic syndrome is an alteration in kidney function secondary to increased glomerular basement membrane permeability to plasma protein. It is characterized by massive proteinuria, hypoalbuminemia, and edema. While hematuria and hypertension may be present, they are not pronounced. Gross hematuria and hypertension are associated with glomerulonephritis. Bacteriuria and fever are associated with a urinary tract infection. Because of the edema, a weight gain, not a weight loss, would be seen. 3. Nephrotic syndrome is an alteration in kidney function secondary to increased glomerular basement membrane permeability to plasma protein. It is characterized by massive proteinuria, hypoalbuminemia, and edema. While hematuria and hypertension may be present, they are not pronounced. Gross hematuria and hypertension are associated with glomerulonephritis. Bacteriuria and fever are associated with a urinary tract infection. Because of the edema, a weight gain, not a weight loss, would be seen. 4. Nephrotic syndrome is an alteration in kidney function secondary to increased glomerular basement membrane permeability to plasma protein. It is characterized by massive proteinuria, hypoalbuminemia, and edema. While hematuria and hypertension may be present, they are not pronounced. Gross hematuria and hypertension are associated with glomerulonephritis. Bacteriuria and fever are associated with a urinary tract infection. Because of the edema, a weight gain, not a weight loss, would be seen. Page Ref: 713-715 Cognitive Level: Analyzing Client Need &Sub: Physiological Integrity: Physiological Adaptation Standards: QSEN Competencies: Patient-centered care | AACN Essential Competencies: Essential IX: Baccalaureate generalist nursing practice | NLN Competencies: Nursing judgement | Nursing/Integrated Concepts: Nursing Process: Assessment/Coordination of care Learning Outcome: LO 26.4 Outline a plan to meet the fluid and dietary restrictions for the child with a renal disorder. MNL LO: Renal and Genitourinary Disorders/Apply the nursing process in providing care for the child and family. 6) A school-age client diagnosed with nephrotic syndrome is severely edematous. The primary healthcare provider has placed the child on bed rest. Which nursing intervention is a priority for this client? 1. Reposition the child every 2 hours. 2. Monitor BP every 30 minutes. 3. Encourage fluids. 4. Limit visitors. Answer: 1 Explanation: 1. A child with severe edema, on bed rest, is at risk for altered skin integrity. To prevent skin breakdown, the child should be repositioned every 2 hours. Vital signs are taken every 4 hours, fluids need to be monitored and should not be encouraged, and the child needs social interaction, so visitors should not be limited. 2. A child with severe edema, on bed rest, is at risk for altered skin integrity. To prevent skin breakdown, the child should be repositioned every 2 hours. Vital signs are taken every 4 hours, fluids need to be monitored and should not be encouraged, and the child needs social interaction, so visitors should not be limited. 3. A child with severe edema, on bed rest, is at risk for altered skin integrity. To prevent skin breakdown, the child should be repositioned every 2 hours. Vital signs are taken every 4 hours, fluids need to be monitored and should not be encouraged, and the child needs social interaction, so visitors should not be limited. 4. A child with severe edema, on bed rest, is at risk for altered skin integrity. To prevent skin breakdown, the child should be repositioned every 2 hours. Vital signs are taken every 4 hours, fluids need to be monitored and should not be encouraged, and the child needs social interaction, so visitors should not be limited. Page Ref: 715-716 Cognitive Level: Analyzing Client Need &Sub: Physiological Integrity: Reduction of Risk Potential Standards: QSEN Competencies: Teamwork and collaboration | AACN Essential Competencies: Essential VI: Interprofessional communication and collaboration for improving patient health outcomes | NLN Competencies: Nursing judgement | Nursing/Integrated Concepts: Nursing Process: Planning/Collaboration Learning Outcome: LO 26.6 Plan nursing care for the child with acute and chronic renal failure. MNL LO: Renal and Genitourinary Disorders/Apply the nursing process in providing care for the child and family. 7) A preschool-age client diagnosed with nephrotic syndrome is placed on prednisone for several weeks. Which teaching point is appropriate for the nurse to include in the teaching plan for this client? 1. Never stop the medication suddenly. 2. This drug is taken once a week on Sunday. 3. The child should always take the medication at night before bed. 4. This drug should be taken with meals. Answer: 1 Explanation: 1. Prednisone, a corticosteroid with anti-inflammatory action, is frequently used to treat nephrotic syndrome. It should never be stopped suddenly. The drug is taken more than once a week and can be taken any time during the day, but should remain on a constant schedule. Taking with food is always appropriate for most medications, but it does not have to be with a meal. 2. Prednisone, a corticosteroid with anti-inflammatory action, is frequently used to treat nephrotic syndrome. It should never be stopped suddenly. The drug is taken more than once a week and can be taken any time during the day, but should remain on a constant schedule. Taking with food is always appropriate for most medications, but it does not have to be with a meal. 3. Prednisone, a corticosteroid with anti-inflammatory action, is frequently used to treat nephrotic syndrome. It should never be stopped suddenly. The drug is taken more than once a week and can be taken any time during the day, but should remain on a constant schedule. Taking with food is always appropriate for most medications, but it does not have to be with a meal. 4. Prednisone, a corticosteroid with anti-inflammatory action, is frequently used to treat nephrotic syndrome. It should never be stopped suddenly. The drug is taken more than once a week and can be taken any time during the day, but should remain on a constant schedule. Taking with food is always appropriate for most medications, but it does not have to be with a meal. Page Ref: 715-716 Cognitive Level: Applying Client Need &Sub: Physiological Integrity: Pharmacological and Parenteral Therapies Standards: QSEN Competencies: Patient-centered care | AACN Essential Competencies: Essential VII: Clinical prevention and population health | NLN Competencies: Human flourishing | Nursing/Integrated Concepts: Nursing Process: Implementation/Health teaching and health promotion Learning Outcome: LO 26.6 Plan nursing care for the child with acute and chronic renal failure. MNL LO: Renal and Genitourinary Disorders/Apply the nursing process in providing care for the child and family. 8) A preschool-age client is diagnosed with acute glomerulonephritis and is admitted to the hospital. Which nursing diagnosis is most appropriate for this client? 1. Risk for Injury Related to Loss of Blood in Urine 2. Fluid-Volume Excess Related to Decreased Plasma Filtration 3. Risk for Infection Related to Hypertension 4. Altered Growth and Development Related to a Chronic Disease Answer: 2 Explanation: 1. The fluid is excessive, and fluid and electrolyte balance should be monitored. There is no risk for injury because the blood loss in the urine is not such that it causes anemia. While a risk for infection may be present, it is not related to the hypertension. Growth and development is not normally affected because this is an acute process, not a chronic one. 2. The fluid is excessive, and fluid and electrolyte balance should be monitored. There is no risk for injury because the blood loss in the urine is not such that it causes anemia. While a risk for infection may be present, it is not related to the hypertension. Growth and development is not normally affected because this is an acute process, not a chronic one. 3. The fluid is excessive, and fluid and electrolyte balance should be monitored. There is no risk for injury because the blood loss in the urine is not such that it causes anemia. While a risk for infection may be present, it is not related to the hypertension. Growth and development is not normally affected because this is an acute process, not a chronic one. 4. The fluid is excessive, and fluid and electrolyte balance should be monitored. There is no risk for injury because the blood loss in the urine is not such that it causes anemia. While a risk for infection may be present, it is not related to the hypertension. Growth and development is not normally affected because this is an acute process, not a chronic one. Page Ref: 718 Cognitive Level: Applying Client Need &Sub: Physiological Integrity: Reduction of Risk Potential Standards: QSEN Competencies: Patient-centered care | AACN Essential Competencies: Essential IX: Baccalaureate generalist nursing practice | NLN Competencies: Nursing judgement | Nursing/Integrated Concepts: Nursing Process: Diagnosis/Coordination of care Learning Outcome: LO 26.4 Outline a plan to meet the fluid and dietary restrictions for the child with a renal disorder. MNL LO: Renal and Genitourinary Disorders/Apply the nursing process in providing care for the child and family. 9) A child diagnosed with acute glomerulonephritis is in the playroom and experiences blurred vision and headache. Which action by the nurse is the most appropriate? 1. Check the urine to see if hematuria has increased. 2. Obtain a blood pressure on the child; notify the healthcare provider. 3. Reassure the child, and encourage bed rest until the headache improves. 4. Obtain serum electrolytes, and send a urinalysis to the lab. Answer: 2 Explanation: 1. Blurred vision and headache may be signs of encephalopathy, a complication of acute glomerulonephritis. A blood pressure (BP) should be obtained and the healthcare provider notified. The healthcare provider may decide to order an antihypertensive to bring down the BP. This is a serious complication, and delay in treatment could mean lethargy and seizures. Therefore, the other options (checking urine for hematuria, encouraging bed rest, and obtaining serum electrolytes) do not directly address the potential problem of encephalopathy. 2. Blurred vision and headache may be signs of encephalopathy, a complication of acute glomerulonephritis. A blood pressure (BP) should be obtained and the healthcare provider notified. The healthcare provider may decide to order an antihypertensive to bring down the BP. This is a serious complication, and delay in treatment could mean lethargy and seizures. Therefore, the other options (checking urine for hematuria, encouraging bed rest, and obtaining serum electrolytes) do not directly address the potential problem of encephalopathy. 3. Blurred vision and headache may be signs of encephalopathy, a complication of acute glomerulonephritis. A blood pressure (BP) should be obtained and the healthcare provider notified. The healthcare provider may decide to order an antihypertensive to bring down the BP. This is a serious complication, and delay in treatment could mean lethargy and seizures. Therefore, the other options (checking urine for hematuria, encouraging bed rest, and obtaining serum electrolytes) do not directly address the potential problem of encephalopathy. 4. Blurred vision and headache may be signs of encephalopathy, a complication of acute glomerulonephritis. A blood pressure (BP) should be obtained and the healthcare provider notified. The healthcare provider may decide to order an antihypertensive to bring down the BP. This is a serious complication, and delay in treatment could mean lethargy and seizures. Therefore, the other options (checking urine for hematuria, encouraging bed rest, and obtaining serum electrolytes) do not directly address the potential problem of encephalopathy. Page Ref: 718 Cognitive Level: Applying Client Need &Sub: Safe and Effective Care Environment: Safety and Infection Control Standards: QSEN Competencies: Teamwork and collaboration | AACN Essential Competencies: Essential VI: Interprofessional communication and collaboration for improving patient health outcomes | NLN Competencies: Nursing judgement | Nursing/Integrated Concepts: Nursing Process: Implementation/Collaboration Learning Outcome: LO 26.4 Outline a plan to meet the fluid and dietary restrictions for the child with a renal disorder. MNL LO: Renal and Genitourinary Disorders/Apply the nursing process in providing care for the child and family. 10) A child, in renal failure, is diagnosed with hyperkalemia. Which food choices will the nurse teach the parents and child to avoid? 1. Carrots and green, leafy vegetables 2. Chips, cold cuts, and canned foods 3. Spaghetti and meat sauce, breadsticks 4. Hamburger on a bun, cherry gelatin Answer: 1 Explanation: 1. Carrots and green, leafy vegetables are high in potassium. Chips, cold cuts, and canned foods are high in sodium but not necessarily potassium. Spaghetti and meat sauce with breadsticks and a hamburger on a bun with cherry gelatin would be acceptable choices for a low- potassium diet. 2. Carrots and green, leafy vegetables are high in potassium. Chips, cold cuts, and canned foods are high in sodium but not necessarily potassium. Spaghetti and meat sauce with breadsticks and a hamburger on a bun with cherry gelatin would be acceptable choices for a low-potassium diet. 3. Carrots and green, leafy vegetables are high in potassium. Chips, cold cuts, and canned foods are high in sodium but not necessarily potassium. Spaghetti and meat sauce with breadsticks and a hamburger on a bun with cherry gelatin would be acceptable choices for a low-potassium diet. 4. Carrots and green, leafy vegetables are high in potassium. Chips, cold cuts, and canned foods are high in sodium but not necessarily potassium. Spaghetti and meat sauce with breadsticks and a hamburger on a bun with cherry gelatin would be acceptable choices for a low-potassium diet. Page Ref: 720-722 Cognitive Level: Applying Client Need &Sub: Health Promotion and Maintenance Standards: QSEN Competencies: Patient-centered care | AACN Essential Competencies: Essential VII: Clinical prevention and population health | NLN Competencies: Human flourishing | Nursing/Integrated Concepts: Nursing Process: Planning/Health teaching and health promotion Learning Outcome: LO 26.6 Plan nursing care for the child with acute and chronic renal failure. MNL LO: Renal and Genitourinary Disorders/Apply the nursing process in providing care for the child and family. 11) A child recently had a kidney transplant and is prescribed cyclosporine. The parents ask the nurse about the reason for the cyclosporine. Which reason will the nurse include in the response for why this medication is prescribed? 1. To boost immunity 2. To suppress rejection 3. To decrease pain 4. To improve circulation Answer: 2 Explanation: 1. Cyclosporine is given to suppress rejection. It doesn't boost immunity, decrease pain, or improve circulation. 2. Cyclosporine is given to suppress rejection. It doesn't boost immunity, decrease pain, or improve circulation. 3. Cyclosporine is given to suppress rejection. It doesn't boost immunity, decrease pain, or improve circulation. 4. Cyclosporine is given to suppress rejection. It doesn't boost immunity, decrease pain, or improve circulation. Page Ref: 730 Cognitive Level: Applying Client Need &Sub: Physiological Integrity: Pharmacological and Parenteral Therapies Standards: QSEN Competencies: Patient-centered care | AACN Essential Competencies: Essential VII: Clinical prevention and population health | NLN Competencies: Human flourishing | Nursing/Integrated Concepts: Nursing Process: Planning/Health teaching and health promotion Learning Outcome: LO 26.7 Summarize psychosocial issues for the child requiring surgery on the genitourinary system. MNL LO: Renal and Genitourinary Disorders/Differentiate treatment options and strategies for continuity of care for the child and family. 12) A child is prescribed hemodialysis for the treatment of kidney failure. When providing care for this child, what will the nurse monitor for during the assessment? Select all that apply. 1. Shock 2. Hypotension 3. Infections 4. Migraines 5. Fluid overload Answer: 1, 2, 3 Explanation: 1. Rapid changes in fluid and electrolyte balance during hemodialysis may lead to shock and hypotension. Other complications to watch for are thromboses and infection. Migraines and fluid overload are not clinical manifestations associated with hemodialysis. 2. Rapid changes in fluid and electrolyte balance during hemodialysis may lead to shock and hypotension. Other complications to watch for are thromboses and infection. Migraines and fluid overload are not clinical manifestations associated with hemodialysis. 3. Rapid changes in fluid and electrolyte balance during hemodialysis may lead to shock and hypotension. Other complications to watch for are thromboses and infection. Migraines and fluid overload are not clinical manifestations associated with hemodialysis. 4. Rapid changes in fluid and electrolyte balance during hemodialysis may lead to shock and hypotension. Other complications to watch for are thromboses and infection. Migraines and fluid overload are not clinical manifestations associated with hemodialysis. 5. Rapid changes in fluid and electrolyte balance during hemodialysis may lead to shock and hypotension. Other complications to watch for are thromboses and infection. Migraines and fluid overload are not clinical manifestations associated with hemodialysis. Page Ref: 728-730 Cognitive Level: Applying Client Need &Sub: Physiological Integrity: Physiological Adaptation Standards: QSEN Competencies: Patient-centered care | AACN Essential Competencies: Essential IX: Baccalaureate generalist nursing practice | NLN Competencies: Human flourishing | Nursing/Integrated Concepts: Nursing Process: Assessment/Coordination of care Learning Outcome: LO 26.4 Outline a plan to meet the fluid and dietary restrictions for the child with a renal disorder. MNL LO: Renal and Genitourinary Disorders/Apply the nursing process in providing care for the child and family. 13) A child is scheduled for a kidney transplant. The nurse completes the preoperative teaching to prepare the child and parents for the surgery and postoperative considerations. Which statement by the parents indicates understanding of the teaching session? 1. "We know it's important to see that our child takes prescribed medications after the transplant." 2. "We'll be glad we won't have to bring our child in to see the doctor again." 3. "We're happy our child won't have to take any more medicine after the transplant." 4. "We understand our child won't be at risk anymore for catching colds from other children at school." Answer: 1 Explanation: 1. It is important that the nurse emphasize compliance with treatments that will need to be followed after the transplant. Follow-up appointments will be necessary, as well as medications and general health promotion. 2. It is important that the nurse emphasize compliance with treatments that will need to be followed after the transplant. Follow-up appointments will be necessary, as well as medications and general health promotion. 3. It is important that the nurse emphasize compliance with treatments that will need to be followed after the transplant. Follow-up appointments will be necessary, as well as medications and general health promotion. 4. It is important that the nurse emphasize compliance with treatments that will need to be followed after the transplant. Follow-up appointments will be necessary, as well as medications and general health promotion. Page Ref: 730 Cognitive Level: Analyzing Client Need &Sub: Health Promotion and Maintenance Standards: QSEN Competencies: Patient-centered care | AACN Essential Competencies: Essential VII: Clinical prevention and population health | NLN Competencies: Human flourishing | Nursing/Integrated Concepts: Nursing Process: Evaluation/Health teaching and health promotion Learning Outcome: LO 26.5 Identify growth and developmental issues for the child with chronic renal failure. MNL LO: Renal and Genitourinary Disorders/Differentiate treatment options and strategies for continuity of care for the child and family. 14) The nurse teaches parents that the anticholinergic drug oxybutynin is used to treat enuresis. The parents ask the nurse why the medication is being prescribed. Which response by the nurse is the most appropriate? 1. "It's an antidepressant that is used to help the child relax." 2. "It will help decrease the spasms sometimes associated with enuresis." 3. "It has an antidiuretic effect, so your child can attend sleepovers." 4. "It will slow the production of urine, so your child does not have to urinate as frequently." Answer: 2 Explanation: 1. Oxybutynin (Ditropan) is an anticholinergic that relaxes the smooth muscle of the bladder and decreases spasms. Oxybutynin is not an antidepressant or an antidiuretic, and does not slow urine production. 2. Oxybutynin (Ditropan) is an anticholinergic that relaxes the smooth muscle of the bladder and decreases spasms. Oxybutynin is not an antidepressant or an antidiuretic, and does not slow urine production. 3. Oxybutynin (Ditropan) is an anticholinergic that relaxes the smooth muscle of the bladder and decreases spasms. Oxybutynin is not an antidepressant or an antidiuretic, and does not slow urine production. 4. Oxybutynin (Ditropan) is an anticholinergic that relaxes the smooth muscle of the bladder and decreases spasms. Oxybutynin is not an antidepressant or an antidiuretic, and does not slow urine production. Page Ref: 712-713 Cognitive Level: Applying Client Need &Sub: Physiological Integrity Standards: QSEN Competencies: Patient-centered care | AACN Essential Competencies: Essential VII: Clinical prevention and population health | NLN Competencies: Human flourishing | Nursing/Integrated Concepts: Nursing Process: Implementation/Health teaching and health promotion Learning Outcome: LO 26.4 Outline a plan to meet the fluid and dietary restrictions for the child with a renal disorder. MNL LO: Renal and Genitourinary Disorders/Differentiate treatment options and strategies for continuity of care for the child and family. 15) The nurse educator is teaching a group of nursing students the pathophysiologic reasons related to genitourinary (GU) disorders in the pediatric population. Which statements are appropriate for the nurse educator to include in the teaching session? Select all that apply. 1. "Incomplete organ development during fetal development is the cause of many GU disorders." 2. "Improper placement of the urethra in vagina is one cause of GU disorders." 3. "GU disorders in the pediatric population can be caused by hydronephrosis." 4. "GU disorders in the pediatric population are not caused by infections." 5. "Anatomic obstruction or incomplete nerve innervation can cause GU disorders." Answer: 1, 3, 5 Explanation: 1. Pathophysiologic causes of GU disorders in the pediatric population include incomplete organ development during fetal development; hydronephrosis; and anatomic obstruction or incomplete nerve innervations. Improper placement of the urethra in the penis, not the vagina, is another pathophysiologic cause of GU disorders. GU disorders can also be caused by infection. 2. Pathophysiologic causes of GU disorders in the pediatric population include incomplete organ development during fetal development; hydronephrosis; and anatomic obstruction or incomplete nerve innervations. Improper placement of the urethra in the penis, not the vagina, is another pathophysiologic cause of GU disorders. GU disorders can also be caused by infection. 3. Pathophysiologic causes of GU disorders in the pediatric population include incomplete organ development during fetal development; hydronephrosis; and anatomic obstruction or incomplete nerve innervations. Improper placement of the urethra in the penis, not the vagina, is another pathophysiologic cause of GU disorders. GU disorders can also be caused by infection. 4. Pathophysiologic causes of GU disorders in the pediatric population include incomplete organ development during fetal development; hydronephrosis; and anatomic obstruction or incomplete nerve innervations. Improper placement of the urethra in the penis, not the vagina, is another pathophysiologic cause of GU disorders. GU disorders can also be caused by infection. 5. Pathophysiologic causes of GU disorders in the pediatric population include incomplete organ development during fetal development; hydronephrosis; and anatomic obstruction or incomplete nerve innervations. Improper placement of the urethra in the penis, not the vagina, is another pathophysiologic cause of GU disorders. GU disorders can also be caused by infection. Page Ref: 710-711 Cognitive Level: Applying Client Need &Sub: Physiological Integrity: Physiological Adaptation Standards: QSEN Competencies: Patient-centered care | AACN Essential Competencies: Essential VII: Clinical prevention and population health | NLN Competencies: Human flourishing | Nursing/Integrated Concepts: Nursing Process: Assessment/Health teaching and health promotion Learning Outcome: LO 26.1 Describe the pathophysiologic processes associated with genitourinary disorders in the pediatric population. MNL LO: Renal and Genitourinary Disorders/Examine etiology, risk factors, pathophysiology, and clinical manifestations as seen in children. 16) The nurse is preparing an educational session for sexually active adolescents. Which statements are appropriate for the nurse to include when educating about sexually transmitted infections (STIs)? Select all that apply. 1. "Frequently diagnosed STIs include chlamydia, genital herpes, gonorrhea, human papillomavirus, trichomoniasis, and syphilis." 2. "Your risk for contracting an STI can be decreased by using a condom when having sex." 3. "Birth control pills are useful in decreasing your risk of contracting an STI." 4. "Risk factors for pelvic inflammatory disease (PID) include multiple sexual partners, lack of barrier protection during intercourse, and history of an STI." 5. "Pelvic inflammatory disease (PID) is an infection of the lower genital tract." Answer: 1, 2, 4 Explanation: 1. It is appropriate for the nurse to include the frequently diagnosed STIs, the fact that the risk is decreased by using a condom, and the risk factors for pelvic inflammatory disease. Birth control pills are useful in decreasing the risk of pregnancy but are not useful for decreasing the risk of contracting an STI. PID is an infection of the upper genital tract, not the lower genital tract. 2. It is appropriate for the nurse to include the frequently diagnosed STIs, the fact that the risk is decreased by using a condom, and the risk factors for pelvic inflammatory disease. Birth control pills are useful in decreasing the risk of pregnancy but are not useful for decreasing the risk of contracting an STI. PID is an infection of the upper genital tract, not the lower genital tract. 3. It is appropriate for the nurse to include the frequently diagnosed STIs, the fact that the risk is decreased by using a condom, and the risk factors for pelvic inflammatory disease. Birth control pills are useful in decreasing the risk of pregnancy but are not useful for decreasing the risk of contracting an STI. PID is an infection of the upper genital tract, not the lower genital tract. 4. It is appropriate for the nurse to include the frequently diagnosed STIs, the fact that the risk is decreased by using a condom, and the risk factors for pelvic inflammatory disease. Birth control pills are useful in decreasing the risk of pregnancy but are not useful for decreasing the risk of contracting an STI. PID is an infection of the upper genital tract, not the lower genital tract. 5. It is appropriate for the nurse to include the frequently diagnosed STIs, the fact that the risk is decreased by using a condom, and the risk factors for pelvic inflammatory disease. Birth control pills are useful in decreasing the risk of pregnancy but are not useful for decreasing the risk of contracting an STI. PID is an infection of the upper genital tract, not the lower genital tract. Page Ref: 736 Cognitive Level: Applying Client Need &Sub: Physiological Integrity: Reduction of Risk Potential Standards: QSEN Competencies: Patient-centered care | AACN Essential Competencies: Essential VII: Clinical prevention and population health | NLN Competencies: Human flourishing | Nursing/Integrated Concepts: Nursing Process: Implementation/Health teaching and health promotion Learning Outcome: LO 26.1 Describe the pathophysiologic processes associated with genitourinary disorders in the pediatric population. MNL LO: Renal and Genitourinary Disorders/Educate the child and family on the care of the child during hospitalization and upon discharge. 17) A 10-year-old diagnosed with chronic renal failure is seen at the dialysis center for dialysis treatment three times a week. The child weighs 35 pounds after dialysis. Physician's order: Epogen 50 U/kg three times weekly after dialysis. Medication on hand: Epogen 2000 U/mL Calculate how many ml of Epogen the child should receive three times a week. Answer: 0.38 mL Explanation: 0.38 mL Page Ref: 723 Cognitive Level: Analyzing Client Need &Sub: Physiological Integrity: Pharmacological and Parenteral Therapies Standards: QSEN Competencies: Patient-centered care | AACN Essential Competencies: Essential IX: Baccalaureate generalist nursing practice | NLN Competencies: Human flourishing | Nursing/Integrated Concepts: Nursing Process: Planning/Coordination of care Learning Outcome: LO 26.6 Plan nursing care for the child with acute and chronic renal failure. MNL LO: Renal and Genitourinary Disorders/Differentiate treatment options and strategies for continuity of care for the child and family. 18) The child is diagnosed with an upper urinary tract infection (UTI). The family asks the nurse what is the difference in the symptoms of an upper versus a lower urinary tract infection? Match each symptom (1-8 below) with the appropriate infection. A. Lower UTI B. Upper UTI 1. High fever 2. Diarrhea 3. Chills 4. Hematuria 5. Costovertebral angle tenderness 6. Cloudy urine 7. Suprapubic or flank pain 8. Moderate/severe dehydration Answer: 1/B, 2/A, 3/B, 4/A, 5/B, 6/A, 7/A, 8/B 1. Upper UTI 2. Lower UTI 3. Upper UTI 4. Lower UTI 5. Upper UTI 6. Lower UTI 7. Lower UTI 8. Upper UTI Explanation: Lower UTI: Fever, diarrhea, vomiting, irritability, lethargy, foul smelling and cloudy urine, dehydration, abdominal pain, enuresis, suprapubic or flank pain, and urgency. Upper UTI: high fever, chills abdominal pain, nausea, vomiting, flak pain, costo-vertebral angle tenderness, moderate to severe dehydration. Page Ref: 706 Cognitive Level: Analyzing Client Need &Sub: Safe and Effective Care Environment: Management of Care Standards: QSEN Competencies: Patient-centered care | AACN Essential Competencies: Essential IX: Baccalaureate generalist nursing practice | NLN Competencies: Human flourishing | Nursing/Integrated Concepts: Nursing Process: Diagnosis/Coordination of care Learning Outcome: LO 26.1 Describe the pathophysiologic processes associated with genitourinary disorders in the pediatric population. MNL LO: Renal and Genitourinary Disorders/Examine etiology, risk factors, pathophysiology, and clinical manifestations as seen in children. Principles of Pediatric Nursing: Caring for Children, 7e (Ball et al.) Chapter 27 Alterations in Neurologic Function 1) A child is diagnosed with epilepsy and is prescribed daily phenytoin (Dilantin). Which topic is most appropriate for the nurse to include in the discharge teaching? 1. Increasing fluid intake 2. Performing good dental hygiene 3. Decreasing intake of vitamin D 4. Taking the medication with milk Answer: 2 Explanation: 1. Because phenytoin (Dilantin) can cause gingival hyperplasia, good dental hygiene should be encouraged. Fluid intake does not affect the drug's effectiveness, an adequate intake of vitamin D should be encouraged, and phenytoin (Dilantin) should not be taken with dairy products. 2. Because phenytoin (Dilantin) can cause gingival hyperplasia, good dental hygiene should be encouraged. Fluid intake does not affect the drug's effectiveness, an adequate intake of vitamin D should be encouraged, and phenytoin (Dilantin) should not be taken with dairy products. 3. Because phenytoin (Dilantin) can cause gingival hyperplasia, good dental hygiene should be encouraged. Fluid intake does not affect the drug's effectiveness, an adequate intake of vitamin D should be encouraged, and phenytoin (Dilantin) should not be taken with dairy products. 4. Because phenytoin (Dilantin) can cause gingival hyperplasia, good dental hygiene should be encouraged. Fluid intake does not affect the drug's effectiveness, an adequate intake of vitamin D should be encouraged, and phenytoin (Dilantin) should not be taken with dairy products. Page Ref: 751 Cognitive Level: Applying Client Need &Sub: Physiological Integrity: Pharmacological and Parenteral Therapies Standards: QSEN Competencies: Patient-centered care | AACN Essential Competencies: Essential VII: Clinical prevention and population health | NLN Competencies: Nursing judgement | Nursing/Integrated Concepts: Nursing Process: Planning/Health teaching and health promotion Learning Outcome: LO 27.3 Differentiate between the signs of a seizure and status epilepticus in infants and children, and describe appropriate nursing management for each condition. MNL LO: Neurological and Sensory Disorders/Examine etiology, risk factors, pathophysiology, and clinical manifestations as seen in children. 2) A toddler-age client has a tonic-clonic seizure while in a crib in the hospital. The client's jaw is clamped. Which nursing action is the priority? 1. Place a padded tongue blade between the child's jaws. 2. Stay with the child and observe the respiratory status. 3. Prepare the suction equipment. 4. Restrain the child to prevent injury. Answer: 2 Explanation: 1. During a seizure, the nurse remains with the child, watching for complications. The child's respiratory rate should be monitored. Be sure nothing is placed in the child's mouth during a seizure. Suction equipment should already be set up at the bedside before a seizure begins. The child should not be restrained during a seizure. 2. During a seizure, the nurse remains with the child, watching for complications. The child's respiratory rate should be monitored. Be sure nothing is placed in the child's mouth during a seizure. Suction equipment should already be set up at the bedside before a seizure begins. The child should not be restrained during a seizure. 3. During a seizure, the nurse remains with the child, watching for complications. The child's respiratory rate should be monitored. Be sure nothing is placed in the child's mouth during a seizure. Suction equipment should already be set up at the bedside before a seizure begins. The child should not be restrained during a seizure. 4. During a seizure, the nurse remains with the child, watching for complications. The child's respiratory rate should be monitored. Be sure nothing is placed in the child's mouth during a seizure. Suction equipment should already be set up at the bedside before a seizure begins. The child should not be restrained during a seizure. Page Ref: 753 Cognitive Level: Analyzing Client Need &Sub: Physiological Integrity: Reduction of Risk Potential Standards: QSEN Competencies: Patient-centered care | AACN Essential Competencies: Essential VII: Clinical prevention and population health | NLN Competencies: Nursing judgement | Nursing/Integrated Concepts: Nursing Process: Implementation/Coordination of care Learning Outcome: LO 27.3 Differentiate between the signs of a seizure and status epilepticus in infants and children, and describe appropriate nursing management for each condition. MNL LO: Neurological and Sensory Disorders/Examine etiology, risk factors, pathophysiology, and clinical manifestations as seen in children. 3) A lumbar puncture is performed on an infant suspected of having meningitis. Which finding does the nurse expect in the cerebral spinal fluid if the infant has meningitis? 1. Elevated WBC count 2. Elevated RBC count 3. Normal glucose 4. Decreased WBC count Answer: 1 Explanation: 1. The lumbar puncture is done to obtain cerebral spinal fluid (CSF). Elevated WBC count is seen with bacterial meningitis. The RBC count is not elevated, and the glucose is decreased in meningitis. 2. The lumbar puncture is done to obtain cerebral spinal fluid (CSF). Elevated WBC count is seen with bacterial meningitis. The RBC count is not elevated, and the glucose is decreased in meningitis. 3. The lumbar puncture is done to obtain cerebral spinal fluid (CSF). Elevated WBC count is seen with bacterial meningitis. The RBC count is not elevated, and the glucose is decreased in meningitis. 4. The lumbar puncture is done to obtain cerebral spinal fluid (CSF). Elevated WBC count is seen with bacterial meningitis. The RBC count is not elevated, and the glucose is decreased in meningitis. Page Ref: 756-757 Cognitive Level: Analyzing Client Need &Sub: Physiological Integrity: Physiological Adaptation Standards: QSEN Competencies: Patient-centered care | AACN Essential Competencies: Essential IX: Baccalaureate generalist nursing practice | NLN Competencies: Human flourishing | Nursing/Integrated Concepts: Nursing Process: Assessment/Coordination of care Learning Outcome: LO 27.4 Differentiate between signs of bacterial meningitis, viral meningitis, encephalitis, Reye syndrome, and Guillain-Barré syndrome in infants and children. MNL LO: Neurological and Sensory Disorders/Examine etiology, risk factors, pathophysiology, and clinical manifestations as seen in children. 4) The nurse is planning care for a school-age child diagnosed with bacterial meningitis. Which intervention is most appropriate? 1. Keeping environmental stimuli at a minimum 2. Avoiding giving pain medications that could dull sensorium 3. Measuring head circumference to assess developing complications 4. Having the child move the head from side to side at least every two hours Answer: 1 Explanation: 1. A quiet environment should be maintained because noise can disturb a child with meningitis. Pain medications are appropriate to give and should be used when needed. Measuring head circumference would only be appropriate for a child less than 2 years. Excessive head movement should be avoided because it can increase irritation of the meninges. 2. A quiet environment should be maintained because noise can disturb a child with meningitis. Pain medications are appropriate to give and should be used when needed. Measuring head circumference would only be appropriate for a child less than 2 years. Excessive head movement should be avoided because it can increase irritation of the meninges. 3. A quiet environment should be maintained because noise can disturb a child with meningitis. Pain medications are appropriate to give and should be used when needed. Measuring head circumference would only be appropriate for a child less than 2 years. Excessive head movement should be avoided because it can increase irritation of the meninges. 4. A quiet environment should be maintained because noise can disturb a child with meningitis. Pain medications are appropriate to give and should be used when needed. Measuring head circumference would only be appropriate for a child less than 2 years. Excessive head movement should be avoided because it can increase irritation of the meninges. Page Ref: 759 Cognitive Level: Applying Client Need &Sub: Physiological Integrity: Basic Care and Comfort Standards: QSEN Competencies: Patient-centered care | AACN Essential Competencies: Essential IX: Baccalaureate generalist nursing practice | NLN Competencies: Nursing judgement | Nursing/Integrated Concepts: Nursing Process: Planning/Coordination of care Learning Outcome: LO 27.4 Differentiate between signs of bacterial meningitis, viral meningitis, encephalitis, Reye syndrome, and Guillain-Barré syndrome in infants and children. MNL LO: Neurological and Sensory Disorders/Examine etiology, risk factors, pathophysiology, and clinical manifestations as seen in children. 5) A nurse is conducting a postoperative assessment on an infant who has just had a ventriculoperitoneal shunt placed for hydrocephalus. Which assessment finding would indicate a malfunction in the shunt? 1. Incisional pain 2. Movement of all extremities 3. Negative Brudzinski sign 4. Bulging fontanel Answer: 4 Explanation: 1. A bulging fontanel would be an abnormal finding and could indicate that the shunt is malfunctioning. Incisional pain, movement of all extremities, and negative Brudzinski sign are all normal findings after a ventriculoperitoneal shunt has been placed. 2. A bulging fontanel would be an abnormal finding and could indicate that the shunt is malfunctioning. Incisional pain, movement of all extremities, and negative Brudzinski sign are all normal findings after a ventriculoperitoneal shunt has been placed. 3. A bulging fontanel would be an abnormal finding and could indicate that the shunt is malfunctioning. Incisional pain, movement of all extremities, and negative Brudzinski sign are all normal findings after a ventriculoperitoneal shunt has been placed. 4. A bulging fontanel would be an abnormal finding and could indicate that the shunt is malfunctioning. Incisional pain, movement of all extremities, and negative Brudzinski sign are all normal findings after a ventriculoperitoneal shunt has been placed. Page Ref: 764-765 Cognitive Level: Analyzing Client Need &Sub: Physiological Integrity: Physiological Adaptation Standards: QSEN Competencies: Patient-centered care | AACN Essential Competencies: Essential IX: Baccalaureate generalist nursing practice | NLN Competencies: Human flourishing | Nursing/Integrated Concepts: Nursing Process: Assessment/Coordination of care Learning Outcome: LO 27.5 Develop a plan of nursing care for the child hospitalized with an acute neurologic condition. MNL LO: Neurological and Sensory Disorders/Apply the nursing process in providing care for the child and family. 6) Which nursing intervention is most appropriate when caring for an infant with a myelomeningocele in the preoperative stage? 1. Placing infant supine to decrease pressure on the sac 2. Appling a heat lamp to facilitate drying and toughening of the sac 3. Measuring head circumference every shift to identify developing hydrocephalus 4. Appling a diaper to prevent contamination of the sac Answer: 3 Explanation: 1. The infant should be monitored for developing hydrocephalus, so the head circumference should be monitored daily. The infant will be placed prone, not supine, and the defect will be protected from trauma or infection. Therefore, applying heat and a diaper around the defect would not be recommended. A sterile saline dressing may be used to cover the sac to maintain integrity. 2. The infant should be monitored for developing hydrocephalus, so the head circumference should be monitored daily. The infant will be placed prone, not supine, and the defect will be protected from trauma or infection. Therefore, applying heat and a diaper around the defect would not be recommended. A sterile saline dressing may be used to cover the sac to maintain integrity. 3. The infant should be monitored for developing hydrocephalus, so the head circumference should be monitored daily. The infant will be placed prone, not supine, and the defect will be protected from trauma or infection. Therefore, applying heat and a diaper around the defect would not be recommended. A sterile saline dressing may be used to cover the sac to maintain integrity. 4. The infant should be monitored for developing hydrocephalus, so the head circumference should be monitored daily. The infant will be placed prone, not supine, and the defect will be protected from trauma or infection. Therefore, applying heat and a diaper around the defect would not be recommended. A sterile saline dressing may be used to cover the sac to maintain integrity. Page Ref: 766 Cognitive Level: Applying Client Need &Sub: Physiological Integrity: Reduction of Risk Potential Standards: QSEN Competencies: Patient-centered care | AACN Essential Competencies: Essential IX: Baccalaureate generalist nursing practice | NLN Competencies: Human flourishing | Nursing/Integrated Concepts: Nursing Process: Implementation/Coordination of care Learning Outcome: LO 27.6 Develop a nursing care plan for the infant with hydrocephalus and spina bifida. MNL LO: Neurological and Sensory Disorders/Apply the nursing process in providing care for the child and family. 7) A child with myelomeningocele, corrected at birth, is now 5 years old. Which is the priority nursing diagnosis for a child with corrected spina bifida at this age? 1. Risk for Altered Nutrition 2. Risk for Impaired Tissue Perfusion–Cranial 3. Risk for Altered Urinary Elimination 4. Risk for Altered Comfort Answer: 3 Explanation: 1. A child with spina bifida will continue to have a risk for altered urinary elimination because the bowel and bladder sphincter controls are affected. Urinary retention is a problem, so bladder interventions are initiated early to prevent kidney damage. Risk for Altered Nutrition, Impaired Tissue Perfusion, and Altered Comfort are not problems once surgery has been performed to close the defect. 2. A child with spina bifida will continue to have a risk for altered urinary elimination because the bowel and bladder sphincter controls are affected. Urinary retention is a problem, so bladder interventions are initiated early to prevent kidney damage. Risk for Altered Nutrition, Impaired Tissue Perfusion, and Altered Comfort are not problems once surgery has been performed to close the defect. 3. A child with spina bifida will continue to have a risk for altered urinary elimination because the bowel and bladder sphincter controls are affected. Urinary retention is a problem, so bladder interventions are initiated early to prevent kidney damage. Risk for Altered Nutrition, Impaired Tissue Perfusion, and Altered Comfort are not problems once surgery has been performed to close the defect. 4. A child with spina bifida will continue to have a risk for altered urinary elimination because the bowel and bladder sphincter controls are affected. Urinary retention is a problem, so bladder interventions are initiated early to prevent kidney damage. Risk for Altered Nutrition, Impaired Tissue Perfusion, and Altered Comfort are not problems once surgery has been performed to close the defect. Page Ref: 769 Cognitive Level: Analyzing Client Need &Sub: Physiological Integrity: Physiological Adaptation Standards: QSEN Competencies: Patient-centered care | AACN Essential Competencies: Essential IX: Baccalaureate generalist nursing practice | NLN Competencies: Human flourishing | Nursing/Integrated Concepts: Nursing Process: Diagnosis/Coordination of care Learning Outcome: LO 27.6 Develop a nursing care plan for the infant with hydrocephalus and spina bifida. MNL LO: Neurological and Sensory Disorders/Apply the nursing process in providing care for the child and family. 8) Which statement made by a parent during a well-child visit would cause the nurse to suspect the child has cerebral palsy? 1. "My 6-month-old baby is rolling from back to prone now." 2. "My 3-month-old seems to have floppy muscle tone." 3. "My 8-month-old can sit without support." 4. "My 10-month-old is not walking." Answer: 2 Explanation: 1. Children with cerebral palsy are delayed in meeting developmental milestones. The infant with hypotonia is showing a clinical manifestation of cerebral palsy. A baby rolls over from back to prone at 6 months, sits without support at 8 months, and walks at 12 months. 2. Children with cerebral palsy are delayed in meeting developmental milestones. The infant with hypotonia is showing a clinical manifestation of cerebral palsy. A baby rolls over from back to prone at 6 months, sits without support at 8 months, and walks at 12 months. 3. Children with cerebral palsy are delayed in meeting developmental milestones. The infant with hypotonia is showing a clinical manifestation of cerebral palsy. A baby rolls over from back to prone at 6 months, sits without support at 8 months, and walks at 12 months. 4. Children with cerebral palsy are delayed in meeting developmental milestones. The infant with hypotonia is showing a clinical manifestation of cerebral palsy. A baby rolls over from back to prone at 6 months, sits without support at 8 months, and walks at 12 months. Page Ref: 774 Cognitive Level: Analyzing Client Need &Sub: Physiological Integrity: Physiological Adaptation Standards: QSEN Competencies: Patient-centered care | AACN Essential Competencies: Essential IX: Baccalaureate generalist nursing practice | NLN Competencies: Nursing judgement | Nursing/Integrated Concepts: Nursing Process: Evaluation/Coordination of care Learning Outcome: LO 27.7 Plan family-centered nursing care for the child with cerebral palsy in a community setting. MNL LO: Neurological and Sensory Disorders/Apply the nursing process in providing care for the child and family. 9) A nurse is caring for a child who is diagnosed with cerebral palsy. Which goal of therapy is most appropriate for the nurse to include in the plan of care? 1. Reversing the degenerative processes that have occurred 2. Curing the underlying defect causing the disorder 3. Preventing the spread to individuals in close contact with the child 4. Promoting optimum development Answer: 4 Explanation: 1. Recognition of the disorder is important so that optimal development can be maintained. Cerebral palsy cannot be reversed or cured. It is not caused by a contagious process, so there is no risk of spread. 2. Recognition of the disorder is important so that optimal development can be maintained. Cerebral palsy cannot be reversed or cured. It is not caused by a contagious process, so there is no risk of spread. 3. Recognition of the disorder is important so that optimal development can be maintained. Cerebral palsy cannot be reversed or cured. It is not caused by a contagious process, so there is no risk of spread. 4. Recognition of the disorder is important so that optimal development can be maintained. Cerebral palsy cannot be reversed or cured. It is not caused by a contagious process, so there is no risk of spread. Page Ref: 775 Cognitive Level: Analyzing Client Need &Sub: Safe and Effective Care Environment: Management of Care Standards: QSEN Competencies: Patient-centered care | AACN Essential Competencies: Essential IX: Baccalaureate generalist nursing practice | NLN Competencies: Human flourishing | Nursing/Integrated Concepts: Nursing Process: Planning/Coordination of care Learning Outcome: LO 27.7 Plan family-centered nursing care for the child with cerebral palsy in a community setting. MNL LO: Neurological and Sensory Disorders/Apply the nursing process in providing care for the child and family. 10) A child sustains a traumatic brain injury and is monitored in the pediatric intensive-care unit (PICU). The nurse is using the Glasgow Coma Scale to assess the child. Which items will the nurse assess when using this tool? Select all that apply. 1. Eye opening 2. Verbal response 3. Motor response 4. Head circumference 5. Pulse oximetry Answer: 1, 2, 3 Explanation: 1. The Glasgow Coma Scale for infants and children scores parameters related to eye opening, verbal response, and motor response. The maximum score is 15, indicating the highest level of neurological functioning. Head circumference and pulse oximetry are not included on the scale. 2. The Glasgow Coma Scale for infants and children scores parameters related to eye opening, verbal response, and motor response. The maximum score is 15, indicating the highest level of neurological functioning. Head circumference and pulse oximetry are not included on the scale. 3. The Glasgow Coma Scale for infants and children scores parameters related to eye opening, verbal response, and motor response. The maximum score is 15, indicating the highest level of neurological functioning. Head circumference and pulse oximetry are not included on the scale. 4. The Glasgow Coma Scale for infants and children scores parameters related to eye opening, verbal response, and motor response. The maximum score is 15, indicating the highest level of neurological functioning. Head circumference and pulse oximetry are not included on the scale. 5. The Glasgow Coma Scale for infants and children scores parameters related to eye opening, verbal response, and motor response. The maximum score is 15, indicating the highest level of neurological functioning. Head circumference and pulse oximetry are not included on the scale. Page Ref: 746 Cognitive Level: Analyzing Client Need &Sub: Physiological Integrity: Reduction of Risk Potential Standards: QSEN Competencies: Patient-centered care | AACN Essential Competencies: Essential IX: Baccalaureate generalist nursing practice | NLN Competencies: Human flourishing | Nursing/Integrated Concepts: Nursing Process: Assessment/Coordination of care Learning Outcome: LO 27.2 Choose the appropriate assessment guidelines and tools to examine infants and children with altered levels of consciousness and other neurologic conditions. MNL LO: Neurological and Sensory Disorders/Apply the nursing process in providing care for the child and family. 11) A child diagnosed with a mild traumatic brain injury is being sedated with a mild sedative so that pain and anxiety are minimized. Which nursing interventions are appropriate for this child? Select all that apply. 1. Place a continuous-pulse oximetry monitor on the child. 2. Place the child in a room near the nurse's station. 3. Allow for several visitors to remain at the child's bedside. 4. Use soft restraints if the child becomes confused. 5. Use sedation around the clock to decrease agitation. Answer: 1, 2 Explanation: 1. When a child is sedated, respiratory status should be monitored with a pulse- oximetry machine. The child should be close to the nurse's station so that frequent monitoring can be done. Several visitors at the bedside would increase the child's anxiety. Soft restraints may increase agitation. Sedation around the clock is not recommended due to the need to evaluate the neurologic system. 2. When a child is sedated, respiratory status should be monitored with a pulse-oximetry machine. The child should be close to the nurse's station so that frequent monitoring can be done. Several visitors at the bedside would increase the child's anxiety. Soft restraints may increase agitation. Sedation around the clock is not recommended due to the need to evaluate the neurologic system. 3. When a child is sedated, respiratory status should be monitored with a pulse-oximetry machine. The child should be close to the nurse's station so that frequent monitoring can be done. Several visitors at the bedside would increase the child's anxiety. Soft restraints may increase agitation. Sedation around the clock is not recommended due to the need to evaluate the neurologic system. 4. When a child is sedated, respiratory status should be monitored with a pulse-oximetry machine. The child should be close to the nurse's station so that frequent monitoring can be done. Several visitors at the bedside would increase the child's anxiety. Soft restraints may increase agitation. Sedation around the clock is not recommended due to the need to evaluate the neurologic system. 5. When a child is sedated, respiratory status should be monitored with a pulse-oximetry machine. The child should be close to the nurse's station so that frequent monitoring can be done. Several visitors at the bedside would increase the child's anxiety. Soft restraints may increase agitation. Sedation around the clock is not recommended due to the need to evaluate the neurologic system. Page Ref: 747 Cognitive Level: Applying Client Need &Sub: Safe and Effective Care Environment: Safety and Infection Control Standards: QSEN Competencies: Patient-centered care | AACN Essential Competencies: Essential IX: Baccalaureate generalist nursing practice | NLN Competencies: Nursing judgement | Nursing/Integrated Concepts: Nursing Process: Implementation/Coordination of care Learning Outcome: LO 27.8 Contrast the appropriate initial nursing management for mild versus severe traumatic brain injury. MNL LO: Neurological and Sensory Disorders/Apply the nursing process in providing care for the child and family. 12) A school-age client sustains a basilar skull fracture. Which symptom is a priority for this nurse to assess for when providing care to this client? 1. Cerebral spinal fluid leakage from the nose or ears 2. Headache 3. Transient confusion 4. Periorbital ecchymosis Answer: 1 Explanation: 1. Cerebral spinal fluid leakage could be present from the nose or ears and, if it persists, may indicate that surgical repair will be needed. Headache, transient confusion, and periorbital ecchymosis are findings that commonly present with a basilar skull fracture but do not indicate that surgical repair will be needed. 2. Cerebral spinal fluid leakage could be present from the nose or ears and, if it persists, may indicate that surgical repair will be needed. Headache, transient confusion, and periorbital ecchymosis are findings that commonly present with a basilar skull fracture but do not indicate that surgical repair will be needed. 3. Cerebral spinal fluid leakage could be present from the nose or ears and, if it persists, may indicate that surgical repair will be needed. Headache, transient confusion, and periorbital ecchymosis are findings that commonly present with a basilar skull fracture but do not indicate that surgical repair will be needed. 4. Cerebral spinal fluid leakage could be present from the nose or ears and, if it persists, may indicate that surgical repair will be needed. Headache, transient confusion, and periorbital ecchymosis are findings that commonly present with a basilar skull fracture but do not indicate that surgical repair will be needed. Page Ref: 783 Cognitive Level: Analyzing Client Need &Sub: Physiological Integrity: Physiological Adaptation Standards: QSEN Competencies: Patient-centered care | AACN Essential Competencies: Essential IX: Baccalaureate generalist nursing practice | NLN Competencies: Nursing judgement | Nursing/Integrated Concepts: Nursing Process: Assessment/Coordination of care Learning Outcome: LO 27.8 Contrast the appropriate initial nursing management for mild versus severe traumatic brain injury. MNL LO: Neurological and Sensory Disorders/Apply the nursing process in providing care for the child and family. 13) A school-age client experiences a near-drowning episode and is admitted to the pediatric intensive-care unit (PICU). The parents express guilt over the near drowning of their child. Which response by the nurse is most appropriate? 1. "You will need to watch the child more closely." 2. "Tell me more about your feelings related to the accident." 3. "The child will be fine, so don't worry." 4. "Why did you let the child almost drown?" Answer: 2 Explanation: 1. In near-drowning cases, the nurse should be nonjudgmental and provide a forum for parents to express guilt. Telling the parents to watch the child more closely or asking them why they let the child almost drown is judgmental. Saying the child will be fine may not be true. The nurse should reassure the parents that the child is receiving all possible medical treatment. 2. In near-drowning cases, the nurse should be nonjudgmental and provide a forum for parents to express guilt. Telling the parents to watch the child more closely or asking them why they let the child almost drown is judgmental. Saying the child will be fine may not be true. The nurse should reassure the parents that the child is receiving all possible medical treatment. 3. In near-drowning cases, the nurse should be nonjudgmental and provide a forum for parents to express guilt. Telling the parents to watch the child more closely or asking them why they let the child almost drown is judgmental. Saying the child will be fine may not be true. The nurse should reassure the parents that the child is receiving all possible medical treatment. 4. In near-drowning cases, the nurse should be nonjudgmental and provide a forum for parents to express guilt. Telling the parents to watch the child more closely or asking them why they let the child almost drown is judgmental. Saying the child will be fine may not be true. The nurse should reassure the parents that the child is receiving all possible medical treatment. Page Ref: 787 Cognitive Level: Applying Client Need &Sub: Psychosocial Integrity Standards: QSEN Competencies: Patient-centered care | AACN Essential Competencies: Essential II: Basic organizational and systems leadership for quality care and patient safety | NLN Competencies: Nursing judgement | Nursing/Integrated Concepts: Nursing Process: Implementation/Communication Learning Outcome: LO 27.9 Discuss initiatives to prevent drowning in children. MNL LO: Neurological and Sensory Disorders/Differentiate treatment options and strategies for continuity of care for the child and family. 14) With a group of new parents, the nurse is reviewing treatment for viral illnesses such as influenza. Which statement by the parents indicates appropriate understanding of the teaching session? 1. "Some over-the-counter medications contain aspirin." 2. "Acetaminophen is good for treatment of fevers in young children." 3. "I can use ibuprofen as needed when my child has aches and pains." 4. "Aspirin is acceptable if my child does not have a virus." Answer: 1 Explanation: 1. Reye syndrome is a serious consequence of aspirin use in children with viral illnesses. Over-the-counter medications should be checked to see whether they contain aspirin before being used. Aspirin is avoided in children. Ibuprofen and acetaminophen are acceptable to use in children. 2. Reye syndrome is a serious consequence of aspirin use in children with viral illnesses. Over- the-counter medications should be checked to see whether they contain aspirin before being used. Aspirin is avoided in children. Ibuprofen and acetaminophen are acceptable to use in children. 3. Reye syndrome is a serious consequence of aspirin use in children with viral illnesses. Over- the-counter medications should be checked to see whether they contain aspirin before being used. Aspirin is avoided in children. Ibuprofen and acetaminophen are acceptable to use in children. 4. Reye syndrome is a serious consequence of aspirin use in children with viral illnesses. Over- the-counter medications should be checked to see whether they contain aspirin before being used. Aspirin is avoided in children. Ibuprofen and acetaminophen are acceptable to use in children. Page Ref: 760-761 Cognitive Level: Applying Client Need &Sub: Physiological Integrity: Reduction of Risk Potential Standards: QSEN Competencies: Patient-centered care | AACN Essential Competencies: Essential VII: Clinical prevention and population health | NLN Competencies: Human flourishing | Nursing/Integrated Concepts: Nursing Process: Evaluation/Health teaching and health promotion Learning Outcome: LO 27.4 Differentiate between signs of bacterial meningitis, viral meningitis, encephalitis, Reye syndrome, and Guillain-Barré syndrome in infants and children. MNL LO: Neurological and Sensory Disorders/Examine etiology, risk factors, pathophysiology, and clinical manifestations as seen in children. 15) A school-age client is transported to the emergency department by ambulance from the scene of a car accident. The client is alert and oriented × 3; pulse, respirations, and blood pressure are stable; and the neck and back are immobilized on a backboard. The nurse sees no obvious bleeding. The client states, "I can't feel or move my legs." Which injury does the nurse suspect? 1. Traumatic brain injury 2. Ruptured spleen 3. Traumatic shock 4. Spinal cord injury Answer: 4 Explanation: 1. Spinal cord injury results in paralysis and anesthesia of the affected body parts below the level of the lesion. Altered levels of consciousness may indicate traumatic brain injury. The child may have a ruptured spleen, but it is not evident from the data given. Traumatic shock results in initially increasing then decreasing pulse and respirations, and falling blood pressure. 2. Spinal cord injury results in paralysis and anesthesia of the affected body parts below the level of the lesion. Altered levels of consciousness may indicate traumatic brain injury. The child may have a ruptured spleen, but it is not evident from the data given. Traumatic shock results in initially increasing then decreasing pulse and respirations, and falling blood pressure. 3. Spinal cord injury results in paralysis and anesthesia of the affected body parts below the level of the lesion. Altered levels of consciousness may indicate traumatic brain injury. The child may have a ruptured spleen, but it is not evident from the data given. Traumatic shock results in initially increasing then decreasing pulse and respirations, and falling blood pressure. 4. Spinal cord injury results in paralysis and anesthesia of the affected body parts below the level of the lesion. Altered levels of consciousness may indicate traumatic brain injury. The child may have a ruptured spleen, but it is not evident from the data given. Traumatic shock results in initially increasing then decreasing pulse and respirations, and falling blood pressure. Page Ref: 785 Cognitive Level: Analyzing Client Need &Sub: Physiological Integrity: Physiological Adaptation Standards: QSEN Competencies: Patient-centered care | AACN Essential Competencies: Essential IX: Baccalaureate generalist nursing practice | NLN Competencies: Human flourishing | Nursing/Integrated Concepts: Nursing Process: Assessment/Coordination of care Learning Outcome: LO 27.8 Contrast the appropriate initial nursing management for mild versus severe traumatic brain injury. MNL LO: Neurological and Sensory Disorders/Apply the nursing process in providing care for the child and family. 16) A child is ready for discharge after surgery for a myelomeningocele repair. Before discharge, the nurse works with the parents to establish a catheterization schedule to prevent urinary tract infection. With what frequency should the nurse instruct the parents to catheterize the child? 1. Every 1 to 2 hours 2. Every 3 to 4 hours 3. Every 6 to 8 hours 4. Every 10 to 12 hours Answer: 2 Explanation: 1. To decrease the incidence of bladder or urinary tract infections, catheterization should occur every 3 to 4 hours. 2. To decrease the incidence of bladder or urinary tract infections, catheterization should occur every 3 to 4 hours. 3. To decrease the incidence of bladder or urinary tract infections, catheterization should occur every 3 to 4 hours. 4. To decrease the incidence of bladder or urinary tract infections, catheterization should occur every 3 to 4 hours. Page Ref: 768 Cognitive Level: Applying Client Need &Sub: Safe and Effective Care Environment: Safety and Infection Control Standards: QSEN Competencies: Patient-centered care | AACN Essential Competencies: Essential VII: Clinical prevention and population health | NLN Competencies: Human flourishing | Nursing/Integrated Concepts: Nursing Process: Implementation/Health teaching and health promotion Learning Outcome: LO 27.6 Develop a nursing care plan for the infant with hydrocephalus and spina bifida. MNL LO: Neurological and Sensory Disorders/Educate the child and family on the care of the child during hospitalization and upon discharge. 17) The nurse educator is describing the pediatric differences associated with the anatomy and physiology of the neurologic system to a group of nursing students. Which statements made by the class indicate appropriate understanding of this topic after the teaching session? Select all that apply. 1. The bones of the cranium are connected by connective tissue to allow for brain growth. 2. The spine of infants is excessively mobile due to immature neck muscles and incompletely developed vertebral bodies. 3. Maturation of the nerves continues until age 10. 4. Myelination is complete at birth, 5. Myelination proceeds in a cephalocaudal direction. Answer: 1, 2, 5 Explanation: 1. There are several pediatric differences associated with the anatomy and physiology of the neurological system and include: the bones of the cranium are connected by connective tissue to allow for brain growth; the spine of infants is excessively mobile due to immature neck muscles and incompletely developed vertebral bodies; and myelination proceeds in a cephalocaudal direction. Maturation of the nerves continues until the age of 4, not 10. Myelination is incomplete at birth. 2. There are several pediatric differences associated with the anatomy and physiology of the neurological system and include: the bones of the cranium are connected by connective tissue to allow for brain growth; the spine of infants is excessively mobile due to immature neck muscles and incompletely developed vertebral bodies; and myelination proceeds in a cephalocaudal direction. Maturation of the nerves continues until the age of 4, not 10. Myelination is incomplete at birth. 3. There are several pediatric differences associated with the anatomy and physiology of the neurological system and include: the bones of the cranium are connected by connective tissue to allow for brain growth; the spine of infants is excessively mobile due to immature neck muscles and incompletely developed vertebral bodies; and myelination proceeds in a cephalocaudal direction. Maturation of the nerves continues until the age of 4, not 10. Myelination is incomplete at birth. 4. There are several pediatric differences associated with the anatomy and physiology of the neurological system and include: the bones of the cranium are connected by connective tissue to allow for brain growth; the spine of infants is excessively mobile due to immature neck muscles and incompletely developed vertebral bodies; and myelination proceeds in a cephalocaudal direction. Maturation of the nerves continues until the age of 4, not 10. Myelination is incomplete at birth. 5. There are several pediatric differences associated with the anatomy and physiology of the neurological system and include: the bones of the cranium are connected by connective tissue to allow for brain growth; the spine of infants is excessively mobile due to immature neck muscles and incompletely developed vertebral bodies; and myelination proceeds in a cephalocaudal direction. Maturation of the nerves continues until the age of 4, not 10. Myelination is incomplete at birth. Page Ref: 743 Cognitive Level: Understanding Client Need &Sub: Physiological Integrity: Physiological Adaptation Standards: QSEN Competencies: Patient-centered care | AACN Essential Competencies: Essential IX: Baccalaureate generalist nursing practice | NLN Competencies: Human flourishing | Nursing/Integrated Concepts: Nursing Process: Evaluation/Education Learning Outcome: LO 27.1 Describe the pediatric differences associated with the anatomy and physiology of the neurologic system. MNL LO: Neurological and Sensory Disorders/Examine etiology, risk factors, pathophysiology, and clinical manifestations as seen in children. 18) A 5-year-old child is admitted to the hospital with increased intracranial pressure after a motor vehicle struck the child. The child weighs 15 kg. The neurosurgeon orders: Mannitol 0.5 g/kg/10 minutes IV first, followed by Mannitol 0.25 g/kg IV every 4 hours. Medication on hand: Mannitol 100 g/500mL D5W. Calculate how many mL/hr to set the IV pump to infuse the Mannitol ordered every 4 hours. Answer: 18.75 or 18.8 mL/hr Explanation: 18.75 or 18.8 mL/hr Page Ref: 781 Cognitive Level: Analyzing Client Need &Sub: Physiological Integrity: Pharmacological and Parenteral Therapies Standards: QSEN Competencies: Patient-centered care | AACN Essential Competencies: Essential IX: Baccalaureate generalist nursing practice | NLN Competencies: Human flourishing | Nursing/Integrated Concepts: Nursing Process: Planning/Coordination of care Learning Outcome: LO 27.5 Develop a plan of nursing care for the child hospitalized with an acute neurologic condition. MNL LO: Neurological and Sensory Disorders/Differentiate treatment options and strategies for continuity of care for the child and family. 19) The nurse is performing an admission assessment on an infant diagnosed with hydrocephalus and a malfunctioning shunt. Which assessment findings should the nurse expect? Select all that apply. 1. Vomiting 2. Fever 3. Irritability 4. Poor appetite 5. Decreased level of consciousness Answer: 1, 2, 3, 4 Explanation: 1. Signs of shunt malfunction in infants are nonspecific and include irritability, vomiting, poor appetite, disordered sleep, and fever. Older children with shunt malfunction may have a headache, nausea, vomiting, and decreased level of consciousness. 2. Signs of shunt malfunction in infants are nonspecific and include irritability, vomiting, poor appetite, disordered sleep, and fever. Older children with shunt malfunction may have a headache, nausea, vomiting, and decreased level of consciousness. 3. Signs of shunt malfunction in infants are nonspecific and include irritability, vomiting, poor appetite, disordered sleep, and fever. Older children with shunt malfunction may have a headache, nausea, vomiting, and decreased level of consciousness. 4. Signs of shunt malfunction in infants are nonspecific and include irritability, vomiting, poor appetite, disordered sleep, and fever. Older children with shunt malfunction may have a headache, nausea, vomiting, and decreased level of consciousness. 5. Signs of shunt malfunction in infants are nonspecific and include irritability, vomiting, poor appetite, disordered sleep, and fever. Older children with shunt malfunction may have a headache, nausea, vomiting, and decreased level of consciousness. Page Ref: 764 Cognitive Level: Analyzing Client Need &Sub: Health Promotion and Maintenance Standards: QSEN Competencies: Patient-centered care | AACN Essential Competencies: Essential VII: Clinical prevention and population health | NLN Competencies: Human flourishing | Nursing/Integrated Concepts: Nursing Process: Intervention/Health teaching and health promotion Learning Outcome: LO 27.2 Choose the appropriate assessment guidelines and tools to examine infants and children with altered levels of consciousness and other neurologic conditions. MNL LO: Neurological and Sensory Disorders/Examine etiology, risk factors, pathophysiology, and clinical manifestations as seen in children. Principles of Pediatric Nursing: Caring for Children, 7e (Ball et al.) Chapter 28 Alterations in Mental Health and Cognitive Function 1) A nurse is caring for four pediatric clients in the hospital. Which client should the nurse refer for play therapy? 1. An adolescent with asthma 2. A preschool-age child with a fractured femur 3. A school-age child having an appendectomy 4. An infant with sepsis Answer: 2 Explanation: 1. Play therapy is often used with preschool and school-age children who are experiencing anxiety, stress, and other specific nonpsychotic mental disorders. In this case, the child who experiences a condition that requires longer hospitalization and recovery, such as a fracture of the femur, should be referred for play therapy. The adolescent with asthma, the school-age child having an appendectomy, and the infant with sepsis do not have as high a need for play therapy as the preschool child with a broken bone. 2. Play therapy is often used with preschool and school-age children who are experiencing anxiety, stress, and other specific nonpsychotic mental disorders. In this case, the child who experiences a condition that requires longer hospitalization and recovery, such as a fracture of the femur, should be referred for play therapy. The adolescent with asthma, the school-age child having an appendectomy, and the infant with sepsis do not have as high a need for play therapy as the preschool child with a broken bone. 3. Play therapy is often used with preschool and school-age children who are experiencing anxiety, stress, and other specific nonpsychotic mental disorders. In this case, the child who experiences a condition that requires longer hospitalization and recovery, such as a fracture of the femur, should be referred for play therapy. The adolescent with asthma, the school-age child having an appendectomy, and the infant with sepsis do not have as high a need for play therapy as the preschool child with a broken bone. 4. Play therapy is often used with preschool and school-age children who are experiencing anxiety, stress, and other specific nonpsychotic mental disorders. In this case, the child who experiences a condition that requires longer hospitalization and recovery, such as a fracture of the femur, should be referred for play therapy. The adolescent with asthma, the school-age child having an appendectomy, and the infant with sepsis do not have as high a need for play therapy as the preschool child with a broken bone. Page Ref: 794 Cognitive Level: Applying Client Need &Sub: Psychosocial Integrity Standards: QSEN Competencies: Teamwork and collaboration | AACN Essential Competencies: Essential VI: Interprofessional communication and collaboration for improving patient health outcomes | NLN Competencies: Nursing judgement | Nursing/Integrated Concepts: Nursing Process: Implementation/Collaboration Learning Outcome: LO 28.3 Plan for the nursing management of children and adolescents with mental health alterations in the hospital and community settings. MNL LO: Mental Health and Cognition/Apply the nursing process in providing care for the child and family. 2) A school-age client diagnosed with autism is admitted to the hospital because of recent vomiting and diarrhea. Which intervention by the nurse is most appropriate upon admission? 1. Take the child on a quick tour of the whole unit. 2. Take the child to the playroom immediately for arts and crafts. 3. Orient the child to the hospital room with minimal distractions. 4. Admit the child to a four-bed unit with small children. Answer: 3 Explanation: 1. Autistic children interpret and respond to the environment differently from other individuals. The child needs to be oriented to new settings and adjusts best to a quiet, controlled environment. A hospital room with only one other child is best. 2. Autistic children interpret and respond to the environment differently from other individuals. The child needs to be oriented to new settings and adjusts best to a quiet, controlled environment. A hospital room with only one other child is best. 3. Autistic children interpret and respond to the environment differently from other individuals. The child needs to be oriented to new settings and adjusts best to a quiet, controlled environment. A hospital room with only one other child is best. 4. Autistic children interpret and respond to the environment differently from other individuals. The child needs to be oriented to new settings and adjusts best to a quiet, controlled environment. A hospital room with only one other child is best. Page Ref: 800 Cognitive Level: Applying Client Need &Sub: Psychosocial Integrity Standards: QSEN Competencies: Patient-centered care | AACN Essential Competencies: Essential IX: Baccalaureate generalist nursing practice | NLN Competencies: Nursing judgement | Nursing/Integrated Concepts: Nursing Process: Planning/Coordination of care Learning Outcome: LO 28.5 Use evidence-based practice to plan nursing management for children with cognitive alterations. MNL LO: Mental Health and Cognition/Develop appropriate coping strategies for the child and family to deal with stress. 3) A nurse is planning preoperative teaching for a school-age client scheduled to have a tonsillectomy. The client has a history of attention deficit hyperactivity disorder (ADHD). Which intervention will the nurse include in the plan of care? 1. Give instructions verbally and use a picture pamphlet, repeating points more than once. 2. Ask other children who have had this procedure to talk to the child. 3. Allow the child to lead the session to gain a sense of control. 4. Play a television show in the background. Answer: 1 Explanation: 1. A teaching session for a child with ADHD should foster attention. Giving instructions verbally and in written form, repeating points, will improve learning for a child with ADHD. The environment needs to be quiet, with minimal distractions. A child who has difficulty concentrating should not lead the session even though the child needs to feel in control. Talking to other children who have had this procedure may not foster understanding, because this child has ADHD. Distractions such as noise from a television should be minimized. 2. A teaching session for a child with ADHD should foster attention. Giving instructions verbally and in written form, repeating points, will improve learning for a child with ADHD. The environment needs to be quiet, with minimal distractions. A child who has difficulty concentrating should not lead the session even though the child needs to feel in control. Talking to other children who have had this procedure may not foster understanding, because this child has ADHD. Distractions such as noise from a television should be minimized. 3. A teaching session for a child with ADHD should foster attention. Giving instructions verbally and in written form, repeating points, will improve learning for a child with ADHD. The environment needs to be quiet, with minimal distractions. A child who has difficulty concentrating should not lead the session even though the child needs to feel in control. Talking to other children who have had this procedure may not foster understanding, because this child has ADHD. Distractions such as noise from a television should be minimized. 4. A teaching session for a child with ADHD should foster attention. Giving instructions verbally and in written form, repeating points, will improve learning for a child with ADHD. The environment needs to be quiet, with minimal distractions. A child who has difficulty concentrating should not lead the session even though the child needs to feel in control. Talking to other children who have had this procedure may not foster understanding, because this child has ADHD. Distractions such as noise from a television should be minimized. Page Ref: 803-804 Cognitive Level: Analyzing Client Need &Sub: Health Promotion and Maintenance Standards: QSEN Competencies: Patient-centered care | AACN Essential Competencies: Essential VII: Clinical prevention and population health | NLN Competencies: Nursing judgement | Nursing/Integrated Concepts: Nursing Process: Planning/Health teaching and health promotion Learning Outcome: LO 28.5 Use evidence-based practice to plan nursing management for children with cognitive alterations. MNL LO: Mental Health and Cognition/Develop appropriate coping strategies for the child and family to deal with stress. 4) A school-age client is prescribed Adderall (amphetamine mixed salts) for attention deficit hyperactivity disorder (ADHD). At which time is it most appropriate for the nurse to teach the parents to administer this medication? 1. At bedtime 2. Before lunch 3. With the evening meal 4. Early in the morning Answer: 4 Explanation: 1. A side effect of Adderall can be insomnia. Administering the medication early in the day can help alleviate the effect of insomnia. 2. A side effect of Adderall can be insomnia. Administering the medication early in the day can help alleviate the effect of insomnia. 3. A side effect of Adderall can be insomnia. Administering the medication early in the day can help alleviate the effect of insomnia. 4. A side effect of Adderall can be insomnia. Administering the medication early in the day can help alleviate the effect of insomnia. Page Ref: 803 Cognitive Level: Applying Client Need &Sub: Physiological Integrity: Pharmacological and Parenteral Therapies Standards: QSEN Competencies: Patient-centered care | AACN Essential Competencies: Essential VII: Clinical prevention and population health | NLN Competencies: Human flourishing | Nursing/Integrated Concepts: Nursing Process: Planning/Health teaching and health promotion Learning Outcome: LO 28.5 Use evidence-based practice to plan nursing management for children with cognitive alterations. MNL LO: Mental Health and Cognition/Apply the nursing process in providing care for the child and family. 5) An adolescent client diagnosed with attention deficit hyperactivity disorder (ADHD) is interested in playing the drums in the school band. Which action by the nurse is the most appropriate? 1. Recommend the child take private lessons and not join the band. 2. Encourage the child to join the band. 3. Consult with the healthcare provider about allowing participation in band activities. 4. Discourage the child from playing in the band. Answer: 2 Explanation: 1. A child with ADHD may lack connectedness with other children. Participation in a school activity where the rules of working with others can be learned should be encouraged. 2. A child with ADHD may lack connectedness with other children. Participation in a school activity where the rules of working with others can be learned should be encouraged. 3. A child with ADHD may lack connectedness with other children. Participation in a school activity where the rules of working with others can be learned should be encouraged. 4. A child with ADHD may lack connectedness with other children. Participation in a school activity where the rules of working with others can be learned should be encouraged. Page Ref: 804 Cognitive Level: Applying Client Need &Sub: Health Promotion and Maintenance Standards: QSEN Competencies: Patient-centered care | AACN Essential Competencies: Essential IX: Baccalaureate generalist nursing practice | NLN Competencies: Nursing judgement | Nursing/Integrated Concepts: Nursing Process: Implementation/Coordination of care Learning Outcome: LO 28.5 Use evidence-based practice to plan nursing management for children with cognitive alterations. MNL LO: Mental Health and Cognition/Apply the nursing process in providing care for the child and family. 6) A school-age client is evaluated for depression. Which assessment tool does the nurse anticipate will be used by the psychologist? 1. Denver Developmental Screening tool 2. Revised Children's Manifest Anxiety Scale 3. Parent Developmental Questionnaire 4. Disruptive Behavior Disorder Scale Answer: 2 Explanation: 1. The Revised Children's Manifest Anxiety Scale is a tool used to assess for depression. The Denver Developmental Screening tool and the Parent Developmental Questionnaire are tools used to assess development. The Disruptive Behavior Disorder Scale is used to assess for autism. 2. The Revised Children's Manifest Anxiety Scale is a tool used to assess for depression. The Denver Developmental Screening tool and the Parent Developmental Questionnaire are tools used to assess development. The Disruptive Behavior Disorder Scale is used to assess for autism. 3. The Revised Children's Manifest Anxiety Scale is a tool used to assess for depression. The Denver Developmental Screening tool and the Parent Developmental Questionnaire are tools used to assess development. The Disruptive Behavior Disorder Scale is used to assess for autism. 4. The Revised Children's Manifest Anxiety Scale is a tool used to assess for depression. The Denver Developmental Screening tool and the Parent Developmental Questionnaire are tools used to assess development. The Disruptive Behavior Disorder Scale is used to assess for autism. Page Ref: 805 Cognitive Level: Analyzing Client Need &Sub: Health Promotion and Maintenance Standards: QSEN Competencies: Patient-centered care | AACN Essential Competencies: Essential IX: Baccalaureate generalist nursing practice | NLN Competencies: Nursing judgement | Nursing/Integrated Concepts: Nursing Process: Assessment/Coordination of care Learning Outcome: LO 28.2 Discuss the clinical manifestations of the major mental health alterations of childhood and adolescence. MNL LO: Mental Health and Cognition/Examine etiology, risk factors, pathophysiology, and clinical manifestations as seen in children. 7) A nurse is calculating the maximum recommended dose that a school-age client diagnosed with depression can receive for sertraline (Zoloft). The recommended pediatric dose for sertraline (Zoloft) is 1.5 to 3 mg/kg/day. If the child weighs 31 kg, the maximum recommended dose for this child would be mg. Round answer to the nearest whole number. Answer: 93 Explanation: The maximum recommended dose for sertraline (Zoloft) is 3 mg/kg/day. If the child weighs 31 kg, it would be 3 × 31 = 93 mg a day. Page Ref: 805 Cognitive Level: Applying Client Need &Sub: Safe and Effective Care Environment: Safety and Infection Control Standards: QSEN Competencies: Patient-centered care | AACN Essential Competencies: Essential IX: Baccalaureate generalist nursing practice | NLN Competencies: Nursing judgement | Nursing/Integrated Concepts: Nursing Process: Implementation/Coordination of care Learning Outcome: LO 28.3 Plan for the nursing management of children and adolescents with mental health alterations in the hospital and community settings. MNL LO: Mental Health and Cognition/Apply the nursing process in providing care for the child and family. 8) The nurse is planning care for a school-age client, who is diagnosed with bipolar disorder and is having suicidal ideations. Which nursing diagnosis is the priority for this client? 1. Powerlessness Related to Mood Instability 2. Social Isolation Related to Disorder 3. Risk for Injury Related to Suicidal Ideas 4. Impaired Social Interaction Answer: 3 Explanation: 1. The priority for a child with bipolar disorder and suicidal ideas is safety. Risk for Injury would be the nursing diagnosis that would address safety for the child. The other diagnoses have a lower priority. 2. The priority for a child with bipolar disorder and suicidal ideas is safety. Risk for Injury would be the nursing diagnosis that would address safety for the child. The other diagnoses have a lower priority. 3. The priority for a child with bipolar disorder and suicidal ideas is safety. Risk for Injury would be the nursing diagnosis that would address safety for the child. The other diagnoses have a lower priority. 4. The priority for a child with bipolar disorder and suicidal ideas is safety. Risk for Injury would be the nursing diagnosis that would address safety for the child. The other diagnoses have a lower priority. Page Ref: 807-808 Cognitive Level: Analyzing Client Need &Sub: Safe and Effective Care Environment: Safety and Infection Control Standards: QSEN Competencies: Patient-centered care | AACN Essential Competencies: Essential IX: Baccalaureate generalist nursing practice | NLN Competencies: Human flourishing | Nursing/Integrated Concepts: Nursing Process: Diagnosis/Coordination of care Learning Outcome: LO 28.2 Discuss the clinical manifestations of the major mental health alterations of childhood and adolescence. MNL LO: Mental Health and Cognition/Develop appropriate coping strategies for the child and family to deal with stress. 9) An adolescent client diagnosed with panic disorder is prescribed paroxetine (Paxil), a selective serotonin reuptake inhibitor (SSRI). The client tells the nurse she often takes diet pills because she is trying to lose weight. Which response by the nurse is the most appropriate? 1. "You can continue with the paroxetine (Paxil) and the diet pills." 2. "It is important to stop both the paroxetine (Paxil) and the diet pills." 3. "Discontinue using the diet pills while taking the paroxetine (Paxil)." 4. "You should discuss the safety of these two medications pills with a pharmacist." Answer: 3 Explanation: 1. Serotonin syndrome, the serious and life-threatening side effect of SSRIs, can develop when the drug is taken with diet pills, St. John's wort, other antidepressants, alcohol, or LSD. In this case, the diet pills should be discontinued in order to avoid serotonin syndrome. The Paxil should not be discontinued, and waiting to discuss the use of diet pills with a pharmacist would not be an appropriate option. 2. Serotonin syndrome, the serious and life-threatening side effect of SSRIs, can develop when the drug is taken with diet pills, St. John's wort, other antidepressants, alcohol, or LSD. In this case, the diet pills should be discontinued in order to avoid serotonin syndrome. The Paxil should not be discontinued, and waiting to discuss the use of diet pills with a pharmacist would not be an appropriate option. 3. Serotonin syndrome, the serious and life-threatening side effect of SSRIs, can develop when the drug is taken with diet pills, St. John's wort, other antidepressants, alcohol, or LSD. In this case, the diet pills should be discontinued in order to avoid serotonin syndrome. The Paxil should not be discontinued, and waiting to discuss the use of diet pills with a pharmacist would not be an appropriate option. 4. Serotonin syndrome, the serious and life-threatening side effect of SSRIs, can develop when the drug is taken with diet pills, St. John's wort, other antidepressants, alcohol, or LSD. In this case, the diet pills should be discontinued in order to avoid serotonin syndrome. The Paxil should not be discontinued, and waiting to discuss the use of diet pills with a pharmacist would not be an appropriate option. Page Ref: 809 Cognitive Level: Applying Client Need &Sub: Physiological Integrity: Pharmacological and Parenteral Therapies Standards: QSEN Competencies: Patient-centered care | AACN Essential Competencies: Essential VII: Clinical prevention and population health | NLN Competencies: Nursing judgement | Nursing/Integrated Concepts: Nursing Process: Implementation/Health teaching and health promotion Learning Outcome: LO 28.3 Plan for the nursing management of children and adolescents with mental health alterations in the hospital and community settings. MNL LO: Mental Health and Cognition/Examine etiology, risk factors, pathophysiology, and clinical manifestations as seen in children. 10) The nurse is conducting a health history for a school-age client. The parents of the client tell the nurse that their child has the following behaviors: excessive handwashing, counting objects, and hoarding substances. Based on these assessment findings, which diagnosis does the nurse anticipate for this client? 1. Depression 2. Separation anxiety disorder 3. Obsessive-compulsive disorder 4. Bipolar disorder Answer: 3 Explanation: 1. Common behaviors of obsessive-compulsive disorder (OCD) are excessive handwashing, counting objects, and hoarding substances. These practices may take up one or more hours each day. 2. Common behaviors of obsessive-compulsive disorder (OCD) are excessive handwashing, counting objects, and hoarding substances. These practices may take up one or more hours each day. 3. Common behaviors of obsessive-compulsive disorder (OCD) are excessive handwashing, counting objects, and hoarding substances. These practices may take up one or more hours each day. 4. Common behaviors of obsessive-compulsive disorder (OCD) are excessive handwashing, counting objects, and hoarding substances. These practices may take up one or more hours each day. Page Ref: 809 Cognitive Level: Analyzing Client Need &Sub: Physiological Integrity Standards: QSEN Competencies: Patient-centered care | AACN Essential Competencies: Essential IX: Baccalaureate generalist nursing practice | NLN Competencies: Human flourishing | Nursing/Integrated Concepts: Nursing Process: Assessment/Coordination of care Learning Outcome: LO 28.2 Discuss the clinical manifestations of the major mental health alterations of childhood and adolescence. MNL LO: Mental Health and Cognition/Examine etiology, risk factors, pathophysiology, and clinical manifestations as seen in children. 11) A nurse is concerned about the safety of a suicidal adolescent client and wants to be prepared for the use of physical restraints, if necessary. Which action by the nurse is the most appropriate in this situation? 1. Obtain a healthcare provider's order, and follow the institution's policy for use of restraints. 2. Apply the restraints, and then obtain a healthcare provider's order later. 3. Apply the restraints if parental permission is obtained. 4. Ask for the child's permission before applying the restraints. Answer: 1 Explanation: 1. Restraints are used only when ordered by the physician and interdisciplinary team caring for the child. Physical restraint is only a short-term approach to provide immediate safety if necessary. It would not be appropriate to apply the restraints, and then obtain a healthcare provider's order. Even if permission is given by the parent and/or child, a healthcare provider's order still needs to be obtained. 2. Restraints are used only when ordered by the physician and interdisciplinary team caring for the child. Physical restraint is only a short-term approach to provide immediate safety if necessary. It would not be appropriate to apply the restraints, and then obtain a healthcare provider's order. Even if permission is given by the parent and/or child, a healthcare provider's order still needs to be obtained. 3. Restraints are used only when ordered by the physician and interdisciplinary team caring for the child. Physical restraint is only a short-term approach to provide immediate safety if necessary. It would not be appropriate to apply the restraints, and then obtain a healthcare provider's order. Even if permission is given by the parent and/or child, a healthcare provider's order still needs to be obtained. 4. Restraints are used only when ordered by the physician and interdisciplinary team caring for the child. Physical restraint is only a short-term approach to provide immediate safety if necessary. It would not be appropriate to apply the restraints, and then obtain a healthcare provider's order. Even if permission is given by the parent and/or child, a healthcare provider's order still needs to be obtained. Page Ref: 812-813 Cognitive Level: Analyzing Client Need &Sub: Safe and Effective Care Environment: Safety and Infection Control Standards: QSEN Competencies: Teamwork and collaboration | AACN Essential Competencies: Essential VI: Interprofessional communication and collaboration for improving patient health outcomes | NLN Competencies: Professional identity | Nursing/Integrated Concepts: Nursing Process: Planning/Collaboration Learning Outcome: LO 28.3 Plan for the nursing management of children and adolescents with mental health alterations in the hospital and community settings. MNL LO: Mental Health and Cognition/Develop appropriate coping strategies for the child and family to deal with stress. 12) A nurse is conducting developmental assessments on several children in the day-care setting. Which child(ren) does the nurse identify as having development delays? Select all that apply. 1. An 18-month-old toddler who is unable to phrase sentences 2. A 5-year-old who is unable to button his shirt 3. A 6-year-old who is unable to sit still for a short story 4. A 2-year-old who is unable to cut with scissors 5. A 2-year-old who cannot recite her phone number Answer: 2, 3 Explanation: 1. A developmental milestone that can indicate learning disability is a kindergartener's being unable to button his shirt. Inability to phrase sentences is considered a delay if not done by 2-1/2 years, inability to sit still for a short story is considered a delay if the child is 3 to 5 years old, and being unable to cut with scissors indicates a delay if not done by kindergarten age. Reciting the phone number is not appropriate for a 2-year-old. 2. A developmental milestone that can indicate learning disability is a kindergartener's being unable to button his shirt. Inability to phrase sentences is considered a delay if not done by 2-1/2 years, inability to sit still for a short story is considered a delay if the child is 3 to 5 years old, and being unable to cut with scissors indicates a delay if not done by kindergarten age. Reciting the phone number is not appropriate for a 2-year-old. 3. A developmental milestone that can indicate learning disability is a kindergartener's being unable to button his shirt. Inability to phrase sentences is considered a delay if not done by 2-1/2 years, inability to sit still for a short story is considered a delay if the child is 3 to 5 years old, and being unable to cut with scissors indicates a delay if not done by kindergarten age. Reciting the phone number is not appropriate for a 2-year-old. 4. A developmental milestone that can indicate learning disability is a kindergartener's being unable to button his shirt. Inability to phrase sentences is considered a delay if not done by 2-1/2 years, inability to sit still for a short story is considered a delay if the child is 3 to 5 years old, and being unable to cut with scissors indicates a delay if not done by kindergarten age. Reciting the phone number is not appropriate for a 2-year-old. 5. A developmental milestone that can indicate learning disability is a kindergartener's being unable to button his shirt. Inability to phrase sentences is considered a delay if not done by 2-1/2 years, inability to sit still for a short story is considered a delay if the child is 3 to 5 years old, and being unable to cut with scissors indicates a delay if not done by kindergarten age. Reciting the phone number is not appropriate for a 2-year-old. Page Ref: 814-815 Cognitive Level: Analyzing Client Need &Sub: Health Promotion and Maintenance Standards: QSEN Competencies: Patient-centered care | AACN Essential Competencies: Essential IX: Baccalaureate generalist nursing practice | NLN Competencies: Human flourishing | Nursing/Integrated Concepts: Nursing Process: Assessment/Coordination of care Learning Outcome: LO 28.1 Define mental health and describe major mental health alterations in childhood. MNL LO: Mental Health and Cognition/Examine etiology, risk factors, pathophysiology, and clinical manifestations as seen in children. 13) The parents of a client recently diagnosed with Down syndrome relate to the nurse that they "feel guilty about causing the condition." Which response by the nurse is the most appropriate? 1. "Down syndrome is a condition caused by an extra chromosome; the cause of it is unknown." 2. "Down syndrome is a condition that is genetically transmitted from both the father and the mother." 3. "Down syndrome is a condition that is carried on the X chromosome, so it came from the mother." 4. "Down syndrome is caused by birth trauma, not by genetics." Answer: 1 Explanation: 1. The therapeutic and accurate response is that Down syndrome is a condition caused by an extra chromosome, but we don't know why it occurs. The other responses are nontherapeutic or inaccurate. 2. The therapeutic and accurate response is that Down syndrome is a condition caused by an extra chromosome, but we don't know why it occurs. The other responses are nontherapeutic or inaccurate. 3. The therapeutic and accurate response is that Down syndrome is a condition caused by an extra chromosome, but we don't know why it occurs. The other responses are nontherapeutic or inaccurate. 4. The therapeutic and accurate response is that Down syndrome is a condition caused by an extra chromosome, but we don't know why it occurs. The other responses are nontherapeutic or inaccurate. Page Ref: 815 Cognitive Level: Applying Client Need &Sub: Psychosocial Integrity Standards: QSEN Competencies: Patient-centered care | AACN Essential Competencies: Essential II: Basic organizational and systems leadership for quality care and patient safety | NLN Competencies: Human flourishing | Nursing/Integrated Concepts: Nursing Process: Implementation/Communication Learning Outcome: LO 28.4 Describe characteristics of common cognitive alterations of childhood. MNL LO: Mental Health and Cognition/Examine etiology, risk factors, pathophysiology, and clinical manifestations as seen in children. 14) A child with a profound intellectual disability is admitted to the hospital for an appendectomy. Which IQ does the nurse anticipate to see documented when reviewing this child's medical record? 1. Between 50 and 70 2. Below 20 3. Between 35 and 50 4. Between 20 and 35 Answer: 2 Explanation: 1. "Profound" intellectual disability is described as an intelligence quotient (IQ) below 20. "Mild" intellectual disability is described as an IQ between 50 and 70, "moderate" intellectual disability is an IQ between 35 and 50, and "severe" intellectual disability is an IQ between 20 and 35. 2. "Profound" intellectual disability is described as an intelligence quotient (IQ) below 20. "Mild" intellectual disability is described as an IQ between 50 and 70, "moderate" intellectual disability is an IQ between 35 and 50, and "severe" intellectual disability is an IQ between 20 and 35. 3. "Profound" intellectual disability is described as an intelligence quotient (IQ) below 20. "Mild" intellectual disability is described as an IQ between 50 and 70, "moderate" intellectual disability is an IQ between 35 and 50, and "severe" intellectual disability is an IQ between 20 and 35. 4. "Profound" intellectual disability is described as an intelligence quotient (IQ) below 20. "Mild" intellectual disability is described as an IQ between 50 and 70, "moderate" intellectual disability is an IQ between 35 and 50, and "severe" intellectual disability is an IQ between 20 and 35. Page Ref: 816 Cognitive Level: Analyzing Client Need &Sub: Health Promotion and Maintenance Standards: QSEN Competencies: Patient-centered care | AACN Essential Competencies: Essential IX: Baccalaureate generalist nursing practice | NLN Competencies: Human flourishing | Nursing/Integrated Concepts: Nursing Process: Assessment/Coordination of care Learning Outcome: LO 28.5 Use evidence-based practice to plan nursing management for children with cognitive alterations. MNL LO: Mental Health and Cognition/Apply the nursing process in providing care for the child and family. 15) The family of a preschool-age client diagnosed with an intellectual disability is expressing difficulty with managing the care needs of the child. Which nursing diagnosis is most appropriate for this situation? 1. Hopelessness Related to Terminal Condition of the Child 2. Compromised Family Coping Related to the Child's Developmental Variations 3. Family Processes Dysfunctional, Related to a Child with Intellectual Disability 4. Impaired Parenting Related to Poor Parenting Skills Answer: 2 Explanation: 1. The family is compromised but not dysfunctional. Hopelessness and impaired parenting are not appropriate in the given situation. 2. The family is compromised but not dysfunctional. Hopelessness and impaired parenting are not appropriate in the given situation. 3. The family is compromised but not dysfunctional. Hopelessness and impaired parenting are not appropriate in the given situation. 4. The family is compromised but not dysfunctional. Hopelessness and impaired parenting are not appropriate in the given situation. Page Ref: 818 Cognitive Level: Analyzing Client Need &Sub: Psychosocial Integrity Standards: QSEN Competencies: Patient-centered care | AACN Essential Competencies: Essential II: Basic organizational and systems leadership for quality care and patient safety | NLN Competencies: Nursing judgement | Nursing/Integrated Concepts: Nursing Process: Diagnosis/Coordination of care Learning Outcome: LO 28.3 Plan for the nursing management of children and adolescents with mental health alterations in the hospital and community settings. MNL LO: Mental Health and Cognition/Educate family on the care of the child with a cognitive disorder. 16) The nurse is planning care for an adolescent client with a newly diagnosed intellectual disability following a traumatic brain injury. Which expected outcomes are appropriate for this client? Select all that apply. 1. The family understands the adolescent's diagnosis. 2. The family understands the specific physical and developmental needs of the adolescent. 3. The adolescent develops self-care skills appropriate to his or her developmental level. 4. The adolescent's family is able to access the necessary community and educational resources. 5. The family's ability to cope with changing needs of the adolescent. Answer: 1, 2, 3, 4 Explanation: 1. All statements are appropriate outcomes for the adolescent and the family except the statement regarding the family's ability to cope with the changing needs of the adolescent. This is an evaluation statement. 2. All statements are appropriate outcomes for the adolescent and the family except the statement regarding the family's ability to cope with the changing needs of the adolescent. This is an evaluation statement. 3. All statements are appropriate outcomes for the adolescent and the family except the statement regarding the family's ability to cope with the changing needs of the adolescent. This is an evaluation statement. 4. All statements are appropriate outcomes for the adolescent and the family except the statement regarding the family's ability to cope with the changing needs of the adolescent. This is an evaluation statement. 5. All statements are appropriate outcomes for the adolescent and the family except the statement regarding the family's ability to cope with the changing needs of the adolescent. This is an evaluation statement. Page Ref: 818 Cognitive Level: Applying Client Need &Sub: Health Promotion and Maintenance Standards: QSEN Competencies: Patient-centered care | AACN Essential Competencies: Essential IX: Baccalaureate generalist nursing practice | NLN Competencies: Nursing judgement | Nursing/Integrated Concepts: Nursing Process: Planning/Coordination of care Learning Outcome: LO 28.6: Establish and evaluate expected outcomes of care for the child with a cognitive alteration. MNL LO: Mental Health and Cognition/Apply the nursing process in providing care for the child and family. 17) Match the classifications of bipolar disorder with their description. A. Bipolar I B. Bipolar II C. Cyclothymic disorder D. Bipolar not otherwise specified 1. Manifests as multiple mild manic and depressive episodes. 2. At least one episode of mild to moderate mania and one of depression. 3. Includes a severe manic episode that requires hospitalization or causes functional impairment in life. 4. Rapid mood fluctuations, mania without depressive episodes, or chronic depression with hypomania episodes. Answer: 1/C, 2/B, 3/A, 4/D 1. Cyclothymic disorder 2. Bipolar II 3. Bipolar I 4. Bipolar not otherwise specified Explanation: Bipolar I—Includes a severe manic episode that requires specified hospitalization or causes functional impairment in life. Bipolar II—At least one episode of mild to moderate mania and one of depression. Cyclothymic disorder—Manifests as multiple mild manic and depressive episodes. Bipolar not otherwise specified—Rapid mood fluctuations, mania without depressive episodes, or chronic depression with hypomania episodes. Page Ref: 808 Cognitive Level: Analyzing Client Need &Sub: Safe and Effective Care Environment: Management of Care Standards: QSEN Competencies: Patient-centered care | AACN Essential Competencies: Essential IX: Baccalaureate generalist nursing practice | NLN Competencies: Human flourishing | Nursing/Integrated Concepts: Nursing Process: Diagnosis/Coordination of care Learning Outcome: LO 28.2 Discuss the clinical manifestations of the major mental health alterations of childhood and adolescence. MNL LO: Mental Health and Cognition/Examine etiology, risk factors, pathophysiology, and clinical manifestations as seen in children. 18) A 15-year-old female is brought to the hospital by her friends after she fainted at the movies. Her friends told the nurse that she had been sad, withdrawn and this is why they asked her to go to the movies. The girl became "excited and energetic" at the movies and then fainted. The physician ordered: Lithium, clonazepam and doxepin. One order is Lithium 300 mg PO tid Medication on hand: Lithium 150 mg/capsule. Calculate how many capsules of lithium will be given by mouth. Answer: 2 capsules Explanation: 2 capsules Page Ref: 807-808 Cognitive Level: Analyzing Client Need &Sub: Physiological Integrity: Pharmacological and Parenteral Therapies Standards: QSEN Competencies: Patient-centered care | AACN Essential Competencies: Essential IX: Baccalaureate generalist nursing practice | NLN Competencies: Human flourishing | Nursing/Integrated Concepts: Nursing Process: Planning/Coordination of care Learning Outcome: LO 28.2 Discuss the clinical manifestations of the major mental health alterations of childhood and adolescence. MNL LO: Mental Health and Cognition/Differentiate treatment options and strategies for continuity of care for the child and family. Principles of Pediatric Nursing: Caring for Children, 7e (Ball et al.) Chapter 29 Alterations in Musculoskeletal Function 1) The nurse completes parent education related to treatment for a pediatric client with congenital clubfoot. Which statement by the parents indicates the need for further education? 1. "We're happy this is the only cast our baby will need." 2. "We'll watch for any swelling of the feet while the casts are on." 3. "We'll keep the casts dry." 4. "We're getting a special car seat to accommodate the casts." Answer: 1 Explanation: 1. Serial casting is the treatment of choice for congenital clubfoot. The cast is changed every one to 2 weeks. Parents should be watching for swelling while the casts are on, keeping the casts dry, and using a car seat to accommodate the casts. 2. Serial casting is the treatment of choice for congenital clubfoot. The cast is changed every one to 2 weeks. Parents should be watching for swelling while the casts are on, keeping the casts dry, and using a car seat to accommodate the casts. 3. Serial casting is the treatment of choice for congenital clubfoot. The cast is changed every one to 2 weeks. Parents should be watching for swelling while the casts are on, keeping the casts dry, and using a car seat to accommodate the casts. 4. Serial casting is the treatment of choice for congenital clubfoot. The cast is changed every one to 2 weeks. Parents should be watching for swelling while the casts are on, keeping the casts dry, and using a car seat to accommodate the casts. Page Ref: 828 Cognitive Level: Analyzing Client Need &Sub: Health Promotion and Maintenance Standards: QSEN Competencies: Patient-centered care | AACN Essential Competencies: Essential VII: Clinical prevention and population health | NLN Competencies: Human flourishing | Nursing/Integrated Concepts: Nursing Process: Evaluation/Health teaching and health promotion Learning Outcome: LO 29.2 Plan nursing care for children with structural deformities of the foot, leg, hip, and spine. MNL LO: Integumentary and Musculoskeletal Disorders/Apply the nursing process in providing care for the child and family. 2) An infant returns from surgery for correction of bilateral congenital clubfeet. The infant has bilateral long-leg casts. The toes on both feet are edematous, but there is color, sensitivity, and movement to them. Which action by the nurse is the most appropriate? 1. Call the healthcare provider to report the edema. 2. Elevate the legs on pillows. 3. Apply a warm, moist pack to the feet. 4. Encourage movement of toes. Answer: 2 Explanation: 1. The legs should be elevated on a pillow for 24 hours to promote healing and help with venous return. Some amount of swelling can be expected, so it would not be appropriate to notify the healthcare provider, especially if the color, sensitivity, and movement remain normal to the toes. Ice should be applied, not heat. An infant would not be able to follow directions to move toes, and in this case it would not be as effective as elevating the legs on pillows. 2. The legs should be elevated on a pillow for 24 hours to promote healing and help with venous return. Some amount of swelling can be expected, so it would not be appropriate to notify the healthcare provider, especially if the color, sensitivity, and movement remain normal to the toes. Ice should be applied, not heat. An infant would not be able to follow directions to move toes, and in this case it would not be as effective as elevating the legs on pillows. 3. The legs should be elevated on a pillow for 24 hours to promote healing and help with venous return. Some amount of swelling can be expected, so it would not be appropriate to notify the healthcare provider, especially if the color, sensitivity, and movement remain normal to the toes. Ice should be applied, not heat. An infant would not be able to follow directions to move toes, and in this case it would not be as effective as elevating the legs on pillows. 4. The legs should be elevated on a pillow for 24 hours to promote healing and help with venous return. Some amount of swelling can be expected, so it would not be appropriate to notify the healthcare provider, especially if the color, sensitivity, and movement remain normal to the toes. Ice should be applied, not heat. An infant would not be able to follow directions to move toes, and in this case it would not be as effective as elevating the legs on pillows. Page Ref: 829-830 Cognitive Level: Applying Client Need &Sub: Physiological Integrity: Reduction of Risk Potential Standards: QSEN Competencies: Patient-centered care | AACN Essential Competencies: Essential IX: Baccalaureate generalist nursing practice | NLN Competencies: Human flourishing | Nursing/Integrated Concepts: Nursing Process: Implementation/Coordination of care Learning Outcome: LO 29.5 Prioritize nursing interventions to promote safety and developmental progression in children who require braces, casts, traction, and surgery. MNL LO: Integumentary and Musculoskeletal Disorders/Apply the nursing process in providing care for the child and family. 3) The nurse in the newborn nursery is performing the admission assessment on a neonate. Which assessment finding indicates the neonate may have congenital hip dysplasia? 1. Asymmetry of the gluteal and thigh fat folds 2. Trendelenburg sign 3. Telescoping of the affected limb 4. Lordosis Answer: 1 Explanation: 1. A sign of congenital hip dysplasia in the infant would be asymmetry of the gluteal and thigh fat folds. Trendelenburg sign and telescoping of the affected limb are signs that present in an older child with congenital hip dysplasia. Lordosis does not occur with hip dysplasia. 2. A sign of congenital hip dysplasia in the infant would be asymmetry of the gluteal and thigh fat folds. Trendelenburg sign and telescoping of the affected limb are signs that present in an older child with congenital hip dysplasia. Lordosis does not occur with hip dysplasia. 3. A sign of congenital hip dysplasia in the infant would be asymmetry of the gluteal and thigh fat folds. Trendelenburg sign and telescoping of the affected limb are signs that present in an older child with congenital hip dysplasia. Lordosis does not occur with hip dysplasia. 4. A sign of congenital hip dysplasia in the infant would be asymmetry of the gluteal and thigh fat folds. Trendelenburg sign and telescoping of the affected limb are signs that present in an older child with congenital hip dysplasia. Lordosis does not occur with hip dysplasia. Page Ref: 832 Cognitive Level: Applying Client Need &Sub: Physiological Integrity: Physiological Adaptation Standards: QSEN Competencies: Patient-centered care | AACN Essential Competencies: Essential IX: Baccalaureate generalist nursing practice | NLN Competencies: Human flourishing | Nursing/Integrated Concepts: Nursing Process: Assessment/Coordination of care Learning Outcome: LO 29.1 Describe pediatric variations in the musculoskeletal system. MNL LO: Integumentary and Musculoskeletal Disorders/Examine etiology, risk factors, pathophysiology, and clinical manifestations as seen in children. 4) The nurse is teaching family members how to care for their infant in a Pavlik harness to treat congenital developmental dysplasia of the hip. Which statement will the nurse include in the teaching session? 1. "Apply lotion or powder to minimize skin irritation." 2. "Put clothing over the harness for maximum effectiveness of the device." 3. "Check at least 2 or 3 times a day for red areas under the straps." 4. "Place a diaper over the harness, preferably using a thin, superabsorbent, disposable diaper." Answer: 3 Explanation: 1. The brace should be checked 2 or 3 times for red areas under the straps. Lotion or powder can contribute to skin breakdown. A light layer of clothing should be under the brace, not over. The diaper should also be under the brace. 2. The brace should be checked 2 or 3 times for red areas under the straps. Lotion or powder can contribute to skin breakdown. A light layer of clothing should be under the brace, not over. The diaper should also be under the brace. 3. The brace should be checked 2 or 3 times for red areas under the straps. Lotion or powder can contribute to skin breakdown. A light layer of clothing should be under the brace, not over. The diaper should also be under the brace. 4. The brace should be checked 2 or 3 times for red areas under the straps. Lotion or powder can contribute to skin breakdown. A light layer of clothing should be under the brace, not over. The diaper should also be under the brace. Page Ref: 832-833 Cognitive Level: Applying Client Need &Sub: Physiological Integrity: Basic Care and Comfort Standards: QSEN Competencies: Patient-centered care | AACN Essential Competencies: Essential VII: Clinical prevention and population health | NLN Competencies: Human flourishing | Nursing/Integrated Concepts: Nursing Process: Implementation/Health teaching and health promotion Learning Outcome: LO 29.5 Prioritize nursing interventions to promote safety and developmental progression in children who require braces, casts, traction, and surgery. MNL LO: Integumentary and Musculoskeletal Disorders/Educate the child and family on the care of the child during hospitalization and upon discharge. 5) The nurse is caring for a pediatric client in Bryant skin traction. Which nursing intervention is most appropriate for this client? 1. Remove the adhesive traction straps daily to prevent skin breakdown. 2. Check the traction frequently to ensure that proper alignment is maintained. 3. Place the child in a prone position to maintain good alignment. 4. Move the child as infrequently as possible to maintain traction. Answer: 2 Explanation: 1. The traction apparatus should be checked frequently to ensure that proper alignment is maintained. The adhesive straps should not be removed. The child should be placed in a supine position, and frequent repositioning is necessary to prevent complications of immobility. 2. The traction apparatus should be checked frequently to ensure that proper alignment is maintained. The adhesive straps should not be removed. The child should be placed in a supine position, and frequent repositioning is necessary to prevent complications of immobility. 3. The traction apparatus should be checked frequently to ensure that proper alignment is maintained. The adhesive straps should not be removed. The child should be placed in a supine position, and frequent repositioning is necessary to prevent complications of immobility. 4. The traction apparatus should be checked frequently to ensure that proper alignment is maintained. The adhesive straps should not be removed. The child should be placed in a supine position, and frequent repositioning is necessary to prevent complications of immobility. Page Ref: 833 Cognitive Level: Applying Client Need &Sub: Physiological Integrity: Basic Care and Comfort Standards: QSEN Competencies: Patient-centered care | AACN Essential Competencies: Essential IX: Baccalaureate generalist nursing practice | NLN Competencies: Human flourishing | Nursing/Integrated Concepts: Nursing Process: Implementation/Coordination of care Learning Outcome: LO 29.5 Prioritize nursing interventions to promote safety and developmental progression in children who require braces, casts, traction, and surgery. MNL LO: Integumentary and Musculoskeletal Disorders/Differentiate treatment options and strategies for continuity of care for the child and family. 6) The nurse has completed discharge teaching for the family of a child diagnosed with Legg- Calve-Perthes disease. Which statement by the family indicates the need for further education? 1. "We're glad this will only take about 6 weeks to correct." 2. "We understand swimming is a good sport for Legg-Calve-Perthes." 3. "We know to watch for areas on the skin the brace may rub." 4. "We understand that abduction of the affected leg is important." Answer: 1 Explanation: 1. The treatment generally takes approximately 2 years. Swimming is a good activity to increase mobility. A brace may be worn, so skin irritation should be monitored. The leg should be kept in the abducted position. 2. The treatment generally takes approximately 2 years. Swimming is a good activity to increase mobility. A brace may be worn, so skin irritation should be monitored. The leg should be kept in the abducted position. 3. The treatment generally takes approximately 2 years. Swimming is a good activity to increase mobility. A brace may be worn, so skin irritation should be monitored. The leg should be kept in the abducted position. 4. The treatment generally takes approximately 2 years. Swimming is a good activity to increase mobility. A brace may be worn, so skin irritation should be monitored. The leg should be kept in the abducted position. Page Ref: 836 Cognitive Level: Analyzing Client Need &Sub: Physiological Integrity: Basic Care and Comfort Standards: QSEN Competencies: Patient-centered care | AACN Essential Competencies: Essential VII: Clinical prevention and population health | NLN Competencies: Human flourishing | Nursing/Integrated Concepts: Nursing Process: Evaluation/Health teaching and health promotion Learning Outcome: LO 29.4 Partner with families to plan care for children with musculoskeletal conditions that are chronic or require long-term care. MNL LO: Integumentary and Musculoskeletal Disorders/Educate the child and family on the care of the child during hospitalization and upon discharge. 7) A school health nurse is screening school-age students for scoliosis. Which assessment findings indicate the need for further evaluation for scoliosis? Select all that apply. 1. Uneven shoulders and hips 2. A one-sided rib hump 3. Prominent scapula 4. Lordosis 5. Pain Answer: 1, 2, 3 Explanation: 1. The classic signs of scoliosis include uneven shoulders and hips, a one-sided rib hump, and prominent scapula. Lordosis and pain are not present with scoliosis. 2. The classic signs of scoliosis include uneven shoulders and hips, a one-sided rib hump, and prominent scapula. Lordosis and pain are not present with scoliosis. 3. The classic signs of scoliosis include uneven shoulders and hips, a one-sided rib hump, and prominent scapula. Lordosis and pain are not present with scoliosis. 4. The classic signs of scoliosis include uneven shoulders and hips, a one-sided rib hump, and prominent scapula. Lordosis and pain are not present with scoliosis. 5. The classic signs of scoliosis include uneven shoulders and hips, a one-sided rib hump, and prominent scapula. Lordosis and pain are not present with scoliosis. Page Ref: 838-839 Cognitive Level: Analyzing Client Need &Sub: Health Promotion and Maintenance Standards: QSEN Competencies: Patient-centered care | AACN Essential Competencies: Essential IX: Baccalaureate generalist nursing practice | NLN Competencies: Human flourishing | Nursing/Integrated Concepts: Nursing Process: Assessment/Coordination Learning Outcome: LO 29.1 Describe pediatric variations in the musculoskeletal system. MNL LO: Integumentary and Musculoskeletal Disorders/Examine etiology, risk factors, pathophysiology, and clinical manifestations as seen in children. 8) An adolescent client must wear a brace for the correction of scoliosis. Which nursing diagnosis is most appropriate for this client? 1. Risk for Impaired Skin Integrity 2. Risk for Altered Growth and Development 3. Risk for Impaired Mobility 4. Risk for Impaired Gas Exchange Answer: 1 Explanation: 1. The skin should be monitored for breakdown in any area the brace may rub. The other diagnoses would not be a priority and should be corrected by the wearing of the brace. 2. The skin should be monitored for breakdown in any area the brace may rub. The other diagnoses would not be a priority and should be corrected by the wearing of the brace. 3. The skin should be monitored for breakdown in any area the brace may rub. The other diagnoses would not be a priority and should be corrected by the wearing of the brace. 4. The skin should be monitored for breakdown in any area the brace may rub. The other diagnoses would not be a priority and should be corrected by the wearing of the brace. Page Ref: 839 Cognitive Level: Analyzing Client Need &Sub: Physiological Integrity: Reduction of Risk Potential Standards: QSEN Competencies: Patient-centered care | AACN Essential Competencies: Essential IX: Baccalaureate generalist nursing practice | NLN Competencies: Human flourishing | Nursing/Integrated Concepts: Nursing Process: Diagnosis/Coordination of care Learning Outcome: LO 29.5 Prioritize nursing interventions to promote safety and developmental progression in children who require braces, casts, traction, and surgery. MNL LO: Integumentary and Musculoskeletal Disorders/Differentiate treatment options and strategies for continuity of care for the child and family. 9) A child returns from spinal-fusion surgery. Which item is the priority assessment for this child? 1. Increased intracranial pressure 2. Seizure activity 3. Impaired pupillary response during neurological checks 4. Impaired color, sensitivity, and movement to lower extremities Answer: 4 Explanation: 1. When the spinal column is manipulated, there is a risk for impaired color, sensitivity, and movement to lower extremities. The other signs are neurological impairment and are not high risk with spinal surgery. 2. When the spinal column is manipulated, there is a risk for impaired color, sensitivity, and movement to lower extremities. The other signs are neurological impairment and are not high risk with spinal surgery. 3. When the spinal column is manipulated, there is a risk for impaired color, sensitivity, and movement to lower extremities. The other signs are neurological impairment and are not high risk with spinal surgery. 4. When the spinal column is manipulated, there is a risk for impaired color, sensitivity, and movement to lower extremities. The other signs are neurological impairment and are not high risk with spinal surgery. Page Ref: 839 Cognitive Level: Applying Client Need &Sub: Physiological Integrity: Reduction of Risk Potential Standards: QSEN Competencies: Patient-centered care | AACN Essential Competencies: Essential IX: Baccalaureate generalist nursing practice | NLN Competencies: Human flourishing | Nursing/Integrated Concepts: Nursing Process: Assessment/Coordination of care Learning Outcome: LO 29.5 Prioritize nursing interventions to promote safety and developmental progression in children who require braces, casts, traction, and surgery. MNL LO: Integumentary and Musculoskeletal Disorders/Educate the child and family on the care of the child during hospitalization and upon discharge. 10) A nurse notes blue sclera during a newborn assessment. Which item will the newborn require further assessment for based on this finding? 1. Marfan syndrome 2. Achondroplasia 3. Osteogenesis imperfecta 4. Muscular dystrophy Answer: 3 Explanation: 1. Clinical manifestations of osteogenesis imperfecta include blue sclera. This is not present in Marfan syndrome, achondroplasia, or muscular dystrophy. 2. Clinical manifestations of osteogenesis imperfecta include blue sclera. This is not present in Marfan syndrome, achondroplasia, or muscular dystrophy. 3. Clinical manifestations of osteogenesis imperfecta include blue sclera. This is not present in Marfan syndrome, achondroplasia, or muscular dystrophy. 4. Clinical manifestations of osteogenesis imperfecta include blue sclera. This is not present in Marfan syndrome, achondroplasia, or muscular dystrophy. Page Ref: 852 Cognitive Level: Analyzing Client Need &Sub: Physiological Integrity: Reduction of Risk Potential Standards: QSEN Competencies: Patient-centered care | AACN Essential Competencies: Essential IX: Baccalaureate generalist nursing practice | NLN Competencies: Human flourishing | Nursing/Integrated Concepts: Nursing Process: Assessment/Coordination of care Learning Outcome: LO 29.2 Plan nursing care for children with structural deformities of the foot, leg, hip, and spine. MNL LO: Integumentary and Musculoskeletal Disorders/Examine etiology, risk factors, pathophysiology, and clinical manifestations as seen in children. 11) The nurse is providing care to a toddler client who is diagnosed with osteogenesis imperfecta. Which nursing intervention is appropriate for this client? 1. Support of the trunk and extremities when moving 2. Traction care 3. Cast care 4. Postop spinal surgery care Answer: 1 Explanation: 1. With osteogenesis imperfecta, nursing care focuses on preventing fractures. Because the bones are fragile, the entire body must be supported when the child is moved. Traction, casts, and spinal surgery are not routinely done for osteogenesis. 2. With osteogenesis imperfecta, nursing care focuses on preventing fractures. Because the bones are fragile, the entire body must be supported when the child is moved. Traction, casts, and spinal surgery are not routinely done for osteogenesis. 3. With osteogenesis imperfecta, nursing care focuses on preventing fractures. Because the bones are fragile, the entire body must be supported when the child is moved. Traction, casts, and spinal surgery are not routinely done for osteogenesis. 4. With osteogenesis imperfecta, nursing care focuses on preventing fractures. Because the bones are fragile, the entire body must be supported when the child is moved. Traction, casts, and spinal surgery are not routinely done for osteogenesis. Page Ref: 847-848 Cognitive Level: Applying Client Need &Sub: Physiological Integrity: Reduction of Risk Potential Standards: QSEN Competencies: Patient-centered care | AACN Essential Competencies: Essential IX: Baccalaureate generalist nursing practice | NLN Competencies: Nursing judgement | Nursing/Integrated Concepts: Nursing Process: Implementation/Coordination of care Learning Outcome: LO 29.2 Plan nursing care for children with structural deformities of the foot, leg, hip, and spine. MNL LO: Integumentary and Musculoskeletal Disorders/Apply the nursing process in providing care for the child and family. 12) An adolescent client who is diagnosed with Duchenne muscular dystrophy is seen in the clinic for a routine health visit. Which nursing diagnosis is the priority for this client? 1. Risk for Impaired Mobility Related to Hypertrophy of Muscles 2. Risk for Infection Related to Altered Immune System 3. Risk for Impaired Skin Integrity Related to Paresthesia 4. Risk for Altered Comfort Related to Effects of the Illness Answer: 1 Explanation: 1. Nursing care for muscular dystrophy (MD) focuses on promoting independence and mobility for this progressive, incapacitating disease. Risk for Infection, Risk for Impaired Skin Integrity, and Risk for Altered Comfort are not as high a priority as Risk for Impaired Mobility. 2. Nursing care for muscular dystrophy (MD) focuses on promoting independence and mobility for this progressive, incapacitating disease. Risk for Infection, Risk for Impaired Skin Integrity, and Risk for Altered Comfort are not as high a priority as Risk for Impaired Mobility. 3. Nursing care for muscular dystrophy (MD) focuses on promoting independence and mobility for this progressive, incapacitating disease. Risk for Infection, Risk for Impaired Skin Integrity, and Risk for Altered Comfort are not as high a priority as Risk for Impaired Mobility. 4. Nursing care for muscular dystrophy (MD) focuses on promoting independence and mobility for this progressive, incapacitating disease. Risk for Infection, Risk for Impaired Skin Integrity, and Risk for Altered Comfort are not as high a priority as Risk for Impaired Mobility. Page Ref: 850-852 Cognitive Level: Analyzing Client Need &Sub: Physiological Integrity: Basic Care and Comfort Standards: QSEN Competencies: Patient-centered care | AACN Essential Competencies: Essential IX: Baccalaureate generalist nursing practice | NLN Competencies: Human flourishing | Nursing/Integrated Concepts: Nursing Process: Diagnosis/Coordination of care Learning Outcome: LO 29.4 Partner with families to plan care for children with musculoskeletal conditions that are chronic or require long-term care. MNL LO: Integumentary and Musculoskeletal Disorders/Differentiate treatment options and strategies for continuity of care for the child and family. 13) The school nurse is providing care to a school-age client who experienced a sprain of the right ankle on the playground. Which intervention is appropriate for the nurse to implement for this client? 1. Apply ice to the extremity 2. Apply a warm, moist pack to the extremity 3. Perform passive range of motion to the extremity 4. Lower the extremity to below the level of the heart Answer: 1 Explanation: 1. For the first 24 hours of a sprain, rest, ice, compression, and elevation should be used. Therefore, the nurse should apply ice to the extremity. 2. For the first 24 hours of a sprain, rest, ice, compression, and elevation should be used. Therefore, the nurse should apply ice to the extremity. 3. For the first 24 hours of a sprain, rest, ice, compression, and elevation should be used. Therefore, the nurse should apply ice to the extremity. 4. For the first 24 hours of a sprain, rest, ice, compression, and elevation should be used. Therefore, the nurse should apply ice to the extremity. Page Ref: 852 Cognitive Level: Applying Client Need &Sub: Physiological Integrity: Basic Care and Comfort Standards: QSEN Competencies: Patient-centered care | AACN Essential Competencies: Essential IX: Baccalaureate generalist nursing practice | NLN Competencies: Human flourishing | Nursing/Integrated Concepts: Nursing Process: Implementation/Coordination of care Learning Outcome: LO 29.6 Develop a nursing care plan for fractures, including teaching for injury prevention and nursing implementations for the child who has sustained a fracture. MNL LO: Integumentary and Musculoskeletal Disorders/Apply the nursing process in providing care for the child and family. 14) A nurse is assessing a child after an open reduction of a fractured femur. Which assessment findings would indicate that the child is experiencing compartment syndrome? Select all that apply. 1. Pink, warm extremity 2. Pain not relieved by pain medication 3. Dorsalis pedis pulse present 4. Prolonged capillary-refill time with paresthesia 5. Skin appears tense. Answer: 2, 4, 5 Explanation: 1. The major serious complication post-fracture reduction is compartment syndrome. A prolonged capillary-refill time with loss of paresthesia, pain not relieved by medication, and skin that appears tense are signs of compartment syndrome. Pink, warm extremity; pain relieved by medication; and a present dorsalis pedis pulse would all be normal findings post-fracture reduction. 2. The major serious complication post-fracture reduction is compartment syndrome. A prolonged capillary-refill time with loss of paresthesia, pain not relieved by medication, and skin that appears tense are signs of compartment syndrome. Pink, warm extremity; pain relieved by medication; and a present dorsalis pedis pulse would all be normal findings post-fracture reduction. 3. The major serious complication post-fracture reduction is compartment syndrome. A prolonged capillary-refill time with loss of paresthesia, pain not relieved by medication, and skin that appears tense are signs of compartment syndrome. Pink, warm extremity; pain relieved by medication; and a present dorsalis pedis pulse would all be normal findings post-fracture reduction. 4. The major serious complication post-fracture reduction is compartment syndrome. A prolonged capillary-refill time with loss of paresthesia, pain not relieved by medication, and skin that appears tense are signs of compartment syndrome. Pink, warm extremity; pain relieved by medication; and a present dorsalis pedis pulse would all be normal findings post-fracture reduction. 5. The major serious complication post-fracture reduction is compartment syndrome. A prolonged capillary-refill time with loss of paresthesia, pain not relieved by medication, and skin that appears tense are signs of compartment syndrome. Pink, warm extremity; pain relieved by medication; and a present dorsalis pedis pulse would all be normal findings post-fracture reduction. Page Ref: 855-856 Cognitive Level: Analyzing Client Need &Sub: Physiological Integrity: Physiological Adaptation Standards: QSEN Competencies: Patient-centered care | AACN Essential Competencies: Essential IX: Baccalaureate generalist nursing practice | NLN Competencies: Human flourishing | Nursing/Integrated Concepts: Nursing Process: Assessment/Coordination of care Learning Outcome: LO 29.6 Develop a nursing care plan for fractures, including teaching for injury prevention and nursing implementations for the child who has sustained a fracture. MNL LO: Integumentary and Musculoskeletal Disorders/Apply the nursing process in providing care for the child and family. 15) A school-age client is admitted to the hospital with osteomyelitis. Which statement regarding the treatment of osteomyelitis is most appropriate for the nurse to share with the parents? 1. "Cultures should be done immediately after the first dose of antibiotic infuses." 2. "Antibiotics are ineffective against this virus." 3. "Methicillin is the antibiotic of choice." 4. "Antibiotic therapy should continue for 3 to 6 weeks." Answer: 4 Explanation: 1. Medical management of osteomyelitis begins with intravenous administration of a broad-spectrum antibiotic. Antibiotic therapy should continue for 3 to 6 weeks. Cultures are always done before an antibiotic is started. Methicillin is not the drug of choice. 2. Medical management of osteomyelitis begins with intravenous administration of a broad- spectrum antibiotic. Antibiotic therapy should continue for 3 to 6 weeks. Cultures are always done before an antibiotic is started. Methicillin is not the drug of choice. 3. Medical management of osteomyelitis begins with intravenous administration of a broad- spectrum antibiotic. Antibiotic therapy should continue for 3 to 6 weeks. Cultures are always done before an antibiotic is started. Methicillin is not the drug of choice. 4. Medical management of osteomyelitis begins with intravenous administration of a broad- spectrum antibiotic. Antibiotic therapy should continue for 3 to 6 weeks. Cultures are always done before an antibiotic is started. Methicillin is not the drug of choice. Page Ref: 844 Cognitive Level: Applying Client Need &Sub: Physiological Integrity: Basic Care and Comfort Standards: QSEN Competencies: Patient-centered care | AACN Essential Competencies: Essential IX: Baccalaureate generalist nursing practice | NLN Competencies: Human flourishing | Nursing/Integrated Concepts: Nursing Process: Assessment/Coordination of care Learning Outcome: LO 29.3 Recognize signs and symptoms of infectious musculoskeletal disorders and refer for appropriate care. MNL LO: Integumentary and Musculoskeletal Disorders/Examine etiology, risk factors, pathophysiology, and clinical manifestations as seen in children. 16) The nurse is caring for the newborn with bilateral clubfoot. What nursing diagnoses would the nurse address? Select all that apply. 1. Activity intolerance 2. Impaired physical mobility 3. Risk for impaired skin integrity 4. Ineffective breathing pattern 5. Impaired parenting Answer: 2, 3, 5 Explanation: 1. Nursing diagnoses that may apply to the newborn with bilateral clubfoot are impaired physical mobility, risk for impaired skin integrity, impaired parenting, and ineffective health maintenance. 2. Nursing diagnoses that may apply to the newborn with bilateral clubfoot are impaired physical mobility, risk for impaired skin integrity, impaired parenting, and ineffective health maintenance. 3. Nursing diagnoses that may apply to the newborn with bilateral clubfoot are impaired physical mobility, risk for impaired skin integrity, impaired parenting, and ineffective health maintenance. 4. Nursing diagnoses that may apply to the newborn with bilateral clubfoot are impaired physical mobility, risk for impaired skin integrity, impaired parenting, and ineffective health maintenance. 5. Nursing diagnoses that may apply to the newborn with bilateral clubfoot are impaired physical mobility, risk for impaired skin integrity, impaired parenting, and ineffective health maintenance. Page Ref: 829 Cognitive Level: Applying Client Need &Sub: Physiological Integrity: Basic Care and Comfort Standards: QSEN Competencies: Patient-centered care | AACN Essential Competencies: Essential IX: Baccalaureate generalist nursing practice | NLN Competencies: Nursing judgement | Nursing/Integrated Concepts: Nursing Process: Diagnosis/Coordination of care Learning Outcome: LO 29.2 Plan nursing care for children with structural deformities of the foot, leg, hip, and spine. MNL LO: Integumentary and Musculoskeletal Disorders/Apply the nursing process in providing care for the child and family. 17) The parents of a child with Duchenne muscular dystrophy are in the clinic after diagnosis and ask the nurse if the family should have genetic testing completed. Who should the nurse suggest to have genetic testing? Select all that apply. 1. Female cousins 2. Aunts 3. Sisters 4. Brothers 5. Uncles and male cousins Answer: 1, 2, 3 Explanation: 1. This is an X-linked disorder so all females in the family should be tested. 2. This is an X-linked disorder so all females in the family should be tested. 3. This is an X-linked disorder so all females in the family should be tested. 4. This is an X-linked disorder so all females in the family should be tested. 5. This is an X-linked disorder so all females in the family should be tested. Page Ref: 850 Cognitive Level: Analyzing Client Need &Sub: Health Promotion and Maintenance Standards: QSEN Competencies: Patient-centered care | AACN Essential Competencies: Essential VII: Clinical prevention and population health | NLN Competencies: Human flourishing | Nursing/Integrated Concepts: Nursing Process: Planning/Health teaching and health promotion Learning Outcome: LO 29.4 Partner with families to plan care for children with musculoskeletal conditions that are chronic or require long-term care. MNL LO: Integumentary and Musculoskeletal Disorders/Apply the nursing process in providing care for the child and family. Principles of Pediatric Nursing: Caring for Children, 7e (Ball et al.) Chapter 30 Alterations in Endocrine Function 1) A school-age client diagnosed with diabetes insipidus (DI) is admitted to the pediatric unit. Which laboratory value does the nurse anticipate for this client based on the diagnosis? 1. Hyperglycemia 2. Hypernatremia 3. Hypercalcemia 4. Hypoglycemia Answer: 2 Explanation: 1. In all forms of diabetes insipidus, serum sodium can increase to pathologic levels, so hypernatremia can occur and should be treated. The glucose level is not affected, so hypoglycemia or hyperglycemia is not caused by the diabetes insipidus. Hypercalcemia (high calcium) does not occur with this endocrine disorder. 2. In all forms of diabetes insipidus, serum sodium can increase to pathologic levels, so hypernatremia can occur and should be treated. The glucose level is not affected, so hypoglycemia or hyperglycemia is not caused by the diabetes insipidus. Hypercalcemia (high calcium) does not occur with this endocrine disorder. 3. In all forms of diabetes insipidus, serum sodium can increase to pathologic levels, so hypernatremia can occur and should be treated. The glucose level is not affected, so hypoglycemia or hyperglycemia is not caused by the diabetes insipidus. Hypercalcemia (high calcium) does not occur with this endocrine disorder. 4. In all forms of diabetes insipidus, serum sodium can increase to pathologic levels, so hypernatremia can occur and should be treated. The glucose level is not affected, so hypoglycemia or hyperglycemia is not caused by the diabetes insipidus. Hypercalcemia (high calcium) does not occur with this endocrine disorder. Page Ref: 869 Cognitive Level: Analyzing Client Need &Sub: Physiological Integrity: Physiological Adaptation Standards: QSEN Competencies: Patient-centered care | AACN Essential Competencies: Essential IX: Baccalaureate generalist nursing practice | NLN Competencies: Human flourishing | Nursing/Integrated Concepts: Nursing Process: Assessment/Coordination of care Learning Outcome: LO 30.2 Summarize signs and symptoms that may indicate a disorder of the endocrine system. MNL LO: Endocrine and Metabolic Disorders/Examine etiology, risk factors, pathophysiology, and clinical manifestations as seen in children. 2) A nurse is conducting a daily weight on a pediatric client diagnosed with diabetes insipidus and notes the child has lost 2 pounds in 24 hours. Which action by the nurse is the most appropriate? 1. Continue to monitor the child. 2. Notify the healthcare provider regarding the weight loss. 3. Chart the weight and report the loss to the next shift. 4. Do nothing more than chart the weight, as this would be a normal finding. Answer: 2 Explanation: 1. With diabetes insipidus, the child may have severe fluid-volume deficit. A weight loss of 2 pounds indicates a loss of 1 liter of fluid, so the healthcare provider should be notified and fluids replaced either orally or intravenously. This is a significant loss in a 24-hour period, so continuing to monitor, charting the weight and reporting to the next shift, and doing nothing would prolong treatment. 2. With diabetes insipidus, the child may have severe fluid-volume deficit. A weight loss of 2 pounds indicates a loss of 1 liter of fluid, so the healthcare provider should be notified and fluids replaced either orally or intravenously. This is a significant loss in a 24-hour period, so continuing to monitor, charting the weight and reporting to the next shift, and doing nothing would prolong treatment. 3. With diabetes insipidus, the child may have severe fluid-volume deficit. A weight loss of 2 pounds indicates a loss of 1 liter of fluid, so the healthcare provider should be notified and fluids replaced either orally or intravenously. This is a significant loss in a 24-hour period, so continuing to monitor, charting the weight and reporting to the next shift, and doing nothing would prolong treatment. 4. With diabetes insipidus, the child may have severe fluid-volume deficit. A weight loss of 2 pounds indicates a loss of 1 liter of fluid, so the healthcare provider should be notified and fluids replaced either orally or intravenously. This is a significant loss in a 24-hour period, so continuing to monitor, charting the weight and reporting to the next shift, and doing nothing would prolong treatment. Page Ref: 869 Cognitive Level: Applying Client Need &Sub: Safe and Effective Care Environment: Management of Care Standards: QSEN Competencies: Patient-centered care | AACN Essential Competencies: Essential IX: Baccalaureate generalist nursing practice | NLN Competencies: Human flourishing | Nursing/Integrated Concepts: Nursing Process: Assessment/Coordination of care Learning Outcome: LO 30.4 Prioritize nursing care for each type of acquired metabolic disorder. MNL LO: Endocrine and Metabolic Disorders/Apply the nursing process in providing care for the child and family. 3) The nurse is caring for a pediatric client diagnosed with syndrome of inappropriate antidiuretic hormone (SIADH) disorder. Which interventions should the nurse implement for this child? Select all that apply. 1. Encouragement of fluids 2. Strict intake and output 3. Administration of ordered diuretics 4. Specific gravity of urine 5. Weight only on admission but not daily Answer: 2, 3, 4 Explanation: 1. SIADH results from an excessive amount of serum antidiuretic hormone, causing water intoxication and hyponatremia. Intake and output should be monitored strictly. Diuretics such as furosemide (Lasix) are administered to eliminate excess body fluid, and urine specific gravity is monitored. Fluids are restricted to prevent further hemodilution. Daily weights should be obtained to monitor fluid balance. 2. SIADH results from an excessive amount of serum antidiuretic hormone, causing water intoxication and hyponatremia. Intake and output should be monitored strictly. Diuretics such as furosemide (Lasix) are administered to eliminate excess body fluid, and urine specific gravity is monitored. Fluids are restricted to prevent further hemodilution. Daily weights should be obtained to monitor fluid balance. 3. SIADH results from an excessive amount of serum antidiuretic hormone, causing water intoxication and hyponatremia. Intake and output should be monitored strictly. Diuretics such as furosemide (Lasix) are administered to eliminate excess body fluid, and urine specific gravity is monitored. Fluids are restricted to prevent further hemodilution. Daily weights should be obtained to monitor fluid balance. 4. SIADH results from an excessive amount of serum antidiuretic hormone, causing water intoxication and hyponatremia. Intake and output should be monitored strictly. Diuretics such as furosemide (Lasix) are administered to eliminate excess body fluid, and urine specific gravity is monitored. Fluids are restricted to prevent further hemodilution. Daily weights should be obtained to monitor fluid balance. 5. SIADH results from an excessive amount of serum antidiuretic hormone, causing water intoxication and hyponatremia. Intake and output should be monitored strictly. Diuretics such as furosemide (Lasix) are administered to eliminate excess body fluid, and urine specific gravity is monitored. Fluids are restricted to prevent further hemodilution. Daily weights should be obtained to monitor fluid balance. Page Ref: 869 Cognitive Level: Applying Client Need &Sub: Physiological Integrity: Physiological Adaptation Standards: QSEN Competencies: Patient-centered care | AACN Essential Competencies: Essential IX: Baccalaureate generalist nursing practice | NLN Competencies: Human flourishing | Nursing/Integrated Concepts: Nursing Process: Implementation/Coordination of care Learning Outcome: LO 30.4 Prioritize nursing care for each type of acquired metabolic disorder. MNL LO: Endocrine and Metabolic Disorders/Apply the nursing process in providing care for the child and family. 4) An adolescent client diagnosed with Graves' disease is admitted to the hospital. Which clinical manifestations would the nurse expect on assessment? 1. Weight gain, hirsutism, and muscle weakness 2. Dehydration, metabolic acidosis, and hypertension 3. Tachycardia, fatigue, and heat intolerance 4. Hyperglycemia, ketonuria, and glucosuria Answer: 3 Explanation: 1. Graves' disease occurs when thyroid hormone levels are increased, resulting in excessive levels of circulating thyroid hormones. Clinical manifestations include tachycardia, fatigue, and heat intolerance. Weight gain, hirsutism, and muscle weakness are signs of Cushing syndrome. Dehydration, metabolic acidosis, and hypertension are signs of congenital adrenal hyperplasia. Hyperglycemia, ketonuria, and glucosuria are signs of diabetes. 2. Graves' disease occurs when thyroid hormone levels are increased, resulting in excessive levels of circulating thyroid hormones. Clinical manifestations include tachycardia, fatigue, and heat intolerance. Weight gain, hirsutism, and muscle weakness are signs of Cushing syndrome. Dehydration, metabolic acidosis, and hypertension are signs of congenital adrenal hyperplasia. Hyperglycemia, ketonuria, and glucosuria are signs of diabetes. 3. Graves' disease occurs when thyroid hormone levels are increased, resulting in excessive levels of circulating thyroid hormones. Clinical manifestations include tachycardia, fatigue, and heat intolerance. Weight gain, hirsutism, and muscle weakness are signs of Cushing syndrome. Dehydration, metabolic acidosis, and hypertension are signs of congenital adrenal hyperplasia. Hyperglycemia, ketonuria, and glucosuria are signs of diabetes. 4. Graves' disease occurs when thyroid hormone levels are increased, resulting in excessive levels of circulating thyroid hormones. Clinical manifestations include tachycardia, fatigue, and heat intolerance. Weight gain, hirsutism, and muscle weakness are signs of Cushing syndrome. Dehydration, metabolic acidosis, and hypertension are signs of congenital adrenal hyperplasia. Hyperglycemia, ketonuria, and glucosuria are signs of diabetes. Page Ref: 871-872 Cognitive Level: Applying Client Need &Sub: Physiological Integrity: Physiological Adaptation Standards: QSEN Competencies: Patient-centered care | AACN Essential Competencies: Essential IX: Baccalaureate generalist nursing practice | NLN Competencies: Human flourishing | Nursing/Integrated Concepts: Nursing Process: Assessment/Coordination of care Learning Outcome: LO 30.2 Summarize signs and symptoms that may indicate a disorder of the endocrine system. MNL LO: Endocrine and Metabolic Disorders/Examine etiology, risk factors, pathophysiology, and clinical manifestations as seen in children. 5) A nurse is planning care for a pediatric client diagnosed with adrenal insufficiency (Addison disease). Which nursing diagnosis is the priority for this client? 1. Risk for Deficient Fluid Volume 2. Risk for Injury Secondary to Hypertension 3. Acute Pain 4. Imbalanced Nutrition: More than Body Requirements Answer: 1 Explanation: 1. Adrenal insufficiency can cause fluid deficit. The goal of care is to maintain fluid and electrolyte balance while normal levels of corticosteroids and mineral corticoids are established. Therefore, Acute Pain and Imbalanced Nutrition: More than Body Requirements are not priority nursing diagnoses. A symptom of adrenal insufficiency is hypotension, not hypertension. 2. Adrenal insufficiency can cause fluid deficit. The goal of care is to maintain fluid and electrolyte balance while normal levels of corticosteroids and mineral corticoids are established. Therefore, Acute Pain and Imbalanced Nutrition: More than Body Requirements are not priority nursing diagnoses. A symptom of adrenal insufficiency is hypotension, not hypertension. 3. Adrenal insufficiency can cause fluid deficit. The goal of care is to maintain fluid and electrolyte balance while normal levels of corticosteroids and mineral corticoids are established. Therefore, Acute Pain and Imbalanced Nutrition: More than Body Requirements are not priority nursing diagnoses. A symptom of adrenal insufficiency is hypotension, not hypertension. 4. Adrenal insufficiency can cause fluid deficit. The goal of care is to maintain fluid and electrolyte balance while normal levels of corticosteroids and mineral corticoids are established. Therefore, Acute Pain and Imbalanced Nutrition: More than Body Requirements are not priority nursing diagnoses. A symptom of adrenal insufficiency is hypotension, not hypertension. Page Ref: 876 Cognitive Level: Analyzing Client Need &Sub: Physiological Integrity: Reduction of Risk Potential Standards: QSEN Competencies: Patient-centered care | AACN Essential Competencies: Essential IX: Baccalaureate generalist nursing practice | NLN Competencies: Human flourishing | Nursing/Integrated Concepts: Nursing Process: Planning/Coordination of care Learning Outcome: LO 30.4 Prioritize nursing care for each type of acquired metabolic disorder. MNL LO: Endocrine and Metabolic Disorders/Apply the nursing process in providing care for the child and family. 6) A pediatric client is admitted to the hospital unconscious. The client has a history of type 1 diabetes, and according to the client's mother, has been to two birthday parties in the last few days and has resisted taking the prescribed insulin. At school the client had two more pieces of birthday cake and some ice cream at a class birthday party. What is the likely reason for this client's unconscious state? 1. Metabolic alkalosis 2. Metabolic ketoacidosis 3. Insulin shock 4. Insulin reaction Answer: 2 Explanation: 1. Metabolic acidosis or ketoacidosis could have occurred because of the excessive intake of sugar with no additional insulin. The body burns fat and protein stores for energy when no insulin is available to metabolize glucose. Altered consciousness occurs as symptoms progress. Metabolic alkalosis, insulin shock, or insulin reaction would not be happening in this case. 2. Metabolic acidosis or ketoacidosis could have occurred because of the excessive intake of sugar with no additional insulin. The body burns fat and protein stores for energy when no insulin is available to metabolize glucose. Altered consciousness occurs as symptoms progress. Metabolic alkalosis, insulin shock, or insulin reaction would not be happening in this case. 3. Metabolic acidosis or ketoacidosis could have occurred because of the excessive intake of sugar with no additional insulin. The body burns fat and protein stores for energy when no insulin is available to metabolize glucose. Altered consciousness occurs as symptoms progress. Metabolic alkalosis, insulin shock, or insulin reaction would not be happening in this case. 4. Metabolic acidosis or ketoacidosis could have occurred because of the excessive intake of sugar with no additional insulin. The body burns fat and protein stores for energy when no insulin is available to metabolize glucose. Altered consciousness occurs as symptoms progress. Metabolic alkalosis, insulin shock, or insulin reaction would not be happening in this case. Page Ref: 877 Cognitive Level: Analyzing Client Need &Sub: Physiological Integrity: Physiological Adaptation Standards: QSEN Competencies: Patient-centered care | AACN Essential Competencies: Essential IX: Baccalaureate generalist nursing practice | NLN Competencies: Human flourishing | Nursing/Integrated Concepts: Nursing Process: Assessment/Coordination of care Learning Outcome: LO 30.6 Distinguish between the nursing care of the child with type 1 and type 2 diabetes. MNL LO: Endocrine and Metabolic Disorders/Differentiate treatment options and strategies for continuity of care for the child and family. 7) A pediatric client is diagnosed with type 1 diabetes. The nurse teaches the client the difference between insulin shock and diabetic hyperglycemia. The nurse evaluates that the client understands the teaching when the client states which characteristics of diabetic hyperglycemia? 1. Tremors and lethargy 2. Hunger and hypertension 3. Thirst and flushed skin 4. Shakiness and pallor Answer: 3 Explanation: 1. Thirst and flushed skin are characteristic of diabetic hyperglycemia. Tremors, lethargy, hunger, shakiness, and pallor are characteristic of hypoglycemia. Hypertension is not a sign associated with hyperglycemia or hypoglycemia. 2. Thirst and flushed skin are characteristic of diabetic hyperglycemia. Tremors, lethargy, hunger, shakiness, and pallor are characteristic of hypoglycemia. Hypertension is not a sign associated with hyperglycemia or hypoglycemia. 3. Thirst and flushed skin are characteristic of diabetic hyperglycemia. Tremors, lethargy, hunger, shakiness, and pallor are characteristic of hypoglycemia. Hypertension is not a sign associated with hyperglycemia or hypoglycemia. 4. Thirst and flushed skin are characteristic of diabetic hyperglycemia. Tremors, lethargy, hunger, shakiness, and pallor are characteristic of hypoglycemia. Hypertension is not a sign associated with hyperglycemia or hypoglycemia. Page Ref: 877 Cognitive Level: Analyzing Client Need &Sub: Physiological Integrity: Physiological Adaptation Standards: QSEN Competencies: Patient-centered care | AACN Essential Competencies: Essential VII: Clinical prevention and population health | NLN Competencies: Human flourishing | Nursing/Integrated Concepts: Nursing Process: Assessment/Health teaching and health promotion Learning Outcome: LO 30.6 Distinguish between the nursing care of the child with type 1 and type 2 diabetes. MNL LO: Endocrine and Metabolic Disorders/Examine etiology, risk factors, pathophysiology, and clinical manifestations as seen in children. 8) The nurse is providing education to a pediatric client diagnosed with diabetes. The client will be playing soccer over the summer. Which change in the client's management will the nurse explore during this education session? 1. Increased food intake 2. Decreased food intake 3. Increased need for insulin 4. Decreased risk of insulin reaction Answer: 1 Explanation: 1. Increased physical activity requires adequate caloric intake to prevent hypoglycemia, so food intake should be increased. Increased activity would not require decreased food intake, and it would not result in a decreased risk of insulin reaction. Exercise causes the insulin to be used more efficiently, so increased insulin would not be needed. 2. Increased physical activity requires adequate caloric intake to prevent hypoglycemia, so food intake should be increased. Increased activity would not require decreased food intake, and it would not result in a decreased risk of insulin reaction. Exercise causes the insulin to be used more efficiently, so increased insulin would not be needed. 3. Increased physical activity requires adequate caloric intake to prevent hypoglycemia, so food intake should be increased. Increased activity would not require decreased food intake, and it would not result in a decreased risk of insulin reaction. Exercise causes the insulin to be used more efficiently, so increased insulin would not be needed. 4. Increased physical activity requires adequate caloric intake to prevent hypoglycemia, so food intake should be increased. Increased activity would not require decreased food intake, and it would not result in a decreased risk of insulin reaction. Exercise causes the insulin to be used more efficiently, so increased insulin would not be needed. Page Ref: 885 Cognitive Level: Applying Client Need &Sub: Physiological Integrity: Reduction of Risk Potential Standards: QSEN Competencies: Patient-centered care | AACN Essential Competencies: Essential VII: Clinical prevention and population health | NLN Competencies: Human flourishing | Nursing/Integrated Concepts: Nursing Process: Implementation/Health teaching and health promotion Learning Outcome: LO 30.6 Distinguish between the nursing care of the child with type 1 and type 2 diabetes. MNL LO: Endocrine and Metabolic Disorders/Examine etiology, risk factors, pathophysiology, and clinical manifestations as seen in children. 9) The nurse is teaching the parent of a type 1 diabetic preschool-age client about management of the disease. Which teaching point is appropriate for the nurse to include in this session? 1. Allowing the client to administer all the insulin injections 2. Allowing the client to choose which finger to stick for glucose testing 3. Allowing the client to draw up the insulin dose 4. Allowing the client to test blood glucose Answer: 2 Explanation: 1. The preschool-age client's need for autonomy and control can be met by allowing the client to pick which finger to stick for glucose testing. Administering the insulin, drawing up the dose, and testing blood glucose should not be done by the client until he or she is middle-school age or older. 2. The preschool-age client's need for autonomy and control can be met by allowing the client to pick which finger to stick for glucose testing. Administering the insulin, drawing up the dose, and testing blood glucose should not be done by the client until he or she is middle-school age or older. 3. The preschool-age client's need for autonomy and control can be met by allowing the client to pick which finger to stick for glucose testing. Administering the insulin, drawing up the dose, and testing blood glucose should not be done by the client until he or she is middle-school age or older. 4. The preschool-age client's need for autonomy and control can be met by allowing the client to pick which finger to stick for glucose testing. Administering the insulin, drawing up the dose, and testing blood glucose should not be done by the client until he or she is middle-school age or older. Page Ref: 881 Cognitive Level: Applying Client Need &Sub: Health Promotion and Maintenance Standards: QSEN Competencies: Patient-centered care | AACN Essential Competencies: Essential VII: Clinical prevention and population health | NLN Competencies: Nursing judgement | Nursing/Integrated Concepts: Nursing Process: Implementation/Health teaching and health promotion Learning Outcome: LO 30.6 Distinguish between the nursing care of the child with type 1 and type 2 diabetes. MNL LO: Endocrine and Metabolic Disorders/Educate the child and family on the care of the child during hospitalization and upon discharge. 10) A pediatric client is seen in the clinic with a possible diagnosis of type 2 diabetes. The mother asks what the healthcare provider uses to make the diagnosis. The nurse explains that type 2 diabetes is suspected if the child has obesity, acanthosis nigricans, and two non-fasting blood-glucose levels above which level? 1. 120 2. 80 3. 200 4. 50 Answer: 3 Explanation: 1. Blood-glucose levels at or above 200 mg/dL without fasting is diagnostic of type 2 diabetes. 2. Blood-glucose levels at or above 200 mg/dL without fasting is diagnostic of type 2 diabetes. 3. Blood-glucose levels at or above 200 mg/dL without fasting is diagnostic of type 2 diabetes. 4. Blood-glucose levels at or above 200 mg/dL without fasting is diagnostic of type 2 diabetes. Page Ref: 891 Cognitive Level: Analyzing Client Need &Sub: Physiological Integrity: Physiological Adaptation Standards: QSEN Competencies: Patient-centered care | AACN Essential Competencies: Essential VII: Clinical prevention and population health | NLN Competencies: Human flourishing | Nursing/Integrated Concepts: Nursing Process: Assessment/Health teaching and health promotion Learning Outcome: LO 30.2 Summarize signs and symptoms that may indicate a disorder of the endocrine system. MNL LO: Endocrine and Metabolic Disorders/Examine etiology, risk factors, pathophysiology, and clinical manifestations as seen in children. 11) A pediatric client diagnosed with Turner syndrome tells the nurse, "I feel different from my peers." Which response by the nurse is the most appropriate? 1. "Tell me more about the feelings you are experiencing." 2. "These feelings are not unusual and should pass soon." 3. "You'll start to grow soon, so don't worry." 4. "You seem to be upset about your disease." Answer: 1 Explanation: 1. The lack of growth and sexual development associated with Turner syndrome presents problems with psychosocial development. Self-image, self-consciousness, and self- esteem are affected by the girl's perception of her body and how she differs from peers. The nurse should encourage more expression of the girl's feelings. Responding that the feelings will pass, that she'll start to grow, or that she is upset about the disease would not be therapeutic. 2. The lack of growth and sexual development associated with Turner syndrome presents problems with psychosocial development. Self-image, self-consciousness, and self-esteem are affected by the girl's perception of her body and how she differs from peers. The nurse should encourage more expression of the girl's feelings. Responding that the feelings will pass, that she'll start to grow, or that she is upset about the disease would not be therapeutic. 3. The lack of growth and sexual development associated with Turner syndrome presents problems with psychosocial development. Self-image, self-consciousness, and self-esteem are affected by the girl's perception of her body and how she differs from peers. The nurse should encourage more expression of the girl's feelings. Responding that the feelings will pass, that she'll start to grow, or that she is upset about the disease would not be therapeutic. 4. The lack of growth and sexual development associated with Turner syndrome presents problems with psychosocial development. Self-image, self-consciousness, and self-esteem are affected by the girl's perception of her body and how she differs from peers. The nurse should encourage more expression of the girl's feelings. Responding that the feelings will pass, that she'll start to grow, or that she is upset about the disease would not be therapeutic. Page Ref: 893 Cognitive Level: Applying Client Need &Sub: Psychosocial Integrity Standards: QSEN Competencies: Patient-centered care | AACN Essential Competencies: Essential IX: Baccalaureate generalist nursing practice | NLN Competencies: Human flourishing | Nursing/Integrated Concepts: Nursing Process: Planning/Coordination of care Learning Outcome: LO 30.7 Plan care for the child with an inherited metabolic disorder. MNL LO: Endocrine and Metabolic Disorders/Apply the nursing process in providing care for the child and family. 12) A parent of a newborn asks the nurse why a heel stick is being done on the baby to test for phenylketonuria (PKU). Which response by the nurse is the most appropriate? 1. "This screening is required and detection can be done before symptoms develop." 2. "The infant has high-risk characteristics." 3. "Because the infant was born by cesarean, this test is necessary." 4. "Because the infant was born by vaginal delivery, this test is recommended." Answer: 1 Explanation: 1. Screening for phenylketonuria is required by law in every state. It is not done according to high-risk characteristics or type of delivery. 2. Screening for phenylketonuria is required by law in every state. It is not done according to high-risk characteristics or type of delivery. 3. Screening for phenylketonuria is required by law in every state. It is not done according to high-risk characteristics or type of delivery. 4. Screening for phenylketonuria is required by law in every state. It is not done according to high-risk characteristics or type of delivery. Page Ref: 894-895 Cognitive Level: Applying Client Need &Sub: Health Promotion and Maintenance Standards: QSEN Competencies: Patient-centered care | AACN Essential Competencies: Essential VII: Clinical prevention and population health | NLN Competencies: Human flourishing | Nursing/Integrated Concepts: Nursing Process: Assessment/Health teaching and health promotion Learning Outcome: LO 30.2 Summarize signs and symptoms that may indicate a disorder of the endocrine system. MNL LO: Endocrine and Metabolic Disorders/Examine etiology, risk factors, pathophysiology, and clinical manifestations as seen in children. 13) The nurse is administering a dose of rapid-acting insulin at 0800 to an insulin-dependent pediatric client. Based on when the insulin peaks, when will the client be at greatest risk for a hypoglycemic episode? 1. At about noon 2. Between bedtime and breakfast the next morning 3. Between lunch and dinner 4. Around 0930 Answer: 4 Explanation: 1. Rapid-acting insulin peaks 30-90 minutes after administration. An injection given at 0800 would peak around 0930. 2. Rapid-acting insulin peaks 30-90 minutes after administration. An injection given at 0800 would peak around 0930. 3. Rapid-acting insulin peaks 30-90 minutes after administration. An injection given at 0800 would peak around 0930. 4. Rapid-acting insulin peaks 30-90 minutes after administration. An injection given at 0800 would peak around 0930. Page Ref: 890 Cognitive Level: Applying Client Need &Sub: Physiological Integrity: Pharmacological and Parenteral Therapies Standards: QSEN Competencies: Patient-centered care | AACN Essential Competencies: Essential IX: Baccalaureate generalist nursing practice | NLN Competencies: Human flourishing | Nursing/Integrated Concepts: Nursing Process: Planning/Coordination of care Learning Outcome: LO 30.6 Distinguish between the nursing care of the child with type 1 and type 2 diabetes. MNL LO: Endocrine and Metabolic Disorders/Differentiate treatment options and strategies for continuity of care for the child and family. 14) Which teaching tips should be included when instructing parents on hydrocortisone administration? Select all that apply. 1. Maintain prescribed administration times. 2. Never discontinue medication abruptly. 3. Injections might be necessary when unable to take by mouth. 4. Lower doses are needed during illness. 5. Keep an emergency kit with the child at all times. Answer: 1, 2, 3, 5 Explanation: 1. Maintaining prescribed administration times is important, as they follow the normal body release of cortisol. Abruptly discontinuing a steroid is not recommended. Giving injections when unable to take by mouth and during emergencies is important to maintain cortisol levels. Higher, not lower, doses are needed during illness. 2. Maintaining prescribed administration times is important, as they follow the normal body release of cortisol. Abruptly discontinuing a steroid is not recommended. Giving injections when unable to take by mouth and during emergencies is important to maintain cortisol levels. Higher, not lower, doses are needed during illness. 3. Maintaining prescribed administration times is important, as they follow the normal body release of cortisol. Abruptly discontinuing a steroid is not recommended. Giving injections when unable to take by mouth and during emergencies is important to maintain cortisol levels. Higher, not lower, doses are needed during illness. 4. Maintaining prescribed administration times is important, as they follow the normal body release of cortisol. Abruptly discontinuing a steroid is not recommended. Giving injections when unable to take by mouth and during emergencies is important to maintain cortisol levels. Higher, not lower, doses are needed during illness. 5. Maintaining prescribed administration times is important, as they follow the normal body release of cortisol. Abruptly discontinuing a steroid is not recommended. Giving injections when unable to take by mouth and during emergencies is important to maintain cortisol levels. Higher, not lower, doses are needed during illness. Page Ref: 876 Cognitive Level: Applying Client Need &Sub: Physiological Integrity: Pharmacological and Parenteral Therapies Standards: QSEN Competencies: Patient-centered care | AACN Essential Competencies: Essential VII: Clinical prevention and population health | NLN Competencies: Nursing judgement | Nursing/Integrated Concepts: Nursing Process: Planning/Health teaching and health promotion Learning Outcome: LO 30.5 Develop a family education plan for the child who needs lifelong cortisol replacement. MNL LO: Endocrine and Metabolic Disorders/Educate the child and family on the care of the child during hospitalization and upon discharge. 15) The nurse is planning care for a pediatric client diagnosed with adrenal hyperplasia. Which nursing diagnosis is most appropriate for this client? 1. Impaired Social Interaction Related to Unnatural Facial Features 2. Nutrition: Less than Body Requirements due to Nausea and Vomiting 3. Depression Related to Inability to Take in Oral Fluids 4. Risk for Deficient Fluid Volume Related to Failure of Regulatory Mechanisms Answer: 4 Explanation: 1. Adrenal hyperplasia alters the regulatory mechanisms, creating a fluid volume deficit. There is no major nutritional deficit, social interaction, or depression related directly to the diagnosis of adrenal hyperplasia. 2. Adrenal hyperplasia alters the regulatory mechanisms, creating a fluid volume deficit. There is no major nutritional deficit, social interaction, or depression related directly to the diagnosis of adrenal hyperplasia. 3. Adrenal hyperplasia alters the regulatory mechanisms, creating a fluid volume deficit. There is no major nutritional deficit, social interaction, or depression related directly to the diagnosis of adrenal hyperplasia. 4. Adrenal hyperplasia alters the regulatory mechanisms, creating a fluid volume deficit. There is no major nutritional deficit, social interaction, or depression related directly to the diagnosis of adrenal hyperplasia. Page Ref: 875 Cognitive Level: Applying Client Need &Sub: Physiological Integrity: Physiological Adaptation Standards: QSEN Competencies: Patient-centered care | AACN Essential Competencies: Essential IX: Baccalaureate generalist nursing practice | NLN Competencies: Human flourishing | Nursing/Integrated Concepts: Nursing Process: Planning/Coordination of care Learning Outcome: LO 30.7 Plan care for the child with an inherited metabolic disorder. MNL LO: Endocrine and Metabolic Disorders/Apply the nursing process in providing care for the child and family. 16) The nurse is planning care for pediatric clients who have diagnoses that impact the endocrine system. Which changes occurring during the school-age and adolescence have a direct impact on the endocrine system? Select all that apply. 1. Puberty 2. Adrenarche 3. Menarche 4. Sexual exploration 5. Risk-taking behavior Answer: 1, 2, 3 Explanation: 1. Puberty, adrenarche, and menarche are all changes that occur during the school age and adolescence that have a direct impact on the endocrine system. Sexual exploration and risk-taking behaviors do not have a direct impact on the endocrine system. 2. Puberty, adrenarche, and menarche are all changes that occur during the school age and adolescence that have a direct impact on the endocrine system. Sexual exploration and risk- taking behaviors do not have a direct impact on the endocrine system. 3. Puberty, adrenarche, and menarche are all changes that occur during the school age and adolescence that have a direct impact on the endocrine system. Sexual exploration and risk- taking behaviors do not have a direct impact on the endocrine system. 4. Puberty, adrenarche, and menarche are all changes that occur during the school age and adolescence that have a direct impact on the endocrine system. Sexual exploration and risk- taking behaviors do not have a direct impact on the endocrine system. 5. Puberty, adrenarche, and menarche are all changes that occur during the school age and adolescence that have a direct impact on the endocrine system. Sexual exploration and risk- taking behaviors do not have a direct impact on the endocrine system. Page Ref: 892 Cognitive Level: Applying Client Need &Sub: Physiological Integrity: Physiological Adaptation Standards: QSEN Competencies: Patient-centered care | AACN Essential Competencies: Essential IX: Baccalaureate generalist nursing practice | NLN Competencies: Human flourishing | Nursing/Integrated Concepts: Nursing Process: Planning/Coordination of care Learning Outcome: LO 30.2 Summarize signs and symptoms that may indicate a disorder of the endocrine system. MNL LO: Endocrine and Metabolic Disorders/Examine etiology, risk factors, pathophysiology, and clinical manifestations as seen in children. 17) The nurse educator is teaching a group of nursing students about the endocrine system. Which statements are appropriate for the educator to include in the teaching session? Select all that apply. 1. Gonadotropin-releasing hormone stimulates the anterior pituitary to produce LH and FSH. 2. Growth hormone regulates linear bone growth and growth of all tissues. 3. Antidiuretic hormone regulates urine concentration by the kidneys. 4. Thyroid hormone regulates serum calcium levels and phosphorus excretion. 5. Parathyroid hormone regulates metabolism of cells and body heat production. Answer: 1, 2, 3 Explanation: 1. All statements are correct except the statements regarding the thyroid hormone and the parathyroid hormone. The thyroid hormone regulates metabolism of the cells and body heat production, not the parathyroid hormone. The parathyroid hormone regulates serum calcium levels and phosphorus excretion. 2. All statements are correct except the statements regarding the thyroid hormone and the parathyroid hormone. The thyroid hormone regulates metabolism of the cells and body heat production, not the parathyroid hormone. The parathyroid hormone regulates serum calcium levels and phosphorus excretion. 3. All statements are correct except the statements regarding the thyroid hormone and the parathyroid hormone. The thyroid hormone regulates metabolism of the cells and body heat production, not the parathyroid hormone. The parathyroid hormone regulates serum calcium levels and phosphorus excretion. 4. All statements are correct except the statements regarding the thyroid hormone and the parathyroid hormone. The thyroid hormone regulates metabolism of the cells and body heat production, not the parathyroid hormone. The parathyroid hormone regulates serum calcium levels and phosphorus excretion. 5. All statements are correct except the statements regarding the thyroid hormone and the parathyroid hormone. The thyroid hormone regulates metabolism of the cells and body heat production, not the parathyroid hormone. The parathyroid hormone regulates serum calcium levels and phosphorus excretion. Page Ref: 867-868 Cognitive Level: Analyzing Client Need &Sub: Physiological Integrity: Physiological Adaptation Standards: QSEN Competencies: Patient-centered care | AACN Essential Competencies: Essential VI: Interprofessional communication and collaboration for improving patient health outcomes | NLN Competencies: Professional identity | Nursing/Integrated Concepts: Nursing Process: Implementation/Education Learning Outcome: LO 30.1 Identify the function of important hormones of the endocrine system. MNL LO: Endocrine and Metabolic Disorders/Examine etiology, risk factors, pathophysiology, and clinical manifestations as seen in children. 18) The nurse is providing education to a group of student nurses regarding disorders of the endocrine system that can cause short stature. Which disorders will the nurse include in the educational session? Select all that apply. 1. Hypothyroidism 2. Turner syndrome 3. Type 1 diabetes mellitus 4. Diabetes insipidus 5. Cushing syndrome Answer: 1, 2, 5 Explanation: 1. There are many disorders of the endocrine system that can cause short stature including hypothyroidism, Turner syndrome, and Cushing syndrome. Type 1 diabetes mellitus and diabetes insipidus are not endocrine disorders that cause short stature. 2. There are many disorders of the endocrine system that can cause short stature including hypothyroidism, Turner syndrome, and Cushing syndrome. Type 1 diabetes mellitus and diabetes insipidus are not endocrine disorders that cause short stature. 3. There are many disorders of the endocrine system that can cause short stature including hypothyroidism, Turner syndrome, and Cushing syndrome. Type 1 diabetes mellitus and diabetes insipidus are not endocrine disorders that cause short stature. 4. There are many disorders of the endocrine system that can cause short stature including hypothyroidism, Turner syndrome, and Cushing syndrome. Type 1 diabetes mellitus and diabetes insipidus are not endocrine disorders that cause short stature. 5. There are many disorders of the endocrine system that can cause short stature including hypothyroidism, Turner syndrome, and Cushing syndrome. Type 1 diabetes mellitus and diabetes insipidus are not endocrine disorders that cause short stature. Page Ref: 862-863 Cognitive Level: Remembering Client Need &Sub: Physiological Integrity: Physiological Adaptation Standards: QSEN Competencies: Patient-centered care | AACN Essential Competencies: Essential VI: Interprofessional communication and collaboration for improving patient health outcomes | NLN Competencies: Professional identity | Nursing/Integrated Concepts: Nursing Process: Implementation/Education Learning Outcome: LO 30.3 Identify all conditions for which short stature is a sign. MNL LO: Endocrine and Metabolic Disorders/Examine etiology, risk factors, pathophysiology, and clinical manifestations as seen in children. 19) An adolescent is admitted to the intensive care unit (ICU) with diabetic ketoacidosis. The client weighs 115 pounds. The healthcare provider orders: Regular insulin 0.15 units/kg bolus via IVF, then regular insulin 0.1 units/kg/hr in 0.9 percent NSS Medication on hand: 250 mL 0.9 percent NSS with 250 units of regular insulin. Calculate the mL/hr for the continuous infusion of regular insulin at 0.1 unit/kg/hr in 0.9 percent NSS. Answer: 7.4 mL/hr Explanation: 7.4 mL/hr Page Ref: 888 Cognitive Level: Analyzing Client Need &Sub: Physiological Integrity: Pharmacological and Parenteral Therapies Standards: QSEN Competencies: Patient-centered care | AACN Essential Competencies: Essential IX: Baccalaureate generalist nursing practice | NLN Competencies: Human flourishing | Nursing/Integrated Concepts: Nursing Process: Planning/Coordination of care Learning Outcome: LO 30.6 Distinguish between the nursing care of the child with type 1 and type 2 diabetes. MNL LO: Endocrine and Metabolic Disorders/Differentiate treatment options and strategies for continuity of care for the child and family. 20) A school-age child is being assessed for syndrome of inappropriate antidiuretic hormone (SIADH). The nurse should watch the child for which symptoms? Select all that apply. 1. Polyphagia 2. Retention of fluid 3. Hypernatremia 4. Hyponatremia 5. Hyperglycemia Answer: 2, 3 Explanation: 1. ADH helps the body retain fluid. Serum osmolality is increased (greater than 300 mOsm/kg) and urine osmolality is decreased (less than 300 mOsm/kg). Urine specific gravity is decreased (less than 1.005) and serum sodium is elevated. 2. ADH helps the body retain fluid. Serum osmolality is increased (greater than 300 mOsm/kg) and urine osmolality is decreased (less than 300 mOsm/kg). Urine specific gravity is decreased (less than 1.005) and serum sodium is elevated. 3. ADH helps the body retain fluid. Serum osmolality is increased (greater than 300 mOsm/kg) and urine osmolality is decreased (less than 300 mOsm/kg). Urine specific gravity is decreased (less than 1.005) and serum sodium is elevated. 4. ADH helps the body retain fluid. Serum osmolality is increased (greater than 300 mOsm/kg) and urine osmolality is decreased (less than 300 mOsm/kg). Urine specific gravity is decreased (less than 1.005) and serum sodium is elevated. 5. ADH helps the body retain fluid. Serum osmolality is increased (greater than 300 mOsm/kg) and urine osmolality is decreased (less than 300 mOsm/kg). Urine specific gravity is decreased (less than 1.005) and serum sodium is elevated. Page Ref: 868-869 Cognitive Level: Analyzing Client Need &Sub: Health Promotion and Maintenance Standards: QSEN Competencies: Patient-centered care | AACN Essential Competencies: Essential IX: Baccalaureate generalist nursing practice | NLN Competencies: Human flourishing | Nursing/Integrated Concepts: Nursing Process: Assessment/Coordination of care Learning Outcome: LO 30.2 Summarize signs and symptoms that may indicate a disorder of the endocrine system. MNL LO: Endocrine and Metabolic Disorders/Differentiate treatment options and strategies for continuity of care for the child and family. Principles of Pediatric Nursing: Caring for Children, 7e (Ball et al.) Chapter 31 Alterations in Skin Integrity 1) The nurse is planning care for a 3-month-old infant diagnosed with eczema. Which should be the focus of the nurse's care for this infant? 1. Maintaining adequate nutrition 2. Keeping the baby content 3. Preventing infection of lesions 4. Applying antibiotics to lesions Answer: 3 Explanation: 1. Nursing care should focus on preventing infection of lesions. Due to impaired skin-barrier function and cutaneous immunity, an infant with eczema is at greater risk for the development of skin infections by organisms. Maintaining adequate nutrition and keeping the infant content are not as high a priority. Antibiotics are not routinely applied to the lesions. 2. Nursing care should focus on preventing infection of lesions. Due to impaired skin-barrier function and cutaneous immunity, an infant with eczema is at greater risk for the development of skin infections by organisms. Maintaining adequate nutrition and keeping the infant content are not as high a priority. Antibiotics are not routinely applied to the lesions. 3. Nursing care should focus on preventing infection of lesions. Due to impaired skin-barrier function and cutaneous immunity, an infant with eczema is at greater risk for the development of skin infections by organisms. Maintaining adequate nutrition and keeping the infant content are not as high a priority. Antibiotics are not routinely applied to the lesions. 4. Nursing care should focus on preventing infection of lesions. Due to impaired skin-barrier function and cutaneous immunity, an infant with eczema is at greater risk for the development of skin infections by organisms. Maintaining adequate nutrition and keeping the infant content are not as high a priority. Antibiotics are not routinely applied to the lesions. Page Ref: 914-915 Cognitive Level: Applying Client Need &Sub: Physiological Integrity: Reduction of Risk Potential Standards: QSEN Competencies: Patient-centered care | AACN Essential Competencies: Essential IX: Baccalaureate generalist nursing practice | NLN Competencies: Human flourishing | Nursing/Integrated Concepts: Nursing Process: Implementation/Coordination of care Learning Outcome: LO 31.4 Plan the nursing care for the child with alterations in skin integrity, including dermatitis, infectious disorders, and infestations. MNL LO: Integumentary and Musculoskeletal Disorders/Apply the nursing process in providing care for the child and family. 2) The nurse is caring for a pediatric client diagnosed with eczema. Which topical medication order does the nurse anticipate for this client? 1. Corticosteroid 2. Retinoid 3. Antifungal 4. Antibacterial Answer: 1 Explanation: 1. Topical corticosteroid is used to reduce inflammation when the child has eczema. Topical retinoid is used for acne. Topical antifungal is used for dermatophytoses. Topical antibacterial would be used for problems such as burns. 2. Topical corticosteroids is used to reduce inflammation when the child has eczema. Topical retinoid is used for acne. Topical antifungal is used for dermatophytoses. Topical antibacterial would be used for problems such as burns. 3. Topical corticosteroids is used to reduce inflammation when the child has eczema. Topical retinoid is used for acne. Topical antifungal is used for dermatophytoses. Topical antibacterial would be used for problems such as burns. 4. Topical corticosteroids is used to reduce inflammation when the child has eczema. Topical retinoid is used for acne. Topical antifungal is used for dermatophytoses. Topical antibacterial would be used for problems such as burns. Page Ref: 914-915 Cognitive Level: Analyzing Client Need &Sub: Physiological Integrity: Pharmacological and Parenteral Therapies Standards: QSEN Competencies: Patient-centered care | AACN Essential Competencies: Essential IX: Baccalaureate generalist nursing practice | NLN Competencies: Human flourishing | Nursing/Integrated Concepts: Nursing Process: Implementation/Coordination of care Learning Outcome: LO 31.4 Plan the nursing care for the child with alterations in skin integrity, including dermatitis, infectious disorders, and infestations. MNL LO: Integumentary and Musculoskeletal Disorders/Differentiate treatment options and strategies for continuity of care for the child and family. 3) The nurse is examining a 12-month-old who is brought to the clinic for persistent diaper rash. The nurse finds perianal inflammation with bright red scaly plaques and small papules. Satellite lesions are also present. What is the most likely cause of this client's diaper rash? 1. Impetigo (staph) 2. Candida albicans (yeast) 3. Urine and feces 4. Infrequent diapering Answer: 2 Explanation: 1. Candida albicans is frequently the underlying cause of severe diaper rash. When a primary or secondary infection with Candida albicans occurs, the rash has bright red scaly plaques with sharp margins. Small papules and pustules may be seen, along with satellite lesions. Even though diaper dermatitis can be caused by impetigo, urine and feces, and infrequent diapering, the lesions and persistent characteristics are common for Candida. 2. Candida albicans is frequently the underlying cause of severe diaper rash. When a primary or secondary infection with Candida albicans occurs, the rash has bright red scaly plaques with sharp margins. Small papules and pustules may be seen, along with satellite lesions. Even though diaper dermatitis can be caused by impetigo, urine and feces, and infrequent diapering, the lesions and persistent characteristics are common for Candida. 3. Candida albicans is frequently the underlying cause of severe diaper rash. When a primary or secondary infection with Candida albicans occurs, the rash has bright red scaly plaques with sharp margins. Small papules and pustules may be seen, along with satellite lesions. Even though diaper dermatitis can be caused by impetigo, urine and feces, and infrequent diapering, the lesions and persistent characteristics are common for Candida. 4. Candida albicans is frequently the underlying cause of severe diaper rash. When a primary or secondary infection with Candida albicans occurs, the rash has bright red scaly plaques with sharp margins. Small papules and pustules may be seen, along with satellite lesions. Even though diaper dermatitis can be caused by impetigo, urine and feces, and infrequent diapering, the lesions and persistent characteristics are common for Candida. Page Ref: 907 Cognitive Level: Analyzing Client Need &Sub: Health Promotion and Maintenance Standards: QSEN Competencies: Patient-centered care | AACN Essential Competencies: Essential IX: Baccalaureate generalist nursing practice | NLN Competencies: Human flourishing | Nursing/Integrated Concepts: Nursing Process: Assessment/Coordination of care Learning Outcome: LO 31.1 Classify the characteristics of skin lesions caused by irritants, drug reactions, mites, infection, and injury. MNL LO: Integumentary and Musculoskeletal Disorders/Examine etiology, risk factors, pathophysiology, and clinical manifestations as seen in children. 4) A 2-month-old client has a candidal diaper rash. Which medication does the nurse anticipate will be prescribed for this client? 1. Bacitracin ointment 2. Hydrocortisone ointment 3. Desitin 4. Nystatin given topically and orally Answer: 4 Explanation: 1. Diaper candidiasis is treated with an antifungal cream (Nystatin). An oral antifungal agent may be given to clear the candidiasis from the intestines. Bacitracin is for an infection caused by staphylococcus. Mild diaper rash is treated with a barrier such as Desitin. Moderate diaper rash is treated with hydrocortisone ointment. 2. Diaper candidiasis is treated with an antifungal cream (Nystatin). An oral antifungal agent may be given to clear the candidiasis from the intestines. Bacitracin is for an infection caused by staphylococcus. Mild diaper rash is treated with a barrier such as Desitin. Moderate diaper rash is treated with hydrocortisone ointment. 3. Diaper candidiasis is treated with an antifungal cream (Nystatin). An oral antifungal agent may be given to clear the candidiasis from the intestines. Bacitracin is for an infection caused by staphylococcus. Mild diaper rash is treated with a barrier such as Desitin. Moderate diaper rash is treated with hydrocortisone ointment. 4. Diaper candidiasis is treated with an antifungal cream (Nystatin). An oral antifungal agent may be given to clear the candidiasis from the intestines. Bacitracin is for an infection caused by staphylococcus. Mild diaper rash is treated with a barrier such as Desitin. Moderate diaper rash is treated with hydrocortisone ointment. Page Ref: 907-908 Cognitive Level: Applying Client Need &Sub: Physiological Integrity: Pharmacological and Parenteral Therapies Standards: QSEN Competencies: Patient-centered care | AACN Essential Competencies: Essential IX: Baccalaureate generalist nursing practice | NLN Competencies: Human flourishing | Nursing/Integrated Concepts: Nursing Process: Implementation/Coordination of care Learning Outcome: LO 31.4 Plan the nursing care for the child with alterations in skin integrity, including dermatitis, infectious disorders, and infestations. MNL LO: Integumentary and Musculoskeletal Disorders/Apply the nursing process in providing care for the child and family. 5) The nurse is teaching a group of adolescents about care for acne vulgaris. Which interventions will the nurse include in the teaching session? Select all that apply. 1. Wash skin with mild soap and water twice a day. 2. Use astringents and vigorous scrubbing. 3. Avoid picking or squeezing the lesions. 4. Apply tretinoin (Retin-A) liberally. 5. Avoid sun exposure if on tetracycline. Answer: 1, 3, 5 Explanation: 1. The adolescent should be taught to wash skin with mild soap and water twice a day, to avoid picking or squeezing acne lesions, and to avoid sun exposure if on tetracycline. Using astringents and scrubbing vigorously can exacerbate acne. Tretinoin (Retin-A) should be applied sparingly (pea-size doses). 2. The adolescent should be taught to wash skin with mild soap and water twice a day, to avoid picking or squeezing acne lesions, and to avoid sun exposure if on tetracycline. Using astringents and scrubbing vigorously can exacerbate acne. Tretinoin (Retin-A) should be applied sparingly (pea-size doses). 3. The adolescent should be taught to wash skin with mild soap and water twice a day, to avoid picking or squeezing acne lesions, and to avoid sun exposure if on tetracycline. Using astringents and scrubbing vigorously can exacerbate acne. Tretinoin (Retin-A) should be applied sparingly (pea-size doses). 4. The adolescent should be taught to wash skin with mild soap and water twice a day, to avoid picking or squeezing acne lesions, and to avoid sun exposure if on tetracycline. Using astringents and scrubbing vigorously can exacerbate acne. Tretinoin (Retin-A) should be applied sparingly (pea-size doses). 5. The adolescent should be taught to wash skin with mild soap and water twice a day, to avoid picking or squeezing acne lesions, and to avoid sun exposure if on tetracycline. Using astringents and scrubbing vigorously can exacerbate acne. Tretinoin (Retin-A) should be applied sparingly (pea-size doses). Page Ref: 919 Cognitive Level: Applying Client Need &Sub: Health Promotion and Maintenance Standards: QSEN Competencies: Patient-centered care | AACN Essential Competencies: Essential VII: Clinical prevention and population health | NLN Competencies: Human flourishing | Nursing/Integrated Concepts: Nursing Process: Implementation/Health teaching and health promotion Learning Outcome: LO 31.5 Prepare an education plan for adolescents with acne to promote self-care. MNL LO: Integumentary and Musculoskeletal Disorders/Apply the nursing process in providing care for the child and family. 6) A pediatric client is hospitalized with a severe case of impetigo contagiosa. Which antibiotic does the nurse anticipate the healthcare provider will order for this client? 1. Dicloxacillin (Pathocil) 2. Rifampin (Rifadin) 3. Sulfamethoxazole and trimethoprim (Bactrim) 4. Metronidazole (Flagyl) Answer: 1 Explanation: 1. A systemic antibiotic will be given for severe impetigo because it is a bacterial infection. Dicloxacillin is used in treatment of skin and soft-tissue infections. It is specific for treating staphylococcal infections. Rifampin is an antitubercular agent, sulfamethoxazole and trimethoprim are used as a prophylaxis against Pneumocystis carinii pneumonia (PCP), and metronidazole is used to treat anaerobic and protozoic infections. 2. A systemic antibiotic will be given for severe impetigo because it is a bacterial infection. Dicloxacillin is used in treatment of skin and soft-tissue infections. It is specific for treating staphylococcal infections. Rifampin is an antitubercular agent, sulfamethoxazole and trimethoprim are used as a prophylaxis against Pneumocystis carinii pneumonia (PCP), and metronidazole is used to treat anaerobic and protozoic infections. 3. A systemic antibiotic will be given for severe impetigo because it is a bacterial infection. Dicloxacillin is used in treatment of skin and soft-tissue infections. It is specific for treating staphylococcal infections. Rifampin is an antitubercular agent, sulfamethoxazole and trimethoprim are used as a prophylaxis against Pneumocystis carinii pneumonia (PCP), and metronidazole is used to treat anaerobic and protozoic infections. 4. A systemic antibiotic will be given for severe impetigo because it is a bacterial infection. Dicloxacillin is used in treatment of skin and soft-tissue infections. It is specific for treating staphylococcal infections. Rifampin is an antitubercular agent, sulfamethoxazole and trimethoprim are used as a prophylaxis against Pneumocystis carinii pneumonia (PCP), and metronidazole is used to treat anaerobic and protozoic infections. Page Ref: 909 Cognitive Level: Applying Client Need &Sub: Physiological Integrity: Pharmacological and Parenteral Therapies Standards: QSEN Competencies: Patient-centered care | AACN Essential Competencies: Essential IX: Baccalaureate generalist nursing practice | NLN Competencies: Human flourishing | Nursing/Integrated Concepts: Nursing Process: Planning/Coordination of care Learning Outcome: LO 31.4 Plan the nursing care for the child with alterations in skin integrity, including dermatitis, infectious disorders, and infestations. MNL LO: Integumentary and Musculoskeletal Disorders/Differentiate treatment options and strategies for continuity of care for the child and family. 7) An infant has a severe case of oral thrush (Candida albicans). Which nursing diagnosis is the priority for this infant? 1. Activity Intolerance Related to Oral Thrush 2. Ineffective Airway Clearance Related to Mucus 3. Ineffective Infant Feeding Pattern Related to Discomfort 4. Ineffective Breathing Pattern Related to Oral Thrush Answer: 3 Explanation: 1. An infant with oral thrush may refuse to nurse or feed because of discomfort and pain. Prompt treatment is necessary so the infant can resume a normal feeding pattern. Activity intolerance, ineffective airway clearance, and ineffective breathing patterns are not usual associated problems. 2. An infant with oral thrush may refuse to nurse or feed because of discomfort and pain. Prompt treatment is necessary so the infant can resume a normal feeding pattern. Activity intolerance, ineffective airway clearance, and ineffective breathing patterns are not usual associated problems. 3. An infant with oral thrush may refuse to nurse or feed because of discomfort and pain. Prompt treatment is necessary so the infant can resume a normal feeding pattern. Activity intolerance, ineffective airway clearance, and ineffective breathing patterns are not usual associated problems. 4. An infant with oral thrush may refuse to nurse or feed because of discomfort and pain. Prompt treatment is necessary so the infant can resume a normal feeding pattern. Activity intolerance, ineffective airway clearance, and ineffective breathing patterns are not usual associated problems. Page Ref: 911-912 Cognitive Level: Analyzing Client Need &Sub: Health Promotion and Maintenance Standards: QSEN Competencies: Patient-centered care | AACN Essential Competencies: Essential IX: Baccalaureate generalist nursing practice | NLN Competencies: Human flourishing | Nursing/Integrated Concepts: Nursing Process: Diagnosis/Coordination of care Learning Outcome: LO 31.4 Plan the nursing care for the child with alterations in skin integrity, including dermatitis, infectious disorders, and infestations. MNL LO: Integumentary and Musculoskeletal Disorders/Apply the nursing process in providing care for the child and family. 8) The nurse is providing education to the parents of a pediatric client who is diagnosed with tinea capitis (ringworm of the scalp). Which statement made the parents indicates an appropriate understanding of the teaching session? 1. "We will give the griseofulvin on an empty stomach." 2. "We're glad ringworm isn't transmitted from person to person." 3. "Once the lesion is gone, we can stop the griseofulvin." 4. "We will give the griseofulvin with milk or peanut butter." Answer: 4 Explanation: 1. Parents are advised to give oral griseofulvin with fatty foods such as milk or peanut butter to enhance absorption. The medication must be used for the entire prescribed period even if the lesions are gone. All members of the family and household pets should be assessed for fungal lesions because person-to-person transmission is common. 2. Parents are advised to give oral griseofulvin with fatty foods such as milk or peanut butter to enhance absorption. The medication must be used for the entire prescribed period even if the lesions are gone. All members of the family and household pets should be assessed for fungal lesions because person-to-person transmission is common. 3. Parents are advised to give oral griseofulvin with fatty foods such as milk or peanut butter to enhance absorption. The medication must be used for the entire prescribed period even if the lesions are gone. All members of the family and household pets should be assessed for fungal lesions because person-to-person transmission is common. 4. Parents are advised to give oral griseofulvin with fatty foods such as milk or peanut butter to enhance absorption. The medication must be used for the entire prescribed period even if the lesions are gone. All members of the family and household pets should be assessed for fungal lesions because person-to-person transmission is common. Page Ref: 912 Cognitive Level: Analyzing Client Need &Sub: Health Promotion and Maintenance Standards: QSEN Competencies: Patient-centered care | AACN Essential Competencies: Essential VII: Clinical prevention and population health | NLN Competencies: Human flourishing | Nursing/Integrated Concepts: Nursing Process: Evaluation/Health teaching and health promotion Learning Outcome: LO 31.4 Plan the nursing care for the child with alterations in skin integrity, including dermatitis, infectious disorders, and infestations. MNL LO: Integumentary and Musculoskeletal Disorders/Educate the child and family on the care of the child during hospitalization and upon discharge. 9) The school nurse is conducting pediculosis capitis (head lice) checks. Which findings would indicate a "positive" head check? 1. White, flaky particles throughout the entire scalp region 2. Maculopapular lesions behind the ears 3. Lesions in the scalp that extend to the hairline or neck 4. White sacs attached to the hair shafts in the occipital area Answer: 4 Explanation: 1. Evidence of pediculosis capitis includes white sacs (nits) that are attached to the hair shafts, frequently in the occiput area. Lesions may be present from itching, but the positive sign is evidence of nits. Lice and nits must be distinguished from dandruff, which appears as white, flaky particles. 2. Evidence of pediculosis capitis includes white sacs (nits) that are attached to the hair shafts, frequently in the occiput area. Lesions may be present from itching, but the positive sign is evidence of nits. Lice and nits must be distinguished from dandruff, which appears as white, flaky particles. 3. Evidence of pediculosis capitis includes white sacs (nits) that are attached to the hair shafts, frequently in the occiput area. Lesions may be present from itching, but the positive sign is evidence of nits. Lice and nits must be distinguished from dandruff, which appears as white, flaky particles. 4. Evidence of pediculosis capitis includes white sacs (nits) that are attached to the hair shafts, frequently in the occiput area. Lesions may be present from itching, but the positive sign is evidence of nits. Lice and nits must be distinguished from dandruff, which appears as white, flaky particles. Page Ref: 922-923 Cognitive Level: Analyzing Client Need &Sub: Health Promotion and Maintenance Standards: QSEN Competencies: Patient-centered care | AACN Essential Competencies: Essential IX: Baccalaureate generalist nursing practice | NLN Competencies: Human flourishing | Nursing/Integrated Concepts: Nursing Process: Assessment/Coordination of care Learning Outcome: LO 31.1 Classify the characteristics of skin lesions caused by irritants, drug reactions, mites, infection, and injury. MNL LO: Integumentary and Musculoskeletal Disorders/Examine etiology, risk factors, pathophysiology, and clinical manifestations as seen in children. 10) A nurse is caring for a toddler client who is diagnosed with scabies and prescribed a 5 percent permethrin lotion. How will the nurse apply this lotion when administering it to the toddler? 1. To the scalp only 2. Over the entire body from the chin down, as well as on the scalp and forehead 3. Only on the areas with evidence of scabies activity 4. Only on the hands Answer: 2 Explanation: 1. Treatment of scabies involves application of a scabicide, such as 5 percent permethrin lotion, over the entire body from the chin down. The scabicide is also applied to the scalp and forehead of younger children, avoiding the rest of the face. 2. Treatment of scabies involves application of a scabicide, such as 5 percent permethrin lotion, over the entire body from the chin down. The scabicide is also applied to the scalp and forehead of younger children, avoiding the rest of the face. 3. Treatment of scabies involves application of a scabicide, such as 5 percent permethrin lotion, over the entire body from the chin down. The scabicide is also applied to the scalp and forehead of younger children, avoiding the rest of the face. 4. Treatment of scabies involves application of a scabicide, such as 5 percent permethrin lotion, over the entire body from the chin down. The scabicide is also applied to the scalp and forehead of younger children, avoiding the rest of the face. Page Ref: 924 Cognitive Level: Applying Client Need &Sub: Health Promotion and Maintenance Standards: QSEN Competencies: Patient-centered care | AACN Essential Competencies: Essential IX: Baccalaureate generalist nursing practice | NLN Competencies: Human flourishing | Nursing/Integrated Concepts: Nursing Process: Implementation/Coordination of care Learning Outcome: LO 31.4 Plan the nursing care for the child with alterations in skin integrity, including dermatitis, infectious disorders, and infestations. MNL LO: Integumentary and Musculoskeletal Disorders/Differentiate treatment options and strategies for continuity of care for the child and family. 11) The nurse is caring for a pediatric client who sustained a severe burn. Determine the order of what would be done for this child when the medical team arrives on the scene: Response 1. Start intravenous fluids. Response 2. Provide for relief of pain. Response 3. Establish an airway. Response 4. Place a Foley catheter. Answer: 3, 1, 2, 4 Establish an airway. Start intravenous fluids. Provide for relief of pain. Place a Foley catheter. Explanation: The first step in burn care is to ensure that the child has an airway, is breathing, and has a pulse. Due to the severity of the burn, establishing IV access and starting resuscitation fluids would be next, followed by addressing the area of pain and inserting a Foley catheter. Page Ref: 927-929 Cognitive Level: Applying Client Need &Sub: Safe and Effective Care Environment: Safety and Infection Control Standards: QSEN Competencies: Patient-centered care | AACN Essential Competencies: Essential IX: Baccalaureate generalist nursing practice | NLN Competencies: Human flourishing | Nursing/Integrated Concepts: Nursing Process: Implementation/Coordination of care Learning Outcome: LO 31.7 Develop a nursing care plan for the child with a full-thickness burn injury. MNL LO: Integumentary and Musculoskeletal Disorders/Apply the nursing process in providing care for the child and family. 12) The nurse is providing care for a pediatric client who has a third-degree circumferential burn of the right arm. Which nursing diagnosis is the priority for this client? 1. Risk for Infection 2. Risk for Altered Tissue Perfusion 3. Risk for Altered Nutrition: Less than Body Requirements 4. Impaired Physical Mobility Answer: 2 Explanation: 1. When the burn is circumferential, blood flow can become restricted due to edema and result in tissue hypoxia; therefore, the priority diagnosis is Risk for Altered Tissue Perfusion to the Extremity. Infection, Nutrition, and Mobility would have second priority in this case. 2. When the burn is circumferential, blood flow can become restricted due to edema and result in tissue hypoxia; therefore, the priority diagnosis is Risk for Altered Tissue Perfusion to the Extremity. Infection, Nutrition, and Mobility would have second priority in this case. 3. When the burn is circumferential, blood flow can become restricted due to edema and result in tissue hypoxia; therefore, the priority diagnosis is Risk for Altered Tissue Perfusion to the Extremity. Infection, Nutrition, and Mobility would have second priority in this case. 4. When the burn is circumferential, blood flow can become restricted due to edema and result in tissue hypoxia; therefore, the priority diagnosis is Risk for Altered Tissue Perfusion to the Extremity. Infection, Nutrition, and Mobility would have second priority in this case. Page Ref: 931 Cognitive Level: Analyzing Client Need &Sub: Safe and Effective Care Environment Standards: QSEN Competencies: Patient-centered care | AACN Essential Competencies: Essential IX: Baccalaureate generalist nursing practice | NLN Competencies: Human flourishing | Nursing/Integrated Concepts: Nursing Process: Diagnosis/Coordination of care Learning Outcome: LO 31.6 Summarize the process to measure the extent of burns and burn severity in children. MNL LO: Integumentary and Musculoskeletal Disorders/Apply the nursing process in providing care for the child and family. 13) During the recovery-management phase of burn treatment, which is the most common complication seen in children? 1. Shock 2. Metabolic acidosis 3. Burn-wound infection 4. Asphyxia Answer: 3 Explanation: 1. Infection of the burned area is a frequent complication in the recovery— management phase. A goal of burn-wound care is protection from infection. 2. Infection of the burned area is a frequent complication in the recovery—management phase. A goal of burn-wound care is protection from infection. 3. Infection of the burned area is a frequent complication in the recovery—management phase. A goal of burn-wound care is protection from infection. 4. Infection of the burned area is a frequent complication in the recovery—management phase. A goal of burn-wound care is protection from infection. Page Ref: 933 Cognitive Level: Analyzing Client Need &Sub: Physiological Integrity: Physiological Adaptation Standards: QSEN Competencies: Patient-centered care | AACN Essential Competencies: Essential IX: Baccalaureate generalist nursing practice | NLN Competencies: Human flourishing | Nursing/Integrated Concepts: Nursing Process: Assessment/Coordination of care Learning Outcome: LO 31.7 Develop a nursing care plan for the child with a full-thickness burn injury. MNL LO: Integumentary and Musculoskeletal Disorders/Apply the nursing process in providing care for the child and family. 14) The nurse explains to the parents of a child with a severe burn that wearing of an elastic pressure garment (Jobst stocking) during the rehabilitative stage can help with the prevention of which complication? 1. Poor circulation 2. Hypertrophic scarring 3. Pain 4. Formation of thrombus in the burn area Answer: 2 Explanation: 1. During the rehabilitation stage, Jobst stockings or pressure garments are used to reduce development of hypertrophic scarring and contractures. 2. During the rehabilitation stage, Jobst stockings or pressure garments are used to reduce development of hypertrophic scarring and contractures. 3. During the rehabilitation stage, Jobst stockings or pressure garments are used to reduce development of hypertrophic scarring and contractures. 4. During the rehabilitation stage, Jobst stockings or pressure garments are used to reduce development of hypertrophic scarring and contractures. Page Ref: 934 Cognitive Level: Analyzing Client Need &Sub: Physiological Integrity: Reduction of Risk Potential Standards: QSEN Competencies: Patient-centered care | AACN Essential Competencies: Essential VII: Clinical prevention and population health | NLN Competencies: Human flourishing | Nursing/Integrated Concepts: Nursing Process: Planning/Health teaching and health promotion Learning Outcome: LO 31.7 Develop a nursing care plan for the child with a full-thickness burn injury. MNL LO: Integumentary and Musculoskeletal Disorders/Apply the nursing process in providing care for the child and family. 15) A pediatric client sustains a minor burn. When teaching the family the treatment for this burn, the nurse would teach that the client's diet should be high in which substance? 1. Fats 2. Protein 3. Minerals 4. Carbohydrates Answer: 2 Explanation: 1. Parents should be taught that management of a minor burn requires a high- calorie, high-protein diet. This is necessary to meet the increased nutritional requirements of healing. 2. Parents should be taught that management of a minor burn requires a high-calorie, high- protein diet. This is necessary to meet the increased nutritional requirements of healing. 3. Parents should be taught that management of a minor burn requires a high-calorie, high- protein diet. This is necessary to meet the increased nutritional requirements of healing. 4. Parents should be taught that management of a minor burn requires a high-calorie, high- protein diet. This is necessary to meet the increased nutritional requirements of healing. Page Ref: 930 Cognitive Level: Applying Client Need &Sub: Health Promotion and Maintenance Standards: QSEN Competencies: Patient-centered care | AACN Essential Competencies: Essential VII: Clinical prevention and population health | NLN Competencies: Human flourishing | Nursing/Integrated Concepts: Nursing Process: Implementation/Health teaching and health promotion Learning Outcome: LO 31.7 Develop a nursing care plan for the child with a full-thickness burn injury. MNL LO: Integumentary and Musculoskeletal Disorders/Educate the child and family on the care of the child during hospitalization and upon discharge. 16) The nurse is teaching a group of students about wound healing. Which items will the nurse include as occurring during the hemostasis and inflammation stage of wound healing? Select all that apply. 1. Clot formation to seal the wound 2. Production of collagen and granulation tissue 3. Scar formation and strengthening 4. Release of inflammatory mediators by platelets 5. Swelling as a result of increased capillary permeability Answer: 1, 2, 5 Explanation: 1. During the hemostasis and inflammation stage of wound healing, the nurse would state that clot formation occurs to seal the wound; platelets release inflammatory mediators; and increased capillary permeability results in swelling. Scar formation and strengthening occur during maturation. Collagen and granulation tissue are produced during tissue formation. 2. During the hemostasis and inflammation stage of wound healing, the nurse would state that clot formation occurs to seal the wound; platelets release inflammatory mediators; and increased capillary permeability results in swelling. Scar formation and strengthening occur during maturation. Collagen and granulation tissue are produced during tissue formation. 3. During the hemostasis and inflammation stage of wound healing, the nurse would state that clot formation occurs to seal the wound; platelets release inflammatory mediators; and increased capillary permeability results in swelling. Scar formation and strengthening occur during maturation. Collagen and granulation tissue are produced during tissue formation. 4. During the hemostasis and inflammation stage of wound healing, the nurse would state that clot formation occurs to seal the wound; platelets release inflammatory mediators; and increased capillary permeability results in swelling. Scar formation and strengthening occur during maturation. Collagen and granulation tissue are produced during tissue formation. 5. During the hemostasis and inflammation stage of wound healing, the nurse would state that clot formation occurs to seal the wound; platelets release inflammatory mediators; and increased capillary permeability results in swelling. Scar formation and strengthening occur during maturation. Collagen and granulation tissue are produced during tissue formation. Page Ref: 905 Cognitive Level: Applying Client Need &Sub: Physiological Integrity: Physiological Adaptation Standards: QSEN Competencies: Patient-centered care | AACN Essential Competencies: Essential VII: Clinical prevention and population health | NLN Competencies: Human flourishing | Nursing/Integrated Concepts: Nursing Process: Implementation/Health teaching and health promotion Learning Outcome: LO 31.2 Differentiate among the stages of wound healing. MNL LO: Integumentary and Musculoskeletal Disorders/Examine etiology, risk factors, pathophysiology, and clinical manifestations as seen in children. 17) The nurse is providing care to a pediatric client who is diagnosed with psoriasis. Which clinical manifestations does the nurse anticipate upon assessment of this client? Select all that apply. 1. Thick, silvery, scaly erythematous plaque 2. Pruritus 3. Dry skin, likely to crack and fissure 4. Fragile skin and blisters 5. Irregular border surrounded by normal skin Answer: 1, 2, 5 Explanation: 1. Clinical manifestations that support the diagnosis of psoriasis include thick, silvery, scaly erythematous plaque; pruritis; and irregular border surrounded by normal skin. Dry skin that is likely to crack and fissure is a clinical manifestation of atopic dermatitis. Fragile skin and blisters are clinical manifestations of epidermolysis bullosa. 2. Clinical manifestations that support the diagnosis of psoriasis include thick, silvery, scaly erythematous plaque; pruritis; and irregular border surrounded by normal skin. Dry skin that is likely to crack and fissure is a clinical manifestation of atopic dermatitis. Fragile skin and blisters are clinical manifestations of epidermolysis bullosa. 3. Clinical manifestations that support the diagnosis of psoriasis include thick, silvery, scaly erythematous plaque; pruritis; and irregular border surrounded by normal skin. Dry skin that is likely to crack and fissure is a clinical manifestation of atopic dermatitis. Fragile skin and blisters are clinical manifestations of epidermolysis bullosa. 4. Clinical manifestations that support the diagnosis of psoriasis include thick, silvery, scaly erythematous plaque; pruritis; and irregular border surrounded by normal skin. Dry skin that is likely to crack and fissure is a clinical manifestation of atopic dermatitis. Fragile skin and blisters are clinical manifestations of epidermolysis bullosa. 5. Clinical manifestations that support the diagnosis of psoriasis include thick, silvery, scaly erythematous plaque; pruritis; and irregular border surrounded by normal skin. Dry skin that is likely to crack and fissure is a clinical manifestation of atopic dermatitis. Fragile skin and blisters are clinical manifestations of epidermolysis bullosa. Page Ref: 921 Cognitive Level: Applying Client Need &Sub: Physiological Integrity: Physiological Adaptation Standards: QSEN Competencies: Patient-centered care | AACN Essential Competencies: Essential IX: Baccalaureate generalist nursing practice | NLN Competencies: Human flourishing | Nursing/Integrated Concepts: Nursing Process: Assessment/Coordination of care Learning Outcome: LO 31.3 Compare skin conditions that have a hereditary cause or predisposition. MNL LO: Integumentary and Musculoskeletal Disorders/Examine etiology, risk factors, pathophysiology, and clinical manifestations as seen in children. 18) The nurse is providing teaching to a community group regarding preventative strategies to reduce the risk of burn injury. Which topics will the nurse include in the teaching session? Select all that apply. 1. Avoid contact with unknown animals and wild animals. 2. Layer children's clothing for warmth. 3. Keep infants and toddlers off the lap when drinking hot beverages or eating soup. 4. Lower the temperature settings for hot water heaters. 5. Wear light-colored clothes and avoid eating sweetened foods and beverages when outside. Answer: 3, 4 Explanation: 1. In order to decrease the risk of burn injury, the nurse would tell the group to keep infants and toddlers off the lap while drinking hot beverages or eating soup and to lower the temperature settings for the hot water heaters. Avoiding contact with unknown animals and wild animals along with wearing light-colored clothes and avoiding eating sweetened foods and beverages when outside are strategies to prevent bites and stings. Layering children's clothing for warmth is a strategy to prevent hypothermia. 2. In order to decrease the risk of burn injury, the nurse would tell the group to keep infants and toddlers off the lap while drinking hot beverages or eating soup and to lower the temperature settings for the hot water heaters. Avoiding contact with unknown animals and wild animals along with wearing light-colored clothes and avoiding eating sweetened foods and beverages when outside are strategies to prevent bites and stings. Layering children's clothing for warmth is a strategy to prevent hypothermia. 3. In order to decrease the risk of burn injury, the nurse would tell the group to keep infants and toddlers off the lap while drinking hot beverages or eating soup and to lower the temperature settings for the hot water heaters. Avoiding contact with unknown animals and wild animals along with wearing light-colored clothes and avoiding eating sweetened foods and beverages when outside are strategies to prevent bites and stings. Layering children's clothing for warmth is a strategy to prevent hypothermia. 4. In order to decrease the risk of burn injury, the nurse would tell the group to keep infants and toddlers off the lap while drinking hot beverages or eating soup and to lower the temperature settings for the hot water heaters. Avoiding contact with unknown animals and wild animals along with wearing light-colored clothes and avoiding eating sweetened foods and beverages when outside are strategies to prevent bites and stings. Layering children's clothing for warmth is a strategy to prevent hypothermia. 5. In order to decrease the risk of burn injury, the nurse would tell the group to keep infants and toddlers off the lap while drinking hot beverages or eating soup and to lower the temperature settings for the hot water heaters. Avoiding contact with unknown animals and wild animals along with wearing light-colored clothes and avoiding eating sweetened foods and beverages when outside are strategies to prevent bites and stings. Layering children's clothing for warmth is a strategy to prevent hypothermia. Page Ref: 934 Cognitive Level: Applying Client Need &Sub: Health Promotion and Maintenance Standards: QSEN Competencies: Patient-centered care | AACN Essential Competencies: Essential VII: Clinical prevention and population health | NLN Competencies: Human flourishing | Nursing/Integrated Concepts: Nursing Process: Implementation/Health teaching and health promotion Learning Outcome: LO 31.8 Contrast preventive strategies to reduce the risk of injury from burns, hypothermia, bites, and stings. MNL LO: Integumentary and Musculoskeletal Disorders/Examine etiology, risk factors, pathophysiology, and clinical manifestations as seen in children. 19) A child is prescribed cephalexin for treatment of cellulitis. The child weighs 15 kg. The pediatrician orders: cephalexin 40 mg/kg/day PO, give twice a day. Medication on hand: 250 mg/5 mL Calculate how many mLs the nurse must draw up for each dose. Answer: 6 mL Explanation: 6 mL Page Ref: 986 Cognitive Level: Analyzing Client Need &Sub: Physiological Integrity: Pharmacological and Parenteral Therapies Standards: QSEN Competencies: Patient-centered care | AACN Essential Competencies: Essential IX: Baccalaureate generalist nursing practice | NLN Competencies: Human flourishing | Nursing/Integrated Concepts: Nursing Process: Planning/Coordination of care Learning Outcome: LO 31.4 Plan the nursing care for the child with alterations in skin integrity, including dermatitis, infectious disorders, and infestations. MNL LO: Integumentary and Musculoskeletal Disorders/Differentiate treatment options and strategies for continuity of care for the child and family. 20) The adolescent is seen in the clinic for a consultation to treat severe acne. The adolescent has tried other medications, but the acne has not been responsive. The nurse knows that what medication is the most effective for this client with severe acne? 1. Oral contraceptives 2. Isotretinoin 3. Antibiotics 4. Benzoyl peroxide Answer: 2 Explanation: 1. Isotretinoin is reserved for severe acne that is not responsive to other therapies. 2. Isotretinoin is reserved for severe acne that is not responsive to other therapies. 3. Isotretinoin is reserved for severe acne that is not responsive to other therapies. 4. Isotretinoin is reserved for severe acne that is not responsive to other therapies. Page Ref: 994 Cognitive Level: Analyzing Client Need &Sub: Health Promotion and Maintenance Standards: QSEN Competencies: Patient-centered care | AACN Essential Competencies: Essential VII: Clinical prevention and population health | NLN Competencies: Human flourishing | Nursing/Integrated Concepts: Nursing Process: Planning/Coordination of care Learning Outcome: LO 31.5 Prepare an education plan for adolescents with acne to promote self-care. MNL LO: Integumentary and Musculoskeletal Disorders/Differentiate treatment options and strategies for continuity of care for the child and family. [Show More]

Last updated: 1 year ago

Preview 1 out of 869 pages

Add to cart

Instant download

document-preview

Buy this document to get the full access instantly

Instant Download Access after purchase

Add to cart

Instant download

Reviews( 0 )

$30.00

Add to cart

Instant download

Can't find what you want? Try our AI powered Search

OR

REQUEST DOCUMENT
90
0

Document information


Connected school, study & course


About the document


Uploaded On

Aug 02, 2022

Number of pages

869

Written in

Seller


seller-icon
securegrades

Member since 4 years

117 Documents Sold


Additional information

This document has been written for:

Uploaded

Aug 02, 2022

Downloads

 0

Views

 90

Document Keyword Tags

Recommended For You

Get more on EXAM »

$30.00
What is Browsegrades

In Browsegrades, a student can earn by offering help to other student. Students can help other students with materials by upploading their notes and earn money.

We are here to help

We're available through e-mail, Twitter, Facebook, and live chat.
 FAQ
 Questions? Leave a message!

Follow us on
 Twitter

Copyright © Browsegrades · High quality services·